Sie sind auf Seite 1von 418

LWBK707-FM_i-viii.

qxd 9/2/10 7:32 AM Page i Aptara Inc

Study
Study Guide for
for

Porths Essentials of Pathophysiology


Third Edition
LWBK707-FM_i-viii.qxd 9/2/10 7:32 AM Page ii Aptara Inc

Acquisitions Editor: Hilarie Surrena


Product Manager: Katherine Burland
Editorial Assistant: Shawn Loht
Design Coordinator: Joan Wendt
Manufacturing Coordinator: Karin Duffield
Prepress Vendor: Aptara, Inc.

3rd edition

Copyright 2011 Wolters Kluwer Health | Lippincott Williams & Wilkins.

All rights reserved. This book is protected by copyright. No part of this book may be reproduced
or transmitted in any form or by any means, including as photocopies or scanned-in or other
electronic copies, or utilized by any information storage and retrieval system without written
permission from the copyright owner, except for brief quotations embodied in critical articles
and reviews. Materials appearing in this book prepared by individuals as part of their official
duties as U.S. government employees are not covered by the above-mentioned copyright.
To request permission, please contact Lippincott Williams & Wilkins at 530 Walnut Street,
Philadelphia, PA 19106, via email at permissions@lww.com, or via our website at lww.com
(products and services).

9 8 7 6 5 4 3 2 1

Printed in the United States of America

ISBN: 978-0-7817-7779-7

Care has been taken to confirm the accuracy of the information presented and to describe
generally accepted practices. However, the authors, editors, and publisher are not responsible for
errors or omissions or for any consequences from application of the information in this book
and make no warranty, expressed or implied, with respect to the currency, completeness, or ac-
curacy of the contents of the publication. Application of this information in a particular situa-
tion remains the professional responsibility of the practitioner; the clinical treatments described
and recommended may not be considered absolute and universal recommendations.
The authors, editors, and publisher have exerted every effort to ensure that drug selection
and dosage set forth in this text are in accordance with the current recommendations and prac-
tice at the time of publication. However, in view of ongoing research, changes in government
regulations, and the constant flow of information relating to drug therapy and drug reactions,
the reader is urged to check the package insert for each drug for any change in indications
and dosage and for added warnings and precautions. This is particularly important when the
recommended agent is a new or infrequently employed drug.
Some drugs and medical devices presented in this publication have Food and Drug Adminis-
tration (FDA) clearance for limited use in restricted research settings. It is the responsibility of
the health care provider to ascertain the FDA status of each drug or device planned for use in his
or her clinical practice.

LWW.com
LWBK707-FM_i-viii.qxd 9/2/10 7:32 AM Page iii Aptara Inc

Preface

This Study Guide was written by Brian Kipp, PhD, APPLYING YOUR KNOWLEDGE
to accompany the third edition of Essentials of
Pathophysiology: Concepts of Altered Health States by The second section of each Study Guide chapter
Carol Mattson Porth. The Study Guide is designed consists of case study-based exercises that ask you
to help you practice and retain the knowledge to begin to apply the knowledge youve gained
youve gained from the textbook, and give you a from the textbook chapter and reinforced in the
basis for applying it in your practice. The following first section of the Study Guide chapter. A case
types of exercises are provided in each chapter of study scenario based on the chapters content is
the Study Guide. presented, and then you are asked to answer some
questions, in writing, related to the case study. The
questions could cover lab values, next steps in
ASSESSING YOUR treatment, anticipated diagnoses, and the like.
UNDERSTANDING
The first section of each Study Guide chapter PRACTICING FOR NCLEX
concentrates on the basic information of the
textbook chapter and helps you to remember key The third and final section of the Study Guide
concepts, vocabulary, and principles. helps you practice NCLEX-style questions while
further reinforcing the knowledge you have been
Fill in the Blanks: Fill-in-the-blank exercises
gaining and testing for yourself through the
test important chapter information, encourag-
textbook chapter and the first two sections of the
ing you to recall key points.
study guide chapter. In keeping with the NCLEX,
Labeling: Labeling exercises are used where the questions presented are multiple-choice and
you need to remember certain visual represen- scenario-based, asking you to reflect, consider,
tations of the concepts presented in the and apply what you know and to choose the best
textbook. answer out of those offered.
Matching: Matching questions test you
knowledge of the definition of key terms.
Sequencing: Sequencing exercises ask you to
ANSWER KEYS
remember particular sequences or orders, for
The answers for all of the exercises and questions
instance of normal or abnormal physiologic
in the Study Guide are provided at the back of
processes.
the book, so you can assess your own learning as
Short Answers: Short-answer questions cover you complete each chapter.
facts, concepts, procedures, and principles of We hope you will find this Study Guide to be
the chapter. These questions ask you to recall helpful and enjoyable, and we wish you every
information as well as demonstrate your success in your studies and future profession.
comprehension of the information.
The Publishers

iii
LWBK707-FM_i-viii.qxd 9/2/10 7:32 AM Page iv Aptara Inc
LWBK707-FM_i-viii.qxd 9/2/10 7:32 AM Page v Aptara Inc

Contents

CHAPTER 9
UNIT 1
Stress and Adaptation 49
CELL AND TISSUE FUNCTION 1
CHAPTER 10
CHAPTER 1
Disorders of Nutritional Status 53
Cell Structure and Function 1

CHAPTER 2 UNIT 3
Cellular Responses to Stress, Injury, HEMATOPOIETIC FUNCTION 57
and Aging 6
CHAPTER 11
CHAPTER 3
Disorders of White Blood Cells and
Inflammation, the Inflammatory Response, Lymphoid Tissues 57
and Fever 12
CHAPTER 12
CHAPTER 4
Disorders of Hemostasis 62
Cell Proliferation and Tissue Regeneration
and Repair 17 CHAPTER 13
CHAPTER 5 Disorders of Red Blood Cells 67
Genetic Control of Cell Function
and Inheritance 20 UNIT 4
INFECTION AND IMMUNITY 73
CHAPTER 6
Genetic and Congenital Disorders 26 CHAPTER 14
CHAPTER 7 Mechanisms of Infectious Disease 73
Neoplasia 32
CHAPTER 15
Innate and Adaptive Immunity 78
UNIT 2
INTEGRATIVE BODY FUNCTIONS 39 CHAPTER 16
Disorders of the Immune Response 84
CHAPTER 8
Disorders of Fluid, Electrolyte,
and Acid-Base Balance 39

v
LWBK707-FM_i-viii.qxd 9/2/10 7:32 AM Page vi Aptara Inc

vi CONTENTS

CHAPTER 27
UNIT 5
Disorders of the Bladder and
CIRCULATORY FUNCTION 90 Lower Urinary Tract 152
CHAPTER 17
Control of Cardiovascular Function 90
UNIT 8
GASTROINTESTINAL AND HEPATOBILIARY
CHAPTER 18 FUNCTION 157
Disorders of Blood Flow and
Blood Pressure 96 CHAPTER 28
Structure and Function
CHAPTER 19 of the Gastrointestinal System 157
Disorders of Cardiac Function 104
CHAPTER 29
CHAPTER 20 Disorders of Gastrointestinal
Heart Failure and Circulatory Shock 112 Function 162

CHAPTER 30
Disorders of Hepatobiliary and
UNIT 6 Exocrine Pancreas Function 169
RESPIRATORY FUNCTION 118

CHAPTER 21
UNIT 9
Control of Respiratory System 118 ENDOCRINE SYSTEM 176

CHAPTER 22 CHAPTER 31
Respiratory Tract Infections, Neoplasms, Mechanisms of Endocrine Control 176
and Childhood Disorders 124
CHAPTER 32
CHAPTER 23 Disorders of Endocrine Control
Disorders of Ventilation of Growth and Metabolism 180
and Gas Exchange 130
CHAPTER 33
Diabetes Mellitus and the Metabolic
UNIT 7 Syndrome 187

KIDNEY AND URINARY TRACT FUNCTION 137


UNIT 10
CHAPTER 24 NERVOUS SYSTEM 194
Structure and Function of the Kidney 137
CHAPTER 34
CHAPTER 25 Organization and Control of Neural
Disorders of Renal Function 142 Function 194

CHAPTER 26 CHAPTER 35
Acute Renal Failure and Chronic Somatosensory Function, Pain, and
Kidney Disease 148 Headache 201
LWBK707-FM_i-viii.qxd 9/2/10 7:32 AM Page vii Aptara Inc

CONTENTS vii

CHAPTER 36 UNIT 12
Disorders of Neuromuscular Function 210
MUSCULOSKELETAL FUNCTION 253
CHAPTER 37
CHAPTER 42
Disorders of Brain Function 218
Structure and Function of the
CHAPTER 38 Skeletal System 253
Disorders of Special Sensory Function: CHAPTER 43
Vision, Hearing, and Vestibular
Function 226 Disorders of the Skeletal System:
Trauma, Infections, Neoplasms, and
Childhood Disorders 258
UNIT 11
CHAPTER 44
GENITOURINARY AND
REPRODUCTIVE FUNCTION 237 Disorders of the Skeletal System:
Metabolic and Rheumatic Disorders 264
CHAPTER 39
Disorders of the Male UNIT 13
Genitourinary System 237
INTEGUMENTARY FUNCTION 269
CHAPTER 40
CHAPTER 45
Disorders of the Female
Genitourinary System 242 Structure and Function of the Skin 269

CHAPTER 41
CHAPTER 46
Sexually Transmitted Infections 249 Disorders of Skin Integrity and Function 273

Answer Key 280


LWBK707-FM_i-viii.qxd 9/2/10 7:32 AM Page viii Aptara Inc
LWBK707-c1_p1-5.qxd 8/19/10 12:43 PM Page 1 Aptara Inc

CHAPTER
Cell Structure and
Function

SECTION I: LEARNING 12. Explain the process of cell differentiation in


terms of development of organ systems in
OBJECTIVES the embryo and the continued regeneration
of tissues in postnatal life.
1. State why the nucleus is called the control
center of the cell. 13. Describe the characteristics of the four
different tissue types.
2. List the cellular organelles and state their
functions. 14. Characterize the composition and functions
of the extracellular components of tissue.
3. State four functions of the cell membrane.
15. Explain the function of intercellular
4. Trace the pathway for cell communication,
adhesions and junctions.
beginning at the receptor and ending with
the effector response, and explain why the
process is often referred to as signal transduc-
tion. SECTION II: ASSESSING YOUR
5. Compare the functions of G-proteinlinked, UNDERSTANDING
ion-channellinked, and enzyme-linked cell
surface receptors. Activity A Fill in the blanks.

6. Relate the function of adenosine 1. is composed of water, proteins,


triphosphate (ATP) to cell metabolism. neutral fats, and glycogen.

7. Compare the processes involved in anaerobic 2. All cells have a nucleus,


and aerobic metabolism. whereas cells do not.

8. Discuss the mechanisms of membrane trans- 3. The nucleus contains , which


port associated with diffusion, osmosis, serves as the template for making all the
endocytosis, and exocytosis and compare , which is later used to direct
them with active transport mechanisms. the synthesis of in the
cytoplasm.
9. Describe the function of ion channels.
4. Ribosomes serve as the site for
10. Describe the basis for membrane potentials. synthesis in the cytoplasm.
11. Explain the relationship between membrane 5. endoplasmic reticulum is
permeability and generation of membrane studded with ribosomes attached to specific
potentials. binding sites on the membrane.

1
LWBK707-c1_p1-5.qxd 8/19/10 12:43 PM Page 2 Aptara Inc

2 UNIT 1 CELL AND TISSUE FUNCTION

6. The complex modifies proteins h. Division of cells


and packages them into secretory granules following mitosis
bound for the membrane. i. Organelle that
7. contain powerful hydrolytic metabolizes
enzymes that are used to break down excess misfolded proteins
and worn-out cell parts as well as foreign j. Carbohydrate and
substances. protein layer that
participates in cell
8. Peroxisomes contain a special enzyme that
recognition
degrades .
9. Mitochondria are the site of cellular
2.
, the product of which is the
formation of . Column A Column B
10. Transport along the axon of neuronal cells 1. Diffusion a. Secondary active
takes place along the primary cytoskeletal transport in which
2. Osmosis
component . substances are
3. Active moved in the same
11. Actin and myosin are examples of functional
transport direction
within muscle cells.
4. Passive b. Any type of
12. Integral proteins span the entire lipid bilayer, transport across the
transport
whereas proteins are bound to cell membrane that
one side of the membrane or the other. 5. Cotransport requires energy as
13. The four tissues of the body are , 6. Facilitated it moves material
, , and diffusion against the concen-
. tration gradient
7. Primary active
c. Secondary active
14. The differences in permeability of transport
transport in which
is responsible for the
8. Secondary substances are
generation of membrane potential.
active moved in the
Permeability is regulated by ion channels.
transport opposite direction
15. Of the four tissue types, only d. The coupling of the
9. Counter
and tissue is excitable. transport of one
transport
solute to a second
Activity B Match the key terms in Column A 10. Symport solute
with their definitions in Column B.
e. Transport across the
1. cell membrane
through a protein
Column A Column B
channel that does
1. tRNA a. Site of synthesis of not require ATP
lipid molecules f. The diffusion of
2. Flagella
b. Transfer RNA water
3. Tubulin
c. Hormone or g. Any type of
4. Glycocalyx neurotransmitter transport across the
d. Second messenger cell membrane that
5. G protein
that mediates does not require
6. Smooth ER cellular responses energy
7. Mitochondria e. Site of aerobic h. Direct use of ATP in
respiration the transport of a
8. Centrioles solute
f. Protein subunit of
9. Proteasomes microtubules i. Utilization of the
energy derived from
10. First messenger g. Sperm motility
the primary active

Copyright 2011. Wolters Kluwer Health | Lippincott Williams & Wilkins. Study Guide for Porths Essentials of Pathophysiology, Third Edition.
LWBK707-c1_p1-5.qxd 8/19/10 12:43 PM Page 3 Aptara Inc

CHAPTER 1 CELL STRUCTURE AND FUNCTION 3

transport of one 3. Signal transduction is a complex and varied


solute for the process. Describe the process starting at the
cotransport of a first messenger and ending in a physiological
second solute response. Be sure to include the various possi-
j. Passive movement bilities at the receptor level as well as the sec-
of solute down the ond messenger level.
concentration
gradient

Activity C Consider the following figure.


1. 4. Large molecules or particles are ingested or
released from cells. Describe the basics of
ingestion and release.

Extracellular
fluid

SECTION III: APPLYING


YOUR KNOWLEDGE
Cytosol
Activity E Consider the following scenario and
answer the questions.
Fourteen-year-old Thomas Kirk is brought to the
In the figure above, label phospholipid by layer, clinic for a routine physical before starting to
an individual phospholipid, an integral protein, play sports in school. He is 77 inches tall and
a peripheral protein, a channel protein, a glyco- weighs 200 pounds. Tom states, I have tried to
protein, and a glycolipid. lose weight so I can wrestle at a lower weight and
I just dont understand why I still weigh 200
Activity D Briefly answer the following. pounds. My science teacher said its because I
1. In many diseases, the root cause is ischemia have white fat and not brown fat. How would
(low blood flow) or hypoxia (decreased delivery you explain to Tom about the two kinds of
of oxygen). Using what you know about aero- adipose tissue in his body?
bic metabolism, explain how alterations in
oxygen delivery to the tissues are detrimental.
SECTION IV: PRACTICING
FOR NCLEX
Activity F Answer the following questions.

2. Tissues must maintain their shape and 1. There are two forms of endoplasmic
integrity in order to function. Explain from reticulum (ER) found in a cell. They are the
the cellular level to the tissue level what is rough and the smooth ER. What does the
responsible for maintaining tissue shape and rough ER do in a cell?
structure. a. Produces proteins
b. Combines protein with other components
of the cytoplasm
c. Exports protein from the cell
d. Destroys ribosomes

Copyright 2011. Wolters Kluwer Health | Lippincott Williams & Wilkins. Study Guide for Porths Essentials of Pathophysiology, Third Edition.
LWBK707-c1_p1-5.qxd 8/19/10 12:43 PM Page 4 Aptara Inc

4 UNIT 1 CELL AND TISSUE FUNCTION

2. The Golgi complex, or Golgi bodies, consists 7. The Krebs cycle provides a common pathway
of stacks of thin, flattened vesicles or sacs for the metabolism of nutrients by the body.
within the cell. These Golgi bodies are found The Krebs cycle forms two pyruvate
near the nucleus and function in association molecules. Each of the two pyruvate
with the ER. What is one purpose of the molecules formed in the cytoplasm from one
Golgi complex? molecule of glucose yields another molecule
a. Produce bile of what?
b. Receive proteins and other substances from a. FAD
the cell surface by a retrograde transport b. NADH  H
mechanism c. ATP
c. Produce excretory granules d. H2O
d. Produce small carbohydrate molecules
8. When cells use energy to move ions against
3. In Tay-Sachs disease, an autosomal recessive an electrical or chemical gradient, the process
disorder, hexosaminidase A, which is the is called what?
lysosomal enzyme needed for degrading the a. Passive transport
GM2 ganglioside found in nerve cell
b. Neutral transport
membranes, is deficient. Although GM2
ganglioside accumulates in many tissues, c. Cotransport
where does it do the most harm? d. Active transport
a. Brain and retinas 9. Groups of cells that are closely associated in
b. Retinas and heart structure and have common or similar func-
c. Nervous system and retinas tions are called tissues. What are the types of
tissue in the human body?
d. Nervous system and brain
a. Connective and muscle tissue
4. The mitochondria are literally the power
b. Binding and connecting tissue
plants of the cell because they transform
organic compounds into energy that is c. Nerve and exothelium tissue
easily accessible to the cell. What do the d. Exothelium and muscle tissue
mitochondria do?
10. Endocrine glands are epithelial structures
a. Make energy that have had their connection with the
b. Form proteasomes surface obliterated during development.
c. Needs DNA from other sources to replicate How are these glands described?
d. Extracts energy from organic compounds a. Ductile and produce secretions
b. Ductless and produce secretions
5. The cell membrane is also called what?
c. Ductile and release their glandular
a. Plasma membrane
products by exocytosis
b. Nuclear membranes
d. Ductless and release their glandular
c. Receptor membrane products by exocytosis
d. Bilayer membrane
6. Some messengers, such as thyroid hormone
and steroid hormones, do not bind to mem-
brane receptors but move directly across the
lipid layer of the cell membrane and are car-
ried to the cell nucleus. What do they do at
the cell nucleus?
a. Transiently open or close ion channels
b. Influence DNA activity
c. Stabilize cell function
d. Decrease transcription of mRNA

Copyright 2011. Wolters Kluwer Health | Lippincott Williams & Wilkins. Study Guide for Porths Essentials of Pathophysiology, Third Edition.
LWBK707-c1_p1-5.qxd 8/19/10 12:43 PM Page 5 Aptara Inc

CHAPTER 1 CELL STRUCTURE AND FUNCTION 5

11. Each skeletal muscle is a discrete organ made 14. Cells in multicellular organisms need to
up of hundreds or thousands of muscle fibers. communicate with one another to coordinate
Although muscle fibers predominate, their function and control their growth.
substantial amounts of connective tissue, The human body has several means of trans-
blood vessels, and nerve fibers are also mitting information between cells, what are
present. What happens during muscle they? (Mark all that apply.)
contraction? a. Direct communication between adjacent
a. When activated by GTP (guanosine cells
5-triphosphate), the cross-bridges swivel b. Express communication between cells
in a fixed arc, much like the oars of a boat,
c. Autocrine and paracrine signaling
as they become attached to the actin fila-
ment. d. Endocrine or synaptic signaling
b. During contraction, each cross-bridge un- 15. The human body has nondividing cells that
dergoes its own cycle of movement, form- have left the cell cycle and are not capable of
ing a bridge attachment and releasing it, mitotic division once an infant is born. What
the same sequence of movement repeats it- are the nondividing cells? (Mark all that
self when the cross-bridge reattaches to the apply.)
same cell. a. Mucous cells
c. The thick myosin and thin actin filaments b. Neurons
slide over each other, causing shortening
c. Skeletal muscle cells
of the muscle fiber.
d. Cardiac muscle cells
d. Calciumcalmodulin complexes produce
the sliding of the filaments that form 16. Smooth muscle is often called
cross-bridges with the thin actin filaments. muscle because it contracts spontaneously or
through activity of the autonomic nervous
12. The three main parts of a cell are the nucleus,
system.
the , and the cell membrane.
13. Bilirubin is a normal major pigment of bile;
its excess accumulation within cells is
evidenced clinically by a yellowish
discoloration of the skin and sclera, a
condition called .

Copyright 2011. Wolters Kluwer Health | Lippincott Williams & Wilkins. Study Guide for Porths Essentials of Pathophysiology, Third Edition.
LWBK707-c2_p6-11.qxd 8/19/10 12:44 PM Page 6 Aptara Inc

CHAPTER
Cellular Responses to
Stress, Injury, and Aging

SECTION I: LEARNING 10. State how nutritional imbalances contribute


to cell injury.
OBJECTIVES
11. Describe three types of reversible cell changes
1. Cite the general purpose of changes in cell that can occur with cell injury.
structure and function that occur as the
12. Define free radical and reactive oxygen species.
result of normal adaptive growth and
differentiation. 13. Relate free radical formation and oxidative
stress to cell injury and death.
2. Describe cell changes that occur with
atrophy, hypertrophy, hyperplasia, meta- 14. Describe cell changes that occur with
plasia, and dysplasia and state general ischemic and hypoxic cell injury.
conditions under which the changes occur.
15. Relate the effects of impaired calcium home-
3. Cite three sources of intracellular accumula- ostasis to cell injury and death.
tions.
16. Differentiate cell death associated with
4. Compare the pathogenesis and effects of necrosis and apoptosis.
dystrophic and metastatic calcifications.
17. Cite the reasons for the changes that occur
5. Describe the mechanisms whereby physical with the wet and dry forms of gangrene.
agents such as blunt trauma, electrical forces,
and extremes of temperature produce cell
injury.
SECTION II: ASSESSING
6. Differentiate between the effects of ionizing YOUR UNDERSTANDING
and nonionizing radiation in terms of their
ability to cause cell injury. Activity A Fill in the blanks.
7. Explain how the injurious effects of biologic 1. Cells may adapt to the environment by
agents differ from those produced by undergoing changes in ,
physical and chemical agents. , and .
8. State the mechanisms and manifestations of 2. Atrophy is seen as a decrease in cell
cell injury associated with lead toxicity. .
9. Identify the causes and outcomes of mercury 3. Denervation will result in cellular
toxicity. .
4. Hypertrophy is an in cell size.

6
LWBK707-c2_p6-11.qxd 8/19/10 12:44 PM Page 7 Aptara Inc

CHAPTER 2 CELLULAR RESPONSES TO STRESS, INJURY, AND AGING 7

5. An increase in muscle mass associated with Activity B Consider the following figure.
exercise is an example of .
6. An increase in the number of cells in an
organ or tissue is known as cellular
. Nucleus

7. Liver regrowth is an example of Basement


hyperplasia. membrane

8. or hyperplasia is
due to excessive hormonal stimulation or
excessive growth factors.
9. represents a reversible change
in which one adult cell type is replaced by
another adult cell type.
10. Metaplasia usually occurs in response to
chronic and and
allows for substitution of cells that are better
able to survive stressful or harmful
conditions.
11. Deranged cell growth of a specific tissue that
results in cells that vary in size, shape, and
organization is known as .
12. Dysplasia is strongly implicated as a precursor
of .
13. Intracellular represent the
buildup of substances that cells cannot
immediately use or eliminate.
14. radicals are highly reactive
chemical species having an unpaired electron
in the outer valence shell of the molecule.
15. deprives the cell of oxygen and
interrupts oxidative metabolism and the
generation of adenosine triphosphate (ATP).
16. Reversible cellular injury is seen as either
cellular or
accumulation.
17. differs from apoptosis in that it
involves unregulated enzymatic digestion of
cell components, loss of cell membrane
integrity with uncontrolled release of the
products of cell death into the intracellular The figure pictured above represents cellular
space, and initiation of the inflammatory adaptation. Label each adaptation and state
response. whether it is a physiologic, pathologic, or if it
could be both types of adaptations.
18. The increased levels may inap-
propriately activate a number of enzymes
with potentially damaging effects.
19. Acidosis develops and denatures the
enzymatic and structural proteins of the cell
during necrosis.

Copyright 2011. Wolters Kluwer Health | Lippincott Williams & Wilkins. Study Guide for Porths Essentials of Pathophysiology, Third Edition.
LWBK707-c2_p6-11.qxd 8/19/10 12:44 PM Page 8 Aptara Inc

8 UNIT 1 CELL AND TISSUE FUNCTION

Activity C Match the pathologic process in 2. List the five categories of cellular injury.
Column A with their description in Column B.
Column A Column B
1. Metastatic a. Macroscopic deposi-
calcification tion of calcium salts
in injured tissue 3. Lead has been found in paint used to give
2. Reactive childrens toys their brilliant colors. Why is
oxygen species b. Oxygen-containing
molecules that are this a concern?
(ROS)
highly reactive
3. Antioxidants
c. Ice crystal
4. Apoptosis formation in
cytosol
5. Dystrophic
calcification d. Natural and 4. List and describe the three major mechanisms
synthetic molecules of cellular injury.
6. Temperature- that inhibit the
induced injury reactions of ROS
7. Ischemia with biological
structures
8. Caseous
e. Occurs in normal
necrosis 5. Oxidative stress has been implicated as the
tissues as the result
9. Ionizing of increased serum causative agent in numerous disease states as
radiation calcium levels well as the cause of physiological aging.
f. Impaired oxygen Explain how oxidative stress can cause
10. Gangrene damage and why it is a concern.
delivery
g. Programmed cell
death
h. Causes injury by
changes in electron
stability 6. Explain why one of the complications of
i. Dead cells persist hypoxia is the development of acidosis and
indefinitely as soft how the acidosis will damage the tissue.
cheeselike debris
j. Term applied when
a considerable mass
of tissue undergoes
necrosis
7. Apoptosis takes place under normal
stimulation or as the result of cellular injury.
Activity D Briefly answer the following. There are two pathways for apoptosis to occur.
What are they and what major protein is
1. Why does chronically damaged tissue result in
involved?
calcification?

Copyright 2011. Wolters Kluwer Health | Lippincott Williams & Wilkins. Study Guide for Porths Essentials of Pathophysiology, Third Edition.
LWBK707-c2_p6-11.qxd 8/19/10 12:44 PM Page 9 Aptara Inc

CHAPTER 2 CELLULAR RESPONSES TO STRESS, INJURY, AND AGING 9

SECTION III: APPLYING 2. Hypertrophy may occur as the result of


normal physiologic or abnormal pathologic
YOUR KNOWLEDGE conditions. The increase in muscle mass asso-
ciated with exercise is an example of physio-
Activity E Consider the scenario and answer
logic hypertrophy. Pathologic hypertrophy
the question.
occurs as the result of disease conditions and
Your child is acting more clumsy than normal may be adaptive or compensatory. Examples
and is not communicating well. In the doctors of adaptive hypertrophy are the thickening
office you are told she has lead poisoning. of the urinary bladder from long-continued
obstruction of urinary outflow and the
1. How does lead affect the nervous system?
myocardial hypertrophy that results from
valvular heart disease or hypertension. What
is compensatory hypertrophy?
a. When the body increases its major organs
during times of malnutrition
b. When one kidney is removed, the remain-
ing kidney enlarges to compensate for the
SECTION IV: PRACTICING loss
FOR NCLEX c. When the body controls myocardial
growth by stimulating actin expression to
Activity F Answer the following questions. enlarge the heart
1. Many molecular mechanisms mediate cellu- d. When the body stimulates gene expression
lar adaptation. Some are factors produced by to begin a progressive decrease in left
other cells and some by the cells themselves. ventricular muscle mass
These mechanisms depend largely on signals 3. Metastatic calcification takes place in normal
transmitted by chemical messengers that tissues as the result of increased serum
exert their effects by altering the function of calcium levels (hypercalcemia). Anything
a gene. Many adaptive cellular responses alter that increases the serum calcium level can
the expression of differentiation genes. lead to calcification in inappropriate places
What can cells do because of this? such as the lung, renal tubules, and blood
a. A cell is able to change size or form with- vessels. What are the major causes of
out compromising its normal function hypercalcemia?
b. A cell incorporates its change in function a. Diabetes mellitus and Paget disease
and passes this change on to other cells b. Hypoparathyroidism and vitamin D
like it. intoxication
c. A cell is able to pass its change on to a c. Hyperparathyroidism and immobilization
housekeeping cell
d. Immobilization and hypoparathyroidism
d. A cell dies once the stimulus to change has
been removed 4. Mercury is a toxic substance, and the hazards
of mercury-associated occupational and
accidental exposures are well known. What is
the primary concern for the general public in
regard to mercury poisoning today?
a. Amalgam fillings in the teeth
b. Mercury from thermometers and blood
pressure machines
c. Mercury found in paint that was made
before 1990
d. Fish such as tuna and swordfish

Copyright 2011. Wolters Kluwer Health | Lippincott Williams & Wilkins. Study Guide for Porths Essentials of Pathophysiology, Third Edition.
LWBK707-c2_p6-11.qxd 8/19/10 12:44 PM Page 10 Aptara Inc

10 UNIT 1 CELL AND TISSUE FUNCTION

5. Small amounts of lead accumulate to reach b. After being out in the cold all night your
toxic levels in the human body. Lead is found toes and feet are frozen and it will be very
in many places in the environment and is painful to warm them again, and the
still a major concern in the pediatric popula- health care team is concerned he might be
tion. What would you teach the parents of a a drug addict.
child who is being tested for lead poisoning? c. It is obvious that you are a homeless per-
a. Keep your child away from peeling paint. son and we were wondering how often this
b. Keep your child away from anything ce- has happened to you before and when it
ramic. will happen again.
c. Do not let your child read newspapers. d. Your toes and feet are frozen and there is a
concern about the formation of blood clots
d. Do not let your child tour a mine on a
as we warm them again.
school field trip.
9. Clinical manifestations of radiation injury
6. In a genetic disorder called xeroderma
result from acute cell injury, dose-dependent
pigmentosum, an enzyme needed to repair
changes in the blood vessels that supply the
sunlight-induced DNA damage is lacking.
irradiated tissues, and fibrotic tissue replace-
This autosomal recessive disorder is
ment. What are these clinical manifestations?
characterized by what?
a. Radiation cystitis, dermatitis, and diarrhea
a. Patches of pink, leathery pigmentation
from enteritis
replace normal skin after a sunburn.
b. Dermatitis, diarrhea from enteritis, and
b. Extreme photosensitivity and a greatly
hunger
increased risk of skin cancer in skin that
has been exposed to the sun c. Diarrhea from enteritis, hunger, and mus-
cle spasms
c. White, scaly patches of skin that appear on
African American people after they have a d. Radiation cystitis, diarrhea from enteritis,
sunburn and muscle spasms
d. Photosensitivity and a decreased risk of 10. Biologic agents differ from other injurious
skin cancer in skin that has been exposed agents in that they are able to replicate and
to the sun. can continue to produce their injurious
effects. How do Gram-negative bacteria cause
7. While presenting a talk to the parents of
harm to the cell?
preschoolers at a local day care center, the
nurse is asked about electrical injury to the a. Gram-negative bacilli excrete elaborate ex-
body. She would know to include what in her otoxins that interfere with cellular produc-
response? tion of ATP.
a. In electrical injuries, the body acts as a b. Gram-negative bacilli release endotoxins
deflector of the electrical current. that cause cell injury and increased capil-
lary permeability.
b. In electrical injuries, the body acts as a
magnifier of the electrical current. c. Gram-negative bacilli enter the cell and
disrupt its ability to replicate.
c. The most severe damage is caused by
lightning and high-voltage wires d. Gram-negative bacilli cannot cause harm
to the cell; only Gram-positive bacilli can
d. When a person touches an electrical
harm the cell.
source, the current passes through the
body and exits to another receptor. 11. When confronted with a decrease in work
demands or adverse environmental
8. A man presents to the emergency department
conditions, most cells are able to revert to a
after being out in below zero weather all
smaller size and a lower and more efficient
night. He asks the nurse why the health care
level of functioning that is compatible with
team is concerned about his toes and feet.
survival. This decrease in cell size is called
How would the nurse respond?
.
a. Cold causes injury to the cells in the body
by injuring the blood vessels, making
them leak into the surrounding tissue.

Copyright 2011. Wolters Kluwer Health | Lippincott Williams & Wilkins. Study Guide for Porths Essentials of Pathophysiology, Third Edition.
LWBK707-c2_p6-11.qxd 8/19/10 12:44 PM Page 11 Aptara Inc

CHAPTER 2 CELLULAR RESPONSES TO STRESS, INJURY, AND AGING 11

12. Match the pigments (Column A) with what 14. You are a nurse preparing an educational
they cause in the body (Column B). event for a group of single parents. You are
going to talk about drugs and the damage
Column A Column B
they can cause to the body. You would know
1. Icterus a. A yellow to include which of these? (Mark all that
discoloration of apply.)
2. Lipofuscin
tissue a. Acetaminophen and aspirin
3. Carbon b. A blue lead line b. Immunosuppressant drugs
4. Melanin along the margins
c. Alcohol and cigarettes
of the gum
d. Vitamin supplements and antineoplastic
c. A brown or dark-
drugs
brown pigment that
is found in the skin
and hair
d. A yellow-brown
pigment that accu-
mulates in neurons

13. Match the type of agent causing cell injury


(Column A) to the agent (Column B).
Column A Column B
1. Physical agent a. Submicroscopic
viruses
2. Chemical agent
b. Mechanical forces
3. Biologic agents that produce tissue
4. Nutritional trauma
factors c. Free radicals
d. Vitamin B
deficiency

Copyright 2011. Wolters Kluwer Health | Lippincott Williams & Wilkins. Study Guide for Porths Essentials of Pathophysiology, Third Edition.
LWBK707-c3_p12-16.qxd 8/19/10 2:22 PM Page 12 Aptara Inc

CHAPTER
Inflammation, the
Inflammatory
Response, and Fever

SECTION I: LEARNING SECTION II: ASSESSING YOUR


OBJECTIVES UNDERSTANDING
1. State the five cardinal signs of acute Activity A Fill in the blanks.
inflammation and describe the physiologic
1. is a protective response
mechanisms involved in production of these
intended to eliminate the initial cause of cell
signs.
injury, remove the damaged tissue, and
2. Describe the vascular changes in an acute generate new tissue.
inflammatory response.
2. The cardinal signs of inflammation are
3. Characterize the interaction of adhesion , , ,
molecules, chemokines, and cytokines in and .
leukocyte adhesion, migration, and
3. In addition to the cardinal signs that appear at
phagocytosis, which are part of the cellular
the site of injury, manifestations
phase of inflammation.
may occur as chemical mediators produced at
4. List four types of inflammatory mediators the site of inflammation gain entrance to the
and state their function. circulatory system.
5. Contrast acute and chronic inflammation. 4. inflammation is of relatively
short duration, lasting from a few minutes,
6. Describe the causes of chronic inflammation
whereas inflammation is of a
and the role of granuloma formation.
longer duration, lasting for days to years.
7. Define the systemic manifestations of
5. Acute inflammation involves two major
inflammation, including the characteristics
components: the and
of an acute-phase response.
stages.
8. Describe the normal mechanism of body 6. Increased circulating white blood cells are a
temperature regulation. condition known as .
9. Characterize the inflammatory initiation of a
febrile response.
10. Explain how age and fever are related.

12
LWBK707-c3_p12-16.qxd 8/19/10 2:22 PM Page 13 Aptara Inc

CHAPTER 3 INFLAMMATION, THE INFLAMMATORY RESPONSE, AND FEVER 13

7. produce prostaglandins and 20. fragments contribute to the


leukotrienes, platelet-activating factor, inflammatory response by causing vasodilation,
inflammatory cytokines, and growth factors increasing vascular permeability; and
that promote regeneration of tissues. enhancing the activity of phagocytes.
8. changes that occur with 21. Activation of the system results
inflammation involve the arterioles, in release of bradykinin, which increases
capillaries, and venules of the microcirculation. vascular permeability and causes contraction
of , dilation of blood vessels,
9. The selectins function in adhesion of
and .
to endothelial cells.
22. , a cytokine that will induce
10. The integrins promote and
endothelial cells to express adhesion
cell-to-extracellular matrix interactions.
molecules and release cytokines, chemokines,
11. Chemotaxis is dynamic and energy-directed and reactive oxygen species, is released from
process of directed . mast cells.
12. Groups of proteins that direct the trafficking 23. The radical, , and
of leukocytes during the early stages of radical are the major free
inflammation or injury are known as oxygen radicals produced within the cell.
.
24. At higher levels, free radicals mediators can
13. The pathways generate toxic produce .
oxygen and nitrogen products.
25. The acute inflammatory response involves
14. The plasma-derived mediators of inflammation the production of ; they can be
include the factors and the serous, hemorrhagic, fibrinous, membranous,
proteins. or purulent.
15. Histamine causes of arterioles 26. Agents that evoke chronic inflammation
and increases the of venules. typically are low-grade, persistent infections
or irritants that are unable to or
16. The family inflammatory medi-
.
ators consist of prostaglandins, leukotrienes,
and related metabolites. 27. The function of the acute-phase protein
is thought to be protective, in
17. The induce inflammation and
that it binds to the surface of invading
potentiate the effects of histamine and other
microorganisms and targets them for destruc-
inflammatory mediators.
tion by complement and phagocytosis.
18. Aspirin and the nonsteroidal anti-inflammatory
28. is one of the most prominent
drugs (NSAIDs) reduce inflammation by inac-
manifestations of the acute-phase response.
tivating the first enzyme in the
pathway for prostaglandin synthesis. 29. Virtually all biochemical processes in the
body are affected by changes in .
19. Eating oily fish and other foods that are high
in results in partial replacement 30. There are numerous under the
of arachidonic acid in inflammatory cell skin surface that allow blood to move directly
membranes, which leads to decreased from the arterial to the venous system.
production of arachidonic acid-derived
inflammatory mediators.

Copyright 2011. Wolters Kluwer Health | Lippincott Williams & Wilkins. Study Guide for Porths Essentials of Pathophysiology, Third Edition.
LWBK707-c3_p12-16.qxd 8/19/10 2:22 PM Page 14 Aptara Inc

14 UNIT 1 CELL AND TISSUE FUNCTION

Activity B Consider the following figure. h. Activation affects


vascular permeabil-
ity, chemotactic,
Injured tissue,
inflammatory mediators adhesive, and prote-
olytic properties
Cell membrane phospholipids i. Swelling due to
Corticosteroid
movement of fluid
medications from vasculature
Arachidonic acid into tissues
j. Outpouring of a
protein-rich fluid
Lipoxygenase Cyclooxygenase
pathway pathway into the tissue and
Aspirin, NSAIDs
extravascular space
Leukotrienes
(LTC 4, LTD 4, LTE 4) Activity D Put the following events into the
Prostaglandins Thromboxane
(PGI2, PGF2a) (TxA2) proper order.
Induces smooth muscle S S S
contraction Induces vasodilation and Vasoconstriction
Constricts pulmonary bronchoconstriction Bronchoconstriction
airways Inhibits inflammatory Promotes platelet Chemotaxis
Increases microvascular cell function function
permeability Margination and adhesion to the endothelium
Activation and phagocytosis
Transmigration across the endothelium
1. What does this figure represent? Explain the
process that is depicted.
Activity E Briefly answer the following.
1. The cardinal signs of inflammation result
Activity C Match the key terms in Column A
from the physiologic processes of the inflam-
with their definitions in Column B.
matory cells and protein systems. List the
Column A Column B signs and give a brief explanation as to its
cause.
1. Endothelial a. Increase in the
cells blood during
allergic reactions
2. Eosinophils
b. Leukocyte
3. Edema accumulation
4. Neutrophils c. Regulate leukocyte 2. Describe and differentiate between acute and
extravasation chronic inflammation.
5. Exudate
d. Stimulate
6. Nitric oxide inflammatory reac-
7. Margination tion in response to
injury or infection
8. Thrombocytes
e. Circulating cells
3. The vascular response of inflammation follows
9. Mast cells similar to mast cells
one of three patterns. Describe these patterns
10. Basophils f. Primary phagocyte and explain why it is necessary to have
that arrives early at multiple responses.
the site of
inflammation
g. Stimulator of
vasodilation

Copyright 2011. Wolters Kluwer Health | Lippincott Williams & Wilkins. Study Guide for Porths Essentials of Pathophysiology, Third Edition.
LWBK707-c3_p12-16.qxd 8/19/10 2:22 PM Page 15 Aptara Inc

CHAPTER 3 INFLAMMATION, THE INFLAMMATORY RESPONSE, AND FEVER 15

4. Many leukocytes have the ability to phagocy- 2. Several days after injury, a family member asks
tose foreign material and dispose of it. The why the client isnt eating. What kind of
process involves three steps. List and explain information would you give the person?
these steps.

5. There are many mediators of the inflammatory


system. They may be grouped by function. SECTION IV: PRACTICING
Describe each group and give a brief example FOR NCLEX
of each.
Activity G Answer the following questions.
1. The cardinal signs of inflammation include
swelling, pain, redness, and heat. What is the
fifth cardinal sign of inflammation?
6. Explain and describe the two types of chronic a. Loss of function
inflammation. b. Altered level of consciousness
c. Sepsis
d. Fever
2. The cells that are associated with allergic disor-
ders and the inflammation associated with
7. What is the purpose of the acute-phase immediate hypersensitive reactions are known
response of inflammation? as what? (Mark all that apply.)
a. Macrophages
b. Eosinophils
c. Mast cells
d. Neutrophils
e. Basophils
SECTION III: APPLYING 3. Inflammation can be either acute or chronic.
YOUR KNOWLEDGE The immune system is thought to play a role
in chronic inflammation and may be one of
Activity F Consider the scenario and answer the reasons chronic inflammation may persist
the questions. for days to months to years. Why is the risk of
scarring and deformity greater in chronic
You are the nurse caring for a burn victim who inflammation than it is in acute
has sustained second- and third-degree burns inflammation?
over 50% of the body. The family is asking you
a. Chronic inflammation is the persistent
questions about the care that is being given to
destruction of healthy tissue.
the burn victim.
b. Fibroblasts instead of exudates proliferate
1. A family member asks about the drainage they in chronic inflammation.
see on the bandages. What would you tell
c. Typically, agents that evoke chronic
them?
inflammation are infections or irritants that
penetrate deeply and spread rapidly.
d. Chronic inflammation is often the result of
allergic reactions.

Copyright 2011. Wolters Kluwer Health | Lippincott Williams & Wilkins. Study Guide for Porths Essentials of Pathophysiology, Third Edition.
LWBK707-c3_p12-16.qxd 8/19/10 2:22 PM Page 16 Aptara Inc

16 UNIT 1 CELL AND TISSUE FUNCTION

4. All wounds are considered contaminated at 6. Inflammation can be either local of systemic.
the time the wound occurs. Usually the natu- What are the most prominent systemic mani-
ral defenses in our bodies can deal with the festations of inflammation?
invading microorganisms at the time the a. Fever, leukocytosis or leukopenia, and the
wound occurs; however, there are times when acute-phase response
a wound is badly contaminated and host
b. Fever, leukocytosis or leukopenia, and the
defenses are overwhelmed. What happens to
transition-phase response
the healing process when host defenses are
overwhelmed by infectious agents? c. Widening pulse pressure,
thrombocytopenia, and the recovery-phase
a. The inflammatory response is shortened
response
and does not complete destruction of the
invading organisms. d. Widening pulse pressure,
thrombocytopenia, and the latent-phase
b. Fibroblast production becomes malignant
response
because of hypersensitization by invading
organisms.
c. The formation of granulation tissue is
impaired.
d. Collagen fibers cannot draw tissues
together.
5. During the acute inflammatory response there
is a period called the transient phase, where
there is increased vascular permeability. What
is considered the principal mediator of the
immediate transient phase?
a. Histamine
b. Arachidonic acid
c. Fibroblasts
d. Cytokines

Copyright 2011. Wolters Kluwer Health | Lippincott Williams & Wilkins. Study Guide for Porths Essentials of Pathophysiology, Third Edition.
LWBK707-c4_p17-19.qxd 8/20/10 11:53 PM Page 17 Aptara

CHAPTER
Cell Proliferation and
Tissue Regeneration
and Repair

SECTION I: LEARNING 10. Explain the effects of malnutrition; ischemia


and oxygen deprivation; impaired immune
OBJECTIVES and inflammatory responses; and infection,
wound separation, and foreign bodies on
1. Distinguish between cell proliferation and
wound healing.
differentiation.
11. Discuss the effect of age on wound
2. Describe the phases of the cell cycle.
healing.
3. Explain the function of cyclins, cyclin-
dependent kinases, and cyclin-dependent
kinase inhibitors in terms of regulating the SECTION II: ASSESSING
cell cycle.
YOUR UNDERSTANDING
4. Describe the properties of stem cells.
Activity A Fill in the blanks.
5. Define the terms parenchymal and stromal as
they relate to the tissues of an organ. 1. Cancer is a disorder of altered cell
and .
6. Compare labile, stable, and permanent cell
types in terms of their capacity for regeneration. 2. The process of cell division results in cellular
.
7. Describe healing by primary and secondary
intention. 3. is the process of specialization
whereby new cells acquire the structure and
8. Explain the effects of soluble mediators and
function of the cells they replace.
the extracellular matrix on tissue repair and
wound healing. 4. Proteins called controls entry
and progression of cells through the cell
9. Trace the wound-healing process through the
cycle.
inflammatory, proliferative, and remodeling
phases. 5. Kinases are enzymes that
proteins.

17
LWBK707-c4_p17-19.qxd 8/20/10 11:53 PM Page 18 Aptara

18 UNIT 1 CELL AND TISSUE FUNCTION

6. Continually renewing cell populations rely 8. Thrombocytes e. Defines the differen-


on cells of the same lineage tiation potential of
9. Mast cells
that have not yet differentiated to the extent stem cells
that they have lost their ability to divide. 10. Cellular f. Process of cell
potency specialization
7. cells remain incompletely
undifferentiated throughout life. g. Stimulator of
vasodilation
8. stem cells are pluripotent cells
derived from the inner cell mass of the h. Activation affects
blastocyst stage of the embryo. vascular
permeability
9. Body organs and tissues are composed of two
i. Swelling due to
types of structures: and
movement of fluid
.
from vasculature
10. are those that continue to into tissues
divide and replicate throughout life, j. Stem cells undergo-
replacing cells that are continually being ing numerous
destroyed. mitotic divisions
11. Cells that are capable of undergoing regener- while maintaining
ation when confronted with an appropriate an undifferentiated
stimulus and are thus capable of reconstitut- state space
ing the tissue of origin are termed
. Activity C Put the following events into the
proper order.
12. tissue is a glistening red, moist
connective tissue that contains newly formed S S S
capillaries, proliferating fibroblasts, and Chemotaxis
residual inflammatory cells.
Margination and adhesion to the endothelium
13. The elderly have reduced and Activation and phagocytosis
synthesis, impaired wound
Transmigration across the endothelium
contraction, and slower reepithelialization of
open wounds.
Activity D Briefly answer the following.
14. The is often born with
1. Not all cells in the body can re-enter the cell
immature organ systems and minimal energy
cycle, but some will do so continuously. In
stores but high metabolic requirementsa
terms of regeneration and differentiation,
condition that predisposes to impaired
which types of cells will or will not re-enter
wound healing.
the cell cycle?
Activity B Match the key terms in Column A
with their definitions in Column B.
Column A Column B
1. Endothelial a. Process of cell
2. Explain the concept of wound healing by first
cells division
and second intent.
2. Proliferation b. Leukocyte
accumulation
3. Edema
c. Regulate leukocyte
4. Differentiation extravasation
5. Renewal d. Stimulate
inflammatory
6. Nitric oxide reaction in response
7. Margination to injury or
infection

Copyright 2011. Wolters Kluwer Health | Lippincott Williams & Wilkins. Study Guide for Porths Essentials of Pathophysiology, Third Edition.
LWBK707-c4_p17-19.qxd 8/20/10 11:53 PM Page 19 Aptara

CHAPTER 4 CELL PROLIFERATION AND TISSUE REGENERATION AND REPAIR 19

SECTION III: APPLYING 2. Hyperbaric treatment for wound healing is


used for wounds that have problems in heal-
YOUR KNOWLEDGE ing due to hypoxia or infection. It works by
raising the partial pressure of oxygen in
Activity E Consider the scenario and answer
plasma. How does hyperbaric oxygen
the questions.
treatment enhance wound healing?
You are the nurse caring for a burn victim who a. Destruction of anaerobic bacteria
has sustained second- and third-degree burns
b. Increased action of eosinophils
over 50% of his body. The family is asking you
questions about the care that is being given to c. Promotion of angiogenesis
the burn victim. d. Decrease in fibroblast activity
1. A family member asks about the drainage they 3. Wound healing is more difficult for persons at
see on the bandages. What would you tell both ends of the age spectrum, although the
them? reasons differ. In the elderly, wound healing is
impaired or delayed because of structural and
functional changes in the skin that occur with
aging and the chronicity of wounds the
elderly have. Why do neonates and small chil-
dren have problems with wound healing?
2. Several days post injury a family member asks a. Their body is not yet capable of an inflam-
why the client isnt eating. What kind of matory response
information would give them?
b. The fragility of their skin
c. They dont have the reserves needed
d. Their immune system is hypersensitive to
infectious agents
4. All wounds are considered contaminated at
the time the wound occurs. Usually the natu-
ral defenses in our bodies can deal with the
SECTION IV: PRACTICING invading microorganisms at the time the
FOR NCLEX wound occurs; however, there are times when
a wound is badly contaminated and host
Activity F Answer the following questions. defenses are overwhelmed. What happens to
1. A class of student nurses is hearing a lecture the healing process when host defenses are
on wound healing. The professor explains overwhelmed by infectious agents?
about primary and secondary healing. The a. The inflammatory response is shortened
professor continues to talk about the phases of and does not complete destruction of the
wound healing and states that in both primary invading organisms.
and secondary healing the phases of wound b. Fibroblast production becomes malignant
healing occur at different rates. What are the due to hypersensitization by invading
phases of wound healing? (Mark all that organisms.
apply.)
c. The formation of granulation tissue is
a. The activation phase impaired.
b. The proliferative phase d. Collagen fibers cant draw tissues together.
c. The nutritional phase
5. In normal tissue the size of the cell population
d. The inflammatory phase is determined by which of the following?
e. The maturational phase a. Balance of cell proliferation
b. Death by apoptosis
c. Emergence of newly differentiated cells

Copyright 2011. Wolters Kluwer Health | Lippincott Williams & Wilkins. Study Guide for Porths Essentials of Pathophysiology, Third Edition.
LWBK707-c5_p20-25.qxd 8/19/10 2:24 PM Page 20 Aptara Inc

CHAPTER
Genetic Control of
Cell Function and
Inheritance

SECTION I: LEARNING 12. Differentiate between genetic and physical


maps.
OBJECTIVES
13. Briefly describe the methods used in linkage
1. Describe the structure and function of DNA. studies, dosage studies, and hybridization
studies.
2. Relate the mechanisms of DNA repair to the
development of a gene mutation. 14. Describe the goals of the International
HapMap Project.
3. Describe the function of messenger RNA,
ribosomal RNA, and transfer RNA as they 15. Describe the process of recombinant DNA
relate to protein synthesis. technology.
4. Cite the effects of posttranslational 16. Characterize the process of RNA interference.
processing on protein structure and function.
5. Explain the role of transcription factors in
regulating gene activity.
SECTION II: ASSESSING
6. Define the terms autosomes, chromatin, meio- YOUR UNDERSTANDING
sis, and mitosis.
7. List the steps in constructing a karyotype Activity A Fill in the blank.
using cytogenetic studies. 1. Our genetic information is stored in the
8. Explain the significance of the Barr body. structure of acid.

9. Construct a hypothetical pedigree for a reces- 2. acid serves as the template for
sive and dominant trait according to protein synthesis.
Mendel's laws. 3. The complete set of proteins encoded by the
10. Contrast genotype and phenotype. genome is known as the .

11. Define the terms allele, locus, expressivity, and 4. A precise complementary pairing of
penetrance. and bases occurs
in the double-stranded DNA molecule.

20
LWBK707-c5_p20-25.qxd 8/19/10 2:24 PM Page 21 Aptara Inc

CHAPTER 5 GENETIC CONTROL OF CELL FUNCTION AND INHERITANCE 21

5. DNA replication is semiconservative, 19. The position of a gene on a chromosome is


meaning the parental DNA strands dissociate called its , and alternate forms
and pair with strands to of a gene at the same locus are called
complete mitosis. .
6. Human somatic cells contain 20. A is a graphic method for
pairs of different chromosomes. portraying a family history of an inherited
trait
7. In the nucleus, DNA is in the form of
and during mitosis, it
Activity B Match the key terms in Column A
condenses into .
with their definitions in Column B.
8. The genetic code is repeat of
1.
bases.
Column A Column B
9. Errors in DNA duplication are known as
. 1. tRNA a. Used to align amino
acids with
10. A represents the variations in 2. Transcription
ribosomes for the
the genetic code that are responsible for the factors
formation of
differences between individuals.
3. Penetrance protein
11. Messenger RNA is formed in the process of b. Ability of a gene to
4. mRNA
. express its function
5. Mitosis
12. The coding sequence of an mRNA molecule is c. Initiate and regulate
known as . 6. Meiosis transcription
13. undergoes the process of 7. Expressivity d. Manner in which
to form a protein in the the gene is expressed
8. Chromosomes
cytosol. e. Template that is
9. Multifactorial copied from DNA
14. Molecular assist in the folding
inheritance f. Replicating germ
of proteins into their three-dimensional
conformation. 10. Single gene cells
inheritance g. Multiple alleles at
15. The degree to which a gene or particular
different loci affect
group of genes is activated is termed gene
the outcome
.
h. Organized and
16. DNA determines the type of biochemical condensed DNA
product that is needed by the cell and directs
i. One pair of genes is
its synthesis, but it is , through
involved in the
the process of transcription and translation
transmission of
that is responsible for the actual assembly
information
of the products.
j. Duplication of
17. occurs in the cell nucleus and somatic cells
involves the synthesis of RNA from a DNA
template.
18. The pattern of gene expression and the
outward presentation is the .

Copyright 2011. Wolters Kluwer Health | Lippincott Williams & Wilkins. Study Guide for Porths Essentials of Pathophysiology, Third Edition.
LWBK707-c5_p20-25.qxd 8/19/10 2:24 PM Page 22 Aptara Inc

22 UNIT 1 CELL AND TISSUE FUNCTION

2. Gene-gene interactions are interesting and Activity D Briefly answer the following.
complex. Match the term with the
description. 1. Gregor Mendel was the first to study and char-
acterize inheritance. Explain what he did and
Column A Column B what he discovered.
1. Collaborative a. More than one
genes allele affects the
same trait
2. Multiple
alleles b. One gene masks the
phenotypic effects
3. Complementary 2. Genetic mapping is done to allow us to know
of another
genes the position of certain genes and sequences
nonallelic gene
on the chromosomes. Explain the difference
4. Epistasis c. Each gene is mutu- between genetic maps and physical maps. In
ally dependent on your explanation, describe the basic method-
5. Alleles
the other ology used to construct these maps.
d. Two different genes
influencing the
same trait interact
to produce a pheno-
type neither gene
alone could 3. During meiosis, a process occurs that increases
produce. genetic variability. Explain how this occurs.
e. Alternate forms of a Is it a good or bad thing?
gene at the same
locus

Activity C Sequencing.
1. The processing of genetic material involves 4. Humans have both somatic and sex chromo-
many well-organized steps. Put the following somes. How many of each do we have and
in order, starting at transcription and ending where do they originate?
with the three-dimensional protein.

S S S S S S S S

a. Transcription
b. Translation begins 5. Only about 2% of the genome encodes
instructions for synthesis of proteins; the
c. mRNA moves to cytosol
remainder consists of noncoding regions that
d. mRNA is read by ribosome complex serve to determine where, when, and in what
e. Posttranslational processing quantity proteins are made. Explain how this
f. tRNA moves to ribosome occurs and describe its significance.
g. Ribosomal subunits come together
h. Formation of peptide bonds
i. Final 3D protein structure

Copyright 2011. Wolters Kluwer Health | Lippincott Williams & Wilkins. Study Guide for Porths Essentials of Pathophysiology, Third Edition.
LWBK707-c5_p20-25.qxd 8/19/10 2:24 PM Page 23 Aptara Inc

CHAPTER 5 GENETIC CONTROL OF CELL FUNCTION AND INHERITANCE 23

SECTION III: APPLYING 3. Chromosomes contain all the genetic


content of the genome. There are 23 pairs of
YOUR KNOWLEDGE different chromosomes in each somatic cell,
half from the mother and half from the
Activity E Consider the scenario and answer
father. One of those chromosomes is the sex
the question.
chromosome. What are the other 22 pairs of
Jessica Jones, an adopted child, has been search- chromosomes called?
ing for her parents for many years. She believes a. Ribosomes
that she has finally found her father but wants to
b. Helixes
be 100% sure before she approaches him.
c. Autosomes
1. Is there any way for her to absolutely identify
d. Haploids
her father before she meets him? Discuss the
use of DNA fingerprinting to identify familial 4. On rare occasions accidental errors in dupli-
relationships. cation of DNA occur. What are these called?
a. Codons
b. Ribosomes
c. Endonucleases
d. Mutations
5. Most human traits are determined by multi-
ple pairs of genes, many with alternate
SECTION IV: PRACTICING codes, accounting for some dissimilar forms
that occur with certain genetic disorders.
FOR NCLEX What type of inheritance involves multiple
genes at different loci, with each gene
Activity F Answer the following questions.
exerting a small additive effect in
1. It is the proteins that the genes encode that determining a trait?
make up the majority of cellular structures a. Polygenic inheritance
and perform most life functions. What is the
b. Multifactorial inheritance
term used to define the complete set of
proteins encoded by a genome? c. Monofactorial inheritance
a. Proteome d. Collaborative inheritance
b. Protogene 6. Two syndromes exhibit mental retardation as
c. Nucleotomics a common feature. Both disorders have the
same deletion in chromosome 15. When
d. Chromosome
the deletion is inherited from the mother,
2. Below are the steps in cell replication. Put the infant presents with one syndrome; when
them in the correct order. the same deletion is inherited from the
A. Complementary molecule is duplicated father, Prader-Willi syndrome results. What is
next to each original strand. the syndrome when the deletion is inherited
from the mother?
B. Separation of the two strands of DNA
a. Turner syndrome
C. Mitosis occurs
b. Angelman syndrome
D. Two strands become four strands
c. Down syndrome
a. ACBD
d. Fragile X syndrome
b. BADC
c. BDAC
d. DBCA

Copyright 2011. Wolters Kluwer Health | Lippincott Williams & Wilkins. Study Guide for Porths Essentials of Pathophysiology, Third Edition.
LWBK707-c5_p20-25.qxd 8/19/10 2:24 PM Page 24 Aptara Inc

24 UNIT 1 CELL AND TISSUE FUNCTION

7. Homozygotes are what people are called in 11. The human genome sequence is almost
whom the two alleles of a given pair are the exactly (99.9%) the same in all people. What
same (AA or aa). Heterozygotes are what is thought to account for the differences in
people are called who have different alleles each human's behaviors, physical traits, and
(Aa) at a gene locus. What kind of trait is the susceptibility to disease is the small varia-
expressed only in homozygous pairing? tion (0.01%) in gene sequence. This is termed
a. Dominant trait a .
b. Single-gene trait 12. Like DNA, RNA is a long string of nucleotides
c. Recessive trait encased in a large molecule. However, there
are three aspects of its structure that makes it
d. Penetrant trait
different from DNA. What are these aspects?
8. The International HapMap Project was (Mark all that apply.)
created with two goals. One is the a. RNA's double strand is missing one pair of
development of methods for applying the chromosomes.
technology of these projects to the diagnosis
b. The sugar in each nucleotide of RNA is
and treatment of disease. The other is to map
ribose.
the (what) of the many closely related single
nucleotide polymorphisms in the human c. RNA is a single-stranded molecule.
genome? d. RNA's thymine base is replaced by uracil.
a. Codons 13. One of the first products to be produced
b. Triplet code using recombinant DNA technology was
c. Alleles human ____________.
d. Haplotypes 14. Cytogenetics is the study of the structure and
numeric characteristics of the cells chromo-
9. DNA fingerprinting is based in part on recom-
somes. Chromosome studies can be done on
binant DNA technology and in part on those
any tissue or cell that grows and divides in
techniques originally used in medical genetics
culture. What are the characteristics of a
to detect slight variations in the genomes of
chromosomal study? (Mark all that apply.)
different individuals. These techniques are
used in forensic pathology to compare a. The completed picture of a chromosomal
specimens from the suspect with those of the study is called karyotyping.
forensic specimen. What is being compared b. Human chromosomes are divided into
when DNA fingerprinting is used in forensic three types according to the position of
pathology? the centromere.
a. The banding pattern c. Special laboratory techniques are used to
b. The triplet code culture body cell. They are then fixed and
stained to display identifiable banding
c. The haplotypes
patterns.
d. The chromosomes
d. Complementary genes and collaborative
10. There are two main approaches used in gene genes are easily recognized.
therapy: transferred genes can replace
defective genes or they can selectively inhibit
deleterious genes. What are the compounds
usually used in gene therapy?
a. mRNA sequences
b. Cloned DNA sequences
c. Sterically stable liposomes
d. Single nucleotide polymorphisms

Copyright 2011. Wolters Kluwer Health | Lippincott Williams & Wilkins. Study Guide for Porths Essentials of Pathophysiology, Third Edition.
LWBK707-c5_p20-25.qxd 8/19/10 2:24 PM Page 25 Aptara Inc

CHAPTER 5 GENETIC CONTROL OF CELL FUNCTION AND INHERITANCE 25

15. Genetics has its own set of definitions. Match


the word with its definition.
1. Genotype a. Traits, physical or
2. Phenotype biochemical, as-
sociated with a
3. Pharmacogenetics
specific genotype
4. Somatic cell that is recogniza-
hybridization ble.
5. Penetrance b. How drugs re-
spond to an in-
dividual's
inherited char-
acteristics.
c. The genetic in-
formation con-
tained in the
base sequence
triplet code.
d. The ability of a
gene to express
its function.
e. The fusion of
human somatic
cells with those
of a different
species to yield
a cell contain-
ing the chromo-
somes of both
species.

Copyright 2011. Wolters Kluwer Health | Lippincott Williams & Wilkins. Study Guide for Porths Essentials of Pathophysiology, Third Edition.
LWBK707-c6_p26-31.qxd 8/19/10 12:48 PM Page 26 Aptara Inc

CHAPTER
Genetic and Congenital
Disorders

SECTION I: LEARNING 10. State the cautions that should be observed


when considering use of drugs during
OBJECTIVES pregnancy, including the possible effects
of alcohol abuse, vitamin A derivatives,
1. Define the terms congenital, allele, gene
and folic acid deficiency on fetal
locus, gene mutation, genotype, phenotype,
development.
homozygous, heterozygous, polymorphism, gene
penetrance, and gene expression. 11. List four infectious agents that cause
congenital defects.
2. Describe three types of single-gene disorders
and their patterns of inheritance. 12. Cite the rationale for prenatal diagnosis.
3. Explain the genetic abnormality responsible 13. Describe methods used in arriving at a
for the fragile X syndrome. prenatal diagnosis, including ultrasonog-
raphy, amniocentesis, chorionic villus
4. Contrast disorders due to multifactorial
sampling, percutaneous umbilical fetal blood
inheritance with those caused by single-gene
sampling, and laboratory methods to deter-
inheritance.
mine the biochemical and genetic makeup
5. Describe three patterns of chromosomal of the fetus.
breakage and rearrangement.
6. Trace the events that occur during meiosis
and explain the events that lead to trisomy
or monosomy.
SECTION II: ASSESSING
YOUR UNDERSTANDING
7. Describe the chromosomal and major clinical
characteristics of Down, Turner, and Klinefel- Activity A Fill in the blanks.
ter syndromes.
1. defects, also known as birth
8. State the primary mechanism of altered body defects, abnormalities of structure, function
function in mitochondrial gene disorders or metabolism that are present at the time
and relate it to the frequent involvement of of birth.
neural and muscular tissues.
2. Genetic disorders are caused either by an
9. Cite the most susceptible period of intrauterine alteration in that disrupts the
life for development of defects due to single-gene sequence or
environmental agents. rearrangements.

26
LWBK707-c6_p26-31.qxd 8/19/10 12:48 PM Page 27 Aptara Inc

CHAPTER 6 GENETIC AND CONGENITAL DISORDERS 27

3. Genes either are expressed in an individual in Activity B Consider the following figures.
a dominate, recessive, or in pairs of
. 1.

4. A gene is a biochemical event


such as nucleotide change, deletion, or
insertion that produces a new allele.
5. Genetic disorders arise in two ways: (1) A
from parents or (2)
Lost
due to an acquired mutation.
6. Someone who carries a gene responsible for a
disease but does not manifest the disease is
said to be a .
B
7. syndrome is an autosomal
dominant disorder of connective tissue.
8. X-linked inheritance patterns are
predominantly .
9. Specific chromosomal abnormalities can be C
linked to more than identi-
fiable syndromes.
Pericentric Paracentric
10. Chromosomal disorders may take the form of
alterations in the of one or
more chromosomes or in an
D Lost
number of chromosomes.
11. occurs when there are simulta-
neous breaks in two chromosomes from
different pairs, with exchange of chromosome
parts. In the reciprocal type, there is no loss
of information. E

Fragments

In the figure above, label the abnormality


(deletion, reciprocal translocation, Robertsonian
translocation, inversion).

Copyright 2011. Wolters Kluwer Health | Lippincott Williams & Wilkins. Study Guide for Porths Essentials of Pathophysiology, Third Edition.
LWBK707-c6_p26-31.qxd 8/19/10 12:48 PM Page 28 Aptara Inc

28 UNIT 1 CELL AND TISSUE FUNCTION

2. j. Genes have more


than one normal
allele at the same
locus

Activity D Briefly answer the following


questions.
1. Inheritance of a genetic disease depends on
the location of the mutation within the
karyotype. What are the potential methods
of inheritance? What will determine the likeli-
Is the pedigree shown in the preceding figure for hood of the offspring developing the disease?
an autosomal dominate, autosomal recessive, or Does the sex of the offspring make any
sex-linked disease? difference?

Activity C Match the key concepts in Column


A with their descriptions in Column B.
Column A Column B
1. Single-gene a. Single set of 2. Multifactorial inheritance patterns involve
disorders chromosomes many different genes and their interactions
with the environment. Predicting such
2. Multifactorial b. Disorders are mani-
disorders is more difficult than others are, but
inheritance fested only when
they do display several characteristics. Explain
both members of
3. Autosomal these characteristics.
the gene pair are
dominant affected
disorder
c. Traits carried by
4. Haploid multiple genes and
influenced by the
5. Chromosomal
environment 3. Chromosomal abnormalities are among the
abnormality
d. The outward expres- most common reasons for first-trimester spon-
6. Autosomal sion of a gene taneous abortions as well as over 60 different
recessive diseases. Structural changes are a common
e. Affected parent has
7. Polymorphism a 50% chance of form of chromosomal abnormalities. Explain
transmitting the what a structural change is and list the poten-
8. Phenotype tial causes.
disorder to each
9. Mutation offspring
10. Mitochondrial f. Follow a nonmen-
disorders delian pattern of
inheritance
g. Trisomy 4. Why are alterations in sex chromosomes
h. Disorders are caused better tolerated than alterations of autosomal
by a defective or chromosomes?
mutant allele at a
single-gene locus
i. A biochemical
event such as
nucleotide change,
deletion, or
insertion

Copyright 2011. Wolters Kluwer Health | Lippincott Williams & Wilkins. Study Guide for Porths Essentials of Pathophysiology, Third Edition.
LWBK707-c6_p26-31.qxd 8/19/10 12:48 PM Page 29 Aptara Inc

CHAPTER 6 GENETIC AND CONGENITAL DISORDERS 29

5. Mitochondrial genetic abnormalities are not 2. An adolescent presents at the clinic with
transmitted via mendelian genetics. In complaints of pedunculated lesions
addition, they tend to affect the brain and projecting from his skin on his trunk area.
muscle tissue. Explain why these two The nurse knows that this is a sign of what?
characteristics of mtDNA inheritance are true. a. Marfan syndrome
b. Neurofibromatosis1
c. Down syndrome
d. Klinefelter syndrome
3. The parents of an infant boy ask the nurse
why their son was born with a cleft lip and
palate. The nurse responds that cleft lip and
SECTION III: APPLYING palate are defects that are caused by many
YOUR KNOWLEDGE factors. The defect may also be caused by
teratogens. Which teratogens can cause cleft
Activity E Consider the scenario and answer lip and palate?
the questions.
a. Mumps
A woman aged 37 is 2 months pregnant and has b. Pertussis
a history of alcohol intake of one to two drinks a
c. Rubella
day. She states, My co-worker told me that
drinking alcohol can harm my baby. d. Measles

1. She asks you how having a drink or two every 4. Sometimes an individual that developed from
day can harm her baby. What would you a single zygote is found to have two or more
respond? kinds of genetically different cell populations.
These individuals are called what?
a. Mutant
b. Monosomy
c. Aneuploidy
2. Discuss the effects of fetal alcohol syndrome. d. Mosaic
5. With increasing age, there is a greater chance
of a woman having been exposed to damaging
environmental agents such as drugs,
chemicals, and radiation. These factors may act
on the aging oocyte to cause what in a fetus?
a. Down syndrome
SECTION IV: PRACTICING b. Marfan syndrome
FOR NCLEX c. Patau syndrome
d. Turner syndrome
Activity F Answer the following questions.
6. The embryo is most susceptible to adverse
1. Chromosomes carry 46 genes, 23 from the influences during the period from 15 to 60 days
mother and 23 from the father. These genes are after conception. This period is referred to as
paired, and if both members of the gene pair what?
are identical the person is considered homozy- a. The period of susceptibility
gous. What is the person considered if both
b. The period of organogenesis
members of the gene pair are not identical?
c. The period of fetal anomalies
a. Heterozygous
d. The period of hormonal imbalance
b. Phenotypic
c. Codominant
d. Mutant

Copyright 2011. Wolters Kluwer Health | Lippincott Williams & Wilkins. Study Guide for Porths Essentials of Pathophysiology, Third Edition.
LWBK707-c6_p26-31.qxd 8/19/10 12:48 PM Page 30 Aptara Inc

30 UNIT 1 CELL AND TISSUE FUNCTION

7. Teratogenic substances cause abnormalities 10. Genetic counseling and prenatal screening
during embryonic and fetal development. are tools both for the parents of a child with
These substances have been divided into a defect and for those couples who want a
three classes. These classes are called child but are at high risk for having a child
what? with a genetic problem. What are the
a. Period of organogenesis, third trimester, objectives of prenatal screening?
second month a. To detect fetal abnormalities and to pro-
b. Outside environmental substances, inside vide information on where they can have
environmental substances, internal the pregnancy terminated if they choose
environmental substances. to.
c. Radiation, drugs and chemical substances, b. To detect fetal abnormalities and to pro-
and infectious agents. vide parents with information needed to
make an informed choice about having a
d. Drugs and chemical substances, smoking,
child with an abnormality.
bacteria and virus
c. To provide parents with information
8. Infections with the TORCH agents are needed to make an informed choice about
reported to occur in 1% to 5% of newborn having a child with an abnormality and to
infants in the United States and are among assure the prospective parents that any de-
the major causes of neonatal morbidity and fect in their hoped for child can be identi-
mortality. Which of these are clinical and fied.
pathologic manifestations of TORCH?
d. To allow parents at risk for having a child
a. Microcephaly, hydrocephalus, spina with a specific defect to begin a pregnancy
bifida with the assurance that knowledge about
b. Pneumonitis, myocarditis, macrocephaly the presence or absence of the disorder in
c. Hydrocephalus, macrocephaly, thrombo- the fetus can be confirmed by testing and to
cytopenia provide information on where they can
have the pregnancy terminated if they
d. Microcephaly, hydrocephalus,
choose to.
thrombocytopenia
11. Match the genetic disorder (Column A) with
9. The birth of a child with a defect brings with
its kind of disorder (Column B).
it two issues that must be resolved quickly.
The traumatized parents need emotional sup- Column A Column B
port from the nurse and guidance in how to
Marfan syndrome Single-gene disorder
resolve these two issues. What are these issues?
Huntingtons chorea Autosomal
a. The immediate and future care of the
dominant
affected child, and the possibility of future Tay-Sachs disease
children in the family having a similar Autosomal recessive
defect. Fragile X syndrome disorders
b. The immediate and future care of the Sex-linked disorders
affected child, and the possibility of the
childs death.
c. The possibility of future children having a
similar defect and the possibility of this
childs death.
d. The need for financial resources and the
possibility of this childs death.

Copyright 2011. Wolters Kluwer Health | Lippincott Williams & Wilkins. Study Guide for Porths Essentials of Pathophysiology, Third Edition.
LWBK707-c6_p26-31.qxd 8/19/10 12:48 PM Page 31 Aptara Inc

CHAPTER 6 GENETIC AND CONGENITAL DISORDERS 31

12. Although multifactorial traits cannot be pre- 15. The U.S. Food and Drug Administration
dicted with the same degree of accuracy as passed a law in 1983 classifying drugs accord-
the mendelian single-gene mutations, charac- ing to their proven teratogenicity. Listed
teristic patterns exist. What are these charac- below are the classes of drugs in random
teristic patterns? (Mark all that apply.) order. Put them in order according to their
a. Multifactorial congenital malformations teratogenicity.
tend to involve a single organ or tissue de- A. Class X
rived from the same embryonic develop- B. Class A
mental field.
C. Class C
b. The risk of recurrence in future pregnan-
D. Class B
cies is for the same or a similar defect.
E. Class D
c. The risk increases with increasing inci-
dence of the defect among relatives. a. BDCEA
d. Multifactorial congenital malformations b. ABCDE
are always present at birth. c. BCDAE
13. is a rare metabolic disorder that d. AEBCD
affects approximately 1 in every 15,000
infants in the United States. The disorder is
caused by a deficiency of the liver enzyme
phenylalanine hydroxylase. Without a special
diet these children will die.
14. After conception, development is influenced
by the environmental factors that the
embryo shares with the mother. Some of
these factors can act on the developing fetus
and cause defects. These factors might be
what? (Mark all that apply.)
a. Drugs
b. Weather
c. Air pollution
d. Radiation

Copyright 2011. Wolters Kluwer Health | Lippincott Williams & Wilkins. Study Guide for Porths Essentials of Pathophysiology, Third Edition.
LWBK707-c7_p32-38.qxd 8/19/10 2:21 PM Page 32 Aptara Inc

CHAPTER
Neoplasia

SECTION I: LEARNING 13. Explain how host factors such as heredity,


levels of endogenous hormones, and
OBJECTIVES immune system function increase the risk for
development of selected cancers.
1. Define neoplasm and explain how neoplastic
growth differs from the normal adaptive 14. Relate the effects of environmental factors
changes seen in atrophy, hypertrophy, and such as chemical carcinogens, radiation, and
hyperplasia. oncogenic viruses to the risk of cancer devel-
opment.
2. Distinguish between cell proliferation and
differentiation. 15. Identify concepts and hypotheses that may
explain the processes by which normal cells
3. Describe the phases of the cell cycle.
are transformed into cancer cells by carcino-
4. Describe the properties of stem cells. gens.
5. Cite the method used for naming benign and 16. Characterize the mechanisms involved in the
malignant neoplasms. anorexia and cachexia, fatigue and sleep dis-
orders, anemia, and venous thrombosis
6. State the ways in which benign and
experienced by patients with cancer.
malignant neoplasms differ.
17. Define the term paraneoplastic syndrome and
7. Relate the properties of cell differentiation to
explain its pathogenesis and manifestations.
the development of a cancer cell clone and
the behavior of the tumor. 18. Cite three characteristics of an ideal
screening test for cancer.
8. Trace the pathway for hematologic spread of
a metastatic cancer cell. 19. Describe the four methods that are used in
the diagnosis of cancer.
9. Use the concepts of growth fraction and dou-
bling time to explain the growth of 20. Differentiate between the methods used for
cancerous tissue. grading and staging of cancers.
10. Describe various types of cancer-associated 21. Explain the mechanism by which radiation
genes and cancer-associated cellular and exerts its beneficial effects in the treatment
molecular pathways. of cancer.
11. Describe genetic events and epigenetic 22. Describe the adverse effects of radiation
factors that are important in tumorigenesis. therapy.
12. State the importance of cancer stem cells, 23. Differentiate between the action of direct
angiogenesis, and the cell microenvironment DNA-interacting and indirect DNA-interact-
in cancer growth and metastasis. ing chemotherapeutic agents and cell cycle-
specific and cell cycle-independent drugs.

32
LWBK707-c7_p32-38.qxd 8/19/10 2:21 PM Page 33 Aptara Inc

CHAPTER 7 NEOPLASIA 33

24. Describe the three mechanisms whereby bio- 11. Malignant neoplasms are less well
therapy exerts its effects. and have the ability to break
loose, enter the circulatory or lymphatic
25. Describe three examples of targeted therapy
systems, and form secondary malignant
used in the treatment of cancer.
tumors at other sites.
26. Cite the most common types of cancer affect-
12. Tumors usually are named by adding the
ing children.
suffix - to the parenchymal
27. Describe how cancers that affect children tissue type from which the growth
differ from those that affect adults. originated.
28. Discuss possible long-term effects of 13. A is growth that projects from a
radiation therapy and chemotherapy on mucosal surface.
adult survivors of childhood cancer.
14. The term is used to designate a
malignant tumor of epithelial tissue origin.
15. There are two categories of malignant
SECTION II: ASSESSING neoplasms, and
YOUR UNDERSTANDING cancers.
16. The term is used to describe the
Activity A Fill in the blanks. loss of cell differentiation in cancerous tissue.
1. Cancer is a disorder of altered cell 17. A characteristic of cancer cells is the ability
and . to proliferate even in the absence of
2. The process of cell division results in cellular .
. 18. With homologous loss of gene
3. is the process of specialization activity, DNA damage goes unrepaired and
whereby new cells acquire the structure and mutations occur in dividing cells, leading to
function of the cells they replace. malignant transformations.

4. Proteins called control entry 19. The types of genes involved in cancer are
and progression of cells through the cell numerous, with two main categories being
cycle. the , which control cell growth
and replication, and tumor
5. Kinases are enzymes that genes, which are growth-inhibiting
proteins. regulatory genes.
6. Continually renewing cell populations rely 20. is the only known retrovirus to
on cells of the same lineage cause cancer in humans.
that have not yet differentiated to the extent
that they have lost their ability to divide. 21. Tumor cells must double times
before there will be a palpable mass.
7. cells remain incompletely
undifferentiated throughout life. 22. A common manifestation of solid tumors is
the cancer syndrome.
8. stem cells are pluripotent cells
derived from the inner cell mass of the 23. As cancers grow, they compress and erode
blastocyst stage of the embryo. blood vessels, causing and
along with frank bleeding and
9. The term refers to an abnormal sometimes hemorrhage.
mass of tissue in which the growth exceeds
and is uncoordinated with that of the normal 24. is a common side effect of
tissues. many cancers. It is related to blood loss,
hemolysis, impaired red cell production, or
10. do not usually cause death treatment effects.
unless the location interferes with a vital
organs function. 25. A tissue involves the removal of
a tissue specimen for microscopic study.

Copyright 2011. Wolters Kluwer Health | Lippincott Williams & Wilkins. Study Guide for Porths Essentials of Pathophysiology, Third Edition.
LWBK707-c7_p32-38.qxd 8/19/10 2:21 PM Page 34 Aptara Inc

34 UNIT 1 CELL AND TISSUE FUNCTION

26. therapy uses high-energy parti- 5. Tumor- c. Mass of cells due to


cles or waves to destroy or damage cancer initiating cells overgrowth
cells. d. Process that removes
6. Tumor
27. is a systemic treatment that senescent and or
7. Apoptosis damaged cells
enables drugs to reach the site of the tumor
as well as other distant sites. 8. Benign mass e. Stem cells undergo-
ing numerous
9. Differentiation
Activity B Consider the following figure. mitotic divisions
10. Oncology while maintaining
an undifferentiated
11. Protooncogene
Carcinogenic Normal state
agent cell 12. Growth f. Cancer stem cells
fraction
g. Process of cell spe-
13. Tumor cialization
suppressor h. Well-differentiated
gene mass of cells
14. Genetic i. Study of tumors
instability and their treatment
15. Epigenetic j. Process of cell divi-
effects sion
k. Loss of cell
16. Anaplasia
differentiation
17. Anchorage l. Changes in gene
dependence expression without
18. Doubling time DNA mutation
m. Normal gene that
19. p53
can cause cancer if
Malignant mutated
neoplasm n. Promote cancer
when less active
o. Ratio of dividing
cells to resting cells
1. In the flow chart above, fill in the missing p. Tumor suppressor
steps using the following terms: DNA damage, gene
alterations in genes that control apoptosis, unregu-
q. Marked by chrom-
lated cell differentiation and growth, inactivation
osomal aberrations
of tumor suppressor genes, activation of growth-
promoting oncogenes, DNA repair, and failure r. Epithelial cells
of DNA repair. must be anchored
to either neighbor-
ing cells or the
Activity C Match the key terms in Column A underlying
with their definitions in Column B. extracellular matrix
Column A Column B to live and grow
s. Time it takes for
1. Malignant a. Defines the differen-
the total mass of
mass tiation potential of
cells in a tumor to
stem cells
2. Cellular double
potency b. Undefined or less
differentiated
3. Renewal cellular mass
4. Proliferation

Copyright 2011. Wolters Kluwer Health | Lippincott Williams & Wilkins. Study Guide for Porths Essentials of Pathophysiology, Third Edition.
LWBK707-c7_p32-38.qxd 8/19/10 2:21 PM Page 35 Aptara Inc

CHAPTER 7 NEOPLASIA 35

Activity D Put the following terms for cellular 6. Chemical carcinogens act in two distinct
potency in order from the least differentiated to ways. What are they?
the most differentiated.
a. Pluripotent
b. Totipotent
c. Unipotent
7. Cachexia is marked by a hypermetabolic
d. Multipotnet state. Give two reasons for this and explain
the consequences.

S S S

Activity E Briefly answer the following.


8. What is paraneoplastic syndrome?
1. Not all cells in the body can re-enter the cell
cycle, but some will do so continuously. In
terms of regeneration and differentiation,
which types of cells will or will not re-enter
the cell cycle?
9. List some of the common methods used for
diagnosing cancer.

2. Compare and contrast benign tumors and


malignant tumors.
10. Cancers are graded and staged on their char-
acteristics in order to determine a treatment
regimen. Explain the grading and staging
system and how it is met.

3. List the five factors used to describe benign


and malignant neoplasm.

SECTION III: APPLYING


4. Describe the process and routes of metastasis.
YOUR KNOWLEDGE
Activity F Consider the scenario and answer
the questions.
Eight year old Joe Cheapson has been diagnosed
with acute lymphocytic leukemia (ALL). His treat-
5. Explain how a diminished immune system ment plan includes placement of an implanted
may play a role in carcinogenesis. central venous catheter and multiple administra-
tions of chemotherapy. Joe says, NO! I dont
want to be stuck with needles all the time.
1. What would you tell Joe to decrease his anxiety?

Copyright 2011. Wolters Kluwer Health | Lippincott Williams & Wilkins. Study Guide for Porths Essentials of Pathophysiology, Third Edition.
LWBK707-c7_p32-38.qxd 8/19/10 2:21 PM Page 36 Aptara Inc

36 UNIT 1 CELL AND TISSUE FUNCTION

2. How would you explain the way chemotherapy 3. It is well known that cancer is not a single
works to Joes parents? disease. It follows then that cancer does not
have a single cause. It seems more likely
that the occurrence of cancer is triggered by
the interactions of multiple risk factors. What
are identified risk factors for cancer?
a. Body type, age, and hereditary
b. Radiation, cancer-causing viruses, and
color of skin
SECTION IV: PRACTICING
c. Hormonal factors, chemicals, and im-
FOR NCLEX munologic mechanisms
Activity G Answer the following questions. d. Immunologic mechanisms, cancer-causing
viruses, and color of skin
1. The nurse has provided an educational session
with a 56-year-old man, newly diagnosed with 4. Several cancers have been identified as inher-
a benign tumor of the colon. The nurse knows itable through an autosomal dominant gene.
that the patient needs further teaching when People who inherit these genes are generally
he makes which remark? only at increased risk for developing the
cancer. There is one type of cancer, however,
a. This tumor I have, will I die from it?
that is almost certain to develop in someone
b. Even though benign tumors cant stop who inherits the dominant gene. Which can-
growing, they arent considered cancer. cer carries the highest risk of developing in
c. Benign tumors still produce normal cells someone who carries the gene?
different from other cells around them. a. Retinoblastoma
d. This kind of tumor cant invade other or- b. Osteosarcoma
gans or travel to other places in the body
c. Acute lymphocytic leukemia
to start new tumors.
d. Colon cancer
2. The nurse on an oncology floor has just admit-
ted a patient with metastatic cancer. The patient 5. One group of chemical carcinogens is called
asks how cancer moves from one place to indirect-reacting agents. Another term for
another in the body. What would the nurse these agents is procarcinogens, which become
answer? active only after metabolic conversion. One
of the most potent procarcinogens is a group
a. The cancer cells are not able to float
of dietary carcinogens called:
around the original tumor in body fluids.
a. Polycyclic aromatic hydrocarbons
b. Cancer cells enter the bodys lymph
system and thereby spread to other parts b. Aflatoxins
of the body. c. Initiators
c. Cancer cells are moved from one place d. Diethylstilbestrol
in the body to another by transporter cells.
d. Cancer cells replicate and form a chain
that spreads from the original tumor site
to the site of the metastatic lesion.

Copyright 2011. Wolters Kluwer Health | Lippincott Williams & Wilkins. Study Guide for Porths Essentials of Pathophysiology, Third Edition.
LWBK707-c7_p32-38.qxd 8/19/10 2:21 PM Page 37 Aptara Inc

CHAPTER 7 NEOPLASIA 37

6. In some cancers, the presenting factor is an 10. The inherent properties of a tumor that deter-
effusion, or fluid, in the pleural, pericardial, mine how the tumor responds to radiation is
or peritoneal spaces. Research has found that called radiosensitivity. When radiation is
almost 50% of undiagnosed effusions in peo- combined with cytotoxic drugs it has been
ple not known to have cancer turn out to be noted that there is a radiosensitizing effect on
malignant. Which cancers are often found tumor cells. Which drug is considered a
because of effusions? radiosensitizer?
a. Colon and rectal cancers a. Doxorubicin
b. Lung and ovarian cancers b. Cisplatin
c. Breast and colon cancers c. Vincristine
d. Ovarian and rectal cancers d. Docetaxel
7. Tumor markers are used for screening, estab- 11. Cancer is a disorder of altered cell differentia-
lishing prognosis, monitoring treatment, and tion and growth. The term
detecting recurrent disease. Which serum refers to an abnormal mass of tissue in which
tumor markers have been proven to be the growth exceeds and is uncoordinated
among the most useful in clinical practice? with that of the normal tissues.
a. Prostate-specific antigen and deoxyribonu- 12. A woman diagnosed with breast cancer asks
cleic acid the nurse how a malignant tumor in her
b. Deoxyribonucleic acid and carcinoembry- breast could spread to other parts of her
onic antigen body. The nurse answers that a malignant
c. Alpha-fetoprotein and human chorionic neoplasm is made of up less well-
gonadotropin differentiated cells that have which of the
following abilities? Select all that apply.
d. Chorionic gonadotropin and cyclin-de-
pendent kinases a. Break loose
b. Reinvade their original site
8. Cranial radiation therapy (CRT) has been
used to treat brain tumors, ALL, head and c. Enter the circulatory or lymphatic systems
neck soft tissue tumors, and retinoblastoma d. Be excreted through the alimentary canal
in children. Childhood cancer survivors who e. Form secondary malignant tumors at
had CRT as therapy for their cancers are other sites
prone to growth hormone deficiency. In
adults, what is growth hormone deficiency 13. Cancer cells differ from normal cells in many
associated with? ways. They have lost the ability to accurately
communicate with other cells, and they do
a. Hypocalcemia
not have to be anchored to other cells to sur-
b. Cardiovascular longevity vive. How else are they different from other
c. Hyperinsulinemia cells? Select all that apply.
d. Dyslipidemia a. Cancer cells have an increased tendency
to stick together.
9. A big difference in the treatment of
childhood cancer as opposed to adult cancer b. Cancer cells have an unlimited life span.
is that chemotherapy is the most widely used c. Cancer cells have lost contact inhibition.
treatment therapy for childhood cancer. d. Cancer cells need increased amounts of
What is the reason for this? growth factor to proliferate.
a. Pediatric tumors are more responsive to e. Cancer cells are termed genetically unstable.
chemotherapy than adult cancers.
b. Children do not tolerate other forms of
therapy as well as adults do.
c. Children do not complain about the
nausea and vomiting caused by
chemotherapy like adults do.
d. Children think losing their hair is cool.

Copyright 2011. Wolters Kluwer Health | Lippincott Williams & Wilkins. Study Guide for Porths Essentials of Pathophysiology, Third Edition.
LWBK707-c7_p32-38.qxd 8/19/10 2:21 PM Page 38 Aptara Inc

38 UNIT 1 CELL AND TISSUE FUNCTION

14. Match the following types of cancer with 15. Childhood cancers are often diagnosed late in
their screening tests. the disease process because the signs and
symptoms mimic other childhood diseases.
Type of Cancer Screening Test
However, with the huge strides in treatment
1. Malignant a. Mammography methods more and more children survive
melanoma b. Self-examination childhood cancer. These survivors face the
uncertainty of what the life-saving treatment
2. Prostatic c. Pap smear
they received during their childhood may
3. Cervical d. PSA produce what late effects? Select all that apply.
4. Breast a. Cardiomyopathy and pulmonary fibrosis
b. Cognitive dysfunction and hormonal
dysfunction
c. Second malignancies and liver failure
d. Impaired growth and second malignancies

Copyright 2011. Wolters Kluwer Health | Lippincott Williams & Wilkins. Study Guide for Porths Essentials of Pathophysiology, Third Edition.
LWBK707-c8_p39-48.qxd 8/19/10 1:46 PM Page 39 Aptara Inc

CHAPTER
Disorders of Fluid,
Electrolyte, and
Acid-Base Balance

SECTION I: LEARNING 8. Describe measures that can be used in assessing


body fluid levels and sodium concentration.
OBJECTIVES
9. Describe the causes, manifestations, and
1. Define the terms electrolyte, ion, and nonelec- treatment of psychogenic polydipsia.
trolytes.
10. Describe the relationship between antidiuretic
2. Differentiate the intracellular from the hormone and aquaporin channels in reabsorp-
extracellular fluid compartments in terms of tion of water by the kidney.
distribution and composition of water, elec-
11. Compare the pathology, manifestations, and
trolytes, and other osmotically active
treatment of diabetes insipidus and the
solutes.
syndrome of inappropriate antidiuretic
3. Relate the concept of a concentration gradi- hormone.
ent to the processes of diffusion and osmosis.
12. Compare and contrast the causes, manifesta-
4. Describe the control of cell volume and the tions, and treatment of isotonic fluid volume
effect of isotonic, hypotonic, and hypertonic deficit, isotonic fluid volume excess,
solutions on cell size. hypotonic hyponatremia, and hypertonic
hyponatremia.
5. Describe factors that control fluid exchange
between the vascular and interstitial fluid 13. Characterize the distribution of potassium in
compartments and relate them to the devel- the body and explain how extracellular
opment of edema and third spacing of extra- potassium levels are regulated in relation to
cellular fluids. body gains and losses.
6. Describe the manifestations and treatment of 14. State the causes of hypokalemia and
edema. hyperkalemia in terms of altered intake, out-
put, and transcellular shifts.
7. State the functions and physiologic mecha-
nisms controlling body water levels and 15. Relate the functions of potassium to the man-
sodium concentration, including the effective ifestations of hypokalemia and hyperkalemia.
circulating volume, sympathetic nervous sys-
16. Describe methods used in diagnosis and
tem, renin-angiotensin-aldosterone system,
treatment of hypokalemia and hyperkalemia.
and antidiuretic hormone.

39
LWBK707-c8_p39-48.qxd 8/19/10 1:46 PM Page 40 Aptara Inc

40 UNIT 2 INTEGRATIVE BODY FUNCTIONS

17. Describe the associations among intestinal 33. Contrast and compare the clinical manifesta-
absorption, renal elimination, bone stores, tions and treatment of metabolic and respira-
and the functions of vitamin D and tory acidosis and of metabolic and respiratory
parathyroid hormone in regulating calcium, alkalosis.
phosphate, and magnesium levels.
18. State the difference between ionized and
bound forms of calcium in terms of
physiologic function.
SECTION II: ASSESSING
YOUR UNDERSTANDING
19. Describe the mechanisms of calcium gain
and loss and relate them to the causes of Activity A Fill in the blanks.
hypocalcemia and hypercalcemia.
1. The consists of fluid contained
20. Relate the functions of calcium to the within all of the billions of cells in the body.
manifestations of hypocalcemia and
hypercalcemia. 2. The contains all the fluids out-
side the cells, including those in the intersti-
21. Describe the mechanisms of phosphate tial or tissue spaces and blood vessels.
gain and loss and relate them to causes of
hypophosphatemia and hyperphosphatemia. 3. are substances that dissociate in
solution to form ions.
22. State the definition of an acid and a base.
4. Particles that do not dissociate into ions such
23. Describe the three forms of carbon dioxide as glucose and urea are called .
transport and their contribution to acid-base
balance. 5. is the movement of charged or
uncharged particles along a concentration
24. Define pH and use the Henderson- gradient.
Hasselbalch equation to calculate the pH
and to compare compensatory mechanisms 6. is the movement of water
for regulating pH. across a semipermeable membrane.

25. Describe the intracellular and extracellular 7. refers to the osmolar concent-
mechanisms for buffering changes in body ration in 1 L of solution and to
pH. the osmolar concentration in 1 kg of water.

26. Compare the role of the kidneys and 8. The predominant osmotically active particles
respiratory system in regulation of acid-base in the extracellular fluid are
balance. and its associated anions (Cl and HCO3).

27. Explain how the transcellular hydrogen- 9. The difference between the calculated and
potassium exchange system contributes to measured osmolality is called the .
the regulation of pH. 10. proteins and other organic
28. Differentiate the terms acidemia, alkalemia, compounds cannot pass through the
acidosis, and alkalosis. membrane.

29. Describe a clinical situation involving an 11. The membrane pump continu-
acid-base disorder in which both primary and ously removes three Na+ ions from the cell
compensatory mechanisms are present. for every two K+ ions that are moved back
into the cell.
30. Define metabolic acidosis, metabolic
alkalosis, respiratory acidosis, and respiratory 12. refers to the movement of water
alkalosis. through capillary pores because of a mechan-
ical, rather than an osmotic, force.
31. Explain the use of the plasma anion gap in
differentiating types of metabolic acidosis. 13. The represents an accessory
route whereby fluid from the interstitial
32. List common causes of metabolic and respi- spaces can return to the circulation.
ratory acidosis and metabolic and respiratory
alkalosis.

Copyright 2011. Wolters Kluwer Health | Lippincott Williams & Wilkins. Study Guide for Porths Essentials of Pathophysiology, Third Edition.
LWBK707-c8_p39-48.qxd 8/19/10 1:46 PM Page 41 Aptara Inc

CHAPTER 8 DISORDERS OF FLUID, ELECTROLYTE, AND ACID-BASE BALANCE 41

14. is a palpable swelling prod- 28. Fluid volume excess represents an


uced by expansion of the interstitial fluid expansion of the ECF
volume. compartment with increases in both interstitial
and vascular volumes.
15. Edema due to decreased capillary colloidal
osmotic pressure usually is the result of inad- 29. represents a plasma sodium
equate production or abnormal loss of concentration below 135 mEq/L.
.
30. hyponatremia represents reten-
16. edema occurs at times when tion of water with dilution of sodium while
the accumulation of interstitial fluid exceeds maintaining the ECF volume within a normal
the absorptive capacity of the tissue gel. range.
17. represent an accumulation or 31. MDMA (Ecstasy) and its metabolites have
trapping of body fluids that contribute to body been shown to produce enhanced release of
weight but not to fluid reserve or function. from the hypothalamus.
18. Water losses that occur through the skin and 32. implies a plasma sodium level
lungs are referred to as because above 145 mEq/L.
they occur without a persons awareness.
33. Hypernatremia represents a deficit of
19. Most sodium losses occur through the in relation to the bodys
. sodium stores.
20. The major regulator of sodium and water 34. The effects of aldosterone on potassium
balance is the maintenance of the elimination are mediated through a
located in the late distal and
21. RAAS exerts its action through
cortical collecting tubules of the kidney.
and .
35. The is determined by the ratio
22. is primarily a regulator of water
of ICF to ECF potassium concentration.
intake and is a regulator of
water output. 36. With severe , the resting
membrane approaches the threshold
23. involves compulsive water
potential causing sustained subthreshold
drinking and is usually seen in persons with
depolarization with a resultant inactivation
psychiatric disorders, most commonly schizo-
of the sodium channels and a net decrease in
phrenia.
excitability.
24. (DI) is caused by a deficiency of
37. The renal processes that conserve potassium
or a decreased response to antidiuretic
during interfere with the
hormone (ADH).
kidneys ability to concentrate urine.
25. Disorders of sodium concentration produce
38. Chronic hyperkalemia is usually associated
a change in the osmolality of the extracellu-
with .
lar fluid (ECF) with movement water from
the ECF compartment into the intracellular 39. The signs and symptoms of potassium
fluid (ICF) compartment, known as are closely related to a decrease
, or from ICF compartment in neuromuscular excitability.
into the ECF fluid compartment known as
40. acts to sustain normal plasma
.
levels of calcium and phosphate by increasing
26. When the effective circulating blood volume their absorption from the intestine. It also is
is compromised, the condition is often referred necessary for normal bone formation.
to as .
41. serves as a cofactor in the gen-
27. cause sequestering of ECF in eration of cellular energy and is important in
the serous cavities, extracellular spaces in the function of second messenger systems.
injured tissues, or lumen of the gut.

Copyright 2011. Wolters Kluwer Health | Lippincott Williams & Wilkins. Study Guide for Porths Essentials of Pathophysiology, Third Edition.
LWBK707-c8_p39-48.qxd 8/19/10 1:46 PM Page 42 Aptara Inc

42 UNIT 2 INTEGRATIVE BODY FUNCTIONS

42. The manifestations of acute 55. Metabolic alkalosis also leads to a compensa-
reflect the increased neuromuscular excitability. tory with development of
various degrees of and respira-
43. The manifestations of result
tory acidosis.
from a decrease in cellular energy stores due
to deficiency in ATP. 56. Respiratory occurs in acute or
chronic conditions that impair effective
44. Many of the signs and symptoms of a
alveolar ventilation and cause an
phosphate excess are related to a
accumulation of PCO2.
deficit.
57. Respiratory is caused by hyper-
45. acts as a cofactor in many intra-
ventilation or a respiratory rate in excess of
cellular enzyme reactions, including the
that needed to maintain normal plasma
transfer of high-energy phosphate groups in
the generation of ATP from adenosine
Activity B Consider the following figure.
diphosphate.
46. Normally, the concentration of body acids
and bases is regulated so that the pH of extra-
cellular body fluids is maintained within a
very narrow range of to Blood volume
. Serum osmolality

47. The H concentration is commonly expressed


in terms of the
48. Acids are continuously generated as by-
products of processes.
49. Physiologically, these acids fall into two
groups: the acid H2CO3 and all
other acids.
50. The content of the blood can
Feedback
be calculated by multiplying the partial
pressure of CO2 (PCO2) by its solubility
coefficient. Complete the above flow chart using the following
terms:
51. The metabolism of and other
substances results in the generation of fixed Extracellular water volume
or nonvolatile acids and bases. Thirst
52. The plasma pH can be calculated using an Secretion of ADH
equation called the . Reabsorption of water by the kidney
53. The buffer system is the princi- Water ingestion
ple ECF buffer.
54. is a systemic disorder caused by
an increase in plasma pH due to a primary
excess in HCO3.

Copyright 2011. Wolters Kluwer Health | Lippincott Williams & Wilkins. Study Guide for Porths Essentials of Pathophysiology, Third Edition.
LWBK707-c8_p39-48.qxd 8/19/10 1:46 PM Page 43 Aptara Inc

CHAPTER 8 DISORDERS OF FLUID, ELECTROLYTE, AND ACID-BASE BALANCE 43

7.4

6.9 7.9

24 1.2
pH = 6.1 + log10 (ratio HCO3-: H2CO3)
HCO3- H2CO3
(mEq/L) (mEq/L)

A Ratio: HCO3-: H2CO3 = 20:1

7.4 7.4
7.7
6.9 7.9 6.9 7.9

12 0.6

HCO3- 1.2 24 H2CO3


(mEq/L) (mEq/L)
H2CO3 HCO3-
(mEq/L) (mEq/L)

B Ratio: HCO3-: H2CO3 = 10:1 D Ratio: HCO3-: H2CO3 = 40:1

7.4 7.4

6.9 7.9 6.9 7.9

12 0.6 12 0.6

HCO3- H2CO3 HCO3- H2CO3


(mEq/L) (mEq/L) (mEq/L) (mEq/L)

C Ratio: HCO3-: H2CO3 = 20:1 E Ratio: HCO3-: H2CO3 = 20:1

In the diagram above, label each scale to reflect Metabolic acidosis with respiratory compensa-
the acid-base state and if there is any compensa- tion
tion present. Respiratory alkalosis
Normal, pH 7.4 Respiratory alkalosis with renal compensation
Metabolic acidosis

Copyright 2011. Wolters Kluwer Health | Lippincott Williams & Wilkins. Study Guide for Porths Essentials of Pathophysiology, Third Edition.
LWBK707-c8_p39-48.qxd 8/19/10 1:46 PM Page 44 Aptara Inc

44 UNIT 2 INTEGRATIVE BODY FUNCTIONS

Activity C Match the key terms in Column A 2.


with their definitions in Column B. Column A Column B
1. 1. SIADH a. State of fluid
Column A Column B volume excess
2. Aldosterone
affecting cardiac
1. Cations a. Effective 3. Hypernatr- function
osmolality same as emia
2. Osmotic b. Hypertonic concen-
the ICF
pressure 4. Circulatory tration
b. Effect that the effec-
3. Capillary overload c. Hypotonic dilution
tive osmotic pressure
colloidal of a solution on cell 5. Hyponatremia d. Failure of the nega-
osmotic size because of water tive feedback system
pressure 6. Baroreceptors that regulates the
movement across
4. Tonicity the cell membrane 7. Nephrogenic release and
c. Positively charged diabetes inhibition of ADH
5. Generalized insipidus e. Renal insensitivity
ions
edema to ADH
d. Negatively charged 8. Osmoreceptors
6. Glomerulone- ions f. Decrease in the abil-
phritis 9. ANP ity to sense thirst
e. Osmotic pressure
7. Isotonic generated by the 10. Hypodipsia g. Respond to changes
solution plasma proteins in ECF osmolality
that do not pass by swelling or
8. Obligatory shrinking
through the pores
urine output
of the capillary wall h. Acts at the cortical
9. Anions f. Increased collecting tubules to
10. Lymphedema permeability of increase sodium
glomerulus to reabsorption
proteins i. Increases sodium
g. Pressure by which excretion by the
water is drawn into kidney
a solution through j. Respond to pressure-
a semipermeable induced stretch of
membrane the vessel walls
h. Urine output that is 3.
required to
eliminate wastes Column A Column B
i. Edema due to 1. Amphoteric a. Molecule that can
impaired lymph release an H
drainage 2. Acid
b. Acute increases in
j. The result of 3. Whole HCO3
increased vascular blood buffer
c. Genetic mitochon-
volume base
drial disorder
4. Delta gap d. Ion or molecule that
5. MELAS can accept an H+
e. Can function as
6. Excess
acid or base
base loading
f. Increase in plasma
7. Base PCO2
8. Carbonic
anhydrase

Copyright 2011. Wolters Kluwer Health | Lippincott Williams & Wilkins. Study Guide for Porths Essentials of Pathophysiology, Third Edition.
LWBK707-c8_p39-48.qxd 8/19/10 1:46 PM Page 45 Aptara Inc

CHAPTER 8 DISORDERS OF FLUID, ELECTROLYTE, AND ACID-BASE BALANCE 45

9. Hypercapnia g. The degree to which 5. What are the three types of polydipsia?
an acid or base in a
10. Dissociation
buffer system disso-
constant
ciates
h. Anion gap of urine
i. Measures the level of
6. What are the physical manifestations of an
all the buffer
isotonic volume expansion?
systems of the blood
j. Catalyzes bicarbon-
ate reaction

Activity D Draw a flow chart that puts the


following steps of calcium concentration by 7. What are the changes seen in an electro-
parathyroid hormone in order. Include the cardiogram during hypokalemia and why are
involved organs: parathyroid glands, bone, they present?
kidney, and intestine.
Increased serum calcium
Increased intestinal calcium absorption
Activated vitamin D
Decreased calcium elimination 8. What are the systemic effects of hypercalcemia?
Release of calcium from bone
Decreased serum calcium
Release of parathyroid hormone

Activity E Briefly answer the following. 9. Why does someone with kidney disease need
1. Compare and contrast the ICF from the ECF. to worry about the integrity of the skeletal
system?

2. What are the forces that control the


movement of water between the capillary 10. How are pH and K+ related? How do they
and interstitial spaces? serve as a buffer?

3. What are the physiological mechanisms that 11. How do the kidneys regulate acid-base
produce edema? balance?

4. How are sodium and water levels maintained


12. What are the two types of acid-base disorders?
in the body?

Copyright 2011. Wolters Kluwer Health | Lippincott Williams & Wilkins. Study Guide for Porths Essentials of Pathophysiology, Third Edition.
LWBK707-c8_p39-48.qxd 8/19/10 1:46 PM Page 46 Aptara Inc

46 UNIT 2 INTEGRATIVE BODY FUNCTIONS

SECTION III: APPLYING 1. What are this patients laboratory values


indicative of?
YOUR KNOWLEDGE
Activity F Consider the scenario and answer
the questions.
Case study: The parents of a 10-year-old child
arrive at the burn unit to see their child for the 2. The physician orders a blood glucose level to
first time since her admission. The client was be drawn. Why would a blood glucose level be
admitted 8 hours ago with second- and third- important for this patient?
degree burns over 60% of her body. She is
edematous and in pain.
a. The parents state, When we left here, just a
few hours ago, she wasnt all swollen like that.
What causes all that swelling? What answer
would you give?
SECTION IV: PRACTICING
FOR NCLEX
Activity H Answer the following questions.

b. The doctor explains to the parents that because 1. Edema is an excess in the interstitial fluid
their daughter has a large burned area she has volume. What mechanisms play a part in the
lost a large amount of fluid. The concern for the formation of edema? (Mark all that apply.)
client is now not only the burn, but a disorder a. Mechanisms that increase capillary perme-
called fluid volume deficit. After the doctor ability
leaves, the parents ask the nurse if the doctor is b. Mechanisms that increase capillary
sure their daughter has fluid volume deficit. filtration pressure
What should the nurse know about fluid
c. Mechanisms that increase capillary
volume deficit?
colloidal osmotic pressure
d. Mechanisms that produce obstruction to
the flow of lymph
e. Mechanisms that decrease capillary
colloidal osmotic pressure
Activity G Consider the scenario and answer 2. Match the following elements with their
the questions. actions in the body.
A college student is brought to the emergency Element Action in the Body
department by her friend. It is reported by the
1. Sodium a. Increases the
young womans friend that they found her
absorption of calcium
wandering around outside the dorm and she did 2. Potassium
from the intestine
not know where she was or why she was there.
3. Calcitriol b. Required for cellular
The friend stated that the young woman had com-
plained of being very tired lately and she had 4. Phosphate energy metabolism
lost weight because she was not eating or drinking. c. Needed for metabolism
5. Magnesium
Vital signs are: blood pressure, 118/78; respiration, of glucose, fat, and
30; pulse, 66. An ABG is ordered and results are: protein
PO2 of 95; PCO2 35; HCO3 of 20, and a pH of 7.1. d. Regulates the ECF
volume
e. Maintenance of the
osmotic integrity of
cells

Copyright 2011. Wolters Kluwer Health | Lippincott Williams & Wilkins. Study Guide for Porths Essentials of Pathophysiology, Third Edition.
LWBK707-c8_p39-48.qxd 8/19/10 1:46 PM Page 47 Aptara Inc

CHAPTER 8 DISORDERS OF FLUID, ELECTROLYTE, AND ACID-BASE BALANCE 47

3. The effective circulating volume is the major 7. Potassium is the major cation in the body. It
regulator of water balance in the body. What plays many important roles, including the
else does it regulate? excitability of nerves and muscles. Where is
a. Sodium this action particularly important?
b. Magnesium a. The heart
c. Calcium b. The brain
d. Potassium c. The lungs
d. The liver
4. Psychogenic polydipsia is most commonly
seen in people with schizophrenia. It is a dis- 8. Vitamin D, officially classified as a vitamin,
ease that involves compulsive water drinking functions as a hormone in the body. What
without thirst and excessive urine output. It other hormone is necessary in the body for
may be worsened by things that cause by vitamin D to work?
excessive ADH secretion. What may be a. Thyroid hormone
reasons that there is excessive ADH secretion
b. Parathyroid hormone
in the body?
c. Antidiuretic hormone
a. Excessive sleeping combined with irregular
eating d. Angiotensin-II
b. Antipsychotic medications and smoking 9. The sodium-phosphate cotransporter (NPT2)
c. An increased need in the aquaporin chan- creates the action by which phosphate is
nel and coffee drinking reabsorbed from the filtrate in the proximal
tubule. NPT2 is inhibited by phosphatonin.
d. Antipsychotic medications and coffee
What condition can cause an overproduction
drinking
of phosphatonin resulting in
5. There are two types of diabetes insipidus (DI), hypophosphatemia?
neurogenic and nephrogenic. In nephrogenic a. Tumor-induced osteomyelitis
DI there is an inability of the kidney to concen-
b. Tumor-induced hypopituitarism
trate urine and to conserve free water. Nephro-
genic DI can be either genetic or acquired. c. Tumor-induced syndrome of antidiuretic
What are the causes of nephrogenic DI? hormone
a. Head injury and cranial surgery d. Tumor-induced osteomalacia
b. Oral antidiabetic drugs and smoking 10. Magnesium levels are important indicators to
c. Lithium and hypokalemia a variety of bodily functions. What is severe
hypermagnesemia associated with?
d. Hypocalcemia and hypernatremia
a. Muscle and respiratory paralysis
6. In a person with fluid volume deficit, there is
b. Cardiac arrest and 2 pulmonary paralysis
a dehydration of brain and nerve cells. What
can occur if fluid volume deficit is corrected c. Complete heart block and cardiac arrythmias
to rapidly? d. Cardiac arrythmias and respiratory paralysis
a. Nerve cells absorb too much sodium and 11. To calculate the H2CO3 content of the
cease to function blood, you need to measure the PCO2 (partial
b. Brain cells shut down to prevent cerebral pressure of CO2) by its solubility coefficient.
edema What is the solubility coefficient of CO2?
c. Fluid volume increases at a rate the body a. 0.03
cannot tolerate b. 0.3
d. Cerebral edema occurs with potentially c. 0.04
severe neurologic impairment
d. 0.4

Copyright 2011. Wolters Kluwer Health | Lippincott Williams & Wilkins. Study Guide for Porths Essentials of Pathophysiology, Third Edition.
LWBK707-c8_p39-48.qxd 8/19/10 1:46 PM Page 48 Aptara Inc

48 UNIT 2 INTEGRATIVE BODY FUNCTIONS

12. The body regulates the pH of its fluids by 17. Metabolic acidosis has four main causes.
what mechanism? (Mark all that apply.) Which laboratory test is used to determine
a. Chemical buffer systems of the body fluids the cause of metabolic acidosis?
b. The liver a. Acid-base deficit
c. The lungs b. Arterial blood gas
d. The cardiovascular system c. Anion gap
e. The kidneys d. Serum bicarbonate

13. By reabsorbing HCO3 from the glomerular 18. A change in the pH of the body affects all
filtrate and excreting H+ from the fixed acids organ systems. When the pH falls to less than
that result from lipid and protein metabolism, 7. 0, what can occur in the cardiovascular
the kidneys work to return or maintain the system? (Mark all that apply?)
pH of the blood to normal or near-normal a. Vasodilate the vascular bed, causing the
values. How long can this mechanism client to go into shock
function when there is a change in the pH of b. Vasoconstrict the vascular bed to preserve
body fluids? the primary organs
a. Minutes c. Increase cardiac contractility, causing
b. Hours cardiac dysrhythmias
c. Days d. Reduce cardiac contractility, causing
d. Weeks cardiac dysrhythmias

14. Laboratory tests give us very valuable 19. Respiratory acidosis occurs at a time when
information about what is happening in the the plasma pH falls below 7.35, and arterial
body. What laboratory test is a good indicator PCO2 rises above 50 mm Hg. Because CO2 eas-
of the how the buffer systems in the body are ily crosses the blood-brain barrier, what signs
working? and symptoms of respiratory acidosis might
you see? (Mark all that apply.)
a. Acid-base test
a. Irritability
b. Urine acidity test
b. Muscle twitching
c. H+ level test
c. Psychological disturbances
d. Base excess or deficit test
d. Seizures
15. There are both metabolic and respiratory
e. Psychotic breaks
effects on the acid-base balance in the body.
How do metabolic disorders change the pH of 20. Respiratory alkalosis is caused by hyperventi-
the body? lation, which is recognized as a respiratory
a. Alter the plasma HCO3 rate in excess of that which maintains
normal plasma Pco2 levels. What is a
b. Alter urine H+ content
common cause of respiratory alkalosis?
c. Alter CO2 levels in the lungs
a. Hyperventilation syndrome
d. Alter O2 levels in the major organ systems
b. Hypoventilation syndrome
16. The body has built-in compensatory mecha- c. Cluster breathing
nisms that take over when correction of pH is
d. Kussmaul breathing
not possible or cannot be immediately
achieved. What are these compensatory
mechanisms considered?
a. Long-term measures that back up first-line
correction mechanisms
b. Interim measures that permit survival
c. Short-term measures that depend on first-
line correction mechanisms
d. Ways to correct the primary disorder

Copyright 2011. Wolters Kluwer Health | Lippincott Williams & Wilkins. Study Guide for Porths Essentials of Pathophysiology, Third Edition.
LWBK707-c9_p49-52.qxd 8/19/10 1:06 PM Page 49 Aptara Inc

CHAPTER
Stress and Adaptation

SECTION I: LEARNING SECTION II: ASSESSING


OBJECTIVES YOUR UNDERSTANDING
1. Cite Cannons four features of homeostasis. Activity A Fill in the blanks.
2. Describe the components of a control 1. The ability of the body to function and
system, including the function of a negative maintain under conditions of
feedback system. change in the internal and external environ-
ment depends on the thousands of
3. State Selyes definition of stress.
control systems that regulate
4. Define stressor. body function.
5. Explain the interactions among components 2. is achieved only through a
of the nervous system in mediating the stress system of carefully coordinated physiologic
response. processes that oppose change.
6. Describe the stress responses of the neuroen- 3. Most control systems in the body operate by
docrine and immune, and the musculoskele- feedback mechanisms.
tal system.
4. Selye described as a state mani-
7. Explain the purpose of adaptation. fested by a specific syndrome of the body
developed in response to any stimuli that
8. List factors that influence a persons adaptive
made an intense systemic demand on it.
capacity.
5. Stress may contribute directly to the produc-
9. Contrast anatomic and physiologic reserve.
tion or exacerbation of a .
10. Propose a way by which social support may
6. There is evidence that the axis,
serve to buffer challenges to adaptation.
the hormonal, and the
11. Describe the physiologic and psychological nervous systems are
effects of a chronic stress response. differentially activated depending on the
type and intensity of the stressor.
12. Describe the three states characteristic of
posttraumatic stress disorder. 7. Human beings, because of their highly devel-
oped nervous system and intellect, usually
13. List five nonpharmacologic methods of treat-
have alternative mechanisms for
ing stress.
and have the ability to control many aspects
of their environment.

49
LWBK707-c9_p49-52.qxd 8/19/10 1:06 PM Page 50 Aptara Inc

50 UNIT 2 INTEGRATIVE BODY FUNCTIONS

8. The means used to attain this balance are 10. Coping d. Stressor that
called . mechanisms produces a response
9. is considered a restorative func- e. Enhances stress-
tion in which energy is restored and tissues induced release of
are regenerated. vasopressin from the
posterior pituitary
10. is commonly used in excess
f. Ability of body sys-
and can suppress the immune system.
tems to increase
their function given
Activity B Consider the following figure.
the need to adapt
g. Regulation of heart
rate and vasomotor
tone
h. Suppresses
osteoblast activity,
hematopoiesis, and
protein synthesis
i. Stimulates the adre-
nal gland to synthe-
size and secrete the
glucocorticoid hor-
mones
j. Increases water
retention by the kid-
neys and produces
vasoconstriction of
1. In the above figure, trace the activation of the blood vessels
hypothalamus to the release of corticotrophin to
the effect on the adrenal gland and to the final
release of cortisol. Also, label the locus ceruleus.
Activity D Briefly answer the following.
Activity C Match the key terms in Column A
with their definitions in Column B. 1. How does the body regulate and maintain
homeostasis? Give one example.
Column A Column B
1. Conditioning a. A personality
factors characteristic that
includes a sense of
2. Antidiuretic
having control over 2. Describe the stages of general adaptation
hormone
the environment syndrome.
3. Baroreflex b. Factors used to cre-
4. Allostasis ate a new balance
between a stressor
5. Physiologic and the ability to
reserve deal with it
6. Angiotensin II c. Physiologic changes 3. Stress will activate numerous body systems.
in the neuro- Many are based in neuroendocrine activity.
7. Hardiness List the effects of neuroendocrine activation
endocrine, auto-
8. Cortisol nomic, and immune in response to stress.
systems in response
9. ACTH
to real or perceived
challenges to home-
ostasis

Copyright 2011. Wolters Kluwer Health | Lippincott Williams & Wilkins. Study Guide for Porths Essentials of Pathophysiology, Third Edition.
LWBK707-c9_p49-52.qxd 8/19/10 1:06 PM Page 51 Aptara Inc

CHAPTER 9 STRESS AND ADAPTATION 51

4. Trained athletes use physiological and 4. A number of responses occur in the body to
anatomic reserve to achieve top-level perform- the release of neurohormones when the body
ance. Explain and give examples of how this is encounters stress, including which of the fol-
accomplished. lowing?
a. Increase in appetite
b. Decreased cerebral blood flow
c. Decrease in awareness
d. Inhibition of reproductive function
5. What are the physiologic and anatomic causes
5. Chronic and excessive activation of the stress
of posttraumatic stress disorder?
response has been shown to play a part in the
development of long-term health problems.
The stress response can also result from
chronic illness. Which health problems have
been linked to a stress response that is
chronic and excessive?
a. Suicide and immune disorders
SECTION III: PRACTICING b. Depression and renal disease
FOR NCLEX c. Immune disorders and brain tumors
d. Suicide and thrombosis in the extremities
Activity E Answer the following questions.
6. Our bodys response to psychological
1. The control systems of the body act in many perceived threats is not regulated to the same
ways to maintain homeostasis. These control degree as our bodys response to physiologic
systems regulate the functions of the cell and perceived threats. The psychological
integrate the functions of different organ sys- responses may be:
tems. What else do they do?
a. Appropriate and limited.
a. Control life processes
b. Inappropriate and sustained.
b. Feed cells under stress
c. Regulated by a positive feedback system
c. Act on invading organisms
d. The result of a baroreflex-mediated response
d. Shut down the body at death
7. Adaptation implies that an individual has
2. It has long been known that our bodies need successfully created a new balance between
a stable internal environment to function the stressor and the ability to deal with it.
optimally. What serves to fulfill this need? The safety margin for adaptation of most
a. Organ systems body systems is considerably greater than
b. Control systems that needed for normal activities. The
method of adaptation that allows the body
c. Biochemical messenger systems
to live with only one of a pair of organs (i.e.,
d. Neurovascular systems one lung or one kidney) is called?
3. The general adaptation syndrome is what a. Genetic endowment
occurs in the body in response to stressors. b. Physiologic reserve
When the bodys defenses are depleted, signs
c. Anatomic reserve
of wear and tear or systemic damage appear.
Which of the following diseases have been d. Health status
linked to stress and are thought to be encour-
aged by the body itself when it can no longer
adapt to the stress in a healthy manner?
a. Psychotic disorders
b. Osteogenesis sarcomas
c. Rheumatic disorders
d. Infections of the head and neck

Copyright 2011. Wolters Kluwer Health | Lippincott Williams & Wilkins. Study Guide for Porths Essentials of Pathophysiology, Third Edition.
LWBK707-c9_p49-52.qxd 8/19/10 1:06 PM Page 52 Aptara Inc

52 UNIT 2 INTEGRATIVE BODY FUNCTIONS

8. Psychosocial factors can impact the bodys 14. Match the following terms with their defini-
response to stress either positively or tions.
negatively. It has been shown that social
Term Definition
networks play a part in the psychosocial and
physical integrity of a person. How do social 1. Corticotropin- a. Increased cortico-
networks affect how a body deals with stress? releasing factor steroid production
a. By stepping in and making decisions for and atrophy of the
2. Fight-or-flight
the person thymus
response
b. By reapportioning the finances of the person b. Endocrine regulator
3. Allostatic load of pituitary and ad-
c. By mobilizing the resources of the person
4. Endocrine- renal activity and
d. By protecting the person from other neurotransmitter
immune
internal stressors involved in auto-
interactions
9. The acute stress response can be detrimental in nomic nervous sys-
people with pre-existing physical or mental tem activity,
health problems. In which of these clients could metabolism, and
the acute stress response cause further problems? behavior
a. Client who is post resection of a brain tumor c. Physiologic
changes in the neu-
b. Client who is schizophrenic and off his or
roendocrine, auto-
her medications
nomic, and
c. Client with a broken femur immune systems
d. Client with heart disease occurring in re-
10. Some clients experience chronic activation of
sponse to real or
the stress response as a result of experiencing perceived chal-
a severe trauma. Which of the following is lenges to home-
the disorder that can occur when the stress ostasis
response is chronically activated? d. Most rapid of the
a. Posttraumatic stress disorder
stress responses,
representing the
b. Chronic renal insufficiency basic survival
c. Schizophrenia response
d. Postdelivery depression
15. It is thought that there is an interaction
11. In a organism it is necessary for between the neuroendocrine system and the
the composition of the internal environment immune system. It has been postulated that
to be compatible with the survival needs of these interactions play a significant role in
the individual cells. autoimmune diseases. These systems have
12. Selye suggested that stress could have positive what in common? Select all that apply.
influences on the body, and these periods of a. They share common signal pathways.
positive stress are called .
b. Hormones and neuropeptides can change
13. The first goal of treatment of stress disorders what immune cells do.
is to aid clients in avoiding those coping c. Mediators of the immune system can
mechanisms that cause their health to be at modify neuroendocrine function.
risk. Secondly, the treatment of stress
d. They are symbiotic systems and cannot
disorders should engage them in alternative
work without each other.
strategies that reduce stress. Which are non-
pharmacologic treatments of stress disorders?
Select all that apply.
a. Lithium therapy
b. Music therapy
c. Education therapy
d. Massage therapy

Copyright 2011. Wolters Kluwer Health | Lippincott Williams & Wilkins. Study Guide for Porths Essentials of Pathophysiology, Third Edition.
LWBK707-c10_p53-56.qxd 8/19/10 1:07 PM Page 53 Aptara Inc

10

CHAPTER
Disorders of
Nutritional Status

SECTION I: LEARNING 12. State the difference between protein-calorie


starvation (i.e., marasmus) and protein mal-
OBJECTIVES nutrition (i.e., kwashiorkor).
1. Define nutritional status. 13. Compare the eating disorders and complica-
tions associated with anorexia nervosa and
2. Define calorie and state the number of
the binge-purge syndrome.
calories derived from the oxidation of 1 g of
protein, fat, or carbohydrate.
3. Describe the function of adipose tissue in
terms of energy storage. SECTION II: ASSESSING
4. State the purpose of the Recommended YOUR UNDERSTANDING
Dietary Allowance of calories, proteins, fats,
carbohydrates, vitamins, and minerals. Activity A Fill in the blanks.

5. Describe methods used for a nutritional 1. describes the condition of the


assessment. body related to the availability and use of
nutrients.
6. State the factors used in determining body
mass index and explain its use in evaluating 2. is the organized process
body weight in terms of undernutrition and through which nutrients such as
overnutrition. carbohydrates, fats, and proteins are broken
down, transformed, or otherwise converted
7. Define and discuss the causes of obesity and into cellular energy.
health risks associated with obesity.
3. Energy expenditure can be increased by
8. Differentiate upper and lower body obesity increasing and/or nonexercise
and their implications in terms of health risk. activity thermogenesis.
9. Discuss the treatment of obesity in terms of diet, 4. More than 90% of body energy is stored in
behavior modification, exercise, social support, the tissues of the body.
pharmacotherapy, and surgical methods.
5. acts at the level of the
10. Explain the use of body mass index in evalu- hypothalamus to decrease food intake and
ating body weight in terms of overnutrition. increase energy expenditure through an
11. List the major causes of malnutrition and increase in thermogenesis and sympathetic
starvation. nervous system activity.

53
LWBK707-c10_p53-56.qxd 8/19/10 1:07 PM Page 54 Aptara Inc

54 UNIT 2 INTEGRATIVE BODY FUNCTIONS

6. The defines the intakes that 23. There is convincing evidence that
meet the nutrient needs of almost all healthy physical activity decreases the
persons in a specific age and sex group. risk of overweight and obesity.
7. (% DV) tells the consumer what 24. does afford significant weight
percent of the DV one serving of a food or loss, long-term weight loss maintenance,
supplement supplies. improved quality of life, decreased incidence
of associated diseases, and decreased all-cause
8. are required for growth and
mortality.
maintenance of body tissues, enzymes and
antibody formation, fluid and electrolyte 25. Obesity is the most prevalent nutritional dis-
balance, and nutrient transport. order affecting the population
in the United States.
9. The rate of protein breakdown can be estimated
by measuring the amount of in 26. and are
the urine. conditions in which a person does not
receive or is unable to use an adequate
10. The saturated fatty acids blood
amount of nutrients for body function.
cholesterol, whereas the monounsaturated
and polyunsaturated fats blood 27. Protein and energy malnutrition represents a
cholesterol. depletion of the bodys lean tissues caused by
and/or catabolic stress.
11. Trans fatty acids LDL cholesterol
and HDL cholesterol. 28. The child with has a wasted
appearance, with loss of muscle mass, stunted
12. There is no specific dietary requirement for
growth, and loss of subcutaneous fat.
.
29. Bulimia nervosa is defined by
13. are a group of organic
binge eating and activities including
compounds that act as catalysts in various
vomiting, fasting, excessive exercise, and use
chemical reactions.
of diuretics, laxatives, or enemas to compen-
14. increases stool bulk and sate for that behavior.
facilitates bowel movements.
Activity B Match the key terms in Column A
15. The contains the feeding center
with their definitions in Column B.
for hunger and satiety.
1.
16. A decrease in blood causes hunger.
Column A Column B
17. measurements provide a means
for assessing body composition, particularly 1. Adipocytes a. The amount of
fat stores and skeletal muscle mass. nitrogen taken in
2. Skinfold
by way of protein is
18. The uses height and weight to thickness
equivalent to the
determine healthy weight.
3. Kwashiorkor nitrogen excreted
19. Studies have indicated that waist b. Malnutrition caused
4. Calorie
at the abdomen is highly corre- by inadequate pro-
lated with insulin resistance. 5. Diet-induced tein intake in the
thermogenesis presence of fair to
20. is defined as having excess
body fat, enlarged fat cells, and even an 6. Metabolites good energy
increased number of fat cells. c. Chemical intermedi-
7. Nitrogen
ates of metabolism
21. Research suggests that may be a balance
more important factor for morbidity and d. A reasonable assess-
8. Catabolism ment of body fat,
mortality than overweight or obesity.
particularly if taken
22. has been found to have little or at multiple sites
no effect on metabolic variables, central obe-
sity, or cardiovascular risk factors or future
amount of weight loss.

Copyright 2011. Wolters Kluwer Health | Lippincott Williams & Wilkins. Study Guide for Porths Essentials of Pathophysiology, Third Edition.
LWBK707-c10_p53-56.qxd 8/19/10 1:07 PM Page 55 Aptara Inc

CHAPTER 10 DISORDERS OF NUTRITIONAL STATUS 55

9. Resting energy e. The amount of 2. How is bioimpedance preformed and what


equivalent energy needed to does it do?
raise the
10. Kilocalorie
temperature of 1 kg
of water by 1C
f. Fat cells
g. Breakdown of com-
3. What are the nongenetic causes of obesity?
plex molecules
h. Amount of heat or
energy required to
raise the
temperature of 1 g
of water by 1C 4. What are the causes of anorexia?
i. Energy used by the
body for the diges-
tion, absorption,
and assimilation
j. Used for predicting
energy expenditure 5. What are the criteria for the diagnosis of
bulimia nervosa?
2.
Column A Column B
1. Anorexia a. Mixture of fatty
nervosa acids and glycerol
2. Ghrelin b. Result of a deficiency 6. Describe the criteria for binge eating.
of linoleic acid
3. Dermatitis
c. Hormone that may
4. Triglycerides stimulate hunger
5. Marasmus d. Unsaturated oils are
partially
6. Trans fatty hydrogenated
acids
e. Minerals present in SECTION III: APPLYING
7. Macrominerals large amounts in
the body
YOUR KNOWLEDGE
f. Characterized by Activity D Consider the scenario and answer
determined dieting, the questions.
often accompanied
by compulsive A 14-year-old girl is brought to the clinic by her
exercise mother for a sports physical. The young lady is
5 feet, 4 inches tall, weighs 95 pounds, and is
g. Protein and calorie
considered to have a small frame. The nurse notes
deficiency
the clients weight and suspects an eating disorder.
Activity C Briefly answer the following. 1. What questions would be appropriate in the
nursing history to assess for an eating
1. What are the two types of adipose tissue? How
disorder?
do they differ?

Copyright 2011. Wolters Kluwer Health | Lippincott Williams & Wilkins. Study Guide for Porths Essentials of Pathophysiology, Third Edition.
LWBK707-c10_p53-56.qxd 8/19/10 1:07 PM Page 56 Aptara Inc

56 UNIT 2 INTEGRATIVE BODY FUNCTIONS

2. The nurse knows that the DSM-IV-TR diagnos- 5. The body mass index (BMI) is the measurement
tic criteria for anorexia nervosa include what? used to determine a persons healthy weight. A
BMI between 18.5 and 24.9 is considered the
lowest health risk in relation to the weight of a
person. How is the BMI calculated?
a. BMI  weight [pounds]/height [feet2]
b. BMI  weight [kg]/height [feet2]
c. BMI  weight [pounds]/height [meter2]
SECTION IV: PRACTICING d. BMI  weight [kg]/height [meter2]
FOR NCLEX 6. Two types of obesity are recognized: upper
body obesity and lower body obesity. How is
Activity E Answer the following questions. the type of obesity determined?
1. Adipose tissue is now known to be both an a. Waist/hip circumference
endocrine and a paracrine organ because of b. Chest circumference/weight
the factors it secretes. What are these factors?
c. Chest/hip circumference
(Mark all that apply.)
d. Waist circumference/weight
a. Leptin
b. Growth hormone 7. Anorexia nervosa, bulimia nervosa, and binge-
eating disorder are becoming more and more
c. Adipokines
common, with assessments for these disorders
d. Insulin resistance factor being made as young 9 years of age. In the
e. Adiponectin adult population, what means of controlling
binge eating is most prevalent in men?
2. When nutritional requirements are needed for
a specific group, what dietary requirements are a. Self-induced vomiting
used? b. Compulsive exercise
a. Estimated average requirement c. Laxative use
b. Adequate intake d. Compulsive working
c. Recommended Dietary Allowance 8. Childhood obesity has now been recognized
d. Dietary Reference Intake as a major problem in the pediatric
population. What diseases are pediatricians
3. Fat is a necessary part of the diet. The Food
now seeing in their clients as a direct result of
and Nutrition Board has set what percent of
childhood obesity?
fat as necessary in our diet?
a. Type I diabetes
a. 10%
b. Dyslipidemia
b. 20%
c. Hypotension
c. 30%
d. Psychosocial acceptance
d. 40%
9. Malnutrition is not something that is consid-
4. It is the hypothalamus that tells us when we
ered common in the general population in the
are hungry or full. Its message is mediated by
United States. However, certain populations
input from the gastrointestinal tract. There are
are more prone to malnutrition than others.
also centers in the hypothalamus that regulate
One of these populations is hospitalized
energy balance and metabolism based on the
patients. Why is this true?
secretion of what hormones?
a. Appetites are increased by fever and pain
a. Cholecystokinin (CCK) and glucagon-like
peptide-1 (GLP-1) b. Special diets can increase appetite
b. Ghrelin and thyroid c. Pain and medications can decrease appetite
c. Thyroid and adrenocortical hormones d. Only healthy diets are served in hospitals.
d. Adrenocortical hormones and
cholecystokinin (CCK)

Copyright 2011. Wolters Kluwer Health | Lippincott Williams & Wilkins. Study Guide for Porths Essentials of Pathophysiology, Third Edition.
LWBK707-c11_p57-61.qxd 8/19/10 1:08 PM Page 57 Aptara Inc

11

CHAPTER
Disorders of White
Blood Cells and
Lymphoid Tissues

SECTION I: LEARNING 10. Use the predominant white blood cell type
and classification of acute or chronic to
OBJECTIVES describe the four general types of leukemia.
1. List the cells and tissues of the hematopoietic 11. Explain the manifestations of leukemia in
system. terms of altered cell differentiation.
2. Trace the development of the different blood 12. Describe the following complications of
cells from their origin in the pluripotent acute leukemia and its treatment: leukostasis,
bone marrow stem cell to their circulation in tumor lysis syndrome, hyperuricemia, and
the bloodstream. blast crisis.
3. Define the terms leukopenia, neutropenia, and 13. Relate the clonal expansion of immunoglob-
aplastic anemia. ulin-producing plasma cells and accompany-
ing destructive skeletal changes that occur
4. Cite two general causes of neutropenia.
with multiple myeloma in terms of manifes-
5. Describe the mechanism of symptom produc- tations and clinical course of the disorder.
tion in neutropenia.
6. Use the concepts regarding the central and
peripheral lymphoid tissues to describe the
site of origin of the malignant lymphomas,
SECTION II: ASSESSING
leukemias, and plasma cell dyscrasias. YOUR UNDERSTANDING
7. Explain how changes in chromosomal struc- Activity A Fill in the blanks.
ture and gene function can contribute to the
development of malignant lymphomas, 1. The white blood cells include the
leukemias, and plasma cell dyscrasias. , monocyte/macrophages, and
lymphocytes.
8. Contrast and compare the signs and
symptoms of non-Hodgkin and Hodgkin 2. T lymphocytes mature in the .
lymphomas. 3. The B lymphocytes differentiate to form
9. Describe the measures used in treatment of immunoglobulin-producing
non-Hodgkin and Hodgkin lymphomas. cells.

57
LWBK707-c11_p57-61.qxd 8/19/10 1:08 PM Page 58 Aptara Inc

58 UNIT 3 HEMATOPOIETIC FUNCTION

4. Another population of lymphocytes includes 17. The are malignant neoplasms


the cells, which do not share the of cells originally derived from hematopoietic
specificity or characteristics of the T or the B precursor cells.
lymphocytes, but have the ability to lyse tar-
18. The leukemias involve
get cells.
immature lymphocytes and their progenitors
5. The granulocyte and monocyte cell lines that originate in the bone marrow but
derive from the stem cells and infiltrate the spleen, lymph nodes, CNS, and
the lymphocytes from the stem other tissues.
cells.
19. Cytogenetic studies have shown that
6. The bodys lymphatic system consists of the recurrent chromosomal changes occur in
lymphatic vessels, lymphoid tissue and over half of all cases of .
lymph nodes, , and
20. usually have a sudden and
.
stormy onset with signs and symptoms
7. T lymphocytes travel to the thymus where related to depressed bone marrow function.
they differentiate into helper
21. There are two types of acute leukemia: acute
T cells and cytotoxic T cells.
and acute .
8. In anemia, all of the myeloid
22. are malignancies involving
stem cells are affected, resulting in anemia,
proliferation of more fully differentiated
thrombocytopenia, and agranulocytosis.
myeloid and lymphoid cells.
9. denotes a virtual absence of
23. Chronic lymphocytic leukemia, a clonal
neutrophils.
malignancy of , is the most
10. Early signs of infection of common form of leukemia in adults in the
include mild skin lesions, stomatitis, pharyn- Western world.
gitis, and diarrhea.
24. Severe congenital neutropenia is known as
11. is a self-limiting lymphoprolifer- syndrome.
ative disorder caused by the Epstein-Barr virus.
25. It is generally believed that chronic myeloge-
12. can involve lymphocytes, gran- nous leukemia (CML) develops when a single,
ulocytes, and other blood cells. pluripotential, hematopoietic stem cell
acquires a chromosome.
13. originate in peripheral
lymphoid structures such as the lymph nodes 26. are characterized by expansion
where B and T lymphocytes undergo differen- of a single clone of immunoglobulin-
tiation and proliferation. producing plasma cells and a resultant
increase in serum levels of a single
14. -cell lymphomas are the most
monoclonal immunoglobulin or its
common type of lymphoma in the Western
fragments.
world.
27. The development of lesions in
15. Four variants of classical Hodgkin lymphoma
multiple myeloma is thought to be related to
have been described: sclerosis,
an increase in expression by osteoblasts of
mixed cellularity, -rich, and
the receptor activator of the nuclear
lymphocyte depleted.
factor-B.
16. Persons with are staged accord-
28. One of the characteristics resulting from the
ing to the number of lymph nodes that are
proliferating osteoclasts in multiple myeloma
involved, whether the lymph nodes are on
is the unregulated production of a monoclonal
one or both sides of the diaphragm, and
antibody referred to as the .
whether there is disseminated disease involv-
ing the bone marrow, liver, lung, or skin.

Copyright 2011. Wolters Kluwer Health | Lippincott Williams & Wilkins. Study Guide for Porths Essentials of Pathophysiology, Third Edition.
LWBK707-c11_p57-61.qxd 8/19/10 1:08 PM Page 59 Aptara Inc

CHAPTER 11 DISORDERS OF WHITE BLOOD CELLS AND LYMPHOID TISSUES 59

Activity B Match the key terms in Column A 3. Describe the clinical manifestations of non-
with their definitions in Column B. Hodgkin lymphoma (NHL) and relate the
symptoms to the pathologic cause.
Column A Column B
1. Heterophil a. Neoplasm involving
B or T cells
2. Leukopoiesis
b. Translocation on
3. Burkitt chromosome 8
lymphoma 4. There are two major differences between
c. Found in more than Hodgkin lymphoma and NHL. Differentiate
4. Non-Hodgkin 90% of persons Hodgkin lymphoma from NHL.
lymphomas with CML
5. Neutropenia d. Production of white
blood cells
6. Reed-Sternberg
e. Used for the diagno-
cells
sis of infectious
7. Kostmann mononucleosis 5. What are the potential causes of leukemia?
syndrome f. An abnormally low
8. Philadelphia number of
chromosome neutrophils
g. Immature precursor
9. Blast cells
cells 6. Compare and contrast acute lymphocytic
10. ZAP-70 h. Definitive marker leukemia (ALL) and acute myelocytic leukemia
for Hodgkin (AML).
lymphoma
i. An arrest in
myeloid maturation
j. Normal T-cell
protein, abnormal
7. Describe the progression of CML through its
in chronic lympho-
three stages.
cytic leukemia
(CLL)

Activity C Briefly answer the following.


1. Neutrophils are very important as a first line of 8. What are the potential causes of multiple
defense against viral/bacterial infection. myeloma?
Explain what a neutrophil does and the condi-
tion that results from a deficiency of
neutrophils.

2. Describe the pathogenesis of infectious


mononucleosis.

Copyright 2011. Wolters Kluwer Health | Lippincott Williams & Wilkins. Study Guide for Porths Essentials of Pathophysiology, Third Edition.
LWBK707-c11_p57-61.qxd 8/19/10 1:08 PM Page 60 Aptara Inc

60 UNIT 3 HEMATOPOIETIC FUNCTION

SECTION III: APPLYING 4. Drug-induced neutropenia is a disease that


has significantly increased in incidence over
YOUR KNOWLEDGE the last several decades. What is the attribut-
ing factor in the increased incidence of drug-
Activity D Consider the scenario and answer
induced neutropenia?
the questions.
a. Treatment of cancer by chemotherapeutic
Lucy, a 2 year old, has been diagnosed with ALL drugs
and has been admitted to your unit for treatment.
b. The decrease in the use of street drugs
How would you answer when the parents ask,
c. The destruction of tissue cells by cocaine
1. What caused Lucys leukemia?
d. The new drugs developed to treat autoim-
mune diseases
5. Infectious mononucleosis is a lymphoprolif-
erative disorder caused by the EBV that is
usually self-limiting and nonlethal. Which of
2. What kind of treatment will Lucy have? the following complications can arise during
this mostly benign disease?
a. Peripheral nerve palsies
b. Rupture of the spleen
c. Rupture of the lymph nodes
d. Severe bacterial infections
SECTION IV: PRACTICING 6. You are presenting an educational event to a
FOR NCLEX group of cancer patients. What would you
cite as the most commonly occurring hema-
Activity E Answer the following questions. tologic cancer?
1. Progenitor cells, or parent cells, for a. Acute lymphocytic leukemia
myelopoiesis and lymphopoiesis are derived b. Hodgkin lymphomas
from which of the following? c. Non-Hodgkin lymphomas
a. Pluripotent stem cells d. Mantle cell lymphoma
b. Unipotent cells
7. Endemic Burkitt lymphoma occurs in regions
c. Multipotential progenitor cells of Africa where what other infections are
d. Myeloproliferative cells common?
2. What is the name of the region of the lymph a. Herpes zoster and Epstein Barr
nodes that contain most of the T cells? b. Herpes zoster and streptococcal
a. The primary follicles c. Malaria and streptococcal
b. The paracortex d. Epstein Barr and malaria
c. The secondary follicles 8. ALL and AML are two distinct disorders with
d. The primary cortex similar presenting clinical features. What
clinical feature do ALL and AML share?
3. Kostmann syndrome is a severe congenital
neutropenia. Which of the following is char- a. Night sweats
acteristic of this condition? b. Weight gain
a. Bone marrow disorders c. High fever
b. Severe viral infections d. Polycythemia
c. Autoimmune disorders
d. Severe bacterial infections

Copyright 2011. Wolters Kluwer Health | Lippincott Williams & Wilkins. Study Guide for Porths Essentials of Pathophysiology, Third Edition.
LWBK707-c11_p57-61.qxd 8/19/10 1:08 PM Page 61 Aptara Inc

CHAPTER 11 DISORDERS OF WHITE BLOOD CELLS AND LYMPHOID TISSUES 61

9. Definitive diagnosis of multiple myeloma 13. You are speaking to a group of genetic
includes the triad of bone marrow plasmacy- students touring your hospitals laboratory.
tosis, lytic bone lesions, and what? You talk about the possibility of a genetic pre-
a. Oligoclonal bands in the CSF disposition for the leukemias being suggested
because of the increased incidence of the dis-
b. Bence-Jones proteins in the urine
ease among a number of congenital disorders.
c. Serum M-protein depression Which congenital disorders are these? (Mark
d. BCR-ABL fusion protein in serum all that apply.)
10. CLL commonly causes hypogammaglobuline- a. Cushing syndrome
mia. This makes clients with CLL more b. Neurofibromatosis
susceptible to infection. What are the most c. Fanconi anemia
common infectious organisms that attack
d. Down syndrome
clients with CLL?
e. Prader-Willi syndrome
a. Acne rosacea
b. Pseudomonas aeruginosa 14. Tumor lysis syndrome, the massive necrosis
of malignant cells that can occur during the
c. Staphylococcus aureus
initial phase of treatment of ALL, can lead to
d. Escherichia coli metabolic disorders that are life-threatening.
11. Large granular lymphocytes, or natural killer Which metabolic disorders can occur because
cells, have the ability to target of tumor lysis syndrome? (Mark all that
cells. apply.)
a. Hyperuricemia
12 Which lymphatic tissue is associated with
mucous membranes and called mucus- b. Hypokalemia
associated lymphatic tissue, or MALT? (Mark c. Acidosis
all that apply.) d. Alkalosis
a. Genitourinary systems and central nerv- e. Hypocalcemia
ous system
15. Secondary malignancies in survivors of
b. Respiratory passages and cardiovascular
Hodgkin lymphoma have been attributed
system
mainly to therapy.
c. Alimentary canal and genitourinary sys-
tems
d. Cardiovascular system central nervous
system

Copyright 2011. Wolters Kluwer Health | Lippincott Williams & Wilkins. Study Guide for Porths Essentials of Pathophysiology, Third Edition.
LWBK707-c12_p62-66.qxd 8/19/10 2:23 PM Page 62 Aptara Inc

12
CHAPTER
Disorders of Hemostasis

SECTION I: LEARNING SECTION II: ASSESSING


OBJECTIVES YOUR UNDERSTANDING
1. Describe the five stages of hemostasis. Activity A Fill in the blanks.
2. Explain the formation of the platelet plug. 1. The term refers to the stoppage
of blood flow.
3. State the purpose of blood coagulation.
2. Platelets have a cell membrane, but have no
4. State the function of clot retraction.
and cannot reproduce.
5. Trace the process of fibrinolysis.
3. The platelet shape is maintained by
6. Compare normal and abnormal clotting. microtubules and and
filaments that support the cell
7. Describe the causes and effects of increased
membrane.
platelet function.
4. The release of from platelets
8. State two conditions that contribute to
results in the proliferation and growth of vas-
increased clotting activity.
cular endothelial cells, smooth muscle cells,
9. State the mechanisms of drug-induced and fibroblasts, and is important in vessel
thrombocytopenia and idiopathic thrombo- repair.
cytopenia and the differing features of the
5. The combined actions of and
disorders in terms of onset and resolution.
lead to the expansion of the
10. Describe the manifestations of enlarging platelet aggregate, which becomes
thrombocytopenia. the primary hemostatic plug.
11. Characterize the role of vitamin K in coagula- 6. The is a step-wise process
tion. resulting in the conversion of the soluble
plasma protein, fibrinogen, into fibrin.
12. State three common defects of coagulation
factors and the causes of each. 7. Most of the coagulation factors are proteins
synthesized in the .
13. Differentiate between the mechanisms of
bleeding in hemophilia A and von 8. It has been suggested that some of these nat-
Willebrand disease. ural anticoagulants may play a role in the
bleeding that occurs with .
14. Describe the effect of vascular disorders on
hemostasis. 9. represents an exaggerated form
of hemostasis that predisposes to thrombosis
15. Explain the physiologic basis of acute dissem-
and blood vessel occlusion.
inated intravascular coagulation.

62
LWBK707-c12_p62-66.qxd 8/19/10 2:23 PM Page 63 Aptara Inc

CHAPTER 12 DISORDERS OF HEMOSTASIS 63

10. , elevated levels of blood lipids Activity B Consider the following figure.
and cholesterol, hemodynamic stress,
diabetes mellitus, and immune mechanisms Intrinsic system
(blood or vessel injury)
may cause vessel damage, platelet adherence,
and, eventually, thrombosis.
11. The common clinical manifestations of
essential are thrombosis and Extrinsic system
(tissue factor)
hemorrhage. Ca++
Ca++
12. In persons with inherited defects in factor V,
the mutant factor Va cannot be inactivated
by .
Ca++
13. Secondary factors that lead to increased
and thrombosis are venous sta-
sis due to prolonged bed rest and immobility, Ca++
myocardial infarction, cancer,
hyperestrogenic states, and oral
contraceptives.
14. from mucous membranes of
1. In the above figure, place the activated factors
the nose, mouth, gastrointestinal tract, and
and proteins in their respective places: Xlla,
uterine cavity is characteristic of platelet
Xla, IXa, Xa, Vlla, thrombin, prothrombin,
bleeding disorders.
fibrinogen, and fibrin.
15. A reduction in platelet number is referred to
Activity C Match the key terms in Column A
as .
with their definitions in Column B.
16. destruction may be caused by
Column A Column B
antiplatelet antibodies, resulting in thrombo-
cytopenia. 1. Thrombin a. Breaks down fibrin
17. thrombocytopenic purpura 2. Fibrinolysis b. May be caused by
results in platelet antibody formation and aplastic anemia
3. Thrombo-
excess destruction of platelets. c. Enzyme that
cytosis
converts fibrinogen
18. may result from inherited
4. Thromboxane to fibrin
disorders of adhesion or acquired defects
A2 d. Factor VIII deficiency
caused by drugs, disease, or extracorporeal
circulation. 5. Plasmin e. Stimulates vasocon-
striction
19. Hemophilia A is an recessive 6. Antiphospho-
disorder that primarily affects males. lipid syndrome f. Autoantibodies that
result in increased
20. In liver disease, synthesis of these 7. Megakaryocytes coagulation activity
is reduced, and bleeding may
8. Factor x g. Process of blood
result.
clot dissolution
9. Hemophilia a
21. Vitamin C deficiency results in , h. Converts prothrom-
where poor collagen synthesis and failure of 10. Thrombocy- bin to thrombin
the endothelial cells to be cemented together topenia
i. Describes elevations
properly causes a fragile wall and bleeding.
in the platelet
22. Common clinical conditions that may cause count above
________ include obstetrical disorders, 1,000,000/L.
massive trauma, shock, sepsis and malignant j. Thrombocyte
disease. precursor

Copyright 2011. Wolters Kluwer Health | Lippincott Williams & Wilkins. Study Guide for Porths Essentials of Pathophysiology, Third Edition.
LWBK707-c12_p62-66.qxd 8/19/10 2:23 PM Page 64 Aptara Inc

64 UNIT 3 HEMATOPOIETIC FUNCTION

Activity D Write the correct sequence of the 5. Disseminated intravascular coagulation is a


terms listed in the boxes provided below. severe condition that is characterized by
widespread coagulation and bleeding. Explain
a. Clot retraction how the disease is initiated and describe its
b. Clot dissolution progression.
c. Activation of coagulation cascade
d. Formation of platelet plug
e. Vessel spasm

S S S S
SECTION III: PRACTICING
Activity E Briefly answer the following.
FOR NCLEX
1. Explain the five stages of hemostasis. Activity F Answer the following questions.
1. Many different proteins, enzymes, and
hormones are involved in maintaining
hemostasis. Which protein is required for
platelet adhesion?
2. Describe the process of platelet activation and a. von Willebrand factor
plug formation. b. Growth factors
c. Ionized calcium
d. Platelet factor 4
2. There are two pathways that can be activated
by the coagulation process. One pathway
3. The coagulation cascade is activated in multiple begins when factor XII is activated. The other
ways and is integral in maintaining hemostasis. pathway begins when there is trauma to a
Explain the general stimulation and end results. blood vessel. What are these pathways?
a. Clotting and bleeding pathways
b. Extrinsic and intrinsic pathways
c. Inner and outer pathways
d. Factor and trauma pathways
4. There are many causes of bleeding disorders.
3. Anticoagulant drugs prevent throm-
One of the more clinically relevant is drug-
boembolic disorders. How does warfarin, one
induced thrombocytopenia. Explain how
of the anticoagulant drugs, act on the body?
drugs such as quinine, quinidine, and certain
sulfa-containing antibiotics may induce a. Alters vitamin K, reducing its ability to
thrombocytopenia. participate in the coagulation of the blood
b. Increases prothrombin
c. Increases vitamin Kdependent factors in
the liver
d. Increases procoagulation factors

Copyright 2011. Wolters Kluwer Health | Lippincott Williams & Wilkins. Study Guide for Porths Essentials of Pathophysiology, Third Edition.
LWBK707-c12_p62-66.qxd 8/19/10 2:23 PM Page 65 Aptara Inc

CHAPTER 12 DISORDERS OF HEMOSTASIS 65

4. Heparin is an anticoagulant given by 9. A teenage girl, seen in the clinic, is diagnosed


injection to prevent the formation of blood with nonthrombocytopenic purpura. The girl
clots. How does heparin work? states, You have taken a lot of blood from
a. Binds to factor X me. Which of my tests came back
abnormal? How should the nurse respond?
b. Promotes the inactivation of clotting
factors a. Your complete blood count (CBC) with dif-
ferential showed a shift to the left.
c. Binds to factor Xa
b. Your CBC with differential showed you do
d. Promotes the inactivation of factor VIII
not have enough iron
5. The process of clot retraction squeezes serum c. Your CBC with differential showed a
from the clot, thereby joining the edges of normal platelet count
the broken vessel. Through the action of
d. Your CBC with differential showed a
actin and myosin, filaments in platelets con-
normal hematocrit
tribute to clot retraction. Failure of clot
retraction is indicative of what? 10. Disseminated intravascular coagulation is a
a. Absence of factor Xa grave coagulopathy resulting from the
overstimulation of clotting and anticlotting
b. A low platelet count
processes in response to what?
c. An overabundance of factor Xa
a. Disease or injury
d. A high platelet count
b. Septicemia and acute hypertension
6. Thrombocytosis is used to describe elevations c. Neoplasms and nonpoisonous snakebites
in the platelet count above 1,000,000/L. It is
d. Severe trauma and acute hypertension
either a primary or a secondary thrombocyto-
sis. Secondary thrombocytosis can occur as a 11. The five stages of hemostasis are given below
reactive process due to what? in random order. Put them into their correct
a. Crohn disease order.
b. Lyme disease A. Clot dissolution
c. Hirschsprung disease B. Blood coagulation
d. Megacolon C. Vessel spasm
D. Clot retraction
7. A 57-year-old man is diagnosed with throm-
bocytopenia. The nurse knows that thrombo- E. Formation of platelet plug
cytopenia refers to a decrease in the number a. CABED
of circulating platelets. The nurse also knows
b. ACBDE
that thrombocytopenia can result from what?
c. CEBDA
a. Decreased platelet production
d. ECDBA
b. Increased platelet survival
c. Decreased sequestration of platelets 12. The coagulation cascade is the third compo-
nent of the hemostatic process. It is a step-
d. Increased platelet production
wise process resulting in the conversion of
8. A young man has been diagnosed with the soluble plasma protein, fibrinogen, into
hemophilia and the nurse is planning his dis- fibrin. This multistep process ensures that a
charge teaching. She knows to include what massive episode of clotting
in her discharge teaching? does not occur by chance.
a. Only use nonsteroidal anti-inflammatory 13. is a natural mucopolysaccharide
drugs for mild pain anticoagulant that occurs in the lungs and
b. Prevent trauma to the body intestinal mucosa.
c. The client will be on IV factor VIII therapy
at home
d. It is an X-linked recessive disorder

Copyright 2011. Wolters Kluwer Health | Lippincott Williams & Wilkins. Study Guide for Porths Essentials of Pathophysiology, Third Edition.
LWBK707-c12_p62-66.qxd 8/19/10 2:23 PM Page 66 Aptara Inc

66 UNIT 3 HEMATOPOIETIC FUNCTION

14. When platelets adhere to the vessel wall, they 15. In a client with DIC, microemboli form, caus-
release growth factors that cause smooth ing obstruction of blood vessels and tissue
muscle to grow. This is a major factor in caus- hypoxia. Common clinical signs may be due
ing atherosclerosis. What are the factors that to what? (Mark all that apply.)
influence platelets to adhere to the vessel a. Circulatory failure
wall? (Mark all that apply.)
b. Immunologic failure
a. Hemodynamic stress
c. Renal failure
b. High cholesterol
d. Right ventricular failure
c. Diabetes
e. Respiratory failure
d. Low blood lipids
e. Smoking

Copyright 2011. Wolters Kluwer Health | Lippincott Williams & Wilkins. Study Guide for Porths Essentials of Pathophysiology, Third Edition.
LWBK707-c13_p67-72.qxd 8/19/10 1:10 PM Page 67 Aptara Inc

13

CHAPTER
Disorders of Red
Blood Cells

SECTION I: LEARNING 11. List three causes of aplastic anemia.


OBJECTIVES 12. Compare characteristics of the RBCs in acute
blood loss, hereditary spherocytosis, sickle
1. Trace the development of a red blood cell cell disease, iron-deficiency anemia, and
(RBC) from erythroblast to erythrocyte. aplastic anemia.
2. Discuss the function of iron in the formation 13. Differentiate red cell antigens from antibod-
of hemoglobin. ies in persons with type A, B, AB, or O blood.
3. Describe the formation, transport, and elimi- 14. Explain the determination of the Rh factor.
nation of bilirubin.
15. List the signs and symptoms of a blood trans-
4. State the meaning of the RBC count, percent- fusion reaction.
age of reticulocytes, hemoglobin, hematocrit,
16. Define the term polycythemia.
mean corpuscular volume, and mean corpus-
cular hemoglobin concentration as it relates 17. Compare causes of polycythemia vera and
to the diagnosis of anemia. secondary polycythemia.
5. Describe the manifestations of anemia and 18. Describe the manifestations of polycythemia.
their mechanisms.
19. Cite the factors that predispose to
6. Explain the difference between intravascular hyperbilirubinemia in the infant.
and extravascular hemolysis.
20. Describe the pathogenesis of hemolytic
7. Compare the hemoglobinopathies associated disease of the newborn.
with sickle cell disease and thalassemia.
21. Compare conjugated and unconjugated
8. Explain the cause of sickling in sickle cell bilirubin in terms of production of
disease. encephalopathy in the neonate.
9. Cite common causes of iron-deficiency 22. Explain the action of phototherapy in the
anemia in infancy, adolescence, and treatment of hyperbilirubinemia in the
adulthood. neonate.
10. Describe the relation between vitamin B12
deficiency and megaloblastic anemia.

67
LWBK707-c13_p67-72.qxd 8/19/10 1:10 PM Page 68 Aptara Inc

68 UNIT 3 HEMATOPOIETIC FUNCTION

SECTION II: ASSESSING 14. Hereditary is caused by


abnormalities of the spectrin and ankyrin
YOUR UNDERSTANDING membrane proteins that lead to a gradual loss
of the membrane surface.
Activity A Fill in the blanks.
15. are caused by deficient synthesis
1. The shape of an erythrocyte
of the  chain and by deficient
provides a larger surface area for oxygen
synthesis of the  chain.
diffusion than would a spherical cell of the
same volume, and the thinness of the 16. The most common inherited enzyme defect
enables oxygen to diffuse that results in hemolytic anemia is a
rapidly between the exterior and the deficiency of .
innermost regions of the cell.
17. anemia results from dietary
2. The rate at which hemoglobin is synthesized deficiency, loss of iron through bleeding, or
depends on the availability of increased demands.
for heme synthesis.
18. Iron deficiency in adults in the Western
3. During its transformation from normoblast world is usually the result of .
to reticulocyte, the RBC accumulates
19. anemias are caused by impaired
hemoglobin as the condenses
DNA synthesis that results in enlarged red
and is finally lost.
cells as a result of impaired maturation and
4. Mature RBCs have a life span of approximately division.
months.
20. anemia is a specific form of
5. The RBC relies on the pathway megaloblastic anemia caused by atrophic
for its metabolic needs. gastritis.
6. Large doses of nitrites can result in high 21. describes a disorder of pluripo-
levels of , causing pseudo- tential bone marrow stem cells that results in
cyanosis and tissue hypoxia. a reduction of all three hematopoietic cell
lines.
7. The measures the total number
of RBCs in a microliter of blood. 22. is an abnormally high total
RBC mass with a hematocrit greater than
8. The measures the volume of red
50%.
cell mass in 100 mL of plasma volume.
23. At birth, changes in the RBC indices reflect
9. The is the concentration of
the transition to extrauterine life and the
hemoglobin in each cell.
need to transport from the
10. is defined as an abnormally low lungs.
number of circulating RBCs or level of hemo-
24. Jaundice in infants is the result of increased
globin.
red cell breakdown and the inability of the
11. Tissue can give rise to fatigue, immature liver to bilirubin.
weakness, dyspnea, and sometimes angina.
25. The diagnosis of in the elderly
12. anemia is characterized by the requires a complete physical examination, a
premature destruction of red cells, the complete blood count, and studies to rule out
retention in the body of iron and the other comorbid conditions such as malignancy,
products of hemoglobin destruction, and an gastrointestinal conditions that cause
increase in erythropoiesis. bleeding, and pernicious anemia.
13. Two main types of hemoglobinopathies can
cause red cell hemolysis: the abnormal substi-
tution of an amino acid in the hemoglobin
molecule, as in anemia, and the
defective synthesis of one of the polypeptide
chains that form the globin portion of hemo-
globin, as in the .

Copyright 2011. Wolters Kluwer Health | Lippincott Williams & Wilkins. Study Guide for Porths Essentials of Pathophysiology, Third Edition.
LWBK707-c13_p67-72.qxd 8/19/10 1:10 PM Page 69 Aptara Inc

CHAPTER 13 DISORDERS OF RED BLOOD CELLS 69

Activity B Consider the following figure. 9. Jaundice f. Regulator of RBC


production
10. Normochromic
cell g. Normal hemoglobin
Spleen
concentration in
RBC
h. Yellow discoloration
of skin due to high
Hemoglobin levels of bilirubin
i. Transports iron to
plasma
j. Conjugated with
bilirubin to render
it water

Activity D Briefly answer the following.


Liver 1. Hemoglobin is the oxygen-carrying protein
found in RBCs. Describe the molecular struc-
ture of hemoglobin. Also, explain how oxygen
interacts with hemoglobin.

Bone
marrow

2. Red blood cells have a finite life span. How


long is the life span, and what is the fate of
RBCs?
In the above figure, fill in the steps associated
with RBC breakdown and secretion from the
body.
Begin by labeling where heme and the globin
proteins separate. Trace the iron as it is
3. What are the three categories of anemic
recycled or as it is conjugated by the liver.
effects?
Activity C Match the key terms in Column A
with their definitions in Column B.

Column A Column B
1. Thalassemia a. Chronic hemolytic 4. Describe and explain the two consequences of
anemia sickle cell anemia.
2. Severe G6PD
deficiency b. Common cause of
megaloblastic
3. Erythropoietin anemias
4. Mean corpus- c. Measure of size of
cular volume RBC
5. Anemia is a common side effect of cancer
5. Transferrin d. Red blood cell treatments. Which type of anemia usually
production develops and why?
6. Glucuronide
e. Caused by deficient
7. B12 deficiency goblin production
8. Erythropoiesis

Copyright 2011. Wolters Kluwer Health | Lippincott Williams & Wilkins. Study Guide for Porths Essentials of Pathophysiology, Third Edition.
LWBK707-c13_p67-72.qxd 8/19/10 1:10 PM Page 70 Aptara Inc

70 UNIT 3 HEMATOPOIETIC FUNCTION

6. Polycythemia vera is a neoplastic disorder of SECTION IV: PRACTICING


RBCs. Describe the complications that arise
from the rapid increase in hematocrit.
FOR NCLEX
Activity F Answer the following questions.
1. All cells of the body age and are replaced in
a natural order. When RBCs age, they are
destroyed in the spleen. During this process
7. Infantile jaundice is caused by the under- the iron from their hemoglobin is released
developed liver being unable conjugate biliru- into the circulation and returned where?
bin. What are the treatment methods for a. To the bone marrow for incorporation into
infantile jaundice and how do they work? new RBCs.
b. To the liver to bind with oxygen.
c. To the lungs to bind with oxygen.
d. To the muscles to be stored for strength.
2. Bilirubin is the pigment of bile and is made
when RBCs die. There are two types of biliru-
SECTION III: APPLYING bin that can be measured in the blood and
YOUR KNOWLEDGE reported on by the laboratory. What does the
laboratory reports them as?
Activity E Consider the scenario and answer a. Conjugated and unconjugated
the questions. b. Soluble and unsoluble
Mrs. McFee, a 62-year-old woman, is in the c. Positive and negative
outpatient procedure area of the hospital. She has d. Direct and indirect
a long history of rheumatoid arthritis and is to
receive a blood transfusion to treat a chronic 3. Neonatal hyperbilirubinemia is an increased
disease anemia. She appears very nervous as the level of bilirubin in the infants blood. It is
nurse begins the transfusion. She states, My usually a benign condition characterized by
friends have told me that there are serious things what?
that can happen to you because of a transfusion. a. A yellow, jaundiced color
1. The nurse would respond that there are several b. Failure to thrive
side effects that need to be watched for during c. Brain damage
a blood transfusion. Together, the nurse and d. A reddish, ruddy complexion
Mrs. McFee will watch for what types of
symptoms of a transfusion reaction? 4. Anemia resulting from blood loss can be
reversed if the blood loss is not so severe that
it results in death. How long does it take for
the red cell concentration to return to
normal?
a. 8 to 10 days
2. The nurse would also explain to Mrs. McFee b. 3 to 4 weeks
that two people always check the donor blood
c. 10 to 14 days
against the recipient information at least two
times before it is transfused. Once, when it d. 5 to 6 weeks
leaves the laboratory, and, again, before it is
infused into the patient. Why is this attention
given to checking the blood?

Copyright 2011. Wolters Kluwer Health | Lippincott Williams & Wilkins. Study Guide for Porths Essentials of Pathophysiology, Third Edition.
LWBK707-c13_p67-72.qxd 8/19/10 1:10 PM Page 71 Aptara Inc

CHAPTER 13 DISORDERS OF RED BLOOD CELLS 71

5. During chronic blood loss, iron-deficiency 10. When an Rh-negative mother gives birth to an
anemia occurs. Most patients are Rh-positive infant, the mother usually produces
asymptomatic until their hemoglobin falls antibodies that will attack any subsequent
below 8 g/dL. The red cells that the body pregnancies in which the fetus is Rh-positive.
does produce have too little hemoglobin. When subsequent babies are Rh-positive,
What is the term for the resulting anemia? erythroblastosis fetalis occurs. What is another
a. Macrocytic hyperchromic name for erythroblastosis fetalis?
b. Macrocytic hypochromic a. Microcytic disease of the newborn
c. Microcytic hypochromic b. Hemolytic iron-deficiency anemia
d. Microcytic hyperchromic c. Hemolytic disease of the newborn
d. Macrocytic disease of the newborn
6. In hemolytic anemia the RBCs are destroyed
prematurely. What distinguishes almost all 11. Pernicious anemia is thought to be an autoim-
types of hemolytic anemia? mune disease that destroys the gastric mucosa.
a. Normocytic hypochromic cells This results in chronic atrophic gastritis and
the production of antibodies that interfere
b. Microcytic normochromic cells
with binding to intrinsic factor.
c. Macrocytic hyperchromic cells
12. Sickle cell anemia is an inherited disorder
d. Normocytic normochromic cells
seen in African American people. It is marked
7. When hemolytic anemia has intravascular by the characteristic sickling of red blood
hemolysis, it can be characterized in different cells. This causes both chronic hemolytic
ways. Which of the following is not a charac- anemia and occlusion of blood vessels.
terization of hemolytic anemia with intravas- Which are considered to be triggers of an
cular hemolysis? episode of sickling? (Mark all that apply.)
a. Hemoglobinemia a. Infection
b. Jaundice b. Stress
c. Hemosiderinuria c. Heat
d. Spherocytosis d. Dehydration
8. Aplastic anemia is a serious anemia that is a e. Alkalosis
disorder of the pluripotential bone marrow 13. The indices of the RBC are used to differentiate
stem cells and causes all three hematopoietic the anemias by size and color of cell. Match
cell lines to be reduced. What is the the term for a red blood cell with its
treatment for aplastic anemia in the young definition:
and severely affected client?
Term Definition
a. There is no treatment for aplastic anemia.
b. Bone marrow transplant 1. Mean corpuscular a. The concentration
hemoglobin of hemoglobin in
c. Spleen transplant
concentration each cell
d. Liver transplant (MCHC) b. The mass of the red
9. When a client is in chronic renal failure, he 2. Mean cell cell
or she almost always has anemia because of a hemoglobin (MCH) c. The volume or size
deficiency of erythropoietin. What else of the red cells
contributes to the anemia experienced by 3. Mean corpuscular
clients in chronic renal failure? volume (MCV)
a. Uremic toxins and retained nitrogen
b. Bleeding tendencies and lack of fibrinogen
in blood
c. Hemodialysis and decreased nitrogen
d. Hemolysis of RBCs and lack of fibrinogen
in blood

Copyright 2011. Wolters Kluwer Health | Lippincott Williams & Wilkins. Study Guide for Porths Essentials of Pathophysiology, Third Edition.
LWBK707-c13_p67-72.qxd 8/19/10 1:10 PM Page 72 Aptara Inc

72 UNIT 3 HEMATOPOIETIC FUNCTION

14. A pregnant woman at her first prenatal visit 16. Thalassemia can be classed as major or minor.
complains to the nurse that she is always In thalassemia major it is necessary to start
tired. The nurse knows that fatigue is one therapy as early as 6 months of
symptom of anemia. What are other age. If therapy is not started in infants who
symptoms of anemia? (Mark all that apply.) present with this disease, severe growth retar-
a. Faintness dation will occur.
b. Dim vision
c. Ruddy skin
d. Bradycardia
15. Polycythemia vera most often occurs in men
with a median age of 62. It is a neoplastic dis-
ease of the bone marrow that is characterized
by which of the following signs and
symptoms? (Mark all that apply.)
a. Headache
b. Dusky red appearance
c. Ability to concentrate better
d. Cyanosis of trunk
e. Hearing difficulty

Copyright 2011. Wolters Kluwer Health | Lippincott Williams & Wilkins. Study Guide for Porths Essentials of Pathophysiology, Third Edition.
LWBK707-c14_p73-77.qxd 9/2/10 7:34 AM Page 73 Aptara Inc

14

CHAPTER
Mechanisms of
Infectious Disease

SECTION I: LEARNING 10. State the two criteria used in the diagnosis of
an infectious disease.
OBJECTIVES
11. Explain the differences among culture,
1. Define the terms host, infectious disease, serology, and antigen, metabolite, or
colonization, microflora, virulence, pathogen, molecular detection methods for diagnosis of
and saprophyte. infectious disease.
2. Describe the concept of host-microorganism 12. Cite three general intervention methods that
interaction using the concepts of can be used in treatment of infectious
commensalism, mutualism, and parasitic illnesses.
relationships.
13. State four basic mechanisms by which
3. Describe the structural characteristics and antibiotics exert their action.
mechanisms of reproduction for prions,
14. Differentiate bactericidal from bacteriostatic.
viruses, bacteria, fungi, and parasites.
15. List the infectious agents considered to pose
4. Use the concepts of incidence, portal of
the highest level of bioterrorism threat.
entry, source of infection, symptomatology,
disease course, site of infection, agent, and
host characteristics to explain the
mechanisms of infectious diseases. SECTION II: ASSESSING
5. Differentiate between incidence and YOUR UNDERSTANDING
prevalence and among endemic, epidemic,
and pandemic. Activity A Fill in the blanks.
6. Describe the stages of an infectious disease after 1. The colonizing bacteria acquire nutritional
the potential pathogen has entered the body. needs and shelter, the host is not adversely
7. List the systemic manifestations of infectious affected by the relationship; an interaction
disease. such as this is called .

8. Describe mechanisms and significance of 2. The term describes the


antimicrobial and antiviral drug resistance. presence, multiplication, and subsequent
injury within a host by another living
9. Explain the actions of intravenous organism.
immunoglobulin and cytokines in the
treatment of infectious illnesses.

73
LWBK707-c14_p73-77.qxd 9/2/10 7:34 AM Page 74 Aptara Inc

74 UNIT 4 INFECTION AND IMMUNITY

3. A relationship is one in which 17. Inflammation of an anatomic location is usu-


only the infecting organism benefits from the ally designated by adding the suffix -
relationship and the host either gains to the name of the involved
nothing from the relationship or sustains tissue in an infection.
injury from the interaction.
18. The suffix - is used to designate
4. All microorganisms can be the presence of a substance in the blood.
pathogens capable of producing an infectious
19. factors are substances or prod-
disease when the health and immunity of the
ucts generated by infectious agents that
host have been severely weakened.
enhance their ability to cause disease.
5. The various prion-associated diseases produce
20. In contrast to , endotoxins do
very similar symptoms and pathology in the
not contain protein, are not actively released
host and are collectively called
from the bacterium during growth, and have
diseases.
no enzymatic activity.
6. are the smallest obligate intra-
cellular pathogens. Activity B Consider the following figure.
7. Bacteria are autonomously replicating unicel-
lular organisms known as Death
because they lack an organized nucleus. Critical threshold
8. characteristics and microscopic
replication of pathogens

morphology are used in combination to


Severity of illness

describe bacteria. Chronic disease

9. The are an eccentric category of


bacteria that are mentioned separately
because of their unusual cellular morphology Clinical
and distinctive mechanism of motility. threshold
Subclinical disease
10. The are unicellular prokaryotes
capable of independent replication.
11. Serious infections are rare and
usually initiated through puncture wounds or
inhalation.
In the above figure, label the areas that represent
12. The fungi can be separated into two groups,
the course through which a disease progresses:
and , based on
resolution, acute phase, convalescent phase, incu-
rudimentary differences in their morphology.
bation phase, infection, and prodromal phase.
13. Parasitic infection results from the ingestion
of highly resistant cysts or spores that are Activity C Match the key terms in Column A
shed in the of an infected host. with their definitions in Column B.
14. The is the initial appearance of 1.
symptoms in the host.
Column A Column B
15. The period during which the host
1. Microflora a. Describes the act of
experiences the maximum impact of the
establishing an
infectious process corresponding to rapid 2. Host
infection
proliferation and dissemination of the
3. Infection b. Microorganisms
pathogen is known as the .
4. Disease that live with a host
16. The is characterized by the con-
c. Microorganisms so
tainment of infection, progressive 5. Colonization
virulent that they
elimination of the pathogen, repair of
6. Virulence are rarely found in
damaged tissue, and resolution of associated
the absence of
symptoms.
disease

Copyright 2011. Wolters Kluwer Health | Lippincott Williams & Wilkins. Study Guide for Porths Essentials of Pathophysiology, Third Edition.
LWBK707-c14_p73-77.qxd 9/2/10 7:34 AM Page 75 Aptara Inc

CHAPTER 14 MECHANISMS OF INFECTIOUS DISEASE 75

7. Pathogens d. The presence, multi- 9. Facultatively h. Virus capable of


plication, and subse- anaerobic transforming a cell
8. Saprophytes
quent injury within bacteria i. Sexually
9. Prions a host by another transmitted genital
10. Chlamydia
living organism infections
10. Rickettsiae trachomatis
e. Disease-producing j. Cannot live long
potential of a the outside strict
microorganism growth
f. Any organism capa- requirements
ble of supporting
the nutritional and Activity D
physical growth
1. Write the correct sequence of the following
requirements of
in the boxes provided below.
another
a. Viral DNA copy is integrated into the host
g. Condition of an
chromosome
organism that
impairs normal b. Host cell lysis
physiological func- c. Reactivation of virus
tion. d. Entry into the host cell
h. Harmless, free- e. Viral RNA genome is first translated into
living organisms DNA
i. Disease-causing f. Replication of virus
protein particles
that lack any kind
S S S S S
of a demonstrable
genome
j. Organisms combin- Activity E Briefly answer the following.
ing characteristics of 1. Explain the general mechanism of cellular
viral and bacterial viral infection and replication. Differentiate
agents to produce between those that cause lysis and those that
disease in humans do not. Also, explain the concept of a latent
virus.
2.
Column A Column B
1. Plasmids a. Organisms are less
than one-third the
2. Mycoplasmas
size of bacteria 2. Describe the various methods of infiltration
3. Fastidious b. Cause Rocky taken by organisms that will cause infection,
bacteria Mountain fever from the organism entering the host to the
4. Ectoparasites c. Flu viruses manifestation of the disease state.

5. Orthomyxo- d. Herpesvirus and


viridae paramyxoviruses
e. Infest external body
6. Enveloped
surfaces
viruses
f. Bacterial DNA that 3. Explain the concept of disease course and
7. Rickettsiae may increase list all the stages that the disease course takes.
8. Oncogenic virulence
viruses g. Bacteria that can
adapt metabolism

Copyright 2011. Wolters Kluwer Health | Lippincott Williams & Wilkins. Study Guide for Porths Essentials of Pathophysiology, Third Edition.
LWBK707-c14_p73-77.qxd 9/2/10 7:34 AM Page 76 Aptara Inc

76 UNIT 4 INFECTION AND IMMUNITY

4. What is the goal of treatment in regard to SECTION IV: PRACTICING


infective organisms? Provide the common
methods of treatment.
FOR NCLEX
Activity G Answer the following questions.
1. What is the term for parasitic relationships
between microorganisms and the human
body in which the human body is harmed?
5. Explain the categorization of organisms that a. Infectious disease
carry the potential for bioterrorism.
b. Mutual disease
c. Communicable disease
d. Commensal disease
2. The infectious agents that cause Rocky
Mountain spotted fever and epidemic typhus
are transmitted to the human body via vector
such as a tick. What are these infectious
SECTION III: APPLYING agents?
YOUR KNOWLEDGE a. Viruses
b. Rickettsiae
Activity F Consider the scenario and answer
the questions. c. Chlamydiae
d. Ehrlichiae
You are a nurse working for a public health
agency. You have been asked to give a talk to 3. Severe acute respiratory syndrome (SARS), a
the local Rotary Club about infectious diseases. highly transmissible respiratory infection,
In your presentation you are going to include crossed international borders in the winter of
information about treatment of these 2002. What terms are used to describe the
diseases. outbreak of SARS?
1. In discussing the role of antibiotics in the a. Pandemic and nosocomial
treatment of infectious diseases, you would b. Regional and endemic
include definitions of the terms bactericidal c. Epidemic and pandemic
and bacteriostatic. What are these
d. Nosocomial and endemic
definitions?
4. The clinical picture, or presentation of a
disease in the body, is called what?
a. Virulence of the disease
b. Source of the disease
2. What drugs are used for HIV infections? How c. Diagnosis of the disease
are these drugs classified? d. Symptomatology of the disease
5. There are two criteria that have to be met in
order for a diagnosis of an infectious disease
to occur. What are these two criteria?
a. Recovery of probable pathogen and
documentation of signs and symptoms
compatible with an infectious process.
b. Propagation of a microorganism outside
the body and testing to see what destroys it.
c. Identification by microscopic appearance
and Gram stain reaction
d. Serology and an antibody titer specific to
the serology

Copyright 2011. Wolters Kluwer Health | Lippincott Williams & Wilkins. Study Guide for Porths Essentials of Pathophysiology, Third Edition.
LWBK707-c14_p73-77.qxd 9/2/10 7:34 AM Page 77 Aptara Inc

CHAPTER 14 MECHANISMS OF INFECTIOUS DISEASE 77

6. Levels A, B, and C are levels assigned to 11. Transmissible neurodegenerative diseases


potential agents of bioterrorism. What are such as Creutzfeldt-Jakob disease are
these categorical assignments based on? associated with .
a. Safety to terrorist 12. infections refer to vertically
b. Transmissibility transmitted infections, infections that are
c. Environmental impact transmitted from mother to infant.
d. Ease of use to terrorist 13. Match the category of infectious diseases
with its source.
7. Global infectious diseases are now being
recognized. These diseases, known as Category Source
endemic to one part of the world, are now 1. Zoonoses a. Passed from mother
being found in other parts of the world to child at birth
because of international travel and a global 2. Perinatal
infections b. Health care facility
marketplace. Which of the following is
considered a global infectious disease? c. Passed from animals
3. Opportunistic
to humans
a. Coxsackie disease
4. Nosocomial d. Acquired from
b. Respiratory syncytial disease
clients own body
c. West Nile virus
14. Infectious agents produce products or
d. Hand, foot, and mouth disease
substances called virulence factors that make
8. Which of the following sequences accurately it easier for them to cause disease. Which of
describes the stages of a disease? these are virulence factors? (Mark all that
a. Incubation, prodromal, current, recovery, apply.)
and resolution a. Invasive factors
b. Subacute, prodromal, acute, postacute, b. Prodromal factors
and convalescent c. Adhesion factors
c. Prodromal, subacute, acute, postdromal, d. Toxins
and resolution
e. Evasive factors
d. Incubation, prodromal, acute, convales-
cent, and resolution. 15. Evasive factors, one type of virulence factor,
are factors produced by infectious
9. Sometimes the hosts white blood cells are microorganisms to keep the hosts immune
unable to eliminate the microorganism, but system from destroying the microorganism.
the body is able to contain the dissemination Which of these are evasive factors? (Mark all
of the pathogen. What is this called? that apply.)
a. Abscess a. Capsules
b. Pimple b. Phospholipases
c. Lesion c. Collagenases
d. Acne d. Slime
10. Escherichia coli (E. coli) produces an exotoxin e. Mucous layers
called Shiga toxin that enters the body when
you eat undercooked hamburger meat and
fruit juices that are not pasteurized. What can
E. coli infection cause?
a. Nephritic syndrome
b. Hemorrhagic colitis
c. Hemolytic thrombocytopenia
d. Neuroleptic malignant syndrome

Copyright 2011. Wolters Kluwer Health | Lippincott Williams & Wilkins. Study Guide for Porths Essentials of Pathophysiology, Third Edition.
LWBK707-c15_p78-83.qxd 9/2/10 7:35 AM Page 78 Aptara Inc

15
CHAPTER
Innate and Adaptive
Immunity

SECTION I: LEARNING 13. Characterize the significance and function of


major histocompatibility complex (MHC)
OBJECTIVES molecules.
1. Discuss the function of the immune system. 14. Describe the antigen-presenting functions of
macrophages and dendritic cells.
2. Contrast and compare the general properties
of innate and adaptive immunity. 15. Contrast and compare the development and
function of the T and B lymphocytes.
3. Describe the cells of the immune system.
16. State the function of the five classes of
4. Characterize the chemical mediators that
immunoglobulins.
orchestrate the immune response.
17. Differentiate between the central and periph-
5. Characterize the function of the innate
eral lymphoid structures.
immune system.
18. Describe the function of cytokines involved
6. Describe components of the innate immune
in the adaptive immune response.
system including epithelial barriers, soluble
chemical agents, and cellular components. 19. Compare passive and active immunity.
7. Describe the recognition systems for 20. Explain the transfer of passive immunity
pathogens in innate immunity. from mother to fetus and from mother to
infant during breast-feeding.
8. State the types and functions of leukocytes
that participate in innate immunity. 21. Characterize the development of active
immunity in the infant and small child.
9. Describe the functions of the various
cytokines involved in innate immunity. 22. Describe changes in the immune response
that occur with aging.
10. Define the role of the complement system in
immunity and inflammation.
11. State the properties associated with adaptive
immunity.
12. Define and describe the characteristics of an
antigen.

78
LWBK707-c15_p78-83.qxd 9/2/10 7:35 AM Page 79 Aptara Inc

CHAPTER 15 INNATE AND ADAPTIVE IMMUNITY 79

SECTION II: ASSESSING 14. cells are specialized, bone


marrow-derived leukocytes found in lymphoid
YOUR UNDERSTANDING tissue that are important intermediaries
between the innate and adaptive immune
Activity A Fill in the blanks.
systems.
1. The has evolved in multicellu-
15. are cytokines that stimulate the
lar organisms to defend against bacteria,
migration and activation of immune and
viruses, and other foreign substances.
inflammatory cells.
2. Although the immune response normally is
16. Cytokines that stimulate bone marrow
protective, it also can produce undesirable
pluripotent stem and progenitor or precursor
effects such as when the response is
cells to produce large numbers of platelets,
excessive, as in , or when it
erythrocytes, lymphocytes, neutrophils,
recognizes self-tissue as foreign, as in
monocytes, eosinophils, basophils, and
disease.
dendritic cells are known as .
3. As the first line of defense,
17. The mucous membrane linings of the
immunity consists of the physical, chemical,
gastrointestinal, respiratory, and urogenital
molecular, and cellular defenses.
tracts are protected by sheets of tightly
4. immunity is the second major packed cells that block the
immune defense. entry of microbes.
5. Substances that elicit adaptive immune 18. The binding of to the pattern
responses are called . recognition receptors on leukocytes initiates
the signaling events that lead to innate
6. immunity, generated by B lym-
immunity and tissue changes associated with
phocytes, is mediated by molecules called
acute inflammation.
antibodies and is the principal defense
against extracellular microbes and toxins. 19. is the coating of a microorgan-
ism with soluble molecules that tag the
7. immunity is mediated by
microorganism for more efficient recognition
specific T lymphocytes and defends against
by phagocytes.
intracellular microbes such as viruses.
20. are substances foreign to the
8. Dendritic cells and function as
host that can stimulate an immune response.
antigen-presenting cells for adaptive
immunity. 21. Antibodies comprise a class of proteins called
.
9. The key cells of innate immunity are
, , and natural 22. immunity depends on matura-
killer cells. tion of B lymphocytes into plasma cells,
which produce and secrete antibodies.
10. are the early responding cells of
innate immunity. 23. The serves as a master regulator
for the immune system.
11. During an inflammation response, the mono-
cyte leaves the blood vessel, transforms into a 24. T cells suppress immune
tissue , and phagocytoses bacte- responses by inhibiting the proliferation of
ria, damaged cells, and tissue debris. other potentially harmful self-reactive
lymphocytes.
12. cells and cells are
the only cells in the body capable of speci- 25. The central lymphoid organs, the
fically recognizing different antigenic and the , provide the enviro-
determinants of microbial agents and other nment for immune cell production and
pathogens. maturation.
13. are part of the innate immune 26. The white pulp layer of the
system, and may be the first line of defense contains concentrated areas of B and T
against viral infections. lymphocytes permeated by macrophages and
dendritic cells.

Copyright 2011. Wolters Kluwer Health | Lippincott Williams & Wilkins. Study Guide for Porths Essentials of Pathophysiology, Third Edition.
LWBK707-c15_p78-83.qxd 9/2/10 7:35 AM Page 80 Aptara Inc

80 UNIT 4 INFECTION AND IMMUNITY

Activity B Consider the following figure. f. The physical barrier


of skin to infection
Antigen g. Disrupts virus
infections
h. Small cationic
Variable region peptides found in
(heavy chain) the stomach
Fab
i. Regulates the
Constant region production of
(heavy chain)
cytokines and
adhesion molecules
j. Hydrolytic enzyme
Variable capable of cleaving
region the walls of bacterial
(light chain)
cell
Constant
region
(light chain)
2.
Column A Column B
Fc 1. Epitopes a. Processing a
 complex antigen
2. CD4
Heavy into epitopes and
chain 3. Perforins then displaying the
foreign and self
4. Cell-mediated
peptides on their
immunity
membranes
1. What does this figure depict? Discuss the 5. Antigen b. Dependent on B
significance of the different parts depicted in presentation cells
this model.
6. Antibody- c. Self-recognition
Activity C Match the key terms in Column A
mediated proteins
with their definitions in Column B immunity d. Type of T helper cell
7. Major e. Dependent on T
1.
histocompati- cells
Column A Column B bility f. Pore-forming
1. Mucins a. Pathogen-associated complex molecules
molecular pattern 8. Haptens g. Immunologically
2. Lysozyme receptors
+ active sites on
b. Renders bacteria 9. CD8
3. Epithelial antigens
barrier and other cells sus- 10. Tolerance h. Combine with
ceptible to phagocy- larger protein
4. Defensins
tosis molecules and serve
5. Collectins c. Traps and washes as antigens
6. Cilia away potential i. Ability of the
invaders immune system to
7. Toll-like
d. Epithelial be nonreactive to
receptors
protrusion that self-antigens
8. Opsonins moves mucus to j. Cytotoxic T cells
throat
9. NF-
e. Surfactant proteins
10. Interferons in respiratory track

Copyright 2011. Wolters Kluwer Health | Lippincott Williams & Wilkins. Study Guide for Porths Essentials of Pathophysiology, Third Edition.
LWBK707-c15_p78-83.qxd 9/2/10 7:35 AM Page 81 Aptara Inc

CHAPTER 15 INNATE AND ADAPTIVE IMMUNITY 81

Activity D Briefly answer the following. 8. Compare and contrast active versus passive
immunity.
1. How do the cells of the immune system com-
municate with each other?

2. What is the innate immune system and what SECTION III: APPLYING
is its function?
YOUR KNOWLEDGE
Activity E Consider the scenario and answer
the question.
A young new mother has her 2-week old infant
3. What is the general function of neutrophils at the clinic for a well-baby check-up. She is con-
and macrophages in the inflammatory cerned because her baby has been exposed to
response? chickenpox. She states, What am I going to do?
I didnt know my friends son had just gotten
over the chickenpox. Will my baby get chicken-
pox?
1. In talking with this mother, the nurse explains
4. What are the methods of initiating the passive immunity. What key points will the
complement system and what are the results nurse be sure to mention?
of activation?

5. What is the function of MHC proteins, and SECTION IV: PRACTICING


how are they classified? FOR NCLEX
Activity F Answer the following questions.
1. Natural killer cells are specialized lymph-
ocytes that are one of the major parts of
6. Explain how a macrophage participates in which immunity?
antigen presentation. a. Innate
b. Adaptive
c. Humoral
d. Cell-mediated
2. Both innate and adaptive immunity have
7. How many classes of antibody are there? Give cells that produce cytokines. Cytokines medi-
a brief definition of function for each one. ate the actions of many cells in both innate
and adaptive immunity. How are the actions
of cytokines described?
a. Rapid and self-limiting
b. Pleiotropic and redundant
c. Cell-specific and targeted
d. Dendritic and morphologic

Copyright 2011. Wolters Kluwer Health | Lippincott Williams & Wilkins. Study Guide for Porths Essentials of Pathophysiology, Third Edition.
LWBK707-c15_p78-83.qxd 9/2/10 7:35 AM Page 82 Aptara Inc

82 UNIT 4 INFECTION AND IMMUNITY

3. Stem cells in the bone marrow produce T 8. The laboratory finds IgA in a sample of cord
lymphocytes or T cells, and release them into blood from a newborn infant. This finding is
the vascular system. The T cells then migrate important because it signifies what?
where to mature? a. Fetal reaction to an infection acquired at
a. Spleen birth
b. Liver b. Maternal reaction to an infection in the
c. Thymus fetus
d. Pancreas c. Maternal exposure to an infection in a
sexual partner
4. Cell-mediated immunity is involved in resis-
d. Fetal reaction to exposure to an
tance to infectious diseases caused by bacteria
intrauterine infection
and some viruses. It is also involved in cell-
mediated hypersensitivity reactions. Which 9. The daughter of a 79-year-old woman asks
of these does not cause a cell-mediated the nurse why her mother gets so many
hypersensitivity reaction? infections. The daughter states, My
a. Latex mother has always been healthy, but now
she has pneumonia. Last month she got
b. Poison ivy
cellulitis from a bug bite she scratched. The
c. X-ray dye month before that was some other
d. Blood transfusion infection. How come she seems to get sick
so often now? What is the nurses best
5. Passive immunity is immunity that is
response?
transferred from another source and lasts
only weeks to months. What is an example a. As people get older their immune system
of passive immunity? does not respond as well as it did when
they were younger.
a. An injection of -globulin
b. About the time we are 75 or 76 years old
b. An immunization
our immune system quits working.
c. Exposure to poison ivy
c. Your mother just seems to be prone to
d. Allergy shots getting infections.
6. An essential property of the immune system d. Your mother gets infections frequently
is self-regulation. An immune response that because she wants attention from you.
is not adequate can lead to immuno-
10. The results of recent research suggest that a
deficiency, while an immune response that is
key role in the origin of some diseases is
excessive can lead to conditions from allergic
played by inflammation. Which of these dis-
responses all the way to autoimmune
eases is it thought that inflammation has a
diseases. Which of these is not an example of
role in its beginnings?
a breakdown of the self-regulation of the
immune system? a. Osteoporosis
a. Multiple sclerosis b. Rheumatoid arthritis
b. Huntington disease c. Osteogenesis imperfecta
c. Systemic lupus d. Hydronephrosis
d. Fibromyalgia 11. , or immunogens, are substances
foreign to the host that can stimulate an
7. One of the self-regulatory actions of the
immune response.
immune system is to identify self-antigens
and be nonreactive to them. What is this
ability of the immune system defined as?
a. Antigen specificity
b. Nonre activity
c. Tolerance
d. Antigen diversity

Copyright 2011. Wolters Kluwer Health | Lippincott Williams & Wilkins. Study Guide for Porths Essentials of Pathophysiology, Third Edition.
LWBK707-c15_p78-83.qxd 9/2/10 7:35 AM Page 83 Aptara Inc

CHAPTER 15 INNATE AND ADAPTIVE IMMUNITY 83

12. Each immunoglobulin has a different role 13. The mucous membrane linings of the
in the immune response. Match each gastrointestinal, respiratory, and urogenital
immunoglobulin with its role. tracts are protected by sheets of tightly
packed cells that block the
Immunoglobulin Role
entry of microbes and destroy them by secret-
1. IgG a. Is the first circulating ing antimicrobial enzymes, proteins, and
immunoglobulin to peptides.
2. IgA
appear in response to an
14. In both the innate and the adaptive immune
3. IgM antigen and is the first
systems, cells communicate information
antibody type made
4. IgD about invading organisms by the secretion of
by a newborn
chemical mediators. Which are these media-
5. IgE b. Involved in inflammation, tors? (Mark all that apply.)
allergic responses, and
a. Virulence factors
combating parasitic
infections b. Chemokines
c. Serves as an antigen c. Colony-stimulating factors
receptor for initiating d. Coxiellas
the differentiation of
15. There are many cells that make up the
B cells
passive and adaptive immune systems.
d. Protects against bacteria, Which cells are responsible for the specificity
toxins, and viruses in and memory of adaptive immunity? (Mark all
body fluids and activates that apply.)
the complement system
a. Phagocytes
e. A primary defense against
b. T lymphocytes
local infections in mucosal
tissues c. Dendritic cells
d. Natural killer cells
e. B lymphocytes

Copyright 2011. Wolters Kluwer Health | Lippincott Williams & Wilkins. Study Guide for Porths Essentials of Pathophysiology, Third Edition.
LWBK707-c16_p84-89.qxd 9/2/10 7:38 AM Page 84 Aptara Inc

16
CHAPTER
Disorders of the
Immune Response

SECTION I: LEARNING 10. Describe the mechanisms and manifestations


of graft-versus-host disease.
OBJECTIVES
11. Name four or more diseases attributed to
1. State the difference in causes of primary and autoimmunity.
secondary immunodeficiency disorders.
12. Describe three or more postulated
2. Compare and contrast pathology and mani- mechanisms underlying autoimmune
festations of humoral (B-cell), cellular disease.
(T-cell), and combined T- and B-cell
13. State the criteria for establishing an autoim-
immunodeficiency disorders.
mune basis for a disease.
3. Differentiate between adaptive immune
responses that protect against microbial
agents and hypersensitivity responses.
SECTION II: ASSESSING
4. Describe the immune mechanisms involved
in a type I, type II, type III, and type IV
YOUR UNDERSTANDING
hypersensitivity reaction.
Activity A Fill in the blanks.
5. Describe the pathogenesis of allergic rhinitis,
1. Under normal conditions, the
food allergy, serum sickness, Arthus reaction,
response deters or prevents disease.
contact dermatitis, and hypersensitivity
pneumonitis. 2. can be defined as an abnormal-
ity in the immune system that renders a per-
6. Characterize the differences in a type I,
son susceptible to diseases normally
immunoglobulin E (IgE)-mediated hypersen-
prevented by an intact immune system.
sitivity response and that caused by a type IV,
cell-mediated response. 3. The immune system is
composed of the phagocytic leukocytes, natu-
7. Relate the mechanisms of self-tolerance to
ral killer (NK) cells, and complement
the possible explanations for development of
proteins.
autoimmune disease.
4. The immune response is
8. Discuss the rationale for matching of human
composed mainly of T and B cells and
leukocyte antigen or major histocompatibil-
responds to infections more slowly, but more
ity complex types in organ transplantation.
specifically, than the innate immune system.
9. Compare the immune mechanisms involved
in allogeneic transplant rejection.

84
LWBK707-c16_p84-89.qxd 9/2/10 7:38 AM Page 85 Aptara Inc

CHAPTER 16 DISORDERS OF THE IMMUNE RESPONSE 85

5. The adaptive immune system is further 18. Chronic cirrhosis of the liver would reduce
divided into the and the production of complement proteins; this
immune systems. type of deficiency would be classified as
.
6. A large number of primary immunodeficiency
diseases have been mapped to the 19. Chdiak-Higashi syndrome is an abnormality
chromosome. of of phagocytes.
7. Defects in humoral immunity increase the 20. Chronic granulomatous disease is a group of
risk of recurrent infections. inherited disorders that greatly reduce or
inactivate the ability of phagocytic cells to
8. During the first few months of life, infants
produce the .
are protected from infection by IgG antibod-
ies that originate in circulation 21. disorders refer to excessive or
during fetal life. inappropriate activation of the immune
system.
9. Of all the primary immunodeficiency
diseases, those affecting 22. Type I hypersensitivity reactions to antigens
production are the most frequent. are referred to as .
10. Abnormal immunoglobulin loss can occur 23. is a systemic life-threatening
with chronic disease; because of hypersensitivity reaction characterized by
abnormal glomerular filtration, patients lose widespread edema, vascular shock secondary
serum IgA and IgG in their urine. to vasodilation, and difficulty breathing.
11. Secondary humoral immunodeficiencies can 24. Persons with allergic conditions
also result from a number of , tend to have high serum levels of IgE and
including chronic lymphocytic leukemia, increased numbers of basophils and mast
lymphoma, and multiple myeloma that inter- cells.
fere with normal immunoglobulin
25. Allergic is characterized by
production.
symptoms of sneezing, itching, and watery
12. T cells can be functionally divided into two discharge from the nose and eyes.
subtypes: and
26. There are three different types of antibody-
T cells.
mediated mechanisms involved in
13. Collectively, protect against reactions: opsonization and
fungal, protozoan, viral, and intracellular complement- and antibody receptor-
bacterial infections; control malignant cell mediated phagocytosis, complement- and
proliferation; and are responsible for coordi- antibody receptor-mediated inflammation,
nating the overall immune response. and antibody-mediated cellular dysfunction.
14. Disorders that affect both B and T lymp- 27. mediated destruction of cells
hocytes, with resultant defects in both that are coated with low levels of IgG
humoral and cell-mediated immunity, fall antibody and are killed by a variety of
under the broad classification of effector cells, which bind to their target by
syndrome. their receptors for IgG, and cell lysis occurs
without phagocytosis.
15. In , genetic mutations lead to
absence of all T and B cell function and, 28. hypersensitivity reactions are
in some cases, a lack of NK cells. responsible for the vasculitis seen in certain
autoimmune diseases such as systemic lupus
16. SCID is more commonly found in
erythematosus (SLE), or the kidney damage
, as it is X linked.
seen with acute glomerulonephritis.
17. Hereditary angioneurotic edema is a form of
deficiency.

Copyright 2011. Wolters Kluwer Health | Lippincott Williams & Wilkins. Study Guide for Porths Essentials of Pathophysiology, Third Edition.
LWBK707-c16_p84-89.qxd 9/2/10 7:38 AM Page 86 Aptara Inc

86 UNIT 4 INFECTION AND IMMUNITY

29. sickness is a systemic immune Activity B Consider the following figure.


complex disorder that is triggered by the
deposition of insoluble antigen-antibody
complexes in blood vessels, joints, heart, and Bone marrow
kidney tissue.
30. A term used by pathologists and immunolo-
gists to describe localized tissue necrosis
caused by immune complexes is the Pre-T cells
.
31. Hypersensitivity reactions that are mediated
by specifically sensitized T lymphocytes are Thymus

divided into two basic types: direct cell-


mediated cytotoxicity and delayed-type
hypersensitivity, and generally classified as
.
32. Allergic denotes an inflam-
matory response confined to the skin that is
initiated by re-exposure to an allergen to
which a person had previously become sensi-
tized.
33. A major barrier to is the process A
of rejection in which the recipients immune
system recognizes the graft as foreign and B
attacks it.
34. Transplanted tissue can be categorized as an C
graft if donor and recipient are
In the above figure, label and diagram the
the same person, graft if the
process of T-cell selection. Be sure to include the
donor and recipient are identical twins, and
end result of each pathway.
if the donor and recipient are
related or unrelated but share similar HLA
Activity C Match the key terms in Column A
types.
with their definitions in Column B.
35. occurs when immunologically
Column A Column B
competent cells or precursors are
transplanted into recipients who are 1. DiGeorge a. Essentially
immunologically compromised. syndrome undetectable levels
of all serum
36. diseases represent a group of 2. Secondary
immunoglobulins
disorders that are caused by a breakdown in immunodefi-
the ability of the immune system to differen- ciency b. Complement-
tiate between self- and nonself-antigens. 3. Hyper-IgM mediated immune
syndrome disorders
37. The ability of the immune system to
c. Decreases in one or
differentiate self from nonself is called 4. X-linked
more of IgG
. agammaglobu-
subgroups
linemia
38. Loss of self-tolerance with development of
is characteristic of a number of
autoimmune disorders.

Copyright 2011. Wolters Kluwer Health | Lippincott Williams & Wilkins. Study Guide for Porths Essentials of Pathophysiology, Third Edition.
LWBK707-c16_p84-89.qxd 9/2/10 7:38 AM Page 87 Aptara Inc

CHAPTER 16 DISORDERS OF THE IMMUNE RESPONSE 87

5. Selective d. Repeated bouts of Activity D


IgA deficiency upper respiratory
and middle ear Put the normal sequence of actions of a polymor-
6. Adenosine phonuclear phagocyte in order in the boxes below.
infections
deaminase
e. Partial or complete a. Phagocytosis
deficiencies
and T-cell failure of develop- b. Kill the ingested pathogens
cytokine ment of the c. Chemotaxis
receptor thymus and
d. Generate microbicidal substances
mutations parathyroid glands
e. Adherence
f. In levels of serum
7. Transient
and secretory IgA
hypogam
maglobuli- g. Antibody-mediated S S S S
nemia disorders
of infancy h. Acquired later in
life Activity E Briefly answer the following.
8. Ataxia-
telangiectasia i. The terminal 1. What is the difference between a primary and
differentiation of a secondary immunodeficiency?
9. Common mature B cells to
variable plasma cells is
immunode- blocked
ficiency
j. Ig-E mediated
10. Immunoglo- disorders
bulin 2. Why does it take up to 6 months for the
k. Lymphopenia and
G subclass symptoms of a primary immunodeficiency to
a decrease in the
show up?
deficiency ratio of CD4
11. Wiskott- helper T cells to
Aldrich CD8 suppressor T
syndrome cells
l. Low IgG and IgA
12. Type I
levels, high IgM 3. Explain how a patient can become sensitized
hypersensitivity
concentrations to an allergen (antigen) in a type I hypersensi-
reaction
m. Cause of SCID tivity reaction.
13. Type II
n. Susceptible to
hypersensitivity
infections caused
reaction
by encapsulated
14. Type III microorganisms
hypersensitivity o. T-cell-mediated
reaction 4. Compare the direct cell-mediated cytotoxicity
disorders
of type IV hypersensitivity reactions with the
15. Type IV delayed-type hypersensitivity reactions.
hypersensi-
tivity
reaction

Copyright 2011. Wolters Kluwer Health | Lippincott Williams & Wilkins. Study Guide for Porths Essentials of Pathophysiology, Third Edition.
LWBK707-c16_p84-89.qxd 9/2/10 7:38 AM Page 88 Aptara Inc

88 UNIT 4 INFECTION AND IMMUNITY

5. What is SCID? 2. Drug-induced secondary


hypogammaglobulinemia is considered
reversible. Which drugs produce hypogam-
maglobulinemia? (Mark all that apply.)
a. Phenytoin
b. Corticosteroids
c. Carbamazepine
SECTION III: APPLYING d. Disease-modifying antirheumatic drugs
YOUR KNOWLEDGE e. Interferon beta-1a drugs
3. Primary cell-mediated disorders of the
Activity F Consider the scenario and answer
immune system cause severe problems with
the questions.
infections. Children with these disorders
A 30-year-old woman has just been diagnosed rarely survive beyond childhood without a
with SLE. She presents with arthritis, a butterfly bone marrow transplant. Which of the follow-
rash, weight loss, weakness, and fatigue. She is ing is a disease that involves primary cell-
distraught and she states, How can the doctor mediated disorders of the immune system?
be sure that I have this disease? a. DiGeorge syndrome
1. The correct response to this patient about the b. Y-linked hyper-IgM syndrome
diagnosis would include information about c. X-linked agammaglobulinemia
which test?
d. Y-linked agammaglobulinemia
4. Combined immunodeficiency syndrome is a
disorder in which both B and T lymphocytes
are affected. This results in defects in both
humoral and cell-mediated immunity. What
2. When planning patient education for this could be the cause of this disorder?
woman, what medications would the nurse a. Multiple misplaced genes that influence
tell the patient about? lymphocyte development and response
b. A single mutation in any gene that
influences major histocompatibility antigens
c. A single misplaced gene that influences
major histocompatibility
d. Multiple mutations in genes that influence
SECTION IV: PRACTICING lymphocyte development and response
FOR NCLEX
Activity G Answer the following questions.
1. Infants are born with a passive immunity
that occurs when immunoglobulin antibodies
cross the placenta from the maternal circula-
tion prior to birth. Which immunoglobulin is
capable of crossing the placenta?
a. IgM
b. IgD
c. IgG
d. IgE

Copyright 2011. Wolters Kluwer Health | Lippincott Williams & Wilkins. Study Guide for Porths Essentials of Pathophysiology, Third Edition.
LWBK707-c16_p84-89.qxd 9/2/10 7:38 AM Page 89 Aptara Inc

CHAPTER 16 DISORDERS OF THE IMMUNE RESPONSE 89

5. Combined immunodeficiency (CID) is 9. The incidence of latex allergy is skyrocketing


distinguished by low, not absent, T-cell because of diseases such as HIV. It is known
function. These diseases are usually that the use of latex examining gloves has
associated with other disorders and arise from played a major role in the increasing
diverse genetic causes. Which of the incidence of latex allergy. What plays a
following diseases is considered a CID? significant role in the allergic response to
a. Pierre-Robin syndrome latex gloves?
b. Angelman syndrome a. Baking powder used inside the gloves
c. Ataxia-telangiectasia b. Airborne pieces of latex
d. Adair-Dighton syndrome c. Latex proteins that attach to clothing
d. Cornstarch powder used inside the gloves
6. The immune system typically responds to
invaders of all types in our body. However, it 10. A transplant reaction that occurs imme-
can also cause tissue injury and disease. What diately after transplantation is caused by
is this effect called? antibodies that are present.
a. Hypersensitivity action 11. It has been postulated that an autoimmune
b. Antigen reaction disease needs a trigger event for it to
c. Mediator response action clinically manifest itself in a body. What are
these trigger events thought to be? (Mark
d. Allergen stimulating reaction
all that apply.)
7. Some people are so sensitive to certain a. A. microorganism or virus
antigens that they react within minutes by
b. A self-antigen from a previously
developing itching, hives, and skin erythema,
sequestered body tissue
followed shortly thereafter by bronchospasm
and respiratory distress. What is this c. A breakdown in the antigen-antibody
commonly known as? response
a. Antigen reaction d. A chemical substance
b. Anaphylactic reaction e. A systemic ability for self-tolerance
c. Hyposensitive reaction
d. Arthus reaction
8. A systemic immune complex disorder that is
caused by insoluble antigen-antibody
complexes being deposited in blood vessels,
the joints, the heart, or kidney tissue is called
what?
a. Anti-immune disease
b. Systemic lupus erythematosus
c. Serum sickness
d. Antigen-antibody sickness

Copyright 2011. Wolters Kluwer Health | Lippincott Williams & Wilkins. Study Guide for Porths Essentials of Pathophysiology, Third Edition.
LWBK707-c17_p90-95.qxd 9/2/10 7:41 AM Page 90 Aptara Inc

17
CHAPTER
Control of
Cardiovascular Function

SECTION I: LEARNING 9. Describe the cardiac reserve and relate it to


the Frank-Starling mechanism.
OBJECTIVES
10. Compare the structure and function of
1. Compare the function and distribution of arteries and veins.
blood flow and blood pressure in the
11. Describe the structure and function of
systemic and pulmonary circulations.
vascular smooth muscle.
2. State the relation between blood volume and
12. Define autoregulation and characterize
blood pressure in arteries, veins, and
mechanisms responsible for short-term and
capillaries of the circulatory system.
long-term regulation of blood flow.
3. Define the term hemodynamics and describe
13. Describe mechanisms involved in the
the effects of blood pressure, vessel radius,
humoral control of blood flow.
vessel length, vessel cross-sectional area, and
blood viscosity on blood flow. 14. Define the term microcirculation.
4. Use the law of Laplace to explain the effect of 15. Describe the structure and function of the
radius size on the pressure and wall tension capillaries.
in a vessel.
16. Explain the forces that control the fluid
5. Use the term compliance to describe the char- exchange between the capillaries and the
acteristics of arterial and venous blood interstitial spaces.
vessels.
17. Describe the structures of the lymphatic
6. Describe the structural components and system and relate them to the role of the
function of the pericardium, myocardium, lymphatics in controlling interstitial fluid
endocardium, and the heart valves and volume.
fibrous skeleton.
18. Describe the roles of the medullary
7. Draw a figure of the cardiac cycle, incorporat- vasomotor and cardioinhibitory centers in
ing the volume, pressure, heart sounds, and controlling the function of the heart and
electrocardiographic changes that occur dur- blood vessels.
ing atrial and ventricular systole and diastole.
8. Define the terms preload and afterload.

90
LWBK707-c17_p90-95.qxd 9/2/10 7:41 AM Page 91 Aptara Inc

CHAPTER 17 CONTROL OF CARDIOVASCULAR FUNCTION 91

19. Relate the performance of baroreceptors and 8. Blood flow in the circulatory system depends
chemoreceptors in the control of on a blood that is sufficient to
cardiovascular function. fill the blood vessels and a dif-
ference across the system that provides the
20. Describe the distribution of sympathetic and
force to move blood forward.
parasympathetic nervous system in the
innervation of the circulatory system and 9. The term refers to the
their effects on heart rate and cardiac principles that govern blood flow in the
contractility. circulatory system.
21. Relate the role of the central nervous system 10. Because flow is directly related to the radius,
in terms of regulating circulatory function. small changes in vessel radius can produce
changes in flow to an organ or
tissue.
11. is the resistance to flow caused
SECTION II: ASSESSING by the friction of molecules in a fluid.
YOUR UNDERSTANDING
12. blood flow may predispose to
Activity A Fill in the blanks. clot formation as platelets and other coagula-
tion factors are exposed to the endothelial
1. The circulatory system delivers lining of the vessel.
and nutrients needed for metabolic processes
to the tissues, carries products 13. Wall tension is inversely related to wall thick-
from the tissues to the kidneys and other ness, such that the the vessel
excretory organs for elimination, and wall, the lower the tension.
circulates electrolytes and 14. The total quantity of blood that can be stored
needed to regulate body function. in a given portion of the circulation for each
2. The circulatory system can be divided into millimeter rise in pressure is termed
two parts: the circulation and compliance, and reflects the of
the circulation. the blood vessel.

3. The circulation consists of the 15. The and valves


right heart, the pulmonary artery, the control the movement of blood out of the
pulmonary capillaries, and the pulmonary ventricles.
veins. 16. The electrical activity, recorded on the
4. The circulation consists of the electrocardiogram, the mechan-
left heart, the aorta and its branches, the cap- ical events of the cardiac cycle.
illaries that supply the brain and peripheral 17. The aorta is highly and as such
tissues, and the systemic venous system and stretches during systole to accommodate the
the vena cava. blood that is being ejected from the left heart
5. The pressure of the pulmonary during systole.
circulation allows blood to move through the 18. is marked by ventricular relax-
lungs more slowly, which is important for gas ation and filling.
exchange.
19. The difference between the end-diastolic and
6. The function as collection end-systolic volumes (approximately 70 mL)
chambers for blood and the are is called the .
the main pumping chambers of the heart.
20. The stroke volume divided by the end-
7. Because it is a closed system, the effective diastolic volume is the fraction.
function of the circulatory system requires
that the outputs of both sides of the heart 21. The efficiency of the heart as a pump often is
pump the amount of blood measured in terms of the or the
over time. amount of blood the heart pumps each minute.

Copyright 2011. Wolters Kluwer Health | Lippincott Williams & Wilkins. Study Guide for Porths Essentials of Pathophysiology, Third Edition.
LWBK707-c17_p90-95.qxd 9/2/10 7:41 AM Page 92 Aptara Inc

92 UNIT 5 CIRCULATORY FUNCTION

22. The refers to the maximum per- 36. The neural control centers for the integration
centage of increase in cardiac output that can and modulation of cardiac function and
be achieved above the normal resting level. blood pressure are located bilaterally in the
.
23. The mechanism allows the
heart to adjust its pumping ability to accom- 37. The neural control of the circulatory system
modate various levels of venous return. occurs primarily through the
and ________ divisions of the autonomic
24. The determines the frequency
nervous system.
with which blood is ejected from the heart.
38. When the intracranial pressure rises to levels
25. The outermost layer of a vessel, the
that equal intra-arterial pressure, blood
, is composed primarily of
vessels to the vasomotor center become com-
loosely woven collagen fibers. The middle
pressed, initiating the CNS ischemic
layer, the , is largely a smooth
response. This is known as the .
muscle layer. The innermost layer, the
consists of a single layer of flat-
Activity B Consider the following figures.
tened endothelial cells.
26. The represents the energy that
is transmitted from molecule to molecule
along the length of the vessel.
27. With peripheral arterial disease, there is a
delay in the transmission of the reflected
wave so that the pulse in
amplitude.
28. Pressure in the right atrium is called the
.
29. in the veins of extremities pre-
vent retrograde flow with the help of skeletal
muscles that surround and intermittently
compress the leg veins to move blood
forward to the heart.
Posterior
30. of blood flow is mediated by
changes in blood vessel tone due to changes
in flow through the vessel or by local tissue
factors.
31. An increase in local blood flow is called
. Anterior

32. In the heart and other vital structures,


1. Label the following structures.
channels exist between some of
the smaller arteries. Pericardium
Pleura
33. The term refers to the functions
of the smallest blood vessels, the capillaries, Right ventricle
and the neighboring lymphatic vessels. Right coronary artery
34. Water-filled junctions, called the Right atrium
, join the capillary endothelial Subclavian vein
cells and provide a pathway for passage of External jugular vein
substances through the capillary wall.
Internal jugular vein
35. The key factor that restrains fluid loss from Intraventricular septum
the capillaries is the pressure
Aortic arch
generated by the plasma proteins.

Copyright 2011. Wolters Kluwer Health | Lippincott Williams & Wilkins. Study Guide for Porths Essentials of Pathophysiology, Third Edition.
LWBK707-c17_p90-95.qxd 9/2/10 7:41 AM Page 93 Aptara Inc

CHAPTER 17 CONTROL OF CARDIOVASCULAR FUNCTION 93

Left atrium Activity C Match the key terms in Column A


Left coronary artery with their definitions in Column B.
Left ventricle Column A Column B
Superior vena cava
1. Diastole a. Contractile phase of
2. Label the following structures. cardiac cycle
2. Pericardium
b. Sac that covers the
3. End-diastolic heart
volume
c. Ventricular filling
4. Preload d. Resting phase of
5. Myocardium cardiac cycle
e. Semilunar and
6. Cardiac output
atrioventricular
7. Heart valves f. Residual blood
8. End-systolic volume following
volume contraction
g. Resistance to
9. Systole
ejection of blood
10. Afterload from heart
h. Heart rate  stroke
volume
i. Muscular wall of
heart
j. Volume in heart
Chordae tendineae following passive
Tricuspid valve filling phase

Superior vena cava Activity D Briefly answer the following.


Inferior vena cava 1. What are the factors involved in regulating
Pulmonic valve the flow of blood and how are they related?

Papillary muscle
Left pulmonary artery
Right pulmonary artery
Pulmonary veins 2. The velocity of blood in the circulatory system
varies considerably between large vessels and
Aortic valve capillaries. Normally, when fluid flows from a
Mitral valve large vessel to a smaller vessel, the velocity
increases, but this does not occur in the
Left atrium circulatory system. Why and for what
Right atrium purpose?

Left ventricle
Right ventricle
Descending aorta
Papillary muscles

Copyright 2011. Wolters Kluwer Health | Lippincott Williams & Wilkins. Study Guide for Porths Essentials of Pathophysiology, Third Edition.
LWBK707-c17_p90-95.qxd 9/2/10 7:41 AM Page 94 Aptara Inc

94 UNIT 5 CIRCULATORY FUNCTION

3. What is the importance of the Frank-Starling 3. The distensibility of the blood vessel is the
mechanism? major factor in which of the vessels
characteristics?
a. Wall tension
b. Compliance
c. Laminar blood flow
4. How is blood vessel diameter controlled? d. Resistance
4. When intracranial pressure (ICP) equals intra-
arterial pressure, the CNS ischemic response
is initiated. This response is directed at
raising arterial pressure above ICP, thereby
re-establishing blood flow to the vasomotor
5. What are the factors that travel in the blood-
center of the brain. What is this response
stream that will regulate blood flow? Indicate
called?
if each factor is a dilator or a vasoconstrictor.
a. Cushings law
b. Cushing response
c. Cushing reflex
d. Cushing syndrome
5. The troponin complex is one of a number of
important proteins that regulate actin-
SECTION III: PRACTICING myosin binding. Troponin works in striated
FOR NCLEX muscle to help regulate calcium-mediated
contraction of the muscle. Which of the
Activity E Answer the following questions. troponin complexes are diagnostic of a
myocardial infarction?
1. Blood volume is dictated by age and body
a. Troponin C and troponin T
weight. Neonates have a higher blood volume
per kilogram than do adults. What is the b. Troponin A and troponin I
blood volume range per kilogram in an adult? c. Troponin T and troponin I
a. 70 to 75 mL/kg d. Troponin A and troponin C
b. 85 to 90 mL/kg 6. The stroke volume is the amount of blood
c. 60 to 65 mL/kg ejected with every contraction of the ventri-
d. 90 to 100 ml/kg cle. It is broken down into quarters. What is
the approximate amount of the stroke
2. Resistance to flow is determined by the blood volume per quarter?
vessels and the blood vessel itself. An equation
a. 25%, 25%, 25%, and 25%
has been developed for understanding the
relationship between the diameter of the b. 50%, 30%, 20%, and little blood
blood vessel, the viscosity of the blood, and c. 40%, 40%, 10%, and 10%
resistance. What is the equation called? d. 60%, 20%, 20%, and little blood
a. LaPlaces law
b. Poiseuilles law
c. Laminars law
d. Pierres law

Copyright 2011. Wolters Kluwer Health | Lippincott Williams & Wilkins. Study Guide for Porths Essentials of Pathophysiology, Third Edition.
LWBK707-c17_p90-95.qxd 9/2/10 7:41 AM Page 95 Aptara Inc

CHAPTER 17 CONTROL OF CARDIOVASCULAR FUNCTION 95

7. Downstream peripheral pulses have a higher 11. Colloidal osmotic pressure acts differently
pulse pressure because the pressure wave than the osmotic effects of the plasma
travels faster than the blood itself. What proteins. What is its action?
occurs in peripheral arterial disease? a. Pulls fluid back into the capillary
a. The pulse decreases rather than increases b. Pushes fluid into the extracellular spaces
in amplitude
c. Controls the direction of the fluid flow in
b. The reflected wave is transmitted more the large arteries
rapidly through the aorta
d. Pulls fluid into the interstitial spaces
c. Downstream peripheral pulses are
increased even more than normal 12. The lymph system correlates with the vascu-
lar system without actually being a part of
d. Downstream peripheral pulses are greater
the vascular system. Among other things, the
than upstream pulses.
lymph system is the main route for the
8. Cardiac output (CO) is used to measure the absorption of fats from the gastrointestinal
efficiency of the heart as a pump. What is the system. The lymph system empties into the
equation used to express CO? right and left thoracic ducts, which are the
a. CO  HR  AV points of juncture with the vascular system.
What are these points of juncture?
b. CO  SV  HR
a. The bifurcation of the common carotid
c. CO  AV  SV
arteries
d. CO  HR  EF
b. The internal and external jugular veins
9. As the needs of the body change, the hearts c. Junctions of the subclavian and internal
ability to increase output necessarily needs to jugular veins
change to. This ability in the heart depends
d. The junction of the subclavian and the
on what factors? (Mark all that apply.)
pulmonary veins
a. Cardiac reserve
13. The heart and blood vessels receive both sym-
b. Cardiac contractility
pathetic and parasympathetic innervation
c. Heart rate from neural control. What controls the
d. Preload parasympathetic-mediated slowing of the
e. Afterload heart rate?
a. The vasomotor center
10. Nitroglycerin is the drug of choice in treating
angina. What does nitroglycerin release into b. The cardioinhibitory center
the vascular smooth muscle of the target tis- c. The medullary center
sues? d. The innervation center
a. Antithrombin factor
b. Platelet aggregating factor
c. Calcium channel blocker
d. Nitric oxide

Copyright 2011. Wolters Kluwer Health | Lippincott Williams & Wilkins. Study Guide for Porths Essentials of Pathophysiology, Third Edition.
LWBK707-c18_p96-103.qxd 8/19/10 1:14 PM Page 96 Aptara Inc

18
CHAPTER
Disorders of Blood Flow
and Blood Pressure

SECTION I: LEARNING 11. Compare the mechanisms and manife-


stations of ischemia associated with
OBJECTIVES atherosclerotic peripheral vascular disease,
Raynaud phenomenon, and thromboangiitis
1. Describe the functions of the endothelial
obliterans (i.e., Buerger disease).
cells and define the term endothelial dysfunc-
tion. 12. Distinguish between the pathology and man-
ifestations of aortic aneurysms and dissection
2. Describe the function of vascular smooth
of the aorta.
muscle and its role in vascular repair.
13. Describe venous return of blood from the
3. List the five types of lipoproteins and state
lower extremities, including the function of
their function in terms of lipid transport and
the muscle pumps and the effects of gravity,
development of atherosclerosis.
and relate to the development of varicose
4. Describe the role of lipoprotein receptors in veins.
removal of cholesterol from the blood.
14. Differentiate primary from secondary
5. Cite the criteria for diagnosis of hypercholes- varicose veins.
terolemia.
15. Characterize the pathology of venous insuffi-
6. Describe possible mechanisms involved in ciency and relate to the development of stasis
the development of atherosclerosis. dermatitis and venous ulcers.
7. List risk factors in atherosclerosis. 16. List the four most common causes of lower
leg ulcer.
8. List the vessels most commonly affected by
atherosclerosis and describe the vessel 17. Cite risk factors associated with venous
changes that occur. thrombosis and describe the manifestation of
the disorder and its treatment.
9. State the signs and symptoms of acute
arterial occlusion. 18. Define the terms systolic blood pressure,
diastolic blood pressure, pulse pressure, and
10. Describe the pathology associated with the
mean arterial blood pressure.
vasculitides and relate it to four disease con-
ditions associated with vasculitis.

96
LWBK707-c18_p96-103.qxd 8/19/10 1:14 PM Page 97 Aptara Inc

CHAPTER 18 DISORDERS OF BLOOD FLOW AND BLOOD PRESSURE 97

19. Explain how cardiac output and peripheral SECTION II: ASSESSING
vascular resistance interact in determining
systolic and diastolic blood pressure.
YOUR UNDERSTANDING
20. Describe the mechanisms for short-term and Activity A Fill in the blanks.
long-term regulation of blood pressure.
1. Although the heart is the center of the
21. Describe the requirements for accurate and cardiovascular system,
reliable blood pressure measurement in terms transport blood throughout the body.
of cuff size, determining the maximum infla-
2. Endothelial cells form a continuous lining for
tion pressure, and deflation rate.
the entire vascular system called the
22. Cite the definition of hypertension put forth .
by the seventh report of the Joint National
3. Vascular smooth muscle cells, which form
Committee on Detection, Evaluation, and
the predominant cellular layer in the tunica
Treatment of Hypertension.
media, produce or
23. Differentiate essential, systolic, and of blood vessels.
secondary forms of hypertension.
4. The term denotes a reduction
24. Describe the possible influence of genetics, in arterial flow to a level that is insufficient to
age, race, obesity, diet and sodium intake, meet the oxygen demands of the tissues.
and alcohol consumption on the
5. refers to an area of ischemic
development of essential hypertension.
necrosis in an organ produced by occlusion
25. Cite the risks of hypertension in terms of tar- of its arterial blood supply or its venous
get organ damage. drainage.
26. Describe behavior modification strategies 6. Elevated levels of blood are
used in the prevention and treatment of implicated in the development of atheroscle-
hypertension. rosis with its attendant risk of heart attack
and stroke.
27. List the different categories of drugs used to
treat hypertension and state their 7. Because and are
mechanisms of action in the treatment of insoluble in plasma, they are encapsulated by
high blood pressure. a stabilizing coat of water-soluble
lipoproteins.
28. Explain the changes in blood pressure that
accompany normal pregnancy and describe 8. The transport cholesterol and
the four types of hypertension that can occur triglycerides to various tissues for energy uti-
during pregnancy. lization, lipid deposition, steroid hormone
production, and bile acid formation.
29. Define systolic hypertension and relate the
circulatory changes that occur with aging 9. Some of the apoproteins activate the
that predispose to the development of enzymes that facilitate the
systolic hypertension. removal of lipids from the lipoproteins.
30. Define the term orthostatic hypotension. 10. There are two sites of lipoprotein synthesis:
the and the
31. Describe the cardiovascular, neurohumoral,
and muscular responses that serve to 11. transfer their triglycerides to
maintain blood pressure when moving from the cells of adipose and skeletal muscle tissue.
the supine to standing position.

Copyright 2011. Wolters Kluwer Health | Lippincott Williams & Wilkins. Study Guide for Porths Essentials of Pathophysiology, Third Edition.
LWBK707-c18_p96-103.qxd 8/19/10 1:14 PM Page 98 Aptara Inc

98 UNIT 5 CIRCULATORY FUNCTION

12. LDL, sometimes called the , is 29. is an inflammatory arterial dis-


the main carrier of cholesterol. order that causes thrombus formation.
13. LDL is removed from the circulation either by 30. is a functional disorder caused
or by cells. by intense vasospasm of the arteries and arte-
rioles in the fingers and, less often, the toes.
14. The uptake of LDL by macrophages in the
arterial wall can result in the accumulation of 31. An is an abnormal localized
insoluble cholesterol esters, the formation dilatation of a blood vessel.
of foam cells, and the development of
32. An aneurysm also may be , with
.
the first evidence of its presence being associ-
15. HDL is synthesized in the liver and often is ated with vessel .
referred to as the .
33. Aortic dissection involves into
16. Lipoprotein measurements are particularly the vessel wall with longitudinal tearing of
important in persons at high risk for develop- the vessel wall to form a blood-filled channel.
ment of .
34. Venous prevent the retrograde
17. Many types of primary hypercholesterolemia flow of blood.
have a basis.
35. The most common cause of secondary
18. Causes of hyperlipoproteinemia varicose veins is .
include obesity with high-calorie intake and
36. leads to tissue congestion,
diabetes mellitus.
edema, and eventual impairment of tissue
19. Excess calories consistently nutrition.
HDL and less consistently LDL.
37. Virchow described the triad that has come
20. is a type of arteriosclerosis or to be associated with venous thrombosis:
hardening of the arteries. , , and
.
21. The major risk factor for atherosclerosis is
. 38. blood pressure reflects the
rhythmic ejection of blood from the left
22. is closely linked with coronary
ventricle into the aorta.
heart disease and sudden death.
39. The pressure at the height of the pressure
23. Considerable interest in the role of
pulse is pressure, and the
in the etiology of atherosclero-
lowest pressure is the pressure.
sis has emerged over the last few years.
40. The difference between the systolic and dias-
24. is a serum marker for systemic
tolic pressure (approximately 40 mm Hg) is
inflammation.
called the .
25. inhibits elements of the antico-
41. The represents the average pres-
agulant cascade and is associated with
sure in the arterial system during ventricular
endothelial damage, which is thought to be
contraction and relaxation.
an important first step in the development
of atherosclerosis. 42. The mean arterial blood pressure is
determined mainly by the and
26. Activated macrophages release
the .
that oxidize LDL.
43. The renin-angiotensin-aldosterone system plays
27. Small vessel are sometimes
a central role in blood pressure by increasing
associated with antineutrophil cytoplasmic
and .
antibodies.
44. The extracellular fluid volume and arterial
28. An is a freely moving particle
blood pressure are regulated around an
such as a blood clot that breaks loose and
point, which represents the
travels in the larger vessels of the circulation
normal pressure for a given individual.
until lodging in a smaller vessel and
occluding blood flow.

Copyright 2011. Wolters Kluwer Health | Lippincott Williams & Wilkins. Study Guide for Porths Essentials of Pathophysiology, Third Edition.
LWBK707-c18_p96-103.qxd 8/19/10 1:14 PM Page 99 Aptara Inc

CHAPTER 18 DISORDERS OF BLOOD FLOW AND BLOOD PRESSURE 99

45. The role that the play in blood 58. The use of pills is probably the
pressure regulation is emphasized by the fact most common cause of secondary hyperten-
that many hypertension medications produce sion in young women.
their blood pressure-lowering effects by
59. is defined as an elevation in
increasing and
blood pressure and proteinuria developing
elimination.
after 20 weeks of gestation.
46. hypertension is the term applied
60. Any disease condition that reduces blood vol-
to 95% of cases in which no cause for hyper-
ume, impairs mobility, results in prolonged
tension can be identified. In
inactivity, or impairs autonomic nervous
hypertension, the elevation of blood pressure
system function may also predispose to
results from some other disorder.
.
47. A diagnosis of hypertension is made if the
systolic blood pressure is or Activity B Consider the following figures.
higher and the diastolic blood pressure is
or higher. LUMEN

48. The risk factors include a family


history of hypertension, race, and age-related
increases in blood pressure.
SHOULDER
49. An elevation in blood pressure increases the
workload of the by increasing
the pressure against which the heart must
pump as it ejects blood into the systemic
circulation.
50. Chronic hypertension leads to ,
a common cause of chronic kidney disease.
51. Hypertension is a major risk factor for
stroke and intracerebral
. In the figure of a fibrofatty plaque above, label
the following: media, lymphocytes, endothelial
52. The main objective for treatment of essential cells, smooth muscle cells, macrophages, CAP
hypertension is to achieve and maintain arte- region, and necrotic core.
rial blood pressure below .
53. lower blood pressure initially Arterial blood pressure

by decreasing vascular volume and cardiac


output. Cardiac output Peripheral vascular resistance

54. The blockers are effective in Sympathetic


activity
Stroke volume Heart rate
treating hypertension because they decrease
heart rate and cardiac output, as they are
Vagal and
cardioselective. Heart sympathetic activity

55. The drugs inhibit the move-


ment of calcium into cardiac and vascular Baroreceptors

smooth muscle. Venous return Angiotensin II

Blood volume Adrenal gland


56. Elevated pressures during favor Aldosterone

the development of left ventricular hypertro- Salt and water


retention
phy, increased myocardial oxygen demands,
and eventual left heart failure. Renin-angiotensin
mechanism

57. Many of the conditions causing Kidney

hypertension can be corrected or cured by What does this figure depict? Describe what the
surgery or specific medical treatment. solid lines represent and what the dashed lines
represent.

Copyright 2011. Wolters Kluwer Health | Lippincott Williams & Wilkins. Study Guide for Porths Essentials of Pathophysiology, Third Edition.
LWBK707-c18_p96-103.qxd 8/19/10 1:14 PM Page 100 Aptara Inc

100 UNIT 5 CIRCULATORY FUNCTION

Activity C Match the key terms in Column A 8. Coarctation g. Narrowing of the


with their definitions in Column B. of the aorta aorta
9. ANG II h. Systolic pressure
1.
over 140 mm Hg
Column A Column B 10. Nondippers
i. Persons whose BP
1. Chylo- a. Necrosis of the follows circadian
microns blood vessel wall rhythms
b. Main carrier of j. Block formation of
2. CRP
cholesterol ANG II
3. Familial
c. Derived from the
hyperchole- Activity D Put the sequence and actions of the
metabolism of
sterolemia renin-angiotensin-aldosterone system into
dietary methionine
chronological order.
4. Xanthomas d. Elevated levels of
blood cholesterol a. Water retention
5. Hyperchol-
esterolemia e. LDL-related b. Stimulation of juxtaglomerular apparatus
arteriosclerosis c. Conversion of angiotensinogen to angiotensin I
6. Vasculitis
f. Carries large d. Conversion of ANG I to ANG II by
7. VLDL amounts of angiotensin-converting enzyme
8. Homocysteine triglycerides e. Increased vascular resistance, release of aldos-
g. Caused by LDL terone
9. Atherosclerosis
receptor deficiency, f. Stimulation of juxtaglomerular apparatus
10. LDL which prevents
uptake of LDL g. Increased vascular resistance, release of aldos-
terone
h. Transfer
triglycerides to h. Na retention, stimulation of ADH release
skeletal muscle,
Activity E Briefly answer the following.
smaller than VLDLs
i. Elevated levels asso- 1. Describe the role of the endothelium.
ciated with arterial
disease
j. Cholesterol deposits
2.
2. Describe the causation of secondary
Column A Column B hyperlipoproteinemia.
1. Dippers a. Abnormal drop in
blood pressure on
2. Systolic
assumption of the
hypertension
standing position
3. Vasopressin b. Noninvasive BP 3. Describe the general mechanisms of drug
4. ACE measurement therapy to lower serum LDL levels.
inhibitors c. Persons with flat BP
profile
5. Postural
hypotension d. Increases renal
water retention
6. Indirect
e. Diastolic pressure 4. What are the seven signs and symptoms of
auscultatory
over 90 mm Hg acute arterial occlusion?
method
f. Strong vasoconstric-
7. Diastolic tor, reduces sodium
hypertension excretion

Copyright 2011. Wolters Kluwer Health | Lippincott Williams & Wilkins. Study Guide for Porths Essentials of Pathophysiology, Third Edition.
LWBK707-c18_p96-103.qxd 8/19/10 1:14 PM Page 101 Aptara Inc

CHAPTER 18 DISORDERS OF BLOOD FLOW AND BLOOD PRESSURE 101

5. What are the physical effects of Raynaud 5. Abnormal f. Arterial aneurysms


phenomenon? vessel dilation (arterial)
6. Acute vessel
obstruction
2. Where in the body is lipoprotein is
synthesized? (Mark all that apply.)
6. How do skeletal muscles of the leg contribute a. The small intestine
to returning blood to the heart?
b. The large intestine
c. The pancreas
d. The liver
3. A 35-year-old man presents to the emergency
department complaining of chest pain for the
7. Explain the short-term regulation of blood
last 2 hours. He describes the pain as
pressure. crushing, like a huge weight is on his chest.
He also states that the pain goes up into his
neck and down his left arm. An acute myocar-
dial infarction (MI) is diagnosed. When taking
his history, the following things are noted:
8. Why does the kidney play a major role in the - Hyperlipoproteinemia for past 7 years
development of secondary hypertension? - Family history of early MI
- Cholesterol deposits along the tendons
(diagnosed 1 year ago)
- Atherosclerosis (diagnosed 6 months ago)
- Diabetes mellitus (type 1) diagnosed at age 16
The nurse suspects which of the following di-
agnosis will be made?
SECTION III: PRACTICING a. Familial hypercholesterolemia (type 2A)
FOR NCLEX b. Homozygotic cutaneous xanthoma
c. Adult-onset hypercholesterolemia (type 1A)
Activity F Answer the following questions.
d. Secondary hyperlipoproteinemia
1. A variety of etiologies are responsible for 4. Atherosclerosis begins in an insidious manner
altering the blood flow in the systemic circu- with symptoms becoming apparent as long as
lation. Match the disturbance of blood flow 20 to 40 years after the onset of the disease.
with the cause. Although an exact etiology of the disease has
Disturbance in not been identified, epidemiologic studies
Blood Flow Cause have shown that there are predisposing risk
factors to this disease. What is the major risk
1. Abnormal vessel a. Atherosclerosis factor for developing atherosclerosis?
dilation (arterial) a. Male sex
2. Pathologic b. Raynaud b. Hypercholesterolemia
changes in vessel phenomenon
(vasospasm) c. Familial history of premature coronary
wall heart disease
c. Venous
3. Acute vessel d. Increasing age
thrombosis (venous)
obstruction
d. Varicose veins
4. Pathologic (venous)
changes in vessel
e. Vasculitis (arterial)
wall

Copyright 2011. Wolters Kluwer Health | Lippincott Williams & Wilkins. Study Guide for Porths Essentials of Pathophysiology, Third Edition.
LWBK707-c18_p96-103.qxd 8/19/10 1:14 PM Page 102 Aptara Inc

102 UNIT 5 CIRCULATORY FUNCTION

5. A group of vascular disorders called 9. A 56-year-old woman presents at the clinic


vasculitides cause inflammatory injury and complaining of the unsightliness of her vari-
necrosis of the blood vessel wall (i.e., vasculi- cose veins and wants to know what can be
tis). These disorders are common pathways done about them. The nurse explains that the
for tissue and organ involvement in many treatment for varicose veins includes which
different disease conditions. What is the most of the following interventions?
common of the vasculitides? a. Surgical or fibrotherapy
a. Polyarteritis nodosa b. Sclerotherapy or surgery
b. Raynaud disease c. Trendelenburg therapy or sclerotherapy
c. Temporal arteritis d. Surgery or Trendelenburg therapy
d. Varicose veins
10. Venous thrombosis most commonly occurs
6. A 69-year-old man is admitted to the floor in the lower extremities. Risk factors for
following a popliteal embolectomy. He asks venous thrombosis include which of the
the nurse why he had to have surgery on his following?
leg. What is the best response by the nurse? a. Stasis of blood, hypercoagulability, inflam-
a. The doctor wanted to look into your artery mation
make sure everything was okay. b. Hypocoagulability, vessel wall injury, in-
b. Didnt the doctor explain everything to creased pressure on deep veins
you before your surgery? c. Vessel wall injury, hypocoagulability,
c. The artery that runs behind your knee was decreased venous blood flow
blocked by a blood clot, and the doctor d. Stasis of blood, hypercoagulability, vessel
removed it. wall injury
d. Your upper leg was not getting enough
11. For people who suffer from hypertension and
blood so the doctor had to fix it.
other diseases that affect blood pressure,
7. A 45-year-old woman with a diagnosis of mul- important information about the status of
tiple sclerosis comes to the clinic complaining their disease is gathered from measurements
of coldness and pain in her fingers. She says including systolic and diastolic pressures,
that her fingers turn blue, and then her pulse pressure, and mean arterial pressure.
fingers get red, and they throb and tingle. The What is the mean arterial pressure estimated
nurse would expect what diagnosis and treat- to be when the blood pressure is 130/85?
ment for this patient? (Mark all that apply.) a. 90
a. Raynaud disease; protecting the digits b. 95
from cold
c. 100
b. Arterial thrombosis; streptokinase
d. 105
c. Peripheral artery disease; aspirin
12. Although the etiology of essential hyperten-
d. Raynaud phenomenon; stop smoking
sion is mainly unknown, several risk factors
8. Aortic aneurysms take varied forms and can have been identified. These risk factors fall
occur anywhere along the aorta. What are the under the categories of constitutional risk fac-
types of aneurysm termed abdominal aortic tors and lifestyle factors. What are the
aneurysms? (Mark all that apply.) primary risk factors for essential
a. Berry aneurysms hypertension? (Mark all that apply.)
b. Dissecting aneurysms a. Race and excessive sodium chloride intake
c. Saccular aneurysms b. Type 2 diabetes and obesity
d. Fusiform aneurysms c. Age and high intake of potassium
e. Bifurcating aneurysms d. Race and smoking
e. Family history and excessive alcohol con-
sumption

Copyright 2011. Wolters Kluwer Health | Lippincott Williams & Wilkins. Study Guide for Porths Essentials of Pathophysiology, Third Edition.
LWBK707-c18_p96-103.qxd 8/19/10 1:14 PM Page 103 Aptara Inc

CHAPTER 18 DISORDERS OF BLOOD FLOW AND BLOOD PRESSURE 103

13. A 37-year-old woman is admitted to your unit c. HELLP syndrome


with a differential diagnosis of rule out pheo- d. Decreased renal filtration rate
chromocytoma. What are the most common
symptoms you would expect this patient to 17. In infants and children, secondary hyperten-
exhibit? sion is the most common form of
hypertension. What is the most common
a. Nervousness and periodic severe headache
cause of hypertension in an infant?
b. Variability in blood pressure and weight loss
a. Cerebral vascular bleed
c. Excessive sweating and pallor
b. Coarctation of the aorta
d. Periodic severe headache and marked vari-
c. Pheochromocytoma
ability in blood pressure
d. Renal artery thrombosis
14. The extended, severe exposure of the walls of
the blood vessels to the exaggerated pressures 18. Hypertension in the elderly is a common
that occur in malignant hypertension cause finding. This is because of the age-related rise
injuries to the walls of the arterioles. Blood in systolic blood pressure. Among the aging
vessels in the renal system are particularly vul- processes, what is a contributor to hyperten-
nerable to this type of damage. Because hyper- sion?
tension is a chronic disease and is associated a. Baroreceptor sensitivity
with autoregulatory changes in the blood flow b. Aortic softening
to major organs, what would be the initial
c. Decreased peripheral vascular resistance
treatment goal for malignant hypertension?
d. Increased renal blood flow
a. Partial reduction in blood pressure to less
critical values 19. A 75-year-old man presents at the clinic for a
b. Reduction to normotensive levels of blood routine physical check-up. He is found to be
pressure hypertensive. While taking his blood pressure
in the sitting, standing, lying positions, the
c. Rapid decrease in blood pressure to less
nurse notes that the brachial artery is
critical levels
pulseless at a high cuff pressure, but she can
d. Slow, gradual decrease in blood pressure to still feel it. What condition would the nurse
normotensive blood pressures suspect?
15. A client with malignant hypertension is at a. Essential hypertension
risk for a hypertensive crisis, including the b. Pseudohypertension
cerebral vascular system often causing
c. Orthostatic hypertension
cerebral edema. As the nurse caring for this
patient, what are the signs and symptoms d. Secondary hypertension
you would assess for? 20. The rennin-angiotensin-aldosterone system is
a. Papilledema and lethargy a negative feedback system that plays a
b. Headache and confusion central role in blood pressure regulation.
How does the end result of this feedback loop
c. Restlessness and nervousness
regulate blood pressure in the body?
d. Stupor and hyperreflexia
a. Vasodilates blood vessels to decrease blood
16. Pregnancy-induced hypertension is a serious pressure
condition affecting between 5% and 10% of b. Vasoconstricts blood vessels to increase
pregnant women. The most serious classifica- blood pressure
tion of hypertension in pregnancy is
c. Increases salt and water retention by the
preeclampsia-eclampsia. It is a pregnancy-
kidney
specific syndrome that can have both mater-
nal and fetal manifestations. What is a d. Decreases salt and water retention by the
life-threatening manifestation of the kidney
preeclampsia-eclampsia classification of preg-
nancy-induced hypertension?
a. Hepatocellular necrosis
b. Thrombocytopenia

Copyright 2011. Wolters Kluwer Health | Lippincott Williams & Wilkins. Study Guide for Porths Essentials of Pathophysiology, Third Edition.
LWBK707-c19_p104-111.qxd 8/19/10 1:14 PM Page 104 Aptara Inc

19
CHAPTER
Disorders of Cardiac
Function

SECTION I: LEARNING 10. Differentiate among the pathophysiologic


changes that occur with hypertrophic
OBJECTIVES cardiomyopathy, arrhythmogenic right ven-
tricular cardiomyopathy, dilated
1. Characterize the function of the pericardium.
cardiomyopathies, and myocarditis.
2. Compare the clinical manifestations of acute
11. List four causes of secondary
pericarditis and chronic pericarditis.
cardiomyopathy.
3. Describe the physiologic impact of pleural
12. Describe the treatment strategies of both pri-
effusion on cardiac function and relate it to
mary and secondary cardiomyopathy.
the life-threatening nature of cardiac
tamponade. 13. Distinguish between the roles of infectious
organisms and the immune system in infec-
4. Relate the pathophysiology of constrictive
tive endocarditis and rheumatic fever.
pericarditis to its clinical manifestations.
14. Describe the relation between the infective
5. Describe blood flow in the coronary circula-
vegetations associated with infective
tion and relate it to the determinants of
endocarditis and the extracardiac manifesta-
myocardial oxygen supply and demand.
tions of the disease.
6. Define the term acute coronary syndrome (ACS)
15. Describe the long-term effects of rheumatic
and distinguish among chronic stable
fever and primary and secondary prevention
angina, unstable angina, nonST-segment
strategies for rheumatic fever and rheumatic
elevation myocardial infarction (MI), and ST-
heart disease.
segment elevation infarction in terms of
pathology, symptomatology, 16. State the function of the heart valves and
electrocardiograph (ECG) changes, and relate alterations in hemodynamic function
serum cardiac markers. of the heart that occur with valvular disease.
7. Compare the treatment goals for stable 17. Compare the effects of stenotic and regurgi-
angina and the acute coronary syndromes. tant mitral and aortic valvular heart disease
on cardiovascular function.
8. Define the term cardiomyopathy as it relates to
both the mechanical and electrical function 18. Compare the methods of and diagnostic
of the myocardium. information obtained from cardiac ausculta-
tion and echocardiography as they relate to
9. Describe the role of genetics in the etiology
valvular heart disease.
of the primary cardiomyopathies.

104
LWBK707-c19_p104-111.qxd 8/19/10 1:14 PM Page 105 Aptara Inc

CHAPTER 19 DISORDERS OF CARDIAC FUNCTION 105

19. Trace the flow of blood in the fetal 6. In pericarditis, fibrous, calcified
circulation, state the function of the foramen scar tissue develops between the visceral and
ovale and ductus arteriosus, and describe the parietal layers of the serous pericardium.
changes in circulatory function that occur at
7. In most cases, coronary artery disease (CAD)
birth.
is caused by .
20. Compare the effects of left-to-right and right-
8. Myocardial blood flow, in turn, is largely reg-
to-left shunts on the pulmonary circulation
ulated by the of the
and production of cyanosis.
myocardium and mechanisms
21. Describe the anatomic defects and altered that control vessel dilation.
patterns of blood flow in children with atrial
9. There is little oxygen reserve in the blood;
septal defects, ventricular septal defects,
therefore, coronary arteries must increase their
endocardial cushion defects, pulmonary
flow to meet the metabolic needs of the
stenosis, tetralogy of Fallot, patent ductus
myocardium during periods of .
arteriosus, transposition of the great vessels,
coarctation of the aorta, and single-ventricle 10. The is the most frequently used
anatomy. cardiovascular diagnostic procedure.
22. Describe the prevalence of the condition and 11. uses ultrasound signals that
issues of concern for adults with congenital inaudible to the human ear.
heart disease.
12. is by far the most common
23. Describe the manifestations related to the cause of CAD.
acute, subacute, and convalescent phases of
13. There are two types of atherosclerotic lesions:
Kawasaki disease.
the plaque, which obstructs
blood flow, and the plaque,
which can rupture and cause platelet
adhesion and thrombus formation.
SECTION II: ASSESSING
YOUR UNDERSTANDING 14. Coronary artery disease is commonly divided
into two types of disorders: and
Activity A Fill in the blanks. .

1. The is a double-layered serous 15. The classic ECG changes that occur with ACS
membrane that isolates the heart from other involve , , and
thoracic structures, maintains its position in .
the thorax, prevents it from overfilling, and 16. Acute severe ischemia reduces the
serves as a barrier to infection. and shortens the duration of
2. Pericardial fluid acts as a lubricant that the action potential in the ischemic area.
prevents forces from developing 17. The have high specificity for
as the heart contracts and relaxes. myocardial tissue and have become the
3. The manifestations of acute primary biomarker for the diagnosis of MI.
include a triad of chest pain, pericardial fric- 18. myocardial infarction is charac-
tion rub, and ECG changes. terized by the ischemic death of myocardial
4. Pericardial refers to the tissue associated with atherosclerotic disease
accumulation of fluid in the pericardial of the coronary arteries.
cavity, usually because of an inflammatory 19. Irreversible myocardial cell death occurs after
and or infectious process. minutes of severe ischemia.
5. Pericardial effusion can lead to cardiac 20. Infarcted and noninfarcted areas of the heart
, in which there is compression muscle in patients with ST-segment elevation
of the heart due to the accumulation of fluid, myocardial infarction (STEMI) can change
pus, or blood in the pericardial sac. size, shape, and thickness, a term referred to
as .

Copyright 2011. Wolters Kluwer Health | Lippincott Williams & Wilkins. Study Guide for Porths Essentials of Pathophysiology, Third Edition.
LWBK707-c19_p104-111.qxd 8/19/10 1:14 PM Page 106 Aptara Inc

106 UNIT 5 CIRCULATORY FUNCTION

21. The gastrointestinal symptoms of STEMI are 33. The function of the heart is to
thought to be related to the severity of the promote directional flow of blood through
pain and stimulation. the chambers of the heart.
22. The medication used to alleviate angina, 34. Mitral valve represents the
, is given because of its incomplete opening of the mitral valve
vasodilating effect. during diastole with left atrial distention and
impaired filling of the left ventricle.
23. is a mechanical technique to
remove atherosclerotic tissue during 35. Mitral valve is characterized by
angioplasty. incomplete closure of the mitral valve, with
the left ventricular stroke volume being
24. Partial or complete rupture of a
divided between the forward stroke volume
is a rare but often fatal complication of trans-
that moves into the aorta and the regurgitant
mural myocardial infarction.
stroke volume that moves back into the left
25. is the initial manifestation of atrium during systole.
ischemic heart disease in approximately half
36. Most persons with mitral valve
of persons with CAD.
are asymptomatic and the disorder is
26. Typically, chronic stable angina is provoked discovered during a routine physical
by or stress and examination.
relieved within minutes by rest or the use of
37. Increased resistance to ejection of blood from
nitroglycerin.
the left ventricle into the aorta characterizes
27. The cardiomyopathies include aortic valve .
hypertrophic cardiomyopathy, arrhythmogenic
38. Aortic is the result of an
right ventricular cardiomyopathy, left
incompetent aortic valve that allows blood to
ventricular noncompaction cardiomyopathy,
flow back to the left ventricle during diastole.
inherited conduction system disorders, and
ion channelopathies. 39. The major development of the
occurs between the fourth and seventh weeks
28. The cardiomyopathies, which
of gestation, and most congenital heart
include dilated cardiomyopathy, are of both
defects arise during this time.
genetic and nongenetic origin.
40. Congenital heart defects produce their effects
29. The physiologic abnormality in
mainly through abnormal shunting of
is reduced left ventricular chamber size, poor
, production of ,
compliance with reduced stroke volume that
and disruption of blood flow.
results from impaired diastolic filling, and
dynamic obstruction of left ventricular 41. Congenital heart defects that result in a left-
outflow. to-right shunt are usually categorized as
disorders because they do not
30. cardiomyopathies are character-
compromise oxygenation of blood in the pul-
ized by atrophic and hypertrophic
monary circulation.
myocardial fibers and interstitial fibrosis.
42. A defect is an opening in the
31. is the most common, and
ventricular septum that results from an
frequently the first, manifestation of
incomplete separation of the ventricles
rheumatic fever.
during early fetal development.
32. The manifestation of rheumatic
43. disease, also known as mucocu-
fever is Sydenham chorea, in which the child
taneous lymph node syndrome, is an acute
often is fidgety, cries easily, begins to walk
febrile disease of young children.
clumsily, and drops things.

Copyright 2011. Wolters Kluwer Health | Lippincott Williams & Wilkins. Study Guide for Porths Essentials of Pathophysiology, Third Edition.
LWBK707-c19_p104-111.qxd 8/19/10 1:14 PM Page 107 Aptara Inc

CHAPTER 19 DISORDERS OF CARDIAC FUNCTION 107

Activity B Consider the following figure.

Superior Right Superior


vena Left pulmonary vena
cava pulmonary veins cava
Aortic veins
arch Aortic valve Coronary
Left sinus
Inferior
atrium vena
cava

Right
atrium Right
atrium

Right
Left ventricle
Right ventricle
ventricle

In the figure above, label the coronary arteries.

Activity C Match the key terms in Column A 10. Pulsus h. Combination of


with their definitions in Column B. paradoxus effusion-tamponade
and constriction
1.
i. Chest pain
Column A Column B occurring while at
rest
1. Unstable a. Chest pain due to a
angina coronary artery j. Exaggeration of the
spasm normal variation in
2. Effusive- the pulse during the
constrictive b. ST elevation
inspiratory phase of
pericarditis myocardial
respiration
infarction
3. Ischemia
c. Decreased blood
4. Pericardial flow to tissue
effusion d. Accumulation of
5. Prinzmetal fluid in the pericar-
angina dial cavity
e. Invasion of the
6. Cardiac
heart valves and the
tamponade
mural endocardium
7. Silent by a microbial
myocardial agent
ischemia f. Mechanical
8. Heart attack compression of the
heart
9. Infective
g. Occurs in the
endocarditis
absence of anginal
pain

Copyright 2011. Wolters Kluwer Health | Lippincott Williams & Wilkins. Study Guide for Porths Essentials of Pathophysiology, Third Edition.
LWBK707-c19_p104-111.qxd 8/19/10 1:14 PM Page 108 Aptara Inc

108 UNIT 5 CIRCULATORY FUNCTION

2. Activity D Briefly answer the following.


Column A Column B 1. Why does pericardial effusion demonstrate
1. Restrictive a. Ventricular enlarge-
signs of right-sided heart failure?
cardiomyo- ment, a reduction
pathy in ventricular wall
thickness, and
2. Ion
impaired systolic
channelo-
function
pathies 2. What factors determine myocardial oxygen
b. An inflammation of supply and demand?
3. Myocarditis the heart
4. Arrhythmo- c. With disproportion-
genic right ate thickening of the
ventricular ventricular septum
cardiomyo- and left ventricle
pathy d. Occurs during the 3. How does an atherosclerotic plaque stimulate
5. Dilated last trimester of thrombosis?
cardiomyo- pregnancy or the
pathy first 6 months after
delivery
6. Stress
e. Conduction
cardiomyo-
disorders in the
pathy 4. What changes are seen in the blood (serum)
heart resulting from
7. Hypertrophic abnormal during ACS?
cardiomyo- membrane
pathy potentials (long
QT/short QT
8. Left ventricular
syndromes)
noncomp-
action f. Left ventricular 5. Describe the pathologic process that is seen
dysfunction in in unstable angina/nonST-segment
9. Secondary response to
cardiomyo- elevation myocardial infarction.
profound
pathy psychological or
10. Peripartum emotional stress
cardiomyo- g. Ventricular filling is
pathy restricted because of
excessive rigidity of 6. What is the damage that results from an
the ventricular walls acute myocardial infarction and what are the
h. Heart muscle factors that determine severity?
disease that affects
primarily the right
ventricle
i. Heart muscle disease
in the presence of a
7. What is meant by reperfusion therapy and
multisystem disorder
what is its goal?
j. Failure of trabecular
compaction in the
developing
myocardium

Copyright 2011. Wolters Kluwer Health | Lippincott Williams & Wilkins. Study Guide for Porths Essentials of Pathophysiology, Third Edition.
LWBK707-c19_p104-111.qxd 8/19/10 1:14 PM Page 109 Aptara Inc

CHAPTER 19 DISORDERS OF CARDIAC FUNCTION 109

8. What is the definition of a cardiomyopathy, 2. What are the emergency department goals of
according to the American Heart Association? management for a patient with a STEMI?

9. What is the relationship between strep throat


and heart valve disorders?
SECTION IV: PRACTICING
FOR NCLEX
Activity F Answer the following questions.

10. Describe the clinical manifestation of patent 1. Nearly everyone with pericarditis has chest
ductus arteriosus. pain. With acute pericarditis the pain is
abrupt in onset, sharp, and radiates to the
neck, back, abdomen, or sides. What can be
done to ease the pain of acute pericarditis?
a. Have patient sit up and lean forward
b. Have patient change positions to unaf-
11. Describe the tetralogy of Fallot. fected side
c. Have patient breathe deeply
d. Have patient swallow slowly and
frequently
2. Cardiac tamponade is a serious life-threatening
condition that can arise from a number of
other conditions. What is a key diagnostic
SECTION III: APPLYING finding in cardiac tamponade?
YOUR KNOWLEDGE a. Increase in stroke volume
b. Pulsus paradoxus
Activity E Consider the scenario and answer
the questions. c. Narrowed pulse pressure
d. Rise in systolic blood pressure
A 55-year-old woman is brought to the
emergency department by ambulance and is 3. The scar tissue that occurs between the layers
complaining of severe, acute chest pain. of the pericardium becomes rigid and
The patient states that It just came on all of a constrictive from scar tissue in constrictive
sudden. Like someone sitting on my chest crush- pericarditis. What is a physiologic sign of
ing me. An ECG shows ST-segment elevation constrictive pericarditis?
and the presumptive diagnosis is acute STEMI. a. Kussmaul breathing
1. While obtaining a history on this patient, b. Pulsus paradoxus
what symptoms would the nurse pay particu- c. Kussmaul sign
lar attention to as they are further indications
d. Widening pulse pressure
of a STEMI?

Copyright 2011. Wolters Kluwer Health | Lippincott Williams & Wilkins. Study Guide for Porths Essentials of Pathophysiology, Third Edition.
LWBK707-c19_p104-111.qxd 8/19/10 1:14 PM Page 110 Aptara Inc

110 UNIT 5 CIRCULATORY FUNCTION

4. Unstable plaque, a condition of 8. ST-elevated myocardial infarction is accompa-


atherosclerotic heart disease, occurs in unstable nied by severe, crushing pain. Morphine is
angina and myocardial infarction. Unstable the drug of choice used to treat the pain of
plaque can rupture, causing platelet aggrega- STEMI when the pain cannot be relieved with
tion and thrombus formation. What are the oxygen and nitrates. Why is morphine
major determinants of the vulnerability of considered the drug of choice in STEMI?
plaque to rupture? (Mark all that apply.) a. Action increases autonomic nervous
a. Size of lipid-rich core system activity
b. Preponderance of smooth muscle cells b. Action decreases metabolic demands of
c. Presence of inflammation the heart
d. Decrease in blood pressure and coronary c. Action increases anxiety increasing meta-
blood flow bolic demands of heart
e. Thickness of fibrous cap d. Action relieves pain and gives sense of de-
pression
5. A patient with a suspected MI is brought to
the emergency department by ambulance. As 9. During an acute MI there is ischemic damage
the nurse caring for this patient, what labora- to the heart muscle. The location and extent
tory work would you expect to receive an of the ischemic damage is the major predictor
order for, to confirm a diagnosis of MI? of complications, ranging from cardiac
insufficiency to death, following an MI. What
a. Creatine kinase marker
is the window of opportunity in restoring
b. Complete blood components blood flow to the affected area so as to
c. Calcium level diminish the ischemic damage to the heart
d. Troponin level and maintain the viability of the cells?
a. 10 to 20 minutes
6. Unstable angina (UA)/nonST-segment eleva-
tion myocardial infarction (NSTEMI) is a clin- b. 30 to 40 minutes
ical syndrome that ranges in severity between c. 20 to 40 minutes
stable angina to MI. It is classified according d. 10 to 30 minutes
to its risk of causing an acute MI and is diag-
nosed based on what? (Mark all that apply.) 10. Angina pectoris is a chronic ischemic CAD
that is characterized by a symptomatic
a. Severity of pain and abruptness of onset
paroxysmal chest pain or pressure sensation
b. Serum biomarkers associated with transient myocardial
c. Coexisting chronic conditions ischemia. What precipitates an attack of
d. ECG pattern angina pectoris?
e. Blood-flow angiography a. Exposure to heat
b. Sedentary lifestyle
7. When an acute MI occurs, many physiologic
changes occur very rapidly. What causes the c. Abrupt change in position
loss of contractile function of the heart d. Emotional stress
within seconds of the onset of an MI?
11. The diagnosis of chronic stable angina is
a. Conversion from aerobic to anaerobic me- based on a detailed pain history, the presence
tabolism of risk factors, invasive and noninvasive stud-
b. Overproduction of energy capable of sus- ies, and laboratory studies. What test is not
taining normal myocardial function used in the diagnosis of angina?
c. Conversion from anaerobic to aerobic me- a. Serum biochemical markers
tabolism b. Cardiac catheterization
d. Inadequate production of glycogen with c. Echocardiogram
mitochondrial shrinkage
d. Nuclear imaging studies

Copyright 2011. Wolters Kluwer Health | Lippincott Williams & Wilkins. Study Guide for Porths Essentials of Pathophysiology, Third Edition.
LWBK707-c19_p104-111.qxd 8/19/10 1:14 PM Page 111 Aptara Inc

CHAPTER 19 DISORDERS OF CARDIAC FUNCTION 111

12. Cardiomyopathies are classified as either 16. Mitral valve prolapse occurs frequently in the
primary or secondary. The primary population at large. Its treatment is aimed at
cardiomyopathies are further classified as relieving the symptoms and preventing com-
genetic, mixed, or acquired. Identify whether plications of the disorder. Which drug is used
the following conditions are classified as in the treatment of mitral valve prolapse to
genetic, acquired, or mixed. relieve symptoms and aid in preventing com-
a. Hypertrophic cardiomyopathy plications?
b. Left ventricular noncompaction a. -Adrenergicblocking drugs
c. Myocarditis b. Calcium channel blocking drugs
d. Dilated cardiomyopathy c. Antianxiety drugs
e. Peripartum cardiomyopathy d. Broad-spectrum antibiotic drugs

13. It is known that over 100 distinct myocardial 17. Heart failure in an infant usually manifests
diseases can demonstrate clinical features itself as tachypnea or dyspnea, both at rest
associated with dilated cardiomyopathy and on exertion. When does this most com-
(DCM). What is the most common monly occur with an infant?
identifiable cause of DCM in the United a. During bathing
States? b. During feeding
a. Hepatic cardiomyopathy c. During burping
b. Alcoholic cardiomyopathy d. During sleep
c. Cardiotoxic cardiomyopathy
18. Tetralogy of Fallot is a congenital condition
d. Exercise induced cardiomyopathy of the heart that manifests in four distinct
14. In infective endocarditis vegetative lesions anomalies of the infant heart. It is considered
grow on the valves of the heart. These vegeta- a cyanotic heart defect because of the right-
tive lesions consist of a collection of to-left shunting of the blood through the
infectious organisms and cellular debris ventricular septal defect. A hallmark of this
enmeshed in the fibrin strands of clotted condition is the tet spells that occur in
blood. What are the possible systemic effects these children. What is a tet spell?
of these vegetative lesions? a. A stressful period right after birth that
a. They can block the heart valves from clos- occurs without evidence of cyanosis.
ing completely b. A hyperoxygenated period when the
b. They can keep the heart valves from infant is at rest
opening c. A hypercyanotic attack brought on by
c. They can fragment and cause cerebral periods of stress
emboli d. A hyperpneic attack in which the infant
d. They can fragment and make the lesions loses consciousness
larger
15. Antibodies directed against the M protein of
certain strains of streptococcal bacteria seem
to cross-react with glycoprotein antigens in
the heart, joint, and other tissues to produce
an autoimmune response resulting in
rheumatic fever and rheumatic heart disease.
This occurs through what phenomenon?
a. The Aschoff reaction
b. The Sydenham reaction
c. C-reactive mimicry
d. Molecular mimicry

Copyright 2011. Wolters Kluwer Health | Lippincott Williams & Wilkins. Study Guide for Porths Essentials of Pathophysiology, Third Edition.
LWBK707-c20_p112-117.qxd 8/19/10 1:15 PM Page 112 Aptara Inc

20
CHAPTER
Heart Failure and
Circulatory Shock

SECTION I: LEARNING 9. Relate the use of cardiac resynchronization,


implantable cardioverter-defibrillators, left
OBJECTIVES ventricular assist devices, heart
transplantation, and other surgical
1. Define heart failure.
alternatives to the treatment of selected types
2. Describe the contractile properties of the of heart failure.
myocardium.
10. State a clinical definition of shock.
3. Explain how the Frank-Starling mechanism,
11. Compare the causes, pathophysiology, and
sympathetic nervous system, renin-
chief characteristics of cardiogenic,
angiotensin-aldosterone mechanism,
hypovolemic, obstructive, and distributive
natriuretic peptides, endothelins, and
shock.
myocardial hypertrophy and remodeling
function as adaptive and maladaptive mecha- 12. Describe the complications of shock as they
nisms in heart failure. relate to the lungs, kidneys, gastrointestinal
tract, and blood clotting.
4. Differentiate high-output versus low-output
heart failure, systolic versus diastolic heart 13. State the rationale for treatment measures to
dysfunction, and right-sided versus left-sided correct and reverse shock.
heart failure in terms of causes, impact on
14. Define multiple organ dysfunction syndrome
cardiac function, and major manifestations.
and cite its significance in shock.
5. Differentiate chronic heart failure from acute
15. Describe the causes of heart failure in infants
heart failure syndromes.
and children.
6. Describe the manifestations of heart failure
16. Cite how the aging process affects cardiac
and relate to the function of the heart.
function and predisposes to ventricular
7. Describe the methods used in diagnosis and dysfunction.
assessment of cardiac function in persons
17. State how the signs and symptoms of heart
with heart failure.
failure may differ between younger and older
8. Relate the pharmacologic actions of adults.
angiotensin-converting enzyme inhibitors
and receptor blockers, -adrenergic blockers,
diuretics, digoxin, and vasodilatory agents to
the treatment of heart failure.

112
LWBK707-c20_p112-117.qxd 8/19/10 1:15 PM Page 113 Aptara Inc

CHAPTER 20 HEART FAILURE AND CIRCULATORY SHOCK 113

SECTION II: ASSESSING 12. Systolic dysfunction commonly results from


conditions that impair the per-
YOUR UNDERSTANDING formance of the heart (e.g., ischemic heart
disease and cardiomyopathy), produce a
Activity A Fill in the blanks.
(e.g., valvular insufficiency and
1. has been defined as a complex anemia), or generate a (e.g.,
syndrome that results from any functional or hypertension and valvular stenosis) on the
structural disorder of the heart that results in heart.
decreased pumping.
13. In dysfunction, cardiac output
2. Among the most common causes of heart is compromised by the abnormal filling of
failure are , , the ventricle.
dilated cardiomyopathy, and
14. Among the conditions that cause diastolic
heart disease.
dysfunction are those that the
3. Endurance athletes have cardiac ventricle (e.g., pericardial effusion, constric-
reserves. tive pericarditis), those that
4. can be expressed as the product wall thickness and reduce chamber size (e.g.,
of the heart rate and stroke volume. myocardial hypertrophy, hypertrophic
cardiomyopathy), and those that
5. The heart rate is regulated by a balance
diastolic relaxation (e.g., aging,
between the activity of the
ischemic heart disease).
nervous system, which produces an increase
in heart rate, and the nervous 15. Diastolic dysfunction can be aggravated by
system, which slows it down. and can be improved by a
reduction in heart rate.
6. The is a function of preload,
afterload, and myocardial contractility. 16. Heart failure can be classified according to
the of the heart that is
7. is the percentage of blood
primarily affected.
pumped out of the ventricles with each
contraction. 17. A major effect of right-sided heart failure is
the development of .
8. In systolic ventricular dysfunction,
myocardial contractility is impaired, leading 18. As venous distention progresses in right-
to a in the ejection fraction sided heart failure, blood backs up in the
and cardiac output. veins that drain into the
inferior vena cava, and the liver becomes
9. Diastolic ventricular dysfunction is character-
engorged.
ized by a ejection fraction but
impaired diastolic ventricular relaxation lead- 19. is the most common cause of
ing to a decrease in ventricular filling, which right ventricular failure.
ultimately causes a decrease in preload,
20. The most common causes of
stroke volume, and cardiac output.
ventricular dysfunction are acute myocardial
10. With both systolic and diastolic ventricular infarction and cardiomyopathy.
dysfunction, are usually able to
21. is an uncommon type of heart
maintain adequate resting cardiac function
failure that is caused by an excessive need for
until the later stages of heart failure.
cardiac output.
11. The rise in preload seen in systolic
22. is caused by disorders that
dysfunction is thought to be a compensatory
impair the pumping ability of the heart, such
mechanism to help maintain stroke volume
as ischemic heart disease and
via the mechanism despite a
cardiomyopathy.
drop in ejection fraction.
23. The development of constitutes
one of the principle mechanisms by which
the heart compensates for an increase in
workload.

Copyright 2011. Wolters Kluwer Health | Lippincott Williams & Wilkins. Study Guide for Porths Essentials of Pathophysiology, Third Edition.
LWBK707-c20_p112-117.qxd 8/19/10 1:15 PM Page 114 Aptara Inc

114 UNIT 5 CIRCULATORY FUNCTION

24. A gradual or rapid change in heart failure 37. A defect in the vasomotor center in the brain
signs and symptoms resulting in a need for stem or the sympathetic outflow to the blood
urgent therapy is defined as vessels is known as .
syndrome.
38. Anaphylactic shock results from an
25. dyspnea is a sudden attack of mediated reaction in which
dyspnea that occurs during sleep. vasodilator substances such as histamine are
released into the blood.
26. is the most dramatic symptom
of acute heart failure syndromes. 39. heart defects are the most com-
mon cause of heart failure in children.
27. In acute or severe left-sided failure, cardiac
output may fall to levels that are insufficient 40. is associated with impaired left
for providing the with ventricular filling that is due to changes in
adequate oxygen. myocardial relaxation and compliance.
28. Ascites is a common manifestation associated
with ventricular failure and Activity B Match the key terms in Column A
long-standing elevation of systemic venous with their definitions in Column B.
pressures.
1.
29. Central cyanosis is caused by conditions that
Column A Column B
impair of the arterial blood.
1. Inotropy a. Volume or loading
30. In persons with ventricular dysfunction, sud-
conditions of the
den death is caused most commonly by 2. Cardiac
ventricle at the end
tachycardia or fibrillation. output
of diastole
31. Measurements of are 3. Afterload b. Right heart failure
recommended to confirm the diagnosis of occurs in response
4. Pulmonary
heart failure to evaluate the severity of left to chronic
congestion
ventricular compromise and estimate the pulmonary disease
prognosis, and predict future cardiac events 5. Cardiac
c. Ability to increase
such as sudden death, and to evaluate the reserve
cardiac output
effectiveness of treatment.
6. Cor during increased
32. -Adrenergic receptor blocking drugs are pulmonale activity
used to decrease dysfunction d. The force that the
7. High-
associated with activation of the sympathetic contracting heart
output failure
nervous system. muscle must gener-
8. Preload ate to eject blood
33. can be described as an acute
failure of the circulatory system to supply the 9. Systolic from the filled heart
peripheral tissues and organs of the body dysfunction e. Failure that is
with an adequate blood supply, resulting in caused by an exces-
10. Endothelins
cellular hypoxia. sive need for cardiac
output
34. The most common cause of cardiogenic
shock is . f. Amount of blood
the ventricles eject
35. shock is characterized by each minute
diminished blood volume such that there is
g. Ejection fraction
inadequate filling of the vascular
less than 40%
compartment.
h. Potent vasoconstric-
36. shock is characterized by loss of tors
blood vessel tone, enlargement of the vascu-
i. Common sign of left
lar compartment, and displacement of the
ventricular failure
vascular volume away from the heart and
central circulation. j. Contractile perform-
ance of the heart

Copyright 2011. Wolters Kluwer Health | Lippincott Williams & Wilkins. Study Guide for Porths Essentials of Pathophysiology, Third Edition.
LWBK707-c20_p112-117.qxd 8/19/10 1:15 PM Page 115 Aptara Inc

CHAPTER 20 HEART FAILURE AND CIRCULATORY SHOCK 115

2. Activity C
Column A Column B 1. The pathophysiology of right- and left-sided
1. Hydrothorax a. Periodic breathing heart failure has distinct features. Construct a
characterized by flow chart of the following symptoms and
2. Cyanosis their causes:
gradual increase in
3. Cheyne-Stokes depth followed by a Right heart failure
respiration decrease resulting in Left heart failure
apnea
4. Dyspnea Orthopnea
b. Bronchospasm due
5. Cardiac asthma to congestion of the Cyanosis
6. Circulatory bronchial mucosa Activity intolerance
failure c. Bluish discoloration Anorexia
of the skin Weight loss
7. Orthopnea
d. Labored breathing Impaired liver function
8. Ascites
e. Transudation of Gastrointestinal (GI) tract congestion
fluid into the Impaired gas exchange
peritoneal cavity
Pulmonary edema
f. Hypoperfusion of
organs and tissues Dependent edema and ascites
g. Transudation of Congestion of peripheral tissues
fluid into the Decreased cardiac output
pleural cavity Pulmonary congestion
h. Shortness of breath
when supine Activity D Briefly answer the following.

3. 1. How is cardiac contractility regulated?

Column A Column B
1. Cardiogenic a. An acute failure of
shock the circulatory
system to supply
2. Obstructive
the peripheral 2. Why is it advisable to test cardiac function
shock during exercise (stress) rather than at rest?
tissues and organs
3. Distributive of the body with an
shock adequate blood
supply
4. Hypovolemic
shock b. Caused by excessive
vasodilation with
5. Circulatory mal distribution of 3. How does diastolic dysfunction produce the
shock blood flow typical signs and symptoms that characterize
the condition?
c. Caused by alteration
in cardiac function
d. Caused by a
decrease in blood
volume
e. Caused by
obstruction of blood
flow through the
circulatory system

Copyright 2011. Wolters Kluwer Health | Lippincott Williams & Wilkins. Study Guide for Porths Essentials of Pathophysiology, Third Edition.
LWBK707-c20_p112-117.qxd 8/19/10 1:15 PM Page 116 Aptara Inc

116 UNIT 5 CIRCULATORY FUNCTION

4. Often, the early signs of heart failure are SECTION III: PRACTICING
silent. This is because of the many compensa-
tory mechanisms of the cardiovascular system.
FOR NCLEX
Explain, briefly, how these mechanisms work
Activity E Answer the following questions.
and why in the end they only serve to make
the heart failure worse. 1. Match the following conditions with the type
of heart failure they cause.
Condition Type of Heart Failure
1. Valvular a. Diastolic dysfunction
insufficiency
b. Left ventricular
5. What are the common manifestations of heart
2. Ischemic dysfunction
failure? Why?
heart disease c. Right ventricular
3. Aortic or dysfunction
mitral stenosis d. Low-output failure
4. Acute e. High-output failure
myocardial f. Systolic dysfunction
6. What effect does diuretic therapy have on
infarction
heart failure?
5. Paget disease
6. Cardiomyopathy
2. What are the signs and symptoms of heart
failure? (Mark all that apply.)
7. What are the cellular consequences of shock? a. Fluid retention
b. Ruddy complexion
c. Fatigue
d. Bradycardia
e. Chronic productive cough
8. What are the five major complications of
severe shock? 3. When an acute event occurs and the circula-
tory system can no longer provide the body
with adequate perfusion of its tissues and
organs, cellular hypoxia occurs and the body
goes into shock. What are the causes of shock
in the human body?
a. Maldistribution of blood flow
b. Hypovolemia
c. Excessive vasoconstriction
d. Obstruction of blood flow
e. Hypervolemia

Copyright 2011. Wolters Kluwer Health | Lippincott Williams & Wilkins. Study Guide for Porths Essentials of Pathophysiology, Third Edition.
LWBK707-c20_p112-117.qxd 8/19/10 1:15 PM Page 117 Aptara Inc

CHAPTER 20 HEART FAILURE AND CIRCULATORY SHOCK 117

4. What are the physiologic signs and 8. Sepsis is growing in incidence in the United
symptoms of cardiogenic shock? (Mark all States. Its pathogenesis includes neutrophil
that apply.) activation, which kills microorganisms. Neu-
a. Decrease in mean arterial blood pressures trophils also injure the endothelium,
releasing mediators that increase vascular per-
b. Increased urine output related to increased
meability. What else do neutrophils do in
renal perfusion
sepsis?
c. Rise in central venous pressure (CVP)
a. Releases nitric oxide
d. Hypercapnic lips and nail beds
b. Vasoconstricts the capillary bed
e. Increased extraction of O2 from hemoglobin
c. Causes bradycardia
5. In hypovolemic shock the main purpose of d. Activates erythropoiesis
treatment is correcting or controlling the
underlying cause of the hypovolemia and 9. What is the primary physiologic result of
improving the perfusion of the tissues and obstructive shock?
organs of the body. Which of the following a. Left ventricular hypertrophy
treatments is not a primary form of therapy b. Elevated right heart pressure
for hypovolemic shock?
c. Right atrial hypertrophy
a. Surgery
d. Decreased right heart pressure
b. Administration of intravenous fluids and
blood 10. An important factor in the mortality of severe
shock is acute renal failure. What is the
c. Vasoconstrictive drugs
degree of renal damage related to in shock?
d. Infusion of blood and blood products
a. Loss of perfusion and duration of shock
6. Neurogenic shock, or spinal shock, is a b. Loss of perfusion and degree of immune-
phenomenon caused by the inability of the mediated response
vasomotor center in the brain stem to control
c. Severity and duration of shock
blood vessel tone through the sympathetic
outflow to the blood vessels. In neurogenic d. Severity of shock and degree of immune-
shock, what happens to the heart rate and mediated response
the skin? 11. The pathogenesis of multiorgan dysfunction
a. Heart rate slower than normal; skin warm syndrome (MODS) is not clearly understood
and dry at this time. Supportive management is
b. Heart rate faster than normal; skin cool currently the focus of treatment in this disor-
and moist der. What is not a major risk factor in MODS?
c. Heart rate slower than normal; skin cool a. Advanced age
and moist b. Alcohol abuse
d. Heart rate slower than normal; skin warm c. Respiratory dysfunction
and dry d. Infarcted bowel
7. Anaphylactic shock is the most severe form 12. What is the primary cause of heart failure in
of systemic allergic reaction. Immunologically infants and children?
medicated substances are released into the
a. Idiopathic heart disease
blood, causing vasodilation and an increase
in capillary permeability. What physiologic b. Structural heart defects
response often accompany the vascular c. Hyperkalemia
response in anaphylaxis? d. Reactions to medications
a. Uterine smooth muscle relaxation
b. Laryngeal edema
c. Bronchodilation
d. Gastrointestinal relaxation

Copyright 2011. Wolters Kluwer Health | Lippincott Williams & Wilkins. Study Guide for Porths Essentials of Pathophysiology, Third Edition.
LWBK707-c21_p118-123.qxd 8/20/10 11:54 PM Page 118 Aptara

21
CHAPTER
Control of Respiratory
System

SECTION I: LEARNING 9. Use the law of Laplace to explain the need


for surfactant in maintaining the inflation of
OBJECTIVES small alveoli.
1. State the difference between the conducting 10. Differentiate between the determinants of
and the respiratory airways. airway resistance and lung compliance and
their effect on the work of breathing.
2. Trace the movement of air through the
airways, beginning in the nose and orophar- 11. Define inspiratory reserve, expiratory reserve,
ynx and moving into the respiratory tissues vital capacity, residual lung volume, and
of the lung. FEV1.0.
3. Describe the function of the mucociliary 12. Trace the exchange of gases between the air
blanket. in the alveoli and the blood in the
pulmonary capillaries.
4. Compare the supporting structures of the
large and small airways in terms of cartilagi- 13. Differentiate between pulmonary and alveo-
nous and smooth muscle support. lar ventilation.
5. State the function of the two types of alveo- 14. Explain why ventilation and perfusion must
lar cells. be matched.
6. Differentiate the function of the bronchial 15. Cite the difference between dead air space
and pulmonary circulations that supply the and shunt.
lungs.
16. List four factors that affect the diffusion of
7. Describe the basic properties of gases in rela- gases in the alveoli.
tion to their partial pressures and their pres-
17. Explain the difference between PO2 and
sures in relation to volume and temperature.
hemoglobin-bound oxygen and O2
8. State the definition of intrathoracic, saturation, and oxygen content.
intrapleural, and intra-alveolar pressures, and
18. Explain the significance of a shift to the right
state how each of these pressures changes in
and a shift to the left in the oxygen-
relation to atmospheric pressure during
hemoglobin dissociation curve.
inspiration and expiration.

118
LWBK707-c21_p118-123.qxd 8/20/10 11:54 PM Page 119 Aptara

CHAPTER 21 CONTROL OF RESPIRATORY SYSTEM 119

19. Compare the neural control of the 8. Each primary bronchus, accompanied by the
respiratory muscles, which control breathing, pulmonary arteries, veins, and lymph vessels,
with that of cardiac muscle, which controls enters the lung through a slit called the
the pumping action of the heart. .
20. Describe the function of the chemoreceptors 9. Each is supplied by a branch of
and lung receptors in the regulation of venti- a terminal bronchiole, an arteriole, the
lation. pulmonary capillaries, and a venule.
21. Trace the integration of the cough reflex 10. The are the terminal air spaces
from stimulus to explosive expulsion of air of the respiratory tract and the actual sites of
that constitutes the cough. gas exchange between the air and the blood.
22. Define dyspnea and list three types of condi- 11. The pulmonary circulation arises from the
tions in which dyspnea occurs. artery and provides for the gas
exchange function of the lungs.
12. Particulate matter entering the lung is partly
removed by vessels, as are the
SECTION II: ASSESSING plasma proteins that have escaped from the
YOUR UNDERSTANDING pulmonary capillaries.

Activity A Fill in the blanks. 13. It is stimulation, through the


vagus nerve, that is responsible for the
1. The primary function of the respiratory slightly constricted smooth muscle tone in
system is . the normal resting lung.
2. Functionally, the respiratory system can be 14. Stimulation of the nervous
divided into two parts: the system causes airway relaxation, blood vessel
airways, through which air moves as it passes constriction, and inhibition of glandular
between the atmosphere and the lungs, and secretion.
the tissues of the lungs, where
gas exchange takes place. 15. The pressure exerted by a single gas in a mix-
ture is called the .
3. The airways consist of the nasal
passages, mouth and pharynx, larynx, 16. Air moves between the atmosphere and the
trachea, bronchi, and bronchioles. lungs because of a .

4. The air we breathe is , 17. The pressure in the pleural cavity is called the
, and as it moves pressure.
through the conducting airways. 18. The maneuver is used to study
5. The produced by the epithelial the cardiovascular effects of increased
cells in the conducting airways forms a layer intrathoracic pressure on peripheral venous
that protects the respiratory system by pressures, cardiac filling and cardiac output,
entrapping dust, bacteria, and other foreign as well as poststrain heart rate and blood
particles that enter the airways. pressure responses.

6. The vocal folds and the elongated opening 19. Lung refers to the ease with
between them are called the . which the lungs can be inflated.

7. The walls of the trachea are supported by 20. The is the volume of air
horseshoe- or C-shaped rings of inspired (or exhaled) with each breath.
cartilage, which prevent it from collapsing 21. The maximum amount of air that can be
when the pressure in the thorax becomes inspired in excess of the normal tidal volume
negative. (TV) is called the , and the max-
imum amount that can be exhaled in excess
of the normal TV is the .

Copyright 2011. Wolters Kluwer Health | Lippincott Williams & Wilkins. Study Guide for Porths Essentials of Pathophysiology, Third Edition.
LWBK707-c21_p118-123.qxd 8/20/10 11:54 PM Page 120 Aptara

120 UNIT 6 RESPIRATORY FUNCTION

22. The is the amount of air a per- 36. The content in the blood regu-
son can breathe in beginning at the normal lates ventilation through its effect on the pH
expiratory level and distending the lungs to of the extracellular fluid of the brain.
the maximal amount.
37. is a subjective sensation or a
23. The equals the IRV plus the TV persons perception of difficulty in breathing
plus the ERV and is the amount of air that that includes the perception of labored
can be exhaled from the point of maximal breathing and the reaction to that sensation.
inspiration.
Activity B Consider the following figure.
24. The is the amount of air that is
exchanged in 1 minute.
25. ventilation refers to the total
exchange of gases between the atmosphere
and the lungs; ventilation is
the exchange of gases within the gas
exchange portion of the lungs.
26. Even at low lung volumes, some air remains
in the alveoli of the lower portion of the
lungs, preventing their .
27. refers to the air that is moved
with each breath but does not participate in
gas exchange.
28. Both dead air space and shunt produce a
of ventilation and perfusion.
29. Although the lungs are responsible for the
exchange of gases with the external environ- In the figure of the respiratory system, label the
ment, the transports gases following structures:
between the lungs and body tissues. Secondary bronchi
30. carries about 98% to 99% of Tracheal cartilage
oxygen in the blood and is the main Left primary bronchus
transporter of oxygen.
Terminal bronchioles
31. Oxygen binds with the heme Segmental bronchi
groups on the hemoglobin molecule.
32. Hemoglobins affinity for oxygen is Activity C Match the key terms in Column A
influenced by , with their definitions in Column B.
concentration, and body . Column A Column B
33. Carbon dioxide is transported in the blood in 1. Mediastinum a. Mucus lining of the
three forms: as (10%), attached conducting airways
to (30%), and as 2. Elastic
recoil b. Form part of
(60%).
respiratory
34. The pacemaker properties of the respiratory 3. Epiglottis membrane
center result from the cycling of the two 4. Type I c. Pressure inside the
groups of respiratory neurons: the pneumocytes airways and alveoli
center in the upper pons and
5. Angiogenesis d. The trachea,
the center in the lower pons.
bronchi, and bron-
35. The automatic regulation of ventilation is 6. Mucociliary chioles
controlled by input from two types of sensors blanket
e. Synthesize
or receptors: and 7. Alveolar pulmonary
receptors. pressure surfactant

Copyright 2011. Wolters Kluwer Health | Lippincott Williams & Wilkins. Study Guide for Porths Essentials of Pathophysiology, Third Edition.
LWBK707-c21_p118-123.qxd 8/20/10 11:54 PM Page 121 Aptara

CHAPTER 21 CONTROL OF RESPIRATORY SYSTEM 121

8. Brush cells f. Space between 4. What is the mathematical formula used to


lungs that contains describe the diffusion of gas across the respira-
9. Tracheobron-
heart, blood vessels, tory membrane?
chial
lymph nodes,
10. Type II nerves and the
pneumocytes esophagus
g. The ability of the
elastic components
of the lung to recoil 5. In the clinic, what type of blood is used for
to their original blood gas measurements and why?
position
h. Routes liquids and
foods into the
esophagus
i. Formation of new 6. What causes us to cough?
blood vessels
j. Act as receptors that
monitor the air
quality of the lungs

Activity D Put these respiratory structures in


anatomic order.
SECTION III: APPLYING
a. Nasopharynx
YOUR KNOWLEDGE
b. Trachea
c. Epiglottis Activity F Consider the scenario and answer
d. Alveoli the questions.
e. Respiratory bronchiole Seventy-nine-year old Mr. Borden is brought to
f. Intrapulmonary bronchus the clinic by his daughter who says, I am
worried about him. He is so stubborn, he just
g. Extrapulmonary bronchus
wont complain. When he walks, he gets so short
of breath. I dont think he is getting enough oxy-
Activity E Briefly answer the following. gen! Mr. Bordens O2 level is 87% and his nail
beds are dusky with a delayed capillary refill
1. Describe the pleura and explain its function.
time. There is no clubbing to Mr. Bordens
fingertips.
1. How would the nurse explain generalized
hypoxia to Mr. Bordens daughter?

2. Describe the events of the respiratory cycle.

2. What diagnostic tests would the doctor order


to confirm a diagnosis of generalized hypoxia?
3. What is the function of pulmonary
surfactant?

Copyright 2011. Wolters Kluwer Health | Lippincott Williams & Wilkins. Study Guide for Porths Essentials of Pathophysiology, Third Edition.
LWBK707-c21_p118-123.qxd 8/20/10 11:54 PM Page 122 Aptara

122 UNIT 6 RESPIRATORY FUNCTION

SECTION IV: PRACTICING 5. An 82-year-old man with chronic obstructive


pulmonary disease (COPD) is at the clinic for
FOR NCLEX a regular check-up. Because of his diagnosis,
the nurse would expect his respiratory rate
Activity G Answer the following questions.
under normal circumstances to be what?
1. The lungs are the working structures of the a. Tachypneic
respiratory system and they have several
b. 18 to 20 bpm
functions. What are the functions of the
lungs? (Mark all that apply.) c. 18 to 20 bpm
a. Produce heparin d. Hyperpneic
b. Activate vasoactive substances 6. Our ability to oxygenate the tissues and
c. Convert angiotensin I to angiotensin II organs of our bodies depends on our ability
to ventilate, or exchange, gases in our respira-
d. Activate bradykinin
tory system. The resultant distribution of
e. Convert glucose to glycogen ventilation or the areas of the body open to
2. Bronchial blood vessels have several the exchange of gases in our respiratory
functions. They warm and humidify incom- system depends on what?
ing air as well as distribute blood to the con- a. Effects of gravity intrathoracic pressure
ducting airways and the supporting structures b. Body position and alveolar pressure
of the lung. What is it that makes bronchial
c. Effects of gravity and body position
blood vessels unique in the body?
d. Intrathoracic pressure and alveolar
a. They can undergo angiogenesis
pressure
b. They drain blood into the bronchiole
arteries 7. Alveolar oxygen levels directly impact the
blood vessels in the pulmonary circulation.
c. They participate in gas exchange
In a person with lung disease, there is
d. They carry oxygenated blood to the lung vasoconstriction throughout the lung,
tissues causing a generalized hypoxia. What can
3. Match the respiratory pressures with their prolonged hypoxia lead to?
definitions. a. Hypertension and increased workload on
the left heart
Pressure Definition
b. Pulmonary hypertension and left ventricu-
1. Alveolar pressure a. The pressure in the lar hypertrophy
thoracic cavity
2. Intrapleural c. Hypertension and increased workload on
pressure b. Pressure inside the the right heart
airways and alveoli
3. Transpulmonary d. Pulmonary hypertension and increased
of the lungs
pressure workload on the right heart.
c. The difference
4. Intrathoracic between the intra- 8. When there is a mismatching of ventilation
pressure alveoli and and perfusion within the lung itself,
intrapleural insufficient ventilation occurs. There is a lack
pressures. of enough oxygen to adequately oxygenate
the blood flowing through the alveolar capil-
d. Pressure in the pleu-
laries, creating a physiologic shunt. What
ral cavity
causes a physiologic right-to-left shunting of
4. What does the equation C  V/P stand blood in the respiratory system?
for? a. Destructive lung disease or heart failure
a. Surface tension inside the lungs b. Obstructive lung disease or heart failure
b. Lung compliance c. Heart failure or pulmonary hypertension
c. Airway resistance d. Heart failure or regional hypoxia
d. Change in peak expiratory flow

Copyright 2011. Wolters Kluwer Health | Lippincott Williams & Wilkins. Study Guide for Porths Essentials of Pathophysiology, Third Edition.
LWBK707-c21_p118-123.qxd 8/20/10 11:54 PM Page 123 Aptara

CHAPTER 21 CONTROL OF RESPIRATORY SYSTEM 123

9. Blood transports both oxygen and carbon 11. There are several actions the body makes to
dioxide in a physically dissolved form to the initiate a cough. Put these actions into the
tissues and organs of the body. It is the mea- correct order.
surements of the components of the gases in a. Elevation of intrathoracic pressures
the blood that are used as indicators of the
b. Rapid opening of glottis
bodys status by health care workers. Why is
it commonly the blood in the arteries that is c. Closure of glottis
measured for its components rather than the d. Rapid inspiration of large volume of air
blood in the veins? e. Forceful contraction of abdominal and
a. Arterial blood most adequately measures expiratory muscles
the metabolic demands of the tissues
12. Dyspnea is defined as an uncomfortable sen-
along with the gas exchange function of
sation or difficulty in breathing that is subjec-
the lungs.
tively defined by the client. Which of the
b. Venous blood measures the metabolic de- following disease states is not characterized by
mands of the tissues rather than the gas dyspnea?
exchange function of the lungs.
a. Pneumonia
c. Arterial blood only measures the gas ex-
b. Emphysema
change function of the lung after it has
met the metabolic demands of the tissues. c. Myasthenia gravis
d. Venous blood only measures the hypoxic d. Multiple sclerosis
reflex of the body, not the gas exchange
function of the lungs.
10. Respiration has both automatic and
voluntary components that are sent to the
respiratory center of the brain from a number
of sources. What physiologic forces can exert
their influence on respiration through the
lower brain centers? (Mark all that apply.)
a. Fever
b. Cold
c. Pain
d. Endorphins
e. Emotion

Copyright 2011. Wolters Kluwer Health | Lippincott Williams & Wilkins. Study Guide for Porths Essentials of Pathophysiology, Third Edition.
LWBK707-c22_p124-129.qxd 8/19/10 1:17 PM Page 124 Aptara Inc

22
CHAPTER
Respiratory Tract
Infections, Neoplasms,
and Childhood
Disorders

SECTION I: LEARNING 8. Compare small cell lung cancer and


nonsmall cell lung cancer in terms of
OBJECTIVES histopathology, prognosis, and treatment
methods.
1. Describe the transmission of the common
cold from one person to another. 9. Describe the manifestations of lung cancer
and list two symptoms of lung cancer that
2. Describe the causes, manifestations, and
are related to the invasion of the
treatment of acute and chronic sinusitis.
mediastinum.
3. Relate the characteristics of the influenza
10. Define the term paraneoplastic and cite three
virus to its contagious properties and the
paraneoplastic manifestations of lung cancer.
need for a yearly flu shot.
11. Characterize the effect of age on treatment of
4. Characterize community-acquired pneumo-
lung cancer.
nia, hospital-acquired pneumonia, and pneu-
monia in immunocompromised persons in 12. Trace the development of the respiratory
terms of pathogens, manifestations, and tract through the five stages of embryonic
prognosis. and fetal development.
5. Describe the immunologic properties of the 13. Cite the function of surfactant in lung func-
tubercle bacillus, and differentiate between tion in the neonate.
primary tuberculosis and reactivated tubercu-
14. Cite the possible cause and manifestations of
losis on the basis of their pathophysiology.
respiratory distress syndrome and
6. State the mechanism for the transmission of bronchopulmonary dysplasia.
fungal infections of the lung.
15. Describe the physiologic basis for sternal and
7. Cite risk factors associated with lung cancer. chest wall retractions and grunting, stridor,
and wheezing as signs of respiratory distress
in infants and small children.

124
LWBK707-c22_p124-129.qxd 8/19/10 1:17 PM Page 125 Aptara Inc

CHAPTER 22 RESPIRATORY TRACT INFECTIONS, NEOPLASMS, AND CHILDHOOD DISORDERS 125

16. Compare croup, epiglottitis, and bronchioli- 12. The term describes inflammation
tis in terms of incidence by age, site of infec- of parenchymal structures of the lung, such as
tion, and signs and symptoms. the alveoli and the bronchioles.
17. List the signs of impending respiratory failure 13. refers to consolidation of a part
in small children. or all of a lung lobe; and signi-
fies a patchy consolidation involving more
than one lobe.
14. Hospital-acquired, or , pneumo-
SECTION II: ASSESSING nia is defined as a lower respiratory tract
YOUR UNDERSTANDING infection that was not present or incubating
on admission to the hospital.
Activity A Fill in the blanks.
15. The term host usually is
1. are the most frequent cause of applied to persons with a variety of underly-
respiratory tract infections. ing defects in host defenses.
2. Viral infections can damage 16. disease is a form of bronchop-
epithelium, airways, and lead neumonia; infection normally occurs by
to secondary infections. acquiring the organism from the
3. The common cold is a viral infection of the environment.
respiratory tract. 17. The primary atypical pneumonias are caused
4. Outbreaks of colds due to are by a variety agents, the most common being
most common in early fall and late spring. pneumonia.

5. are popular over-the-counter 18. is the worlds foremost cause of


treatments for colds because of their action death from a single infectious agent.
in drying nasal secretions. 19. Mycobacteria are similar to other bacterial
6. refers to inflammation of the organisms except for an outer
nasal passages, and sinusitis as inflammation that makes them more resistant to
of the sinuses. destruction.

7. The lower content in the 20. tuberculosis is a form of the


sinuses facilitates the growth of organisms, disease that develops in previously
impairs local defenses, and alters the unexposed, and therefore unsensitized,
function of immune cells. persons.

8. Host antibodies to and 21. The most frequently used screening methods
prevent or ameliorate infection for pulmonary tuberculosis are the
by the influenza virus. tests and chest .

9. The influenza viruses can cause three types of 22. is caused by the dimorphic fun-
infections: an uncomplicated gus Histoplasma capsulatum and is one of the
respiratory infection, pneumo- most common fungal infections in the
nia, and a respiratory viral infection followed United States.
by a infection. 23. respiratory infections produce
10. Because influenza is so highly contagious, pulmonary manifestations that resemble
prevention relies primarily on . tuberculosis.

11. Avian strains of the influenza virus do not 24. The number of Americans who develop lung
usually cause outbreaks of disease in humans cancer is decreasing, primarily because of a
unless a of the virus genome decrease in .
has occurred within an intermediate 25. Cigarette smoking causes more than
mammalian host such as a pig. of cases of lung cancer.

Copyright 2011. Wolters Kluwer Health | Lippincott Williams & Wilkins. Study Guide for Porths Essentials of Pathophysiology, Third Edition.
LWBK707-c22_p124-129.qxd 8/19/10 1:17 PM Page 126 Aptara Inc

126 UNIT 6 RESPIRATORY FUNCTION

26. are aggressive, locally invasive, Maxillary sinus


and widely metastatic tumors that arise from Superior turbinate
the epithelial lining of major bronchi.
Middle turbinate
27. The are small, round to oval Inferior turbinate
cells that are approximately the size of a lym-
phocyte and grow in clusters that exhibit nei- Activity C Match the key terms in Column A
ther glandular nor squamous organization. with their definitions in Column B.
28. The include squamous cell car- Column A Column B
cinomas, adenocarcinomas, and large cell
carcinomas. 1. SCLCs a. Audible crowing
sound during inspi-
29. is characterized by inspiratory 2. Typical
ration
stridor, hoarseness, and a barking cough. pneumonias
b. False-negative
30. By the weeks of gestation, suffi- 3. Stridor tuberculin skin tests
cient terminal air sacs are present to permit 4. Anergy c. Result from
survival of the premature infant. infection by bacteria
5. Hemagglutinin
d. Attachment protein
Activity B Consider the following figures. 6. Squamous cell that allows the
7. Paraneoplastic influenza virus to
syndrome enter epithelial cells
in the respiratory
8. Atypical tract
pneumonias
e. Symptoms that
9. Neuraminidase develop when sub-
stances released by
10. Reye syndrome
some cancer cells
disrupt the normal
function
f. Viral and
mycoplasma infec-
tion
g. Facilitates influenza
viral replication and
release from the cell
h. Fatty liver with
encephalitis
Sphenoidal i. Highly aggressive
sinus
lung cancer
j. Carcinoma is associ-
ated with the para-
neoplastic
syndromes that pro-
duce hypercalcemia

In the figures above, label the following


structures:
Frontal sinus
Ethmoid sinuses

Copyright 2011. Wolters Kluwer Health | Lippincott Williams & Wilkins. Study Guide for Porths Essentials of Pathophysiology, Third Edition.
LWBK707-c22_p124-129.qxd 8/19/10 1:17 PM Page 127 Aptara Inc

CHAPTER 22 RESPIRATORY TRACT INFECTIONS, NEOPLASMS, AND CHILDHOOD DISORDERS 127

Activity D 2. How does the influenza virus reinfect some-


one? How is it so contagious?

Inhalation of
tubercle bacillus

3. What is a common complication of influenza


(usually of the elderly or those with
cardiopulmonary disease)?

Primary Secondary
tuberculosis tuberculosis

Development of
cell-mediated
4. What type of pneumonia results from inhala-
immunity tion or aspiration of nasopharyngeal
secretions during sleep?

Positive skin
test

5. What are the pathophysiologic stages of


pneumococcal pneumonia infection?

6. How is Mycobacterium tuberculosis hominis


Use the following terms to complete the spread?
flowchart above:
Reinfection
Ghon complex
Granulomatous inflammatory response
Healed dormant lesion 7. Describe the pathogenic mechanisms of
Cell-mediated hypersensitivity response M. tuberculosis hominis.
Reactivated tuberculosis
Progressive or disseminated tuberculosis

Activity E Briefly answer the following.


1. How is the cold virus spread? 8. How is lung cancer categorized?

Copyright 2011. Wolters Kluwer Health | Lippincott Williams & Wilkins. Study Guide for Porths Essentials of Pathophysiology, Third Edition.
LWBK707-c22_p124-129.qxd 8/19/10 1:17 PM Page 128 Aptara Inc

128 UNIT 6 RESPIRATORY FUNCTION

9. What causes the varied manifestations of SECTION IV: PRACTICING


lung cancer?
FOR NCLEX
Activity G Answer the following questions.
1. A 23-year-old woman goes to the drug store
to buy a medication to ease the symptoms of
10. What is the result of the absence of her cold. Her friends have told her to buy a
surfactant in premature infants? medication with an antihistamine in it to
help dry up her runny nose and make it eas-
ier to breath. The woman talks with the phar-
macist, who has known her many years. The
pharmacist recommends that this young
woman not buy a cold medication with a
decongestant in it. Why would he do that?
a. Client has history of hyperthyroidism
SECTION III: APPLYING b. Client has history of hypotension
YOUR KNOWLEDGE c. Client has history of type I diabetes melli-
tus
Activity F Consider the scenario and answer
the questions. d. Client has history of juvenile rheumatoid
arthritis
Mr. Jones, who is 68 years old, presents to the
clinic with lack of appetite and weight loss of 2. The early stages of influenza pass by as if the
30 pounds over the past 6 months. He has a his- infection were any other viral infection.
tory of a chronic, nonproductive cough; shortness What is the distinguishing feature of an
of breath, which is worse on exertion; and wheez- influenza viral infection that makes it differ-
ing. He tells the nurse that he is now coughing ent from other viral infections?
up bloody stuff, and he wants to know what is a. Slow onset of upper respiratory symptoms
wrong with him. When asked about pain he says, b. Rapid onset of profound malaise
I get heartburn once in awhile, but the pain is
c. Slow onset of fever and chills
dull instead of burning. Routine laboratory
work is ordered and the only abnormal finding is d. Rapid onset of productive cough
hypercalcemia. The suspected diagnosis is 3. Influenza A subtype H5N1 has been
squamous cell cancer of the lung. documented in poultry in both East and
1. What diagnostic tests would the nurse expect Southeast Asian Countries. This form of
to be ordered? Avian flu (bird flu) is highly contagious from
bird to bird, but rarely is passed from human
to human. There is a large amount of
concern that the H5N1 strain might mutate,
making it easier to be passed from human to
human, carrying with it a high mortality rate.
2. Mr. Jones wants to know how his cancer will What is the main concern if the H5N1 strain
be treated. The nurse knows that treatments does mutate?
are available. Which treatments are used for a. An epidemic in Southeast Asia.
squamous cell (NSCLC) cancer of the lung? b. Inability to develop a vaccine for the
newly infected poultry
c. Initiation of a pandemic
d. Several small pockets of infection so wide-
spread it will be difficult to control them

Copyright 2011. Wolters Kluwer Health | Lippincott Williams & Wilkins. Study Guide for Porths Essentials of Pathophysiology, Third Edition.
LWBK707-c22_p124-129.qxd 8/19/10 1:17 PM Page 129 Aptara Inc

CHAPTER 22 RESPIRATORY TRACT INFECTIONS, NEOPLASMS, AND CHILDHOOD DISORDERS 129

4. Community-acquired pneumonia can be cat- 8. Coccidioidomycosis is a pulmonary fungal


egorized according to several indexes. What infection resembling tuberculosis. Less severe
are these indexes? (Mark all that apply.) forms of the infection are treated with oral
a. Radiologic findings antifungal medications. For persons with
progressive disease, what is the drug of
b. Serologic findings
choice?
c. Age
a. IV fluconazole
d. Presence of coexisting disease
b. IV BCG
e. Need for hospitalization in long-term care
c. IV amphotericin B
facility
d. IV rifampin
5. An immunocompromised host is open to
pneumonia from all types of organisms. 9. Nonsmall cell lung cancers (NSCLCs) mimic
There is, however, a correlation between Small cell lung cancers (SCLCs) through their
specific types of immunologic deficits and abilities to do what?
specific invading organisms. What organism a. Synthesize bioactive products and produce
is most likely to cause pneumonia in an pan-neoplastic syndromes
immunocompromised host with neutropenia b. Neutralize bioactive products, which
and impaired granulocyte function? produce paraneoplastic syndromes
a. -Hemolytic streptococcus c. Produce paraneoplastic syndromes and
b. Gram-positive bacilli synthesize adrenocorticotropic hormone
c. Eosinophilic bacillus subtilis (ACTH)
d. Haemophilus influenza d. Synthesize bioactive products and produce
paraneoplastic syndromes
e. Staphylococcus aureus
10. Premature infants who are treated with
6. Elderly people are very susceptible to
mechanical ventilation, mostly for
pneumonia in all its varieties. The symptoms
respiratory distress syndrome, are at risk for
the elderly exhibit can be very different than
developing bronchopulmonary dysplasia
those of other age groups who have pneumo-
(BPD), a chronic lung disease. What are the
nia. What signs and symptoms are elderly
signs and symptoms of BPD?
people with pneumonia less likely to experi-
ence than people with pneumonia in other a. Rapid and shallow breathing and chest
age groups? retractions
a. Marked elevation in temperature b. Weight loss and a barrel chest
b. Loss of appetite c. Tachycardia and slow shallow breathing
c. Deterioration in mental status d. A barrel chest and rapid weight gain
d. Pleuritic pain 11. For each of the following conditions, identify
where it occurs in the respiratory tract of
7. Tuberculosis is a highly destructive disease
children: upper airway or lower airway.
because the tubercle bacillus activates a tissue
hypersensitivity to the tubercular antigens. Epiglottitis
What does the destructive nature of tubercu- Acute bronchiolitis
losis cause in a previously unexposed Asthma
immunocompetent person?
Spasmodic croup
a. Cavitation and rapidly progressing pul-
Laryngotracheobronchitis
monary lesions
b. Caseating necrosis and cavitation 12. What is the underlying cause of respiratory
failure in a child with bronchiolitis?
c. Rapidly progressing lesions and purulent
necrosis a. Obstructive process
d. Caseating necrosis and purulent pul- b. Impaired gas exchange
monary lesions c. Hypoxemia and hypercapnia
d. Metabolic acidosis

Copyright 2011. Wolters Kluwer Health | Lippincott Williams & Wilkins. Study Guide for Porths Essentials of Pathophysiology, Third Edition.
LWBK707-c23_p130-136.qxd 8/19/10 2:24 PM Page 130 Aptara Inc

23
CHAPTER
Disorders of Ventilation
and Gas Exchange

SECTION I: LEARNING 10. Characterize the acute- or early-phase and


late-phase responses in the pathogenesis of
OBJECTIVES bronchial asthma and relate them to current
methods for treatment of the disorder.
1. Define the terms hypoxemia and hypercapnia.
11. Explain the distinction between chronic
2. Characterize the mechanisms whereby disor-
bronchitis and emphysema in terms of
ders of ventilation and diffusion cause
pathology and clinical manifestations.
hypoxemia and hypercapnia.
12. State the chief manifestations of bronchiecta-
3. Compare the manifestations of hypoxemia
sis.
and hypercapnia.
13. Describe the genetic abnormality responsible
4. Characterize the pathogenesis and manifesta-
for cystic fibrosis and relate it to the manifes-
tions of transudative and exudative pleural
tations of the disorder.
effusion, chylothorax, and hemothorax.
14. State the difference between chronic obstruc-
5. Differentiate among the causes and manifes-
tive pulmonary diseases and chronic restric-
tations of spontaneous pneumothorax,
tive lung diseases in terms of their pathology
secondary pneumothorax, and tension
and manifestations.
pneumothorax.
15. Describe the causes of hypersensitivity pneu-
6. Describe the causes of pleuritis and differenti-
monitis.
ate the characteristics of pleural pain from
other types of chest pain. 16. Characterize the organ involvement in
sarcoidosis.
7. Describe the causes and manifestations of
atelectasis. 17. State the most common cause of pulmonary
embolism and the clinical manifestations of
8. Describe the physiology of bronchial smooth
the disorder.
muscle as it relates to airway disease.
18. Describe the pathophysiology of pulmonary
9. Describe the interaction between heredity,
arterial hypertension and state three causes
alterations in the immune response, and
of secondary pulmonary hypertension.
environmental agents in the pathogenesis of
bronchial asthma.

130
LWBK707-c23_p130-136.qxd 8/19/10 2:24 PM Page 131 Aptara Inc

CHAPTER 23 DISORDERS OF VENTILATION AND GAS EXCHANGE 131

19. Describe the alterations in cardiovascular 8. Elevated levels of PCO2 produce a decrease in
function that are characteristic of cor and respiratory .
pulmonale.
9. refers to an abnormal
20. Describe the pathologic lung changes that collection of fluid in the pleural cavity.
occur in acute respiratory distress syndrome
10. is a specific type of pleural
and relate them to the clinical manifestations
effusion in which there is blood in the
of a general definition of respiratory failure.
pleural cavity.
21. Differentiate between the causes and
11. Primary atelectasis of the newborn implies
manifestations of hypoxemic and hyper-
that the lung has never been .
capnic/hypoxemic respiratory failure.
12. Obstructive airway disorders are caused by
22. Describe the treatment of respiratory failure.
disorders that limit airflow.
13. Bronchial is a chronic disorder
of the airways that causes episodes of airway
SECTION II: ASSESSING YOUR obstruction, bronchial hyperresponsiveness,
UNDERSTANDING and airway inflammation that are usually
reversible.
Activity A Fill in the blanks. 14. Recent research has focused on the role of
1. The primary function of the respiratory in the pathogenesis of
system is to remove appropriate amounts of bronchial asthma.
from the blood entering the 15. pulmonary disease (COPD) is
pulmonary circulation and to add adequate characterized by chronic and recurrent
amounts of to the blood obstruction of airflow in the pulmonary
leaving the pulmonary circulation. airways.
2. involves the movement of 16. In COPD, and of
fresh atmospheric air to the alveoli for deliv- the bronchial wall, along with excess mucus
ery provision of O2 and removal of CO2. secretion, obstruct airflow and cause
3. As a general rule, of the blood mismatching of ventilation and perfusion.
primarily depends on factors that promote 17. is thought to result from the
diffusion of O2 from the alveoli into the pul- breakdown of elastin and other alveolar
monary capillaries; whereas, wall components by enzymes, called
primarily depends on the minute ventilation , that digest proteins.
and elimination of CO2 from the alveoli.
18. A hereditary deficiency in
4. refers to a reduction in blood accounts for approximately 1% of all cases
O2 levels. of COPD and is more common in young per-
5. Hypoxemia produces its effects through sons with emphysema.
tissue and the compensatory 19. The earliest feature of chronic bronchitis is
mechanisms that the body uses to adapt to in the large airways, associated
the lowered oxygen level. with hypertrophy of the submucosal glands
6. The body compensates for chronic in the trachea and bronchi.
hypoxemia by increased , 20. Persons with predominant emphysema are
pulmonary , and increased classically referred to as ,a
production of cells. reference to the lack of cyanosis, the use of
7. can occur in a number of disor- accessory muscles, and pursed-lip
ders that cause hypoventilation or mismatch- breathing.
ing of ventilation and perfusion resulting in
increased arterial CO2.

Copyright 2011. Wolters Kluwer Health | Lippincott Williams & Wilkins. Study Guide for Porths Essentials of Pathophysiology, Third Edition.
LWBK707-c23_p130-136.qxd 8/19/10 2:24 PM Page 132 Aptara Inc

132 UNIT 6 RESPIRATORY FUNCTION

21. Persons with a clinical syndrome of chronic Activity B Consider the following figure.
bronchitis are classically labeled
, a reference to cyanosis and
fluid retention associated with right-sided
heart failure. Smoking

22. is a permanent dilation of


the bronchi and bronchioles caused by
destruction of the muscle and elastic
supporting tissue resulting from a vicious
cycle of infection and inflammation.
23. is an autosomal recessive disor-
der involving fluid secretion in the exocrine
glands in the epithelial lining of the respira-
tory, gastrointestinal, and reproductive tracts.
24. The diffuse diseases are a Macrophages
diverse group of lung disorders that produce and neutrophils
similar inflammatory and fibrotic changes in
the interalveolar septa of the lung.
25. The interstitial lung disorders exert their Emphysema
effects on the and
connective tissue found between the delicate
interstitium of the alveolar walls.
Complete the above flowchart using the items
26. Pulmonary develops when a below:
blood-borne substance lodges in a branch of
the pulmonary artery and obstructs the flow, Destruction of elastic fibers in lung
almost all of which are thrombi that arise Decreased 1-antitrypsin activity
from deep vein thrombosis. Action inhibited by 1-antitrypsin
27. Chest pain, dyspnea, and increased Inherited 1-antitrypsin deficiency
respiratory rate are the most frequent signs Release of elastase
and symptoms of .
Attraction of inflammatory cells
28. is a disorder characterized by
an elevation of pressure within the Activity C Match the key terms in Column A
pulmonary circulation, namely the with their definitions in Column B.
pulmonary arterial system.
1.
29. Continued exposure of the pulmonary
Column A Column B
vessels to is a common cause of
pulmonary hypertension. 1. Ventilation a. Ratio of carbon
dioxide production
30. can be viewed as a failure in 2. PF ratio
to oxygen
the gas exchange due either to pump or lung
3. Cyanosis consumption
failure, or both.
4. Respiratory b. Difference between
quotient arterial PO2 and the
fraction of inspired
5. Empyema oxygen
6. Hypercapnia c. Infection of the
pleura
7. Venous oxygen
saturation d. Movement of gas
into or out of lungs
8. Pneumothorax

Copyright 2011. Wolters Kluwer Health | Lippincott Williams & Wilkins. Study Guide for Porths Essentials of Pathophysiology, Third Edition.
LWBK707-c23_p130-136.qxd 8/19/10 2:24 PM Page 133 Aptara Inc

CHAPTER 23 DISORDERS OF VENTILATION AND GAS EXCHANGE 133

9. Hypoxemia e. Increase in the i. Acute respiratory


carbon dioxide distress syndrome
10. Pleuritis
content of the arte- j. Areas of the lung are
rial blood ventilated but not
f. Reflects the bodys perfused, or when
extraction and areas are perfused
utilization of O2 at but not ventilated
the tissue levels
g. Air in pleural space Activity D Put the events of IgE-mediated
asthma reaction in order in the boxes below:
h. Decreased oxygena-
tion a. Infiltration of inflammatory cells
i. Results from an b. Mast cell activation
excessive concentra- c. Bronchospasm
tion of reduced
d. Increased airway responsiveness
hemoglobin
e. Exposure to allergen
j. Infection in the
pleural cavity f. Airway inflammation

2.
Column A Column B
S S S S S
1. Cor pulmonale a. Lung tissue destruc-
tion resulting from Activity E Briefly answer the following.
2. Pneumoconi-
a vicious cycle of
oses 1. What are the mechanisms of hypoxemia?
infection and
3. CFTR inflammation
4. ARDS b. Caused by the
inhalation of inor-
5. Atelectasis ganic dusts and
6. Mismatching particulate matter
2. What are the clinical features of atelectasis?
of ventilation c. With increased
and perfusion mucus production,
obstruction of small
7. Bronchiectasis
airways, and a
8. Emphysema chronic productive
cough 3. Explain what is meant by the acute-response
9. Sarcoidosis
d. Incomplete expan- and the late-phase reactions of asthma.
10. Chronic sion of a lung or
bronchitis portion of a lung
e. Right heart failure
resulting from
primary lung disease
4. What factors are causative to the development
f. Granulomas found
of bronchiectasis?
in the lung and
lymphatic system
g. Cystic fibrosis trans-
membrane regulator
h. Enlargement of air
spaces and destruc-
tion of lung tissue

Copyright 2011. Wolters Kluwer Health | Lippincott Williams & Wilkins. Study Guide for Porths Essentials of Pathophysiology, Third Edition.
LWBK707-c23_p130-136.qxd 8/19/10 2:24 PM Page 134 Aptara Inc

134 UNIT 6 RESPIRATORY FUNCTION

5. Describe the pathogenic mechanism of cystic SECTION IV: PRACTICING


fibrosis.
FOR NCLEX
Activity G Answer the following questions.
1. There can be many reasons for a patient to
present with hypoxemia. For a clients PO2 to
6. What are the effects of a pulmonary embolism fall, a respiratory disease is usually involved.
on lung tissue? Often, patients have involvement from more
than one mechanism. Match the mechanism
involved with the end result (hypoxemia or
decreased levels of PO2).

Mechanism Outcome
7. Describe the disease-producing changes of Decreased oxygen
acute respiratory distress syndrome. in air
Inadequate circulation
through pulmonary
capillaries
Hypoventilation
Disease in respiratory
SECTION III: APPLYING YOUR system
KNOWLEDGE Mismatched ventilation
and perfusion
Activity F Consider the scenario and answer
Dysfunction of neurologic
the questions.
system
The parents of a 14-year-old girl arrive in the
emergency department after being notified by 2. When CO2 levels in the blood rise, a state of
the school nurse that their daughter had a hypercapnia occurs in the body. What factors
spell at school and was taken to the emergency contribute to hypercapnia? (Mark all that
department by ambulance. When they arrive apply.)
their daughter is sitting up on the stretcher, has a. Alteration in carbon dioxide production
oxygen on at 1 L/min, and is answering b. Abnormalities in respiratory function
questions asked by the nurse.
c. Disturbance in gas exchange function
1. The doctor talks to the family and tells them d. Decrease in carbon dioxide production
he suspects their daughter has asthma. What
e. Changes in neural control of respiration
diagnostic tests would the nurse expect to be
ordered to confirm the diagnosis of asthma? 3. The complications of a hemothorax can
impact the total body. Left untreated, what
can a moderate or large hemothorax cause?
a. Calcification of the lung tissue
b. Fibrothorax
2. The parents mention to the nurse that their c. Pleuritis
daughter values her independence. They want d. Atelectasis
to know how her treatment plan will impact
her independence. How would the nurse cor-
rectly respond?

Copyright 2011. Wolters Kluwer Health | Lippincott Williams & Wilkins. Study Guide for Porths Essentials of Pathophysiology, Third Edition.
LWBK707-c23_p130-136.qxd 8/19/10 2:24 PM Page 135 Aptara Inc

CHAPTER 23 DISORDERS OF VENTILATION AND GAS EXCHANGE 135

4. Talc lung can occur from injected or inhaled 9. Bronchiectasis is considered a secondary
talc powder that has been mixed with heroin, COPD and, with the advent of antibiotics, it
methamphetamine, or codeine as a filler. What is not a common disease entity. In the past,
are people with talc lung very susceptible to? bronchiectasis often followed specific
a. Hemothorax diseases. Which disease did it not follow?
b. Chylothorax a. Necrotizing bacterial pneumonia
c. Fibrothorax b. Complicated measles
d. Pneumothorax c. Chickenpox
d. Influenza
5. Pleuritis, an inflammatory process of the
pleura, is a common in infectious processes 10. Cystic fibrosis (CF) is an autosomal recessive
that spread to the pleura. Which are the disorder involving the secretion of fluids in
drugs of choice for treating pleural pain? specific exocrine glands. The genetic defect in
a. Indomethacin CF inclines a person to chronic respiratory
infections from a small group of organisms.
b. Aspirin
Which organisms create chronic infection in
c. Acetaminophen a child with cystic fibrosis?
d. Inderal a. Pseudomonas aeruginosa and Escherichia coli
6. Atelectasis is the term used to designate an b. Staphylococcus aureus and Hepatitis C
incomplete expansion of a portion of the c. Haemophilus influenzae and Influenza A
lung. Depending on the size of the collapsed
d. Pseudomonas aeruginosa and S. aureus
area and the type of atelectasis occurring, you
may see a shift of the mediastinum and 11. What etiologic determinants are important in
trachea. Which way does the mediastinum the development of the pneumoconioses?
and trachea shift in compression atelectasis? (Mark all that apply.)
a. Toward the affected lung a. Chemical nature of the dust particle
b. Toward the mediastinum b. Size of dust particle
c. Away from the affected lung c. Density of dust particle
d. Away from the trachea d. Biologic nature of the dust particle
7. Infants and small children have asthma and e. Ability of particle to incite lung destruction
need to be medicated, just as adults do. There 12. There are cytotoxic drugs used in the
are special systems manufactured for the treatment of cancer that cause pulmonary
delivery of inhaled medications to children. damage because of their direct toxicity and
At what age is it recommended that children because they stimulate an influx of
may begin using an metered-dose inhaler inflammatory cells into the alveoli. Which
(MDI) with a spacer? cardiac drug is known for its toxic effect in
a. 3 to 5 years the lungs?
b. 4 to 6 years a. Amiodarone
c. 2 to 4 years b. Inderal
d. 5 to 7 years c. Methotrexate
8. Chronic obstructive pulmonary disease d. Busulfan
(COPD) is a combination of disease processes.
What disease processes have been identified
as being part of COPD?
a. Emphysema and asthma
b. Chronic obstructive bronchitis and em-
physema
c. Chronic obstructive bronchitis and asthma
d. Chronic bronchitis and emphysema

Copyright 2011. Wolters Kluwer Health | Lippincott Williams & Wilkins. Study Guide for Porths Essentials of Pathophysiology, Third Edition.
LWBK707-c23_p130-136.qxd 8/19/10 2:24 PM Page 136 Aptara Inc

136 UNIT 6 RESPIRATORY FUNCTION

13. A pulmonary embolism occurs when there is 16. Acute lung injury/acute respiratory distress
an obstruction in the pulmonary artery blood syndrome (ALI/ARDS) are distinguishable
flow. Classic signs and symptoms of a between the two by the extent of hypoxemia
pulmonary embolism include dyspnea, chest involved. What is the clinical presentation of
pain, and increased respiratory rate. What is ARDS? (Mark all that apply.)
a classic sign of pulmonary infarction? a. Diffuse bilateral infiltrates of lung tissue
a. Mediastinal shift to the left without cardiac dysfunction
b. Pleuritic pain b. Rapid onset
c. Tracheal shift to the right c. Signs of respiratory distress
d. Pericardial pain d. Increase in respiratory rate
14. Pulmonary hypertension is usually caused by e. Hypoxemia refractory to treatment
long-term exposure to hypoxemia. When 17. Acute respiratory failure is commonly
pulmonary vessels are exposed to signaled by varying degrees of hypoxemia
hypoxemia, what is their response? and hypercapnia. Respiratory acidosis
a. Pulmonary vessels dilate develops manifested by what?
b. Pulmonary vessels constrict a. Decrease in cerebral blood flow
c. Pulmonary vessels spasm b. Arterial vasoconstriction
d. Pulmonary vessels infarct c. Increase in cardiac contractility
15. The management of cor pulmonale is d. Increased cerebral spinal fluid pressure
directed at the underlying lung disease and
heart failure. Why is low-flow oxygen
therapy a part of the management of cor
pulmonale?
a. Stimulates body to breathe on its own
b. Inhibits the respiratory center of the brain
from initiating tachypnea
c. Reduces pulmonary hypertension and
polycythemia associated with chronic
lung disease
d. Reduces pulmonary hypertension and
formation of pulmonary embolism

Copyright 2011. Wolters Kluwer Health | Lippincott Williams & Wilkins. Study Guide for Porths Essentials of Pathophysiology, Third Edition.
LWBK707-c24_p137-141.qxd 8/19/10 1:21 PM Page 137 Aptara Inc

24

CHAPTER
Structure and Function
of the Kidney

SECTION I: LEARNING 13. Explain the concept of the glomerular filtra-


tion rate.
OBJECTIVES
14. Explain the value of serum creatinine levels
1. Describe the location and gross structure of in evaluating renal function.
the kidney.
2. Explain why the kidney receives such a large
percentage of the cardiac output and describe
the mechanisms for regulating renal blood
SECTION II: ASSESSING YOUR
flow. UNDERSTANDING
3. Describe the structure and function of the Activity A Fill in the blanks.
glomerulus and tubular components of the
nephron in terms of regulating the composi- 1. The are paired, bean-shaped
tion of the extracellular fluid compartment. organs that lie outside the peritoneal cavity
in the back of the upper abdomen.
4. Explain the concept of tubular transport
mechanisms. 2. The is the place where blood
vessels and nerves enter and leave the
5. Describe how the kidney produces kidney.
concentrated or diluted urine.
3. are the functional units of the
6. Characterize the function of the kidney.
juxtaglomerular complex.
4. The contains the glomeruli and
7. Relate the function of the kidney to drug convoluted tubules of the nephron and
elimination. blood vessels.
8. Explain the endocrine functions of the 5. The medulla consists of the
kidney. that extend into the medulla.
9. Relate the sodium reabsorption function of 6. Each kidney is supplied by a single renal
the kidney to action of diuretics. artery that arises on either side of the
10. Describe the characteristics of normal urine. .

11. Explain the significance of casts in the urine. 7. The afferent arterioles that supply the
arise from the intralobular
12. Explain the value of urine specific gravity in arteries.
evaluating renal function.

137
LWBK707-c24_p137-141.qxd 8/19/10 1:21 PM Page 138 Aptara Inc

138 UNIT 7 KIDNEY AND URINARY TRACT FUNCTION

8. The is a unique, high-pressure 19. The assists in maintenance of


capillary filtration system. the extracellular fluid volume by controlling
the permeability of the medullary collecting
9. are low-pressure vessels that are
tubules.
adapted for reabsorption rather than
filtration. 20. Increased activity causes
constriction of the afferent and efferent arte-
10. The passes through each of
rioles and thus a decrease in renal blood flow.
these segments before reaching the pelvis of
the kidney. 21. Renal is the volume of plasma
that is completely cleared each minute of
11. The is regulated by the
any substance that finds its way into the
constriction and relaxation of the afferent
urine.
and efferent arterioles.
22. functions in the regulation of
12. Substances move from the tubular filtrate
sodium and potassium elimination.
into the tubular cell along a
gradient, but they require facilitated transport 23. Atrial natriuretic peptide contributes to the
or carrier systems to move across the regulation of elimination.
membrane into the interstitial
24. The kidneys regulate body pH by conserving
fluid, where they are absorbed into the
base and eliminating
peritubular capillaries.
ions.
13. uses a carrier system in which
25. is an end product of protein
the downhill movement of one substance
metabolism.
such as sodium is coupled to the uphill
movement of another substance such as 26. The synthesis of is stimulated
glucose or an amino acid. by tissue hypoxia, which may be brought
about by anemia, residence at high altitudes,
14. In the tubule, there is almost
or impaired oxygenation of tissues due to
complete reabsorption of nutritionally
cardiac or pulmonary disease.
important substances from the filtrate.
27. represents excessive protein
15. The plasma level at which the substance
excretion in the urine.
appears in the urine is called the
. 28. Urine provides a valuable
index of the hydration status and functional
16. The establishes a high concen-
ability of the kidneys.
tration of osmotically active particles in the
interstitium surrounding the medullary 29. levels in the blood and urine
collecting tubules where the antidiuretic can be used to measure glomerular filtration
hormone (ADH) exerts its effects. rate (GFR).
17. The thick portion of the loop of Henle 30. , therefore, is related to the GFR
contains a cotransport but, unlike creatinine, also is influenced by
system. protein intake, gastrointestinal bleeding, and
hydration status.
18. The tubule is relatively imper-
meable to water, and reabsorption of sodium
chloride from this segment further dilutes
the tubular fluid.

Copyright 2011. Wolters Kluwer Health | Lippincott Williams & Wilkins. Study Guide for Porths Essentials of Pathophysiology, Third Edition.
LWBK707-c24_p137-141.qxd 8/19/10 1:21 PM Page 139 Aptara Inc

CHAPTER 24 STRUCTURE AND FUNCTION OF THE KIDNEY 139

Activity B Match the key terms in Column A Activity D Briefly answer the following.
with their definitions in Column B.
1. Describe the three layers of the glomerular
Column A Column B membrane.
1. Counter- a. Originate in the
transport superficial part of
the cortex
2. Glomerular
filtration rate b. Originate deeper in
the cortex 2. Describe the various methods of transport
3. Vasopressin
c. Contribute to regu- across the epithelial layer of the renal tubule.
4. Cortical lation of glomerular
nephrons blood flow
5. Vitamin D d. Milliliter of filtrate
formed per minute
6. Principal cells
e. The movement of
3. How does the juxtaglomerular apparatus regu-
7. Juxtamedullary one substance
late GFR?
nephrons enables the
movement of a sec-
8. Countercurrent
ond substance in the
9. Transport opposite direction
maximum f. Maximum amount
10. Mesangial cells of substance that 4. What are the actions of atrial natriuretic pep-
can be reabsorbed tide (ANP)?
per unit of time
g. Site of aldosterone
action
h. Flow of fluids in
opposite directions
5. What are the endocrine functions of the kid-
i. Stimulate expression
ney?
of aquaporin-2
channels
j. Converted to active
form in kidney

Activity C
6. How do Na blockers function as a diuretic?
1. Put the components of the renin-angiotensin-
aldosterone system in order from stimulation
to end hormone action:
Conversion of angiotensin I to angiotensin
II by angiotensin converting enzyme
Decreased GFR
Sodium and Water retention
Angiotensin II stimulates release of ADH
and aldosterone
Juxtaglomerular release of renin
Conversion of angiotensinogen to
angiotensin I by renin

Copyright 2011. Wolters Kluwer Health | Lippincott Williams & Wilkins. Study Guide for Porths Essentials of Pathophysiology, Third Edition.
LWBK707-c24_p137-141.qxd 8/19/10 1:21 PM Page 140 Aptara Inc

140 UNIT 7 KIDNEY AND URINARY TRACT FUNCTION

SECTION III: APPLYING YOUR 2. You are admitting to the floor a 45-year-old
woman with a presumptive diagnosis of dia-
KNOWLEDGE betes mellitus. While taking her history, she
mentions that she has been eating a lot of
Activity E Consider the scenario and answer
sweets lately. How would you expect this diet
the questions.
to impact her renal system?
An 18-year-old girl is brought to the emergency a. Decrease tubular reabsorption
department by her friends. Her blood pressure is
b. Increase renal blood flow
115/85; pulse is 99; respiratory rate in 35 bpm.
The girl is doubled over and she is holding her c. Decrease renal blood flow
abdomen saying, I hurt so bad; I hurt so bad. d. Increase sodium excretion
Her friends deny the girl has been using
3. The renal clearance of a substance is
recreational drugs. They tell the triage nurse that
measured independently. What are the
the girl started complaining that her side hurt
factors that determine renal clearance of a
about 3 hours prior to the trip to the emergency
substance? (Mark all that apply.)
department. Asked if the girls parents had been
notified, the friends tell the triage nurse that a. The ability of the substance to be filtered
they have been unable to reach the girls parents. in the glomeruli
On examination, a suspected diagnosis of kidney b. The capacity of the renal tubules to reab-
impairment is arrived at. sorb or secrete the substance
1. What tests would the nurse expect to be c. The normal electrolyte and pH composi-
ordered to either confirm or deny the tion of the blood
diagnosis? d. The rate of renal blood flow
e. The rate sodium is excreted from the body
4. It is known that high levels of uric acid in the
blood can cause gout, while high levels in the
urine can cause kidney stones. What medica-
2. The girl says, My father just had a kidney tion competes with uric acid for secretion in
stone removed. Is that what I have? What to the tubular fluid, thereby reducing uric
noninvasive test would the nurse expect to be acid secretion?
ordered to rule out a kidney stone? a. Ibuprofen
b. Acetaminophen
c. Aspirin
d. Advil
5. Many drugs are eliminated in the urine.
These drugs cannot be bound to plasma pro-
teins if the glomerulus is going to filter them
SECTION IV: PRACTICING out of the blood. In what situation would it
FOR NCLEX be necessary to create either an alkaline or
acid diuresis in a client?
Activity F Answer the following questions. a. Nontherapeutic drug levels in blood
1. Many substances are both filtered out of the b. Noncompliance with medication regimen
blood and reabsorbed into the blood in the c. The need to use a loading dose of a specific
kidneys. What is the plasma level at which a drug and keep it in the system for a long
specific substance can be found in the urine? time.
a. Renal threshold d. In the case of a drug overdose
b. Renal clearance
c. Renal filtration rate
d. Renal transport level

Copyright 2011. Wolters Kluwer Health | Lippincott Williams & Wilkins. Study Guide for Porths Essentials of Pathophysiology, Third Edition.
LWBK707-c24_p137-141.qxd 8/19/10 1:21 PM Page 141 Aptara Inc

CHAPTER 24 STRUCTURE AND FUNCTION OF THE KIDNEY 141

6. The anemia that occurs with end-stage 9. An elderly man is brought into the clinic by
kidney disease is often caused by the kidneys his daughter who states, My father hasnt
themselves. What inability of the kidney dis- been himself lately. Now I think he looks a
ease causes anemia in end-stage kidney little yellow. What test would the nurse
disease? expect to have ordered to check this mans
a. Produce erythropoietin creatinine level?
b. Produce rennin a. BUN level
c. Produce angiotensin b. 24 hour urine test
d. Inactivate vitamin D c. Urine test, first void in morning
d. Serum creatinine
7. Diuretics can either block the reabsorption of
components of the urine, or they can block 10. A patient suffering from a previous
the reabsorption of water back into the body. myocardial infarction is displaying an inabil-
What does the increase in urine flow from ity to dilate the blood vessels and increased
the body depend on with a patient taking sodium retention. Which hormone level may
diuretics? have been affected by the MI?
a. The amount of water reabsorption back a. ANP
into the body b. ADH
b. The amount of sodium and chloride reab- c. BNP
sorption that it blocks
d. ACTH
c. The amount of sodium and chloride that it
excretes through the kidney
d. The amount of water excreted by the body
8. Urine specific gravity is normally 1.010 to
1.025 with adequate hydration. When there
is loss of renal concentrating ability due to
impaired renal function, low concentration
levels are exhibited. When would the nurse
consider the low levels of concentration to be
significant?
a. At noon
b. First void in morning
c. Last void at night
d. After a nap

Copyright 2011. Wolters Kluwer Health | Lippincott Williams & Wilkins. Study Guide for Porths Essentials of Pathophysiology, Third Edition.
LWBK707-c25_p142-147.qxd 9/2/10 7:47 AM Page 142 Aptara Inc

25
CHAPTER
Disorders of Renal
Function

SECTION I: LEARNING 9. List four common causes of urinary tract


obstruction.
OBJECTIVES
10. Define the term hydronephrosis and relate
1. Describe the two types of immune it to the destructive effects of urinary tract
mechanisms involved in glomerular obstructions.
disorders.
11. Describe the role of urine supersaturation,
2. Use the terms proliferation, sclerosis, membra- nucleation, and inhibitors of stone formation
nous, diffuse, focal, segmental, and mesangial in the development of kidney stones.
to explain changes in glomerular structure
12. Explain the mechanisms of pain and
that occur with glomerulonephritis.
infection that occur with kidney stones.
3. Relate the proteinuria, hematuria, pyuria,
13. Describe methods used in the diagnosis and
oliguria, edema, hypertension, and azotemia
treatment of kidney stones.
that occur with glomerulonephritis to
changes in glomerular structure. 14. Cite the organisms most responsible for uri-
nary tract infections (UTIs) and state why
4. Briefly describe the difference among the
urinary catheters, obstruction, and reflux
nephritic syndromes, rapidly progressive
predispose to infections.
glomerulonephritis, nephrotic syndrome,
asymptomatic glomerular disorders, and 15. List three physiologic mechanisms that
chronic glomerulonephritis. protect against UTIs.
5. Cite a definition of tubulointerstitial kidney 16. Describe the signs and symptoms of UTIs.
disease.
17. Describe factors that predispose to UTIs in
6. Differentiate between the defects in tubular children, sexually active women, pregnant
function that occur in proximal and distal women, and older adults.
tubular acidosis.
18. Compare the manifestations of UTIs in differ-
7. Explain the pathogenesis of kidney damage ent age groups, including infants, toddlers,
in acute and chronic pyelonephritis. adolescents, adults, and older adults.
8. Describe the inheritance, pathology, and 19. Cite measures used in the diagnosis and
manifestations of the different types of treatment of UTIs.
polycystic kidney disease.
20. Explain the vulnerability of the kidneys to
injury caused by drugs and toxins.

142
LWBK707-c25_p142-147.qxd 9/2/10 7:47 AM Page 143 Aptara Inc

CHAPTER 25 DISORDERS OF RENAL FUNCTION 143

21. Characterize Wilms tumor in terms of age of 11. refers to urine-filled dilatation
onset, possible oncogenic origin, manifesta- of the renal pelvis and calyces associated
tions, and treatment. with progressive atrophy of the kidney due
to obstruction of urine outflow.
22. Cite the risk factors for renal cell carcinoma,
describe its manifestations, and explain why 12. Obstruction of the urinary track may provoke
the 5-year survival rate has been so low. pain due to of the collecting
system and renal capsule.
13. The most common cause of upper urinary
tract obstruction is urinary .
SECTION II: ASSESSING YOUR
UNDERSTANDING 14. In addition to a supersaturated urine, kidney
stone formation requires a that
Activity A Fill in the blanks. facilitates crystal aggregation.

1. Anomalies in and 15. Most kidney stones are stones.


of the kidneys are the most common form of 16. The major manifestation of kidney stones is
congenital renal disorder. .
2. The term dysgenesis refers to a failure of an 17. Urinary tract infections are the
organ to develop normally and most common type of bacterial infection
refers to complete failure of an organ to seen by health care providers.
develop.
18. Most uncomplicated lower UTIs are caused
3. Newborns with renal agenesis often have by .
characteristic facial features, termed
, resulting from the effects 19. Most UTIs are caused by bacteria that enter
of oligohydramnios. through the .

4. In renal , the kidneys do not 20. Urinary tract infections are


develop to normal size. common in women than men.

5. Renal is due to an abnormality 21. In UTIs associated with stasis of urine flow,
in the differentiation of kidney structures the obstruction may be or
during embryonic development. .

6. Unilateral renal dysplasia is the 22. -associated bacteriuria remains


most common cause of an abdominal mass the most frequent cause of Gram-negative
in newborns. septicemia in hospitalized patients.

7. kidney diseases are a group of 23. An acute episode of is


kidney disorders characterized by fluid-filled characterized by frequency of urination,
sacs or segments that have their origin in the lower abdominal or back discomfort, and
tubular structures of kidney. burning and pain on urination.

8. In the form of polycystic kidney 24. is second leading cause of kid-


disease, thousands of large cysts are derived ney failure worldwide and it ranks third, after
from every segment of the nephron. diabetes and hypertension, as a cause of
chronic kidney disease in the United States.
9. The effects of urinary obstruc-
tion on kidney structures are determined 25. The syndromes produce a pro-
by the degree and the duration of the liferative inflammatory response, whereas the
obstruction. syndrome produces increased
permeability of the glomerulus.
10. of urine predisposes to
infection, which may spread throughout 26. syndrome is characterized by
the urinary tract. sudden onset of hematuria, variable degrees
of proteinuria, diminished glomerular filtra-
tion rate, oliguria, and signs impaired renal
function.

Copyright 2011. Wolters Kluwer Health | Lippincott Williams & Wilkins. Study Guide for Porths Essentials of Pathophysiology, Third Edition.
LWBK707-c25_p142-147.qxd 9/2/10 7:47 AM Page 144 Aptara Inc

144 UNIT 7 KIDNEY AND URINARY TRACT FUNCTION

27. Acute postinfectious glomerulonephritis Activity B Consider the following figure.


usually occurs after infection with certain
strains of group A -hemolytic streptococci
and is caused by of immune
complexes.
28. The of postinfectious glomeru-
lonephritis is caused by infiltration of leuko-
cytes, both neutrophils and monocytes;
proliferation of endothelial and mesangial
cells; and, in severe cases, formation of
crescents.
29. syndrome is an uncommon
and aggressive form of glomerulonephritis
that is caused by antibodies to the glomerular
basement membrane.
30. syndrome is characterized by
massive proteinuria and lipiduria, along with
an associated hypoalbuminemia, generalized
edema, and hyperlipidemia.
31. glomerulonephritis is caused by
diffuse thickening of the glomerular basement
membrane due to deposition of immune
complexes. In the figure above, identify the common
32. is a primary glomerulonephritis locations and causes of urinary track obstructions:
characterized by the presence of glomerular Pregnancy or tumor
IgA immune complex deposits.
Ureterovesical junction stricture
33. Alport syndrome represents a hereditary Kidney stone
defect of the glomerular that
Scar tissue
results in hematuria and may progress to
chronic renal failure. Neurogenic bladder
Bladder outflow obstruction
34. refers to a group of tubular
defects in reabsorption of bicarbonate ions
or excretion of hydrogen ions (H) that result Activity C Match the key terms in Column A
in metabolic acidosis and its subsequent with their definitions in Column B.
complications, including metabolic bone Column A Column B
disease, kidney stones, and growth failure
in children. 1. Urease a. Low renal mass in
infant
35. Proximal renal tubular acidosis involves a 2. Hypogenesis
b. Blood cells in
defect in proximal tubular reabsorption of 3. Oliguria urine
.
4. PKD 1 and 2 c. Urea splitting bac-
36. represents an infection of the terial enzyme
upper urinary tract, specifically the renal 5. Hydronephrosis
d. Change in renal
parenchyma and renal pelvis. 6. Proteinuria structure
37. is one of the most common 7. Renal dysplasia e. Dilatation of the
primary neoplasms of young children. renal pelvis and
8. Nephrolithiasis
38. Kidney cancer is suspected when there calyces associated
are findings of and a renal 9. Hematuria with progressive
. atrophy
10. Oligohydramnios

Copyright 2011. Wolters Kluwer Health | Lippincott Williams & Wilkins. Study Guide for Porths Essentials of Pathophysiology, Third Edition.
LWBK707-c25_p142-147.qxd 9/2/10 7:47 AM Page 145 Aptara Inc

CHAPTER 25 DISORDERS OF RENAL FUNCTION 145

f. Very low urine Activity E Briefly answer the following.


production
1. What is the mechanism of tissue damage in
g. Genes responsible
urinary track obstructions?
for autosomal
dominant polycys-
tic kidney disease
(ADPKD)
h. Kidney stone
formation 2. What are the factors involved in kidney stone
i. Protein loss in formation?
urine
j. Low amniotic fluid
levels

Activity D
3. For whom are the risk factors for UTIs higher?

4. What are the host defense mechanisms


against the development of a UTI?
Glomerular damage

5. What are the cellular changes associated with


glomerular disease?

6. Describe the disease progress and the produc-


tion of symptoms in poststreptococcal
glomerulonephritis.

Complete the above flowchart using the bullet


points below.
Edema
Hyperlipidemia
Increased permeability to proteins 7. Describe the mechanisms of a diabetic
nephropathy.
Decreased plasma oncotic pressure
Hypoproteinemia
Compensatory synthesis of proteins by liver

Copyright 2011. Wolters Kluwer Health | Lippincott Williams & Wilkins. Study Guide for Porths Essentials of Pathophysiology, Third Edition.
LWBK707-c25_p142-147.qxd 9/2/10 7:47 AM Page 146 Aptara Inc

146 UNIT 7 KIDNEY AND URINARY TRACT FUNCTION

8. How do medications and toxins from the 2. Match the type of polycystic kidney disorder
environment damage renal structures? with the characteristic cysts.
Type of Polycystic Kidney Disorder
1. Autosomal dominant polycystic kidney
disease (ADPKD)
2. Autosomal recessive polycystic kidney dis-
ease (ARPKD)
3. Acquired cysts
SECTION III: APPLYING YOUR 4. Nephronophthisis-medullary cystic kidney
KNOWLEDGE disease

Activity F Consider the scenario and answer Characteristic Cysts


the question. a. Small elongated cysts form in the
collecting ducts and maintain contact
An elderly woman, hospitalized with a broken
with the nephron of origin
hip, has an indwelling catheter in place. On the
third day of hospitalization the womans urine b. The tubule wall, which is lined by a single
becomes cloudy and foul smelling. The nurse layer of tubular cells, expands and then
knows that catheters have a high incidence of rapidly closes the cyst off from the tubule
causing UTIs in hospitalized patients. of origin.
c. Cysts are restricted to the corticomedullary
1. What orders would the nurse expect to receive
border.
for this patient to determine if there is an
infection? d. Cysts that develop in the kidney as a
consequence of aging, dialysis, or other
conditions that affect tubular function
3. A young woman presents with signs and
symptoms of a UTI. The nurse notes that this
is the fifth UTI in as many months. What
would this information lead the nurse to
believe?
SECTION IV: PRACTICING a. There is possible obstruction in the urinary
FOR NCLEX tract
b. The woman has multiple sexual partners
Activity G Answer the following questions.
c. The woman takes too many bubble baths
1. Congenital disorders of the kidneys are fairly
d. The woman does not clean herself as she
common, occurring in approximately 1:1000
should
live births. What is the result to the newborn
when bilateral renal dysplasia occurs? (Mark 4. Staghorn kidney stones, or struvite stones,
all that apply.) are usually located in the renal pelvis. These
a. Potter facies stones are made from what?
b. Oligohydramnios a. Calcium oxalate
c. Pulmonary hypoplasia b. Magnesium ammonium phosphate
d. Multicystic kidneys c. Cystine
e. Renal failure d. Uric acid
5. What is the most common cause of a lower
UTI?
a. Staphylococcus saprophyticus
b. Pseudomonas aeruginosa
c. Escherichia coli
d. Staphylococcus aureus

Copyright 2011. Wolters Kluwer Health | Lippincott Williams & Wilkins. Study Guide for Porths Essentials of Pathophysiology, Third Edition.
LWBK707-c25_p142-147.qxd 9/2/10 7:47 AM Page 147 Aptara Inc

CHAPTER 25 DISORDERS OF RENAL FUNCTION 147

6. Urinary tract infections in children do not 9. Acute pyelonephritis is an infection of


generally present as UTIs as they do in adults. the renal parenchyma and renal pelvis.
What are the signs and symptoms of a UTI in What is the most common cause of acute
a toddler? (Mark all that apply.) pyelonephritis?
a. Frequency a. Group A -hemolytic streptococci
b. Diarrhea b. P. aeruginosa
c. Abdominal pain c. Haemophilus influenza
d. Poor growth d. Candida albicans
e. Burning 10. Drug-related nephropathies occur all too
7. Acute postinfectious glomerulonephritis, as often. They involve functional and/or struc-
its name implies, follows an acute infection tural changes to the kidney after exposure to
somewhere else in the body. What is the a drug. What does the tolerance to drugs
most common cause of acute postinfectious depend on?
glomerulonephritis? a. Vesicoureteral reflux
a. E. coli b. Glomerular filtration rate
b. S. aureus c. State of hydration
c. P. aeruginosa d. Proteinuria
d. Group A -hemolytic streptococci 11. Wilms tumor is a tumor of childhood. It is
8. Both type I and type II diabetes mellitus can usually an encapsulated mass occurring in
cause damage to the glomeruli of the kidneys. any part of the kidney. What are the common
What renal disease is diabetic nephropathy presenting signs of a Wilms tumor?
associated with? a. Hypotension and a large abdominal mass
a. Nephrotic syndrome b. Vomiting and oliguria
b. Acute glomerulonephritis c. Abdominal pain and diarrhea
c. Nephritic syndrome d. Large asymptomatic abdominal mass and
d. Acute glomerulonephritis hypertension

Copyright 2011. Wolters Kluwer Health | Lippincott Williams & Wilkins. Study Guide for Porths Essentials of Pathophysiology, Third Edition.
LWBK707-c26_p148-151.qxd 8/19/10 1:22 PM Page 148 Aptara Inc

26
CHAPTER
Acute Renal Failure and
Chronic Kidney Disease

SECTION I: LEARNING 8. State the basis for adverse drug reactions in


patients with chronic kidney disease.
OBJECTIVES
9. Describe the scientific principles underlying
1. Describe acute renal failure in terms of its dialysis treatment, and compare hemodialysis
causes, treatment, and outcome. with peritoneal dialysis.
2. Differentiate the prerenal, intrinsic, and 10. Cite the possible complications of kidney
postrenal forms of acute renal failure in transplantation.
terms of the mechanisms of development
11. State the goals for dietary management of
and manifestations.
persons with chronic kidney disease.
3. Cite the two most common causes of acute
12. List the causes of chronic kidney disease
tubular necrosis and describe the course of
(CKD) in children and describe the special
the disease in terms of the initiation, mainte-
problems of children with kidney failure.
nance, and recovery phases.
13. State why CKD is more common in the
4. State the most common causes of chronic
elderly and describe measures to prevent or
kidney disease.
delay the onset of kidney failure in this
5. Describe the five stages of chronic kidney population.
disease.
14. Describe the treatment of CKD in children
6. Describe the methods used to arrive at an and the elderly.
accurate estimation of the glomerular
filtration rate (GFR) and explain the rationale
for its use in defining the stages of chronic
kidney disease.
SECTION II: ASSESSING
YOUR UNDERSTANDING
7. Explain the physiologic mechanisms under-
lying the common problems associated with Activity A Fill in the blanks.
chronic kidney disease, including alterations
in fluid and electrolyte balance and disorders 1. represents a rapid decline in
of skeletal, hematologic, cardiovascular, kidney function sufficient to increase blood
immune system, neurologic, skin, and sexual levels of nitrogenous wastes and impair fluid
function. and electrolyte balance.

148
LWBK707-c26_p148-151.qxd 8/19/10 1:22 PM Page 149 Aptara Inc

CHAPTER 26 ACUTE RENAL FAILURE AND CHRONIC KIDNEY DISEASE 149

2. The causes of acute renal failure commonly are 15. The acidosis that occurs in persons with kid-
categorized as , , ney failure seems to stabilize as the disease
or . progresses, probably as a result of the tremen-
dous buffering capacity of .
3. failure, the most common form
of acute renal failure, is characterized by a 16. The term renal is used to
marked decrease in renal blood flow. describe the skeletal complications of CKD.
4. Because of their high metabolic rate, the 17. commonly is an early manifes-
cells are most vulnerable to tation of chronic renal failure.
ischemic injury.
18. Anorexia, nausea, and vomiting are common
5. Prerenal failure is manifested by a sharp in patients with , along with a
decrease in urine output and a disproportion- metallic taste in the mouth that further
ate elevation of in relation to depresses the appetite.
serum creatinine levels.
19. Neuropathy is caused by and
6. failure results from obstruction of nerve fibers, possibly caused
of urine outflow from the kidneys. by uremic toxins.
7. A major concern in the treatment of acute 20. Normal aging is associated with a decline in
renal failure is identifying and correcting the the and subsequently with
. reduced homeostatic regulation under stress-
ful conditions.
8. Regardless of cause, represents a
loss of functioning kidney nephrons with
Activity B Consider the following figure.
progressive deterioration of glomerular filtra-
tion, tubular reabsorptive capacity, and
endocrine functions of the kidneys.
9. The normal GFR, which varies with age, gen-
der, and body size, is approximately
mL/minute (1.73 mL/minute
per square millimeter) for normal young
healthy adults.
10. In clinical practice, GFR is usually estimated
using the serum concentration.
11. Increased excretion of low-molecular-weight
globulins is a marker of disease,
and excretion of a marker of
CKD.
12. The state includes signs and
symptoms of altered fluid, electrolyte, and
acid-base balance; and alterations in
regulatory functions.
In the figure above, label the sites of prerenal,
13. Chronic renal failure can produce intrinsic, and postrenal causes of renal failure.
or fluid , depend-
ing on the pathology of the kidney disease.
14. In chronic renal failure, the kidneys lose the
ability to regulate excretion.

Copyright 2011. Wolters Kluwer Health | Lippincott Williams & Wilkins. Study Guide for Porths Essentials of Pathophysiology, Third Edition.
LWBK707-c26_p148-151.qxd 8/19/10 1:22 PM Page 150 Aptara Inc

150 UNIT 7 KIDNEY AND URINARY TRACT FUNCTION

Activity C Match the key terms in Column A 4. Why is chronic kidney disease considered to
with their definitions in Column B. have an insidious progression?

Column A Column B
1. Isosthenuria a. Decreased urine
production
2. Azotemia
b. Polyuria with
3. Creatinine 5. What are the clinical manifestations of
urine that is
chronic kidney disease?
4. Salt wasting almost isotonic
with plasma
5. Oliguria
c. Increased bone
6. Uremic resorption and
encephalopathy formation
7. Prostatic d. By-product of mus- 6. How is anemia related to chronic kidney
hyperplasia cle metabolism disease?
e. Decreased CNS
8. Hemodialysis
activity
9. Uremia f. Presence of exces-
10. Osteitis fibrosa sive amounts of
urea in the blood
7. How does renal disease cause cardiovascular
g. Impaired tubular disease?
reabsorption of
sodium
h. Most common
cause of postrenal
failure
i. Use of artificial kid-
ney to filter blood
j. Accumulation of
SECTION III: APPLYING
nitrogenous wastes YOUR KNOWLEDGE
in the blood
Activity E Consider the scenario and answer
Activity D Briefly answer the following. the questions.
1. Name the most common intrarenal cause of The parents of a hospitalized 4-year-old boy have
renal failure and describe its different forms. just been told that their son has a chronic renal
disease. The nurse is planning discharge teaching
for this family.
1. What would the nurse know to include in the
discharge teaching for this child and his family?
2. Describe the progression of acute tubular
necrosis (ATN).

2. The parents inquire about treatment for their


son and if kidney transplantation could occur.
What would be the nurses best response?
3. How is chronic kidney disease classified?

Copyright 2011. Wolters Kluwer Health | Lippincott Williams & Wilkins. Study Guide for Porths Essentials of Pathophysiology, Third Edition.
LWBK707-c26_p148-151.qxd 8/19/10 1:22 PM Page 151 Aptara Inc

CHAPTER 26 ACUTE RENAL FAILURE AND CHRONIC KIDNEY DISEASE 151

SECTION IV: PRACTICING 6. Neuromuscular disorders can be triggered by


CKD. For those clients on dialysis,
FOR NCLEX approximately two-thirds suffer from what
peripheral neuropathy?
Activity F Answer the following questions.
a. Reynaud syndrome
1. Acute renal failure occurs at a high rate in
b. Burning hands and feet
seriously ill people who are in intensive care
units. What is the most common indicator of c. Tingling and loss of sensation in lower
acute renal failure? limbs
a. Azotemia and a decrease in the GFR d. Restless leg syndrome
b. Proteinuria and a decrease in the GFR 7. People with CKD have impaired immune
c. Azotemia and an increase in the GFR responses to infection because of high levels
of urea and metabolic wastes in the blood.
d. Proteinuria and an increase in the GFR
What is one thing that is missing in an
2. Acute tubular necrosis is the most common immune response in people with CKD?
cause of intrinsic renal failure. One of the a. Failure to mount a fever with infection
causes of ATN is ischemia. What are the
b. Failure of a phagocytic response with
most common causes of ischemic ATN?
infection
(Mark all that apply.)
c. Decrease in granulocyte count
a. Severe hypovolemia
d. Impaired humoral immunity response
b. Severe hypertension
with infection
c. Burns
8. Sexual dysfunction in people with CKD is
d. Overwhelming sepsis
thought to be multifactorial. What are
e. Severe hypervolemia thought to be causes of sexual dysfunction in
3. The GFR is considered to be the best measure people with CKD? (Mark all that apply.)
of renal function. What is used to estimate a. Antihypertensive drugs
the GFR? b. Psychological factors
a. BUN c. Uremic toxins
b. Serum creatinine d. Inability to vasodilate veins
c. Albumin level e. High incidence of sexually transmitted dis-
d. Serum protein eases
4. Chronic kidney disease impacts many 9. In hemodialysis, access to the vascular system
systems in the body. What is the number one is most commonly through what?
hematologic disorder caused by CKD? a. External arteriovenous shunt
a. Polycythemia b. Internal arteriovenous fistula
b. Erythrocythemia c. Internal arteriovenous shunt
c. Anemia d. External arteriovenous fistula
d. Leukocytosis
10. Dietary restrictions placed on people with
5. Uremic pericarditis is a disorder that accom- CKD include limiting protein in their diet.
panies CKD. What are its presenting signs The recommended sources of protein for peo-
and symptoms? (Mark all that apply.) ple with CKD include what source of protein?
a. Pericardial friction rub a. Red meat
b. Chest pain with respiratory accentuation b. Fowl
c. Fever without infection c. Milk
d. Shortness of breath d. Fish
e. Thromboangiitis

Copyright 2011. Wolters Kluwer Health | Lippincott Williams & Wilkins. Study Guide for Porths Essentials of Pathophysiology, Third Edition.
LWBK707-c27_p152-156.qxd 8/19/10 1:23 PM Page 152 Aptara Inc

27
CHAPTER
Disorders of the Bladder
and Lower Urinary Tract

SECTION I: LEARNING 9. List the treatable causes of incontinence in


the elderly.
OBJECTIVES
10. Discuss the difference between superficial
1. Trace the ascending sensory and descending and invasive bladder cancer in terms of blad-
motor impulses between the detrusor muscle der involvement, extension of the disease,
and external urinary sphincter and the spinal and prognosis.
cord, pontine micturition center, and
11. State the most common sign of bladder
cerebral cortex.
cancer.
2. Explain the mechanism of low-pressure urine
storage in the bladder.
3. Describe at least three urodynamic studies SECTION II: ASSESSING
that can be used to assess bladder function. YOUR UNDERSTANDING
4. Describe the causes of and compensatory
changes that occur with urinary tract Activity A Fill in the blanks.
obstruction. 1. The stores urine and controls
5. Differentiate lesions that produce storage its elimination from the body.
dysfunction associated with spastic bladder 2. The bladder is a freely movable organ located
from those that produce emptying on the pelvic floor, just
dysfunction associated with flaccid bladder posterior to the pubic .
in terms of the level of the lesions and their
effects on bladder function. 3. In the male, the gland
surrounds the neck of the bladder where it
6. Describe methods used in treatment of neu- empties into the urethra.
rogenic bladder.
4. Urine passes from the kidneys to the bladder
7. Define incontinence and differentiate through the .
between stress incontinence, overactive
bladder/urge incontinence, and overflow 5. The tonicity and composition of the urine
incontinence. often is quite different from that of the
blood, and the of the bladder
8. Describe behavioral, pharmacologic, and acts as an effective barrier to prevent the
surgical methods used in treatment of the passage of water and other urine elements
different types of incontinence. between the bladder and the blood.

152
LWBK707-c27_p152-156.qxd 8/19/10 1:23 PM Page 153 Aptara Inc

CHAPTER 27 DISORDERS OF THE BLADDER AND LOWER URINARY TRACT 153

6. The operates as a reserve mech- 17. A mild form of reflex neurogenic bladder can
anism to stop micturition when it is develop after a .
occurring and to maintain continence in the
18. of the detrusor muscle and loss
face of unusually high bladder pressure.
of the perception of bladder fullness permit
7. The motor component of the neural reflex the overstretching of the detrusor muscle
that causes bladder emptying is controlled that contributes to weak and ineffective blad-
by the nervous system, while der contractions seen in detrusor muscle
the relaxation and storage function of the areflexia.
bladder is controlled by the
19. is the involuntary loss of urine
nervous system.
during coughing, laughing, sneezing, or lift-
8. The parasympathetic lower motor neurons ing that increases intra-abdominal pressure.
for the detrusor muscle of the bladder are
20. Two mechanisms are thought to contribute
located in the segments of the
to its symptomatology of overactive bladder:
spinal cord; their axons travel to the bladder
CNS and neural control of bladder sensation
by way of the .
and emptying, and those
9. The immediate coordination of the normal involving the smooth muscle of the bladder
micturition reflex occurs in the micturition itself, .
center in the , facilitated by
21. Approximately 90% of bladder cancers are
descending input from the forebrain and
derived from the epithelial cells
ascending input from the reflex centers in the
that line the bladder.
spinal cord.
22. The most common sign of bladder cancer is
10. brain centers enable inhibition
painless .
of the micturition center in the pons and
conscious control of urination.
Activity B Consider the following figure.
11. The receptors are found in the
detrusor muscle; they produce relaxation of Epithelium when Epithelium when
bladder is empty bladder is full
the detrusor muscle, increasing the bladder
volume at which the micturition reflex is
triggered.
12. The activation of produces
contraction of the intramural ureteral muscu-
lature, bladder neck, and internal sphincter.
13. Alterations in bladder function include
urinary with retention or stasis
of urine and urinary with
involuntary loss of urine.
14. The most important cause of urinary obstruc-
tion in males is external compression of the
urethra caused by the enlargement of the
.
15. Neurogenic disorders of bladder function
commonly are manifested in one of two
ways: failure to urine or failure 1. In the diagram of the bladder above, please
to . locate and label the following:
16. Spastic bladder is caused by conditions that Detrusor muscle
produce partial or extensive neural damage Ureters
above the center in the sacral
Trigone
cord.
Internal sphincter
External sphincter

Copyright 2011. Wolters Kluwer Health | Lippincott Williams & Wilkins. Study Guide for Porths Essentials of Pathophysiology, Third Edition.
LWBK707-c27_p152-156.qxd 8/19/10 1:23 PM Page 154 Aptara Inc

154 UNIT 7 KIDNEY AND URINARY TRACT FUNCTION

Activity C Match the key terms in Column A 3. Describe the activities of the pontine micturi-
with their definitions in Column B. tion center and cortical brain centers.

Column A Column B
1. Incontinence a. Muscle-tensing
exercises of the
2. Micturition
pelvic muscles
3. Kegel exercises b. Uninhibited spinal 4. Describe the actions that take place in the
reflex-controlled bladder during micturition.
4. Muscarinic
contraction of the
5. Nocturia bladder without
6. Antimuscarinic relaxation of the
drugs external sphincter
c. Produce relaxation 5. What are the necessary factors that every
7. Detrusor-
of the detrusor child must possess in order to attain conscious
sphincter
muscle, increasing control of bladder function?
dyssynergia
the bladder
8. May cause volume at which
urinary retention the micturition
reflex is triggered
9. Nicotinic
d. Cholinergic recep-
10. 2-adrenergic tor found on 6. Describe the effects of prolonged urinary tract
receptors external sphincter obstruction disorders on the bladder.
muscle
e. Antihistamine
f. Passage of urine
g. Decrease detrusor
muscle tone and 7. Why do many women develop incontinence
increase bladder following childbirth?
capacity
h. Cholinergic recep-
tor found on stri-
ated muscle fibers
of bladder
i. Involuntary loss or 8. Describe how chronic neurologic disorders
leakage of urine can contribute to overactive bladder.
j. Excessive urination
at night

Activity D Briefly answer the following.


1. Describe the structural layers of the bladder. 9. What are the factors associated with age that
contribute to incontinence in the elderly?

2. List the name and function of the major


nerves that regulate bladder function.

Copyright 2011. Wolters Kluwer Health | Lippincott Williams & Wilkins. Study Guide for Porths Essentials of Pathophysiology, Third Edition.
LWBK707-c27_p152-156.qxd 8/19/10 1:23 PM Page 155 Aptara Inc

CHAPTER 27 DISORDERS OF THE BLADDER AND LOWER URINARY TRACT 155

SECTION III: APPLYING 2. Children usually achieve bladder control by


age 5. Girls generally achieve bladder control
YOUR KNOWLEDGE before boys do. What is the general rule for
bladder capacity in a child?
Activity E Consider the scenario and answer
the questions. a. Up to the age of 12 to 14, the capacity of
the bladder is the childs age in years plus 2.
A 53-year-old woman with multiple sclerosis
b. Up to the age of 5, the capacity of the blad-
presents at the clinic with urinary frequency and
der is the childs age in years plus 3.
bladder spasms. A urinalysis is negative for infec-
tion. A complete history is taken and a physical c. The capacity of the bladder is equal to the
examination is performed by the primary care childs age in years.
physician. The woman asks the nurse why she is d. Age has nothing to do with bladder capac-
having bladder spasms and urinary frequency if ity; it has adult capacity from toddlerhood.
she does not have a bladder infection.
3. One of the many tests done during uro-
1. What would the nurse respond? dynamic studies is the sphincter electromyo-
graph (EMG). What does this test study?
a. Ability of the bladder to store urine
b. Activity of the voluntary muscles of the
perineal area
2. What would the nurse expect the doctor to do c. The pressure of the bladder during filling
for this woman to treat her bladder spasms? and emptying
d. The flow rate during urination
4. Urinary obstruction in the lower urinary tract
triggers changes to the urinary system to
compensate for the obstruction. What is an
early change the system makes in its effort to
cope with an obstruction?
SECTION IV: PRACTICING a. Ability to suppress urination is increased
FOR NCLEX b. The stretch receptors in the bladder wall
become hypersensitive
Activity F Answer the following questions. c. The bladder begins to shrink
1. You are caring for a 16-year-old male patient, d. Bladder contraction weakens
newly diagnosed with a spinal cord injury. He 5. What is a common cause of spastic bladder
asks you why he can no longer control his dysfunction?
bladder. What would you explain to him?
a. Central nervous system lesions
(Mark all that apply.)
b. Constriction of the internal sphincter mus-
a. Your spinal cord injury has disrupted the cles
control your brain has over your bladder.
c. External sphincter spasticity
b. You will always have to wear an internal
d. Vesicoureteral reflux
catheter.
c. You will have to learn how to in-and-out 6. Acute overdistention of the bladder can occur
catheterize yourself. in anyone with a neurogenic bladder that
does not empty. How much urine would the
d. You have a condition known as a relaxed
nurse empty out of the bladder at one time?
bladder.
a. Everything in the bladder, no matter how
e. You have a condition known as detrusor-
full it is
sphincter dyssynergia.
b. No more than 600 mL of urine at one time
c. No more than 500 mL of urine at one time
d. No more than 1000 mL of urine at one
time.

Copyright 2011. Wolters Kluwer Health | Lippincott Williams & Wilkins. Study Guide for Porths Essentials of Pathophysiology, Third Edition.
LWBK707-c27_p152-156.qxd 8/19/10 1:23 PM Page 156 Aptara Inc

156 UNIT 7 KIDNEY AND URINARY TRACT FUNCTION

7. In women, stress incontinence is a common 9. Urinary incontinence can be a problem with


problem. The loss of the angle between the the elderly. One method of treatment is habit
urethrovesical junction and the bladder con- training, or bladder training. When using this
tributes to stress incontinence. What is the treatment with an elderly person how
normal angle between the bladder and the frequently should they be voiding?
urethrovesical junction? a. Every 1 to 3 hours
a. 90 to 100 degrees b. Every 2 to 4 hours
b. 100 to 1110 degrees c. Every 3 to 5 hours
c. 80 to 90 degrees d. Every 4 to 6 hours
d. 95 to 105 degrees
10. One of the treatments for bladder cancer in
8. Incontinence can be transient. What are the situ is the intervesicular administration of
causes of transient urinary incontinence? what drug?
(Mark all that apply.) a. Adriamycin
a. Spinal cord injury b. Mitomycin C
b. Confusional states c. Bacillus Calmette-Gurin vaccine
c. Stool impaction d. Thiotepa
d. Diarrhea
e. Recurrent urinary tract infections

Copyright 2011. Wolters Kluwer Health | Lippincott Williams & Wilkins. Study Guide for Porths Essentials of Pathophysiology, Third Edition.
LWBK707-c28_p157-161.qxd 8/19/10 1:23 PM Page 157 Aptara Inc

28

CHAPTER
Structure and Function
of the Gastrointestinal
System

SECTION I: LEARNING 12. List three major GI hormones and cite their
OBJECTIVES function.
13. Describe the site of gastric acid and pepsin
1. Describe the anatomic structures of the upper, production and secretion in the stomach.
middle, and lower gastrointestinal (GI) tract.
14. Describe the function of the gastric mucosal
2. List the five layers of the GI tract wall and barrier.
describe their function.
15. Describe the functions of the secretions of
3. Characterize the structure and function of the small and the large intestine.
the peritoneum and describe its attachment
to the abdominal wall. 16. Describe and differentiate between anorexia,
nausea, and vomiting.
4. Characterize the properties of the interstitial
smooth muscle cells that act as pacemakers
for the GI tract. SECTION II: ASSESSING
5. Compare the actions of the enteric and auto-
YOUR UNDERSTANDING
nomic nervous systems as they relate to
Activity A Fill in the blanks.
motility of the GI tract.
6. Trace a bolus of food through the stages of 1. The major physiologic function of the
swallowing. is to digest food and absorb
nutrients into the bloodstream
7. Differentiate tonic and peristaltic movements
in the GI tract. 2. The upper esophageal sphincter, the
sphincter, consists of a circular
8. Describe the action of the internal and exter- layer of striated muscle.
nal sphincters in the control of defecation.
3. The lower esophageal sphincter, the
9. State the source and function of water and elec- sphincter, lies just above the
trolytes that are secreted in digestive secretions. area where the esophagus joins the stomach.
10. Explain the protective function of saliva. 4. The lies in the left side of the
11. Describe the function of the gastric secretions abdomen and serves as a food storage
in the process of digestion. reservoir during the early stages of digestion.

157
LWBK707-c28_p157-161.qxd 8/19/10 1:23 PM Page 158 Aptara Inc

158 UNIT 8 GASTROINTESTINAL AND HEPATOBILIARY FUNCTION

5. The small intestine, which forms the middle on food intake and digestive function, while
portion of the digestive tract, consists of reducing energy expenditure.
three subdivisions: the , 20. potentiates the action of
, and . secretin, increasing the pancreatic bicarbon-
6. Bile and pancreatic juices enter the intestine ate response to low circulating levels of
through openings for the common bile duct secretin, stimulates biliary secretion of fluid
and the main pancreatic duct in the and bicarbonate, and regulates gallbladder
. contraction and gastric emptying.
7. The cells carry out the secretory 21. The cells secrete hydrochloric
and absorptive functions of the GI tract and acid and intrinsic factor, which is necessary
they produce the that for the absorption of .
lubricates and protects the inner surface of 22. The chief cells secrete , an
the alimentary canal. enzyme that initiates proteolysis or breakdown
8. fluid forms a moist and slippery of proteins.
surface that prevents friction between the 23. G cells secrete .
continuously moving abdominal structures.
24. secrete large amounts of
9. The contains the blood vessels, alkaline mucus that protect the duodenum
nerves, and lymphatic vessels that supply the from the acid content in the gastric chyme
intestinal wall. and from the action of the digestive enzymes.
10. Like the self-excitable cardiac muscle cells in 25. The stomach and small intestine contain
the heart, some smooth muscle cells of the only a few species of , probably
GI tract function as cells. because of the composition of luminal
11. The nervous system consists contents.
of the myenteric and submucosal plexuses in 26. The major metabolic function of colonic
the wall of the GI tract. microflora is the fermentation of
12. monitor the stretch and disten- and endogenous mucus
tion of the GI tract wall, and produced by the epithelial cells.
monitor the chemical composition of its 27. is the process of dismantling
contents. foods into their constituent parts.
13. Numerous reflexes influence 28. is the process of moving nutri-
motility and secretions of the digestive tract. ents and other materials from the external
environment of the GI tract into the internal
14. Swallowing consists of three phases: an
environment.
phase, a phase,
and an phase. 29. Each villus is covered with cells called
that contribute to the absorptive
15. The is the major site for the
and digestive functions of the small bowel,
digestion and absorption of food.
and goblet cells that provide mucus.
16. normally is initiated by the
30. The enterocytes secrete that
mass movements of the large intestine.
adhere to the border of the villus structures.
17. The GI tract produces that act
31. Triglycerides are broken down by pancreatic
locally, pass into the general circulation for
.
distribution to more distant sites, and interact
with the central nervous system by way of 32. represents a loss of appetite.
the enteric and autonomic nervous systems. 33. is the conscious sensation
18. The primary function of is the resulting from stimulation of the medullary
stimulation of gastric acid secretion. vomiting center that often precedes or
accompanies vomiting.
19. has potent growth hormone-
releasing activity and has a stimulatory effect 34. is the sudden and forceful oral
expulsion of the contents of the stomach.

Copyright 2011. Wolters Kluwer Health | Lippincott Williams & Wilkins. Study Guide for Porths Essentials of Pathophysiology, Third Edition.
LWBK707-c28_p157-161.qxd 8/19/10 1:23 PM Page 159 Aptara Inc

CHAPTER 28 STRUCTURE AND FUNCTION OF THE GASTROINTESTINAL SYSTEM 159

Activity B Consider the following figures.

1. In the transfers section of the digestive tract Longitudinal muscle


above, locate and label the following Circular muscle
layers/structures:
Submucosa
Mesentery Mucosa
Muscularis mucosae Serosa (connective tissue)
Serosa (mesothelium) Muscularis externa

2. In the transfers section of the digestive tract


above, locate and label the following
layers/structures:
Extruded enterocyte
Enterocyte
Vein
Lacteal
Artery
Crypt of Lieberkhn

Copyright 2011. Wolters Kluwer Health | Lippincott Williams & Wilkins. Study Guide for Porths Essentials of Pathophysiology, Third Edition.
LWBK707-c28_p157-161.qxd 8/19/10 1:23 PM Page 160 Aptara Inc

160 UNIT 8 GASTROINTESTINAL AND HEPATOBILIARY FUNCTION

Activity C Match the key terms in Column A 4. Describe the incretin effect.
with their definitions in Column B.
Column A Column B
1. Amylase a. Responsible for
motility along the
2. Mastication 5. What are the three functions of saliva?
length of the gut
3. Mesentery b. Blood vessels,
4. Interstitial cells nerves, and
of Cajal lymphatic vessels
that supply the
5. Peritoneum intestinal wall
6. What is the mechanism of acid secretion by
6. Submucosal c. Breaks down starch the parietal cells of the stomach?
plexus d. Result of chemical
7. Haustrations breakdown of pro-
teins in stomach
8. Chyme
e. Chewing of food
9. Myenteric f. Generate slow
plexus 7. How are carbohydrates broken down to
waves of electrical
activity absorbable units?
10. Secretin
g. The largest serous
membrane in the
body
h. Segmental mixing
movements of the 8. Describe protein digestion and absorption.
large intestine
i. Controls function
of each segment of
intestinal tract
j. Inhibits gastric
acid secretion
Activity D SECTION III: APPLYING
1. Describe the functional divisions of the GI tract. YOUR KNOWLEDGE
Activity E Consider the scenario and answer
the question.
The nurse is preparing an educational event for a
group of children in elementary school who are
2. What factors are involved in stimulating the
studying the GI tract.
emptying of the stomach?
1. What facts would the nurse know to include
for these children?

3. Describe the two types of contractions seen in


the small intestine.

Copyright 2011. Wolters Kluwer Health | Lippincott Williams & Wilkins. Study Guide for Porths Essentials of Pathophysiology, Third Edition.
LWBK707-c28_p157-161.qxd 8/19/10 1:23 PM Page 161 Aptara Inc

CHAPTER 28 STRUCTURE AND FUNCTION OF THE GASTROINTESTINAL SYSTEM 161

SECTION IV: PRACTICING 6. Saliva has more than one function. What are
the functions of saliva? (Mark all that apply.)
FOR NCLEX
a. Protection
Activity F Answer the following questions. b. Lubrication
1. The circular layer of smooth muscle that lies c. Antibacterial
between the stomach and the small intestine d. Initiate digestion of starches
is called what? e. Initiate digestion of protein
a. Pyloric sphincter
7. The colon is home to between 300 and 500
b. Cardiac sphincter different species of bacteria. What is their
c. The antrum main metabolic function?
d. The cardiac orifice a. Digestion of insoluble fiber
2. Where in the GI tract is food digested and b. Fermentation of undigestible dietary
absorbed? residue
a. The colon and the ileum c. Compaction of metabolic waste prior to
leaving the body
b. The jejunum and ileum
d. Absorption of calcium
c. The stomach and the jejunum
d. The jejunum and the colon 8. Absorption is a major function of the GI
tract. How is absorption accomplished in the
3. Some smooth muscle cells in the GI tract GI tract?
serve as pacemakers. They display rhythmic
a. Osmosis and diffusion
spontaneous oscillations in membrane poten-
tials. What are these called? b. Active transport and osmosis
a. Peristalsis c. Active transport and diffusion
b. Intestinal spasms d. Diffusion and inactive transport
c. Slow waves 9. Nausea and vomiting can be side effects of
d. Rapid contractility many drugs as well as physiologic
disturbances within the body. What is a com-
4. Defecation is controlled by both an internal mon cause of nausea?
and an external sphincter. What nerve
a. Distention of the stomach
controls the external sphincter?
b. Distention of the cecum
a. Vagus nerve
c. Distention of the jejunum
b. Femoral nerve
d. Distention of the duodenum
c. Phrenic nerve
d. Pudendal nerve 10. Several neurotransmitters have been
identified with nausea and vomiting. In this
5. The stomach secretes two important hormones capacity they act as neuromediators. What
in the GI tract. One is gastrin. What is the sec- neuromediator is thought to be involved in
ond hormone secreted by the stomach? the nausea and vomiting that accompanies
a. Ghrelin chemotherapy?
b. Secretin a. Serotonin
c. Incretin b. Dopamine
d. Cholecystokinin c. Acetylcholine receptors
d. Opioid receptors

Copyright 2011. Wolters Kluwer Health | Lippincott Williams & Wilkins. Study Guide for Porths Essentials of Pathophysiology, Third Edition.
LWBK707-c29_p162-168.qxd 9/2/10 7:50 AM Page 162 Aptara Inc

29
CHAPTER
Disorders of
Gastrointestinal Function

SECTION I: LEARNING 11. State the diagnostic criteria for irritable bowel
syndrome.
OBJECTIVES
12. Compare the characteristics of Crohn disease
1. Define and cite the causes of dysphagia, and ulcerative colitis.
odynophagia, and achalasia.
13. Relate an increase in dietary fiber to the
2. Relate the pathophysiology of gastroesophageal treatment of diverticular disease.
reflux to measures used in the diagnosis and
14. Describe the pathogenesis of the symptoms
treatment of the disorder in adults and
associated with appendicitis.
children.
15. Compare the causes and manifestations of
3. State the reason for the poor prognosis asso-
small-volume diarrhea and large-volume
ciated with esophageal cancer.
diarrhea.
4. Describe the anatomic and physiologic
16. Explain why a failure to respond to the defe-
factors that contribute to the gastric mucosal
cation urge may result in constipation.
barrier.
17. Differentiate between mechanical and
5. Differentiate between the causes and
paralytic intestinal obstruction in terms of
manifestations of acute and chronic gastritis.
cause and manifestations.
6. Characterize the proposed role of Helicobacter
18. Describe the characteristics of the
pylori in the development of chronic gastritis
peritoneum that increase its vulnerability to
and peptic ulcer and cite methods for
and protect it against the effects of peritoni-
diagnosis and treatment of the infection.
tis.
7. Describe the predisposing factors in develop-
19. List three causes of intestinal malabsorption
ment of peptic ulcer and cite the three com-
and describe their manifestations.
plications of peptic ulcer.
20. Describe the pathophysiology of celiac
8. Describe the goals for pharmacologic
disease.
treatment of peptic ulcer disease.
21. List the risk factors associated with colorectal
9. Cite the etiologic factors in ulcer formation
cancer and cite the screening methods for
related to Zollinger-Ellison syndrome and
detection.
stress ulcer.
10. List risk factors associated with gastric cancer.

162
LWBK707-c29_p162-168.qxd 9/2/10 7:50 AM Page 163 Aptara Inc

CHAPTER 29 DISORDERS OF GASTROINTESTINAL FUNCTION 163

SECTION II: ASSESSING 14. is denoted by the absence of


grossly visible erosions and the presence of
YOUR UNDERSTANDING chronic inflammatory changes leading even-
tually to atrophy of the glandular epithelium
Activity A Fill in the blanks.
of the stomach.
1. The functions primarily as a
15. Autoimmune gastritis results from the
conduit for passage of food and liquid from
presence of to components of
the pharynx to the stomach.
gastric gland parietal cells and intrinsic
2. anomalies of the esophagus factor.
require early detection and correction
16. is a term used to describe a
because they are incompatible with life.
group of ulcerative disorders that occur in
3. can result from disorders that areas of the upper gastrointestinal tract that
produce narrowing of the esophagus, lack of are exposed to acid-pepsin secretions.
salivary secretion, weakness of the muscular
17. The most common complications of peptic
structures that propel the food bolus, or neu-
ulcer are , perforation and pene-
ral networks coordinating the swallowing
tration, and gastric outlet .
mechanism.
18. Laboratory findings of hypochromic anemia
4. is characterized by a protrusion
and occult blood in the stools indicate
of the stomach through the esophageal
.
hiatus of the diaphragm.
19. is the major physiologic media-
5. The most frequent symptom of
tor for hydrochloric acid secretion.
is heartburn.
20. Persons at high risk for development of
6. There is considerable evidence linking
include those with large
gastroesophageal reflux with .
surface area burns, trauma, sepsis, acute respi-
7. involves mucosal injury to the ratory distress syndrome, severe liver failure,
esophagus, hyperemia, and inflammation. and major surgical procedures.
8. Symptoms of reflux esophagitis in an 21. Gastric is the second most
include evidence of pain when common tumor in the world.
swallowing, hematemesis, and anemia due to
22. is a functional gastrointestinal
esophageal bleeding, heartburn, irritability,
disorder characterized by a variable combina-
and sudden or inconsolable crying.
tion of chronic and recurrent intestinal
9. Most squamous cell esophageal carcinomas symptoms not explained by structural or
are attributable to and biochemical abnormalities.
use.
23. The term inflammatory bowel disease is used to
10. The stomach lining usually is designate two related inflammatory intestinal
to the acid it secretes, a property that allows disorders: disease and
the stomach to contain acid and pepsin with- .
out having its wall digested.
24. disease is a recurrent, granulo-
11. The are thought to exert their matous type of inflammatory response that
effect through improved mucosal blood flow, can affect any area of the gastrointestinal
decreased acid secretion, increased bicarbon- tract.
ate ion secretion, and enhanced mucus
25. Ulcerative colitis is confined to
production.
and .
12. refers to inflammation of the
26. deficiencies are common in
gastric mucosa.
Crohn disease because of diarrhea,
13. is most commonly associated steatorrhea, and other malabsorption
with local irritants such as aspirin or other problems.
nonsteroidal anti-inflammatory agents, alco-
hol, or bacterial toxins.

Copyright 2011. Wolters Kluwer Health | Lippincott Williams & Wilkins. Study Guide for Porths Essentials of Pathophysiology, Third Edition.
LWBK707-c29_p162-168.qxd 9/2/10 7:50 AM Page 164 Aptara Inc

164 UNIT 8 GASTROINTESTINAL AND HEPATOBILIARY FUNCTION

27. Characteristic of ulcerative colitis are the 42. Celiac disease is an immune-mediated disor-
lesions that form in the crypts of der triggered by ingestion of
in the base of the mucosal containing grains.
layer.
43. provides a means for direct
28. of the colon is one of the feared visualization of the rectum and colon.
complications of ulcerative colitis.
Activity B Match the key terms in Column A
29. The complications of result
with their definitions in Column B.
from massive fluid loss or destruction of
intestinal mucosa. 1.
30. is a condition in which the Column A Column B
mucosal layer of the colon herniated through
1. Achalasia a. Swallowing is
the muscularis layer.
painful
2. Esophageal
31. is a complication of diverticulo- b. Most common
atresia
sis in which there is inflammation and gross cause of chronic
or microscopic perforation of the 3. Odynophagia gastritis in the
diverticulum. United States
4. Gastroesophageal
32. The pain associated with is reflux c. An ulcer erodes
caused by stretching of the appendix during through all the
5. Dysphagia
the early inflammatory process. layers of the
6. Barrett stomach
33. The usual definition of is exces-
esophagus d. Esophagus is con-
sively frequent passage of stools.
7. Tracheoeso- nected to the
34. Toxin-producing bacteria or other agents that trachea
phageal fistulae
disrupt the normal absorption or secretory
e. Backward
process in the small bowel commonly cause 8. Mallory-Weiss
movement of gas-
. syndrome
tric contents into
35. diarrhea is often associated 9. Perforation the esophagus
with conditions such as inflammatory bowel The upper esopha-
10. Helicobacter pylori f.
disease, irritable bowel syndrome, malabsorp- gus ends in a blind
tion syndrome, endocrine disorders, or radia- pouch
tion colitis.
g. Difficulty passing
36. commonly is associated with food into the
acute or chronic inflammation or intrinsic stomach
disease of the colon, such as ulcerative colitis h. Squamous mucosa
or Crohn disease. that lines the
37. can be defined as the esophagus gradu-
infrequent and/or difficult passage of stools. ally is replaced
by columnar
38. is the retention of hardened or epithelium
puttylike stool in the rectum and colon,
i. Tears in the esoph-
which interferes with normal passage of
agus at the esoph-
feces.
agogastric
39. Intestinal obstruction designates an junction
impairment of movement of intestinal j. Difficulty in
contents in a direction. swallowing
40. obstruction results from neuro-
genic or muscular impairment of peristalsis.
41. Peritonitis is an inflammatory response of the
that lines the abdominal cavity
and covers the visceral organs.

Copyright 2011. Wolters Kluwer Health | Lippincott Williams & Wilkins. Study Guide for Porths Essentials of Pathophysiology, Third Edition.
LWBK707-c29_p162-168.qxd 9/2/10 7:50 AM Page 165 Aptara Inc

CHAPTER 29 DISORDERS OF GASTROINTESTINAL FUNCTION 165

2. 2. Describe how the gastric mucosal barrier func-


tions to protect the stomach from it own
Column A Column B
secretions.
1. Fistulas a. Immune-mediated
disorder triggered
2. Zollinger-Ellison
by ingestion of
syndrome
gluten-containing
3. Celiac disease grains
Water is pulled 3. Describe the progression and remission of
4. Osmotic diarrhea b.
into the bowel by peptic ulcers.
5. Hypergastrinemia the hyperosmotic
6. Steatorrheic nature of its
contents
7. Cobblestone
c. Tubelike passages
appearance
that form connec-
4. What is the relationship between H. pylori infec-
8. Penetration tions between dif-
tion and the development of stomach cancer?
ferent sites in the
9. Adenomatous
gastrointestinal
polyps
tract
10. Rotavirus d. Hallmark
symptom of
Crohn disease 5. What are the typical characteristics of irritable
e. Presence of an bowel syndrome?
excess of gastrin in
the blood
f. Ulcer crater erodes
into adjacent
organs
6. What is hypothesized to be a cause of inflam-
g. Gastrin-secreting
matory bowel disease (ulcerative colitis and
tumor
Crohn disease)?
h. Causes diarrhea in
children
i. Benign neoplasms
that arise from the
mucosal
epithelium of the 7. What is the mechanism of diverticulosis forma-
intestine tion?
j. Stools contain
excess fat

Activity C Briefly answer the following.


1. What is GERD? What is the mechanism of 8. What is the pathophysiology of constipation?
damage?

Copyright 2011. Wolters Kluwer Health | Lippincott Williams & Wilkins. Study Guide for Porths Essentials of Pathophysiology, Third Edition.
LWBK707-c29_p162-168.qxd 9/2/10 7:50 AM Page 166 Aptara Inc

166 UNIT 8 GASTROINTESTINAL AND HEPATOBILIARY FUNCTION

9. How does diet expose a patient to colon cancer? SECTION IV: PRACTICING
FOR NCLEX
Activity E Answer the following questions.
1. Hiatal hernias can cause severe pain if the
hernia is large. Gastroesophageal reflux is a
common comorbidity of hiatal hernia, and,
SECTION III: APPLYING when this occurs, what might the hernia do?
YOUR KNOWLEDGE a. Increase esophageal acid clearance
b. Retard esophageal acid clearance
Activity D Consider the scenario and answer
c. Decrease esophageal acid clearance
the questions.
d. Accelerate esophageal acid clearance
A 67-year-old black man presents at the clinic
with complaints of difficulty swallowing foods of 2. Infants and children commonly have gastroe-
any kind. He states, It always feels like I have sophageal reflux. Many times it is asympto-
something caught in my throat. His medical matic and resolves on its own. What are the
history is significant for Barrett esophagus, unin- signs and symptoms of gastroesophageal reflux
tentional weight loss of 15 pounds over past 4 in infants with severe disease?
months, and some pain when swallowing. The a. Consolable crying and early satiety
gentleman is scheduled for an esophagoscopy, b. Delayed satiety and sleeping after feeding
and a diagnosis of esophageal cancer is
c. Tilting of the head to one side and arching
subsequently confirmed. The physician explains
of the back
that, depending on the stage of the tumor, there
are options for treatment. The physician recom- d. Inconsolable crying and delayed satiety
mends chemotherapy followed by surgical resec- 3. The stomach secretes acid to begin the diges-
tion of the tumor. tive process on the food that we eat. The gas-
1. The man arrives for his first treatment of tric mucosal barrier works to prevent acids
chemotherapy and asks the nurse why he has secreted by the stomach from actually
to have chemotherapy before having the sur- damaging the wall of the stomach. What are
gery to remove the tumor. The nurse correctly the factors that make up the gastric mucosal
responds by stating: barrier? (Mark all that apply.)
a. An impermeable epithelial cell surface
covering
b. Mechanisms for selective transport of
bicarbonate and potassium ions
c. Characteristics of gastric mucus
2. Subsequent studies show that this clients
tumor has already metastasized. The physician d. Cell coverings that act as antacids
recommends that surgery be done right away, e. Mechanisms for selective transport of
but emphasizes to the client that there is no hydrogen and bicarbonate ions
cure for his cancer. The client arrives for
4. Helicobacter pylori gastritis has a prevalence of
surgery and asks the preoperative nurse why
over 50% of American adults over the age of
he needs the surgery if it will not cure his can-
50, which is thought to be caused by a previous
cer. What would be the correct response by
infection when the client was younger. What
the nurse?
can chronic gastritis caused by H. pylori cause?
a. Decreased risk of gastric adenocarcinoma
b. Decreased risk of low-grade B-cell gastric
lymphoma
c. Duodenal ulcer
d. Gastric atrophy

Copyright 2011. Wolters Kluwer Health | Lippincott Williams & Wilkins. Study Guide for Porths Essentials of Pathophysiology, Third Edition.
LWBK707-c29_p162-168.qxd 9/2/10 7:50 AM Page 167 Aptara Inc

CHAPTER 29 DISORDERS OF GASTROINTESTINAL FUNCTION 167

5. A 39-year-old white woman presents at the 8. Crohn disease is a recurrent inflammatory


clinic with complaints of epigastric pain that disease that can affect any area of the bowel.
is cramplike, rhythmic, and just below the Characteristic of Crohn disease is granuloma-
xiphoid. She states that it wakes her up tous lesions that are sharply demarcated from
around 1 AM, and she is not sleeping well the surrounding tissue. As the nurse caring
because of it. She further states that this is for a client with newly diagnosed Crohn dis-
the third episode of having this pain in the ease, you would know to include what in
past year. The nurse suspects the client has a your teaching?
peptic ulcer and expects to receive what a. Definition of Crohn disease that includes
orders from the physician? that it is a recurrent disease that affects
a. Schedule client for a complete metabolic only the large intestine.
panel and a complete blood count b. Information on which nonsteroidal anti-
b. Schedule client for laparoscopic examina- inflammatory drugs to take and how often
tion to take them.
c. Schedule client for a swallow study c. Information on sulfasalazine including
d. Schedule client for a lower gastrointestinal dosage, route, frequency, and side effects
study of the drug.
d. Information on the chemotherapy that
6. A client in a nursing home complains to her
will be ordered to cure the disease.
nurse that she is not feeling well. When
asked to describe how she feels, the client 9. Rotavirus is a common infection in children
states that she really is not hungry anymore younger than 5 years of age. Like other
and seems to have indigestion a lot. The diseases, rotavirus is most severe in children
nurse checks the clients chart and finds that under 24 months of age. What is a symptom
her vital signs are normal, but that she has of rotavirus infection?
lost weight over the past 2 months. She also a. Mild to moderate fever that gets higher
notes that there is a history of gastric cancer after the second day
in the clients family. The nurse notifies the
b. Vomiting that lasts for the course of the
physician and expects to receive what orders?
disease
(Mark all that apply.)
c. Fever that disappears after 7 days
a. Schedule a barium radiograph and an en-
doscopy d. Vomiting that disappears around the
second day
b. Perform a Papanicolaou smear on the
clients gastric secretions 10. Diverticulitis is the herniation of tissue of the
c. Order cytologic studies to be done during large intestine through the muscularis layer
the endoscopy of the colon. It is often asymptomatic and is
found in approximately 80% of people over
d. Schedule a lower gastrointestinal study
the age of 85. Diverticulitis is often asympto-
e. Have the technician do an endoscopic ul- matic, but when symptoms do occur what is
trasound. the most common complaint of the client?
7. Irritable bowel syndrome is thought to be pres- a. Lower left quadrant pain with nausea and
ent in 10% to 15% of the population in the vomiting
United States. What is its hallmark symptom? b. Right lower quadrant pain with nausea
a. Nausea and abdominal pain unrelieved by and vomiting
defecation. c. Midepigastric pain with nausea and vomit-
b. Abdominal pain relieved by defecation ing
with a change in consistency or frequency d. Right lower quadrant pain with rebound
of stools. tenderness on the left
c. Diarrhea and abdominal pain unrelieved
by defection.
d. Abdominal pain relieved by defecation
and bowel impaction.

Copyright 2011. Wolters Kluwer Health | Lippincott Williams & Wilkins. Study Guide for Porths Essentials of Pathophysiology, Third Edition.
LWBK707-c29_p162-168.qxd 9/2/10 7:50 AM Page 168 Aptara Inc

168 UNIT 8 GASTROINTESTINAL AND HEPATOBILIARY FUNCTION

11. Diarrhea is described as a change in frequency 13. Celiac disease commonly presents in infancy
of stool passage to a point where it is as failure to thrive. It is an inappropriate
excessively frequent. Diarrhea can be acute or T-cellmediated immune response and there
chronic, inflammatory, or noninflammatory. is no cure for it. What is the treatment of
What are the symptoms of noninflammatory choice for celiac disease?
diarrhea? (Mark all that apply.) a. Removal of protein from the diet
a. Small volume watery stools b. Removal of fat from the diet
b. Nonbloody stools c. Removal of gluten from the diet
c. Periumbilical cramps d. Removal of sugar from the diet
d. Nausea and/or vomiting
14. One of the accepted methods of screening for
e. Large-volume blood stools colorectal cancer is testing for occult blood in
12. Peritonitis is an inflammatory condition of the the stool. Because it is possible to get a false-
lining of the abdominal cavity. What is one of positive result on these tests, you would
the most important signs of peritonitis? instruct the client to do what?
a. Vomiting of coffee ground-appearing a. Eat lots of red meat for 3 or 4 days before
emesis the test is done.
b. The translocation of extracellular fluid b. Take 1000 mg of vitamin C in supplement
into the peritoneal cavity form for 1 week prior to testing.
c. The translocation of intracellular fluid into c. Eat citrus fruits at least 5 times a day for
the peritoneal cavity 2 days prior to testing.
d. Vomiting of bloody emesis d. Avoid nonsteroidal anti-inflammatory
drugs for 1 week prior to testing.

Copyright 2011. Wolters Kluwer Health | Lippincott Williams & Wilkins. Study Guide for Porths Essentials of Pathophysiology, Third Edition.
LWBK707-c30_p169-175.qxd 8/19/10 1:25 PM Page 169 Aptara Inc

30

CHAPTER
Disorders of
Hepatobiliary and
Exocrine Pancreas
Function

SECTION I: LEARNING 8. Characterize the metabolism of alcohol by


OBJECTIVES the liver and state metabolic mechanisms
that can be used to explain liver injury.
1. Describe the function of the liver in terms of 9. Summarize the three patterns of injury that
carbohydrate, protein, and fat metabolism. occur with alcohol-induced liver disease.
2. Characterize the function of the liver in 10. Describe the pathogenesis of intrahepatic
terms of bilirubin elimination and describe biliary disorders.
the pathogenesis of unconjugated and conju-
gated hyperbilirubinemia. 11. Characterize the liver changes that occur
with cirrhosis.
3. Relate the mechanism of bile formation and
elimination to the development of cholestasis. 12. Describe the physiologic basis for portal
hypertension and relate it to the
4. List four laboratory tests used to assess liver development of ascites, esophageal varices,
function and relate them to impaired liver and splenomegaly.
function.
13. Relate the functions of the liver to the mani-
5. State the three ways by which drugs and festations of liver failure.
other substances are metabolized or
inactivated in the liver and provide examples 14. Characterize etiologies of hepatocellular can-
of liver disease related to the toxic effects of cer and state the reason for the poor progno-
drugs and chemical agents. sis in persons with this type of cancer.
6. Compare hepatitis A, B, C, D, and E in terms 15. Explain the function of the gallbladder in
of source of infection, incubation period, regulating the flow of bile into the
acute disease manifestations, development duodenum and relate to the formation of
of chronic disease, and the carrier state. cholelithiasis (gallstones).
7. Define chronic hepatitis and compare the 16. Describe the clinical manifestations of acute
pathogenesis of chronic autoimmune and and chronic cholecystitis.
chronic viral hepatitis.

169
LWBK707-c30_p169-175.qxd 8/19/10 1:25 PM Page 170 Aptara Inc

170 UNIT 8 GASTROINTESTINAL AND HEPATOBILIARY FUNCTION

17. Characterize the effects of choledocholithia- 10. represents a decrease in bile


sis and cholangitis on bile flow and the flow through the intrahepatic canaliculi and
potential for hepatic and pancreatic a reduction in secretion of water, bilirubin,
complications. and bile acids by the hepatocytes.
18. Cite the possible causes and describe the man- 11. Common to all types of obstructive and hepa-
ifestations and treatment of acute pancreatitis. tocellular cholestasis is the accumulation of
pigment in the liver.
19. Describe the manifestations of chronic
pancreatitis. 12. jaundice occurs when red
blood cells are destroyed at a rate in excess of
20. State the reason for the poor prognosis in
the livers ability to remove the bilirubin
pancreatic cancer.
from the blood.
13. of bilirubin is impaired when-
SECTION II: ASSESSING ever liver cells are damaged, when transport
of bilirubin into liver cells becomes deficient,
YOUR UNDERSTANDING or when the enzymes needed to conjugate
the bile are lacking.
Activity A Fill in the blanks.
14. result in chemical modification
1. The liver, the gallbladder, and the exocrine
of reactive drug groups by oxidation,
pancreas are classified as organs
reduction, hydroxylation, or other chemical
of the gastrointestinal tract.
reactions carried out in hepatocytes.
2. Approximately 300 mL of blood per minute
15. Drugs such as alcohol and barbiturates can
enters the liver through the hepatic
induce certain members of the
; another 1050 mL/minute
family to increase enzyme production, accel-
enters by way of the .
erating drug metabolism and decreasing the
3. The venous blood delivered by the pharmacologic action of the drug.
comes from the digestive tract
16. , which involve the conversion
and major abdominal organs, including the
of lipid-soluble derivatives to water-soluble
pancreas and spleen.
substances, may follow phase 1 reactions or
4. A major exocrine function of the liver is proceed independently.
secretion.
17. Direct hepatotoxic reactions result from drug
5. The most important of the secretory proteins metabolism and the generation of
of the liver is . .
6. Acetyl-CoA units from fat metabolism also 18. drug reactions result in
are used to synthesize and decreased secretion of bile or obstruction of
acids in the liver. the biliary tree.
7. Almost all the synthesis in the 19. refers to inflammation of the
body from carbohydrates and proteins occurs liver.
in the liver.
20. Currently, recreational use is
8. Whenever a greater quantity of carbohydrates the most common mode of hepatitis C virus
enters the body than can be immediately transmission in the United States and
used, the excess is converted to Canada. The main route of transmission of
in the liver. hepatitis C virus in the past was through
contaminated or blood
9. Bile salts serve an important function in
products and
digestion; they aid in dietary
fats, and they are necessary for the formation 21. hepatitis is a severe type of
of the that transport fatty acids chronic hepatitis of unknown origin that is
and fat-soluble vitamins to the surface of the associated with high levels of serum
intestinal mucosa for absorption. immunoglobulins, including autoantibodies.

Copyright 2011. Wolters Kluwer Health | Lippincott Williams & Wilkins. Study Guide for Porths Essentials of Pathophysiology, Third Edition.
LWBK707-c30_p169-175.qxd 8/19/10 1:25 PM Page 171 Aptara Inc

CHAPTER 30 DISORDERS OF HEPATOBILIARY AND EXOCRINE PANCREAS FUNCTION 171

22. biliary diseases disrupt the flow 37. The pancreas is made up of lob-
of bile through the liver, causing cholestasis ules that consist of acinar cells, which secrete
and biliary cirrhosis. digestive enzymes into a system of
microscopic ducts.
23. biliary cirrhosis results from
prolonged obstruction of the extrabiliary tree. 38. Acute represents a reversible
inflammatory process of the pancreatic acini
24. Obesity, type 2 diabetes, the metabolic
brought about by premature activation of
syndrome, and hyperlipidemia are coexisting
pancreatic enzymes.
conditions frequently associated with
liver disease. 39. is characterized by progressive
destruction of the exocrine pancreas, fibrosis,
25. represents the end stage of
and in the later stages, by destruction of the
chronic liver disease in which much of the
endocrine pancreas.
functional liver tissue has been replaced by
fibrous tissue. 40. The most significant and reproducible
environmental risk factor of pancreatic can-
26. is characterized by increased
cer is
resistance to flow in the portal venous system
and sustained portal vein pressure above
Activity B Consider the following figure.
12 mm Hg.
27. Complications of portal hypertension arise
from the pressure and
of the venous channels behind
the obstruction.
28. occurs when the amount of
fluid in the peritoneal cavity is increased.
29. is a complication in persons
with both cirrhosis and ascites.
30. The syndrome refers to a func-
tional renal failure sometimes seen during the
terminal stages of liver failure with ascites.
31. Hepatic refers to the totality of
central nervous system manifestations of
liver failure.
In the figure above, label the following
32. Among the factors identified as etiologic
structures:
agents in are chronic viral
hepatitis, cirrhosis, long-term exposure to Liver
environmental agents such as aflatoxin, and Gallbladder
drinking water contaminated with arsenic.
Cystic duct
33. The is a distensible, pear- Common bile duct
shaped, muscular sac located on the ventral
Duodenum
surface of the liver.
Tail of pancreas
34. provides a strong stimulus for
Head of pancreas
gallbladder contraction and is released when
food enters the intestines. Pancreatic duct
Hepatic duct
35. Gallstones are caused by precipitation of
substances contained in bile, mainly Spleen
and . Diaphragm
36. Acute is a diffuse inflammation Ampulla of Vater
of the gallbladder, usually secondary to Sphincter of Oddi
obstruction of the gallbladder outlet.

Copyright 2011. Wolters Kluwer Health | Lippincott Williams & Wilkins. Study Guide for Porths Essentials of Pathophysiology, Third Edition.
LWBK707-c30_p169-175.qxd 8/19/10 1:25 PM Page 172 Aptara Inc

172 UNIT 8 GASTROINTESTINAL AND HEPATOBILIARY FUNCTION

Activity C Match the key terms in Column A 2.


with their definitions in Column B. Column A Column B
1. 1. Hepatitis A a. Infection is linked
Column A Column B to hepatitis B
2. Hepatitis B
b. Does not cause
1. Kupffer cells a. formed from 3. Hepatitis C chronic hepatitis
senescent red
2. Albumin 4. Hepatitis D or the carrier state
blood cells
3. Gluconeogenesis c. Inoculation with
b. conversion of 5. Hepatitis E
infected blood
4. Oxidative amino acids to
and/or spread by
deamination ketoacids and
oral or sexual
ammonia
5. Beta oxidation contact
c. capable of remov-
d. Occurs primarily
6. Extrahepatic ing and phagocy-
by the fecal-oral
cholestasis tosing old and
route
defective blood
7. Bilirubin e. The most common
cells
8. Jaundice cause of chronic
d. abnormally high
hepatitis, cirrhosis,
9. Steatosis accumulation of
and hepatocellular
bilirubin in the
10. Cholestatic cancer in the
blood
jaundice world
e. the splitting of
fatty acids into Activity D
two-carbon acetyl-
coenzyme A
f. transport
protein/plasma Gluconeogenesis
colloidal osmotic
pressure
g. obstruction of the
large bile ducts
that reduces bile
secretion
Bloodstream
h. amino acids are
used for producing 1. Complete the flowchart above for the hepatic
glucose pathways for the storage and synthesis of glu-
i. fatty infiltration of cose
the liver Triglycerides
j. occurs when bile Glucose
flow is obstructed
between the liver Amino acids
and the intestine Glycogen
Glycerol
Lactic acid

Copyright 2011. Wolters Kluwer Health | Lippincott Williams & Wilkins. Study Guide for Porths Essentials of Pathophysiology, Third Edition.
LWBK707-c30_p169-175.qxd 8/19/10 1:25 PM Page 173 Aptara Inc

CHAPTER 30 DISORDERS OF HEPATOBILIARY AND EXOCRINE PANCREAS FUNCTION 173

Portal hypertension

2. Complete the flowchart above using the 3. List the major causes and categories of
following terms. jaundice.
Increased pressure in peritoneal capillaries
Portosystemic shunting of blood
Splenomegaly
Ascites
4. What is measured in the serum to asses liver
Development of collateral channels
dysfunction?
Shunting of ammonia and toxins into gen-
eral circulation
Anemia
Leukopenia
Thrombocytopenia
5. Describe the clinical course of viral hepatitis.
Hepatic encephalopathy
Hemorrhoids
Esophageal varices
Caput medusae
Bleeding 6. How does ethanol cause tissue damage?

Activity E Briefly answer the following.


1. What are the basic functions of the liver?

7. What changes take place in the liver resulting


from the toxic affects of alcohol?

2. Describe the pathogenesis of cholestasis.

Copyright 2011. Wolters Kluwer Health | Lippincott Williams & Wilkins. Study Guide for Porths Essentials of Pathophysiology, Third Edition.
LWBK707-c30_p169-175.qxd 8/19/10 1:25 PM Page 174 Aptara Inc

174 UNIT 8 GASTROINTESTINAL AND HEPATOBILIARY FUNCTION

8. What is cirrhosis of the liver? SECTION IV: PRACTICING


FOR NCLEX
Activity G Answer the following questions.
1. The liver has many jobs. One of the most
9. What are the factors that lead to the develop- important functions of the liver is to cleanse
ment of ascites? the portal blood of old and defective blood
cells, bacteria in the bloodstream, and any
foreign material. Which cells in the liver are
capable of removing bacteria and foreign
material from the portal blood?
a. Kupffer cells
10. How does biliary venous obstruction lead to
b. Langerhans cells
hemorrhoid formation?
c. Epstein cells
d. Davidoff cells
2. Cholestasis is a condition in which there is a
decrease in bile flow through the intrahepatic
canaliculi and a reduction in secretion of
water, bilirubin, and bile acids by the hepato-
cytes. Cholestasis can have more than one
SECTION III: APPLYING cause, but, in all types of cholestasis, there is
YOUR KNOWLEDGE what?
a. Accumulation of bile pigment in the gall-
Activity F Consider the scenario and answer
bladder
the questions.
b. Accumulation of bile pigment in the
A 16-year-old female patient is brought to the liver
clinic by her mother. She complains of recurrent
c. Accumulation of bile pigment in the blood
fatigue and loss of appetite. Her mother states, I
am concerned because she has a yellow look in d. Accumulation of bile pigment in the por-
her eyes. It sort of comes and goes. While taking tal vein
the clients history, the nurse finds that the client 3. What is considered the normal amount of
became sexually active 1 year ago and has had serum bilirubin found in the blood?
multiple partners during the past 12 months. On
a. 1 to 2 mg/dL
physical examination the physician notes an
enlarged liver. The presumptive diagnosis is b. 0.01to 0.02 mg/dL
hepatitis C. c. 0.1 to 0.2 mg/dL
1. What confirmatory tests would the nurse d. 0.001to 0.002 mg/dL
expect to be ordered? 4. Many drugs are metabolized and detoxified
in the liver. Most drug metabolizing occurs
in the central zones of the liver. What condi-
tion is caused by these drug-metabolizing
actions?
a. Central cirrhosis
2. The clients tests come back positive for hepa-
titis C. What medications might be ordered b. Lobular cirrhosis
for this client? c. Lobular necrosis
d. Centrilobular necrosis

Copyright 2011. Wolters Kluwer Health | Lippincott Williams & Wilkins. Study Guide for Porths Essentials of Pathophysiology, Third Edition.
LWBK707-c30_p169-175.qxd 8/19/10 1:25 PM Page 175 Aptara Inc

CHAPTER 30 DISORDERS OF HEPATOBILIARY AND EXOCRINE PANCREAS FUNCTION 175

5. Primary biliary cirrhosis is an autoimmune 9. Gall stones are made up mostly of


disease that destroys the small intrahepatic cholesterol. What is thought to be a precursor
bile ducts causing cholestasis. It is insidious of gallstones?
in onset and is a progressive disease. What a. Gallbladder sludge
are the earliest symptoms of the disease?
b. Thinned mucoprotein
a. Unexplained pruritus
c. Pieces of hard food trapped in the
b. Weight gain gallbladder
c. Pale urine d. Thickened bile
d. Dark stools
10. What laboratory markers are most commonly
6. One of the jobs the liver performs is to export used to diagnose acute pancreatitis?
triglyceride. When the livers capacity to a. Amylase and cholesterol
export triglyceride is maximized, excess fatty
b. Lipase and amylase
acids accumulate in the liver. What is the dis-
ease these excess fatty acids contribute to? c. Lipase and triglycerides
a. Biliary cirrhosis d. Cholesterol and triglycerides
b. Nonalcoholic fatty liver disease 11. All diseases have risk factors. What is the
c. Cholelithiasis most significant environmental risk factor for
pancreatic cancer?
d. Alcoholic fatty liver disease
a. Air pollution
7. Ascites is an accumulation of fluid in the
b. Water pollution
peritoneal cavity and usually occurs in
advanced cirrhosis. What is the treatment of c. Cigarette smoking
choice for ascites? d. Heavy metal toxicity
a. Paracentesis
b. Thoracentesis
c. Diuretics
d. DDAVP
8. A client is suspected of having liver cancer.
What diagnostic tests would be ordered to
confirm the diagnosis?
a. Serum -fetoprotein
b. Endoscopy
c. Ultrasound of liver
d. MRI of liver

Copyright 2011. Wolters Kluwer Health | Lippincott Williams & Wilkins. Study Guide for Porths Essentials of Pathophysiology, Third Edition.
LWBK707-c31_p176-179.qxd 9/2/10 7:52 AM Page 176 Aptara Inc

31
CHAPTER
Mechanisms of
Endocrine Control

SECTION I: LEARNING 2. The functions of the endocrine system are


closely linked with those of the
OBJECTIVES system and the system.
1. Characterize a hormone. 3. When hormones act locally on cells other
than those that produced the hormone, the
2. Differentiate vesicle-mediated and non
action is called .
vesicle-mediated mechanisms of hormone
synthesis in terms of their stimuli for 4. Hormones also can exert an
hormone synthesis and release. action on the cells from which they were
produced.
3. Describe mechanisms of hormone transport
and inactivation. 5. Hormones that are released into the
bloodstream circulate either as
4. State the function of a hormone receptor and
molecules, or as hormones to
the difference between cell surface hormone
transport carriers.
receptors and nuclear hormone receptors.
6. Hormones produce their effects through
5. Describe the role of the hypothalamus in regu-
interaction with , which in turn
lating pituitary control of endocrine function.
are linked to one or more effector systems
6. State the major difference between positive within the cell.
and negative feedback control mechanisms.
7. The structure of hormone
7. Describe methods used in diagnosis of varies in a manner that allows target cells to
endocrine disorders. respond to one hormone and not to others.
8. hormones attach to intracellu-
lar receptors and form a hormone-receptor
SECTION II: ASSESSING complex that travels to the cell nucleus.
YOUR UNDERSTANDING 9. The synthesis and release of anterior pituitary
hormones is regulated by the action of releas-
Activity A Fill in the blanks. ing or inhibiting hormones from the
1. The endocrine system uses chemical , which is the coordinating cen-
substances called as a means of ter of the brain for endocrine, behavioral,
regulating and integrating body functions. and autonomic nervous system function.

176
LWBK707-c31_p176-179.qxd 9/2/10 7:52 AM Page 177 Aptara Inc

CHAPTER 31 MECHANISMS OF ENDOCRINE CONTROL 177

10. The pituitary gland has been called the 3. How do tissues regulate a hormones affect?
because its hormones control
the functions of many target glands and cells.
11. The easiest way to measure hormone levels
during a specific period are by either blood
samples or urine tests to measure
4. What are the main types of cell membrane
or .
receptors and how do they exert their effects?
Activity B Match the key terms in Column A
with their definitions in Column B.
Column A Column B
1. Autocrine a. Time it takes for
5. Describe the global role of the anterior
the body to reduce
2. Half-life of a pituitary hormones.
the concentration
hormone
of the hormone by
3. Hormones one-half
4. Hypophysis b. Hormone acts on
cell that produced
5. Paracrine it 6. How does negative feedback regulate
6. Second c. Hormone affecting hormone levels?
messenger neighboring cells
7. Hormone d. The hypothalamus
response and the pituitary
element e. Highly specialized
organic molecules
produced by
endocrine organs
that exert their
SECTION III: APPLYING
action on specific YOUR KNOWLEDGE
target cells
Activity D Consider the scenario and answer
f. Intracellular signal
the questions.
g. Activate or suppress
intracellular mech- An 87-year-old woman has come to the clinic for
anisms such as a routine physical examination. She says she has
gene activity no complaints and is concerned only about a
20-pound weight gain in the past 2 years. She
says that she is not as active as she used to be.
She also mentions that she has fallen several
Activity C Briefly answer the following. times and now has a large bruise on her right
1. What is a hormone? hip.
1. The nurse knows that this client is at risk for
osteoporosis because of her decrease in activ-
ity. What test would the nurse expect to be
ordered to either confirm or rule out
osteoporosis in this patient?
2. What is the structure of a hormone?

Copyright 2011. Wolters Kluwer Health | Lippincott Williams & Wilkins. Study Guide for Porths Essentials of Pathophysiology, Third Edition.
LWBK707-c31_p176-179.qxd 9/2/10 7:52 AM Page 178 Aptara Inc

178 UNIT 9 ENDOCRINE SYSTEM

2. With the clients weight gain over the past 5. The hypophysis is a unit formed by the pitu-
2 years and her decrease in activity level, the itary and the hypothalamus. These two
nurse would expect what test to be ordered to glands are connected by the blood flow in
either rule out or confirm type II diabetes in what system?
this client? a. Hypophyseal portal system
b. Supraoptic portal system
c. Paraventricular portal system
d. Hypothalamic portal system
6. The hormone levels in the body need to be
kept within an appropriate range. How is this
SECTION IV: PRACTICING accomplished for many of the hormones in
FOR NCLEX the body?
a. Positive feedback loop
Activity E Answer the following questions. b. Negative feedback loop
1. The endocrine system is closely linked with c. Regulated feedback loop
both the immune system and the nervous d. Sensory feedback loop
system. What neurotransmitter can also act
as a hormone? 7. Many hormones are measured for diagnostic
reasons by using the plasma levels of the
a. Epinephrine
hormones. What is used today to measure
b. Norepinephrine plasma hormone levels?
c. Dopamine a. Nucleotide assay methods
d. Succinylcholine b. Selective binding methods
2. When hormones act locally rather than being c. Radioimmunoassay methods
secreted into the bloodstream, their actions d. Radiolabeled hormone-antibody
are termed what? methods
a. Autocratic and paracratic
8. Sometimes the measurement of hormones is
b. Autocrine and paracrine done through a urine sample. What is an
c. Localized and influential advantage of measuring hormone levels
d. Preventers and inhibitors through a urine sample rather than a blood
sample?
3. Hormones can be synthesized by both
a. Urine has more accurate measurements of
vesicle-mediated pathways and nonvesicle-
hormones
mediated pathways. What hormones are syn-
thesized by nonvesicle-mediated pathways? b. There are more hormone metabolites in
urine than in blood
a. Neurotransmitters that are also hormones
c. Blood sampling has more pure hormone
b. Renin and angiotensin
than urine does
c. Androgens and estrogens
d. Urine samples are easily obtained
d. Pepcin and ghrelin
4. To prevent the accumulation of hormones in
our bodies, the hormones are constantly being
metabolized and excreted. Where are adrenal
and gonadal steroid hormones excreted?
a. Feces and urine
b. Bile and lungs
c. Cell metabolites and lungs
d. Bile and urine

Copyright 2011. Wolters Kluwer Health | Lippincott Williams & Wilkins. Study Guide for Porths Essentials of Pathophysiology, Third Edition.
LWBK707-c31_p176-179.qxd 9/2/10 7:52 AM Page 179 Aptara Inc

CHAPTER 31 MECHANISMS OF ENDOCRINE CONTROL 179

9. In an adult with acromegaly, a growth 10. Imaging has proven useful in both the
hormone (GH)-secreting tumor is suspected. diagnosis and follow-up of endocrine
What diagnostic test would be used for this disorders. Two types of imaging studies are
client? useful when dealing with endocrine
a. A GH suppression test disorders, isotopic imaging and nonisotopic
imaging. What is an example of isotopic
b. A GH stimulation test
imaging?
c. A GH serum assay test
a. MRI
d. A GH urine assay test
b. Thyroid scan
c. Renal angiography
d. PET scan

Copyright 2011. Wolters Kluwer Health | Lippincott Williams & Wilkins. Study Guide for Porths Essentials of Pathophysiology, Third Edition.
LWBK707-c32_p180-186.qxd 8/19/10 1:26 PM Page 180 Aptara Inc

32
CHAPTER
Disorders of Endocrine
Control of Growth and
Metabolism

SECTION I: LEARNING 11. Characterize the synthesis, transport, and


regulation of thyroid hormone.
OBJECTIVES
12. Diagram the hypothalamic-pituitary-thyroid
1. Describe the mechanisms of endocrine hypo- feedback system.
function and hyperfunction.
13. Describe tests in the diagnosis and
2. Differentiate primary, secondary, and tertiary management of thyroid disorders.
endocrine disorders.
14. Relate the functions of thyroid hormone to
3. Discuss the classification of pituitary tumors. hypothyroidism and hyperthyroidism.
4. Describe the clinical features and causes of 15. Describe the effects of congenital
hypopituitarism. hypothyroidism.
5. State the effects of a deficiency in growth 16. Characterize the manifestations and
hormone (GH). treatment of myxedematous coma and
thyroid storm.
6. Differentiate genetic short stature from con-
stitutional short stature. 17. Describe the function of the adrenal cortical
hormones and their feedback regulation.
7. State the mechanisms of short stature in
hypothyroidism, poorly controlled diabetes 18. State the underlying cause of congenital
mellitus, chronic treatment with excessive adrenal hyperplasia.
glucocorticoid hormones, malnutrition, and
19. Relate the functions of the adrenal cortical
psychosocial dwarfism.
hormones to Addison disease (i.e., adrenal
8. List three causes of tall stature. insufficiency) and Cushing syndrome (i.e.,
glucocorticoid excess).
9. Relate the functions of GH to the manifesta-
tions of acromegaly and adult-onset GH defi-
ciency.
10. Explain why children with precocious
puberty are tall-statured children but short-
statured adults.

180
LWBK707-c32_p180-186.qxd 8/19/10 1:26 PM Page 181 Aptara Inc

CHAPTER 32 DISORDERS OF ENDOCRINE CONTROL OF GROWTH AND METABOLISM 181

SECTION II: ASSESSING 13. Thyroid hormone has two major functions: it
increases and
YOUR UNDERSTANDING synthesis, and it is necessary for growth and
development in children.
Activity A Fill in the blanks.
14. Thyroid hormone increases the
1. Disturbances of endocrine function usually
of all body tissues except the retina, spleen,
can be divided into two categories:
testes, and lungs.
and .
15. Measures of T3, T4, and TSH have been made
2. defects result in endocrine
available through methods.
hypofunction due to the absence or impaired
development of the gland or the absence of 16. Congenital hypothyroidism is a common
an enzyme needed for hormone synthesis. cause of .
3. Several hormones are essential for normal 17. The term implies the presence
body and maturation, including of a nonpitting mucus-type edema caused by
growth hormone (GH), insulin, thyroid the accumulation of hydrophobic extracellu-
hormone, and androgens. lar matrix substances in the connective
tissues of a number of body tissues.
4. Growth hormone cannot directly produce
bone growth; instead, it acts indirectly by 18. is the clinical syndrome that
causing the liver to produce . results when tissues are exposed to high lev-
els of circulating thyroid hormone.
5. secretion is stimulated by hypo-
glycemia, fasting, starvation, increased blood 19. The most common cause of hyperthyroidism
levels of amino acids, and stress conditions is disease, which is accompanied
such as trauma, excitement, emotional stress, by ophthalmopathy (or dermopathy) and
and heavy exercise. diffuse goiter.
6. is a term used to describe chil- 20. Many of the manifestations of hyperthyroidism
dren (particularly boys) who have moderately are related to the increase in
short stature, thin build, delayed skeletal and consumption and use of fuels
sexual maturation, and absence of other associated with the hypermetabolic state, as
causes of decreased growth. well as to the increase in sympathetic
nervous system activity that occurs.
7. The term is used to describe a
child who is taller than his or her peers and is 21. is manifested by a very high
growing at a velocity that is within the fever, extreme cardiovascular effects, and
normal range for bone age. severe CNS effects.
8. Growth hormone excess occurring before 22. The forms the bulk of the gland
puberty and the fusion of the epiphyses of and is responsible for secreting three types of
the long bones results in . hormones: the glucocorticoids, the mineralo-
corticoids, and the adrenal androgens.
9. When GH excess occurs in adulthood or after
the epiphyses of the long bones have fused, 23. secretion is regulated by the
the condition is referred to as . renin-angiotensin mechanism and by blood
levels of potassium.
10. Long-term elevation of GH results in
of the beta cells, causing them 24. When produced as part of the stress response,
literally to burn out. hormones aid in regulating the
metabolic functions of the body and in con-
11. sexual development may be
trolling the inflammatory response.
idiopathic or may be caused by gonadal,
adrenal, or hypothalamic disease. 25. stimulates glucose production
by the liver, promotes protein breakdown,
12. hormones are bound to
and causes mobilization of fatty acids.
thyroxine-binding globulin and other plasma
proteins for transport in the blood.

Copyright 2011. Wolters Kluwer Health | Lippincott Williams & Wilkins. Study Guide for Porths Essentials of Pathophysiology, Third Edition.
LWBK707-c32_p180-186.qxd 8/19/10 1:26 PM Page 182 Aptara Inc

182 UNIT 9 ENDOCRINE SYSTEM

26. Primary adrenal insufficiency, or 27. The term refers to the manifes-
disease, is caused by destruction of the tations of hypercortisolism from any cause.
adrenal gland.

Activity B Consider the following figure.

Hypothalamus

Anterior
pituitary

Growth hormone

Liver

IGF-1

Adipose Carbohydrate
tissue metabolism

Bone and Body


Muscle
cartilage organs

Complete the flowchart above with the following Increased lean muscle mass
terms Increased linear growth
Anti-insulin effects Increased lipolysis
Decreased glucose use Increased protein synthesis
Decrease in adiposity Increased size and function
Growth-promoting actions
Increased blood glucose

Copyright 2011. Wolters Kluwer Health | Lippincott Williams & Wilkins. Study Guide for Porths Essentials of Pathophysiology, Third Edition.
LWBK707-c32_p180-186.qxd 8/19/10 1:26 PM Page 183 Aptara Inc

CHAPTER 32 DISORDERS OF ENDOCRINE CONTROL OF GROWTH AND METABOLISM 183

Activity C Match the key terms in Column A Activity D Briefly answer the following.
with their definitions in Column B.
1. Explain the grouping of the root causes of
Column A Column B endocrine disorders.
1. Laron-type a. Growth hormone-
dwarfism secreting cells
2. Hypopituitarism b. Deficiency of all
pituitary-derived
3. Cretinism hormones 2. What hormones are directly affected by
4. Hashimoto c. Dry skin and hypopituitarism? What affect does it have on
thyroiditis swellings around the rest of the endocrine system?
lips and nose as
5. Panhypo-
well as mental
pituitarism
deterioration
6. Ophthalmopathy d. Manifestations
7. Goiter of untreated
congenital 3. What are the normal actions of GH?
8. Myxedema hypothyroidism
9. Somatotropes e. An autoimmune
disorder in which
10. Pendred
the thyroid gland
syndrome
may be totally
4. How is GH release stimulated? How is it inhib-
destroyed
ited?
f. Increase in the size
of the thyroid
gland
g. Eyelid retraction,
bulging eyes, light
sensitivity, 5. Describe the stimulation of the thyroid gland
discomfort, double and explain the mechanism of negative feed-
vision, and vision back to inhibit thyroid activity.
loss
h. Patients with goi-
ter and congenital
deafness
i. Growth hormone
6. Describe the manifestations of
levels are normal
hypothyroidism.
or elevated, but
there is a
hereditary defect
in insulinlike
growth factor
production
7. What is the result of adrenal insufficiency?
j. Decreased
secretion of
pituitary
hormones

Copyright 2011. Wolters Kluwer Health | Lippincott Williams & Wilkins. Study Guide for Porths Essentials of Pathophysiology, Third Edition.
LWBK707-c32_p180-186.qxd 8/19/10 1:26 PM Page 184 Aptara Inc

184 UNIT 9 ENDOCRINE SYSTEM

8. What are the manifestations of Cushing syn- SECTION IV: PRACTICING


drome?
FOR NCLEX
Activity F Answer the following questions.
1. Advances in technology have made it
possible to assess hypothalamic-pituitary
function by newly developed imaging and
radioimmunoassay methods. When baseline
tests are not sufficient, what suppression test
SECTION III: APPLYING gives information about combined hypothal-
YOUR KNOWLEDGE amic-pituitary function?
a. GH suppression test
Activity E Consider the scenario and answer b. ACTH suppression test
the questions.
c. Cortisol suppression test
The parents of a newborn have been notified by d. Prolactin suppression test
the hospital that they need to bring their
newborn back to the hospital for further testing. 2. Growth hormone is secreted by adults as well
The parents are informed that one of the tests as by children. Growth hormone deficiency
done on the baby when it was first born needs in children is treated by injections of GH on
to be repeated. When the parents arrive at the a daily basis. When teaching a family or child
hospital, they meet with a pediatrician who to give injections of GH, what is it important
explains that their infants thyroid tests have to teach them?
come back abnormal and need to be repeated. a. Give the injections in the morning so the
He goes on to say that it might be a false- peak effect is before noon, like the body
negative result on the original test and not to does.
worry. b. Give the injections at bedtime to produce
1. As the nurse prepares to take the infants blood, the greatest effect at night, like the body
the parents ask what it means if the first test does.
result is not a mistake. The nurse knows the c. Give the injections about 3 in the after-
best information to give the parents is what? noon to produce the greatest effect in the
evening, like the body does.
d. Give the injections in the early afternoon
to produce the greatest effect at dinner
time, like the body does.

2. The parents want to know what will happen to 3. Growth hormone exerts its effects on the
their baby if the thyroid gland is not working body in many ways. Which of these are
correctly. The nurse correctly answers what? effects of GH? (Mark all that apply.)
a. Enhances fatty acid mobilization
b. Increases insulin levels
c. Facilitates the rate of protein synthesis
d. Decreases ACTH production
e. Decreases use of fatty acids for fuel

Copyright 2011. Wolters Kluwer Health | Lippincott Williams & Wilkins. Study Guide for Porths Essentials of Pathophysiology, Third Edition.
LWBK707-c32_p180-186.qxd 8/19/10 1:26 PM Page 185 Aptara Inc

CHAPTER 32 DISORDERS OF ENDOCRINE CONTROL OF GROWTH AND METABOLISM 185

4. Acromegaly is a disorder that is caused by 8. Hyperthyroidism that is inadequately treated


the production of excessive GH in the adult. can cause a life-threatening condition known
Because the person cannot grow taller, the as a thyroid storm. What are the manifestations
soft tissues continue to grow, presenting of a thyroid storm? (Mark all that apply.)
a very distinctive appearance. What is it a. Tachycardia
that is distinctive in a person with
b. Very low fever
acromegaly?
c. Delirium
a. Small hands and feet compared to length
of arms and legs d. Bradycardia
b. Broad, bulbous nose and a protruding e. Very high fever
lower jaw 9. At times, it is necessary to give medications
c. Slanting forehead and a receding lower that suppress the adrenal glands on a long-
jaw term basis. When the suppression of the
d. Protruding lower jaw and forehead adrenals becomes chronic, the adrenal glands
atrophy. What does the abrupt withdrawal of
5. Precocious puberty is a disorder that occurs in these suppressive drugs cause?
both boys and girls. What does precocious
a. Acute adrenal hyperplasia
puberty cause in adults?
b. Acute adrenal insufficiency
a. Early menopause in females
c. Acute adrenal hypoplasia
b. Early erectile dysfunction problems in
males d. Acute adrenal cortical hyperplasia
c. Short stature in adults 10. Congenital adrenal hyperplasia is a congeni-
d. Gigantism in adults tal disorder in which a deficiency exists in
any of the enzymes necessary for the synthe-
6. When the assessment of thyroid autoantibod- sis of cortisol. Infants of both sexes are
ies is performed, what is the suspected affected, although boys are not diagnosed at
diagnosis? birth unless of enlarged genitalia. Female
a. Goiter infants often have ambiguous genitalia
b. Thyroid tumor because of the oversecretion of adrenal
androgens. What are the manifestations of
c. Congenital hypothyroidism
the ambiguous genitalia caused by congenital
d. Hashimoto thyroiditis adrenal hyperplasia?
7. An elderly woman is brought to the a. Small clitoris, fused labia, and urogenital
emergency department by her family. They sinus
relate to the nurse that the client has had b. Small clitoris, open labia, and urogenital
mental status changes and cannot remember sinus
her grandchildrens names. They go on to say
c. Enlarged clitoris, fused labia, and urogeni-
that she is intolerant of cold and is lethargic.
tal sinus
On physical examination the nurse notes
that the client has a husky voice, her face is d. Enlarged clitoris, open labia, and urogeni-
puffy around the eyes, and her tongue tal sinus
appears to be enlarged. What diagnosis 11. In Addison disease the majority of the adrenal
would the nurse suspect? cortex has been destroyed. This causes a lack
a. Myxedema of mineralocorticoids and glucocorticoids.
b. Hashimoto thyroiditis Therapy consists of oral replacement with
what drug?
c. Hyperthyroidism
a. Cortisol
d. Congenital hypothyroidism
b. Aldosterone
c. Glucocorticoid
d. Hydrocortisone

Copyright 2011. Wolters Kluwer Health | Lippincott Williams & Wilkins. Study Guide for Porths Essentials of Pathophysiology, Third Edition.
LWBK707-c32_p180-186.qxd 8/19/10 1:26 PM Page 186 Aptara Inc

186 UNIT 9 ENDOCRINE SYSTEM

12. In an acute adrenal crisis, the onset of symp- 13. The hallmark manifestations of Cushing syn-
toms is sudden, and in the case of Addison drome are a moon face, a buffalo hump
disease, can be precipitated by exposure to a between the shoulder blades, and a protrud-
minor illness or stress. What are the manifes- ing abdomen. What other manifestations of
tations of acute adrenal crisis? (Mark all that Cushing syndrome occur?
apply.) a. Thin extremities and muscle weakness
a. Hypertension b. Muscle wasting and thickened extremities
b. Muscle weakness c. Muscle weakness and thickened extremi-
c. Dehydration ties
d. Altered mental status d. Thin extremities and increased strength
e. Vascular collapse

Copyright 2011. Wolters Kluwer Health | Lippincott Williams & Wilkins. Study Guide for Porths Essentials of Pathophysiology, Third Edition.
LWBK707-c33_p187-193.qxd 8/19/10 1:27 PM Page 187 Aptara Inc

33

CHAPTER
Diabetes Mellitus and
the Metabolic Syndrome

SECTION I: LEARNING 10. Name and describe the types (according to


duration of action) of insulin.
OBJECTIVES
11. Differentiate between the causes and clinical
1. State the functions of glucose, fats, and pro- manifestations of diabetic ketoacidosis and
teins in meeting the energy needs of the the hyperosmolar hyperglycemic state.
body.
12. Describe alterations in physiologic function
2. Characterize the actions of insulin with refer- that accompany diabetic peripheral neuropa-
ence to glucose, fat, and protein metabolism. thy, retinopathy, and nephropathy.
3. Explain what is meant by counter-regulatory hor- 13. Describe the causes of foot ulcers in people
mones, and describe the actions of glucagon, with diabetes mellitus.
epinephrine, growth hormone, and the gluco-
14. Explain the relation between diabetes melli-
corticoid hormones in regulation of blood
tus and infection.
glucose levels.
4. Compare the distinguishing features of type
1 and type 2 diabetes mellitus, list causes of
other specific types of diabetes, and cite the
SECTION II: ASSESSING
criteria for gestational diabetes. YOUR UNDERSTANDING
5. Describe what is meant by the term Activity A Fill in the blanks.
prediabetes.
1. The primary source of energy for the body is
6. Relate the physiologic functions of insulin to .
the manifestations of diabetes mellitus.
2. Because the can neither synthe-
7. Define the metabolic syndrome and describe size nor store more than a few minutes sup-
its associations with the development of type ply of glucose, normal cerebral function
2 diabetes. requires a continuous supply from the circu-
8. Discuss the role of diet and exercise in the lation.
management of diabetes mellitus. 3. Severe and prolonged can cause
9. Characterize the blood glucose-lowering brain death.
actions of the hypoglycemic agents used in 4. Glucose that is not needed for energy is
treatment of type 2 diabetes. removed from the blood and stored as
or converted to fat.

187
LWBK707-c33_p187-193.qxd 8/19/10 1:27 PM Page 188 Aptara Inc

188 UNIT 9 ENDOCRINE SYSTEM

5. When blood glucose levels fall below normal, 18. The term type 1B diabetes is
as they do between meals, a process called used to describe those cases of beta cell
breaks down glycogen, and destruction in which no evidence of autoim-
glucose is released. munity is present.
6. In addition to mobilizing its glycogen stores, 19. diabetes mellitus is a heteroge-
the liver synthesizes glucose from amino neous condition that describes the presence
acids, glycerol, and lactic acid in a process of hyperglycemia in association with relative
called . insulin deficiency.
7. Fat is the most efficient form of fuel storage, 20. Insulin initially stimulates an
providing kcal/g of stored increase in insulin secretion, often to a level
energy, compared with the of modest hyperinsulinemia, as the beta cells
kcal/g provided by carbohydrates and attempt to maintain a normal blood glucose
proteins. level.
8. are essential for the formation 21. While the insulin resistance seen in persons
of all body structures, including genes, with type 2 diabetes can be caused by a num-
enzymes, contractile structures in muscle, ber of factors, it is strongly associated with
matrix of bone, and hemoglobin of red blood and .
cells.
22. A major factor in persons with the metabolic
9. Because cannot be converted to syndrome that leads to type 2 diabetes is
glucose, the body must break down .
and use the amino acids as a
23. diabetes mellitus refers to any
major substrate for gluconeogenesis during
degree of glucose intolerance that is first
periods when metabolic needs exceed food
detected during pregnancy.
intake.
24. The plasma glucose has been
10. Because cell membranes are impermeable to
suggested as the preferred diagnostic test
glucose, they require a special carrier, called a
because of ease of administration, convenience,
, to move glucose from the
patient acceptability, and cost.
blood into the cell.
25. A plasma glucose
11. is the insulin-dependent
concentration that is unequivocally elevated
glucose transporter for skeletal muscle and
( 200 mg/dL) in the presence of
adipose tissue.
classic symptoms of diabetes such as polydip-
12. maintains blood glucose sia, polyphagia, polyuria, and blurred vision
between meals and during periods of fasting. is diagnostic of diabetes mellitus at any age.
13. The most dramatic effect of glucagon is its 26. In uncontrolled diabetes or diabetes with
ability to initiate and hyperglycemia, there is an increase in the
. level in circulation.
14. The secretion of growth hormone normally 27. Type 1 diabetes mellitus always requires treat-
is inhibited by and increased ment with , and many people
levels of blood glucose. with type 2 diabetes eventually require simi-
lar therapy.
15. is a disorder of carbohydrate,
protein, and fat metabolism resulting from 28. Diabetic most commonly
an imbalance between insulin availability occurs in a person with type 1 diabetes, in
and insulin need. whom the lack of insulin leads to
mobilization of fatty acids from adipose
16. A fasting plasma glucose of or a
tissue because of the unsuppressed adipose
2-hour oral glucose tolerance test result
cell lipase activity that breaks down
is considered normal.
triglycerides into fatty acids and glycerol.
17. diabetes mellitus is
characterized by destruction of the pancreatic
beta cells.

Copyright 2011. Wolters Kluwer Health | Lippincott Williams & Wilkins. Study Guide for Porths Essentials of Pathophysiology, Third Edition.
LWBK707-c33_p187-193.qxd 8/19/10 1:27 PM Page 189 Aptara Inc

CHAPTER 33 DIABETES MELLITUS AND THE METABOLIC SYNDROME 189

29. The is characterized by hyper- g. Stimulate


glycemia (blood glucose 600 mg/dL), gluconeogenesis
hyperosmolarity (plasma osmolarity 310 by the liver
mOsm/L) and dehydration, the absence of h. Increases tissue
ketoacidosis, and depression of the sensitivity to
sensorium. insulin
30. are thought to produce i. Nuclear receptor
structural defects in the basement membrane that leads to the
of the microcirculation and to contribute to regulation of genes
eye, kidney, and vascular complications. controlling free
fatty acid levels
31. The term is used to describe the
and glucose
combination of lesions that often occur con-
metabolism
currently in the diabetic kidney.
j. Cycle of insulin-
32. is characterized by abnormal induced
retinal vascular permeability, microaneurysm posthypoglycemic
formation, neovascularization and associated episodes
hemorrhage, scarring, and retinal
k. Increased levels of
detachment.
fasting blood
33. Multiple risk factors for , includ- glucose without
ing obesity, hypertension, hyperglycemia, precursor
hyperinsulinemia, hyperlipidemia, altered hypoglycemia
platelet function, endothelial dysfunction,
systemic inflammation, and elevated fibrino-
gen levels, frequently are found in people Activity C
with diabetes.
1. Construct a flowchart, using the terms below,
that reflects hormonal and hepatic regulation
Activity B Match the key terms in Column A
of blood glucose.
with their definitions in Column B.
Decreased blood glucose
Column A Column B
Removal of glucose from blood
1. Incretin effect a. Three fatty acids
Decreased glucagon
linked by a
2. Somatostatin Increased insulin release from beta cells
glycerol molecule
3. Epinephrine b. Produce inhibition Deceased hepatic glucose production
4. Secretagogues of gastric Increased blood glucose
emptying and Decreased insulin and increased glucagon
5. Adiponectin glucagon secretion and gluconeogenesis
6. Triglyceride c. Inhibit the release
of insulin and
7. Somogyi effect Activity D Briefly answer the following.
glucagon
8. PPAR- d. Increase insulin 1. What are the results/actions of insulin
release after an release?
9. Amylin
oral nutrient load
10. Glucocorticoid
e. Agents that cause
11. Dawn or stimulate secre-
phenomenon tion
f. Inhibits insulin
release and
promotes
glycogenolysis

Copyright 2011. Wolters Kluwer Health | Lippincott Williams & Wilkins. Study Guide for Porths Essentials of Pathophysiology, Third Edition.
LWBK707-c33_p187-193.qxd 8/19/10 1:27 PM Page 190 Aptara Inc

190 UNIT 9 ENDOCRINE SYSTEM

2. How is insulin secretion from beta cells stim- 9. What are the three polys and why are they
ulated? significant?

3. Why are patients with type 1 diabetes melli- 10. Why do patients with type 1 diabetes lose
tus especially prone to develop ketoacidosis? weight?

4. What is thought to cause type 1 diabetes 11. How does continuous subcutaneous insulin
mellitus? infusion work?

5. What are the metabolic changes that precede 12. What are the three major challenges to nor-
the development of type 2 diabetes? mal physiology from diabetic ketoacidosis
(DKA)?

6. How does beta cell dysfunction develop in


type 2 diabetics? 13. What are the common complications of
chronic diabetes mellitus? How do they
develop?

7. What are the systemic manifestations of


metabolic syndrome?
14. What are the pathologic changes observed in
peripheral neuropathies that are associated
with chronic diabetes mellitus?

8. What are the effects of insulin resistance and


increased glucose production in obese
patients with type 2 diabetes?
15. What are the effects of diabetes mellitus on
renal tissue?

Copyright 2011. Wolters Kluwer Health | Lippincott Williams & Wilkins. Study Guide for Porths Essentials of Pathophysiology, Third Edition.
LWBK707-c33_p187-193.qxd 8/19/10 1:27 PM Page 191 Aptara Inc

CHAPTER 33 DIABETES MELLITUS AND THE METABOLIC SYNDROME 191

SECTION III: APPLYING 2. Hormones that counteract insulins storage


function when regulating blood glucose dur-
YOUR KNOWLEDGE ing times when glucose intake is limited or
glucose stores are depleted are called counter-
Activity E Consider the scenario and answer
regulatory hormones. What are the counter
the questions.
regulatory hormones? (Mark all that apply.)
A 16-year-old boy is admitted to your unit with a a. Glucocorticoids
new diagnosis of type 1A diabetes mellitus. His
b. Growth hormone
blood sugar on admission is 735; he is lethargic;
his parents state that he has started eating c. Catecholamines
continuously; and he is urinating much more d. Mineralocorticoids
than he usually does. They say he has lost 10 e. Glucagon
pounds over the past few months without trying.
The client and his family state that they know 3. During periods of fasting and starvation, the
nothing about diabetes and ask the nurse for an glucocorticoid and other corticosteroid
explanation of what the disease is. hormones are critical for survival because of
their stimulation of gluconeogenesis by the
1. The nurse would know to include what infor- liver. When the glucocorticoid hormones
mation in educating the client and his family remain elevated for extended periods of time
about type 1A diabetes mellitus? what can occur?
a. Hepatomegaly
b. Portal hypertension
c. Hyperglycemia
d. Adrenal hyperplasia
2. The client asks if there is any cure for type
1A diabetes. The nurse would know to 4. Type 1A diabetes is now considered an
respond: autoimmune disorder. What factors are con-
sidered necessary for type 1A diabetes to
occur?
a. Genetic predisposition, environmental
triggering event, and a T-lymphocyte
mediated hypersensitivity reaction against
some beta cell antigen
b. Genetic predisposition, physiologic trigger-
ing event, allergic reaction to pancreatic
SECTION IV: PRACTICING alpha cells
FOR NCLEX c. Diabetogenic gene from both parents,
physiologic triggering event, and an aller-
Activity F Answer the following questions. gic reaction to pancreatic delta cells
1. The pancreas is an endocrine organ that is d. Diabetogenic gene from both parents,
composed of the acini and the islets of environmental triggering event, and a B-
Langerhans. The islets of Langerhans have lymphocyte reaction to alpha cell antigens
alpha, beta, and delta cells as well as the PP
5. Type 2 diabetes is caused by metabolic abnor-
cell. Which cells secrete insulin?
malities in the presence of insulin. What are
a. Alpha cells these metabolic abnormalities? (Mark all that
b. Beta cells apply.)
c. Delta cells a. Deranged secretion of insulin
d. PP cells b. Decreased glucose production by the liver
c. Insulin resistance
d. Increased glucose production by the liver
e. Hypersensitivity to insulin

Copyright 2011. Wolters Kluwer Health | Lippincott Williams & Wilkins. Study Guide for Porths Essentials of Pathophysiology, Third Edition.
LWBK707-c33_p187-193.qxd 8/19/10 1:27 PM Page 192 Aptara Inc

192 UNIT 9 ENDOCRINE SYSTEM

6. Secondary diabetes occurs because of 10. Diabetic ketoacidosis is a condition that


disorders that produce hyperglycemia by mostly occurs in type 1 diabetics. What are
stimulating the hepatic production of glucose the definitive diagnostic criteria for DKA?
or decrease the cellular use of glucose. Which a. Blood glucose level 350 mg/dL; bicarbon-
disorders can be causes of secondary ate 05 mEq/L and pH 7.4
diabetes?
b. Blood glucose level 250 mg/dL; bicarbon-
a. Pheochromocytoma and Cushing syn- ate 25 mEq/L and pH 7.3
drome
c. Blood glucose level 350 mg/dL; bicarbon-
b. Pancreatic disease and dwarfism ate 05 mEq/L and pH 7.4
c. Acromegaly and pancreatic hyperplasia d. Blood glucose level 250 mg/dL; bicarbon-
d. Hepatomegaly and pheochromocytoma ate 15 mEq/L and pH 7.3
7. Gestational diabetes mellitus is a disorder of 11. A man is brought into the emergency
glucose intolerance that occurs during department by paramedics who state that
pregnancy. It is associated with increased risk the client passed out on the street. The man
for developing type 2 diabetes and with fetal smells of alcohol, and when roused says he
abnormalities. What fetal abnormalities are has not eaten since yesterday. He is wearing
associated with gestational diabetes mellitus? a medic alert bracelet that says he is a
a. Microsomia and polycythemia diabetic. What would the nurse suspect as
a diagnosis?
b. Macrosomia and hypocalcemia
a. Hypoglycemia
c. Hypercalcemia and hyperbilirubinemia
b. Hyperglycemia
d. Hypoglycemia and hypercalcemia
c. Hyponatremia
8. What are the hallmark signs of diabetes
d. Hypernatremia
mellitus?
a. Polyuria, polydipsia, and pheochromocy- 12. Hypoglycemia has a sudden onset with a pro-
toma gression of symptoms. What are the signs
and symptoms of hypoglycemia?
b. Polyuria, polyphagia, and polycythemia
a. Difficulty problem solving and muscle
c. Polyuria, polydipsia, and polyphagia
spasms
d. Polycythemia, polydipsia, and pheochro-
b. Altered cerebral function and headache
mocytoma
c. Muscle spasms and headache
9. Match the type of oral antidiabetic agents
d. Altered cerebral function and muscle
with the name of a drug in its class.
spasms
Type of Antidiabetic
13. Research has identified a cycle of insulin-
Agent Drug
induced posthypoglycemic episodes. What is
1. Insulin a. Exenatide this phenomenon called?
secretagogues b. Rosiglitazone a. Dawn phenomenon
2. Biguanides c. Metformin b. Joslin phenomenon
3. -Glucosidase d. Repaglinide c. Somogyi effect
inhibitors e. Acarbose d. Sunset effect
4. Thiazolidine- f. Alogliptin
diones
5. Dipeptidyl
peptidase 4
(DPP-4) enzyme
inhibitors
6. Glucagonlike
polypeptide 1
agonists

Copyright 2011. Wolters Kluwer Health | Lippincott Williams & Wilkins. Study Guide for Porths Essentials of Pathophysiology, Third Edition.
LWBK707-c33_p187-193.qxd 8/19/10 1:27 PM Page 193 Aptara Inc

CHAPTER 33 DIABETES MELLITUS AND THE METABOLIC SYNDROME 193

14. Peripheral neuropathies occur in people with 17. Diabetics are hospitalized for a number of
diabetes mellitus. With the loss of sensation reasons. What is the most common
in the lower extremities diabetics become complication of diabetes requiring
predisposed to what? hospitalization?
a. Denervation of the large muscles of the a. Diabetic ketoacidosis
foot and bunions b. Foot problems
b. Displacement of the submetatarsal fat pad c. Hypertensive crisis
posteriorly and hammer toes
d. Macrovascular disease
c. Impairment of temperature and touch sen-
sations 18. Infections are common in people with diabetes.
Which infection is thought to be related to a
d. Clawing of toes and denervation of the
neurogenic bladder?
small muscles of the foot
a. Nephrotic syndrome
15. Diabetics are at higher risk than the majority
b. Urinary retention
of the population for injury to organ systems
in the body. Which organs are most at risk? c. Pyelonephritis
a. Kidneys and eyes d. Urinary incontinence
b. Kidneys and liver
c. Liver and eyes
d. Pancreas and eyes
16. Macrovascular disease includes coronary
artery disease, cerebrovascular disease, and
peripheral vascular disease. People with both
type 1 and type 2 diabetes are at high risk for
developing macrovascular disease. What are
the risk factors for macrovascular disease in
diabetics? (Mark all that apply.)
a. Elevated fibrinogen levels and hyperinsu-
linemia
b. Hyperlipidemia and hypotension
c. Hyperglycemia and hypoinsulinemia
d. Decreased fibrinogen levels and systemic
inflammation

Copyright 2011. Wolters Kluwer Health | Lippincott Williams & Wilkins. Study Guide for Porths Essentials of Pathophysiology, Third Edition.
LWBK707-c34_p194-200.qxd 8/19/10 1:27 PM Page 194 Aptara Inc

34
CHAPTER
Organization and
Control of Neural
Function

SECTION I: LEARNING 10. Use the segmental approach to explain the


development of the nervous system and the
OBJECTIVES organization of the postembryonic nervous
system.
1. Distinguish between the functions of the
neurons and supporting cells of the nervous 11. Define the terms afferent, efferent, ganglia,
system. association neuron, cell column, and tract.
2. List the three parts of a neuron and describe 12. State the origin and destination of nerve
their structure and function. fibers contained in the dorsal and ventral
roots.
3. Name the supporting cells in the central
nervous system (CNS) and peripheral 13. State the structures innervated by general
nervous system and state their functions. somatic afferent, special visceral afferent,
general visceral afferent, special somatic
4. Describe the metabolic requirements of nerv-
afferent, general visceral efferent, pharyngeal
ous tissue.
efferent, and general somatic efferent
5. Describe the three phases of an action poten- neurons.
tial and relate the functional importance of
14. Describe the longitudinal and transverse
ion channels to the different phases.
structures of the spinal cord.
6. State the difference between electrical and
15. Trace an afferent and efferent neuron from
chemical synapses.
its site in the periphery through its entrance
7. Describe the interaction of the presynaptic into or exit from the spinal cord.
and postsynaptic terminals.
16. Explain the innervation and function of
8. Characterize the role of excitatory and spinal cord reflexes.
inhibitory postsynaptic potentials as they
17. List the structures of the hindbrain, midbrain,
relate to spatial and temporal summation of
and forebrain and describe their functions.
membrane potentials.
18. Name the cranial nerves and cite their
9. Briefly describe how neurotransmitters are
location and function.
synthesized, stored, released, and inactivated.

194
LWBK707-c34_p194-200.qxd 8/19/10 1:27 PM Page 195 Aptara Inc

CHAPTER 34 ORGANIZATION AND CONTROL OF NEURAL FUNCTION 195

19. Describe the characteristics of the cerebrospinal 7. In some pathologic conditions, such as multi-
fluid and trace its passage through the ventric- ple sclerosis in the CNS and Guillain-Barr
ular system. syndrome in the PNS, the may
degenerate or be destroyed.
20. Contrast and compare the blood-brain and
cerebrospinal fluid-brain barriers. 8. The increase nerve conduction
by allowing the impulse to jump from node
21. Compare the sensory and motor components
to node through the extracellular fluid in a
of the autonomic nervous system with those
process called .
of the CNS.
9. The form the myelin in the
22. Compare the anatomic location and
CNS.
functions of the sympathetic and parasympa-
thetic nervous systems. 10. is the major fuel source for the
nervous system.
23. Describe neurotransmitter synthesis, release,
and degradation, and receptor function in 11. Nerve signals are transmitted by
the sympathetic and parasympathetic , which are abrupt, pulsatile
nervous systems. changes in the membrane potential.
12. The excitability of neurons can be affected by
conditions that alter the mov-
SECTION II: ASSESSING ing it either closer to or further from the
YOUR UNDERSTANDING threshold potential.
13. Neurons communicate with each other
Activity A Fill in the blanks. through structures known as .
1. The are the functional cells of 14. synapses involve special presy-
the nervous system. naptic and postsynaptic membrane
2. The supporting cells, such as in structures, separated by a synaptic cleft.
the peripheral nervous system (PNS) and the 15. The secreted neurotransmitters diffuse into
cells in the CNS, protect the the and unite with receptors on
nervous system and provide metabolic the postsynaptic membrane.
support for the neurons.
16. In excitatory synapses, binding of the neuro-
3. Neurons have three distinct parts: the cell transmitter to the receptor produces
, and its cytoplasm-filled of the postsynaptic membrane,
processes, the and where as the binding of the neurotransmitter
which form the functional connections, or to the receptor in an inhibitory synapse
, with other nerve cells, with induces of the postsynaptic
receptor cells, or with effector cells. membrane by making the membrane more
4. are multiple, short-branched permeable to potassium or chloride.
extensions of the nerve cell body; they 17. When the combination of a neurotransmitter
conduct information toward the cell body with a receptor site causes partial depolariza-
and are the main source of information for tion of the postsynaptic membrane, it is
the neuron. called an potential.
5. Supporting cells of the nervous system, the 18. The process of involves the syn-
and cells of the thesis, storage, and release of a neurotransmit-
PNS and the several types of neuroglial cells ter; the reaction of the neurotransmitter with
of the CNS, give the neurons protection and a receptor; and termination of the receptor
metabolic support. action.
6. cells secrete a basement 19. molecules react with presynap-
membrane that protects the cell body from tic or postsynaptic receptors to alter the
the diffusion of large molecules. release of or response to neurotransmitters.

Copyright 2011. Wolters Kluwer Health | Lippincott Williams & Wilkins. Study Guide for Porths Essentials of Pathophysiology, Third Edition.
LWBK707-c34_p194-200.qxd 8/19/10 1:27 PM Page 196 Aptara Inc

196 UNIT 10 NERVOUS SYSTEM

20. factors are required to maintain 33. The special sensory afferent is
the long-term survival of the postsynaptic attached laterally at the junction of the
cell and are secreted by axon terminals inde- medulla oblongata and the pons, often called
pendent of action potentials. the caudal pons.
21. A functional system called the 34. The innervates the nasopharynx
operates in the lateral portions of the reticu- and taste buds of the palate.
lar formation of the medulla, pons, and espe-
35. The nerve abducts the eye.
cially the midbrain.
36. The is the main sensory nerve
22. The spinal cord and the dorsal and ventral
conveying the modalities of pain,
roots are covered by a connective tissue
temperature, touch, and proprioception to
sheath, the , which also
the superficial and deep regions of the face.
contains the blood vessels that supply the
white and gray matter of the cord. 37. The makes continuous
adjustments, resulting in smoothness of
23. The peripheral nerves that carry information
movement, particularly during the delicate
to and from the spinal cord are called
maneuvers.
.
38. The plays a role in relaying crit-
24. Each spinal cord segment communicates with
ical information regarding motor activities to
its corresponding body segment through the
and from selected areas of the motor cortex.
.
39. A is the ridge between two
25. Spinal nerves do not go directly to skin and
grooves, and the groove is called a
muscle fibers; instead, they form complicated
.
nerve networks called .
40. The supply axial and proximal
26. A is a highly predictable
unlearned and learned postures and move-
relationship between a stimulus and an
ments, which enhance and add gracefulness
elicited motor response.
to upper motor neuron-controlled manipula-
27. The reflex is stimulated by a tive movements.
damaging stimulus and quickly moves the
41. The is necessary for somesthetic
body part away from the offending stimulus,
perception, especially concerning perception
usually by flexing a limb part.
of where the stimulus is in space and in
28. Based on its embryonic development, the relation to body parts.
brain is divided into three regions, the
42. Inside the skull and vertebral column, the
, the , and the
brain and spinal cord are loosely suspended
.
and protected by several connective tissue
29. Damage to the nerve results in sheaths called the .
weakness or paralysis of tongue muscles.
43. The provides a supporting and
30. Sensory and motor components of the protective fluid in which the brain and spinal
nerve innervate the pharynx, cord float.
the gastrointestinal tract, the heart, the
44. The ability to maintain homeostasis and per-
spleen, and the lungs.
form the activities of daily living in an ever-
31. The sternocleidomastoid, a powerful head- changing physical environment is largely
turning muscle, and the trapezius muscle, vested in the .
which elevates the shoulders, are innervated
45. The functions of the are
by the .
concerned with conservation of energy,
32. The dorsolateral contains the resource replenishment and storage, and
same components as the vagus nerve but for maintenance of organ function during
a more rostral segment of the gastrointestinal periods of minimal activitythe rest and
tract and the pharynx. digest response.

Copyright 2011. Wolters Kluwer Health | Lippincott Williams & Wilkins. Study Guide for Porths Essentials of Pathophysiology, Third Edition.
LWBK707-c34_p194-200.qxd 8/19/10 1:27 PM Page 197 Aptara Inc

CHAPTER 34 ORGANIZATION AND CONTROL OF NEURAL FUNCTION 197

Activity B Consider the following figure. Activity C Match the key terms in Column A
with their definitions in Column B.
1.
Column A Column B
1. Microglia a. Forms the lining
of the neural tube
2. Depolarization
cavity
3. Neurotran- b. Phase during
smitters which the polarity
4. Repolarization of the resting
membrane poten-
5. Astrocytes tial is re-established
6. Synaptic vesicles c. Membrane-bound
1. In the figure above of the segments of the sacs that store
7. Ependymal
spinal cord, please label the following neurotransmitters
structures: 8. Plexus d. Form the blood-
9. Threshold brain barrier
IA neuron
potential e. Chemical transmit-
Segments ter molecules
Ventral root 10. Oligodendro-
cytes f. Small phagocytic
Dorsal root ganglion cell that is
Spinal nerve available for clean-
Dorsal root ing up debris after
cellular damage,
infection, or cell
Septum
pellucidum death
g. Membrane poten-
tial at which neu-
Pineal
rons or other
body excitable tissues
Interventricular Cerebral are stimulated
foramen aqueduct
h. Flow of electrically
Anterior
commissure charged ions
toward an equilib-
Central canal
rium
i. Production of CNS
2. In the above figure of the brain, please label myelin
the following structures: j. Site of intermixing
nerve branches
Spinal cord
Medulla oblongata
Pons
Midbrain
Frontal lobe
Corpus callosum
Occipital lobe
Third ventricle
Fourth ventricle
Cerebellum

Copyright 2011. Wolters Kluwer Health | Lippincott Williams & Wilkins. Study Guide for Porths Essentials of Pathophysiology, Third Edition.
LWBK707-c34_p194-200.qxd 8/19/10 1:27 PM Page 198 Aptara Inc

198 UNIT 10 NERVOUS SYSTEM

2. 2. Explain the fragileness of neural cells in regard


to metabolic requirements.
Column A Column B
1. Afferent a. Neurons that com-
municate with CNS
2. Bells palsy
and peripheral
3. Efferent neural cells
b. Nerves that 3. How do neural cell bodies interpret the
4. Proprioception
conduct impulses numerous incoming signals (action potentials)
5. Ganglia from the periphery from other neurons?
6. Association of the body to the
neuron brain or spinal
cord.
7. Limbic system
c. Longitudinal
8. Cell column columns of
4. How are neurotransmitters inactivated in the
neurons
9. Tract synaptic space following release?
d. Communication
over distances
between neighbor-
ing and distal seg-
ment of neural
tube 5. Describe the basic embryologic development
e. Carrying impulses of the nervous system.
from the CNS to
an effector
f. Group of neural
cell bodies
g. Sense of body
6. How are the cell columns organized in the
movement and
dorsal and ventral horns of the spinal cord?
position
h. Involved in
emotional experi-
ence and release
of emotional
behaviors, is 7. What is the importance of cerebral spinal
located in the fluid?
medial aspect of
the cerebrum
i. Unilateral loss of
facial nerve
function
8. How does the blood-brain barrier affect
Activity D Briefly answer the following. drug/toxin actions on the brain?
1. Describe the formation and attachment of
myelin to the axonal membrane.

Copyright 2011. Wolters Kluwer Health | Lippincott Williams & Wilkins. Study Guide for Porths Essentials of Pathophysiology, Third Edition.
LWBK707-c34_p194-200.qxd 8/19/10 1:27 PM Page 199 Aptara Inc

CHAPTER 34 ORGANIZATION AND CONTROL OF NEURAL FUNCTION 199

SECTION III: APPLYING 2. Ion channels in nervous system cells generate


action potentials in the cells. What are the
YOUR KNOWLEDGE ion channels guarded by?
Activity E Consider the scenario and answer a. Schwann cells
the questions. b. Voltage-dependent gates
A woman in her fourth month of pregnancy c. Ligand-gates
comes to the clinic to have an ultrasound done. d. Leyte cells
When the ultrasound is read, the physician tells
3. Neurons communicate through the use of
the woman that her fetus has a neural tube
synapses. These synapses may link neurons
defect and, when the infant is born, it will have a
into functional circuits. What is the most
cystlike pouch on its lower back that contains
common type of synapse?
cerebrospinal fluid, meninges, and spinal
nerves. a. Electrical synapse
b. Excitatory synapse
1. The client asks if there is a name for the defect
her child has. What is the correct response to c. Chemical synapse
the clients question? d. Inhibitory synapse
4. Neurotransmitters are small molecules that
exert their actions through specific proteins,
called receptors, embedded in the postsynaptic
membrane. Where are neurotransmitters syn-
thesized?
2. The client asks what this defect will mean for
her baby. What would be the correct response a. In the dendrite terminal
from the health care professional? b. In the presynaptic junction
c. In the postsynaptic junction
d. In the axon terminal
5. Neuromodulators can produce slower and
longer-lasting changes in membrane excitabil-
ity by acting on postsynaptic receptors. What
do neuromodulators do?
a. Alter the release or response to neurotrans-
SECTION IV: PRACTICING mitters
FOR NCLEX b. Alter the inhibitory response of postsynap-
tic electrical receptors
Activity F Answer the following questions.
c. Alter the metabolic function of Schwann
1. There are two types of nervous tissue cells. cells
One type is neurons, and the other type is d. Alter the Ligand-gate response to electrical
the supporting cells. What is the function of activity
the supporting cells?
6. The basis for assessing the function of any
a. Protect nervous system and provide meta-
peripheral nerve lies in what?
bolic support for the neurons
a. Peripheral nerves contain only afferent
b. Transmit messages between parts of the
processes from the cell columns
PNS
b. Peripheral nerves contain processes of
c. Transmit messages between the CNS and
more than one of the four afferent and
the PNS
three efferent cell columns.
d. Provide metabolic support for the neurons
c. Peripheral nerves contain only efferent
and the PNS
processes from the cell columns
d. Peripheral nerves contain no processes
from the seven cell columns

Copyright 2011. Wolters Kluwer Health | Lippincott Williams & Wilkins. Study Guide for Porths Essentials of Pathophysiology, Third Edition.
LWBK707-c34_p194-200.qxd 8/19/10 1:27 PM Page 200 Aptara Inc

200 UNIT 10 NERVOUS SYSTEM

7. The spinal cord does not hang freely within 10. The basal ganglia, part of the cerebral
the spinal column. What is it supported by? hemispheres, are damaged by diseases such as
a. The pia mater and the posterior vertebra Parkinson disease and Huntington chorea.
What does this result in?
b. The denticulate ligaments and the verte-
bral blood vessels a. Uncontrollable tremors on movement
c. The pia mater and the denticulate liga- b. Abnormal movement patterns
ments c. Explosive, inappropriate speech
d. The vertebral blood vessels and the poste- d. Inappropriate emotions
rior vertebra
11. The sympathetic and the parasympathetic
8. One of the spinal motor reflexes is the nervous systems are continuously at work in
myotatic reflex. What does this reflex do for our bodies. This continual action gives a
the body? basal activity to all parts of the body. What is
a. Provides information to withdraw the this basal activity referred to as?
body from noxious stimuli a. Tension
b. Provides information about nociceptive b. Relaxation
stimuli c. Tone
c. Provides information about equilibrium d. Strength
d. Provides information about propriocep-
12. Dopamine is an intermediate compound
tion
made during the synthesis of norepinephrine.
9. The cerebellum, separated from the cerebral It is the principal inhibitory transmitter of the
hemispheres by the tentorium cerebelli, lies internuncial neurons in the sympathetic gan-
in the posterior fossa of the cranium. What is glia. What other action does it have?
one of the functions of the cerebellum? a. Vasoconstricts renal and coronary blood
a. Coordinates smooth and accurate move- vessels when given intravenously
ments of the body b. Acts as a neuromodulator in the hindbrain
b. Conveys the senses of pain, temperature, c. Acts as a neuromodulator in the forebrain
touch, and proprioception to the superfi-
d. Vasodilates renal and coronary blood ves-
cial and deep regions of the face
sels when given intravenously
c. Contains the pontine nuclei
d. Contains the main motor pathways be-
tween the forebrain and the pons

Copyright 2011. Wolters Kluwer Health | Lippincott Williams & Wilkins. Study Guide for Porths Essentials of Pathophysiology, Third Edition.
LWBK707-c35_p201-209.qxd 8/19/10 1:28 PM Page 201 Aptara Inc

35

CHAPTER
Somatosensory
Function, Pain,
and Headache

SECTION I: LEARNING 10. Trace the transmission of pain signals with


reference to the neospinothalamic and pale-
OBJECTIVES ospinothalamic, and reticulospinal pathways,
including the role of chemical mediators and
1. Describe the four major classes of somatosen-
factors that modulate pain transmission.
sory modalities and define a sensory unit.
11. Describe the function of endogenous
2. Describe the organization of the somatosen-
analgesic mechanisms as they relate to trans-
sory system in terms of first-, second-, and
mission of pain information.
third-order neurons.
12. Compare pain threshold and pain tolerance.
3. Characterize the structure and function of
the dorsal root ganglion neurons in terms of 13. Differentiate acute pain from chronic pain in
sensory receptors, conduction velocities, and terms of mechanisms, manifestations, and
spinal cord projections. treatment.
4. Compare the discriminative pathway with 14. Describe the mechanisms of referred pain,
the anterolateral pathway, and explain the and list the common sites of referral for car-
clinical usefulness of this distinction. diac and other types of visceral pain.
5. Compare the tactile, thermal, and position 15. Describe three methods for assessing pain.
sense modalities in terms of receptors,
16. State the proposed mechanisms of pain relief
adequate stimuli, ascending pathways, and
associated with the use of heat, cold, transcu-
central integrative mechanisms.
taneous electrical nerve stimulation, and
6. Describe the role of clinical examination in acupuncture.
assessing somatosensory function.
17. Cite the mechanisms whereby nonnarcotic
7. Differentiate among the specificity, pattern, and narcotic analgesics, tricyclic antidepres-
gate control, and neuromatrix theories of pain. sants, and antiseizure drugs relieve pain.
8. Characterize the response of nociceptors to 18. Define allodynia, hypoesthesia,
stimuli that produce pain. hyperesthesia, paresthesias, hyperpathia,
analgesia, and hypoalgesia and hyperalgesia.
9. State the difference between the A- and
C-fiber neurons in the transmission of pain
information.

201
LWBK707-c35_p201-209.qxd 8/19/10 1:28 PM Page 202 Aptara Inc

202 UNIT 10 NERVOUS SYSTEM

19. Describe the cause and characteristics and 5. Somatosensory information from the face
treatment of neuropathic pain, trigeminal and cranial structures is transmitted by the
neuralgia, postherpetic neuralgia, and sensory neurons, which
complex regional pain syndrome. function in the same manner as the dorsal
root ganglion neurons.
20. Cite possible mechanisms of phantom limb
pain. 6. The region of the body wall that is supplied
by a single pair of dorsal root ganglia is called
21. State the importance of distinguishing
a .
between primary and secondary types of
headache. 7. The pathway is used for the
rapid transmission of sensory information
22. Differentiate between the periodicity of
such as discriminative touch.
occurrence and manifestations of migraine
headache, cluster headache, tension-type 8. The pathways provide for
headache, and headache due to transmission of sensory information such as
temporomandibular joint syndrome. pain, thermal sensations, crude touch, and
pressure that does not require discrete local-
23. Characterize the nonpharmacologic and
ization of signal source or fine discrimination
pharmacologic methods used in treatment of
of intensity.
headache.
9. Somatosensory experience can be divided
24. Cite the most common cause of
into , a term used for
temporomandibular joint pain.
qualitative, subjective distinctions between
25. State how the pain response may differ in sensations such as touch, heat, and pain.
children and older adults.
10. The receptive endings of different afferent
26. Explain how pain assessment may differ in neurons can initiate to many
children and older adults. forms of energy at high energy levels, but
they usually are highly tuned to be differen-
27. Explain how pain treatment may differ in
tially sensitive to low levels of a particular
children and older adults.
energy type.
11. The ability to discriminate the location of a
somesthetic stimulus is called
SECTION II: ASSESSING and is based on the sensory field in a
YOUR UNDERSTANDING dermatome innervated by an afferent
neuron.
Activity A Fill in the blanks.
12. The system, which relays
1. The system is designed to pro- sensory information regarding touch,
vide the central nervous system (CNS) with pressure, and vibration, is considered the
information related to deep and superficial basic somatosensory system.
body structures as contrasted to special
senses such sight and hearing. 13. sensation is discriminated by
three types of receptors: cold receptors,
2. somatic afferent neurons have warmth receptors, and pain receptors.
branches with widespread distribution
throughout the body and with many distinct 14. Attention, motivation, past experience, and
types of receptors that result in sensations the meaning of the situation can influence
such as pain, touch, and temperature. the individuals reaction to .

3. somatic afferent neurons sense 15. The experience of pain depends on both
position and movement of the body. stimulation and .

4. General afferent neurons have 16. pain arises from direct injury or
receptors on various visceral structures that dysfunction of the sensory axons of
sense fullness and discomfort. peripheral or central nerves.

Copyright 2011. Wolters Kluwer Health | Lippincott Williams & Wilkins. Study Guide for Porths Essentials of Pathophysiology, Third Edition.
LWBK707-c35_p201-209.qxd 8/19/10 1:28 PM Page 203 Aptara Inc

CHAPTER 35 SOMATOSENSORY FUNCTION, PAIN, AND HEADACHE 203

17. The theory proposes that the 31. is characterized by severe, brief,
brain contains a widely distributed neural often repetitive attacks of lightning-like or
network that contains somatosensory, limbic, throbbing pain.
and thalamocortical components.
32. headache is a type of primary
18. stimuli are objectively defined neurovascular headache that typically
as stimuli of such intensity that they cause or includes severe, unrelenting, unilateral pain
are close to causing tissue damage. located, in order of decreasing frequency, in
the orbital, retro-orbital, temporal,
19. Nociceptive stimulation that activates
supraorbital, and infraorbital region.
can cause a response known as
neurogenic inflammation that produces vasodi- 33. The most common type of headache is
lation and an increased release of chemical headache.
mediators to which nociceptors respond.
34. A common cause of head pain is
20. The faster-conducting fibers in the syndrome.
tract are associated mainly with
the transmission of sharp-fast pain informa- Activity B Consider the following figures.
tion to the thalamus.
21. The tract is a slower-
conducting, multisynaptic tract concerned
with the diffuse, dull, aching, and unpleasant
sensations that commonly are associated
with chronic and visceral pain.
22. Through research, it was found that electrical
stimulation of the midbrain
regions produced a state of analgesia that
lasted for many hours.
23. Three families of endogenous opioid peptides
have been identifiedthe ,
, and .
24. Pain and tolerance affect an
individuals response to a painful stimulus.
25. pain arises from superficial
structures, such as the skin and subcutaneous
tissues. 1. In the figure above, label the flowing
structures:
26. pain originates in deep body
structures, such as the periosteum, muscles, Receptor
tendons, joints, and blood vessels. Dorsal root ganglion
27. The purpose of acute pain is to serve as a First-order neuron
system. Second-order neuron
28. An drug is a medication that acts Thalamus
on the nervous system to decrease or eliminate Somatosensory cortex
pain without inducing loss of consciousness. Third-order neuron
29. Primary describes pain sensitiv-
ity that occurs directly in damaged tissues.
30. is the absence of pain on nox-
ious stimulation or the relief of pain without
loss of consciousness.

Copyright 2011. Wolters Kluwer Health | Lippincott Williams & Wilkins. Study Guide for Porths Essentials of Pathophysiology, Third Edition.
LWBK707-c35_p201-209.qxd 8/19/10 1:28 PM Page 204 Aptara Inc

204 UNIT 10 NERVOUS SYSTEM

8. Hyperpathia d. Convey cutaneous


pressure and touch
9. Type C fibers
sensation, cold
10. Type A fibers sensation,

Hip
Trunk
Head
Neck
Shoulder

Leg
Arm
Elbow m
mechanical pain,

Forea
Wri d
ot
Ha
and heat pain.
Fo
Lit g

st
n

r
Ri iddl

es
tle
e. Circulation to a
n e

To
M dex mb

n
In hu

E Ge cooled area under-


T

Noye goes alternating


s
Fac e periods of pallor
e
Uppe
r lip
caused by ischemia
Lips and flushing
caused by
Lower lip hyperemia
Teeth, gums, and jaw
f. Awareness of the
Tongue
stimuli, localization
Pharynx and discrimination
Intra- of their characteris-
abdominal tics, and interpreta-
tion of their
meaning.
g. Receives primary
sensory
2. Using the figure above, please answer the fol- information by
lowing questions: way of direct pro-
jections from the
Which area has the smallest receptor field? thalamus
Which area has the largest receptor field? h. Convey warm-hot
Which area has the highest acuity? sensation and
mechanical and
Activity C Match the key terms in Column A chemical as well
with their definitions in Column B. as heat- and cold-
induced pain
1. sensation
Column A Column B i. Respond to
1. Perception a. The perception of
mechanical, ther-
tactual, mal, and chemical
2. Somesthesia stimuli
proprioceptive, or
3. Type A fibers gut sensations j. Transmit informa-
b. Transmit informa-
tion from
4. Polymodal cutaneous and
receptors tion about muscle
length and tendon subcutaneous
5. Type B fibers stretch mechanoreceptors
6. Hunting reflex c. Sensory threshold
is raised
7. Primary
somatosensory
cortex

Copyright 2011. Wolters Kluwer Health | Lippincott Williams & Wilkins. Study Guide for Porths Essentials of Pathophysiology, Third Edition.
LWBK707-c35_p201-209.qxd 8/19/10 1:28 PM Page 205 Aptara Inc

CHAPTER 35 SOMATOSENSORY FUNCTION, PAIN, AND HEADACHE 205

2. 3. How much information can be obtained from


a single pinprick to the bottom of your patients
Column A Column B
foot?
1. Free nerve a. Stimulated by
endings rapid movements
of the tissues and
2. Meissner
adapts within a
corpuscles
few hundredths of
3. Merkel disks a second 4. What is the gate control theory of pain?
4. Pacinian b. Unmyelinated
corpuscles fibers entwined
around most of the
5. Hair follicle length of the hair
end-organs follicle that detect
movement on the 5. How can the phenomena of referred pain be
6. Ruffini
surface of the body explained?
end-organs
c. Are responsible for
giving steady-state
signals that allow
for continuous
determination of
6. In many sports injuries the athlete may be
touch against the
instructed to place heat on the injured area.
skin
What is the effect on pain originating from
d. Detect touch and the injury?
pressure
e. Found in joint cap-
sules
f. Elongated
encapsulated nerve
ending that is pres- 7. What is phantom limb pain and what are
ent in nonhairy some of the theories postulated to explain its
parts of the skin presence?

Activity D Briefly answer the following.


1. How are sensory systems organized?

8. What are the differences and similarities


between migraine headaches with aura and
migraine headaches without aura?

2. What are the types of sensory information


that can be perceived by our sensory
receptors?

9. What is known about the pathology of pain


during a migrainous headache?

Copyright 2011. Wolters Kluwer Health | Lippincott Williams & Wilkins. Study Guide for Porths Essentials of Pathophysiology, Third Edition.
LWBK707-c35_p201-209.qxd 8/19/10 1:28 PM Page 206 Aptara Inc

206 UNIT 10 NERVOUS SYSTEM

SECTION III: APPLYING 3. General visceral b. Transmit sensory


afferent neurons information from
YOUR KNOWLEDGE the periphery to the
4. First-order
CNS
Activity E Consider the scenario and answer neurons
the questions. c. Communicate with
5. Second-order various reflex net-
An 82-year-old woman is brought to the neurons works and sensory
emergency department by ambulance from a pathways in the
6. Third-order
local nursing home. The report from the accom- spinal cord and
neurons
panying staff member is that the client suffers travel directly to the
from a physiologic dementia, and that 2 days ago 7. Dorsal root thalamus
she suffered a fall in the bathroom. The client ganglion
d. Transmits all so-
denies pain, but has been restless and agitated neurons
matosensory infor-
since the fall, and today she will not use her
8. Trigeminal mation from the
right arm.
sensory neurons limbs and trunk
1. The caregiver asks the nurse how the health e. Sense position and
care team is going to assess this clients pain as movement of the
the client cannot give them any accurate body
information. What is the nurses best
f. Somatosensory in-
response?
formation from the
face and cranial
structures
g. Sense fullness and
discomfort
2. The client is diagnosed with a fractured right h. Relay information
ulna. She is taken to the operating room, from the thalamus
where the arm is aligned and cast. When the to the cerebral
client is ready for release back to the nursing cortex
home, the caregiver asks what can be done for
the clients discomfort. What teaching would 2. Match the term with the definition.
the nurse include at discharge? Term Definition
1. Discriminative a. The region of the
touch body wall that is
supplied by a single
2. Sensory unit
pair of dorsal root
3. Type A fibers ganglia
4. Type A and b. Stimulate auto-
SECTION IV: PRACTICING A fibers nomic nervous sys-
FOR NCLEX tem responses, such
5. Type B fibers as a rise in heart
Activity F Answer the following questions. 6. Dermatome rate and blood pres-
sure, dilation of the
1. Match the type of neuron with the informa- 7. Discriminative pupils, and the
tion they transmit and where they transmit it pathway pale, moist skin
to. that results from
8. Stereognosis
Information constriction of the
9. Anterolateral cutaneous blood
Type of Neuron Transmitted and Site
pathway vessels and activa-
1. Special somatic a. Sensations such as tion of the sweat
afferent neurons pain, touch, and glands
temperature
2. General somatic
afferent neurons

Copyright 2011. Wolters Kluwer Health | Lippincott Williams & Wilkins. Study Guide for Porths Essentials of Pathophysiology, Third Edition.
LWBK707-c35_p201-209.qxd 8/19/10 1:28 PM Page 207 Aptara Inc

CHAPTER 35 SOMATOSENSORY FUNCTION, PAIN, AND HEADACHE 207

10. Free nerve c. Identifies the size 3. A neurologic assessment of the


endings and shape of ob- somatosensory function of the body is often
jects and their necessary for diagnostic information. How is
11. Nociceptors
movement across this assessment done?
the skin; tempera- a. Testing the integrity of spinal segmental
ture sensation; nerves
sense of movement
b. Testing the integrity of cranial nerves
of the limbs and
joints of the body; c. Testing the integrity of peripheral nerves
and nociception, or d. Testing the integrity of the CNS
pain
4. When testing nociceptive stimuli to elicit a
d. Transmit informa- withdrawal reflex in the body, what stimuli
tion about muscle are commonly used?
length and tendon
a. Weak electrical current
stretch
b. Pressure from a sharp object
e. Sensory receptors
that are activated c. Skin temperature damp cotton ball
by noxious insults d. Water heated to 5C above skin tempera-
to peripheral tissues ture
f. Carry the informa- 5. One of the neurotransmitters between the
tion from the spinal nociceptive neurons and the dorsal horn neu-
cord to the thala- rons is a major excitatory neurotransmitter.
mic level of sensa- What is this neurotransmitter?
tion and relays
a. Norepinephrine
precise information
regarding spatial b. Substance P
orientation c. Glutamate
g. The cell body of the d. Dopamine
dorsal root ganglion
6. Which tract in the spinal cord conducts the
neuron, its periph-
diffuse, dull, aching sensations that are asso-
eral branch (which
ciated with chronic and visceral pain?
innervates a small
area of periphery), a. Multisynaptic tract
and its central axon b. Neospinothalamic tract
(which projects to c. Anterolateral tract
the CNS)
d. Paleospinothalamic tract
h. Transmit informa-
tion from cuta-
neous and
subcutaneous
mechanoreceptors
i. Convey cutaneous
pressure and touch
sensation, cold sen-
sation, mechanical
pain, and heat pain.
j. The sense of shape
and size of an ob-
ject in the absence
of visualization
k. Detect touch and
pressure

Copyright 2011. Wolters Kluwer Health | Lippincott Williams & Wilkins. Study Guide for Porths Essentials of Pathophysiology, Third Edition.
LWBK707-c35_p201-209.qxd 8/19/10 1:28 PM Page 208 Aptara Inc

208 UNIT 10 NERVOUS SYSTEM

7. Match the type of pain with its description 9. When giving pain medicine for acute pain,
health care workers are reluctant to provide
Type of Pain Description of Pain
much needed opioid pain medicine. What is
1. Deep somatic a. Extends for long pe- the major concern of health care workers
pain riods of time and when providing opioid pain relief?
generally represents a. Fear of addiction
2. Cutaneous
low levels of under-
pain b. Fear of depressed respirations
lying pathology that
3. Visceral pain does not explain the c. Fear of oversedation
presence and/or ex- d. Fear of adverse reactions
4. Referred pain
tent of the pain.
10. Chronic pain is difficult to treat. Cancer, a
5. Guarding b. The pains location, common cause of chronic pain, has been
6. Acute pain radiation, intensity, especially addressed by the World Health
and duration, as Organization (WHO). What has WHO created
7. Chronic pain well as those factors to assist clinicians in choosing appropriate
that aggravate or re- analgesics?
lieve it, provide es-
a. An opioid ladder for pain control
sential diagnostic
clues. b. An analgesic ladder for pain control
c. Type of pain experi- c. Stepping stones for pain control
enced from a d. A list of nonpharmacologic ways to con-
sprained ankle. trol pain
d. A sharp pain with a 11. In describing the ideal analgesic, what factors
burning quality and would be included? (Mark all that apply.)
may be abrupt or
a. Inexpensive
slow in onset.
b. Have minimal adverse effects
e. A protective reflex
rigidity; its purpose c. Effective
is to protect the af- d. Addictive
fected body parts. e. Decrease the level of consciousness
f. Diffuse and poorly
12. Using surgery to relieve severe, intractable
localized nature
pain has been successful to a degree. What
with a tendency to
can surgery be used for when a person is in
be referred to other
pain?
locations
a. Relief of severe peripheral contractures
g. Perceived at a site
different from its b. Cure inoperable cancer
point of origin but c. Block transmission of phantom limb pain
innervated by the d. Cure severe myalgia
same spinal segment
13. When a peripheral nerve is irritated enough,
8. It is often necessary to assess a clients pain. it becomes hypersensitive to the noxious
What factors would you assess when stimuli, which results in increased painfulness
assessing pain? (Mark all that apply.) or hyperalgesia. Health care professionals rec-
ognize both primary and secondary forms of
a. Nature and severity of pain
hyperalgesia. What is primary hyperalgesia?
b. Severity and spinal reflex involvement of
a. Pain that occurs in the tissue surrounding
pain
an injury.
c. Location and radiation of pain
b. Pain sensitivity that lasts longer than 1 week
d. Spinal reflex involvement and nature of
c. Pain sensitivity that occurs in the viscera
pain
d. Pain sensitivity that occurs directly in
e. Spinal tract involvement and radiation of
damaged tissues
pain

Copyright 2011. Wolters Kluwer Health | Lippincott Williams & Wilkins. Study Guide for Porths Essentials of Pathophysiology, Third Edition.
LWBK707-c35_p201-209.qxd 8/19/10 1:28 PM Page 209 Aptara Inc

CHAPTER 35 SOMATOSENSORY FUNCTION, PAIN, AND HEADACHE 209

14. Match the type of pain with its description. 15. Phantom limb pain is a little understood pain
that develops after an amputation. Because it
Type of Pain Description
is little understood, it is difficult to treat,
1. Neuropathic a. Manifested by facial even though the client is experiencing severe
pain tics or spasms and pain. What are the treatments for phantom
characterized by limb pain?
2. Neuralgia
paroxysmal attacks a. Sympathetic blocks and hypnosis
3. Tic douloureux of stabbing pain
b. Relaxation training and transcutaneous
that usually are lim-
4. Postherpetic electrical nerve stimulation on the effer-
ited to the unilateral
neuralgia ents in the area
sensory distribution
of one or more c. Narcotic analgesics and relaxation training
branches of the d. Biofeedback and nonsteroidal anti-inflam-
trigeminal nerve, matory drugs
most often the max-
16. Migraine headaches affect millions of people
illary or mandibular
worldwide. What are first-line agents for the
divisions.
treatment of migraine headaches?
b. Characterized by
a. Ondansetron and morphine
severe, brief, often
repetitive attacks of b. Naproxen sodium and metoclopramide
lightning-like or c. Sumatriptan and tramadol
throbbing pain. d. Caffeine and syrup of ipecac
c. Affected sensory
17. A severe type of headache that occurs more fre-
ganglia and the pe-
quently in men than women and is described
ripheral nerve to
as having unrelenting, unilateral pain located
the skin of the cor-
most frequently in the orbit is called what?
responding der-
matomes cause a a. Migraine headache
unilateral localized b. Tension headache
vesicular eruption c. Cluster headache
and hyperpathia
d. Chronic daily headache
(i.e., abnormally ex-
aggerated subjective 18. When assessing pain in children, it is impor-
response to pain). tant to use the correct pain rating scale. What
d. Widespread pain would be the appropriate pain rating scale
that is not other- with children from the 3- to 8-year-old range?
wise explainable, a. COMFORT pain scale
burning pain, and b. FLACC pain scale
attacks of pain that
c. CRIES pain scale
occur without seem-
ing provocation. d. FACES pain scale
19. Children feel pain just as much as adults do.
What is the major principle in pain manage-
ment in the pediatric population?
a. Treat on individual basis and match anal-
gesic agent with cause and level of pain.
b. Always use nonpharmacologic pain man-
agement before using pharmacologic pain
management.
c. Base treatment of pain on gender and age
group.
d. Treat pediatric pain the way the parents
want you to.

Copyright 2011. Wolters Kluwer Health | Lippincott Williams & Wilkins. Study Guide for Porths Essentials of Pathophysiology, Third Edition.
LWBK707-c36_p210-217.qxd 8/19/10 2:22 PM Page 210 Aptara Inc

36
CHAPTER
Disorders of
Neuromuscular Function

SECTION I: LEARNING 10. Trace the steps in regeneration of an injured


peripheral nerve.
OBJECTIVES
11. Compare the cause and manifestations of
1. Define the term motor unit and characterize peripheral mononeuropathies with polyneu-
its mechanism of controlling skeletal muscle ropathies.
movement.
12. Describe the manifestation of peripheral
2. Describe the distribution of upper and lower nerve root injury due to a ruptured interver-
motor neurons in relation to the central tebral disk.
nervous system (CNS).
13. Relate the functions of the cerebellum to pro-
3. Differentiate between the functions of the duction of vestibulocerebellar dysfunction,
primary, premotor, and supplemental motor decomposition of movement, and cerebellar
cortices. tremor.
4. Compare the effect of upper and lower motor 14. Describe the functional organization of the
neuron lesions on the spinal cord stretch basal ganglia and communication pathways
reflex function and muscle tone. with the thalamus and cerebral cortex.
5. Describe muscle atrophy and differentiate 15. State the possible mechanisms responsible
between disuse and denervation atrophy. for the development of Parkinson disease and
characterize the manifestations and
6. Relate the molecular changes in muscle struc-
treatment of the disorder.
ture that occur in Duchenne muscular
dystrophy to the clinical manifestations of 16. Relate the pathologic upper motor neuron
the disease. and lower motor neuron (LMN) changes that
occur in amyotrophic lateral sclerosis to the
7. Describe the actions of Clostridium botulinum
manifestations of the disease.
neurotoxins in terms of their pathologic and
therapeutic potential. 17. Explain the significance of demyelination
and plaque formation in multiple sclerosis.
8. Relate the clinical manifestations of myasthe-
nia gravis to its cause. 17. Describe the manifestations of multiple
sclerosis.
9. Define the term peripheral nervous system and
describe the characteristics of peripheral 18. Relate the structures of the vertebral column
nerves. to mechanisms of spinal cord injury.

210
LWBK707-c36_p210-217.qxd 8/19/10 2:22 PM Page 211 Aptara Inc

CHAPTER 36 DISORDERS OF NEUROMUSCULAR FUNCTION 211

19. Explain how loss of upper motor neuron 10. are found in muscle tendons
function contributes to the muscle spasms and transmit information about muscle ten-
that occur after recovery from spinal cord sion or force of contraction at the junction of
injury. the muscle and the tendon that attaches to
bone.
20. State the effects of spinal cord injury on ven-
tilation and communication, the autonomic 11. Stretch reflexes tend to be hypoactive or
nervous system, cardiovascular function, sen- absent in cases of nerve damage
sorimotor function, and bowel, bladder, and or ventral horn injury involving the test area.
sexual function.
12. Abnormalities in any part of the
pathway can produce muscle
weakness.
SECTION II: ASSESSING 13. Muscular usually results from
YOUR UNDERSTANDING LMN lesions as well as diseases of the muscle
themselves.
Activity A Fill in the blanks.
14. Any interruption of the myotatic or stretch
1. , whether it involves walking, reflex circuitry by peripheral nerve injury,
running, or precise finger movements, pathology of the neuromuscular junction,
requires movement and maintenance of injury to the spinal cord, or damage to the
posture. corticospinal system can results in
disturbances of .
2. The contains the neuronal cir-
cuits that mediate a variety of reflexes and 15. Hyperactive reflexes are suggestive of a
automatic rhythmic movements. disorder.

3. Most reflexes are , meaning that 16. suggests the presence of a LMN
they involve one or more interposed lesion.
interneurons. 17. Disorders affecting the nerve cell body are
4. The medial descending systems of the often referred to , those
brain stem contribute to the control of affecting the nerve axon, as
by integrating visual, vestibular, neuropathies; and primary disorders affecting
and somatosensory information. the muscle fibers as .

5. The is the highest level of 18. Muscular is a term applied to a


motor function. number of genetic disorders that produce
progressive deterioration of skeletal muscles
6. The primary cortex is located because of mixed muscle cell hypertrophy,
on the rostral surface and adjacent portions atrophy, and necrosis.
of the central sulcus.
19. If the LMN dies or its axon is destroyed, the
7. The and provide skeletal muscle cell begins to have temporary
feedback circuits that regulate cortical and spontaneous contractions, called .
brain stem motor areas.
20. muscular dystrophy is inherited
8. Cerebellar are involved with as a recessive single-gene defect on the X
the timing and coordination of movements chromosome and is transmitted from the
that are in progress and with learning of mother to her male offspring.
motor skills.
21. The serves as a synapse between
9. The , which are distributed a motor neuron and a skeletal muscle fiber.
throughout the belly of a muscle, relay infor-
mation about muscle length and rate of 22. Neurotoxins from the botulism organism
stretch. (C. botulinum) produce paralysis by blocking
release.

Copyright 2011. Wolters Kluwer Health | Lippincott Williams & Wilkins. Study Guide for Porths Essentials of Pathophysiology, Third Edition.
LWBK707-c36_p210-217.qxd 8/19/10 2:22 PM Page 212 Aptara Inc

212 UNIT 10 NERVOUS SYSTEM

23. is a disorder of transmission at 34. affects motor neurons in three


the neuromuscular junction that affects com- locations: the anterior horn cells of the spinal
munication between the motor neuron and cord; the motor nuclei of the brain stem, par-
the innervated muscle cell. ticularly the hypoglossal nuclei; and the
UMNs of the cerebral cortex.
24. Lower motor neuron diseases are progressive
neurologic illnesses that selectively affect the 35. is characterized by inflammation
anterior horn cells of the and and selective destruction of CNS myelin.
motor neurons.
36. The pathophysiology of multiple sclerosis
25. There are two main types of involves the of nerve fibers in
injury based on the target of the insult: the white matter of the brain, spinal cord,
segmental demyelination involving the and optic nerve.
Schwann cell and axonal degeneration
37. The most common cause of is
involving the neuronal cell body and/or its
motor vehicle accidents, followed by falls,
axon.
violence (primarily gunshot wounds), and
26. usually are caused by localized recreational sporting activities.
conditions such as trauma, compression, or
38. Sudden complete transection of the spinal
infections that affect a single spinal nerve,
cord results in complete of
plexus, or peripheral nerve trunk.
motor, sensory, reflex, and autonomic
27. involve demyelination or function below the level of injury.
axonal degeneration of multiple peripheral
39. is the impairment or loss of
nerves that leads to symmetric sensory,
motor or sensory function (or both) after
motor, or mixed sensorimotor deficits.
damage to neural structures in the cervical
28. The signs and symptoms of a segments of the spinal cord.
are localized to the area of the body
40. refers to impairment or loss of
innervated by the nerve roots and include
motor or sensory function (or both) in the
both motor and sensory manifestations.
thoracic, lumbar, or sacral segments of the
29. Loss of function can result in spinal cord from damage of neural elements
total incoordination of these functions even in the spinal canal.
though its loss does not result in paralysis.
41. Vagal stimulation that causes a marked
30. The are a group of deep, inter- bradycardia is called the
related subcortical nuclei that play an essen- response.
tial role in control of movement.
42. hypotension usually occurs in
31. Disorders of the basal ganglia comprise a persons with injuries at T4 to T6 and above
complex group of motor disturbances charac- and is related to the interruption of
terized by and other descending control of sympathetic outflow
involuntary movements, changes in posture to blood vessels in the extremities and
and muscle tone, and poverty and slowness abdomen.
of movement.
43. The high risk for in acute spinal
32. disease is a degenerative disor- cord injury patients is due to immobility,
der of basal ganglia function that results in decreased vasomotor tone below the level of
variable combinations of tremor, rigidity, and injury, and hypercoagulability and stasis of
bradykinesia. blood flow.
33. The cardinal manifestations of Parkinson dis-
ease are tremor, rigidity, and or
slowness of movement.

Copyright 2011. Wolters Kluwer Health | Lippincott Williams & Wilkins. Study Guide for Porths Essentials of Pathophysiology, Third Edition.
LWBK707-c36_p210-217.qxd 8/19/10 2:22 PM Page 213 Aptara Inc

CHAPTER 36 DISORDERS OF NEUROMUSCULAR FUNCTION 213

Activity B Consider the following figure. f. The failure to accu-


rately perform
rapid alternating
movements
g. Sense of body
movement and
position
h. A wide-based
unsteady gait
i. Visible squirming
and twitching
movements of
muscle
j. Involuntary motor
In the figure above, locate and label the responses
following areas of the brain:
2.
Premotor cortex
Motor cortex Column A Column B
Broca area 1. Bradykinesia a. Muscle shrinkage
Vestibular cortex due to loss of neu-
2. Dystonia
ral stimulus
Primary auditory cortex
3. Chorea b. Involuntary
Primary visual cortex
4. Truncal ataxia jerking movement
Somatosensory cortex
c. Slowness of move-
Frontal eye fields 5. Myoclonus
ments
6. Parkinsonism d. Rhythmic
Activity C Match the key terms in Column A movements of a
7. Dysmetria
with their definitions in Column B. particular body
8. Denervation part
1.
atrophy
e. Abnormal simulta-
Column A Column B
9. Constant neous contractions
1. Clonus a. Increased muscle conjugate of agonist and
resistance that readjustment of antagonist muscles
2. Paralysis varies and eye position f. Abnormal writhing
3. Fasciculations commonly movements
becomes worse at 10. Tremor
4. Motor the extremities of g. Inaccuracies of
homunculus the range of movements
motion leading to a failure
5. Spasticity
to reach a specified
6. Dysdiado- b. Incomplete loss of target
chokinesia strength
h. Nystagmus
c. Loss of movement
7. Reflex i. Unsteadiness of
d. Rhythmic contrac- the trunk
8. Ataxia tion and alternate
relaxation of a j. Syndrome arising
9. Proprioception from the degenera-
limb
10. Paresis tive changes in
e. Somatotopic array basal ganglia
of the body repre- function
senting motor
areas

Copyright 2011. Wolters Kluwer Health | Lippincott Williams & Wilkins. Study Guide for Porths Essentials of Pathophysiology, Third Edition.
LWBK707-c36_p210-217.qxd 8/19/10 2:22 PM Page 214 Aptara Inc

214 UNIT 10 NERVOUS SYSTEM

Activity D 6. Compare segmental demyelination with


axonal degeneration in relation to peripheral
1. In the boxes below, put the following events nerve injuries.
regarding synaptic transmission in order:

S S S S

1. Inactivation by acetylcholinesterase
2. Action potential arrives at synaptic 7. What is carpal tunnel syndrome?
terminal
3. Depolarization of motor-end plate
4. Release of acetylcholine into synapse
5. Influx of Ca2

Activity E Briefly answer the following. 8. What are the clinical manifestations of Guil-
lain-Barr syndrome?
1. Describe the basic hierarchy of organization
of motor movement.

9. What is the current theory of the pathogene-


2. What is the basic unit of motor control? How sis of Parkinson disease?
does it vary between gross motor movement
and fine motor movements?

10. What does amyotrophic lateral sclerosis


imply?
3. What is a muscle spindle and how does it
work?

11. What are the two pathologic types of spinal


4. What for areas must be integrated in order chord injury?
for muscle movement to be coordinated?

5. Describe the molecular causation of


Duchenne muscular dystrophy.

Copyright 2011. Wolters Kluwer Health | Lippincott Williams & Wilkins. Study Guide for Porths Essentials of Pathophysiology, Third Edition.
LWBK707-c36_p210-217.qxd 8/19/10 2:22 PM Page 215 Aptara Inc

CHAPTER 36 DISORDERS OF NEUROMUSCULAR FUNCTION 215

SECTION III: APPLYING 2. Match the neurons with their function/


description.
YOUR KNOWLEDGE
Neuron Function/Description
Activity F Consider the scenario and answer
1. Motor neurons a. Motor neuron and
the questions.
the group of muscle
2. Motor unit
A 27-year-old man is brought into the emergency fibers it innervates
department after falling out of a tree stand while 3. Lower motor in a muscle
deer hunting. He is awake and alert and states neurons b. Control motor
that he cannot feel or move his legs. An MRI function
4. Upper motor
indicates a subluxation of the vertebrae with
neurons c. Project from the
fractures above and below the subluxation.
motor strip in the
1. The mans wife arrives at the emergency cerebral cortex to
department. She asks the nurse what medicine the ventral horn
is in the intravenous line and why her and are fully con-
husband is receiving it. What would the nurse tained within the
include in her answer to the wife? CNS
d. The motor neurons
supplying a motor
unit are located in
the ventral horn of
the spinal cord
2. The client is transferred to a neurosurgical
intensive care unit. As the nurse caring for
this client, what orders would you expect to 3. Reflexes are basically hard-wired into the
receive? CNS. Anatomically, the basis of a reflex is an
afferent neuron that synapses directly with
an effector neuron that causes muscle move-
ment. Sometimes the afferent neuron
synapses with what intermediary between the
afferent and effector neurons?
a. Neurotransmitter
b. Interneuron
SECTION IV: PRACTICING c. Intersegmental effectors
FOR NCLEX d. Suprasegmental effectors
Activity G Answer the following questions. 4. The signs and symptoms produced by disor-
ders of the motor system are useful in finding
1. The spinal cord contains the basic factors
the disorder. What signs and symptoms
necessary to coordinate function when a
would you assess when looking for a disorder
movement is planned. It is the lowest level of
of the motor system? (Mark all that apply.)
function. What is the highest level of
function in planning movement? a. Spinal reflex activity
a. Frontal cortex b. Bulk
b. Cerebral cortex c. Motor coordination
c. Pons d. Muscle innervation
d. Cerebellum e. Tone

Copyright 2011. Wolters Kluwer Health | Lippincott Williams & Wilkins. Study Guide for Porths Essentials of Pathophysiology, Third Edition.
LWBK707-c36_p210-217.qxd 8/19/10 2:22 PM Page 216 Aptara Inc

216 UNIT 10 NERVOUS SYSTEM

5. Duchenne muscular dystrophy usually does 10. Match the cerebellar pathway with its
not produce any signs or symptoms until function.
between the ages of 2 and 3. What muscles
Cerebellar Pathway Function
are usually first to be affected in Duchenne
muscular dystrophy? 1. Vestibulocere- a. Maintains equilib-
a. Muscles of the upper arms bellar pathway rium and posture
b. Large muscles of the legs 2. Spinocerebellar b. Provides the cir-
pathway cuitry for coordi-
c. Postural muscles of hip and shoulder
nating the
d. Spinal and neck muscles 3. Cerebrocerebellar movements of the
pathway distal portions of
6. Antibiotics such as gentamicin can produce a
disturbance in the body that is similar to botu- the limbs
lism by preventing the release of acetylcholine c. Coordinates se-
from nerve endings. In persons with pre-exist- quential body and
ing neuromuscular transmission disturbances limb movements.
these drugs can be dangerous. What disease
falls into this category? 11. The basal ganglia play a role in coordinated
a. Multiple sclerosis movements. Part of the basal ganglia system
is the striatum, which involves local choliner-
b. Duchenne muscular dystrophy
gic interneurons. What disease is thought to
c. Becker muscular dystrophy be related to the destruction of the choliner-
d. Myasthenia gravis gic interneurons?
7. In myasthenia gravis, periods of stress can a. Parkinson syndrome
produce myasthenia crisis. When does myas- b. Guillain-Barr syndrome
thenia crisis occur? c. Myasthenia gravis
a. When muscle weakness becomes severe d. Huntington disease
enough to compromise ventilation
12. What disease results from the degeneration of
b. When the client is too weak to hold the
the dopamine nigrostriatal system of the
head up
basal ganglia?
c. When the client is so weak he or she can-
a. Parkinson disease
not lift the arms
b. Huntington disease
d. When the client can no longer walk
c. Guillain-Barr syndrome
8. Peripheral nerve disorders are not
d. Myasthenia gravis
uncommon. What is an example of a fairly
common mononeuropathy? 13. Amyotrophic lateral sclerosis is considered a
a. Guillain-Barr syndrome disease of the upper motor neurons. What is
the most common clinical presentation of
b. Carpal tunnel syndrome
amyotrophic lateral sclerosis?
c. Myasthenia gravis
a. Rapidly progressive weakness and
d. Phalen syndrome atrophy in distal muscles of both upper
9. Herniated disks occur when the nucleus pulpo- extremities
sus is compressed enough that it protrudes b. Slowly progressive weakness and
through the annulus fibrosus, putting pressure atrophy in distal muscles of one upper
on the nerve root. This type of injury occurs extremity
most often in the cervical and lumbar region c. Rapidly progressive weakness and
of the spine. What is an important diagnostic atrophy in distal muscles of both lower
test for a herniated disk in the lumbar region? extremities
a. Hip flexion test d. Slowly progressive weakness and
b. CT scan atrophy in distal muscles of one lower
c. Straight-leg test extremity
d. Electromyelography

Copyright 2011. Wolters Kluwer Health | Lippincott Williams & Wilkins. Study Guide for Porths Essentials of Pathophysiology, Third Edition.
LWBK707-c36_p210-217.qxd 8/19/10 2:22 PM Page 217 Aptara Inc

CHAPTER 36 DISORDERS OF NEUROMUSCULAR FUNCTION 217

14. While there is no laboratory test that is 17. Approximately 6 months after a spinal cord
diagnostic for multiple sclerosis, some patients injury, a 29-year-old man has an episode of
have alterations in their cerebrospinal fluid autonomic dysreflexia. What are the charac-
(CSF) that can be seen when a portion of the teristics of autonomic dysreflexia? (Mark all
CSF is removed during a spinal tap. What find- that apply.)
ing in CSF is suggestive of multiple sclerosis? a. Hypertension
a. Decreased immunoglobulin G levels b. Fever
b. Decreased total protein levels c. Skin pallor
c. Oligoclonal patterns d. Vasoconstriction
d. Decreased lymphocytes e. Piloerector response
15. At what level of the cervical spine would an 18. Bowel dysfunction is one of the most difficult
injury allow finger flexion? problems to deal with after a spinal cord
a. C5 injury. After a spinal cord injury, most people
b. C6 experience constipation. Why does this
occur?
c. C7
a. Innervation of the bowel is absent
d. C8
b. Defecation reflex is lost
16. A 14-year-girl has been thrown from the back
c. Internal anal sphincter will not relax
of a pick-up truck. MRI shows broken
vertebrae at the C2 level. What is the main d. Peristaltic movements are not strong
significance of an injury at this level of the enough to move stool through the colon
spinal column?
a. Cannot breathe on own, needs ventilator
assistance
b. Partial or full diaphragmatic function; ven-
tilation is diminished because of the loss
of intercostal muscle function, resulting in
shallow breaths and a weak cough
c. Intercostal and abdominal musculature is
affected; the ability to take a deep breath
and cough is less impaired
d. Needs maintenance therapy to strengthen
existing muscles for endurance and mobi-
lization of secretions

Copyright 2011. Wolters Kluwer Health | Lippincott Williams & Wilkins. Study Guide for Porths Essentials of Pathophysiology, Third Edition.
LWBK707-c37_p218-225.qxd 8/19/10 2:22 PM Page 218 Aptara Inc

37
CHAPTER
Disorders of
Brain Function

SECTION I: LEARNING 11. Define consciousness and trace the rostral-to-


caudal progression of consciousness in terms
OBJECTIVES of pupillary changes, respiration, and motor
function as the effects of brain dysfunction
1. Differentiate cerebral hypoxia from cerebral
progress to involve structures in the
ischemia and focal from global ischemia.
diencephalon, midbrain, pons, and medulla.
2. Characterize the role of excitatory amino
12. List the major vessels in the cerebral circula-
acids as a common pathway for neurologic
tion and state the contribution of the
disorders.
internal carotid arteries, the vertebral arteries,
3. State the determinants of intracranial and the circle of Willis to the cerebral circula-
pressure and describe compensatory mecha- tion.
nisms used to prevent large changes in
13. Describe the autoregulation of cerebral blood
intracranial pressure when there are changes
flow.
in brain, blood, and cerebrospinal fluid (CSF)
volumes. 14. Explain the substitution of brain attack for
stroke in terms of making a case for early
4. Explain the causes of tentorial herniation of
diagnosis and treatment.
the brain and its consequences.
15. Differentiate the pathologies of ischemic and
5. Compare the causes of communicating and
hemorrhagic stroke.
noncommunicating hydrocephalus.
16. Explain the significance of transient ischemic
6. Compare cytotoxic, vasogenic, and
attacks, the ischemic penumbra, and
interstitial cerebral edema.
watershed zones of infarction and how these
7. Differentiate primary and secondary brain conditions relate to ischemic stroke.
injuries due to head trauma.
17. Cite the most common cause of
8. Describe the mechanism of brain damage in subarachnoid hemorrhage and state the com-
coupcontrecoup injuries. plications associated with subarachnoid hem-
orrhage.
9. List the constellation of symptoms involved
in the postconcussion syndrome. 18. Describe the alterations in cerebral
vasculature that occur with arteriovenous
10. Differentiate among the location, manifesta-
malformations.
tions, and morbidity of epidural, subdural,
and intracerebral hematoma.

218
LWBK707-c37_p218-225.qxd 8/19/10 2:22 PM Page 219 Aptara Inc

CHAPTER 37 DISORDERS OF BRAIN FUNCTION 219

19. Describe the patterns of motor deficits and 6. refers to short serpiginous seg-
typical problems with speech and language ments of necrosis that occur within and par-
that occur as a result of stroke. allel to the cerebral cortex, in areas supplied
by the penetrating arteries during an
20. List the sequence of events that occur with
ischemic event.
meningitis.
7. In many neurologic disorders, various media-
21. Describe the symptoms of encephalitis.
tors including excitatory , cate-
22. List the major categories of brain tumors and cholamines, nitric oxide, free radicals,
interpret the meaning of benign and inflammatory cells, apoptosis, and intracellu-
malignant as related to brain tumors. lar may cause injury to
neurons.
23. Describe the general manifestations of brain
tumors. 8. Increased pressure is a common
pathway for brain injury from different types
24. List the methods used in diagnosis and treat-
of insults and agents.
ment of brain tumors.
9. Brain represents a displacement
25. Explain the difference between a seizure and
of brain tissue under the falx cerebri or
epilepsy.
through the tentorial notch or incisura of the
26. State four or more causes of seizures other tentorium cerebelli.
than epilepsy.
10. Cerebral is an increase in tissue
27. Differentiate between the origin of seizure volume secondary to abnormal fluid accumu-
activity in partial and generalized forms of lation.
epilepsy and compare the manifestations of
11. The functional manifestations of
simple partial seizures with those of complex
edema include focal neurologic deficits,
partial seizures and major and minor motor
disturbances in consciousness, and severe
seizures.
intracranial hypertension.
28. Characterize status epilepticus.
12. edema involves an increase in
intracellular fluid.
13. The effects of traumatic head injuries can be
SECTION II: ASSESSING divided into two categories:
YOUR UNDERSTANDING injuries, in which damage is caused by
impact; and secondary injuries, in which
Activity A Fill in the blanks. damage results from the subsequent brain
1. A number of regulatory mechanisms, includ- swelling, infection, or .
ing the blood-brain barrier and autoregulatory 14. usually are caused by head
mechanisms that ensure its blood supply, injury in which the skull is fractured.
maintains the electrically active
cells. 15. A subdural hematoma develops in the area
between the dura and the arachnoid and usu-
2. Although the brain makes up only 2% of ally is the result of a in the
the body weight, it receives 15% of the small bridging veins that connect veins on
resting cardiac output and accounts for the surface of the cortex to dural sinuses.
% of the oxygen consumption.
16. is the state of awareness of self
3. Because indicates decreased and the environment and of being able to
oxygen levels in all brain tissue, it produces a become oriented to new stimuli.
generalized depressant effect on the brain.
17. Brain death is defined as the irreversible loss
4. Cerebral ischemia can be , as in of function of the , including
stroke, or , as in cardiac arrest. the brain stem.
5. Excessive influx of during neu- 18. The state is characterized by
ral ischemia results in neuronal and intersti- loss of all cognitive functions and the
tial edema. unawareness of self and surroundings.

Copyright 2011. Wolters Kluwer Health | Lippincott Williams & Wilkins. Study Guide for Porths Essentials of Pathophysiology, Third Edition.
LWBK707-c37_p218-225.qxd 8/19/10 2:22 PM Page 220 Aptara Inc

220 UNIT 10 NERVOUS SYSTEM

19. Cerebral has been classically 33. The use of for brain tumors is
defined as the ability of the brain to maintain somewhat limited by the blood-brain barrier.
constant cerebral blood flow despite changes
34. A represents the abnormal
in systemic arterial pressure.
behavior caused by an electrical discharge
20. At least three metabolic factors affect cerebral from neurons in the cerebral cortex.
blood flow: , , and
35. seizures usually involve only
concentration.
one hemisphere and are not accompanied by
21. is the syndrome of acute focal loss of consciousness or responsiveness.
neurologic deficit from a vascular disorder
36. seizures involve impairment of
that injures brain tissue.
consciousness and often arise from the
22. strokes are caused by an temporal lobe.
interruption of blood flow in a cerebral vessel,
37. Myoclonic seizures involve brief involuntary
and strokes are caused by bleed-
induced by stimuli of cerebral
ing into brain tissue.
origin.
23. TIA or is equivalent to brain
38. seizures usually present with a
angina and reflects a temporary disturbance
person having a vague warning and
in focal cerebral blood flow, which reverses
experience a sharp tonic contraction of the
before infarction occurs, analogous to
muscles with extension of the extremities
in relation to heart attack.
and immediate loss of consciousness.
24. are the most common cause of
39. Seizures that do not stop spontaneously or
ischemic strokes, usually occurring in athero-
occur in succession without recovery are
sclerotic blood vessels.
called .
25. infarcts result from occlusion of
the smaller penetrating branches of large Activity B Consider the following figure.
cerebral arteries, commonly the middle cere-
bral and posterior cerebral arteries. Anterior

26. An stroke is caused by a moving


blood clot that travels from its origin to the
brain.
27. The most frequently fatal stroke is a
spontaneous into the brain.
28. The specific manifestations of stroke or TIA
are determined by the that is
affected, by the area of brain tissue that is
supplied by that vessel, and by the adequacy
of the collateral circulation.
29. Aneurysmal subarachnoid hemorrhage repre-
sents bleeding into the subarachnoid space
caused by a ruptured .
30. malformations are a complex
tangle of abnormal arteries and veins linked
by one or more fistulas.
31. represents a generalized
infection of the parenchyma of the brain or
spinal cord.
Posterior
32. occurs with or without nausea,
may be projectile, and is a common symptom In the figure above, identify the subdural
of increased intracranial pressure (ICP) and hematoma, the epidural hematoma, and the
brain stem compression. intracerebral hematoma.

Copyright 2011. Wolters Kluwer Health | Lippincott Williams & Wilkins. Study Guide for Porths Essentials of Pathophysiology, Third Edition.
LWBK707-c37_p218-225.qxd 8/19/10 2:22 PM Page 221 Aptara Inc

CHAPTER 37 DISORDERS OF BRAIN FUNCTION 221

Activity C Match the key terms in Column A Activity D


with their definitions in Column B.
1. Write the correct sequence in the boxes
Column A Column B provided below.
1. Vasogenic a. To attend to and
S S S S S S S
edema react to stimuli
coming from the
2. Hypoxia Put the pathologic process of bacterial menin-
contralateral side
gitis in order:
3. Tentorium b. Inability to
cerebelli comprehend, inte- 1. Release endotoxins
4. Hydrocephalus grate, and express 2. Development of a cloudy, purulent exudate
language in CSF
5. Aphasia
c. Small cells 3. Endotoxins initiate inflammatory response
6. Microneurons intimately involved 4. Meninges thicken and adhesions form
in local circuitry
7. Ischemia 5. Bacteria replicate and undergo lysis in CSF
d. Divides the cranial
8. Decorticate 6. Vascular congestion and infarction in the
cavity into anterior
posturing surrounding tissues
and posterior fossae
7. Pathogens, neutrophils, and albumin to
9. Hemineglect e. Reduced or
move across the capillary wall into the CSF
interrupted blood
10. Macroneurons 8. Adhesions may impinge on the cranial
flow
nerves or may impair the outflow of CSF
f. Occurs when
integrity of the
blood-brain barrier Glutamate
Glutamate
is disrupted
g. Deprivation of
oxygen with main-
tained blood flow
h. Results from
lesions of the cere-
bral hemisphere
i. Large cells with
long axons that
leave the local net-
work of intercom-
municating
neurons to send
action potentials
to other regions of
the nervous system
2. Complete the flowchart above using the
j. Abnormal increase following terms:
in CSF volume in
any part or all of Release of intracellular proteases, free
the ventricular radicals, and fragmentation of nuclei
system Calcium cascade
Opening calcium channels
NMDA receptor activation

Copyright 2011. Wolters Kluwer Health | Lippincott Williams & Wilkins. Study Guide for Porths Essentials of Pathophysiology, Third Edition.
LWBK707-c37_p218-225.qxd 8/19/10 2:22 PM Page 222 Aptara Inc

222 UNIT 10 NERVOUS SYSTEM

Activity E Briefly answer the following. 8. What is the ischemic penumbra of an ischemic
stroke and how does it affect the amount of
1. What does global ischemia refer to and irreversible damage?
what is the result?

9. Why do arteriovenous malformations predis-


2. Explain what watershed infarcts are and why pose a patient to stroke?
they occur.

10. What are some of the possible causes of a


3. What is the mechanism of toxicity of excito- seizure?
toxic amino acids?

4. What is postconcussion syndrome?


SECTION III: APPLYING
YOUR KNOWLEDGE
Activity F Consider the scenario and answer
the questions.
5. Compare the general manifestations of global
and focal brain injury. Case Study: A 78-year-old African American
woman is brought to the emergency department
by ambulance. She was found on the floor of her
bedroom by her daughter in a confused state,
and she could not move her left leg. A diagnosis
of stroke is suspected.
6. What are the two components of conscious-
a. When taking the nursing history, what risk
ness? What are the signs of altered conscious-
factors would the nurse assess for?
ness?

b. The diagnosis of ischemic stroke is confirmed.


7. How are pupillary reflexes used to evaluate
What orders would the nurse expect to receive
levels of brain function?
from the physician for acute ischemic stroke?

Copyright 2011. Wolters Kluwer Health | Lippincott Williams & Wilkins. Study Guide for Porths Essentials of Pathophysiology, Third Edition.
LWBK707-c37_p218-225.qxd 8/19/10 2:22 PM Page 223 Aptara Inc

CHAPTER 37 DISORDERS OF BRAIN FUNCTION 223

SECTION IV: PRACTICING 5. Excitotoxic c. Clouding of con-


sciousness, bilater-
FOR NCLEX 6. Hydrocephalus
ally small pupils
7. Cerebral edema (approximately
Activity G Answer the following questions.
2 mm in diameter)
1. Match the type of brain insult to its with a full range of
definition. constriction, and
motor responses to
Type Definition
pain that are pur-
1. Hypoxic a. Excessive activity of poseful or semipur-
the excitatory neu- poseful (localizing)
2. Ischemic
rotransmitters and and often
3. Excitotoxic their receptor-medi- asymmetric
ated effects. d. Depends on the
4. Increased
intercranial b. Displacement of brains compensa-
volume and brain tissue under tory mechanisms
pressure the falx cerebri or and the extent of
through the tentor- the swelling
5. Brain herniation ial notch or incisura e. Generalized depres-
6. Cerebral edema of the tentorium sant effect on the
cerebelli brain
7. Hydrocephalus
c. Interferes with de- f. Cerebral hemi-
livery of oxygen spheres become
and glucose as well enlarged, and the
as the removal of ventricular system
metabolic wastes beyond the point of
d. An abnormal in- obstruction is di-
crease in CSF vol- lated. The sulci on
ume in any part or the surface of the
all of the ventricu- brain become
lar system effaced and shal-
e. Swelling of the brain lowed, and the
white matter is re-
f. Increase in intercra-
duced in volume.
nial tissue causing
an increase in ICP g. Tissue perfusion be-
comes inadequate,
g. Decreased oxygen
cellular hypoxia re-
levels in all brain
sults, and neuronal
tissue
death may occur.
2. Match the type of brain insult to its effect on
the brain 3. There are several types of brain injuries that
can occur. What are the primary (or direct)
Type Effect on Brain
brain injuries? (Mark all that apply.)
1. Brain herniation a. Can be focal or a. Focal lesions of laceration
global with only
2. Hypoxic b. Contusion
one part of the
3. Ischemic brain being under c. Hypoxic
perfused or all of d. Diffuse axonal
4. Increased
the brain being e. Hemorrhage
intercranial
compromised
volume and
pressure b. Neuronal cell injury
and death

Copyright 2011. Wolters Kluwer Health | Lippincott Williams & Wilkins. Study Guide for Porths Essentials of Pathophysiology, Third Edition.
LWBK707-c37_p218-225.qxd 8/19/10 2:22 PM Page 224 Aptara Inc

224 UNIT 10 NERVOUS SYSTEM

4. Global and focal brain injuries manifest 8. The regulation of cerebral blood flow is
differently. What is almost always a manifes- accomplished through both autoregulation
tation of a global brain injury? and local regulation. This allows for the brain
a. Altered level of consciousness to meet its metabolic needs. What is the low
parameter for blood pressure before cerebral
b. Change in behavior
blood flow becomes severely compromised?
c. Respiratory instability
a. 30 mm Hg
d. Loss of eye movement reflexes
b. 40 mm Hg
5. You are the nurse caring for a 31-year-old c. 50 mm Hg
trauma victim is admitted to the neurologic
d. 60 mm Hg
intensive care unit. While doing your initial
assessment you find that the client is flexing 9. Intracranial aneurysms that rupture cause
the arms, wrists and fingers. There is subarachnoid hemorrhage in the client. How
abduction of the upper extremities with is the diagnosis of intracranial aneurysms and
internal rotation and plantar flexion of the subarachnoid hemorrhage made?
lower extremities. How would you describe a. Lumbar puncture
this in your nursing notes?
b. MRI
a. Decerebrate posturing
c. Loss of cranial nerve reflexes
b. Decorticate posturing
d. Venography
c. Extensor posturing
10. When the suspected diagnosis is bacterial
d. Diencephalon posturing
meningitis, what assessment techniques can
6. Brain death is the term that is used when the assist in determining of meningeal irritation
loss of function of the entire brain is is present?
irreversible. A clinical examination must be a. Kernig sign and Chadwick sign
done and repeated at least 6 hours later with
b. Brudzinski sign and Kernig sign
the same findings for brain death to be
declared. What is not assessed in the clinical c. Brudzinski sign and Chadwick sign
examination for brain death? d. Chvosteks sign and Guedel sign
a. Blink reflex 11. Manifestations of brain tumors are focal dis-
b. Responsiveness turbances in brain function and increased
c. Electrocardiogram ICP. What causes the focal disturbances mani-
fested by brain tumors?
d. Respiratory effort
a. Tumor infiltration and increased blood
7. Much like brain death, there are criteria for pressure
the diagnosis of a persistent vegetative state,
b. Brain compression and decreased ICP
and the criteria have to have lasted for more
than 1 month. What are criteria for the diag- c. Brain edema and disturbances in blood
nosis of persistent vegetative state? (Mark all flow
that apply.) d. Tumor infiltration and decreased ICP
a. Bowel and bladder incontinence
b. Ability to open the eyes
c. Lack of language comprehension
d. Lack of enough hypothalamic function to
maintain life
e. Variable preserved cranial nerve reflexes

Copyright 2011. Wolters Kluwer Health | Lippincott Williams & Wilkins. Study Guide for Porths Essentials of Pathophysiology, Third Edition.
LWBK707-c37_p218-225.qxd 8/19/10 2:22 PM Page 225 Aptara Inc

CHAPTER 37 DISORDERS OF BRAIN FUNCTION 225

12. Match the type of seizure with its definition. 13. For seizure disorders that do not respond to
anticonvulsant medications, the option for
Type of Seizure Definition
surgical treatment exists. What is removed in
1. Unprovoked a. Motion takes the the most common surgery for seizure
form of automa- disorders?
2. Complex partial
tisms such as lip a. Temporal neocortex
seizures
smacking, mild
b. Hippocampus
3. Generalized- clonic motion (usu-
onset ally in the eyelids), c. Entorhinal cortex
increased or de- d. Amygdala
4. Absence seizures
creased postural
14. Generalized convulsive status epilepticus is a
5. Atonic tone, and auto-
medical emergency caused by a tonic-clonic
nomic phenomena
6. Tonic-clonic seizure that does not spontaneously end, or
b. These seizures also recurs in succession without recovery. What
are known as drop is the first-line drug of choice to treat status
attacks epilepticus?
c. Most common a. Intravenous diazepam
major motor seizure
b. Intramuscular lorazepam
d. Clinical signs,
c. Intravenous cyclobenzaprine
symptoms, and sup-
porting electroen- d. Intramuscular cyproheptadine
cephalogram
changes indicate in-
volvement of both
hemispheres at
onset
e. Begins in a local-
ized area of the
brain but may
progress rapidly to
involve both hemi-
spheres
f. No identifiable
cause can be deter-
mined

Copyright 2011. Wolters Kluwer Health | Lippincott Williams & Wilkins. Study Guide for Porths Essentials of Pathophysiology, Third Edition.
LWBK707-c38_p226-236.qxd 8/19/10 1:30 PM Page 226 Aptara Inc

38
CHAPTER
Disorders of Special
Sensory Function:
Vision, Hearing, and
Vestibular Function

SECTION I: LEARNING 9. Describe the changes in lens structure that


occur with cataract.
OBJECTIVES
10. Cite risk factors and visual changes associated
1. Compare symptoms associated with red eye with cataract.
caused by conjunctivitis, corneal irritation,
11. Describe the treatment of persons with
and acute glaucoma.
cataracts.
2. Describe the appearance of corneal edema.
12. Relate the phagocytic function of the retinal
3. Characterize the manifestations, treatment, pigment epithelium to the development of
and possible complications of bacterial, Acan- retinitis pigmentosa.
thamoeba, and herpes keratitis.
13. Cite the manifestations and long-term visual
4. Describe tests used in assessing the pupillary effects of papilledema.
reflex and cite the possible causes of
14. Describe the pathogenesis of background and
abnormal pupillary reflexes.
proliferative diabetic retinopathies and their
5. Describe the formation and outflow of aque- mechanisms of visual impairment.
ous humor from the eye and relate to the
15. Relate the role of posterior vitreous
development of glaucoma.
detachment to the development of retinal
6. Compare open-angle and angle-closure tears and detachment.
glaucoma in terms of pathology, symptoma-
16. Explain the pathology and visual changes
tology, and diagnosis and treatment.
associated with macular degeneration.
7. Explain why glaucoma leads to blindness.
17. Characterize what is meant by a visual field
8. Describe changes in eye structure that occur defect.
with nearsighted and farsighted vision.
18. Explain the use of perimetry in the diagnosis
of a visual field defect.

226
LWBK707-c38_p226-236.qxd 8/19/10 1:30 PM Page 227 Aptara Inc

CHAPTER 38 DISORDERS OF SPECIAL SENSORY FUNCTION: VISION, HEARING, AND VESTIBULAR FUNCTION 227

19. Define the terms hemianopia, quadrantanopia, 37. Differentiate the structures of peripheral and
heteronymous hemianopia, and homonymous central vestibular function.
hemianopia and relate them to disorders of
38. Characterize the physiologic cause of motion
the optic pathways.
sickness.
20. Describe visual defects associated with disor-
39. Compare the manifestations and pathologic
ders of the visual cortex and visual
processes associated with benign positional
association areas.
vertigo and Mnire disease.
21. Describe the function and innervation of the
40. Differentiate the manifestations of peripheral
extraocular muscles.
and central vestibular disorders.
22. Explain the difference between paralytic and
nonparalytic strabismus.
23. Define amblyopia and explain its pathogenesis. SECTION II: ASSESSING
24. Explain the need for early diagnosis and
YOUR UNDERSTANDING
treatment of eye movement disorders in
Activity A Fill in the blanks.
children.
1. The optic globe, commonly called the
25. List the structures of the external, middle,
, is a remarkably mobile, nearly
and inner ear and cite their function.
spherical structure contained in a pyramid-
26. Describe two common disorders of the outer shaped cavity of the skull called the orbit.
ear.
2. The outer layer of the eyeball consists of a
27. Relate the functions of the eustachian tube to tough, opaque, white, fibrous layer called the
the development of middle ear problems, .
including acute otitis media and otitis media
3. Two striated muscles, the and
with effusion.
the , provide for movement
28. Describe anatomic variations as well as risk of the eyelids.
factors that make infants and young children
4. Symptoms of are a foreign body
more prone to develop acute otitis media.
sensation, a scratching or burning sensation,
29. List three common symptoms of acute otitis itching, and photophobia.
media.
5. conjunctivitis is a severe, sight-
30. Describe the disease process associated with threatening ocular infection.
otosclerosis and relate it to the progressive
6. The is avascular and obtains its
conductive hearing loss that occurs.
nutrient and oxygen supply by diffusion
31. Characterize tinnitus. from blood vessels of the adjacent sclera,
from the aqueous humor at its deep surface,
32. Differentiate between conductive,
and from tears.
sensorineural, and mixed hearing loss and
cite the more common causes of each. 7. refers to inflammation of the
cornea caused by infections, misuse of
33. Describe methods used in the diagnosis and
contact lenses, hypersensitivity reactions,
treatment of hearing loss.
ischemia, trauma, defects in tearing, and
34. Characterize the causes of hearing loss in interruption in sensory innervation, as occurs
infants and children and describe the need with local anesthesia.
for early diagnosis and treatment.
8. Herpes simplex virus with stro-
35. Explain the function of the vestibular system mal scarring is the most common cause of
with respect to postural reflexes and corneal ulceration and blindness in the West-
maintaining a stable visual field despite ern world.
marked changes in head position.
9. Herpes zoster usually presents
36. Relate the function of the vestibular system with malaise, fever, headache, and burning
to nystagmus and vertigo. and itching of the periorbital area.

Copyright 2011. Wolters Kluwer Health | Lippincott Williams & Wilkins. Study Guide for Porths Essentials of Pathophysiology, Third Edition.
LWBK707-c38_p226-236.qxd 8/19/10 1:30 PM Page 228 Aptara Inc

228 UNIT 10 NERVOUS SYSTEM

10. The is an adjustable diaphragm 23. The refers to the area that is
that permits changes in pupil size and in the visible during fixation of vision in one
light entering the eye. direction.
11. Inflammation of the entire uveal tract, which 24. Three pairs of extraocular musclesthe supe-
supports the lens and neural components of rior and , the medial, and
the eye, is called . , and the superior and inferior
control the movement of each
12. With diffuse damage to the forebrain involv-
eye.
ing the thalamus and hypothalamus, the
are typically small but respond 25. movements are those in which
to light. the optical axes of the two eyes are kept par-
allel, sharing the same visual field.
13. includes a group of conditions
that produce an elevation in intraocular pres- 26. refers to any abnormality of eye
sure. coordination or alignment that results in loss
of binocular vision.
14. In persons with glaucoma, temporary or per-
manent impairment of vision results from 27. describes a decrease in visual
changes in the retina and acuity resulting from abnormal visual devel-
optic nerve and from corneal edema and opment in infancy or early childhood.
opacification.
28. The external consists of the
15. glaucoma is caused by a auricle, which collects sound, and external
disorder in which the anterior chamber acoustic meatus or ear canal, which conducts
retains its fetal configuration, with aberrant the sound to the tympanic membrane.
trabecular meshwork extending to the root of
29. Impacted usually produces no
the iris, or is covered by a membrane.
symptoms unless it hardens and touches the
16. Nonuniform curvature of the refractive tympanic membrane, or the canal becomes
medium comparing the horizontal and verti- irritated resulting in symptoms of pain, itch-
cal planes is called . ing, and a sensation of fullness.
17. is neurologically associated 30. is an inflammation of the exter-
with convergence of the eyes, pupillary con- nal ear that can vary in severity from mild
striction, and results from thickening of the allergic dermatitis to severe cellulitis.
lens through contraction of the ciliary
31. The tympanic cavity is a small, mucosa-lined
muscle.
cavity within the petrous portion of the
18. A is a lens opacity that bone.
interferes with the transmission of light to
32. The tube, which connects the
the retina.
nasopharynx with the middle ear, is located
19. The function of the is to receive in a gap in the bone between the anterior
visual images, partially analyze them, and and medial walls of the middle ear.
transmit this modified information to the
33. The eustachian tube does not
brain.
close or does not close completely.
20. The genetically person has
34. refers to inflammation of the
never experienced the full range of normal
middle ear without reference to etiology or
color vision and is unaware of what he or she
pathogenesis.
is missing.
35. is characterized by acute onset
21. represents a group of hereditary
of otalgia (or pulling of the ears in an infant),
diseases that cause slow degenerative changes
fever, and hearing loss.
in the retinal photoreceptors.
36. refers to the formation of new
22. degeneration is characterized
spongy bone around the stapes and oval win-
by degenerative changes in the central
dow, which results in progressive deafness.
portion of the retina that results primarily in
loss of central vision.

Copyright 2011. Wolters Kluwer Health | Lippincott Williams & Wilkins. Study Guide for Porths Essentials of Pathophysiology, Third Edition.
LWBK707-c38_p226-236.qxd 8/19/10 1:30 PM Page 229 Aptara Inc

CHAPTER 38 DISORDERS OF SPECIAL SENSORY FUNCTION: VISION, HEARING, AND VESTIBULAR FUNCTION 229

37. The spiral canal of the , which is Activity B Consider the following figures.
shaped like a snail shell, begins at the
vestibule and winds around a central core of
spongy bone called the modiolus.
38. hearing loss occurs with disor-
ders that affect the inner ear, auditory nerve,
or auditory pathways of the brain.
39. Deafness or some degree of hearing
impairment is the most common serious
complication of in infants and
children.
40. Acoustic neuromas are benign Schwann cell
tumors affecting .
41. The most common infectious cause of
congenital sensorineural hearing loss is
.
42. The system maintains and
assists recovery of stable body and head posi- 1. In the figure above, locate and label the
tion through control of postural reflexes, and following structures:
it maintains a stable visual field despite
marked changes in head position. Conjunctiva
Cornea
43. Disorders of vestibular function are character-
ized by a condition called , in Lens
which an illusion of motion occurs. Iris
44. is a form of normal physiologic Meibomian gland
vertigo, caused by repeated rhythmic stimula- Orbicularis oculi muscle
tion of the vestibular system, and such as is Inferior oblique muscle
encountered in car, air, or boat travel. Inferior rectus
45. Benign vertigo is the most com- Superior rectus
mon cause of pathologic vertigo. Levator palpebrae superioris
46. Acute is characterized by an Choroid
acute onset (usually hours) of vertigo, nausea, Retina
and vomiting lasting several days and not
associated with auditory or other neurologic Superior tarsal plate
manifestations. Ciliary body
47. disease is a disorder of the Sclera
inner ear due to distention of the Optic nerve
endolymphatic compartment of the inner
ear, causing a triad of hearing loss, vertigo,
and tinnitus.
48. Abnormal nystagmus and vertigo can occur
as a result of CNS lesions involving the
and lower brain stem.

Copyright 2011. Wolters Kluwer Health | Lippincott Williams & Wilkins. Study Guide for Porths Essentials of Pathophysiology, Third Edition.
LWBK707-c38_p226-236.qxd 8/19/10 1:30 PM Page 230 Aptara Inc

230 UNIT 10 NERVOUS SYSTEM

B
A

2. In the figure above, which eye represents


myopia? Which eye represents hyperopia?
Which eye represents normal focal length?

Middle Inner
Cochlear
ear ear
portion
Vestibular
portion

Pharynx

3. In the figure above, locate and label the Malleus


following structures: Stapes
Auricle Eustachian tube
External acoustic meatus Cochlea

Copyright 2011. Wolters Kluwer Health | Lippincott Williams & Wilkins. Study Guide for Porths Essentials of Pathophysiology, Third Edition.
LWBK707-c38_p226-236.qxd 8/19/10 1:30 PM Page 231 Aptara Inc

CHAPTER 38 DISORDERS OF SPECIAL SENSORY FUNCTION: VISION, HEARING, AND VESTIBULAR FUNCTION 231

Cranial nerve VIII 2.


Tympanic membrane Column A Column B
Incus
1. Anopia a. The vitreous
Semicircular canals shrinks and partly
2. Hyperopia
separates from the
Activity C Match the key terms in Column A 3. Cycloplegia retinal surface
with their definitions in Column B. b. A hole in the
4. Scotoma
1. visual field
5. Rhegmatogenous
c. Anterior-posterior
Column A Column B detachment
dimension of the
1. Arcus senilis a. Caused by 6. Tonometry eyeball is too short
infection of the d. Anterior-posterior
2. Entropion 7. Presbyopia
sebaceous glands dimension of the
3. Ophthalmia b. An infection of the 8. Myopia eyeball is too long
neonatorum lacrimal sac 9. Direct e. Measurement of
4. Hordeolum c. Chronic inflamma- pupillary intraocular
tory granuloma of light reflex pressure
5. Pink eye
a meibomian gland f. Blindness in one
10. Papilledema
6. Chalazion d. Drooping of the eye
7. Ptosis eyelid g. Paralysis of the cil-
e. Extracellular lipid iary muscle, with
8. Dacryocystitis
infiltration of the loss of accommo-
9. Ectropion cornea dation
10. Sjgren f. Turning in of the h. Leakage of fluid
syndrome lid margin results in edema of
g. Conjunctivitis that the optic papilla
occurs in newborns i. Rapid constriction
and is related to of the pupil
sexually transmit- exposed to light
ted diseases j. Decrease in
h. Diminished accommodation
salivary and that occurs
lacrimal secretions, because of aging
resulting in kerato-
conjunctivitis sicca
and xerostomia
i. Eversion of the
lower lid margin
j. Inflammation of
the conjunctiva

Copyright 2011. Wolters Kluwer Health | Lippincott Williams & Wilkins. Study Guide for Porths Essentials of Pathophysiology, Third Edition.
LWBK707-c38_p226-236.qxd 8/19/10 1:30 PM Page 232 Aptara Inc

232 UNIT 10 NERVOUS SYSTEM

3. Activity D Briefly answer the following.


Column A Column B 1. Where are tears formed and what purpose(s)
1. Otitis media a. Degenerative hear-
do they serve?
with effusion ing loss that occurs
with advancing age
2. Cerumen
b. Cystlike lesions of
3. Tinnitus the middle ear
4. Streptococcus c. Most common 2. What is the most common cause of chronic
pneumoniae cause of bacterial bacterial conjunctivitis and what are the
meningitis that symptoms?
5. Electrony-
results in
stagmography
sensorineural hear-
6. Presbycusis ing loss after the
neonatal period
7. Nystagmus
d. Injury resulting
8. Barotrauma from the inability 3. How do the different levels of abrasion
9. Cholesteatomas to equalize middle trauma (less severe to more severe) affect the
ear with ambient cornea and how fast to the abrasions heal?
10. Frequency pressures
e. Ringing of the ears;
it may also assume
a hissing, roaring,
buzzing, or
4. What is the mechanism of a primary herpes
humming sound
simplex virus optical epithelial infection?
f. Number of waves
per unit time
g. Involuntary
rhythmic and
oscillatory eye
movements that 5. What is the cause of acanthamoeba keratitis
preserve eye and what are the primary symptoms?
fixation on stable
objects in the
visual field during
angular and rota-
tional movements
of the head 6. Explain how the pupil is able to change
h. Earwax shape.
i. Examination that
records eye move-
ments in response
to vestibular, visual,
cervical, rotational,
7. What is glaucoma? What is primary and
and positional
stimulation. secondary glaucoma?
j. Presence of fluid
in the middle ear
without signs and
symptoms of acute
ear infection

Copyright 2011. Wolters Kluwer Health | Lippincott Williams & Wilkins. Study Guide for Porths Essentials of Pathophysiology, Third Edition.
LWBK707-c38_p226-236.qxd 8/19/10 1:30 PM Page 233 Aptara Inc

CHAPTER 38 DISORDERS OF SPECIAL SENSORY FUNCTION: VISION, HEARING, AND VESTIBULAR FUNCTION 233

8. What is presbyopia and how does it affect 15. What are the purported causes of subjective
vision? tinnitus?

9. Retinal hemorrhage can occur at many layers. 16. What is the cause of hearing loss in conduc-
What are the types of retinal bleeding and tive hearing loss?
where do they occur?

17. How does the vestibular system inform the


10. Why is proliferative diabetic retinopathy a brain about head and body position?
major concern for all diabetic patients?

18. What is the test used to determine vestibular


11. What is the relationship between hypertension function in an unconscious patient?
and the development of a retinopathy?

12. What is/are the functions of the eustachian


tube? SECTION III: APPLYING
YOUR KNOWLEDGE
Activity E Consider the scenario and answer
the questions.

13. What are the complications associated with 1. Case study: The mother of an 18-month-old
otitis media? girl brings her daughter to the clinic for a
well-baby check. During the physical
examination, the physician notices that the
client has a white reflex in her left eye. He sus-
pects retinoblastoma.
a. What diagnostic measures would the nurse
14. How does otosclerosis lead to progressive expect the doctor to order?
deafness?

Copyright 2011. Wolters Kluwer Health | Lippincott Williams & Wilkins. Study Guide for Porths Essentials of Pathophysiology, Third Edition.
LWBK707-c38_p226-236.qxd 8/19/10 1:30 PM Page 234 Aptara Inc

234 UNIT 10 NERVOUS SYSTEM

b. Retinoblastoma is confirmed and a 3. Keratitis can be caused by different infectious


treatment plan is being made. What are the agents. What is the treatment goal with her-
treatment options for retinoblastoma? pes simplex virus keratitis?
a. Minimizing pain
b. Cure for the disease
c. Eliminating viral replication within the
cornea
2. Case study: You are the nurse preparing an d. Minimizing spread of virus to other parts
educational event for the local junior league, of the eye
which has asked you to speak on hearing loss
4. Corneal transplants are done everyday in
and deafness. One of the subjects that you
hospitals around the world. All of these trans-
will address is ototoxicity.
planted corneas come from cadavers. Why do
a. What drugs would you include when talk- corneal transplants have such a low rejection
ing about ototoxicity? rate? (Mark all that apply.)
a. Cornea is very vascular
b. Antigen-presenting cells are not present in
great numbers
c. The cornea secretes immunosuppressive
factors
d. The cornea has no lymphatics
SECTION IV: PRACTICING e. Corneal cells secrete substances that pro-
FOR NCLEX tect against keratitis
5. Pharmacologic agents can affect dilation
Activity F Answer the following questions.
of the pupil and the papillary response.
1. Dacryocystitis is an infection in the lacrimal What types of drugs produce papillary
sac. What symptoms indicate dacryocystitis? constriction?
a. Purulent discharge a. Sympathomimetic agents
b. Swelling b. Antihistamine agents
c. Inflamed conjunctiva c. Cycloplegic agents
d. Lack of tears d. Miotic agents
2. Ophthalmia neonatorum is a conjunctivitis 6. In open-angle glaucoma, there is an increased
that develops in newborns. It is caused by pressure within the globe of the eye without
the agents that cause sexually transmitted obstruction at the iridocorneal angle. Usually,
diseases. When should ophthalmia neonato- this is caused by an abnormality in the
rum be suspected? trabecular meshwork, which controls the
a. When a conjunctivitis develops 24 hours flow or aqueous humor. Where is aqueous
after birth humor in a normal eye?
b. When a conjunctivitis develops 12 hours a. Canal of Schlemm
after birth b. Ocular canal
c. When a conjunctivitis develops 48 hours c. Ductus lacrimalis
after birth d. Behind the pupil
d. When a conjunctivitis develops 36 hours
after birth

Copyright 2011. Wolters Kluwer Health | Lippincott Williams & Wilkins. Study Guide for Porths Essentials of Pathophysiology, Third Edition.
LWBK707-c38_p226-236.qxd 8/19/10 1:30 PM Page 235 Aptara Inc

CHAPTER 38 DISORDERS OF SPECIAL SENSORY FUNCTION: VISION, HEARING, AND VESTIBULAR FUNCTION 235

7. Match the terms with their definitions. 11. Age-related macular degeneration that is dry
is characterized by what?
Term Definition
a. Atrophy of the Bruch membrane
1. Presbyopia a. The anterior-poste-
b. Leakage of serous or hemorrhagic fluid
rior dimension of
2. Cycloplegia c. New blood vessels in the eye
the eyeball is too
3. Myopia long; the focus point d. Formation of a choroidal neovascular
for an infinitely dis- membrane
4. Hyperopia
tant target is anterior
12. Cortical blindness is the bilateral loss of the
5. Astigmatism to the retina
primary visual cortex. What is retained in
b. Paralysis of the cil- cortical blindness?
iary muscle, with
a. Red spots seen behind the eyelids
loss of accommoda-
tion b. Pupillary reflexes
c. The anterior-poste- c. Ptosis
rior dimension of d. Myopia
the eyeball is too
13. Adult strabismus is almost always of the paraly-
short; the image is
tic variety. What is a cause of adult strabismus?
theoretically focused
posterior to (behind) a. Huntington disease
the retina b. Parkinson disease
d. Range of focus or c. Graves disease
accommodation is d. Addison disease
diminished
14. Amblyopia, or lazy eye, occurs at a time
e. An asymmetric
when visual deprivation or abnormal binocu-
bowing of the
lar interactions occur in visual infancy.
cornea
Whether or not amblyopia is reversible
8. Age-related cataracts are characterized by what? depends on what?
a. Everything looking grey a. The child has to be older than 5
b. Visual distortion b. The maturity of the visual system at time
c. Narrowing visual field of onset
d. Blind spots in visual field c. The child has to have bilateral congenital
cataracts
9. Vitreous humor occupies the posterior
d. The child has to be able to wear contact
portion of the eyeball. It is an amorphous
lenses
biologic gel. When liquefaction of the gel
occurs, as in aging, what can be seen during 15. Otitis externa is an inflammation of the outer
head movement? ear. What fungi cause otitis externa?
a. Blind spots a. Aspergillus
b. Meshlike structures b. Pseudomonas aeruginosa
c. Floaters c. Staphylococcus aureus
d. Red spots d. Escherichia coli
10. When conditions occur that impair retinal 16. The eustachian tube connects the nasophar-
blood flow, such as hyperviscosity of the ynx and the middle ear. In infants and
blood or a sickle cell crisis, what can occur in children with abnormally patent tubes, what
the eye? are let into the eustachian tube when the
a. Microaneurysms infant or child cries or blows the nose?
b. Hypertensive retinopathy a. Air and cerumen
c. Microinfarcts b. Air and secretions
d. Neovascularization c. Secretions and saliva
d. Cerumen and saliva

Copyright 2011. Wolters Kluwer Health | Lippincott Williams & Wilkins. Study Guide for Porths Essentials of Pathophysiology, Third Edition.
LWBK707-c38_p226-236.qxd 8/19/10 1:30 PM Page 236 Aptara Inc

236 UNIT 10 NERVOUS SYSTEM

17. Acute otitis media is the disorder in children 22. Tumors affecting cranial nerve VIII are
for which antibiotics are most prescribed. acoustic neuromas. What are these tumors of?
What are the risk factors for acute otitis a. Inner ear
media? (Mark all that apply.)
b. Organ of Corti
a. Ethnicity
c. Schwann cells
b. Premature birth
d. Labyrinth
c. Only child in household
23. It is important to differentiate between the
d. Genetic syndromes
kinds of hearing loss so they can be appropri-
e. Female gender ately treated. What is used to test between
18. Otosclerosis is a condition in which spongy, conductive and sensorineural hearing loss?
pathologic bone grows around the stapes and a. Audioscope
oval window. It can be treated either b. Audiometer
medically or surgically. What is the surgical
c. Tone analysis
treatment for otosclerosis?
d. Tuning fork
a. Otosclerotomy
b. Ovalectomy 24. Hearing loss in children can be either
conductive or sensorineural, as it is in adults.
c. Stapedectomy
What is the major cause of sensorineural
d. Amplification surgery hearing loss in children?
19. What separates the scala vestibule and the a. Genetic causes
scala media? b. Acute otitis media
a. Corti membrane c. Paget disease
b. Tympani membrane d. Ototoxicity
c. Modiolus membrane
25. Presbycusis is degenerative hearing loss asso-
d. Reissner membrane ciated with aging. What is the first symptom
20. Objective tinnitus is tinnitus that someone of this disorder?
else can hear. What does the tinnitus that is a. Inability to localize sounds
caused by vascular disorders sound like? b. Reduction in ability to understand speech
a. Pulses c. Inability to detect sound
b. Rings d. Reduction in ability to identify sounds
c. Hums
d. Roars
21. Conductive hearing loss can occur for a vari-
ety of reasons, including foreign bodies in the
ear canal, damage to the ear drum, or disease.
What disease is associated with conductive
hearing loss?
a. Huntington disease
b. Paget disease
c. Alzheimer disease
d. Parkinson disease

Copyright 2011. Wolters Kluwer Health | Lippincott Williams & Wilkins. Study Guide for Porths Essentials of Pathophysiology, Third Edition.
LWBK707-c39_p237-241.qxd 8/19/10 1:30 PM Page 237 Aptara Inc

39

CHAPTER
Disorders of the Male
Genitourinary System

SECTION I: LEARNING 14. Compare the pathology and symptoms of


acute bacterial prostatitis, chronic bacterial
OBJECTIVES prostatitis, and chronic prostatitis/pelvic pain
syndrome.
1. State the difference between hypospadias and
epispadias. 15. Describe the urologic manifestations and
treatment of benign prostatic hyperplasia.
2. Cite the significance of phimosis.
16. List the methods used in the diagnosis and
3. Describe the anatomic changes that occur
treatment of prostatic cancer.
with Peyronie disease.
4. Explain the physiology of penile erection and
relate it to erectile dysfunction and priapism.
SECTION II: ASSESSING
5. Describe the appearance of balanitis xerotica YOUR UNDERSTANDING
obliterans.
6. List the signs of penile cancer. Activity A Fill in the blanks.

7. State the physical manifestations and poten- 1. and are congeni-


tial risks associated with uncorrected tal disorders of the penis resulting from
cryptorchidism. embryologic defects in the development of
the urethral groove and penile urethra.
8. Compare the cause, appearance, and signifi-
cance of hydrocele, hematocele, 2. involves a localized and
spermatocele, and varicocele. progressive fibrosis of unknown origin that
affects the tunica albuginea.
9. State the difference between extravaginal and
intravaginal testicular torsion. 3. The manifestations of disease
include painful erection, bent erection, and
10. Describe the symptoms of epididymitis. the presence of a hard mass at the site of
11. State the manifestations and possible compli- fibrosis.
cations of mumps orchitis. 4. Erection is under the control of
12. Relate environmental factors to development nervous system, and ejaculation and
of scrotal cancer. detumescence (penile relaxation) are under
the control of the nervous
13. State the cell types involved in seminoma,
system.
embryonal carcinoma, teratoma, and chorio-
carcinoma tumors of the testes.

237
LWBK707-c39_p237-241.qxd 8/19/10 1:30 PM Page 238 Aptara Inc

238 UNIT 11 GENITOURINARY AND REPRODUCTIVE FUNCTION

5. Parasympathetic stimulation results in release 17. The cause of noninflammatory prostatitis is


of , which causes relaxation of unknown, but because of the absence of
trabecular smooth muscle of the corpora cav- inflammation, the search for the cause of
ernosa, permitting inflow of blood into the symptoms has been directed toward
sinuses of the cavernosa at pressures sources.
approaching those of the
18. The level of correlates with the
system.
volume and stage of prostate cancer.
6. is commonly classified as
psychogenic, organic, or mixed psychogenic Activity B Consider the following figure.
and organic.
7. Erectile dysfunction is now recognized as a A
marker for disease, and is now
considered a component of the
syndrome.
8. is caused by impaired blood
flow in the corpora cavernosa of the penis.
9. Several risk factors for have
been suggested, including increasing age,
poor hygiene, smoking, human
papillomavirus infections, ultraviolet
radiation exposure, and immunodeficiency
states. B

10. The consequences of include


infertility, malignancy, testicular torsion, and
the possible psychological effects of an empty
scrotum.
11. Sperm concentration and are
decreased in men with varicocele.
12. is an inflammation of the
epididymis, the elongated cordlike structure
that lies along the posterior border of the In the figure above, locate and label the
testis, whose function is the storage, structures responsible for penile erection:
transport, and maturation of spermatozoa. Deep dorsal vein
13. refers to a variety of inflamma- Tunica albuginea
tory disorders of the prostate gland, some Corpora cavernosa
bacterial and some not. Cavernous artery
14. The manifestations of include Sinusoidal spaces
fever and chills, malaise, myalgia, arthralgia, Circumflex vein
frequent and urgent urination, dysuria, and
urethral discharge. Circumflex artery
Cavernous nerve
15. As with other cancers, it appears that the
development of cancer is a Dorsal nerve
multistep process involving genes that Dorsal artery
control cell differentiation and growth. Subtunical venular plexus
16. Men with typically have recur-
rent urinary tract infections with persistence
of the same strain of pathogenic bacteria in
prostatic fluid and urine.

Copyright 2011. Wolters Kluwer Health | Lippincott Williams & Wilkins. Study Guide for Porths Essentials of Pathophysiology, Third Edition.
LWBK707-c39_p237-241.qxd 8/19/10 1:30 PM Page 239 Aptara Inc

CHAPTER 39 DISORDERS OF THE MALE GENITOURINARY SYSTEM 239

Activity C Match the key terms in Column A 3. What is testicular torsion? What are the differ-
with their definitions in Column B. ent types?

Column A Column B
1. Balanitis a. Inflammation of
the glans penis
2. Smegma
b. Undescended
3. Epispadias testes 4. How are testicular cancers staged?

4. Hydrocele c. Accumulation
under the
5. Cryptorchidism
phimotic foreskin
6. Orchitis d. Excess fluid
7. Prostatitis collects between 5. How does benign prostatic hyperplasia (BPH)
the layers of the cause obstruction of the urethra?
8. Benign prostatic tunica vaginalis
hyperplasia
e. Infection of the
9. Phimosis testes
10. Priapism f. Inflammation of
the prostate
6. How is prostate cancer diagnosed?
g. Tightening of the
penile foreskin
h. Opening of the
urethra is on the
dorsal surface of
the penis
i. Involuntary,
prolonged, abnor- SECTION III: APPLYING
mal and painful YOUR KNOWLEDGE
erection
j. An age-related, Activity E Consider the scenario and answer
nonmalignant the questions.
enlargement of the A 50-year-old man presents at the clinic
prostate gland complaining of a lump on his penis that has pro-
gressed over the past 4 months until he can no
Activity D Briefly answer the following. longer retract his foreskin over his glans. He
states the condition is now painful. He is having
1. What are some of the known causes of erectile
difficulty urinating, and there is a discharge com-
dysfunction?
ing from under his foreskin. He is scheduled for
surgery the following day to relieve the phimosis
and biopsy the lump. The physician explains the
surgery to the client and states that, if the lump
is malignant, a partial or total penectomy may be
necessary.
2. How do drugs like Viagra treat erectile
dysfunction? 1. Before the client leaves, he asks the nurse
what causes penile cancer. The nurse correctly
responds:

Copyright 2011. Wolters Kluwer Health | Lippincott Williams & Wilkins. Study Guide for Porths Essentials of Pathophysiology, Third Edition.
LWBK707-c39_p237-241.qxd 8/19/10 1:30 PM Page 240 Aptara Inc

240 UNIT 11 GENITOURINARY AND REPRODUCTIVE FUNCTION

2. The client is admitted to your unit after 4. Cryptorchidism, left untreated, is a high risk
undergoing a total penectomy for penile can- for testicular cancer and infertility. What are
cer with inguinal lymph node involvement. the treatment goals for boys with
While you are caring for him, the client asks cryptorchidism?
what his prognosis is. What would be your a. Prevention of testicular cancer
correct response to the client?
b. Prevention of an associated inguinal her-
nia
c. Easier cancer detection
d. Decreased fertility
5. The mother of a 5-year-old boy brings him
into the clinic because there is a firm feeling
swelling around one of his testes. What
SECTION IV: PRACTICING would the suspected diagnosis be?
FOR NCLEX a. Peyronie disease
Activity F Answer the following questions. b. Cryptorchism
c. Priapism
1. In hypospadias, the treatment of choice is
surgery to repair the defect. What influences d. Hydrocele
the timing of the surgical repair? (Mark all 6. In the neonatal and pediatric population,
that apply.) there can be many physiologic problems with
a. Penile size the male genitourinary system. What is the
b. Testicular involvement most common acute scrotal disorder in the
pediatric population?
c. Psychological effects on the child
a. Testicular torsion
d. Presence of an abdominal hernia
b. Hypospadias
e. Anesthetic risk
c. Balanitis
2. A 75-year-old man presents at the clinic com-
d. Paraphimosis
plaining of pain during intercourse and an
upward bowing of his penis during erection. 7. Epididymitis can be sexually transmitted, or
The clients history mentions an it can be caused by a variety of other reasons,
inflammation of the penis that was treated including abnormalities in the genitourinary
3 months ago. The physicians physical tract. What are the most common causes of
examination of the client notes beads of scar epididymitis in young men without underly-
tissue along the dorsal midline of the penile ing genitourinary disease?
shaft. What would be the suspected diagnosis a. Chlamydia trachomatis and Candida albi-
of this client? cans
a. Peyronie disease b. Chlamydia trachomatis and Neisseria gonor-
b. Cavernosa disease rhoeae
c. Balanitic disease c. Escherichia coli and Neisseria gonorrhoeae
d. Paraphimosis disease d. Candida albicans and Escherichia coli
3. Priapism (a prolonged painful erection not 8. Testicular cancer is highly curable if found
associated with sexual excitement) can occur and treated early in the disease. What are
at any age. In boys, ages 5 to 10, what are the signs of metastatic spread of testicular
most common causes of priapism? cancer? (Mark all that apply.)
a. Neoplasms or hemophilia a. Hemoptysis
b. Sickle cell disease or neoplasms b. Back pain
c. Hemophilia or sickle cell disease c. Neck mass
d. Hypospadias or neoplasms d. Chest mass
e. Hoarse voice

Copyright 2011. Wolters Kluwer Health | Lippincott Williams & Wilkins. Study Guide for Porths Essentials of Pathophysiology, Third Edition.
LWBK707-c39_p237-241.qxd 8/19/10 1:30 PM Page 241 Aptara Inc

CHAPTER 39 DISORDERS OF THE MALE GENITOURINARY SYSTEM 241

9. A 40-year-old man presents at the clinic com- 10. While the cause of BPH is unknown, we do
plaining of painful urination and rectal pain. know that the incidence of BPH increases with
His vital signs are temperature, 101.7F; blood age. What ethnic group is BPH highest in?
pressure, 105/74; pulse, 98; respiration, 22. a. Japanese
While taking a history, the nurse notes the
b. White
client has had chills, malaise, and myalgia.
What would the nurse suspect as a diagnosis? c. Native American
a. Benign prostatic hyperplasia d. African American
b. Epididymitis
c. Acute bacterial prostatitis
d. Orchitis

Copyright 2011. Wolters Kluwer Health | Lippincott Williams & Wilkins. Study Guide for Porths Essentials of Pathophysiology, Third Edition.
LWBK707-c40_p242-248.qxd 8/19/10 1:31 PM Page 242 Aptara Inc

40
CHAPTER
Disorders of the Female
Genitourinary System

SECTION I: LEARNING 10. Cite the major early symptom of endometrial


cancer and describe the relationship between
OBJECTIVES unopposed estrogen stimulation of the
endometrium and development of endome-
1. Compare the abnormalities associated with
trial cancer.
Bartholin cyst, nonneoplastic epithelial
disorders, vulvodynia, and cancer of the 11. Compare the location and manifestations of
vulva. intramural and subserosal leiomyomas.
2. State the role of Dderlein bacilli in 12. List the common causes and symptoms of
maintaining the normal ecology of the pelvic inflammatory disease.
vagina.
13. Describe the risk factors and symptoms of
3. Describe the conditions that predispose to ectopic pregnancy.
vaginal infections and the methods used to
14. State the underlying cause of ovarian cysts.
prevent and treat these infections.
15. Differentiate benign ovarian cyst from poly-
4. Describe the importance of the cervical trans-
cystic ovary syndrome (PCOS).
formation zone in the pathogenesis of cervi-
cal cancer. 16. List the hormones produced by the three
types of functioning ovarian tumors.
5. Compare the lesions associated with naboth-
ian cysts and cervical polyps. 17. State the reason that ovarian cancer may be
difficult to detect in an early stage.
6. List the complications of untreated cervicitis.
18. Characterize the function of the supporting
7. Describe the development of cervical cancer
ligaments and pelvic floor muscles in
from the appearance of atypical cells to the
maintaining the position of the pelvic
development of invasive cervical cancer and
organs, including the uterus, bladder, and
relate to the importance of the Pap smear in
rectum.
early detection of cervical cancer.
19. Describe the manifestations of cystocele, rec-
8. Cite the rationale for describing cervical can-
tocele, and enterocele.
cer as a sexually transmitted infection and
the rationale for use of the human papilloma 20. Describe the cause and manifestations of
virus vaccine in prevention of cervical cancer. uterine prolapse.
9. Compare the pathology and manifestations
of endometriosis and adenomyosis.

242
LWBK707-c40_p242-248.qxd 8/19/10 1:31 PM Page 243 Aptara Inc

CHAPTER 40 DISORDERS OF THE FEMALE GENITOURINARY SYSTEM 243

21. Define the terms amenorrhea, hypomenorrhea, 6. represents an inflammation of


oligomenorrhea, menorrhagia, metrorrhagia, and the vagina that is characterized by vaginal
menometrorrhagia. discharge and burning, itching, redness, and
swelling of vaginal tissues.
22. Relate the alteration in estrogen and proges-
terone levels to the development of dysfunc- 7. The most common symptom of vaginal carci-
tional menstrual cycles. noma is abnormal .
23. Differentiate between primary dysmenorrhea 8. During , the newly developed
and secondary dysmenorrhea. squamous epithelial cells of the cervix are
vulnerable to development of dysplasia and
24. Characterize the manifestations of the
genetic change if exposed to cancer-
premenstrual syndrome, its possible causes,
producing agents.
and the methods of treatment.
9. cancer is readily detected and,
25. Describe changes in breast function that
if detected early, is the most easily cured of
occur with mastitis.
all the cancers of the female reproductive
26. Describe the manifestations of nonprolifera- system.
tive (fibrocystic) breast changes.
10. Untreated may extend to
27. Cite the risk factors for breast cancer, the include the development of pelvic cellulitis,
importance of clinical breast examination, low back pain, dyspareunia, cervical stenosis,
and recommendations for mammography. dysmenorrhea, and ascending infection of
the uterus or fallopian tubes.
28. Describe the methods used in the diagnosis
and treatment of breast cancer. 11. are the most common lesions
of the cervix.
29. Provide a definition of infertility.
12. A preponderance of evidence suggests a
30. List male and female factors that contribute
causal link between infection
to infertility.
and cervical cancer.
31. Briefly describe methods used in the
13. is the condition in which func-
treatment of infertility.
tional endometrial tissue is found in ectopic
sites outside the uterus.
14. is the condition in which
SECTION II: ASSESSING endometrial glands and stroma are found
YOUR UNDERSTANDING within the myometrium, interspersed
between the smooth muscle fibers.
Activity A Fill in the blanks.
15. and , which con-
1. Diseases of the external genitalia are similar sists of dilating the cervix and scraping the
to those that affect skin uterine cavity, is the definitive procedure for
elsewhere in the body. diagnosis of endometrial cancer because it
2. The is particularly prone to skin provides a more thorough evaluation.
infections because it is constantly being 16. Uterine are benign neoplasms
exposed to secretions and moisture. of smooth muscle origin.
3. A is a fluid-filled sac that results 17. is a polymicrobial infection of
from occlusion of the duct system in the upper reproductive tract associated with
Bartholin gland. the sexually transmitted organisms as well as
4. presents as thickened, gray- endogenous organisms.
white plaques with an irregular surface. 18. occurs when a fertilized ovum
5. The normal vaginal depends on implants outside the uterine cavity, the most
the delicate balance of hormones and bacter- common site being the fallopian tube.
ial flora.

Copyright 2011. Wolters Kluwer Health | Lippincott Williams & Wilkins. Study Guide for Porths Essentials of Pathophysiology, Third Edition.
LWBK707-c40_p242-248.qxd 8/19/10 1:31 PM Page 244 Aptara Inc

244 UNIT 11 GENITOURINARY AND REPRODUCTIVE FUNCTION

19. Disorders of the ovaries frequently cause 34. is the inability to conceive a
and problems. child after 1 year of unprotected intercourse.
20. syndrome is characterized by
Activity B Consider the following figures.
varying degrees of menstrual irregularity,
signs of hyperandrogenism, and infertility.
21. Most women with PCOS have elevated
levels with normal estrogen
and follicle-stimulating hormone production.
22. tumors are common; most are
benign, but malignant tumors are the leading
cause of death from reproductive cancers.
23. The most significant risk factor for ovarian
cancer appears to be the length
of time during a womans life when her ovar-
ian cycle is not suppressed by pregnancy,
lactation, or oral contraceptive use.
24. The breast cancer susceptibility genes,
BRACA1 and BRCA2, which are
genes are incriminated in approximately 10%
of hereditary ovarian cancers despite being
identified as breast cancer genes.
25. is the herniation of the rectum
into the vagina.
26. Uterine prolapse is the bulging of the uterus In the figure above, locate and label all the com-
into the vagina that occurs when the mon locations of endometriosis within the pelvis
ligaments are stretched. and abdomen.
27. Removal of the uterus through the vagina Activity C Match the key terms in Column A
with appropriate repair of the vaginal wall with their definitions in Column B.
often is done when is accompa-
nied by cystocele or rectocele. 1.

28. Primary is the failure to Column A Column B


menstruate by 15 years of age or by 13 years 1. Papanicolaou a. Insertion of
of age if failure to menstruate is accompanied smear radioactive materi-
by absence of secondary sex characteristics. als into the body
2. Menorrhagia
29. is the secretion of breast milk b. Commonly called
in a nonlactating breast. 3. Vulvodynia fibroids
30. is inflammation of the breast. 4. Leiomyomas c. Unexplained vul-
var pain
31. are firm, rubbery, sharply 5. Cystocele
defined round masses in breast tissue. d. Herniation of the
6. Curettage bladder into the
32. changes usually present as 7. Atrophic vagina
nodular granular breast masses that are more vaginitis e. Inflammation of
prominent and painful during the luteal or the vagina that
progesterone-dominant portion of the men- 8. Lichen
occurs after
strual cycle. sclerosus
menopause
33. disease presents as an 9. Brachytherapy
eczematoid lesion of the nipple and areola. 10. Cervicitis

Copyright 2011. Wolters Kluwer Health | Lippincott Williams & Wilkins. Study Guide for Porths Essentials of Pathophysiology, Third Edition.
LWBK707-c40_p242-248.qxd 8/19/10 1:31 PM Page 245 Aptara Inc

CHAPTER 40 DISORDERS OF THE FEMALE GENITOURINARY SYSTEM 245

f. Surgical procedure 3. What is the normal method of detecting/


used to scrape out diagnosing cervical cancer?
the surface of the
endometrium
g. Inflammation of
the cervix
h. Excessive
4. What are the three most prominent theories
menstrual bleeding
of the pathogenesis of endometriosis?
i. The vaginal cytol-
ogy to detect vagi-
nal or cervical
cancer
j. Inflammatory dis-
ease of the vulva 5. What is the mechanism of infection in pelvic
2. inflammatory disease?
Column A Column B
1. Amenorrhea a. Frequent menstru-
ation
2. Hypomenorrhea
b. Infrequent
3. Oligomenorrhea menstruation 6. Why should you be concerned about the
future of your patient with PCOS?
4. Polymenorrhea c. Bleeding between
periods
5. Menorrhagia
d. Absence of
6. Metrorrhagia menstruation
7. Menometror- e. Heavy bleeding
rhagia during and 7. Describe the functional anatomy of normal
between menstrual pelvic support.
periods
f. Scanty menstrua-
tion
g. Excessive menstru-
ation
8. Describe the alterations in a normal period
and give the hormone that thought to be
Activity D Briefly answer the following.
responsible.
1. What measures should be taken to avoid vagi-
nal infections?

9. What is the genetic component of breast


cancer?
2. What is the only approved vaccine for cervical
cancer and how does it work?

Copyright 2011. Wolters Kluwer Health | Lippincott Williams & Wilkins. Study Guide for Porths Essentials of Pathophysiology, Third Edition.
LWBK707-c40_p242-248.qxd 8/19/10 1:31 PM Page 246 Aptara Inc

246 UNIT 11 GENITOURINARY AND REPRODUCTIVE FUNCTION

SECTION III: APPLYING 2. There are two types of vulvar cancer. One
type is found in older women, and one type
YOUR KNOWLEDGE is found in younger women, generally less
than 40 years of age. What is the type found
Activity E Consider the scenario and answer
in younger women thought to be caused by?
the questions.
a. Multiple sexual partners
A 23-year-old woman is being seen in her physi-
b. Human papilloma virus
cians office as a follow-up to an abnormal Pap
smear. The physician explains to the client that c. Nonsquamous cell lesions
she may have cervical cancer, and he wants to do d. Lichen sclerotic lesions
a colposcopy so he can diagnose and treat any
3. Vaginal infections can occur in young girls
lesions he may find. The client gives her consent.
prior to menarche. These infections generally
1. While the nurse is preparing the client for the have nonspecific causes. What are some of
procedure, the client asks what a colposcopy the causes of vaginal infections in premenar-
is and what it is for. What would the nurse chal girls? (Mark all that apply.)
would correctly respond? a. Presence of foreign bodies
b. Intestinal parasites
c. Poor hygiene
d. Vaginal deodorants
e. Tampax
2. The colposcopy shows dysplastic lesions, and
the physician wants to do a large loop 4. The endocervix is covered with large-
excision of the transformation zone (LEEP branched mucous-secreting glands. During
procedure). The client gives her consent, but the menstrual cycle, they undergo functional
wants to know what this procedure is. How changes, and the amount and properties of
would the procedure be explained to the the mucous that they secret varies as to the
client? stage of the cycle. When one of these glands
get blocked, what kind of cyst forms within
the cervix?
a. Bartholin cysts
b. Bulbourethral cysts
c. Nabothian cysts
d. Metaplastic cysts
SECTION IV: PRACTICING 5. Endometriosis is the condition in which
FOR NCLEX endometrial tissue is found growing outside
the uterus in the pelvic cavity. What are risk
Activity F Answer the following questions. factors for endometriosis?
1. Bartholin gland obstruction of the ductal sys- a. Late menarche and regular periods with
tem will cause a cyst. Sometimes the cyst longer cycles than 27 days
becomes infected and an abscess occurs. b. Early menarche and lighter flow
What is the surgical removal of a Bartholin c. Increased menstrual pain and periods of
cyst or abscess when a wedge of vulvar skin shorter duration than 7 days
is removed along with the cyst wall?
d. Periods longer than 7 days and increased
a. Marsupialization menstrual pain
b. Vulvectomy
c. Bartholectomy
d. Incision and drainage

Copyright 2011. Wolters Kluwer Health | Lippincott Williams & Wilkins. Study Guide for Porths Essentials of Pathophysiology, Third Edition.
LWBK707-c40_p242-248.qxd 8/19/10 1:31 PM Page 247 Aptara Inc

CHAPTER 40 DISORDERS OF THE FEMALE GENITOURINARY SYSTEM 247

6. Leiomyomas, or intrauterine fibroids, are the 10. Ovarian cancer, once thought to be asympto-
most common form of pelvic tumor. Approx- matic, has now been shown to produce non-
imately half the time leiomyomas are asymp- specific symptoms, which make the diagnosis
tomatic. What are the symptoms of of ovarian cancer difficult. What symptoms
leiomyomas that are not asymptomatic? are believed to have a strong association with
a. Anemia and urinary frequency ovarian cancer? (Mark all that apply.)
b. Diarrhea and rectal pressure a. Difficulty eating
c. Menorrhagia and urinary retention b. Increased intestinal gas
d. Abdominal distention and diarrhea c. Bloating
d. Increased appetite
7. An 18-year-old woman presents at the clinic
complaining new-onset breakthrough bleed- e. Abdominal or pelvic pain
ing, even though she is taking contraceptives. 11. Uterine prolapse is a disorder of pelvic
What contraceptive use, along with new- support and uterine position. It can range in
onset breakthrough bleeding, has been asso- severity from a slight descent of the uterus
ciated with pelvic inflammatory disease? into the vagina, all the way to the entire
a. Intrauterine device uterus protruding through the vaginal open-
b. Depo-Provera ing. In women who want to have children, or
in older women who are at significant risk if
c. Spermicidal foam
surgery is performed, what device is inserted
d. Diaphragm to hold the uterus in place?
8. Ectopic pregnancies are true gynecologic a. A pessary
emergencies and are considered the leading b. A Colpexin sphere
cause of maternal death in the first trimester.
c. A vesicourethral suspender
What diagnostic test would you expect to
have ordered for a suspected ectopic d. A retroversion inducer
pregnancy? 12. In primary dysmenorrheal when
a. Transvaginal ultrasound if pregnancy is contraception is not desired, what is the
less than 5 weeks gestation treatment of choice?
b. Serial -human chorionic gonadotropin a. Aspirin
(hCG) with higher than normal hCG pro- b. Ibuprofen
duction
c. Acetaminophen
c. Ultrasonography followed by serial hCG
d. Metformic acid
tests
d. Amniocentesis 13. Mastitis is an inflammation of the breast that
can occur at any time. What is the treatment
9. Polycystic ovary syndrome is an endocrine for mastitis?
disorder and a common cause of chronic
a. Opioid analgesics
anovulation. In addition to the clinical mani-
festations of PCOS, long-term health b. Nonsteroidal anti-inflammatory drugs
problems including cardiovascular disease c. Application of heat or cold
and diabetes have been linked to PCOS. d. Tylenol 3
What drug has emerged as an important part
of PCOS treatment? 14. Fibrocystic changes in the breast are not
uncommon. How is the diagnosis of fibrocys-
a. DHEAS
tic changes made?
b. Methotrexate
a. Physical examination and client history
c. Mineralocorticoids
b. Galactography and biopsy
d. Metformin
c. Mammography and galactography
d. Ultrasonography and mammography

Copyright 2011. Wolters Kluwer Health | Lippincott Williams & Wilkins. Study Guide for Porths Essentials of Pathophysiology, Third Edition.
LWBK707-c40_p242-248.qxd 8/19/10 1:31 PM Page 248 Aptara Inc

248 UNIT 11 GENITOURINARY AND REPRODUCTIVE FUNCTION

15. Cancer of the breast is the most common 17. Couples who are being treated for infertility
cancer in women. Many breast cancers are often choose to try in vitro fertilization.
found by women themselves while doing When using this technique, the females eggs
breast self-examination. When should are inseminated with the males sperm in a
postmenopausal women do breast self- culture dish. After a period of time, the ova
examination? are evaluated for signs of fertilization. If signs
a. Any day of the month of fertilization are present, when are the fer-
tilized eggs placed in the womans uterus?
b. 2 days following menses
a. 12 to 24 hours after egg retrieval
c. On the first day of every month
b. 36 to 48 hours after egg retrieval
d. On the 15th of every month
c. 48 to 72 hours after egg retrieval
16. The causes of infertility can be in either the
d. 24 to 36 hours after egg retrieval
male or the female. Male tests for infertility
require a specimen of ejaculate that is
collected when?
a. Any time
b. After 3 days of abstinence
c. After 3 consecutive days of intercourse
d. After 3 weeks of abstinence

Copyright 2011. Wolters Kluwer Health | Lippincott Williams & Wilkins. Study Guide for Porths Essentials of Pathophysiology, Third Edition.
LWBK707-c41_p249-252.qxd 8/19/10 2:04 PM Page 249 Aptara Inc

41

CHAPTER
Sexually Transmitted
Infections

SECTION I: LEARNING SECTION II: ASSESSING


OBJECTIVES YOUR UNDERSTANDING
1. Define what is meant by a sexually transmit- Activity A Fill in the blanks.
ted infection (STI).
1. Sexually transmitted infections can
2. List common portals of entry for STIs. selectively infect the tissues
of the external genitalia, primarily cause
3. Name the organisms responsible for condylo-
vaginitis in women, or they can produce
mata acuminata, genital herpes, chancroid,
both genitourinary and systemic effects.
and lymphogranuloma venereum.
2. Sexually transmitted infections may be
4. State the significance of being infected with
transmitted by an infected mother to a
high-risk strains of the human
, causing congenital defects or
papillomavirus (HPV).
death of the child.
5. Explain the pathogenesis of recurrent genital
3. are caused by the HPV.
herpes infections.
4. Genital warts typically present as soft, raised,
6. State the difference between wet-mount slide
fleshy lesions on the , including
and culture methods of diagnosis of STIs.
the penis, vulva, scrotum, perineum, and
7. Compare the signs and symptoms of perianal skin.
infections caused by Candida albicans,
5. is one of the most common
Trichomonas vaginalis, and bacterial vaginosis.
causes of genital ulcers in the United States.
8. Compare the signs and symptoms of gonor-
6. Herpes simplex virus type-1 and herpes sim-
rhea in the male and female patient.
plex virus type-2 are viruses,
9. Describe the three stages of syphilis. meaning that they grow in neurons and
share the biologic property of latency.
10. State the genital and nongenital
complications that can occur with 7. Herpes simplex virus is respon-
chlamydial infections, gonorrhea, and sible for more than 90% of recurrent genital
syphilis. herpes infections.
11. State the treatment for chlamydial urogenital 8. The initial symptoms of infec-
infections, gonorrhea, nonspecific urogenital tions include tingling, itching, and pain in the
infections, and syphilis. genital area, followed by eruption of small
pustules and vesicles.

249
LWBK707-c41_p249-252.qxd 8/19/10 2:04 PM Page 250 Aptara Inc

250 UNIT 11 GENITOURINARY AND REPRODUCTIVE FUNCTION

9. Candida albicans is the most commonly iden- f. Yeast infection


tified organism in vaginal infec- g. Spirochete that is
tions, but other Candida species, such as responsible for
Candida glabrata and Candida tropicalis may syphilitic infection
also be present.
h. Genital warts
10. can reside in the paraurethral i. Obligate intracellu-
glands of both sexes. lar bacterial
11. vaginosis is the most prevalent pathogen that
form of vaginal infection seen by health care resembles a virus,
professionals. but like a bacteria
has RNA and DNA
12. exist in two forms: elementary and is susceptible
bodies, which are the infectious particles to some antibiotics
capable of entering uninfected cells, and the
j. Through skin-to-
initiator or reticulate bodies, which multiply
skin contact
by binary fission to produce the inclusions
identified in stained cells.
Activity C Briefly answer the following.
13. Untreated chlamydial infection results in
1. What are the risk factors for acquiring the
damage in female patients.
HPV and how is it spread?
14. The is a pyogenic (i.e., pus-
forming) gram-negative diplococcus that
evokes inflammatory reactions characterized
by purulent exudates.
15. is spread by direct contact with
2. How do herpes simplex virus type-1 and her-
an infectious moist lesion, usually through
pes simplex virus type-2 spread, and where do
sexual intercourse.
they reside in the body?
Activity B Match the key terms in Column A
with their definitions in Column B.
Column A Column B
1. Transmission a. Anaerobic proto-
3. What are the risk factors for developing a
of HPV zoan that can
candidiasis infection?
be transmitted
2. Condylomata
sexually
acuminata
b. Development of
3. Trichomonas large, tender, and
vaginalis sometimes fluc-
4. Chancroid tuant inguinal 4. What are the potential complications of
lymph nodes trichomoniasis in male and female patients?
5. Treponema called buboes
pallidum
c. Excess of
6. Candidiasis lactobacilli
7. Chlamydia d. Disease of the
trachomatis external genitalia
and lymph nodes 5. What are the sex-specific manifestations of
8. Lymphogranuloma gonorrhea?
e. Large
venereum
mononuclear
9. Donovan bodies cells filled with
intracytoplasmic
10. Dderlein
gram-negative
cytolysis
rods

Copyright 2011. Wolters Kluwer Health | Lippincott Williams & Wilkins. Study Guide for Porths Essentials of Pathophysiology, Third Edition.
LWBK707-c41_p249-252.qxd 8/19/10 2:04 PM Page 251 Aptara Inc

CHAPTER 41 SEXUALLY TRANSMITTED INFECTIONS 251

6. What is the clinical course of syphilis? 2. Primary genital herpes is a sexually transmit-
ted disease (STD) caused by either the Herpes
simplex virus type-1 or type-2. What are the
initial symptoms of primary genital herpes
infections? (Mark all that apply.)
a. Itching
b. Chancres
SECTION III: APPLYING YOUR c. Genital pain
KNOWLEDGE d. Eczemalike lesions
e. Small pustules
Activity D Consider the scenario and answer
3. There is no known cure for genital herpes,
the questions.
and methods of treatment are often sympto-
A 35-year-old man presents at the clinic complain- matic. Pharmacologic management of genital
ing of painful joints of the left leg and pain on uri- herpes includes which drugs?
nation. He also is noted to have mucocutaneous a. AZT
lesions on the palms of his hands.
b. Famciclovir
1. What would be important for the nurse to c. Nonsteroidal anti-inflammatory drugs
note while taking a nursing history?
d. Topical corticosteroid compounds
4. Chancroid or soft chancre is a highly conta-
gious STD usually found in the Southeast
Asian and North African populations.
What is the recommended treatment for
2. The client is diagnosed with a chlamydial Chancroid?
infection. What would be the expected treat- a. Tetracycline
ment for this client?
b. Sulfamethoxazole
c. Erythromycin
d. Acyclovir
5. A male client presents at the clinic with
flulike symptoms and reports a weight loss
of 10 pounds without trying. On physical
examination, the client is found to have
SECTION IV: PRACTICING splenomegaly and large, tender, fluctuant
FOR NCLEX inguinal lymph nodes. While taking the
nursing history, it is discovered that the
Activity E Answer the following questions. client prefers male sexual partners, and that
1. After inoculation with HPV, genital warts 2 weeks ago the client had small, painless
may begin to grow. They usually manifest as papules. What disease would the nurse
soft, raised fleshy lesions on the external gen- suspect the client has?
italia of either male of female. What is the a. Genital herpes
incubation period for HPV-induced genital b. Chancroid
warts?
c. Syphilis
a. 6 weeks to 8 months
d. Lymphogranuloma venereum
b. 6 weeks to 8 weeks
c. 6 months to 8 months
d. 6 days to 8 days

Copyright 2011. Wolters Kluwer Health | Lippincott Williams & Wilkins. Study Guide for Porths Essentials of Pathophysiology, Third Edition.
LWBK707-c41_p249-252.qxd 8/19/10 2:04 PM Page 252 Aptara Inc

252 UNIT 11 GENITOURINARY AND REPRODUCTIVE FUNCTION

6. Candidiasis is a leading cause of vaginal 9. Gonorrhea is an STD that affects both men
infections. Which antifungal agent is not and women. When diagnosing gonorrhea,
available without prescription to treat specimens should be collected from the
candidiasis? appropriate site and inoculated onto the cor-
a. Terconazole rect medium. What sites can specimens be
collected from when diagnosing gonorrhea?
b. Clotrimazole
(Mark all that apply.)
c. Miconazole
a. Oropharynx
d. Butaconazole
b. Urethra
7. Trichomoniasis is an STD that can occur c. Nasal passages
in either sex. Men carry the protozoan in
d. Exocervix
the urethra and prostate and remain asymp-
tomatic. This anaerobic protozoan can cause e. Anal canal
a number of complications. What is it a risk 10. Tertiary syphilis is a delayed response of
factor for in both men and women? untreated primary syphilis and can occur as
a. Atypical pelvic inflammatory disease long as 20 years after the primary disease.
b. HIV transmission When tertiary syphilis progresses to a sympto-
matic stage, it can produce localized necrotic
c. Blockage of tubes and ducts
lesions. What are these lesions called?
d. Ovarian and testicular cysts
a. Chancres
8. Bacterial vaginosis is the most common vagi- b. Chancroids
nal infection seen by health care providers.
c. Gummies
What is the predominant symptom of
bacterial vaginosis? d. Gummas
a. Thick, cottage cheeselike discharge with
a fishy odor
b. Painless chancres
c. Grayish-white discharge with a fishy odor
d. Small, painless papules

Copyright 2011. Wolters Kluwer Health | Lippincott Williams & Wilkins. Study Guide for Porths Essentials of Pathophysiology, Third Edition.
LWBK707-c42_p253-257.qxd 8/19/10 2:04 PM Page 253 Aptara Inc

42

CHAPTER
Structure and Function
of the Skeletal System

SECTION I: LEARNING SECTION II: ASSESSING


OBJECTIVES YOUR UNDERSTANDING
1. Describe locations and characteristics of com- Activity A Fill in the blanks.
pact and cancellous bone.
1. The bones of the skeletal system serve
2. Describe the structure of a long bone. as a framework for the attachment of
, , and
3. Cite the characteristics and name at least one
.
location of elastic cartilage, hyaline cartilage,
and fibrocartilage. 2. The bones act as a storage reservoir for
, and the central cavity of some
4. Name and characterize the function of the
bones contains the hematopoietic connective
four types of bone cells.
tissue in which cells are
5. State the function of parathyroid hormone, formed.
calcitonin, and vitamin D in terms of bone
3. The skeletal system consists of the
formation and metabolism.
and skeleton.
6. State the characteristics of tendons and
4. bone has a densely packed
ligaments.
calcified intercellular matrix that makes it
7. State the difference between synarthroses more rigid than cancellous bone.
and synovial joints.
5. are classified by shape as long,
8. Describe the source of blood supply to a short, flat, and irregular.
synovial joint.
6. Bones are covered, except at their articular
9. Explain why pain is often experienced in all ends, by a membrane called the .
the joints of an extremity when only a single
7. Bone occupies the medullary
joint is affected by a disease process.
cavities of the long bones throughout the
10. Describe the structure and function of a skeleton and the cavities of cancellous bone
bursa. in the vertebrae, ribs, sternum, and flat bones
of the pelvis.
11. Explain the pathology associated with a torn
meniscus of the knee.

253
LWBK707-c42_p253-257.qxd 8/19/10 2:04 PM Page 254 Aptara Inc

254 UNIT 12 MUSCULOSKELETAL FUNCTION

8. The enter the bone through a Activity B Consider the following figure.
nutrient foramen and supply the marrow
space and the internal half of the cortex.
9. Bone is tissue in which the
intercellular matrix has been impregnated
with inorganic salts so that it
has great tensile and compressible strength
but is light enough to be moved by
coordinated muscle contractions.
10. The undifferentiated cells are
found in the periosteum, endosteum, and
epiphyseal plate of growing bone.
A
11. are bone-chewing cells
that function in the resorption of bone,
removing the mineral content and the
organic matrix.
12. cartilage is found in areas, such
as the ear, where some flexibility is
important.
13. is found in the intervertebral
disks, in areas where tendons are connected B C
to bone, and in the symphysis pubis.
14. cartilage forms the costal carti-
lages that join the ribs to the sternum and
vertebrae, many of the cartilages of the respi-
ratory tract, the articular cartilages, and the In the figure above, locate and label the
epiphyseal plates. following structures:
15. inhibits the release of calcium Proximal epiphysis
from bone into the extracellular fluid. Medullary cavity
16. , which attach skeletal muscles Periosteum
to bone, are relatively inextensible because of Nutrient artery
their richness in collagen fibers. Compact bone
17. are fibrous thickenings of the Spongy bone
articular capsule that join one bone to its Yellow marrow
articulating mate.
Diaphysis
18. are joints that lack a joint
cavity and move a little or not at all. Activity C Match the key terms in Column A
19. joints are freely movable joints. with their definitions in Column B.
20. The purpose of a sac is to Column A Column B
prevent friction on a tendon. 1. Trabeculae a. Connect adjacent
haversian canals
2. Appendicular
skeleton b. Prebone that will be
ossified
3. Osteoid c. The bones of the
4. Lamellar bone skull, thorax, and
vertebral column
5. Haversian
canal d. Mature bone found
in the adult skeleton

Copyright 2011. Wolters Kluwer Health | Lippincott Williams & Wilkins. Study Guide for Porths Essentials of Pathophysiology, Third Edition.
LWBK707-c42_p253-257.qxd 8/19/10 2:04 PM Page 255 Aptara Inc

CHAPTER 42 STRUCTURE AND FUNCTION OF THE SKELETAL SYSTEM 255

6. Osteons e. Contains the blood Reabsorption of calcium via 1,25-dihydroxy


vessels and nerve vitamin D3
7. Volkmann
supply for the osteon Synthesis of 1,25-dihydroxy vitamin D3
canals
f. Bones of the upper Release of calcium and phosphate
8. Axial skeleton and lower extremities,
Reabsorption of calcium
9. Endosteum including the shoul-
der and hip Urinary excretion of phosphate
10. Chondrocytes
g. Membrane that lines
the spaces of spongy Activity E Briefly answer the following.
bone, the marrow 1. What is the typical structure of a long bone?
cavities, and the
haversian canals
h. Cells that form carti-
lage
i. Concentric lamellae
of bone matrix, sur- 2. What are the two types of bone marrow and
rounding a central what are their functions?
canal
j. Lined with osteogenic
cells and filled with
red or yellow bone
marrow
3. What is the makeup of the intercellular matrix
of bone tissue?
Activity D In the flow chart shown, put the
following in the proper sequence.

Parathyroid
glands
Kidney
4. What are the similarities and differences
between bone and cartilage?
Bone

Calcium
concentration
in extracellular 5. How does parathyroid hormone maintain
fluid serum calcium levels?

6. What is the action of vitamin D?


Intestine

Copyright 2011. Wolters Kluwer Health | Lippincott Williams & Wilkins. Study Guide for Porths Essentials of Pathophysiology, Third Edition.
LWBK707-c42_p253-257.qxd 8/19/10 2:04 PM Page 256 Aptara Inc

256 UNIT 12 MUSCULOSKELETAL FUNCTION

SECTION III: APPLYING 3. Lamellar bone is the bone tissue that is found
in the adult body. What is lamellar bone
YOUR KNOWLEDGE largely composed of?
Activity F Consider the scenario and answer a. Hematopoietic cells
the questions. b. Spicules
Case study: A 62-year-old woman with multiple c. Osteons
sclerosis was referred to the orthopedic clinic by d. Macrocrystalline cells
her primary care physician because of pain on
4. Our bodies contain three types of cartilage:
movement in her upper arms. Because of the
elastic, hyaline, and fibrocartilage. Which of
multiple sclerosis, the clients legs were extremely
these types of cartilage is found in the
weak, and the client had to lift herself out of a
symphysis pubis?
chair with her arms. After a physical examination,
the orthopedic physician diagnosed her as having a. None
bilateral biceps tendinitis. b. Elastic
1. The client asks what causes tendinitis. What c. Hyaline
would be the correct answer? d. Fibrocartilage
5. Parathyroid hormone functions to maintain
serum calcium levels. How does it fulfill this
function? (Mark all that apply.)
a. Initiates calcium release from bone
2. The client asks if all tendons are like the b. Enhances intestinal absorption of calcium
biceps tendons. What would be the correct c. Activates conservation of calcium by the
answer? kidney
d. Decreases intestinal absorption of calcium
e. Inhibits conservation of calcium by the
kidney
6. When vitamin D is metabolized it takes
breaks down into various metabolites.
1,25(OH)2D3 is the most potent of the
SECTION IV: PRACTICING Vitamin D metabolites. What is the function
FOR NCLEX of this metabolite of vitamin D?
a. Promotes actions of parathyroid hormone
Activity G Answer the following questions. on resorption of calcium and phosphate
1. The metaphysis is the part of the bone that from bone
fans out toward the epiphysis. What is the b. Decreases intestinal absorption of calcium
metaphysis composed of? c. Promotes absorption of calcium and phos-
a. Trabeculae phate by bone
b. Cancellous bone d. Decreases absorption of phosphate and in-
c. Red bone marrow creases absorption of calcium by bone
d. Endosteum 7. There are two types of joints in the human
body. They are synarthroses and synovial
2. We have both red and yellow bone marrow
joints. Synarthroses joints are further broken
in our bodies. What is yellow bone marrow
down into three types of joint. What type of
largely composed of?
joint occurs when bones are connected by
a. Hematopoietic cells hyaline cartilage?
b. Adipose cells a. Synovial
c. Cancellous cells b. Synchondroses
d. Osteogenic cells c. Syndesmoses
d. Diarthrodial

Copyright 2011. Wolters Kluwer Health | Lippincott Williams & Wilkins. Study Guide for Porths Essentials of Pathophysiology, Third Edition.
LWBK707-c42_p253-257.qxd 8/19/10 2:04 PM Page 257 Aptara Inc

CHAPTER 42 STRUCTURE AND FUNCTION OF THE SKELETAL SYSTEM 257

8. Rheumatic disorders attack the joints of the 10. Synovial membranes can form sacs, called
body. Which joints are most frequently bursae. What is the function of bursae?
attacked by rheumatic disorders? a. Prevent friction on a tendon
a. Synchondroses b. Prevent injury to a joint
b. Articular c. Prevent friction on a ligament
c. Diarthrodial d. Cushion the joint
d. Synarthroses
9. Each joint capsule has tendons and
ligaments? What are the tendons and
ligaments of the joint capsule sensitive to?
a. Position and elevating
b. Position and lowering
c. Position and turning
d. Position and movement

Copyright 2011. Wolters Kluwer Health | Lippincott Williams & Wilkins. Study Guide for Porths Essentials of Pathophysiology, Third Edition.
LWBK707-c43_p258-263.qxd 8/19/10 2:25 PM Page 258 Aptara Inc

43
CHAPTER
Disorders of the
Skeletal System:
Trauma, Infections,
Neoplasms, and
Childhood Disorders

SECTION I: LEARNING 11. Differentiate among osteomyelitis due to


spread from a contaminated wound,
OBJECTIVES hematogenous osteomyelitis, and osteomyelitis
due to vascular insufficiency in terms of etiolo-
1. Describe the physical agents responsible for
gies, manifestations, and treatment.
soft-tissue trauma.
12. Cite the characteristics of chronic
2. Differentiate among the types of soft-tissue
osteomyelitis.
injuries.
13. Describe the most common sites of tubercu-
3. Compare muscle strains and ligamentous
losis of the bone.
sprains.
14. Define osteonecrosis.
4. Describe the healing process of soft-tissue
injuries. 15. Cite four major causes of osteonecrosis.
5. Differentiate open from closed fractures. 16. Characterize the blood supply of bone and
relate it to the pathologic features of the
6. List the signs and symptoms of a fracture.
condition.
7. Explain the measures used in treatment of
17. Describe the methods used in diagnosis and
fractures.
treatment of the condition.
8. Describe the fracture healing process.
18. Differentiate between the properties of
9. Differentiate the early complications of benign and malignant bone tumors.
fractures from later complications of fracture
19. Contrast osteogenic sarcoma, Ewing sarcoma,
healing.
and chondrosarcoma in terms of the most
10. Explain the implications of bone infection. common age groups and anatomic sites that
are affected.

258
LWBK707-c43_p258-263.qxd 8/19/10 2:25 PM Page 259 Aptara Inc

CHAPTER 43 DISORDERS OF THE SKELETAL SYSTEM: TRAUMA, INFECTIONS, NEOPLASMS, AND CHILDHOOD DISORDERS 259

20. List the primary sites of tumors that 12. A fracture occurs in bones that
frequently metastasize to the bone. already are weakened by disease or tumors.
21. State the three primary goals for treatment of 13. The signs and symptoms of a
metastatic bone disease. include pain, tenderness at the site of bone dis-
ruption, swelling, loss of function, deformity
of the affected part, and abnormal mobility.
14. is another method for achieving
SECTION II: ASSESSING immobility and maintaining alignment of
YOUR UNDERSTANDING the bone ends and maintaining the reduction,
particularly if the fracture is unstable or
Activity A Fill in the blanks. comminuted.
1. A broad spectrum of injuries 15. are skin bullae and blisters
result from numerous physical forces, includ- representing areas of epidermal necrosis with
ing blunt tissue trauma, disruption of separation of epidermis from the underlying
tendons and ligaments, and fractures of bony dermis by edema fluid.
structures.
16. Because of inactivity and restrictions in weight
2. Unintentional are the number- bearing, the individual with a lower extremity
one cause of nonfatal injuries in all age fracture is at risk for the development of
groups. venous , which includes
3. injuries include contusions, pulmonary embolism and deep venous
hematomas, and lacerations. thrombosis.

4. A is a stretching injury to a 17. The syndrome refers to a


muscle or a musculotendinous unit caused by constellation of clinical manifestations result-
mechanical overloading. ing from the presence of fat droplets in the
small blood vessels of the lung or other
5. A usually is caused by abnormal organs after a long bone fracture or other
or excessive movement of the joint. major trauma.
6. A involves the displacement or 18. osteomyelitis symptoms
separation of the bone ends of a joint with include pain, immobility, and muscle
loss of articulation. atrophy; joint swelling, mild fever, and leuko-
7. bodies are small pieces of bone cytosis also may occur.
or cartilage within a joint space. 19. , or death of a segment of bone,
8. injuries and impingement dis- is a condition caused by the interruption of
orders can result from a number of causes, blood supply to the marrow, medullary bone,
including excessive use, a direct blow, or or cortex.
stretch injury, usually involving throwing or 20. Malignant bone tumors, such as ,
swinging, as with baseball pitchers or tennis grow rapidly and can spread to other parts
players. of the body through the bloodstream or
9. Meniscus injury commonly occurs as the lymphatics.
result of a injury from a sudden 21. bone tumors usually are limited
or sharp pivot or a direct blow to the knee, as to the confines of the bone, have well-demar-
in hockey, basketball, or football. cated edges, and are surrounded by a thin rim
10. of the hip commonly result of sclerotic bone.
from the knee being struck while the hip and 22. A is a tumor composed of
knee are in a flexed position. hyaline cartilage.
11. Grouped according to cause, fractures can be 23. , a malignant tumor of cartilage
divided into three major categories: fractures that can develop in the medullary cavity or
caused by , fatigue or stress frac- peripherally, is the second most common
tures, and fractures. form of malignant bone tumor.

Copyright 2011. Wolters Kluwer Health | Lippincott Williams & Wilkins. Study Guide for Porths Essentials of Pathophysiology, Third Edition.
LWBK707-c43_p258-263.qxd 8/19/10 2:25 PM Page 260 Aptara Inc

260 UNIT 12 MUSCULOSKELETAL FUNCTION

Activity B Consider the following figure. 9. Greenstick e. Healing of bone with


fracture deformity, angulation,
or rotation
10. Osteomyelitis
f. Large area of local
Proximal hemorrhage
g. Softening of the artic-
Midshaft ular cartilage
h. A partial break in
bone continuity;
Distal resembles what is
seen when a young
sapling is broken
i. Area becomes ecchy-
motic (i.e., black and
blue) because of local
hemorrhage
j. C-shaped plates of
fibrocartilage that
are superimposed
between the condyles
of the femur and tibia

Activity D Put the following events of healing


a bone fracture into the proper order in the
In the figure above, label the type of fracture boxes below.
Impacted
Butterfly S S S
Comminuted
Transverse
A. Development of fibrin meshwork within
Oblique
the hematoma
Segmental
B. Replacement of callus with mature bone
Spiral
C. Formation of fibrocartilaginous callus
D. Remodeling of bone
Activity C Match the key terms in Column A
with their definitions in Column B.
Activity E Briefly answer the following.
Column A Column B
1. What joints are most commonly involved in
1. Subluxation a. A partial sprain-type injuries?
dislocation
2. Malunion
b. Acute or chronic
3. Contusion infection of the
4. Compound bone
fracture c. Bone fragments
2. What is the normal healing process of a
have broken
5. Menisci sprain? What are some of the greatest
through the skin
concerns?
6. Hematoma d. Injury in which
7. Chondromalacia the skin is torn or
its continuity is
8. Laceration disrupted

Copyright 2011. Wolters Kluwer Health | Lippincott Williams & Wilkins. Study Guide for Porths Essentials of Pathophysiology, Third Edition.
LWBK707-c43_p258-263.qxd 8/19/10 2:25 PM Page 261 Aptara Inc

CHAPTER 43 DISORDERS OF THE SKELETAL SYSTEM: TRAUMA, INFECTIONS, NEOPLASMS, AND CHILDHOOD DISORDERS 261

3. What is the structure of rotator cuff and how SECTION III: APPLYING
is it usually injured?
YOUR KNOWLEDGE
Activity F Consider the scenario and answer
the questions.
Case study: A 15-year-old boy is brought to the
4. When someone breaks a hip, what is usually emergency department after an injury playing
occurring? football. The doctor suspects an injury to the
meniscus of the knee.
1. As the nurse, what orders would you expect to
receive to confirm the suspected diagnosis?

5. What is compartment syndrome and how


does it relate to bone tissue?

2. The diagnosis of torn meniscus is confirmed.


What would be the first-line treatment for this
type of injury?
6. What are the manifestations of osteomyelitis?

3. The client asks what will happen if his knee


does not heal right. The correct answer would
7. What is the pathogenesis of osteonecrosis?
include what?

8. What are the general characteristics of bone


tumors?
SECTION IV: PRACTICING
FOR NCLEX
Activity G Answer the following questions.
9. What is metastatic bone disease? 1. Athletic injuries fall into two types, acute or
overuse injuries. Where do overuse injuries
commonly occur?
a. Knee
b. Wrist
c. Neck
d. Fingers

Copyright 2011. Wolters Kluwer Health | Lippincott Williams & Wilkins. Study Guide for Porths Essentials of Pathophysiology, Third Edition.
LWBK707-c43_p258-263.qxd 8/19/10 2:25 PM Page 262 Aptara Inc

262 UNIT 12 MUSCULOSKELETAL FUNCTION

2. Match the injury to its definition. 5. There are times when fractures of long bones
need enhancement to promote healing.
Injury Definition
What can be done to induce bone formation
1. Contusion a. The ligaments may be and repair bone defects?
incompletely torn or, as a. The use of steroids to induce bone growth
2. Hematoma
in a severe sprain, com-
b. The use of growth factors to induce bone
3. Laceration pletely torn or ruptured
growth
4. Puncture b. An injury in which the
c. The use of vibration therapy to induce
wounds skin is torn or its conti-
bone growth
nuity is disrupted
5. Strain d. The use of physical therapy to induce bone
c. A stretching injury
growth
6. Sprain caused by mechanical
overloading 6. Determining the extent of the injury when a
7. Dislocation
d. Blood accumulates and fracture occurs is important. It is also impor-
exerts pressure on nerve tant to obtain a thorough history. What is
endings important to determine during the history
taking? (Mark all that apply.)
e. Displacement or separa-
tion of the bone ends of a. Anyone else in family prone to fractures
a joint with loss of artic- b. Recognition of symptoms
ulation c. Any treatment initiated
f. Provide the setting for d. Mechanism of injury
growth of anaerobic bac-
e. What patient has eaten
teria
g. The skin overlying the 7. Match the complication with the definition.
injury remains intact Complication
of Fracture Definition
3. Shoulder and rotator cuff injuries usually
occur from trauma or overuse. What orders 1. Fracture blisters a. Areas of epidermal
would be given for conservative treatment necrosis with separa-
2. Compartment
of an injured shoulder? (Mark all that tion of epidermis from
syndrome
apply.) the underlying dermis
a. Anesthetic injections 3. Complex by edema fluid
regional pain b. Reflex sympathetic
b. Physical therapy
syndrome dystrophy
c. Corticosteroid injections
c. A condition of in-
d. Anti-inflammatory agents
creased pressure
e. Pain medicine within a limited space
4. Hip injuries include dislocations and (e.g., abdominal and
fractures of the hip. Why is dislocation of a limb compartments)
hip considered a medical emergency? that compromises the
circulation and func-
a. The dislocation causes great pain
tion of the tissues
b. Avascular necrosis may result from the within the space.
dislocation
c. The longer the hip is dislocated, the less 8. Fat emboli syndrome can occur after a
chance of putting it back in place fracture of a long bone. What are the clinical
d. Dislocation interrupts the blood supply to features of this syndrome?
the femoral head a. Petechiae on soles of feet and palms of
hands
b. Respiratory insufficiency
c. Encephalopathy
d. Global neurologic deficits

Copyright 2011. Wolters Kluwer Health | Lippincott Williams & Wilkins. Study Guide for Porths Essentials of Pathophysiology, Third Edition.
LWBK707-c43_p258-263.qxd 8/19/10 2:25 PM Page 263 Aptara Inc

CHAPTER 43 DISORDERS OF THE SKELETAL SYSTEM: TRAUMA, INFECTIONS, NEOPLASMS, AND CHILDHOOD DISORDERS 263

9. Osteomyelitis is an infection of the bone. 12. Osteosarcoma is an aggressive malignancy of


Chronic osteomyelitis is complicated by a the bone. What is the primary clinical feature
piece of infected dead bone that has of osteosarcoma?
separated from the living bone. How long a. Pain, worse during the day
does the initial intravenous antibiotic
b. Erythema in the overlaying skin
therapy last for chronic osteomyelitis?
c. Nighttime awakening
a. 4 weeks
d. Soreness in nearest joint
b. 8 weeks
c. 12 weeks 13. Metastatic bone disease is a frequent disorder.
It occurs at a time when primary tumors in the
d. 6 weeks
lungs, breasts, and prostate seed themselves
10. Tuberculosis can spread from the lungs into (metastasize) to the musculoskeletal system.
the musculoskeletal system. What is the most What are the primary goals of treatment for
common site in the skeletal system for tuber- metastatic bone disease? (Mark all that apply.)
culosis to be found? a. Prevent pathologic fractures
a. Spine b. Cure the disease
b. Ankles c. Promote survival with maximum
c. Shoulders functioning
d. Hips d. Prevent ischemia to the bone segment
11. Osteonecrosis is a condition in which part of e. Maintain mobility and pain control
a bone dies because of the interruption of its
blood supply. What is the most common
cause of osteonecrosis other than fracture?
a. Vessel injury
b. Prior steroid therapy
c. Radiation therapy
d. Embolism

Copyright 2011. Wolters Kluwer Health | Lippincott Williams & Wilkins. Study Guide for Porths Essentials of Pathophysiology, Third Edition.
LWBK707-c44_p264-268.qxd 8/20/10 11:54 PM Page 264 Aptara

44
CHAPTER
Disorders of the
Skeletal System:
Metabolic and
Rheumatic Disorders

SECTION I: LEARNING 10. Contrast and compare ankylosing


spondylitis, reactive arthritis, and psoriatic
OBJECTIVES arthritis in terms of cause, pathogenesis, and
clinical manifestations.
1. Characterize the common characteristics of
the different systemic autoimmune 11. Compare rheumatoid arthritis and
rheumatic disorders. osteoarthritis in terms of joint involvement,
level of inflammation, and local and systemic
2. Describe the pathologic changes that may be
manifestations.
found in the joint of a person with rheuma-
toid arthritis. 12. Describe the pathologic joint changes associ-
ated with osteoarthritis.
3. List the extra-articular manifestations of
rheumatoid arthritis. 13. Characterize the treatment of osteoarthritis.
4. Describe the immunologic process that occurs 14. Relate the metabolism and elimination of
in systemic lupus erythematosus (SLE). uric acid to the pathogenesis of crystal-
induced arthropathy.
5. List four major organ systems that may be
involved in SLE. 15. State why asymptomatic hyperuricemia is a
laboratory finding and not a disease.
6. Describe the manifestations of systemic
sclerosis. 16. Describe the clinical manifestations, diagnos-
tic measures, and methods used in the
7. Cite a definition of the seronegative
treatment of gouty arthritis.
spondyloarthropathies.
17. List three types of juvenile arthritis and
8. Cite the primary features of ankylosing
differentiate among their major
spondylitis.
characteristics.
9. Describe how the site of inflammation differs
18. Name one rheumatic disease that affects only
in spondyloarthropathies from that in
the elderly population.
rheumatoid arthritis.

264
LWBK707-c44_p264-268.qxd 8/20/10 11:54 PM Page 265 Aptara

CHAPTER 44 DISORDERS OF THE SKELETAL SYSTEM: METABOLIC AND RHEUMATIC DISORDERS 265

SECTION II: ASSESSING 11. syndrome includes acute arthri-


tis with recurrent attacks of severe articular
YOUR UNDERSTANDING and periarticular inflammation; tophi or the
accumulation of crystalline deposits in articu-
Activity A Fill in the blanks.
lar surfaces, bones, soft tissue, and cartilage;
1. The cause of remains gouty nephropathy or renal impairment; and
uncertain, but evidence points to a genetic uric acid kidney stones.
predisposition and the development of joint
12. is characterized by synovitis
inflammation that is immunologically
and can influence epiphyseal growth by stim-
mediated.
ulating growth of the affected side.
2. It has been suggested that rheumatoid
13. Children with may present
arthritis is initiated in a genetically predis-
with constitutional symptoms, including
posed individual by the activation of a
fever, malaise, anorexia, and weight loss, just
response to an immunologic
as adults.
trigger, such as a microbial agent.
14. Juvenile is an inflammatory
3. Systemic lupus erythematosus is a
myopathy primarily involving skin and mus-
disease that can affect
cle and associated with a characteristic rash.
virtually any organ system, including the
musculoskeletal system. 15. is the most common complaint
of elderly persons.
4. Almost all persons with
develop polyarthritis and Raynaud phenome- 16. is by far the most common
non, a vascular disorder characterized by form of arthritis among the elderly.
reversible vasospasm of the arteries supplying
17. is an inflammatory condition
the fingers.
of unknown origin characterized by aching
5. is a chronic, systemic and morning stiffness in the cervical regions
inflammatory disease of the joints of the and shoulder and pelvic girdle areas.
axial skeleton manifested by pain and
progressive stiffening of the spine. Activity B Consider the following figure.
6. The reactive may be defined as
sterile inflammatory joint disorders that are
distant in time and place from the initial
inciting infective process.
7. is considered a clinical manifes-
tation of reactive arthritis that may be
accompanied by extra-articular symptoms
such as uveitis, bowel inflammation, and
carditis.
8. Arthritis that is associated with an inflamma-
tory bowel disease usually is considered an
arthritis because the intestinal
disease is directly involved in the pathogenesis.
9. is the most prevalent form of
arthritis and is a leading cause of disability
and pain in the elderly.
In the figure above, locate the following joint
10. Popularly known as arthritis,
changes seen in osteoarthritis.
osteoarthritis is characterized by significant
changes in both the composition and Joint space narrows
mechanical properties of cartilage. Erosion of cartilage and bone
Osteophyte development
Bone cysts

Copyright 2011. Wolters Kluwer Health | Lippincott Williams & Wilkins. Study Guide for Porths Essentials of Pathophysiology, Third Edition.
LWBK707-c44_p264-268.qxd 8/20/10 11:54 PM Page 266 Aptara

266 UNIT 12 MUSCULOSKELETAL FUNCTION

Activity C Match the key terms in Column A Activity D Put the following processes involved
with their definitions in Column B. in rheumatoid arthritis in proper sequence.
Column A Column B
1. Spondy- a. Autoimmune disease
S S S S S
loarthropathies of connective tissue
A. Inflammatory response
characterized by
2. Reactive B. Recruitment of inflammatory cells
excessive collagen
arthritis
deposition C. Destruction of articular cartilage
3. Systemic lupus b. Bone spurs D. Complement fixation
erythematosus
c. Multisystem inflam- E. T-cellmediated response
4. Joint mice matory disorders F. Release of enzymes and prostaglandins
that primarily affect
5. Scleroderma G. RH antigen/immunoglobulin G (IgG)
the axial skeleton
interaction
6. Osteophytes d. An inflammatory
7. Ankylosing erosion of the sites Activity E Briefly answer the following.
spondylitis where tendons and
ligaments attach to 1. What is the pathogenesis of rheumatoid
8. Baker cyst bone arthritis?
9. Polymyalgia e. Result from the pres-
rheumatica ence of a foreign sub-
stance in the joint
10. Gout
tissue
f. Inflammatory condi- 2. What causes the degradation of a joint in
tion marked by anti- rheumatoid arthritis?
nuclear antibodies
g. Enlargement of
the bursa in the
popliteal area behind
the knee
h. Disorder of the mus- 3. What are the musculoskeletal manifestations
cles and joints, typi- of SLE?
cally of older persons
characterized by pain
and stiffness, affect-
ing both sides of the
body, and involving
the shoulders, arms, 4. What are the typical joint changes seen in
neck, and buttock osteoarthritis?
areas
i. Uric acid crystals are
found in the joint
cavity
j. Fragments of 5. What is the pathogenesis of primary and
cartilage and bone secondary gout?
often become
dislodged, creating
free-floating
osteocartilaginous
bodies

Copyright 2011. Wolters Kluwer Health | Lippincott Williams & Wilkins. Study Guide for Porths Essentials of Pathophysiology, Third Edition.
LWBK707-c44_p264-268.qxd 8/20/10 11:54 PM Page 267 Aptara

CHAPTER 44 DISORDERS OF THE SKELETAL SYSTEM: METABOLIC AND RHEUMATIC DISORDERS 267

SECTION III: APPLYING 2. Systemic lupus erythematosus has been called


the great imitator because it can affect many
YOUR KNOWLEDGE different body systems. What is among the
most commonly occurring symptoms in the
Activity F Consider the scenario and answer
early stages of SLE?
the questions.
a. Arthritis
Case study: A 5-year-old girl is brought to the
b. Avascular necrosis
clinic by her mother because she just isnt feeling
well. While taking the history, the nurse notes a c. Rupture of the Achilles tendon
weight loss of 5 pounds during the past year and d. Classic malar rash
complaints of malaise. The childs growth chart
3. Scleroderma is an autoimmune disease of
shows she is in the 20th percentile for height.
connective tissue that is characterized by
During the physical examination, the physician
hardening of the skin. What diseases do
notes pain in three joints, hepatosplenomegaly,
almost all people with scleroderma develop?
and lymph adenopathy. The suspected diagnosis
(Mark all that apply.)
is juvenile idiopathic arthritis. The mother asks
the nurse to explain juvenile idiopathic arthritis. a. Dumping syndrome
b. Chronic diarrhea
1. What information would the nurse include in
her response? c. Polyarthritis
d. Raynaud phenomenon
e. Chronic vasoconstriction
4. Polymyositis and dermatomyositis are
chronic inflammatory myopathies that com-
2. What confirmatory test would the nurse monly manifest systemically. What is the
expect to see ordered? treatment of choice for these myopathies?
a. Muscle relaxants
b. Corticosteroids
c. IgG
d. Nonsteroidal anti-inflammatory drugs
(NSAIDs)
5. Ankylosing spondylitis is a disease that typi-
SECTION IV: PRACTICING cally manifests in late adolescence and early
FOR NCLEX adulthood. What is characteristic of the pain
in ankylosing spondylitis?
Activity G Answer the following questions. a. Worse when active
1. Joint destruction in rheumatoid arthritis b. Worse when sitting
occurs by an obscure process. The cellular c. Worse when lying
changes, however, have been documented.
d. Worse when standing
Place the process in the correct order.
a. Vasodilation 6. Reiter syndrome is a reactive arthropathy.
What disease is Reiter syndrome associated
b. Joint swelling
with?
c. Neutrophils, macrophages, and lympho-
a. Pelvic inflammatory disease
cytes arrive
b. Gonorrhea
d. Lysosomal enzymes released
c. Syphilis
e. Immune complexes phagocytized
d. Human immunodeficiency virus (HIV)
f. Inflammatory response
g. Reactive hyperplasia of synovial cells and
subsynovial tissues
h. Increased blood flow to joint
i. Destructive changes in joint cartilage

Copyright 2011. Wolters Kluwer Health | Lippincott Williams & Wilkins. Study Guide for Porths Essentials of Pathophysiology, Third Edition.
LWBK707-c44_p264-268.qxd 8/20/10 11:54 PM Page 268 Aptara

268 UNIT 12 MUSCULOSKELETAL FUNCTION

7. A seronegative inflammatory arthropathy is 9. Gout, or gouty arthritis, cannot be diagnosed


psoriatic arthritis. What drug has been found on the basis of hyperuricemia. What is the
to be beneficial in controlling both the psori- diagnostic criterion for gout?
asis and the arthritis in these patients? a. Finding of tophaceous deposits
a. Etanercept b. Finding of monosodium urate crystals in
b. Acetaminophen the synovial fluid
c. Interferon B c. Finding of sodium urate crystals in the
d. Econazole tissues
d. Finding of urate crystal deposits in the
8. Osteoarthritis is the most common cause of
synovial fluid
arthritis and a significant cause of disability
in the elderly. What joint changes occur in 10. The elderly population needs special consid-
osteoarthritis? (Mark all that apply.) eration in the treatment of the arthritic
a. Creation of spurs diseases. The NSAIDs, a first-line group of
drugs used in the general population for
b. Loss of synovial fluid
arthritic diseases, may not be well tolerated
c. Loss of articular cartilage by the elderly. What side effects of NSAIDs
d. Inflammation of cartilage might be seen in the elderly?
e. Synovitis a. Malaise
b. Lethargy
c. Sleeplessness
d. Mania

Copyright 2011. Wolters Kluwer Health | Lippincott Williams & Wilkins. Study Guide for Porths Essentials of Pathophysiology, Third Edition.
LWBK707-c45_p269-272.qxd 8/19/10 2:06 PM Page 269 Aptara Inc

45

CHAPTER
Structure and Function
of the Skin

SECTION I: LEARNING SECTION II: ASSESSING


OBJECTIVES YOUR UNDERSTANDING
1. Describe the protective functions of skin. Activity A Fill in the blanks.
2. Characterize the changes in a keratinocyte 1. The skin, also called the , is one
from its inception in the basal lamina to its of the largest organs and most versatile
arrival on the outer surface of the skin. organs of the body, accounting for roughly
16% of the bodys weight.
3. List the four specialized cells of the epidermis
and describe their functions. 2. Variations are found in the properties of the
skin, such as the of skin layers,
4. Describe the structure and function of the
the distribution of sweat glands, and the
dermis and subcutaneous layers of skin.
number and size of hair follicles.
5. Describe the following skin appendages and
3. The covers the body, and it is
their functions: sebaceous gland, eccrine
specialized in areas to form the various skin ap-
gland, apocrine gland, nails, and hair.
pendages: hair, nails, and glandular structures.
6. Characterize the skin in terms of sensory and
4. The top or surface layer of the skin, the
immune functions.
, consists of dead, keratinized
7. Describe the following skin rashes and cells.
lesions: macule, patch, papule, plaque, nod-
5. produce keratin, a complex pro-
ule, tumor, wheal, vesicle, bulla, and pustule.
tein that that forms the surface of the skin, is
8. Describe the characteristics and causes of also the structural protein of the hair, and nails.
blisters, calluses, and corns.
6. are pigment-synthesizing cells
9. Cite two physiologic explanations for that are located at or in the basal layer.
pruritus.
7. Exposure to the suns ultraviolet rays
10. Describe the causes and treatment of dry increases the production of ,
skin. causing tanning to occur.
11. State common variations found in dark skin. 8. cells are potent antigen-
presenting cells.
9. The is involved in skin disorders
that cause bullae or blister formation.

269
LWBK707-c45_p269-272.qxd 8/19/10 2:06 PM Page 270 Aptara Inc

270 UNIT 13 INTEGUMENTARY FUNCTION

10. The dermis supports the and 1. In the figure above, locate and label the
serves as its primary source of nutrition. following structures:
11. The receptors for touch, pressure, heat, cold, Nerve
and pain are widely distributed in the Sebaceous gland
.
Blood vessel
12. The layer of the dermis is Arrector pili muscle
supplied with free nerve endings that serve as
Dermis
nociceptors and thermoreceptors.
Sweat gland
13. sweat glands are simple tubular
Papillae
structures that originate in the dermis and
open directly to the skin surface. Epidermis

14. sweat glands open through a


hair follicle and are found primarily in the
axillae and groin.
15. Hair is a structure that is
pushed upward from the hair follicle.
16. The nails are hardened plates,
called fingernails and toenails, that protect
the fingers and toes and enhance dexterity.
17. A is a vesicle or fluid-filled
papule.
18. A is a hyperkeratotic plaque of
skin due to chronic pressure or friction.
19. are small, well-circumscribed,
conical, keratinous thickenings of the skin.
20. Dry skin, also called , may be a
natural occurrence, as in the drying of skin
associated with aging, or it may be sympto-
matic of underlying systemic disease or skin 2. In the figure above, locate and label the
disorder such as contract dermatitis. following structures:
Activity B Consider the following figures. Epidermis
Hair papilla
Dermis
Hair shaft
Arrector pili
Sebaceous gland
Keratinized cells
Hair follicle
Dermal blood vessels

Copyright 2011. Wolters Kluwer Health | Lippincott Williams & Wilkins. Study Guide for Porths Essentials of Pathophysiology, Third Edition.
LWBK707-c45_p269-272.qxd 8/19/10 2:06 PM Page 271 Aptara Inc

CHAPTER 45 STRUCTURE AND FUNCTION OF THE SKIN 271

Activity C Match the key terms in Column A 4. What is the relationship between melanin and
with their definitions in Column B. different colors of skin?

Column A Column B
1. Keratinocytes a. Consists of collagen
fibers and ground sub-
2. Merkel cells
stance
3. Keratinization b. Responsible for skin 5. Describe the structure and function of
sebaceous glands.
4. Epidermis color, tanning, and
protecting against
5. Papillary ultraviolet radiation
dermis
c. Outer layer of skin
6. Langerhans d. Produce a fibrous pro-
cells tein called keratin, 6. How does and itch differ from pain?
7. Dermis which is essential to
the protective function
8. Ruffini
e. Complex meshwork
corpuscles
of three-dimensional
9. Melanin collagen bundles inter-
connected with large
10. Reticular
elastic fibers and
dermis SECTION III: APPLYING
ground substance
f. Inner layer of skin YOUR KNOWLEDGE
g. Mechanoreceptors
Activity E Consider the scenario and answer
h. Immune cells the questions.
i. Transformation from
Case study: You are the nurse preparing an edu-
viable cells to the dead
cational event for the local chapter of the Daugh-
cells of the stratum
ters of the American Revolution (DAR). You have
corneum
been asked to speak on skin disorders.
j. Provide sensory
information a. What information would you include about
dark-skinned people?
Activity D Briefly answer the following.
1. What are the vital functions of the skin?

SECTION IV: PRACTICING


2. What are the layers of the epidermis? FOR NCLEX
Activity F Answer the following questions.
1. Among the skins protective functions is the
fact that it serves as an immunologic barrier.
3. How is it that a person with albinism cannot What cells detect foreign antigens?
synthesize melanin? a. The Langerhans cells
b. The Merkel cells
c. The keratinocytes
d. The melanocytes

Copyright 2011. Wolters Kluwer Health | Lippincott Williams & Wilkins. Study Guide for Porths Essentials of Pathophysiology, Third Edition.
LWBK707-c45_p269-272.qxd 8/19/10 2:06 PM Page 272 Aptara Inc

272 UNIT 13 INTEGUMENTARY FUNCTION

2. Match the cells of the epidermis with their 7. Fingernails and toenails, unlike hair, grow
description or function. continuously. The nail plate itself is nearly
transparent and acts as a window for viewing
Cell Description or Function
what?
1. Keratinocytes a. Thought to be neu- a. The amount of oxygen in the blood
roendocrine cells
2. Melanocytes b. The color of the blood in the subcuta-
b. Pigment-synthesizing neous tissue
3. Merkel cells cells
c. The health of the nail plate
4. Langerhans c. Replaces lost skin cells
d. The color of the stratum corneum
cells d. Immunologic cells
8. When a degeneration of the epidermal cells
3. The basement membrane separates the occurs, there is separation of the layers of the
epithelium from the underlying connective skin because of a disruption of the intercellu-
tissue. It is a major site of what is in skin lar junctions. When this occurs what is
disease? formed?
a. Melanocytes a. Lichenifications
b. Complement deposition b. Vesicles
c. The lamina lucida c. Petechiae
d. Type IV collagen d. Pressure ulcer
4. What is the pars reticularis characterized by? 9. Pruritis, or the itch sensation, is a by-product
a. Dendritic cells of almost all skin disorders. However, we can
b. Its color itch without having a skin disorder. Itch then
can be local or central in our bodies. Where
c. Three-dimensional collagen bundles
is it postulated that a central itch center
d. Its immunologic function exists?
5. Why is the subcutaneous tissue considered a. Pons
part of the skin? (Mark all that apply.) b. Medulla oblongata
a. Eccrine glands extend to this layer c. Somatosensory cortex
b. The keratinocytes are formed in the subcu- d. Sensory area of the cerebrum
taneous tissue
10. The first-line treatment for dry skin is
c. Skin diseases can involve the subcuta-
moisturizing agents. How do these agents
neous tissue
work?
d. The Merkel cells are formed in the subcu-
a. Decreasing pruritis
taneous tissue
b. Penetrating the lipid barrier of the skin
e. Deep hair follicles can be found in the sub-
cutaneous tissue c. Increasing transepidermal water loss
d. Repairing the skin barrier
6. Cerumen glands excrete a mixture that lubri-
cates the hair and skin. What is this mixture
called?
a. Sweat
b. Chalasia
c. Cerumen
d. Sebum

Copyright 2011. Wolters Kluwer Health | Lippincott Williams & Wilkins. Study Guide for Porths Essentials of Pathophysiology, Third Edition.
LWBK707-c46_p273-279.qxd 8/20/10 11:54 PM Page 273 Aptara

46

CHAPTER
Disorders of Skin
Integrity and Function

SECTION I: LEARNING 11. Describe the three types of ultraviolet radia-


tion and relate them to sunburn, aging skin
OBJECTIVES changes, and the development of skin cancer.
1. Describe common pigmentary disorders of 12. Describe the manifestations and treatment of
the skin. sunburn.
2. Relate the behavior of fungi to the 13. State the properties of an effective sunscreen.
production of superficial skin lesions associ-
14. Compare the tissue involvement in first-
ated with tinea or ringworm.
degree, second-degree full-thickness, and
3. State the cause and describe the appearance third-degree burns.
of impetigo and ecthyma.
15. State how the rule of nine is used in
4. Compare the viral causes, manifestations, determining the body surface area involved
and treatments of verrucae, herpes simplex, in a burn.
and herpes zoster lesions.
16. Cite the determinants for grading burn sever-
5. Compare acne vulgaris, acne conglobata, and ity using the American Burn Association
rosacea in terms of appearance and location classification of burns.
of lesions.
17. Describe the systemic complications of
6. Describe the pathogenesis of acne vulgaris burns.
and relate it to measures used in treating the
18. Describe the major considerations in
disorder.
treatment of burn injury.
7. Differentiate allergic and contact dermatitis
19. Cite two causes of pressure ulcers.
and atopic and nummular eczema.
20. Explain how shearing forces contribute to
8. Describe the differences and similarities
ischemic skin damage.
between erythema multiforme minor,
Stevens-Johnson syndrome, and toxic epider- 21. List four measures that contribute to the pre-
mal necrolysis. vention of pressure ulcers.
9. Define the term papulosquamous and use the 22. Describe the origin of nevi and state their
term to describe the lesions associated with relationship to skin cancers.
psoriasis, pityriasis rosea, and lichen planus.
23. Compare the appearance and outcome of
10. Relate the life cycle of Sarcoptes scabiei to the basal cell carcinoma, squamous cell
skin lesions seen in scabies. carcinoma, and malignant melanoma.

273
LWBK707-c46_p273-279.qxd 8/20/10 11:54 PM Page 274 Aptara

274 UNIT 13 INTEGUMENTARY FUNCTION

24. Differentiate a strawberry hemangioma of 9. Herpes is an acute, localized


infancy from a port-wine stain hemangioma vesicular eruption distributed over a
in terms of appearance and outcome. dermatomal segment of the skin.
25. Describe the manifestations and probable 10. is a disorder of the pilosebaceous
causes of diaper dermatitis, prickly heat, and unit.
cradle cap.
11. consists of a mixture of free
26. Describe the distinguishing features of rashes fatty acids, triglycerides, diglycerides, mono-
associated with the following infectious glycerides, sterol esters, wax esters, and
childhood diseases: roseola infantum, squalene.
rubeola, rubella, and varicella.
12. Noninflammatory acne lesions consist of
27. Characterize the physiologic changes of ; are plugs of
aging skin. material that accumulate in sebaceous
glands that open to the skin surface and
28. Describe the appearance of skin tags,
are pale, slightly elevated
keratoses, lentigines, and vascular lesions
papules with no visible orifice.
that are commonly seen in the elderly.
13. acne lesions consist of papules,
pustules, nodules, and, in severe cases, cysts.
SECTION II: ASSESSING 14. Hypersensitivity are usually
YOUR UNDERSTANDING characterized by epidermal edema with sepa-
ration of epidermal cells; they include irritant
Activity A Fill in the blanks. contact dermatitis, allergy contact dermatitis,
atopic and nummular eczema, urticaria, and
1. skin disorders include pigmen- drug-induced skin eruptions.
tary skin disorders, infectious processes, acne,
rosacea, papulosquamous dermatoses, allergic 15. dermatitis results from a cell-
disorders and drug reactions, and arthropod mediated, type IV hypersensitivity response
infestations. brought about by sensitization to an allergen.

2. is a genetic disorder in which 16. Acute immunologic is


there is complete or partial congenital commonly the result of an immunoglobulin
absence of pigment in the skin, hair, and eyes E-mediated immune reaction that usually
is found in all races. occurs within 1 hour of exposure to an
antigen.
3. are free-living, saprophytic
plantlike organisms, certain strains of which 17. drugs are usually responsible
are considered part of the normal skin flora. for localized contact dermatitis types of
rashes, whereas drugs cause
4. is a yeastlike fungus that is a generalized skin lesions.
normal inhabitant of the gastrointestinal
tract, mouth, and vagina. 18. dermatoses are a group of skin
disorders characterized by scaling papules
5. Primary infections are superficial and plaques.
skin infections such as impetigo or ecthyma.
19. is a relatively common chronic,
6. is common, superficial bacterial pruritic disease that involves inflammation
infection caused by staphylococci or group A and papular eruption of the skin and mucous
-hemolytic streptococci that appears as a membranes.
small vesicle or pustule or as a large bulla on
the face or elsewhere on the body. 20. Lichen simplex chronicus is a localized
lichenoid, pruritic dermatitis resulting from
7. is a deeper infection affecting rubbing and scratching.
the dermis and subcutaneous tissues.
21. A mite, Sarcoptes scabiei, which burrows into
8. and occur on the the epidermis, causes .
soles of the feet and palms of the hands,
respectively.

Copyright 2011. Wolters Kluwer Health | Lippincott Williams & Wilkins. Study Guide for Porths Essentials of Pathophysiology, Third Edition.
LWBK707-c46_p273-279.qxd 8/20/10 11:54 PM Page 275 Aptara

CHAPTER 46 DISORDERS OF SKIN INTEGRITY AND FUNCTION 275

22. , commonly referred to as 35. represent slow-growing


sunburn rays, are responsible for nearly capillary malformations that grow
all the skin effects of sunlight, including proportionately with a child and persist
photoagingthe wrinkles, pigmentary throughout life.
changes, dryness, and loss of skin tone that
36. is a form of contact dermatitis
occurs with and is enhanced by exposure to
that is caused by an interaction with several
sunlight.
factors, including prolonged contact of the
23. Some drugs are classified as skin with a mixture of urine and feces.
drugs because they produce an exaggerated
37. results from constant macera-
response to ultraviolet light when the drug is
tion of the skin because of prolonged
taken in combination with sun exposure.
exposure to a warm, humid environment.
24. is caused by excessive exposure
38. is a greasy crust or scale forma-
of the epidermal and dermal layers of the
tion on the scalp that is usually attributed to
skin to ultraviolet radiation, resulting in an
infrequent and inadequate washing of the
erythematous inflammatory reaction.
scalp.
25. are typically classified
according to the depth of involvement as Activity B Match the key terms in Column A
first-degree, second-degree, and third-degree. with their definitions in Column B.
26. victims often are confronted 1.
with hemodynamic instability, impaired res-
Column A Column B
piratory function, hypermetabolic response,
major organ dysfunction, and sepsis. 1. Herpes a. Pain that persists
simplex virus longer than 1 to
27. Pressure ulcers are lesions of
3 months after the
the skin and underlying structures caused by 2. Vitiligo
resolution of herpes
unrelieved pressure that impairs the flow of
3. Postherpetic zoster rash
blood and lymph.
neuralgia b. Responsible for cold
28. Another form of nevi, the , is sore
4. Dermatophytid
important because of its capacity to
c. Superficial mycoses
transform to malignant melanoma. 5. Melasma
d. Warts that are com-
29. Malignant melanoma is a malignant tumor of 6. Ecthyma mon benign papil-
the . lomas caused by
7. Verrucae
30. Severe, blistering sunburns in early childhood DNA-containing
8. Dermatophy- human
and intermittent intense sun exposures
toses papillomaviruses
contribute to increased susceptibility to
in young and middle-aged 9. Tinea capitis e. Sudden appearance
adults. of white patches on
10. Shingles
the skin
31. , which is a neoplasm of the
nonkeratinizing cells of the basal layer of the f. Darkened macules
epidermis, is the most common skin cancer on the face
in white-skinned people. g. Allergic reaction
during an acute
32. are the second most frequent
episode of a fungal
occurring malignant tumors of the outer
infection
epidermis.
h. Ulcerative form of
33. Pigmented represent abnormal impetigo
migration or proliferation of melanocytes
i. Caused by infection
seen in infants.
of herpes zoster
34. of infancy are generally benign j. Ringworm of the
vascular tumors produced by proliferation of scalp
the endothelial cells.

Copyright 2011. Wolters Kluwer Health | Lippincott Williams & Wilkins. Study Guide for Porth's Essentials of Pathophysiology, Third Edition.
LWBK707-c46_p273-279.qxd 8/20/10 11:54 PM Page 276 Aptara

276 UNIT 13 INTEGUMENTARY FUNCTION

2. 2. What is the mechanism of skin irritation


with a fungal infection?
Column A Column B
1. Verrucae a. Comedones form pri-
marily on the face and
2. Acne
neck and, to a lesser
vulgaris
extent, on the back,
3. Psoriasis chest, and shoulders 3. What are the types of tinea capitis and what
b. Disorder characterized are the mechanisms of irritation?
4. Acne
conglobata the development of
edematous wheals
5. Decubitus accompanied by
ulcers intense itching
6. Rosacea c. Thickening of the skin
4. What is the port of entry for cellulitis
associated with rosacea
7. Urticaria infections? What are the most common
d. Bed sore symptoms?
8. Nevi
e. Comedones, papules,
9. Rhinophyma pustules, nodules,
abscesses, cysts, and
10. Hyper-
scars occur on the
keratosis
back, buttocks, and
chest 5. Herpes simplex virus type-1 virus will have
f. Mole episodic recurrences. What is the mechanism
of recurrence? What are the signs and symp-
g. Erythema (flushing
toms of recurrence?
and redness) on the
central face and
across the cheeks,
nose, or forehead
h. Benign papillomas
caused by the DNA- 6. What are the factors believed to contribute to
containing human the development of acne?
papillomavirus
i. Increased epidermal
cell turnover with
marked epidermal
thickening
j. Chronic 7. What is atopic dermatitis and how does it
inflammatory skin affect adults differently than infants?
disease characterized
by circumscribed red,
thickened plaques
with an overlying
silvery-white scale
8. What is thought to be the cause of psoriasis?

Activity C Briefly answer the following.


1. What is the cause and symptomology of
albinism?

Copyright 2011. Wolters Kluwer Health | Lippincott Williams & Wilkins. Study Guide for Porths Essentials of Pathophysiology, Third Edition.
LWBK707-c46_p273-279.qxd 8/20/10 11:54 PM Page 277 Aptara

CHAPTER 46 DISORDERS OF SKIN INTEGRITY AND FUNCTION 277

9. What is the hypothesized mechanism of skin 2. The parents ask what specific complication can
damage brought about by UV-B rays? occur because of the burns their son has. The
nurses correct response would include what?

10. What are the steps recommended to protect a


patient from UV exposure?

SECTION IV: PRACTICING


FOR NCLEX
Activity E Answer the following questions.
11. Why are severe burns an immediate medical
emergency? 1. Match the skin disorder with its description.
Skin Disorder Description
1. Vitiligo A. Darkened macules on the
face
2. Albinism
B. Sudden appearance of
12. Describe the two main types of basal cell 3. Melasma white patches on the skin
carcinoma. C. A genetic disorder in
which there is complete
or partial congenital ab-
sence of pigment in the
skin, hair, and eyes; it is
found in all races.

2. Our bodies have, as endemic organisms, both


SECTION III: APPLYING yeast (Candida albicans) and molds. When a
YOUR KNOWLEDGE fungus invades the skin of our body, what is
used as a confirmatory diagnostic test?
Activity D Consider the scenario and answer
a. Potassium hydroxide preparations
the questions.
b. The Forest light
A 17-year-old patient with second- and third- c. Tinea preparations
degree burns on his trunk, arms, and neck is
brought to the emergency department, where he d. Sodium chloride preparations
is being stabilized for shipment to the nearest 3. Match the bacterial or viral skin infection
burn unit. with its preferred treatment.
1. The mother asks why her son must be sent to Skin Infection Preferred Treatment
another hospital. The nurse explains that the
client is at high risk for complications from 1. Impetigo a. Systemic antibiotics
his burns. What does the massive loss of skin 2. Ecthyma b. Bactroban or systemic
tissue predispose the client to? antibiotics
3. Cellulitis
c. Acyclovir
4. Verrucae d. Oral acyclovir
5. Herpes simplex e. Penciclovir cream
virus type-1 f. Oral and intravenous
6. Herpes simplex antibiotics
virus type-2 g. A keratolytic agent
7. Herpes zoster

Copyright 2011. Wolters Kluwer Health | Lippincott Williams & Wilkins. Study Guide for Porth's Essentials of Pathophysiology, Third Edition.
LWBK707-c46_p273-279.qxd 8/20/10 11:54 PM Page 278 Aptara

278 UNIT 13 INTEGUMENTARY FUNCTION

4. Acne vulgaris is typically an infection in the 10. What skin disease manifests with lesions on
adolescent population. What topical agent the skin and oral lesions that look like milky
used in the treatment of acne is both an anti- white lacework?
bacterial and a comedolytic? a. Eczema
a. Alcohol b. Psoriasis
b. Benzoyl peroxide c. Lichen planus
c. Bactroban d. Pityriasis rosea
d. Resorcinol
11. Scabies infections are caused by mites that
5. Rosacea is a chronic inflammatory process burrow under the skin. They are usually eas-
that occurs in middle-aged and older adults. ily treated by bathing with a mite-killing
What are common manifestations of rosacea? agent and leaving it on for 12 hours. When
(Mark all that apply.) scabies are resistant to the mite-killing agent,
a. Swelling of the eyelid what oral drug is prescribed?
b. Heat sensitivity a. Clindamycin
c. Burning eyes b. Interferon B
d. Telangiectasia c. Potassium hydroxide
e. Erythema d. Ivermectin

6. Allergic contact dermatitis is a common inflam- 12. Pressure ulcers can occur quickly in the
mation of the skin. It produces lesions in the elderly and in those who are immobile. What
affected areas. What do these lesions look like? is a method for preventing pressure ulcers?
a. Papules a. Preventing dehydration
b. Papulosquamous pustules b. Frequent position changes
c. Vesicles c. Use of water-based skin moisturizers
d. Ulcers d. Infrequent changing of incontinent clients

7. Atopic dermatitis, or eczema, occurs at all 13. Nevi are benign tumors of the skin. There is
ages and in all races. What happens in black- one type of nevi that is important because of
skinned people who have eczema? its capacity to transform to malignant
melanoma. What type of nevus is this?
a. Hyperpigmentation of skin
a. Nevocellular
b. Papules cover the area affected
b. Compound nevi
c. Erythema is a prominent symptom
c. Dysplastic
d. Loss of pigmentation from lichenified skin
d. Dermal
8. In severe Stevens-Johnson syndrome and
toxic epidermal necrolysis, hospitalization is 14. Malignant melanomas are metastatic tumors
required. When large areas of the skin are of the skin. In the past decades the incidence
lost, what intravenous medication may speed of malignant melanoma has grown. This is
up the healing process? related to more exposure to UV light, such as
tanning salons. What are risk factors for
a. Immunoglobulin
developing malignant melanoma?
b. Broad-spectrum antibiotics
a. Freckles across the bridge of the nose
c. Diflucan
b. Blistering sunburns after age 20
d. Corticosteroids
c. Palmar nevi
9. What disease has primary lesions that have a d. Presence of actinic keratoses
silvery scale over thick red plaques?
a. Pityriasis rosea
b. Psoriasis vulgaris
c. Lichen planus
d. Lichen simplex chronicus

Copyright 2011. Wolters Kluwer Health | Lippincott Williams & Wilkins. Study Guide for Porths Essentials of Pathophysiology, Third Edition.
LWBK707-c46_p273-279.qxd 8/20/10 11:54 PM Page 279 Aptara

CHAPTER 46 DISORDERS OF SKIN INTEGRITY AND FUNCTION 279

15. Basal cell carcinoma is the most common 18. Rubella, or 3-day measles, is a childhood dis-
skin cancer in white-skinned people. The ease caused by a togavirus. Because rubella
treatment goal that is most important is elim- can be easily transmitted and because it is
ination of the lesion, but it is also important dangerous to the fetus if contracted by preg-
to maintain the function and cosmetic effect. nant women early in their gestational period,
What treatment is used for basal cell immunization is required. What type of vac-
carcinoma? cine is the rubella vaccine?
a. Curettage with electrodesiccation a. Attenuated virus vaccine
b. Systemic chemotherapy b. Antibody/antigen vaccine
c. Topical chemotherapy c. Dead-virus vaccine
d. Simple radiographic radiation d. Live-virus vaccine
16. Squamous cell carcinoma in light-skinned 19. Lentigines are skin lesions common in the
people is a red, scaling, keratotic, slightly ele- elderly. A type of lentigines is tan to brown in
vated lesion with an irregular border, usually color with benign spots. Lentigines are
with a shallow chronic ulcer. How do they removed because they are considered precur-
appear in black-skinned people? sors to skin cancer. How are lentigines
a. Keratotic lesions with rolling, irregular removed?
borders a. Cryotherapy
b. Hyperpigmented nodules b. Chemotherapy
c. Hypopigmented nodules c. Bleaching agents
d. Lichenous plaques with silvery scales d. Curettage
17. Hemangiomas of infancy are small, red
lesions that are noticed shortly after birth
and grow rapidly. What is the treatment of
choice for hemangiomas of infancy?
a. Surgical excision
b. Laser surgery
c. No treatment
d. Chemotherapy

Copyright 2011. Wolters Kluwer Health | Lippincott Williams & Wilkins. Study Guide for Porth's Essentials of Pathophysiology, Third Edition.
LWBK707-Ans_p280-408.qxd 9/2/10 8:02 AM Page 280 Aptara Inc

Answer Key

CHAPTER 1 11. microfilaments


12. peripheral
13. epithelial, connective, muscle, neuronal
SECTION II: ASSESSING YOUR 14. ion
UNDERSTANDING 15. muscle, neural
Activity A Activity B
1. Cytoplasm 1.
2. Eukaryotic, prokaryotic 1. b 2. g 3. f 4. j 5. d
3. DNA, RNA, proteins 6. a 7. e 8. g 9. i 10. c
4. Protein
5. Rough 2.
6. Golgi 1. j 2. f 3. b 4. g 5. d
7. Lysosomes 6. e 7. h 8. i 9. c 10. a
8. Peroxides
Activity C
9. Respiration, ATP
10. Microtubules 1.

Hydrophilic polar head

Cholesterol
Extracellular
molecule
Pore fluid
Carbohydrate
Hydrophobic Glycoprotein
fatty acid chain
Glycolipid

Phospholipids:
polar head
(hydrophilic)

Fatty acid tails


(hydrophobic)
Cytosol
Peripheral
Channel protein
Filaments of protein
cytoskeleton Transmembrane Integral
Cholesterol protein proteins
LWBK707-Ans_p280-408.qxd 9/2/10 8:02 AM Page 281 Aptara Inc

ANSWER KEY 281

Activity D adipose tissue is composed of cells that contain multi-


ple droplets of fat and numerous mitochondria.
1. In ischemia and hypoxia (an anoxia), the cells
We have deposits of brown fat when we are born
do not receive enough oxygen. As a result, the
but they decrease over time. White fat is the kind
electron transport chain cannot pass electrons from
we have most of and it is what we add to our body
complex to complex. Proton pumping slows or is
when we gain weight.
halted and the proton gradient decreases, resulting
in a decreased production or a complete lack of
ATP. With no ATP, the cell cannot maintain normal
SECTION IV: PRACTICING FOR NCLEX
functioning (e.g., membrane potential, transport) Activity F
and begins to malfunction. 1. Answer: a
2. Individual cells produce extracellular matrix pro- RATIONALE: Rough ER is studded with ribosomes
teins that form a basement membrane where cells attached to specific binding sites on the membrane.
can form anchors. Cells will then form connections Proteins produced by the rough ER are usually
between each other via cell junctions (tight, gap, destined for incorporation into cell membranes
desmosome, hemidesmosome). This interaction and lysosomal enzymes or for exportation from
between cytoskeletal elements, the basement mem- the cell. The rough ER segregates (rather than
brane, and cellular adhesion is the basis for tissue combines) these proteins from other components
formation. of the cytoplasm and modifies their structure for
3. First messengers can be neurotransmitters, protein a specific function. Rough ER does not transport
hormones and growth factors, steroids, and/or anything through the cell membrane. Rough ER is
other chemical messengers. They will bind to recep- studded with ribosomes; it does not destroy them.
tors either on the cell membrane (hydrophilic first 2. Answer: b
messengers) or in the cytoplasm (hydrophobic RATIONALE: Recent data suggest that the Golgi
first messengers). The activation of a receptor via apparatus has yet another function: it can receive
first messenger results in the activation of a second proteins and other substances from the cell surface
messenger. Cell surface receptors are transmembrane by a retrograde transport mechanism. Golgi bodies
proteins that will activate an array of second mes- do not produce bile. They produce secretory, not
sengers (cAMP, G proteins, and tyrosine kinases) excretory, granules and they produce large carbo-
that will have direct effects on membrane potential hydrate molecules rather than small ones.
or a host of other cellular functions. Activation of 3. Answer: c
an intracellular receptor involves the activation of a RATIONALE: Although GM2 ganglioside
transcription factor that will directly influence the accumulates in many tissues, such as the heart, liver,
expression of a gene product. The gene product will and spleen, its accumulation in the nervous system
then have an effect on cellular function. and retina of the eye causes the most damage.
4. Endocytosis is the process of bringing in large mol- 4. Answer: d
ecules or substances to a cell. Receptor-mediated RATIONALE: They do not make energy, but they
endocytosis is triggered by a specific ligand. The extract it from organic compounds. Proteasomes are
inflammatory system contains cells (macrophages, small organelles composed of protein complexes
neutrophils) that will endocytose dead cell that are thought to be present in both the
material, bacteria, or foreign material. This process cytoplasm and the nucleus. They are not formed by
is known as phagocytosis. Exocytosis is the release of mitochondria. Mitochondria contain their own
large quantities of material, such as the exocytosis DNA and ribosomes and are self-replicating.
of a neurotransmitter. 5. Answer: a
RATIONALE: The cell membrane is often called the
SECTION III: APPLYING YOUR KNOWLEDGE plasma membrane. The nuclear membrane is
Activity E another type of membrane within the cell. The
cell membrane provides receptors for hormones
1. In our bodies, fat is stored in tissue called adipose
and other biologically active substances; it is not a
tissue. Adipose tissue is a special form of connective
receptor membrane. A main structural component
tissue, so it helps to connect different types of tissue
of the membrane is its lipid bilayer. It is not a
in our body to each other. Adipose cells contain big
bilayer membrane.
empty spaces so they can store large quantities of
6. Answer: b
triglycerides and are the largest storage spaces of
RATIONALE: At the membrane of the cell nucleus
energy in the body. The subcutaneous fat we store
both thyroid and steroid hormones cross into the
helps to shape our body. It also helps to insulate our
cell nucleus itself where they influence DNA activ-
body because fat is a poor conductor of heat.
ity. Ion-channellinked receptors transiently open
Adipose tissue exists in two forms: unilocular and
or close ion channels. Thyroid and steroid
multilocular. Unilocular (white) adipose tissue is com-
hormones act within the cell nucleus to increase
posed of cells in which the fat is contained in a single,
transcription of mRNA to alter cell function.
large droplet in the cytoplasm. Multilocular (brown)

Copyright 2011. Wolters Kluwer Health | Lippincott Williams & Wilkins. Study Guide for Porths Essentials of Pathophysiology, Third Edition.
LWBK707-Ans_p280-402.qxd 8/20/10 6:38PM Page 282 Aptara Inc

282 ANSWER KEY

7. Answer: c 12. Answer: cytoplasm


RATIONALE: Each of the two pyruvate molecules RATIONALE: When seen under a light microscope,
formed in the cytoplasm from one molecule of three major components of the cell become
glucose yields another molecule of ATP, which is a evident: the nucleus, the cytoplasm, and the cell
special carrier for cellular energy. FAD, or flavin membrane.
adenine dinucleotide, is a coenzyme of protein 13. Answer: jaundice
metabolism that accepts electrons and is reduced. RATIONALE: When bilirubin collects within the
NADH  H is an end product of glycolysis. The cells, they take on a yellowish color, which is
electron transport chain oxidizes NADH  H and called jaundice.
FADH2 and donates the electrons to oxygen, 14. Answers: a, c, d
which is reduced to water. RATIONALE: The human body has several means of
8. Answer: d transmitting information between cells. These
RATIONALE: Active transport is what happens when mechanisms include direct communication
cells use energy to move ions against an electrical between adjacent cells through gap junctions,
or chemical gradient. Passive transport is another autocrine and paracrine signaling, and endocrine
term for diffusion. There is no such thing as neutral or synaptic signaling. There is no such thing as
transport. Cotransport is when the sodium ion and express communication between cells.
the solute are transported in the same direction. 15. Answers: b, c, d
9. Answer: a RATIONALE: Nondividing cells, such as neurons and
RATIONALE: Four categories of tissue exist: (1) epithe- skeletal and cardiac muscle cells, have left the cell
lium, (2) connective (supportive) tissue, (3) mus- cycle and are not capable of mitotic division in
cle, and (4) nerve. Binding, connecting, and postnatal life. The cells that produce mucous are
exothelium tissue are not categories of tissue. capable of mitotic division. Smooth muscle is
10. Answer: b often called involuntary muscle because it contracts
RATIONALE: These glands are ductless and produce spontaneously or through activity of the
secretions (i.e., hormones) that move directly into autonomic nervous system.
the bloodstream. Exocrine glands retain their con- 16. Answer: involuntary
nection with the surface epithelium from which RATIONALE: Three types of muscle tissue exist:
they originated. This connection takes the form of skeletal, cardiac, and smooth. Smooth muscle
epithelium-lined tubular ducts through which the is often called involuntary muscle because it con-
secretions pass to reach the surface. Exocytosis tracts without the person willing it to contract.
occurs when part of the cell membrane ruptures to
release particles that are too large to pass through
the cell membrane. These cells are ductless, but do CHAPTER 2
not necessarily secrete their contents into the
bloodstream. SECTION II: ASSESSING YOUR
11. Answer: c UNDERSTANDING
RATIONALE: Thin and thick filaments are the two
Activity A
types of muscle fibers that are responsible for mus-
cle contraction. The thin filaments are composed 1. size, number, and type
primarily of actin and the thick filaments are com- 2. size
posed of myosin. During muscle contraction, the 3. atrophy
thick myosin and thin actin filaments slide over 4. increase
each other, causing shortening of the muscle fiber, 5. physiologic hypertrophy
although the length of the individual thick and 6. hyperplasia
thin filaments remains unchanged. When 7. compensatory
activated by ATP, the cross-bridges swivel in a fixed 8. Pathologic, nonphysiologic
arc, much like the oars of a boat, as they become 9. Metaplasia
attached to the actin filament. During contraction, 10. irritation, inflammation
each cross-bridge undergoes its own cycle of 11. dysplasia
movement, forming a bridge attachment and 12. cancer
releasing it, and moving to another site where the 13. accumulations
same sequence of movement occurs. This pulls the 14. Free
thin and thick filaments past each other. The cal- 15. Hypoxia
ciumcalmodulin complex is in smooth muscle. It 16. swelling, fatty
binds to and activates the myosin-containing 17. Necrosis
thick filaments, which interact with actin. 18. calcium
19. coagulation

Copyright 2011. Wolters Kluwer Health | Lippincott Williams & Wilkins. Study Guide for Porths Essentials of Pathophysiology, Third Edition.
LWBK707-Ans_p280-402.qxd 8/20/10 6:38PM Page 283 Aptara Inc

ANSWER KEY 283

Activity B from the bodies of dead or dying cells as well as


from the circulation and interstitial fluid. As tissues
die, the calcium crystallizes and deposits form.
2. (1) Injury from physical agents, (2) radiation injury,
Nucleus (3) chemical injury, (4) injury from biologic agents,
and (5) injury from nutritional imbalances.
Basement 3. The toxicity of lead is related to its multiple
Normal cellsPhysiologic membrane biochemical effects. It has the ability to inactivate
enzymes, compete with calcium for incorporation
into bone, and interfere with nerve transmission
and brain development. Lead exposure in children
has been demonstrated to result in neurobehavioral
AtrophyPathologic and cognitive deficits.
4. The three major mechanisms of cellular damage
are free radical formation, hypoxia and ATP deple-
tion, and disruption of intracellular calcium home-
ostasis. Multiple pathologies, be it mechanical,
chemical, biological, or blunt force, will result in a
combination of these mechanisms being activated.
HypertrophyBoth 5. Oxidative stress leads to the oxidation of cell com-
ponents, activation of signal transduction
pathways, and changes in gene and protein expres-
sion. DNA modification and damage can occur
because of oxidative stress. In addition, mitochon-
drial DNA as a target of oxidation and subsequent
cause of mitochondrial dysfunction may be the
HyperplasiaBoth
cause of diseases.
6. As oxygen concentrations fall, oxidative metabolism
slows down. To make ATP, the cell reverts to anaer-
obic metabolism. With a decrease in ATP, the ion
distribution is altered and cells will swell. The prod-
uct of anaerobic metabolism is lactic acid, and as
lactic acid accumulates, the pH falls. Low pH will
change protein conformation, resulting in total loss
MetaplasiaBoth of enzyme function.
7. Two basic pathways for apoptosis are the extrinsic
pathway, which is death receptordependent and
is under cellular control, and the intrinsic pathway,
which is death receptorindependent and results
from injury. The execution phase of both pathways
is initiated by proteolytic enzymes called caspases.

SECTION III: APPLYING YOUR KNOWLEDGE


Activity E
1. In the nervous system, lead toxicity is characterized
DysplasiaPathologic by demyelination of cerebral and cerebellar white
matter and death of cortical cells. The demyelination
results in loss of action potential generation and
Activity C decreased neurotransmitter release.

1. e 2. b 3. e 4. g 5. a
SECTION IV: PRACTICING FOR NCLEX
6. c 7. f 8. i 9. h 10. j
Activity D Activity F
1. The pathogenesis of dystrophic calcification 1. Answer: a
RATIONALE: There are numerous molecular mecha-
involves the intracellular and/or extracellular
formation of crystalline calcium phosphate. The nisms mediating cellular adaptation, including
components of the calcium deposits are derived factors produced by other cells or by the cells
themselves. These mechanisms depend largely on

Copyright 2011. Wolters Kluwer Health | Lippincott Williams & Wilkins. Study Guide for Porths Essentials of Pathophysiology, Third Edition.
LWBK707-Ans_p280-402.qxd 8/20/10 6:38PM Page 284 Aptara Inc

284 ANSWER KEY

signals transmitted by chemical messengers that lead when you read the newspaper. You have to
exert their effects by altering gene function. In work directly with ore to be exposed to toxic levels
general, the genes expressed in all cells fall into of lead. Walking through part of a mine on a field
two categories: housekeeping genes that are nec- trip is not a contributing factor to lead poisoning.
essary for normal function of a cell, and genes that 6. Answer: b
determine the differentiating characteristics of a RATIONALE: In a genetic disorder called xeroderma
particular cell type. In many adaptive cellular pigmentosum, an enzyme needed to repair
responses, the expression of the differentiation sunlight-induced DNA damage is lacking. This
genes is altered, whereas that of the housekeeping autosomal recessive disorder is characterized by
genes remains unaffected. Thus, a cell is able to extreme photosensitivity and a 2000-fold increased
change size or form without compromising its risk of skin cancer in sun-exposed skin. Exposure
normal function. Once the stimulus for adaptation to sun causes the skin to toughen and become
is removed, the effect on expression of the differ- leathery feeling, but not in patches of pink
entiating genes is removed and the cell resumes its pigmented skin. Vitiligo is a benign acquired skin
previous state of specialized function. disease of unknown cause, consisting of irregular
2. Answer: b patches of various sizes totally lacking in pigment
RATIONALE: Compensatory hypertrophy is the and often having hyperpigmented borders. It can
enlargement of a remaining organ or tissue after a appear in the skin of any race and is not scaly.
portion has been surgically removed or rendered Photosensitivity is a sign of xeroderma pigmento-
inactive. The body does not enlarge its major sum but this disease increases, not decreases, the
organs during times of malnutrition. Gene expres- persons risk of skin cancer.
sion, not actin expression, stimulates the body to 7. Answer: c
increase the muscle mass of the heart. Hypertrophy RATIONALE: Lightning and high-voltage wires that
is not a progressive decrease in the size of anything. carry several thousand volts produce the most
3. Answer: c severe damage. In electrical injuries, the body acts
RATIONALE: Metastatic calcification occurs in as a conductor of the electrical current.
normal tissues as the result of increased serum cal- 8. Answer: d
cium levels (hypercalcemia). Almost any condition RATIONALE: Injury from freezing probably results
that increases the serum calcium level can lead to from a combination of ice crystal formation and
calcification in inappropriate sites such as the vasoconstriction. The decreased blood flow leads
lung, renal tubules, and blood vessels. The major to capillary stasis and arteriolar and capillary
causes of hypercalcemia are: hyperparathyroidism, thrombosis. Edema results from increased capillary
either primary or secondary to phosphate permeability. Exposure to low-intensity heat (43C
retention in renal failure; increased mobilization to 46C), such as occurs with partial-thickness
of calcium from bone as in Paget disease, cancer burns and severe heat stroke, causes cell injury by
with metastatic bone lesions, or immobilization; inducing vascular injury. The process of warming
and vitamin D intoxication. Diabetes mellitus and tissue that has been frozen or partially frozen
hypoparathyroidism do not cause hypercalcemia; causes pain. If the pain is bad enough, then med-
therefore, they cannot be a cause of metastatic ication to control the pain is given. Health team
calcification. members are always concerned about giving pain
4. Answer: d medication to someone who might be an addict.
RATIONALE: The main source of methyl mercury Asking if this is the first time this person has
exposure is from consumption of long-lived fish, had an injury induced by the cold is appropriate
such as tuna and swordfish. Although there is mer- when taking a health history. However, pointing
cury in amalgam fillings, the amount of mercury out that it is obvious you are a homeless person
vapor given off by the fillings is very small. Most is not an appropriate remark for the nurse to make.
thermometers today are made without mercury. Also not appropriate is wondering when it will
The same holds true for most blood pressure happen again.
machines. Lead in paint is a concern, not mercury. 9. Answer: a
5. Answer: a RATIONALE: Destructive changes occur in small
RATIONALE: Children are exposed to lead through blood vessels such as the capillaries and venules.
ingestion of peeling lead paint, by breathing dust Acute reversible necrosis is represented by such
from lead paint (e.g., during remodeling), or from disorders as radiation cystitis, dermatitis, and diar-
playing in contaminated soil. The lead danger to rhea from enteritis. More persistent damage can be
potters is from the ceramic glaze before it is fired. attributed to acute necrosis of tissue cells that are
You do not have to keep children away from not capable of regeneration and chronic ischemia.
everything ceramic. Newsprint contains lead, but Hunger is not a sign of radiation injury, nor are
you are not exposed to a significant amount of muscle spasms.

Copyright 2011. Wolters Kluwer Health | Lippincott Williams & Wilkins. Study Guide for Porths Essentials of Pathophysiology, Third Edition.
LWBK707-Ans_p280-402.qxd 8/20/10 6:38PM Page 285 Aptara Inc

ANSWER KEY 285

10. Answer: b continue to produce injury. Among the nutritional


RATIONALE: Gram-negative bacilli release endotox- factors that contribute to cell injury are excesses and
ins that cause cell injury and increased capillary deficiencies of nutrients, vitamins, and minerals.
permeability. Certain bacteria excrete elaborate 14. Answers: a, b, c, d
exotoxins that interfere with cellular production RATIONALE: Many drugsalcohol, prescription
of ATP. Gram-negative bacilli do not disrupt a drugs, over-the-counter drugs, and street drugs
cells ability to replicate. Many gram-negative are capable of directly or indirectly damaging tis-
bacilli cause harm to cells. sues. Ethyl alcohol can harm the gastric mucosa,
11. Answer: atrophy liver, developing fetus, and other organs. Antineo-
RATIONALE: When confronted with a decrease in plastic (anticancer) and immunosuppressant drugs
work demands or adverse environmental can directly injure cells. Other drugs produce
conditions, most cells are able to revert to a metabolic end products that are toxic to cells.
smaller size and a lower and more efficient level Acetaminophen, a commonly used over-the-
of functioning that is compatible with survival. counter analgesic drug, is detoxified in the liver,
This decrease in cell size is called atrophy. where small amounts of the drug are converted
12. Answers: 1-a, 2-d, 3-b, 4-c to a highly toxic metabolite.
RATIONALE: Pigments are colored substances that
may accumulate in cells. They can be endogenous
(i.e., arising from within the body) or exogenous CHAPTER 3
(i.e., arising from outside the body). Icterus, also
called jaundice, is characterized by a yellow discol- SECTION II: ASSESSING YOUR
oration of tissue due to the retention of bilirubin, UNDERSTANDING
an endogenous bile pigment. This condition may
Activity A
result from increased bilirubin production from
red blood cell destruction, obstruction of bile pas- 1. Inflammation
sage into the intestine, or toxic diseases that affect 2. rubor, tumor, calor, dolor
the livers ability to remove bilirubin from the 3. systemic
blood. Lipofuscin is a yellow-brown pigment that 4. Acute, chronic
results from the accumulation of the indigestible 5. vascular, cellular
residues produced during normal turnover of cell 6. leukocytosis
structures (Fig. 2-3). The accumulation of lipofus- 7. Monocyte/macrophages
cin increases with age and is sometimes referred 8. Vascular
to as the wear-and-tear pigment. It is more com- 9. leukocytes
mon in heart, nerve, and liver cells than other 10. cell-to-cell
tissues and is seen more often in conditions asso- 11. cell migration
ciated with atrophy of an organ. One of the most 12. chemokines
common exogenous pigments is carbon in the 13. metabolic burst
form of coal dust. In coal miners or persons 14. coagulation, complement
exposed to heavily polluted environments, the 15. dilation, permeability
accumulation of carbon dust blackens the lung 16. eicosanoid
tissue and may cause serious lung disease. The for- 17. prostaglandins
mation of a blue lead line along the margins of 18. cyclooxygenase
the gum is one of the diagnostic features of lead 19. omega-3 fatty acids
poisoning. Melanin is a black or dark brown pig- 20. Complement
ment that occurs naturally in the hair, skin, and 21. kinin, smooth muscle, pain
iris and choroid of the eye. 22. Tumor necrosis factor-
13. Answers: 1-b, 2-c, 3-a, 4-d 23. superoxide, hydrogen peroxide, hydroxyl
RATIONALE: Cell injury can be caused by a number 24. endothelial cell damage
of agents, including physical agents, chemicals, 25. exudates
biologic agents, and nutritional factors. Among 26. penetrate deeply, spread rapidly
the physical agents that generate cell injury are 27. CRP (C-reactive protein)
mechanical forces that produce tissue trauma, 28. Fever
extremes of temperature, electricity, radiation, and 29. body temperature
nutritional disorders. Chemical agents can cause 30. arteriovenous (AV) shunts
cell injury through several mechanisms: they can Activity B
block enzymatic pathways, cause coagulation of 1. The figure depicts the cyclooxygenase and lipoxy-
tissues, or disrupt the osmotic or ionic balance of genase pathways and sites where the corticosteroids
the cell. Biologic agents differ from other injurious and nonsteroidal anti-inflammatory drugs (NSAIDs)
agents in that they are able to replicate and exert their action. Inflammation is essential to the

Copyright 2011. Wolters Kluwer Health | Lippincott Williams & Wilkins. Study Guide for Porths Essentials of Pathophysiology, Third Edition.
LWBK707-Ans_p280-402.qxd 8/20/10 6:38PM Page 286 Aptara Inc

286 ANSWER KEY

first phase of wound healing, and immune mecha- endothelium by injurious stimuli. The third pattern
nisms prevent infections that impair wound is a delayed hemodynamic response in which the
healing. Among the conditions that impair inflam- increased permeability begins after a delay of 2 to
mation and immune function is administration of 12 hours, lasts for several hours or even days, and
corticosteroid drugs. Release of arachidonic acid involves venules as well as capillaries. A delayed
by phospholipases initiates a series of complex reac- response often accompanies radiation types of
tions that lead to the production of inflammatory injuries.
mediators. The cyclooxygenase pathway culminates 4. Phagocytosis involves three distinct steps: (1)
in the synthesis of prostaglandins, and the lipoxy- recognition and adherence, (2) engulfment, and
genase pathway culminates in the synthesis of the (3) intracellular killing. Phagocytosis is initiated
leukotrienes. Aspirin and the NSAIDs reduce by recognition and binding of particles by specific
inflammation by inactivating the first enzyme in receptors on the surface of phagocytic cells.
the cyclooxygenase pathway for prostaglandin Microbes can be bound directly to the membrane
synthesis. of the phagocytic cells by several types of pattern
Activity C recognition receptors or indirectly by receptors that
recognize microbes coated with carbohydrate bind-
1. c 2. a 3. i 4. f 5. j
ing-lectins, antibody, and/or complement. Endocyto-
6. g 7. b 8. h 9. d 10. e
sis is accomplished through cytoplasmic extensions
Activity D that surround and enclose the particle in a
Margination and adhesion S transmigration across membrane-bound phagocytic vesicle. Intracellular
endothelium S chemotaxis S activation and killing of pathogens is accomplished through several
phagocytosis. mechanisms, including toxic oxygen and nitrogen
Activity E products, lysozymes, proteases, and defensins.
5. Mediators can be classified by function: (1) those
1. These signs are rubor (redness), tumor (swelling), with vasoactive and smooth muscleconstricting
calor (heat), and dolor (pain). The rubor is the result properties such as histamine, arachidonic acid
of increased blood flow due to histamine release. metabolites, and platelet-activating factor; (2)
The tumor, or swelling, is due to an increased per- plasma proteases that activate members of the com-
meability of blood vessels due to histamine and plement system, coagulation factors of the clotting
other long-term vasoactive mediators. The calor, or cascade, and vasoactive peptides of the kinin sys-
heat, is the result of increased perfusion of the tis- tem; (3) chemotactic factors such as complement
sues at the wound site. Dolor, or pain, is due to fragments and chemokines; and (4) reactive
bradykinin, prostaglandins, and histamines effects molecules and cytokines liberated from leukocytes,
on sensory nerve endings. which when released into the extracellular environ-
2. Acute inflammation is the early (almost immediate) ment can affect the surrounding tissue and cells.
reaction of local tissues and their blood vessels to 6. The types of chronic inflammation are nonspecific
injury. It typically occurs before adaptive immunity and granulomatous. Nonspecific chronic inflam-
becomes established and is aimed primarily at mation involves a diffuse accumulation of
removing the injurious agent and limiting the macrophages and lymphocytes at the site of injury.
extent of tissue damage. Acute inflammation can be Ongoing chemotaxis causes macrophages to
triggered by a variety of stimuli, including infections, infiltrate the inflamed site, where they accumulate
immune reactions, blunt and penetrating trauma, owing to prolonged survival and immobilization.
physical or chemical agents, and tissue necrosis These mechanisms lead to fibroblast proliferation,
from any cause. In contrast to acute inflammation, with subsequent scar formation that in many
chronic inflammation is self-perpetuating and may cases replaces the normal connective tissue or the
last for weeks, months, or even years. It may develop functional parenchymal tissues of the involved
as the result of a recurrent or progressive acute structures. A granulomatous lesion is a small, 1- to
inflammatory process or from low-grade, smoldering 2-mm lesion in which there is a massing of epithe-
responses that fail to evoke an acute response. lioid cells surrounded by lymphocytes. Granuloma-
3. The first pattern is an immediate transient response, tous inflammation is associated with foreign bodies
which occurs with minor injury. It develops rapidly and with microorganisms that are poorly digested
after injury and is usually reversible and of short and usually not easily controlled by other
duration. Typically, this type of leakage affects inflammatory mechanisms.
venules 20 to 60 mm in diameter, leaving capillar- 7. The acute-phase response includes changes in the
ies and arterioles unaffected. The second pattern concentrations of plasma proteins, skeletal muscle
is an immediate sustained response, which occurs catabolism, negative nitrogen balance, elevated
with more serious types of injury and continues for erythrocyte sedimentation rate, and increased
several days. It affects all levels of the microcircula- numbers of leukocytes. These responses are gener-
tion and is usually due to direct damage of the ated by the release of cytokines that affect the

Copyright 2011. Wolters Kluwer Health | Lippincott Williams & Wilkins. Study Guide for Porths Essentials of Pathophysiology, Third Edition.
LWBK707-Ans_p280-402.qxd 8/20/10 6:38PM Page 287 Aptara Inc

ANSWER KEY 287

thermoregulatory center in the hypothalamus 4. Answer: c


to produce fever. The metabolic changes provide RATIONALE: Infection impairs all dimensions of
amino acids that can be used in the immune wound healing. It prolongs the inflammatory
response and for tissue repair. In general, the acute- phase, impairs the formation of granulation tissue,
phase response serves to coordinate the various and inhibits proliferation of fibroblasts and
changes in body activity to enable an optimal deposition of collagen fibers. All wounds are con-
host response. taminated at the time of injury. Although body
defenses can handle the invasion of microorganisms
SECTION III: APPLYING YOUR KNOWLEDGE at the time of wounding, badly contaminated
wounds can overwhelm host defenses. Trauma and
Activity F
existing impairment of host defenses also can con-
1. After an injury the body initiates what is called the tribute to the development of wound infections.
inflammatory response. This means the body sends 5. Answer: c
cells and fluids that are specific to destroying infec- RATIONALE: Histamine causes dilation of arterioles
tious organisms and healing the injury to the site and increases the permeability of venules. It acts at
of the wound. What you are seeing on the bandages the level of the microcirculation by binding to his-
is a serous exudate from the plasma in the circula- tamine 1 (H1) receptors on endothelial cells and is
tory system that has responded to the burn injury. considered the principal mediator of the immediate
2. The bodys response to an injury activates many transient phase of increased vascular permeability
different types and kinds of cells. This response is in the acute inflammatory response. Arachidonic
called the acute-phase response, and some of the acid is a 20-carbon unsaturated fatty acid found in
cells that are released during this response act on phospholipids of cell membranes. Release of arachi-
the central nervous system. Their actions can cause donic acid by phospholipases initiates a series of
outward manifestations of their work such as complex reactions that lead to the production of
anorexia, somnolence, and malaise. the eicosanoid family of inflammatory mediators
(prostaglandins, leukotrienes, and related metabo-
SECTION IV: PRACTICING FOR NCLEX lites). Fibroblasts and cytokines are not the princi-
Activity G pal mediator of the transient phase of an acute
inflammatory response.
1. Answer: a 6. Answer: a
RATIONALE: The classic description of inflammation RATIONALE: The most prominent systemic manifesta-
has been handed down through the ages. In the first tions of inflammation include the acute-phase
century AD, the Roman physician Celsus described response, alterations in white blood cell count
the local reaction of injury in terms now known as (leukocytosis or leukopenia), and fever. A widening
the cardinal signs of inflammation. These signs are pulse pressure is not indicative of systemic inflamma-
rubor (redness), tumor (swelling), calor (heat), and tion, and thrombocytopenia is a hematologic disor-
dolor (pain). In the second century AD, the Greek der, not an indication of systemic inflammation.
physician Galen added a fifth cardinal sign, functio
laesa (loss of function). Altered level of consciousness
is not a cardinal sign of inflammation. Sepsis and CHAPTER 4
fever are systemic signs of infection.
2. Answers: b, c, e SECTION II: ASSESSING YOUR
RATIONALE: Eosinophils, basophils, and mast cells UNDERSTANDING
produce lipid mediators and cytokines that induce
inflammation. They are particularly important in Activity A
inflammation associated with immediate hypersen- 1. differentiation
sitivity reactions and allergic disorders. Neutrophils growth
and macrophages are white blood cells that respond 2. proliferation
to inflammation and destroy invading bacteria. 3. Differentiation
They do not induce inflammation. 4. cyclins
3. Answer: b 5. phosphorylate
RATIONALE: Chronic inflammation involves the pro- 6. progenitor
liferation of fibroblasts instead of exudates. As a 7. Stem
result, the risk of scarring and deformity usually is 8. Embryonic
greater than in acute inflammation. Chronic 9. parenchymal, stromal
inflammation is not the persistent destruction of 10. Labile cells
healthy tissue. Typically, agents that cause chronic 11. stabile
inflammation are agents that do not penetrate 12. Granulation
deeply or spread rapidly. Acute inflammation, not 13. collagen, fibroblast
chronic, is the result of allergic reactions. 14. premature infant

Copyright 2011. Wolters Kluwer Health | Lippincott Williams & Wilkins. Study Guide for Porths Essentials of Pathophysiology, Third Edition.
LWBK707-Ans_p280-402.qxd 8/20/10 6:38PM Page 288 Aptara Inc

288 ANSWER KEY

Activity B 2. Answer: c
RATIONALE: An increase in tissue oxygen tension by
1. c 2. a 3. i 4. f 5. j
hyperbaric oxygen enhances wound healing by a
6. g 7. b 8. h 9. d 10. e
number of mechanisms, including the increased
Activity D killing of bacteria by neutrophils, impaired growth
Margination and adhesion to the endothelium S of anaerobic bacteria, and the promotion of angio-
transmigration across endothelium S chemotaxis S genesis and fibroblast activity. Eosinophil activity is
activation and phagocytosis. not affected by hyperbaric treatment of wounds.
Activity E 3. Answer: c
RATIONALE: The child has a greater capacity for
1. In terms of cell proliferation, the cells may be divided repair than the adult but may lack the reserves
into three groups: (1) the well-differentiated needed to ensure proper healing. Such lack is
neurons and cells of skeletal and cardiac muscle evidenced by an easily upset electrolyte balance,
that rarely divide and reproduce; (2) the progenitor sudden elevation or lowering of temperature, and
or parent cells, that continue to divide and rapid spread of infection. The neonate and small
reproduce, such as blood cells, skin cells, and liver child may have an immature immune system with
cells; and (3) the undifferentiated stem cells that no antigenic experience with organisms that
can be triggered to enter the cell cycle and produce contaminate wounds. The younger the child, the
large numbers of progenitor cells when the need more likely that the immune system is not fully
arises. developed. The skin of a neonate or a small child is
2. Depending on the extent of tissue loss, wound clo- not as fragile as the skin of an elderly person.
sure and healing occur by primary or secondary 4. Answer: c
intention. Small or clean wounds (such as a surgi- RATIONALE: Infection impairs all dimensions of
cal incision) are an example of healing by primary wound healing. It prolongs the inflammatory
intention. Larger wounds that have a greater loss of phase, impairs the formation of granulation tis-
tissue and contamination heal by secondary inten- sue, and inhibits proliferation of fibroblasts and
tion. Healing by secondary intention is slower than deposition of collagen fibers. All wounds are
healing by primary intention and results in the for- contaminated at the time of injury. Although
mation of larger amounts of scar tissue. body defenses can handle the invasion of microor-
ganisms at the time of wounding, badly contami-
SECTION III: APPLYING YOUR KNOWLEDGE nated wounds can overwhelm host defenses.
Activity F Trauma and existing impairment of host defenses
also can contribute to the development of wound
1. After an injury the body initiates what is called the
infections.
inflammatory response. This means the body sends
5. Answers: a, b, c
cells and fluids that are specific to destroying infec-
tious organisms and healing the injury to the
site of the wound. What you are seeing on the CHAPTER 5
bandages is a serous exudate from the plasma in
the circulatory system that has responded to the
SECTION II: ASSESSING YOUR
burn injury.
2. The bodys response to an injury activates many
UNDERSTANDING
different types and kinds of cells. This response is Activity A
called the acute phase response and some of the 1. deoxyribonucleic
cells that are released during this response act on 2. Ribonucleic
the central nervous system. Their actions can cause 3. proteome
outward manifestations of their work such as 4. purine, pyrimidine
anorexia, somnolence, and malaise. 5. complementary
6. 23
SECTION IV: PRACTICING FOR NCLEX 7. chromosomes
Activity G 8. triplet
9. mutations
1. Answers: b, d, e
10. haplotype
RATIONALE: Wound healing is commonly divided
11. transcription
into three phases: (1) the inflammatory phase,
12. exons
(2) the proliferative phase, and (3) the maturational
13. Translation
or remodeling phase. There is no activation or
14. chaperones
nutritional phase in wound healing.
15. expression
16. RNA

Copyright 2011. Wolters Kluwer Health | Lippincott Williams & Wilkins. Study Guide for Porths Essentials of Pathophysiology, Third Edition.
LWBK707-Ans_p280-402.qxd 8/20/10 6:38PM Page 289 Aptara Inc

ANSWER KEY 289

17. Transcription at specific sites. There is a chance of 1 in 30 billion


18. phenotype that two persons who are not monozygotic twins
19. alleles would have identical DNA fingerprints. Because
20. pedigree genetic variations are so distinctive, DNA fingerprint-
Activity B ing (analysis of DNA sequence differences) can be used
to determine family relationships or help identify per-
1.
sons involved in criminal acts.
1. a 2. c 3. b 4. e 5. j
6. f 7. d 8. h 9. g 10. i
2.
SECTION IV: PRACTICING FOR NCLEX
1. d 2. a 3. c 4. b 5. e Activity F
Activity C 1. Answer: a
1. RATIONALE: The term proteome is a relatively
new term, created to define the complete set of
a S c S g S f S b S d S e S h S i
proteins encoded by a genome. A chromosome is
Activity D any of the threadlike structures in the nucleus of a
cell that function in the transmission of genetic
1. Mendel discovered the basic pattern of inheritance
information. The terms protogene and nucleotomics
by conducting carefully planned experiments with
are not real words.
garden peas. Experimenting with phenotypic traits
2. Answer: b
in peas, Mendel proposed that inherited traits are
RATIONALE: The two strands of the helix separate
transmitted from parents to offspring by means of
and a complementary molecule is duplicated next
independently inherited factors, now known as
to each original strand. Two strands become four
genes, and that these factors are transmitted as
strands. During cell division, the newly duplicated
recessive and dominant traits from patents to their
double-stranded molecules are separated and placed
offspring.
in each daughter cell by the mechanics of mitosis.
2. Genetic maps use linkage studies to estimate the
As a result, each of the daughter cells again contains
distances between chromosomal landmarks. They
the meaningful strand and the complementary
are similar to a road map. Physical maps are similar
strand joined together as a double helix.
to a surveyors map. They make use of cytogenetic
3. Answer: c
and molecular techniques to determine the actual
RATIONALE: Of the 23 pairs of human chromosomes,
physical locations of genes on chromosomes.
3. While in metaphase I, chromosomes are paired and 22 are called autosomes and are alike in both
condensed. Over time an interchange of chromatid males and females. The double helix is the shape
segments can occur. This process is called crossing of the DNA molecule. Ribosomes are areas in a cell
over. Crossing over allows for new combinations of that synthesize proteins. Haploid have only one
genes resulting in an increase in genetic variability. complete set of nonhomologous chromosomes.
This is a very beneficial process. 4. Answer: d
RATIONALE: Rarely, accidental errors in duplication
4. There are 22 pairs of somatic chromosomes. Half
of each pair is received from the female and the of DNA occur. These errors are called mutations.
other half are from the male. We then have two Ribosomes are areas in a cell that synthesize
sex chromosomes, an X from our mother and, in proteins. Several repair mechanisms exist, and
the case of females, an X from the father, for a total each depends on specific enzymes called endonu-
of two Xs. Males only have one X chromosome cleases that recognize local distortions of the
from their mother and one Y chromosome from DNA helix, cleave the abnormal chain, and
their father. remove the distorted region. Four basesguanine,
5. Gene activator and repressor sites within DNA com- adenine, cytosine, and thymine (uracil is substi-
monly monitor levels of the synthesized product tuted for thymine in RNA)make up the alpha-
and regulate gene transcription through a negative bet of the genetic code. A sequence of three of
feedback mechanism. Expression is also regulated these bases forms the fundamental triplet code
at the transcription level by transcription factors used in transmitting the genetic information
that directly affect protein structure and function. needed for protein synthesis. This triplet code is
called a codon.
5. Answer: a
SECTION III: APPLYING YOUR KNOWLEDGE RATIONALE: Polygenic inheritance involves multi-
Activity E ple genes at different loci, with each gene exerting a
DNA fingerprinting is a technique for comparing the small additive effect in determining a trait. Multi-
nucleotide sequences of fragments of DNA from differ- factorial inheritance is similar to polygenic inheri-
ent sources. The fragments are obtained by treating tance in that multiple alleles at different loci affect
the DNA with various enzymes that break DNA strands the outcome; the difference is that multifactorial

Copyright 2011. Wolters Kluwer Health | Lippincott Williams & Wilkins. Study Guide for Porths Essentials of Pathophysiology, Third Edition.
LWBK707-Ans_p280-402.qxd 8/20/10 6:38PM Page 290 Aptara Inc

290 ANSWER KEY

inheritance includes environmental effects on the carries the instructions for protein synthesis. Steri-
genes. Monofactorial inheritance is nonexistent, as cally stable liposomes are stable liposomes with
is collaborative inheritance. long circulation times. Sites in the DNA sequence
6. Answer: b where individuals differ at a single DNA base are
RATIONALE: When the deletion is inherited from called single nucleotide polymorphisms (SNPs,
the mother, the infant presents with Angelman pronounced snips).
(happy puppet) syndrome. Turner syndrome is 11. Answer: haplotype
a chromosomal anomaly seen in about 1 in 3000 RATIONALE: As the Human Genome Project was
live female births, characterized by the absence progressing it became evident that the human
of one X chromosome. Down syndrome is a con- genome sequence is almost exactly (99.9%) the
genital condition characterized by varying degrees same in all people. It is the small variation (0.01%)
of mental retardation and multiple defects. It is in gene sequence (termed a haplotype) that is
the most common chromosomal abnormality thought to account for the individual differences
of a generalized syndrome and is caused by the in physical traits, behaviors, and disease suscepti-
presence of an extra chromosome 21 in the G bility.
group. Fragile X syndrome is a reproductive 12. Answers: b, c, d
disorder characterized by a nearly broken X chro- RATIONALE: RNA is a single-stranded rather than a
mosome, which has a tip hanging by a flimsy double-stranded molecule. Second, the sugar in
thread. It is the most common inherited cause of each nucleotide of RNA is ribose instead of
mental retardation. deoxyribose. Third, the pyrimidine base thymine
7. Answer: c in DNA is replaced by uracil in RNA. All cells are
RATIONALE: A recessive trait is one that is expressed supposed to have 23 pairs of chromosomes.
only when a two homozygous people have a child. 13. Answer: insulin
A dominant trait is one expressed in either a RATIONALE: Recombinant DNA technology has also
homozygous or a heterozygous pairing. A single- made it possible to produce proteins that have
gene trait and a penetrant trait do not exist. How- therapeutic properties. One of the first products to
ever, single-gene inheritance does exist. be produced was human insulin.
8. Answer: d 14. Answers: a, b, c
RATIONALE: The establishment of the International RATIONALE: A karyotype is a photograph of a
HapMap Project was to map the haplotypes of persons chromosomes. It is prepared by special
the many closely related single nucleotide laboratory techniques in which body cells are
polymorphisms in the human genome; and the cultured, fixed, and then stained to display iden-
development of methods for applying the tech- tifiable banding patterns. A centromere is the con-
nology of these projects to the diagnosis and treat- stricted region of a chromosome that joins the two
ment of disease. Four basesguanine, adenine, chromatids to each other and attaches to spindle
cytosine, and thymine (uracil is substituted for fibers in mitosis and meiosis. Human chromosomes
thymine in RNA)make up the alphabet of the are classified as one of three types, depending
genetic code. A sequence of three of these bases on the position of their centromere. Two types
forms the fundamental triplet code used in trans- of genes, complementary genes, in which each
mitting the genetic information needed for protein gene is mutually dependent on the other; and
synthesis. This triplet code is called a codon. Alter- collaborative genes, in which two different genes
nate forms of a gene at the same locus are called influencing the same trait interact, play a part in
alleles. multifactorial inheritance.
9. Answer: a 15. Answers: 1-c, 2-a, 3-b, 4-e, 5-d
RATIONALE: Banding patterns are analyzed to see RATIONALE: The genotype of a person is the
if they match. Four basesguanine, adenine, cyto- genetic information stored in the base sequence
sine, and thymine (uracil is substituted for thymine triplet code. The phenotype refers to the recogniz-
in RNA)make up the alphabet of the genetic able traits, physical or biochemical, associated
code. A sequence of three of these bases forms the with a specific genotype. Pharmacogenetics is
fundamental triplet code used in transmitting the the variability of drug response due to inherited
genetic information needed for protein synthesis. characteristics in individuals. Somatic cell
The small variation in gene sequence (termed a hybridization involves the fusion of human
haplotype) that is thought to account for the indi- somatic cells with those of a different species
vidual differences in physical traits, behaviors, and (typically, the mouse) to yield a cell containing
disease susceptibility. Chromosomes contain all the chromosomes of both species. Penetrance rep-
the genetic content of the genome. resents the ability of a gene to express its function.
10. Answer: b Seventy-five percent penetrance means 75% of
RATIONALE: Cloned DNA sequences are usually the persons of a particular genotype present with a
compounds used in gene therapy. Messenger RNA recognizable phenotype.

Copyright 2011. Wolters Kluwer Health | Lippincott Williams & Wilkins. Study Guide for Porths Essentials of Pathophysiology, Third Edition.
LWBK707-Ans_p280-408.qxd 9/2/10 8:02 AM Page 291 Aptara Inc

ANSWER KEY 291

CHAPTER 6 2. The figure represents a simple pedigree for inheri-


tance of an autosomal dominant trait. The colored
circle or square represents an affected parent with a
SECTION II: ASSESSING YOUR mutant gene. An affected parent with an autosomal
UNDERSTANDING dominant trait has a 50% chance of passing the
Activity A mutant gene on to each child regardless of sex.
1. Congenital Activity C
2. DNA, chromosomal 1. h 2. c 3. e 4. a 5. g
3. codominate 6. b 7. j 8. d 9. i 10. f
4. mutation
Activity D
5. inherited, spontaneous
6. carrier 1. Conditions are inherited as dominant or recessive
7. Marfan on one of the autosomal chromosomes or one of
8. recessive the sex chromosomes. If the trait is dominant, the
9. 60 patient will inherit the condition. For a recessive
10. structure, abnormal trait to be expressed, both parents must carry the
11. Translocation mutation. If the mutation is on the Y chromosome,
all male children will be affected; if it is on the X
Activity B LABELING
chromosome, about half of the offspring potentially
1. may be affected.
2. Multifactorial congenital malformations involve a
single organ or tissue derived from the same embry-
onic developmental field. Second, the risk of recur-
rence in future pregnancies is increased for the same
A Deletion
or a similar defect. Third, the risk increases with
increasing incidence of the defect among relatives.
Lost
3. Structural changes in chromosomes usually results
from breakage in one or more of the chromosomes
followed by rearrangement or deletion of the chro-
mosome parts. Among the factors believed to cause
Balanced chromosome breakage are exposure to radiation
B translocation
sources, such as x-rays; influence of certain chemi-
cals; extreme changes in the cellular environment;
and viral infections.
4. (1) The inactivation of all but one X chromosome
and (2) the modest amount of genetic material that
is carried on the Y chromosome.
C Inversion 5. Ova contain the majority of the mitochondria,
whereas spermatozoa have very few, if any, so
the embryo will inherit most if not all of the
Pericentric Paracentric
mitochondria from the mother. The neural and
muscular tissues are most affected by mtDNA
mutations because of their great dependence on
Robertsonian
D oxidative phosphorylation. These tissues will have
Lost translocation
numerous mitochondria and suffer from their
malfunction.

SECTION III: APPLYING YOUR KNOWLEDGE


Isochromosomal
Activity E
E translocation 1. Alcohol passes freely across the placental barrier so
concentrations of alcohol in the fetus are at least as
high as in the mother. Unlike other agents harmful
to the fetus, the harmful effects of alcohol are not
restricted to the sensitive period of early gestation
but extend throughout pregnancy. Alcohol
F Ring formation consumption during pregnancy can cause fetal
alcohol syndrome in the baby.
Fragments

Copyright 2011. Wolters Kluwer Health | Lippincott Williams & Wilkins. Study Guide for Porths Essentials of Pathophysiology, Third Edition.
LWBK707-Ans_p280-408.qxd 9/3/10 11:02 AM Page 292 Aptara Inc

292 ANSWER KEY

2. Alcohol has widely variable effects on fetal develop- may also be caused by teratogens (e.g., rubella,
ment, ranging from minor abnormalities to fetal anticonvulsant drugs) and is often encountered in
alcohol syndrome. There may be prenatal or post- children with chromosomal abnormalities.
natal growth retardation; central nervous system 4. Answer: d
involvement, including neurologic abnormalities, RATIONALE: Occasionally, mitotic errors in early
developmental delays, behavioral dysfunction, development give rise to two or more cell lines
intellectual impairment, and skull and brain malfor- characterized by distinctive karyotypes, a condition
mation; and a characteristic set of facial features referred to as mosaicism. A gene mutation is a bio-
that include small eye openings, a thin upper lip, chemical event such as nucleotide change, deletion,
and an elongated, flattened midface and philtrum or insertion that produces a new allele. Referring to
(i.e., the groove in the middle of the upper lip). someone as a mutant is a derogatory expression.
Each of these defects can vary in severity, probably Monosomy refers to the presence of only one mem-
reflecting the timing of alcohol consumption in ber of a chromosome pair. It is not a term a person
terms of the period of fetal development, amount is called. Having an abnormal number of chromo-
of alcohol consumed, and hereditary and environ- somes is referred to as aneuploidy; it is not a term
mental influences. a person is called.
5. Answer: a
SECTION IV: PRACTICING FOR NCLEX RATIONALE: The risk of having a child with Down
syndrome increases with maternal ageit is 1 in
Activity F 1250 at 25 years of age, 1 in 400 at 35 years, and 1
1. Answer: a in 100 at 45 years of age. The reason for the corre-
RATIONALE: If the members of a gene pair are iden- lation between maternal age and nondisjunction
tical (i.e., code the exact same gene product), the is unknown, but is thought to reflect some aspect
person is homozygous, and if the two members are of aging of the oocyte. Although males continue
different, the person is heterozygous. The pheno- to produce sperm throughout their reproductive
type is the observable expression of a genotype in life, females are born with all the oocytes they ever
terms of morphologic, biochemical, or molecular will have. These oocytes may change as a result of
traits. Although gene expression usually follows a the aging process. With increasing age, there is a
dominant or recessive pattern, it is possible for greater chance of a woman having been exposed
both alleles (members) of a gene pair to be fully to damaging environmental agents such as drugs,
expressed in the heterozygote, a condition called chemicals, and radiation. There is no correlation
codominance. A gene mutation is a biochemical with maternal age and the other syndromes.
event such as nucleotide change, deletion, or 6. Answer: b
insertion that produces a new allele. RATIONALE: The embryos development is most
2. Answer: b easily disturbed during the period when differenti-
RATIONALE: In more than 90% of persons with neu- ation and development of the organs are taking
rofibromatosis-1, cutaneous and subcutaneous place. This time interval, which is often referred to
neurofibromas develop in late childhood or adoles- as the period of organogenesis, extends from day
cence. The cutaneous neurofibromas, which vary 15 to day 60 after conception. There are no periods
in number from a few to many hundreds, manifest of susceptibility, fetal anomalies, or hormonal
as soft, pedunculated lesions that project from imbalance.
the skin. Marfan syndrome affects several organ 7. Answer: c
systems including the ocular system (eyes), the RATIONALE: Teratogenic agents have been divided
cardiovascular system (heart and blood vessels), into three groups: radiation, drugs and chemical
and the skeletal system (bones and joints). Down substances, and infectious agents. The period of
syndrome is a congenital condition characterized organogenesis, the third trimester, and the second
by varying degrees of mental retardation and mul- trimester are not teratogenic substances. They are
tiple defects. Klinefelter syndrome is a condition time periods during the pregnancy. Teratogenic
that occurs in men who have an extra X chromo- substances are not classified as outside, inside, or
some in most of their cells. The syndrome can internal. Although drugs and chemical substances
affect different stages of physical, language, and are a class of teratogenic agents, smoking is included
social development. The most common symptom in that class as a teratogenic agent. It is not a class
is infertility. unto itself. Bacteria and virus are considered
3. Answer: c infectious agents and are therefore teratogenic
RATIONALE: Cleft lip with or without cleft palate is agents.
one of the most common birth defects. This 8. Answer: d
process is under the control of many genes, and RATIONALE: The acronym TORCH stands for toxo-
the disturbances in gene expression (hereditary or plasmosis, other, rubella (i.e., German measles),
environmental) at this time may result in cleft lip cytomegalovirus, and herpes, which are the agents
with or without cleft palate (Fig. 6-6). The defect most frequently implicated in fetal anomalies.

Copyright 2011. Wolters Kluwer Health | Lippincott Williams & Wilkins. Study Guide for Porths Essentials of Pathophysiology, Third Edition.
LWBK707-Ans_p280-402.qxd 8/20/10 6:38PM Page 293 Aptara Inc

ANSWER KEY 293

Common clinical and pathologic manifestations Third, the increased risk (compared with the
include growth retardation and abnormalities of general population) among first-degree relatives
the brain (microcephaly, hydrocephalus), eye, ear, of the affected person is 2% to 7%, and among
liver, hematopoietic system (anemia, thrombocy- second-degree relatives, it is approximately half
topenia), lungs (pneumonitis), and heart that amount. The risk increases with increasing
(myocarditis, congenital heart disorders). incidence of the defect among relatives. Disorders
9. Answer: a of multifactorial inheritance can be expressed dur-
RATIONALE: The birth of a defective child is a trau- ing fetal life and be present at birth, or they may
matic event in any parents life. Usually two issues be expressed later in life.
must be resolved. The first deals with the immedi- 13. Answer: Phenylketonuria
ate and future care of the affected child, and the RATIONALE: Phenylketonuria (PKU) is a rare meta-
second with the possibility of future children in bolic disorder that affects approximately 1 in every
the family having a similar defect. 15,000 infants in the United States. The disorder,
10. Answer: b which is inherited as a recessive trait, is caused by
RATIONALE: The purpose of prenatal screening and a deficiency of the liver enzyme phenylalanine
diagnosis is not just to detect fetal abnormalities. hydroxylase. As a result of this deficiency, toxic
Rather, it has the following objectives: to provide levels of the amino acid phenylalanine accumulate
parents with information needed to make an in the blood and other tissues.
informed choice about having a child with an 14. Answers: a, d
abnormality; to provide reassurance and reduce RATIONALE: The physiologic status of the mother
anxiety among high-risk groups; and to allow par- her hormone balance, her general state of health,
ents at risk for having a child with a specific defect, her nutritional status, and the drugs she takes
who might otherwise forgo having a child, to undoubtedly influences the development of the
begin a pregnancy with the assurance that knowl- unborn child. Other agents, such as radiation,
edge about the presence or absence of the disorder can cause chromosomal and genetic defects and
in the fetus can be confirmed by testing. It is not produce developmental disorders. Neither the
the object of genetic counseling and prenatal weather nor air pollution has been linked with
screening to provide information on where to fetal abnormalities or developmental disorders.
terminate a pregnancy if that is what the parents 15. Answer: a
choose to do. Prenatal screening cannot be used RATIONALE: In 1983, the U.S. Food and Drug
to rule out all possible fetal abnormalities. It is Administration established a system for classifying
limited to determining whether the fetus has (or drugs according to probable risks to the fetus.
probably has) designated conditions indicated by According to this system, drugs are put into five
late maternal age, family history, or well-defined categories: A, B, C, D, and X. Drugs in category A
risk factors. are the least dangerous, and categories B, C,
11. Answers: 1-a, 2-b, 3-c, 4-d, 5-e and D are increasingly more dangerous. Those in
RATIONALE: A single mutant gene may be expressed category X are contraindicated during pregnancy
in many different parts of the body. Marfan because of proven teratogenicity.
syndrome, for example, is a defect in connective tis-
sue that has widespread effects involving skeletal,
eye, and cardiovascular structures. In autosomal CHAPTER 7
dominant disorders, a single mutant allele from an
affected parent is transmitted to an offspring SECTION II: ASSESSING YOUR
regardless of sex. In many conditions, the age of UNDERSTANDING
onset is delayed, and the signs and symptoms of
Activity A
the disorder do not appear until later in life, as in
Huntingtons chorea. Tay-Sachs is inherited as an 1. differentiation growth
autosomal recessive trait. Fragile X syndrome is a 2. proliferation
single-gene disorder in which the mutation is char- 3. Differentiation
acterized by a long repeating sequence of three 4. kinases
nucleotides within the fragile X gene. 5. phosphorylate
12. Answers: a, b, c 6. progenitor
RATIONALE: First, multifactorial congenital malfor- 7. Stem
mations tend to involve a single organ or tissue 8. Embryonic
derived from the same embryonic developmental 9. neoplasm
field. Second, the risk of recurrence in future preg- 10. Benign tumors
nancies is for the same or a similar defect. This 11. differentiated
means that parents of a child with a cleft palate 12. -oma
defect have an increased risk of having another 13. polyp
child with a cleft palate, but not with spina bifida. 14. carcinoma

Copyright 2011. Wolters Kluwer Health | Lippincott Williams & Wilkins. Study Guide for Porths Essentials of Pathophysiology, Third Edition.
LWBK707-Ans_p280-402.qxd 8/20/10 6:38PM Page 294 Aptara Inc

294 ANSWER KEY

15. solid tumors hematological Activity E


16. anaplasia
1. In terms of cell proliferation, the cells may be
17. growth factors
divided into three groups: (1) the well-differentiated
18. p53
neurons and cells of skeletal and cardiac muscle
19. protooncogenes suppressor
that rarely divide and reproduce; (2) the progeni-
20. Human T-cell leukemia virus-1
tor or parent cells that continue to divide and
21. 30
reproduce, such as blood cells, skin cells, and
22. anorexia-cachexia
liver cells; and (3) the undifferentiated stem cells
23. ulceration necrosis
that can be triggered to enter the cell cycle and
24. Anemia
produce large numbers of progenitor cells when
25. biopsy
the need arises.
26. Radiation
2. Both benign and malignant tumors have lost the
27. Chemotherapy
ability to suppress growth. As a result, the tumor
Activity B cells continue to proliferate. Benign tumors are
composed of well-differentiated cells and are
confined to the area of tissue origin. In contrast,
Carcinogenic Normal malignant tumors are composed of less differenti-
agent cell
ated cells that will re-enter circulation and establish
secondary tumors in another region of the body.
DNA repair
(DNA repair genes) 3. (1) Cell characteristics, (2) rate of growth, (3) man-
ner of growth, (4) capacity to invade and metasta-
DNA damage size to other parts of the body, and (5) potential
for causing death.
4. Metastasis occurs via lymph channels and blood
Failure of DNA
repair vessels. When metastasis occurs by way of the
lymphatic channels, the tumor cells lodge first in
Activation of growth-promoting oncogenes the initial lymph node that receives drainage from
Inactivation of tumor suppressor genes the tumor site. If they survive, cancer cells may
Alterations in genes that control apoptosis
spread from more distant lymph nodes to the
thoracic duct, and then gain access to the blood
vasculature. With hematologic spread, the blood-
borne cancer cells may enter the venous flow that
Unregulated cell drains the site of the primary neoplasm. Cancer
differentiation and growth
cells may also enter tumor-associated blood vessels
that either infiltrate the tumor or are found at the
periphery of the tumor.
5. Cancer cells express abnormal cell surface
Malignant proteins. Normally, the immune system recognizes
neoplasm
these abnormal proteins and destroys the cancer-
ous cell. With a compromised immune system,
these abnormalities are missed and allowed to
persist in the body.
6. Chemicals will cause cellular transformation either
directly (direct reacting agents) or indirectly,
Activity C only becoming activated via a metabolic process
1. b 2. a 3. e 4. j 5. f (initiators).
6. c 7. d 8. h 9. g 10. i 7. Hypermetabolism is the result of the rapidly
11. m 12. o 13. n 14. q 15. l growing tumor and the increased expression of
16. k 17. r 18. s 19. p uncoupling proteins. The tumor uses large quan-
tities of glucose via glycolysis, therefore produc-
Activity D ing high levels of lactic acid. The lactic acid
1. undergoes the energy-requiring process of gluco-
neogenesis in order to convert it back to glucose.
b S a S d S c This uses large amounts of glucose and wastes
large amounts of adenosine triphosphate (ATP).
The second reason is the presence of uncoupling
proteins. The uncoupling proteins uncouple

Copyright 2011. Wolters Kluwer Health | Lippincott Williams & Wilkins. Study Guide for Porths Essentials of Pathophysiology, Third Edition.
LWBK707-Ans_p280-408.qxd 9/2/10 8:02 AM Page 295 Aptara Inc

ANSWER KEY 295

oxidative phosphorylation, thereby reducing the differentiation. They do not have the capacity to
amount of ATP produced. infiltrate, invade, or metastasize to distant sites.
8. Paraneoplastic syndromes are characterized by 2. Answer: b
manifestations in sites that are not directly RATIONALE: Metastasis occurs by way of the lymph
affected by the disease. Most commonly, manifes- channels (i.e., lymphatic spread) and the blood
tations are caused by the elaboration of hormones vessels (i.e., hematogenic spread). In many types
by cancer cells, and others from the production of of cancer, the first evidence of disseminated
circulating factors that produce hematopoietic, disease is the presence of tumor cells in the lymph
neurologic, and dermatological syndromes. nodes that drain the tumor area. When metastasis
9. Blood tests for tumor markers, cytologic studies occurs by way of the lymphatic channels, the
and tissue biopsy, endoscopic examinations, ultra- tumor cells lodge first in the initial lymph node
sound, x-ray studies, MRI, computed tomography, that receives drainage from the tumor site. Once
and positron-emission tomography. in this lymph node, the cells may die because of
10. The clinical staging of cancer is intended to group the lack of a proper environment, grow into a dis-
patients according to the extent of their disease. cernible mass, or remain dormant for unknown
Grading of tumors involves the microscopic exam- reasons. If they survive and grow, the cancer cells
ination of cancer cells to determine their level of may spread from more distant lymph nodes to the
differentiation and the number of mitoses. thoracic duct, and then gain access to the blood
Cancers are classified as grades I, II, III, and IV vasculature. Because cancer cells have the ability
with increasing anaplasia or lack of differentiation. to shed themselves from the original tumor, they
The two basic methods for classifying cancers are are often found floating in the body fluids around
grading according to the histologic or cellular char- the tumor. Cancer cells are not moved from one
acteristics of the tumor and staging according to place to another by transporter cells. Cancer cells
the clinical spread of the disease. do not form a chain to grow to the new place in
the body to form a new tumor.
SECTION III: APPLYING YOUR KNOWLEDGE 3. Answer: c
RATIONALE: Cancer occurs because of interactions
Activity F
among multiple risk factors or repeated exposure
1. To make it better for you, the doctor is going to to a single carcinogenic (cancer-producing) agent.
put a tube just under your skin that the nurses can Among the traditional risk factors that have been
put your medication in so they wont have to stick linked to cancer are heredity, hormonal factors,
you in the hands and arms so many times. You immunologic mechanisms, and environmental
will still get stuck by a needle but it will not be as agents such as chemicals, radiation, and cancer-
painful as trying to start an IV in your arms. causing viruses. More recently, there has been
2. Since Joes cancer is found in his blood and interest in obesity and type 2 diabetes mellitus as
bone marrow, you cannot use surgery to cure it. risk factors for a number of cancers. Body type,
Chemotherapy is the primary treatment for most age, and color of skin have not been identified as
hematologic and some solid tumors. Chemotherapy risk factors for cancer.
is a systemic treatment that enables drugs to 4. Answer: d
reach the site of the tumor as well as other RATIONALE: Familial adenomatous polyposis of the
distant sites. Cancer chemotherapeutic drugs colon also follows an autosomal dominant inheri-
exert their effects through several mechanisms. tance pattern. It is caused by mutation of another
At the cellular level, they exert their lethal action tumor suppressor gene, the APC gene. In people
by targeting processes that prevent cell growth and who inherit this gene, hundreds of adenomatous
replication. These mechanisms include disrupting polyps may develop, some of which inevitably
the production of essential enzymes; inhibiting become malignant. Retinoblastoma is inheritable
DNA, RNA, and protein synthesis; and preventing through an autosomal dominant gene, but only
cell reproduction. about 40% of retinoblastomas are inherited.
Osteosarcoma and ALL are not inheritable through
SECTION IV: PRACTICING FOR NCLEX an autosomal dominant process.
5. Answer: a
Activity G
RATIONALE: Most known dietary carcinogens
1. Answer: a occur either naturally in plants (e.g., aflatoxins)
RATIONALE: Asking if his tumor will make him die or are produced during food preparation. Among
shows lack of understanding of educational mate- the most potent of the procarcinogens are the
rial he has been given. For unknown reasons, polycyclic aromatic hydrocarbons. The polycyclic
benign tumors have lost the ability to suppress aromatic hydrocarbons are of particular interest
the genetic program for cell proliferation but because they are produced from animal fat in
have retained the program for normal cell the process of charcoal-broiling meats and are

Copyright 2011. Wolters Kluwer Health | Lippincott Williams & Wilkins. Study Guide for Porths Essentials of Pathophysiology, Third Edition.
LWBK707-Ans_p280-402.qxd 8/20/10 6:38PM Page 296 Aptara Inc

296 ANSWER KEY

present in smoked meats and fish. They also are more forms of cancer treatment than adults do.
produced in the combustion of tobacco and Children do complain about the nausea and vom-
are present in cigarette smoke. Initiators is another iting chemotherapy can cause, just like adults do.
term for procarcinogens. Diethylstilbestrol And they do not like losing their hair, just like
was a drug that was widely used in the United adults.
States from the mid-1940s to 1970 to prevent 10. Answer: b
miscarriages. RATIONALE: The combination of selected cytotoxic
6. Answer: b drugs with radiation has demonstrated a radiosen-
RATIONALE: Lung cancers, breast cancers, and lym- sitizing effect on tumor cells by altering the cell
phomas account for about 75% of malignant pleu- cycle distribution, increasing DNA damage, and
ral effusions. Complaints of abdominal discomfort, decreasing DNA repair. Some radiosensitizers are
swelling and a feeling of heaviness, and an increase 5-fluorouracil, capecitabine, paclitaxel, gemcitabine,
in abdominal girth, which reflect the presence and cisplatin. Doxorubicin is an antitumor antibi-
of peritoneal effusions or ascites, are the most otic; vincristine is a vinca alkaloid; and docetaxel
common presenting symptoms in ovarian cancer, is a taxane.
occurring in up to 65% of women with the 11. Answer: neoplasm
disease. RATIONALE: An abnormal mass of tissue in which
7. Answer: c the growth exceeds and is uncoordinated with
RATIONALE: Tumor markers are antigens expressed that of the normal tissues is called a neoplasm.
on the surface of tumor cells or substances released Unlike normal cellular adaptive processes such as
from normal cells in response to the presence of hypertrophy and hyperplasia, neoplasms do not
tumor. The serum markers that have proven most obey the laws of normal cell growth. They serve
useful in clinical practice are the human chorionic no useful purpose, they do not occur in response
gonadotropin (hCG), CA 125, prostate-specific to an appropriate stimulus, and they continue to
antigen (PSA), alpha-fetoprotein, carcinoembryonic grow at the expense of the host.
antigen, and CD blood cell antigens. Deoxyribonu- 12. Answers: a, c, e
cleic acid is DNA and is not a serum tumor marker. RATIONALE: Malignant neoplasms are less well dif-
Cyclin-dependent kinases come from a family of ferentiated and have the ability to break loose,
proteins called cyclins, which control entry and enter the circulatory or lymphatic systems, and
progression of cells through the cell cycle. Cyclins form secondary malignant tumors at other sites.
act by complexing with (and thereby activating) Malignant neoplasms frequently cause suffering
proteins called cyclin-dependent kinases (CDKs). and death if untreated or uncontrolled. Malignant
They are not serum tumor markers. neoplasms form secondary tumors at sites other
8. Answer: d than the original tumor site. Malignant neoplasms
RATIONALE: Growth hormone deficiency in adults are not passed out of the body as waste through
is associated with increased prevalence of dyslipi- the alimentary canal.
demia, insulin resistance, and cardiovascular 13. Answers: b, c, e
mortality. Hypocalcemia is a deficiency of calcium RATIONALE: Cancer cells differ from normal cells by
in the serum that may be caused by hypoparathy- being immortal with an unlimited life span. Can-
roidism, vitamin D deficiency, kidney failure, acute cer cells often lose cell density-dependent inhibi-
pancreatitis, or inadequate amounts of plasma tion, which is the cessation of growth after cells
magnesium and protein. It does not result from reach a particular density. This is sometimes
cancer therapy during childhood. Hyperinsuline- referred to as contact inhibition because cells often
mia is associated with syndrome X, which is a con- stop growing when they come into contact with
dition characterized by hypertension with obesity, each other. Another characteristic of cancer cells is
type 2 diabetes mellitus, hypertriglyceridemia, the ability to proliferate even in the absence of
increased peripheral insulin resistance, hyperinsu- growth factors. Most cancer cells exhibit a charac-
linemia, and elevated catecholamine levels. teristic called genetic instability that is often con-
9. Answer: a sidered to be a hallmark of cancer.
RATIONALE: Chemotherapy is more widely used in 14. Answers: 1-b, 2-d, 3-c, 4-a
the treatment of children with cancer than in RATIONALE: Cancers for which current screening
adults because children better tolerate the acute or early detection has led to improvement in out-
adverse effects, and in general, pediatric tumors comes include cancers of the breast (breast self-
are responsive to chemotherapy than adult examination and mammography), cervix (Pap
cancers. Children are very adaptable and tolerate smear), colon and rectum (rectal examination,

Copyright 2011. Wolters Kluwer Health | Lippincott Williams & Wilkins. Study Guide for Porths Essentials of Pathophysiology, Third Edition.
LWBK707-Ans_p280-402.qxd 8/20/10 6:38PM Page 297 Aptara Inc

ANSWER KEY 297

fecal occult blood test, and flexible sigmoidoscopy 27. Third-space losses
and colonoscopy), prostate (PSA testing and trans- 28. isotonic
rectal ultrasonography), and malignant melanoma 29. Hyponatremia
(self-examination). 30. Normovolemic hypotonic
15. Answers: a, b, d 31. ADH
RATIONALE: With improvement in treatment meth- 32. Hypernatremia
ods, the number of children who survive child- 33. water
hood cancer is continuing to increase. As these 34. Na-K exchange mechanism
children approach adulthood, there is continued 35. resting membrane potential
concern that the life-saving therapy they received 36. hyperkalemia
during childhood may produce late effects, such 37. hypokalemia
as impaired growth, cognitive dysfunction, hor- 38. renal failure
monal dysfunction, cardiomyopathy, pulmonary 39. excess
fibrosis, and risk for second malignancies. Liver 40. Vitamin D
failure is not viewed as a late effect of childhood 41. Magnesium
cancer therapy. 42. hypocalcemia
43. hypophosphatemia
44. calcium
CHAPTER 8 45. Magnesium
46. 7.35, 7.45
SECTION II: ASSESSING YOUR 47. pH
UNDERSTANDING 48. metabolic
49. volatile, nonvolatile
Activity A
50. H2CO3
1. ICF compartment 51. dietary proteins
2. ECF compartment 52. Henderson-Hasselbalch equation
3. Electrolytes 53. bicarbonate
4. nonelectrolytes 54. Metabolic alkalosis
5. Diffusion 55. hypoventilation, hypoxemia
6. Osmosis 56. acidosis
7. Osmolarity, osmolality 57. alkalosis
8. Na
9. osmolar gap
Activity B
10. Osmotically active
11. Na-K ATPase
12. Capillary filtration
13. lymphatic system
14. Edema Blood volume
Serum osmolality
15. plasma proteins
16. Pitting
17. Third-space fluids
18. insensible water losses Secretion of
Thirst
ADH
19. kidney
20. effective circulating volume
21. angiotensin II, aldosterone Water ingestion Reabsorption of
water by the kidney
22. Thirst, ADH
23. Psychogenic polydipsia
24. Diabetes insipidus Extracellular
25. hyponatremia, hypernatremia water volume
26. hypovolemia Feedback

Copyright 2011. Wolters Kluwer Health | Lippincott Williams & Wilkins. Study Guide for Porths Essentials of Pathophysiology, Third Edition.
LWBK707-Ans_p280-402.qxd 8/20/10 6:38PM Page 298 Aptara Inc

298 ANSWER KEY

7.4

6.9 7.9

24 1.2
pH = 6.1 + log10 (ratio HCO3-: H 2CO3)
HCO3- H2CO3
(mEq/L) (mEq/L)

A Ratio: HCO 3-: H 2CO3 = 20:1


normal, ph 7.4

7.4 7.4
7.7
6.9 7.9 6.9 7.9

12 0.6

HCO3- 1.2 24 H2CO3


(mEq/L) (mEq/L)
H2CO3 HCO3-
(mEq/L) (mEq/L)

B Ratio: HCO 3-: H 2CO3 = 10:1 D Ratio: HCO 3-: H 2CO3 = 40:1
metabolic acidosis respiratory alkalosis

7.4 7.4

6.9 7.9 6.9 7.9

12 0.6 12 0.6

HCO3- H2CO3 HCO3- H2CO3


(mEq/L) (mEq/L) (mEq/L) (mEq/L)

C Ratio: HCO 3-: H 2CO3 = 20:1 E Ratio: HCO 3-: H 2CO3 = 20:1
metabolic acidosis with respiratory alkalosis
respiratory compensation with renal compensation

Activity C
1.
1. c 2. g 3. e 4. b 5. j
6. f 7. a 8. h 9. d 10. i
2.
1. d 2. h 3. b 4. a 5. c
6. j 7. e 8. g 9. i 10. f
3.
1. e 2. a 3. i 4. h 5. c
6. b 7. d 8. j 9. f 10. g

Copyright 2011. Wolters Kluwer Health | Lippincott Williams & Wilkins. Study Guide for Porths Essentials of Pathophysiology, Third Edition.
LWBK707-Ans_p280-402.qxd 8/20/10 6:38PM Page 299 Aptara Inc

ANSWER KEY 299

Activity D pressure, decrease the capillary colloidal osmotic


pressure, increase capillary permeability, or
produce obstruction to lymph flow.
Decreased serum 4. The major regulator of sodium and water balance
calcium is the maintenance of the effective circulating vol-
ume, which can described as that portion of the
ECF that fills the vascular compartment and is
effectively perfusing the tissues. A low effective
Parathyroid circulating volume results in feedback mechanisms
glands that produce an increase in renal and sodium and
water retention and a high circulating volume in
feedback mechanisms that decrease sodium and
Parathyroid
hormone water retention.
Activated 5. Three types of polydipsia include (1) symptomatic
vitamin D or true thirst, (2) inappropriate or false thirst that
occurs despite normal levels of body water and
Bone
serum osmolality, and (3) compulsive water
drinking.
Release of 6. There may be a decrease in BUN and hematocrit
calcium
Kidney because of dilution due to expansion of the
plasma volume. An increase in vascular volume
Intestine may be evidenced by distended neck veins, slow-
Increased calcium emptying peripheral veins, a full and bounding
absorption pulse, and an increase in central venous pressure.
When excess fluid accumulates in the lungs (i.e.,
Decreased calcium pulmonary edema), there are complaints of short-
elimination and increased
phosphate elimination
ness of breath and difficult breathing, respiratory
crackles, and a productive cough. Ascites and pleu-
ral effusion may occur with severe fluid volume
excess.
Increased 7. These changes include prolongation of the PR
serum calcium
interval, depression of the ST segment, flattening
Feedback
of the T wave, and appearance of a prominent U
wave. Normally, potassium leaves the cell during
the repolarization phase of the action potential,
returning the membrane potential to its normal
Activity E
resting value. Hypokalemia reduces the permeabil-
1. The ECF, including blood plasma and interstitial ity of the cell membrane to potassium and thus
fluids, contain large amounts of sodium and chlo- produces a decrease in potassium efflux that
ride, moderate amounts of bicarbonate, but only prolongs the rate of repolarization and lengthens
small quantities of potassium, magnesium, the relative refractory period. The U wave normally
calcium, and phosphate. In contrast to the ECF, may be present on the electrocardiogram but
the ICF contains almost no calcium; small should be of lower amplitude than the T wave.
amounts of sodium, chloride, bicarbonate, and With hypokalemia, the amplitude of the T wave
phosphate; moderate amounts of magnesium; and decreases as the U-wave amplitude increases.
large amounts of potassium. 8. Systemic effects of hypercalcemia are (1) changes
2. a. the capillary filtration pressure, which pushes in neural excitability, (2) alterations in smooth
water out of the capillary into the interstitial and cardiac muscle function, and (3) exposure of
spaces the kidneys to high concentrations of calcium.
b. the capillary colloidal osmotic pressure, which 9. Excess H ions can be exchanged for Na and K
pulls water back into the capillary on the bone surface and dissolution of bone min-
c. the interstitial hydrostatic pressure, which erals with release of compounds such as sodium
opposes the movement of water out of the bicarbonate (NaHCO3), and calcium carbonate
capillary (CaCO3) into the ECF can be used for buffering
d. the tissue colloidal osmotic pressure, which excess acids. It has been estimated that as much as
pulls water out of the capillary into the intersti- 40% of buffering of an acute acid load takes place
tial spaces in bone. The role of bone buffers is even greater in
3. Mechanisms that contribute to edema formation the presence of chronic acidosis. The consequences
include factors that increase the capillary filtration of bone buffering include demineralization of

Copyright 2011. Wolters Kluwer Health | Lippincott Williams & Wilkins. Study Guide for Porths Essentials of Pathophysiology, Third Edition.
LWBK707-Ans_p280-402.qxd 8/20/10 6:38PM Page 300 Aptara Inc

300 ANSWER KEY

bone and predisposition to development of kidney Blood pressure


stones due to increased urinary excretion of Testing for venous refill
calcium. Persons with chronic kidney disease are Capillary refill time
at particular risk for reduction in bone calcium Activity G
because of acid retention.
1. Metabolic acidosis
10. The transcompartmental exchange of H and
2. To rule out diabetes mellitus as a cause of the meta-
potassium ions (K) provides an important system
bolic acidosis
for regulation of acid-base balance. Both ions are
positively charged, and both ions move freely
between the ICF and ECF compartments. When SECTION IV: PRACTICING FOR NCLEX
excess H is present in the ECF, it moves into the Activity H
ICF in exchange for K, and when excess K is 1. Answers: a, b, d, e
present in the ECF, it moves into the ICF in RATIONALE: The physiologic mechanisms that con-
exchange for H. Thus, alterations in potassium tribute to edema formation include factors that
levels can affect acid-base balance, and changes in (1) increase the capillary filtration pressure, (2)
acid-base balance can influence potassium levels. decrease the capillary colloidal osmotic pressure,
11. The kidneys play two major roles in regulating (3) increase capillary permeability, or (4) produce
acid-base balance. The first is accomplished obstruction to lymph flow.
through the reabsorption of the HCO3 that is fil- 2. Answers: 1-d, 2-e, 3-a, 4-c, 5-b
tered in the glomerulus so this important buffer is 3. Answer: a
not lost in the urine. The second is through the RATIONALE: The major regulator of sodium and
excretion of H from fixed acids that result from water balance is the maintenance of the effective
protein and lipid metabolism. circulating volume. The other answers are not reg-
12. There are two types of acid-base disorders: ulated by the effective circulating volume.
metabolic and respiratory. Metabolic disorders 4. Answer: b
produce an alteration in the plasma HCO3 concen- RATIONALE: Psychogenic polydipsia may be
tration and result from the addition or loss of compounded by antipsychotic medications that
nonvolatile acid or alkali to or from the extra- increase ADH levels and interfere with water excre-
cellular fluids. A reduction in pH due to a decrease tion by the kidneys. Cigarette smoking, which is
in HCO3 is called metabolic acidosis, and an common among persons with psychiatric
elevation in pH due to increased HCO3 levels is disorders, also stimulates ADH secretion.
called metabolic alkalosis. Respiratory disorders 5. Answer: c
involve an alteration in the PCO2, reflecting an RATIONALE: Other acquired causes of nephrogenic
increase or decrease in alveolar ventilation. Respi- DI are drugs such as lithium and electrolyte disor-
ratory acidosis is characterized by a decrease in ders such as potassium depletion or chronic hyper-
pH, reflecting a decrease in ventilation and an calcemia. The other answers are not acquired
increase in PCO2. Respiratory alkalosis involves an causes of nephrogenic DI.
increase in pH, resulting from an increase in alveo- 6. Answer: c
lar ventilation and a decrease in PCO2. RATIONALE: When this occurs, water moves into
the brain cells, causing cerebral edema and poten-
SECTION III: APPLYING YOUR KNOWLEDGE tially severe neurologic impairment. The other
Activity F CASE STUDY cells are not correct.
7. Answer: a
a. When a client has burns over a large area of the
RATIONALE: Changes in nerve and muscle excitabil-
body there is a loss of protein in the plasma of the
ity are particularly important in the heart, where
body. In a burn there is also injury to the capillaries
alterations in plasma potassium can produce seri-
in the burned area. When a person is burned, large
ous cardiac arrhythmias and conduction defects.
amounts of albumin are moved out of the blood
The other answers are not correct.
and are lost in the urine. We are working very hard
8. Answer: b
to infuse fluid that the body needs with our IV
RATIONALE: The small, but vital, amount of ECF
solutions.
calcium, phosphate, and magnesium is directly or
b. The nurse knows that the diagnosis of fluid volume
indirectly regulated by vitamin D and parathyroid
deficit is based on these factors:
hormone. The other answers are not correct.
History of conditions that predispose to sodium
9. Answer: d
and water losses
RATIONALE: The NPT2 is also inhibited by the
Weight loss
recently identified hormone called phosphatonin.
Intake and output
When this hormone is overproduced, as in tumor-
Heart rate
induced osteomalacia, marked hypophosphatemia

Copyright 2011. Wolters Kluwer Health | Lippincott Williams & Wilkins. Study Guide for Porths Essentials of Pathophysiology, Third Edition.
LWBK707-Ans_p280-402.qxd 8/20/10 6:38PM Page 301 Aptara Inc

ANSWER KEY 301

occurs. The other conditions are not caused by blurred vision, irritability, muscle twitching, and
hypophosphatemia. psychological disturbances. Seizures and psychotic
10. Answer: a breaks are not signs or symptoms of respiratory
RATIONALE: Severe hypermagnesemia (12 mg/dL) acidosis.
is associated with muscle and respiratory paralysis, 20. Answer: a
complete heart block, and cardiac arrest. RATIONALE: One of the most common causes of res-
11. Answer: a piratory alkalosis is hyperventilation syndrome,
RATIONALE: The H2CO3 content of the blood can be which is characterized by recurring episodes of
calculated by multiplying the partial pressure of CO2 overbreathing, often associated with anxiety.
(PCO2) by its solubility coefficient, which is 0.03.
12. Answers: a, c, e
RATIONALE: The pH of body fluids is regulated by CHAPTER 9
three major mechanisms: (1) chemical buffer sys-
tems of the body fluids, which immediately com- SECTION II: ASSESSING YOUR
bine with excess acids or bases to prevent large UNDERSTANDING
changes in pH; (2) the lungs, which control the
Activity A
elimination of CO2; and (3) the kidneys, which
eliminate H and both reabsorb and generate 1. homeostasis, physiologic
HCO3. None of the other answers are correct. 2. Homeostasis
13. Answer: c 3. negative
RATIONALE: The renal mechanisms for regulating 4. stress
acid-base balance cannot adjust the pH within 5. disease
minutes, as respiratory mechanisms can, but they 6. hypothalamic-pituitary-adrenocortical,
continue to function for days, until the pH has adrenomedullary, sympathetic
returned to normal or near-normal range. It is the 7. adapting
respiratory system that responds within minutes 8. coping strategy
to return the bodys pH near to its normal limits. 9. Sleep
The other answers are wrong. 10. Alcohol
14. Answer: d Activity B
RATIONALE: The total base excess or deficit, also
referred to as the whole blood buffer base, measures
the level of all the buffer systems of the blood
hemoglobin, protein, phosphate, and HCO3. For
clinical purposes, base excess or deficit can be viewed
as a measurement of bicarbonate excess or deficit. Immune system
(cytokines)
15. Answer: a Hypothalamus
RATIONALE: Metabolic disorders produce an CRF
alteration in the plasma HCO3 concentration and
result from the addition or loss of nonvolatile acid
or alkali to or from the extracellular fluids. None Adrenal
Brain stem
Locus
of the other answers are correct. gland
Ceruleus
16. Answer: b Cortisol Pituitary
ACTH
RATIONALE: Often, compensatory mechanisms are
Autonomic
interim measures that permit survival while the nervous system
body attempts to correct the primary disorder. All manifestations

of the other answers are wrong.


17. Answer: c
RATIONALE: The anion gap is often useful in deter-
mining the cause of the metabolic acidosis. None
of the other tests are used to determine the cause Activity C
of metabolic acidosis.
1. d 2. j 3. g 4. c 5. f
18. Answer: d
6. e 7. a 8. h 9. i 10. b
RATIONALE: A fall in pH to less than 7.0 to 7.10 can
reduce cardiac contractility and predispose to Activity D
potentially fatal cardiac dysrhythmias. No other 1. Negative feedback mechanisms are the primary
answer is correct. mechanism used to maintain homeostasis. The
19. Answers: a, b, c negative feedback mechanism that controls blood
RATIONALE: Elevated levels of CO2 produce vasodi- glucose levels, an increase in blood glucose stimu-
lation of cerebral blood vessels, causing headache, lates an increase in insulin, which enhances the

Copyright 2011. Wolters Kluwer Health | Lippincott Williams & Wilkins. Study Guide for Porths Essentials of Pathophysiology, Third Edition.
LWBK707-Ans_p280-408.qxd 9/2/10 8:02 AM Page 302 Aptara Inc

302 ANSWER KEY

removal of glucose from the blood. When glucose


has been taken up by cells and blood glucose levels
SECTION III: PRACTICING FOR NCLEX
fall, insulin secretion is inhibited and glucagon and Activity E
other counterregulatory mechanisms stimulate the 1. Answer: a
release of glucose from the liver, which causes the RATIONALE: The bodys control systems regulate cel-
blood glucose to return to normal. lular function, control life processes, and integrate
2. The stages of general adaptation syndrome are functions of the different organ systems. Homeo-
the alarm stage, the resistance stage, and the static control systems do not feed cells when they
exhaustion stage. The alarm stage is characterized are under stress, they do not act on invading organ-
by a generalized stimulation of the sympathetic isms, and they do not shut down the body at
nervous system and the hypothalamic-pituitary- death.
adrenocortical axis, resulting in the release of cate- 2. Answer: b
cholamines and cortisol. During the resistance RATIONALE: A homeostatic control system consists
stage, the body selects the most effective and eco- of a collection of interconnected components that
nomic channels of defense. During this stage, the function to keep a physical or chemical parameter
increased cortisol levels present during the first of the body relatively constant. Organ systems
stage drop because they are no longer needed. If are a group of organs that function together to
the stressor is prolonged or overwhelms the abil- accomplish necessary functions in the body; for
ity of the body to defend itself, the exhaustion example, the cardiovascular system provides
stage ensues, during which resources are depleted blood to all the bodys components. Biochemical
and signs of wear and tear or systemic damage messengers are in the brain; they are not control
appear. systems. Neuroendocrine systems are control sys-
3. The results of the coordinated release of these neu- tems that help to regulate our response to stress.
rohormones include the mobilization of energy, a Neurovascular systems do not aid in the control
sharpened focus and awareness, increased cerebral of homeostasis in the body.
blood flow and glucose utilization, enhanced 3. Answer: c
cardiovascular and respiratory functioning, redistri- RATIONALE: Selye contended that many ailments,
bution of blood flow to the brain and muscles, such as various emotional disturbances, mildly
modulation of the immune response, inhibition of annoying headaches, insomnia, upset stomach,
reproductive function, and decrease in appetite. gastric and duodenal ulcers, certain types of rheu-
4. Many organs are functioning at much less than matic disorders, and cardiovascular and kidney
maximum capacity, giving the organ a safety mar- diseases, appear to be initiated or encouraged by
gin. The safety margin for adaptation of most body the body itself because of its faulty adaptive reac-
systems is considerably greater than that needed for tions to potentially injurious agents. Psychotic
normal activities. The red blood cells carry more disorders are not caused by stress. Osteogenesis
oxygen than the tissues can use, the liver and fat refers to the origin of bone tissue; this is not due
cells store excess nutrients, and bone tissue stores to stress. Sarcomas are a type of cancer. There is no
calcium in excess of that needed for normal neuro- such thing as osteogenesis sarcomas. Infections in
muscular function. Many of the body organs, such the head and neck are caused by bacterial or viral
as the lungs, kidneys, and adrenals, are paired to invaders of the body; they are not due to stress.
provide anatomic reserve as well. Both organs are 4. Answer: d
not needed to ensure the continued existence and RATIONALE: The results of the coordinated release of
maintenance of the internal environment. As they these neurohormones include the mobilization of
expend greater amounts of energy, athletes are able energy, a sharpened focus and awareness, increased
to tap into these reserves. cerebral blood flow and glucose utilization, enhanced
5. Physiologic symptoms arise from exaggerated sym- cardiovascular and respiratory functioning, redis-
pathetic nervous system activation in response to tribution of blood flow to the brain and muscles,
the traumatic event. Persons with chronic post- modulation of the immune response, inhibition of
traumatic stress disorder (PTTSD) have been reproductive function, and decrease in appetite.
shown to have increased levels of norepinephrine 5. Answer: a
and increased activity of 2-adrenergic receptors. RATIONALE: Diseases of the cardiovascular, gastroin-
The increases in catecholamines, in tandem with testinal, immune, and neurologic systems, as well
increased thyroid levels in persons with PTSD, are as depression, chronic alcoholism and drug abuse,
thought to explain some of the intrusive and eating disorders, accidents, and suicide have all
somatic symptoms of the disorder. In the CNS, been linked to the chronic and excessive
reactivity of the amygdala and hippocampus and activation of the stress response.
decreased reactivity of the anterior cingulate and 6. Answer: b
orbitofrontal areas and are thought to contribute RATIONALE: The response to physiologic distur-
to PTSD also. bances that threaten the integrity of the internal

Copyright 2011. Wolters Kluwer Health | Lippincott Williams & Wilkins. Study Guide for Porths Essentials of Pathophysiology, Third Edition.
LWBK707-Ans_p280-402.qxd 8/20/10 6:38PM Page 303 Aptara Inc

ANSWER KEY 303

environment is specific to the threat; the body usu- nal environment is compatible with the survival
ally does not raise the body temperature when needs of the individual cells.
an increase in heart rate is needed. In contrast, 12. Answer: eustress
the response to psychological disturbances is not RATIONALE: Selye suggested that mild, brief, and
regulated with the same degree of specificity and controllable periods of stress could be perceived as
feedback control; instead, the effect may be inap- positive stimuli to emotional and intellectual
propriate and sustained. No systems in the body are growth and development. These periods of stress
regulated by a positive feedback system. In cardio- are called eustress.
vascular physiology, the baroreflex or baroreceptor 13. Answers: b, d
reflex is one of the bodys homeostatic mecha- RATIONALE: The treatment of stress should be
nisms for maintaining blood pressure. It has directed toward helping people avoid coping
nothing to do with the bodys response to a psy- behaviors that impose a risk to their health and
chological threat. providing them with alternative stress-reducing
7. Answer: c strategies. Nonpharmacologic methods used for
RATIONALE: The ability of body systems to increase stress reduction are relaxation techniques, guided
their function given the need to adapt is known as imagery, music therapy, massage, and biofeedback.
the physiologic reserve. Many of the body organs, 14. Answers: 1-b, Corticotropin-releasing factor is a small
such as the lungs, kidneys, and adrenals, are paired peptide hormone found in both the hypothalamus
to provide anatomic reserve as well. Both organs and in extrahypothalamic structures, such as the
are not needed to ensure the continued existence limbic system and the brain stem. It is both an
and maintenance of the internal environment. important endocrine regulator of pituitary and
Genetic endowment, physiologic reserve, and adrenal activity and a neurotransmitter involved
health status are all coping mechanisms but they in autonomic nervous system activity, metabolism,
do not impact the bodys need to survive when one and behavior.
organ of a pair is missing. 2-d, The sympathetic nervous system manifestation
8. Answer: c of the stress reaction has been called the fight-or-
RATIONALE: The configuration of significant others flight response. This is the most rapid of the stress
that constitutes the social network functions to responses and represents the basic survival response
mobilize the resources of the person; these friends, of our primitive ancestors when confronted with
colleagues, and family members share the persons the perils of the wilderness and its inhabitants.
tasks and provide monetary support, materials and 3-c, The term allostasis has been used by some
tools, and guidance in improving problem-solving investigators to describe the physiologic changes
capabilities. Social networks cannot protect the in the neuroendocrine, autonomic, and immune
person from other internal stressors. systems that occur in response to either real or
9. Answer: d perceived challenges to homeostasis. The
RATIONALE: In persons with limited coping persistence and/or accumulation of these allostatic
abilities, either because of physical or mental changes (e.g., immunosuppression, activation of
health, the acute stress response may be detrimen- the sympathetic nervous and renin-angiotensin-
tal. This is true of persons with pre-existing heart aldosterone systems) has been called an allostatic
disease in whom the overwhelming sympathetic load, and this concept has been used to measure
behaviors associated with the stress response can the cumulative effects of stress on humans.
lead to arrhythmias. The acute stress response is 4-a, The hallmark of the stress response, as first
not necessarily going to be detrimental to the described by Selye, is the endocrine-immune inter-
client who has undergone the resection of a brain actions (i.e., increased corticosteroid production
tumor or is a schizophrenic client who is off his or and atrophy of the thymus) that are known to
her medication, or a client with a broken femur. suppress the immune response. In concert, these
10. Answer: a two components of the stress system, through
RATIONALE: Posttraumatic stress disorder is an exam- endocrine and neurotransmitter pathways,
ple of chronic activation of the stress response as a produce the physical and behavioral changes
result of experiencing a severe trauma. In this disor- designed to adapt to acute stress.
der, memory of the traumatic event seems to be 15. Answers: a, b, c
enhanced. Flashbacks of the event are accompanied RATIONALE: The most significant arguments for inter-
by intense activation of the neuroendocrine system. action between the neuroendocrine and immune
Chronic renal insufficiency, schizophrenia, and systems derive from evidence that the immune and
postdelivery depression in a new mother are not neuroendocrine systems share common signal path-
the result of chronic activation of the stress ways (i.e., messenger molecules and receptors), that
response following a severe trauma. hormones and neuropeptides can alter the function
11. Answer: multicellular of immune cells, and that the immune system and its
RATIONALE: A multicellular organism is able to sur- mediators can modulate neuroendocrine function.
vive only as long as the composition of the inter- These systems do not need each other to function.

Copyright 2011. Wolters Kluwer Health | Lippincott Williams & Wilkins. Study Guide for Porths Essentials of Pathophysiology, Third Edition.
LWBK707-Ans_p280-402.qxd 8/20/10 6:38PM Page 304 Aptara Inc

304 ANSWER KEY

CHAPTER 10 neonatal life in humans and in animals that hiber-


nate. In humans, brown fat decreases with age but
is still detectable in the sixth decade. This small
SECTION II: ASSESSING YOUR amount of brown fat has a minimal effect on
UNDERSTANDING energy expenditure.
Activity A 2. Bioimpedance is performed by attaching electrodes
1. Nutritional status at the wrist and ankle that send a harmless current
2. Metabolism through the body. The flow of the current is affected
3. voluntary physical activity by the amount of water in the body. Because fat-free
4. adipose tissue contains virtually all the water and the con-
5. Leptin ducting electrolytes, measurements of the resistance
6. Recommended Dietary Allowance (RDA) (i.e., impedance) to current flow can be used to esti-
7. Percent daily value mate the percentage of body fat present.
8. Proteins 3. Family eating patterns, inactivity because of labor-
9. nitrogen saving devices and time spent on the computer and
10. elevate, lower watching television, reliance on the automobile for
11. increase, decrease transportation, easy access to food, energy density
12. carbohydrates of food, increased consumption of sugar-sweetened
13. Vitamins beverages, and increasing portion sizes. The obese
14. Fiber may be greatly influenced by the availability of
15. hypothalamus food, the flavor of food, time of day, and other
16. glucose cues. The composition of the diet also may be a
17. Anthropometric causal factor, and the percentage of dietary fat inde-
18. body mass index (BMI) pendent of total calorie intake may play a part in
19. circumference the development of obesity. Psychological factors
20. Obesity include using food as a reward, comfort, or means
21. fat distribution of getting attention. Eating may be a way to cope
22. Weight cycling with tension, anxiety, and mental fatigue. Some
23. increased persons may overeat and use obesity as a means of
24. Bariatric surgery avoiding emotionally threatening situations.
25. pediatric 4. The causes of anorexia appear to be multifactorial,
26. Malnutrition, starvation with determinants that include genetic influence,
27. starvation personality traits of perfectionism and compulsive-
28. marasmus ness, anxiety disorders, family history of depression
29. recurrent and obesity, and peer, familial, and cultural
pressures with respect to appearance. The DSM-IV-
Activity B TR diagnostic criteria for anorexia nervosa are (1) a
1. refusal to maintain a minimally normal body
1. f 2. d 3. b 4. h 5. i weight for age and height (e.g., at least 85% of min-
6. c 7. a 8. g 9. j 10. e imal expected weight or BMI 17.5); (2) an intense
2. fear of gaining weight or becoming fat; (3) a distur-
1. f 2. c 3. b 4. a 5. g bance in the way ones body size, weight, shape is
6. d 7. e perceived; and (4) amenorrhea (in girls and women
Activity C after menarche). Other psychiatric disorders often
coexist with anorexia nervosa, including major
1. There are two types of adipose tissue: white fat and
depression or dysthymia, and obsessive-compulsive
brown fat. White fat is the prevalent form. At body
disorder. Alcohol and substance abuse may also be
temperature, the lipid content of fat cells exists
present, more often among those with binging-
as an oil of triglycerides. Triglycerides have the
purging type of anorexia nervosa.
highest caloric content of all nutrients and are an
5. The criteria to diagnose bulimia nervosa are: (1)
efficient form of energy storage. Fat cells synthesize
recurrent binge eating (at least two times per week
triglycerides, from dietary fats and carbohydrates.
for 3 months); (2) inappropriate compensatory
When calorie intake is restricted for any reason, fat
behaviors such as self-induced vomiting, abuse of
cell triglycerides are broken down and the resultant
laxatives or diuretics, fasting, or excessive exercise
fatty acids and glycerol are released as energy sources.
that follow the binge-eating episode; (3) self-
Brown fat differs from white fat in terms of its ther-
evaluation that is unduly influenced by body shape
mogenic capacity or ability to produce heat. Brown
and weight; and (4) a determination that the eating
fat, the site of diet-induced thermogenesis and non-
disorder does not occur exclusively during episodes
shivering thermogenesis, is found primarily in early
of anorexia nervosa.

Copyright 2011. Wolters Kluwer Health | Lippincott Williams & Wilkins. Study Guide for Porths Essentials of Pathophysiology, Third Edition.
LWBK707-Ans_p280-402.qxd 8/20/10 6:38PM Page 305 Aptara Inc

ANSWER KEY 305

6. Binge eating is characterized by recurrent episodes and adrenocortical hormones) that regulate energy
of compulsive eating at least 2 days per week for balance and metabolism. Cholecystokinin (CCK)
6 months and at least three of the following: and glucagon-like peptide-1 (GLP-1) are intestinal
(1) eating rapidly; (2) eating until becoming uncom- hormones. Ghrelin is secreted mostly in the stomach.
fortably full; (3) eating large amounts when not 5. Answer: d
hungry; (4) eating alone because of embarrassment; RATIONALE: The body mass index (BMI) uses
and (5) disgust, depression, or guilt because of eat- height and weight to determine healthy weight
ing episodes. (Table 10-2). It is calculated by dividing the weight
in kilograms by the height in meters squared
SECTION III: APPLYING YOUR KNOWLEDGE (BMI  weight [kg]/height [m2]). The other answers
are incorrect.
Activity D 6. Answer: a
1. Questions include: RATIONALE: The obesity type is determined by divid-
Do you consider yourself a perfectionist? ing the waist by the hip circumference. The other
Do you do things compulsively? answers are incorrect.
Is there a family history of obesity? 7. Answer: b
Is anyone in your family overweight? RATIONALE: Compared with women, men tend to
Does anyone in your family have an anxiety disorder? experience less pressure to engage in behaviors
Does anyone in your family have a history of such as self-induced vomiting or laxative use when
depression? overeating, less on a subjective sense or loss of con-
2. Criteria include: trol when binge eating, and a greater tendency to
Refusal to maintain a minimally normal body use compulsive exercise rather than purging for
weight for age and height weight control.
An intense fear of gaining weight or becoming fat 8. Answer: b
A disturbance in the way ones body size, weight, RATIONALE: Pediatricians are now beginning to
shape is perceived see hypertension, dyslipidemia, type II diabetes,
Amenorrhea (in girls and women after menarche) and psychosocial stigma in obese children and ado-
lescents. The other answers are not correct.
SECTION IV: PRACTICING FOR NCLEX 9. Answer: c
Activity E RATIONALE: The hospitalized patient often finds eat-
ing a healthful diet difficult and commonly has
1. Answers: a, c, e restrictions on food and water intake in preparation
RATIONALE: The factors secreted by adipose tissue
for tests and surgery. Pain, medications, special
are termed adipokines and include leptin, certain diets, and stress can decrease appetite. Even when
cytokines (e.g., tumor necrosis factor-), growth the patient is well enough to eat, being alone in a
factors, and adiponectin (important in insulin room, where unpleasant treatments may be given,
resistance). is not conducive to eating. The other answers are
2. Answer: a not correct.
RATIONALE: An estimated average requirement is the
intake that meets the estimated nutrient need of
half of the persons in a specific group. The adequate CHAPTER 11
intake is set when there is not enough scientific evi-
dence to estimate an average requirement. The Rec- SECTION II: ASSESSING YOUR
ommended Dietary Allowance (RDA) defines the UNDERSTANDING
intakes that meet the nutrient needs of almost all
healthy persons in a specific age and sex group. Activity A
The Dietary Reference Intake includes a set of at 1. granulocytes
least four nutrient-based reference valuesthe rec- 2. thymus
ommended dietary allowance, the adequate intake, 3. plasma
the estimated average requirement, and the tolera- 4. natural killer
ble upper intake level. 5. myeloid, lymphoid
3. Answer: b 6. thymus, spleen
RATIONALE: The Food and Nutrition Board has set 7. CD4 , CD8
an acceptable macronutrient distribution range for 8. aplastic
fat of no less than 20% to prevent the fall of HDL 9. Agranulocytosis
cholesterol associated with very low fat diets. The 10. neutropenia
other answers are incorrect. 11. Infectious mononucleosis
4. Answer: c 12. Leukemias
RATIONALE: Centers in the hypothalamus also con- 13. Lymphomas
trol the secretion of several hormones (e.g., thyroid 14. B

Copyright 2011. Wolters Kluwer Health | Lippincott Williams & Wilkins. Study Guide for Porths Essentials of Pathophysiology, Third Edition.
LWBK707-Ans_p280-402.qxd 8/20/10 6:38PM Page 306 Aptara Inc

306 ANSWER KEY

15. nodular, lymphocyte fungal infections, and a poor humoral antibody


16. Hodgkin lymphoma response.
17. leukemias 4. First, Hodgkin lymphoma usually arises in a single
18. lymphocytic node or chain of nodes, while NHL frequently
19. leukemia originates at extranodal sites and spreads to
20. Acute leukemias anatomically contiguous nodes. Second, Hodgkin
21. lymphocytic, monocytic lymphoma is characterized by the presence of large,
22. Chronic leukemias atypical, mononuclear tumor cells, called Reed-
23. B lymphocytes Sternberg cells. The cells, which frequently consti-
24. Kostmann tute less than 1% of the total cell population, are a
25. Philadelphia diagnostic hallmark of the disease.
26. Plasma cell dyscrasias 5. The incidence of leukemia among persons who
27. bone have been exposed to high levels of radiation is
28. M protein unusually high. An increased incidence of leukemia
Activity B also is associated with exposure to benzene and the
use of antitumor drugs. Leukemia may occur as a
1. e 2. d 3. b 4. a 5. f
second cancer after aggressive chemotherapy for
6. h 7. i 8. c 9. g 10. j
other cancers. The existence of a genetic predisposi-
Activity C tion to develop acute leukemia is suggested by the
1. Neutrophils migrate to sites of infection and increased leukemia incidence among a number of
engulf, digest, and destroy microorganisms. Thus, a congenital disorders. In individuals with Down
decrease in the number of neutrophils places a per- syndrome, the incidence of acute leukemia is 10
son at risk for infection. The risk for and severity of times that of the general population. Also there are
neutropenia-associated infection are directly numerous reports of multiple cases of acute
proportional to the absolute neutrophil count and leukemia occurring within the same family.
duration of the neutropenia (defined as a circulat- 6. Both are characterized by an abrupt onset of symp-
ing neutrophil count of less than 1500/mL). toms including fatigue resulting from anemia; low-
2. The Epstein-Barr virus (EBV) initially penetrates grade fever, night sweats, and weight loss due to
the nasopharyngeal, oropharyngeal, and salivary the rapid proliferation and hypermetabolism of the
epithelial cells. It then spreads to the underlying leukemic cells; bleeding because of a decreased
oropharyngeal lymphoid tissue and, more specifi- platelet count; and bone pain and tenderness from
cally, to B lymphocytes, all of which have receptors bone marrow expansion. Infection results from
for EBV. Infection of the B cells may take one of neutropenia. Generalized lymphadenopathy,
two forms: it may kill the infected B cell or it may splenomegaly, and hepatomegaly caused by infiltra-
become incorporated into its genome. The B cells tion of leukemic cells occur in all acute leukemias
that harbor the EBV genome proliferate in the cir- but are more common in acute lymphoblastic
culation and produce the well-known heterophil leukemia (ALL). In addition to the common mani-
antibodies that are used for the diagnosis of infec- festations of acute leukemia, infiltration of malig-
tious mononucleosis. The resultant destruction of nant cells in the skin, gums, and other soft tissue is
B cells and production of large T cells result in particularly common in the monocytic form of
enlarged lymph nodes, particularly in the cervical, acute myelogenous leukemia (AML). The leukemic
axillary, and groin areas. Hepatitis and cells may also cross the blood-brain barrier and
splenomegaly are common manifestations of the establish sanctuary in the CNS. The CNS involvement
disease and are thought to be immune-mediated. is more common in ALL than AML and is more
3. The manifestations of non-Hodgkin lymphoma common in children than adults. Signs and symp-
(NHL) depend on lymphoma type and the stage toms of CNS involvement include cranial nerve
of the disease. Persons with indolent or slow-growing palsies, headache, nausea, vomiting, papilledema,
lymphomas usually present with painless lym- and, occasionally, seizures and coma. Leukostasis
phadenopathy due to increased cell filtering, and blood clotting are seen in severe cases.
which may be isolated or widespread. The indolent 7. The early chronic stage is marked by leukocytosis,
lymphomas are usually disseminated at the time anemia, and thrombocytopenia. Splenomegaly and
of diagnosis, and bone marrow involvement is hepatomegaly are often present. The accelerated
frequent. Many low-grade lymphomas eventu- phase of CML is characterized by enlargement of
ally transform into more aggressive forms of the spleen, resulting in a feeling of abdominal full-
lymphoma/leukemia. Persons with intermediate ness and discomfort. An increase in basophil count
or more aggressive forms of lymphoma usually and more immature cells in the blood or bone mar-
present with symptoms such as fever, drenching row confirm transformation to the accelerated
night sweats, or weight loss. Frequently, there are phase. Symptoms such as low-grade fever, night
increased susceptibility to bacterial, viral, and sweats, bone pain, and weight loss develop because

Copyright 2011. Wolters Kluwer Health | Lippincott Williams & Wilkins. Study Guide for Porths Essentials of Pathophysiology, Third Edition.
LWBK707-Ans_p280-402.qxd 8/20/10 6:38PM Page 307 Aptara Inc

ANSWER KEY 307

of rapid proliferation and hypermetabolism of the inactive follicles, called primary follicles, and
leukemic cells. Bleeding and easy bruising may arise active follicles that contain germinal centers called
from dysfunctional platelets. The terminal blast secondary follicles. There is no primary cortex in
crisis phase of CML represents evolution to acute the lymph nodes.
leukemia and is characterized by an increasing 3. Answer: d
number of myeloid precursors, especially blast cells, RATIONALE: Severe congenital neutropenia, or
in the blood. Constitutional symptoms become Kostmann syndrome, is characterized by an arrest
more pronounced during this period, and in myeloid maturation at the promyelocyte stage
splenomegaly may increase significantly. Isolated of development resulting in an absolute neutrophil
infiltrates of leukemic cells can involve the skin, count of less than 200 cells/ L. The disorder
lymph nodes, bones, and CNS. is characterized by severe bacterial infections.
8. The cause of multiple myeloma is unknown. Risk Kostmann syndrome is not characterized by bone
factors are thought to include chronic immune marrow disorders, viral infections, or autoimmune
stimulation, autoimmune disorders, exposure to disorders.
ionizing radiation, and occupational exposure to 4. Answer: a
pesticides or herbicides. Myeloma has been associ- RATIONALE: The incidence of drug-induced
ated with exposure to Agent Orange during the neutropenia has increased significantly over the
Vietnam War. A number of viruses have been asso- last several decades and is attributed primarily to a
ciated with the pathogenesis of myeloma. There is a wider use of drugs in general and more specifically
4.5-fold increase in the likelihood of developing to the use of chemotherapeutic drugs in the treat-
myeloma for persons with HIV. ment of cancer.
5. Answer: b
SECTION III: APPLYING YOUR KNOWLEDGE RATIONALE: Hepatitis and splenomegaly are common
manifestations of infectious mononucleosis and
Activity D
are thought to be immune-mediated. Hepatitis is
1. The causes of leukemia are really unknown. We do characterized by hepatomegaly, nausea, anorexia,
know that the event or events causing the and jaundice. Although discomforting, it usually is a
leukemias exert their effects through disruption or benign condition that resolves without causing
dysregulation of genes that normally regulate blood permanent liver damage. The spleen may be
cell development, blood cell stability, or both. enlarged two to three times its normal size, and
2. Treatment of ALL consists of a number of rupture of the spleen is an infrequent complica-
chemotherapeutic agents designed to achieve tion. Cranial nerve palsies, not peripheral nerve
remission followed by high doses of chemotherapy palsies, can occur. Lymph nodes do not rupture.
given to patients who have achieved remission Severe bacterial infections are complications of
with their induction therapy. This part of Lucys Kostmann syndrome.
treatment is designed to reduce the number of can- 6. Answer: c
cer cells in her body even more once remission has RATIONALE: Non-Hodgkin lymphomas represent
been achieved. Then she will receive lower doses of the cancer with the second fastest rate of increase
chemotherapy given over a long period of time in in the United States, and the most commonly
an attempt to cure her. occurring hematologic cancer. Neoplasms of
immature B cells include lymphoblastic leukemia/
SECTION IV: PRACTICING FOR NCLEX lymphoma (i.e., ALL). They are not classed as
NHLs. Mantle cell lymphoma is one of the mature
Activity E
B-cell lymphomas.
1. Answer: a 7. Answer: d
RATIONALE: A small population of cells called RATIONALE: Endemic Burkitt lymphoma is the most
pluripotent stem cells are capable of providing pro- common childhood cancer (peak age 3 to 7 years)
genitor cells, or parent cells, for myelopoiesis and in central Africa, often beginning in the jaw. It
lymphopoiesis, processes by which myeloid and occurs in regions of Africa where both EBV and
lymphoid blood cells are made. Unipotent cells are malaria infection are common. Neither herpes
the progenitors for each of the blood cell types and zoster nor streptococcal infections are associated
come from pluripotent stem cells. Multipotential with endemic Burkitt lymphoma.
progenitor cells act as parent cells for multiple types 8. Answer: a
of blood cells. Myeloproliferative cells do not exist. RATIONALE: Although ALL and AML are distinct
2. Answer: b disorders, they typically present with similar clini-
RATIONALE: The portion of the cortex between the cal features. Both are characterized by an abrupt
medullary and superficial cortex is called the para- onset of symptoms including fatigue resulting
cortex. The region contains most of the T cells in from anemia; low-grade fever, night sweats, and
the lymph nodes. The B-celldependent cortex weight loss due to the rapid proliferation and
consists of two types of follicles: immunologically hypermetabolism of the leukemic cells; bleeding

Copyright 2011. Wolters Kluwer Health | Lippincott Williams & Wilkins. Study Guide for Porths Essentials of Pathophysiology, Third Edition.
LWBK707-Ans_p280-402.qxd 8/20/10 6:38PM Page 308 Aptara Inc

308 ANSWER KEY

because of a decreased platelet count; and bone available, the importance of the late effects of
pain and tenderness due to bone marrow expan- treatment, including secondary malignancies, has
sion. Polycythemia is an increase in the erythrocytes become more apparent. Because these malignancies
in the blood. It is not an indication of leukemia. have mainly been attributed to radiation therapy,
9. Answer: b studies are being conducted to determine the low-
RATIONALE: Diagnosis of multiple myeloma is est effective radiation dose.
based on clinical manifestations, blood tests, and
bone marrow examination. The classic triad of
bone marrow plasmacytosis (more than 10% CHAPTER 12
plasma cells), lytic bone lesions, and either the
serum M-protein spike or the presence of Bence- SECTION II: ASSESSING YOUR
Jones proteins in the urine is definitive for a diag- UNDERSTANDING
nosis of multiple myeloma. Oligoclonal bands are
Activity A
indicative of multiple sclerosis and BCR-ABL
fusion protein is found in CML. 1. hemostasis
10. Answer: c 2. nucleus
RATIONALE: Hypogammaglobulinemia is common 3. actin, myosin
in CLL, especially in persons with advanced 4. growth factors
disease. An increased susceptibility to infection 5. ADP, TXA2
reflects an inability to produce specific antibodies 6. coagulation cascade
and abnormal activation of complement. The 7. liver
most common infectious organisms are those that 8. disseminated intravascular coagulation (DIC)
require opsonization for bacterial killing, such as 9. Hypercoagulability
Streptococcus pneumoniae, Staphylococcus aureus, and 10. Smoking
Haemophilus influenzae. Acne rosacea, Pseudomonas 11. thrombocytosis
aeruginosa, and Escherichia coli are not infectious 12. protein C
agents common in clients with CLL. 13. coagulation
11. Answer: lyse 14. Bleeding
12. Answer: c 15. thrombocytopenia
RATIONALE: The alimentary canal, respiratory pas- 16. Platelet
sages, and genitourinary systems are guarded by 17. Immune
accumulations of lymphatic tissue that are not 18. Thrombocytopathia
enclosed in a capsule. This form of lymphatic tis- 19. X-linked.
sue is called diffuse lymphatic tissue or mucus- 20. clotting factors
associated lymphatic tissue (MALT) because of its 21. scurvy
association with mucous membranes. Lymphocytes 22. DIC
are found in the subepithelial of these tissues. Lym- Activity B
phomas can arise from MALT as well as lymph 1.
node tissue. The cardiovascular system and the cen-
tral nervous system do not have MALT. Intrinsic system
13. Answers: b, c, d (blood or vessel injury)

RATIONALE: The existence of a genetic predisposition XII XIIa


to develop acute leukemia is suggested by the
increased leukemia incidence among a number of XI XIa
congenital disorders, including Down syndrome, Extrinsic system
neurofibromatosis, and Fanconi anemia. Cushing IX
IXa (tissue factor)
Ca++
syndrome is not a genetic disorder, nor is Prader- VIII VIIa VII
Ca++
Willi syndrome. Thrombin
14. Answers: a, c, e
VIIIa
RATIONALE: Massive necrosis of malignant cells can
occur during the initial phase of treatment. This Xa
X X
phenomenon, known as tumor lysis syndrome, can Ca++
lead to life-threatening metabolic disorders, including
hyperkalemia, hyperphosphatemia, hyperuricemia, Prothrombin Thrombin
Ca++
hypomagnesemia, hypocalcemia, and acidosis, with
Fibrinogen Fibrin (monomer)
the potential for causing acute renal failure.
15. Answer: radiation
RATIONALE: As the cure rate for Hodgkin lymphoma Fibrin (polymer)
has risen and longer-term follow-up data became

Copyright 2011. Wolters Kluwer Health | Lippincott Williams & Wilkins. Study Guide for Porths Essentials of Pathophysiology, Third Edition.
LWBK707-Ans_p280-402.qxd 8/20/10 6:38PM Page 309 Aptara Inc

ANSWER KEY 309

Activity C resulting in systemic formation of fibrin. In


addition, levels of all the major anticoagulants are
1. c 2. g 3. i 4. e 5. a
reduced. The microthrombi that result cause vessel
6. f 7. j 8. h 9. d 10. c
occlusion and tissue ischemia. Multiple organ fail-
Activity D ure may ensue. Clot formation consumes all avail-
1. able coagulation proteins and platelets, and severe
hemorrhage results.
e S d S c S a S b
SECTION III: PRACTICING FOR NCLEX
Activity E Activity F
1. Answer: a
1. (1) Vessel spasm, which constricts the vessel and
RATIONALE: Platelet adhesion requires a protein
reduces blood flow, (2) formation of the platelet
molecule called von Willebrand factor. This factor
plug initiated platelet contact with subendothelial
is produced by the endothelial cells of blood ves-
tissue, (3) blood coagulation via fibrin polymeriza-
sels and circulates in the blood as a carrier protein
tion, (4) clot retraction in order to squeeze out
for coagulation factor VIII. The release of growth
serum, and (5) clot dissolution by fibrinolysis by
factors results in the proliferation and growth of
plasminogen.
vascular endothelial cells, smooth muscle cells,
2. Platelets are attracted to a damaged vessel wall,
and fibroblasts, and is important in vessel repair.
become activated, and change from smooth disks
Ionized calcium contributes to vasoconstriction.
to spiny spheres, exposing glycoprotein receptors
Platelet factor 4 is a heparin-binding chemokine.
on their surfaces. Platelet adhesion requires a protein
2. Answer: b
molecule called von Willebrand factor, which is pro-
RATIONALE: The coagulation process results from
duced by the endothelial cells of blood vessels
the activation of what has traditionally been
and circulates in the blood as a carrier protein
designated the intrinsic or the extrinsic pathways.
for coagulation factor VIII. Adhesion to the vessel
The intrinsic pathway, which is a relatively slow
subendothelial layer occurs when the platelet recep-
process, begins in the circulation with the activa-
tor binds to von Willebrand factor at the injury site,
tion of factor XII. The extrinsic pathway, which is
linking the platelet to exposed collagen fibers.
a much faster process, begins with trauma to the
3. The intrinsic pathway, which is a relatively slow
blood vessel or surrounding tissues and the release
process, begins in the circulation with the
of tissue factor, an adhesive lipoprotein released
activation of factor XII, which is activated as blood
from the subendothelial cells. The terminal steps
comes in contact with collagen in the injured ves-
in both pathways are the same: the activation of
sel wall. The extrinsic pathway, which is a much
factor X and the conversion of prothrombin to
faster process, begins with trauma to the blood ves-
thrombin. All other answers do not exist in the
sel or surrounding tissues and the release of tissue
formation of clots.
factor, an adhesive lipoprotein released from the
3. Answer: a
subendothelial cells. The terminal steps in both
RATIONALE: The anticoagulant drugs warfarin
pathways are the same: the activation of factor X
and heparin are used to prevent thromboembolic
and the conversion of prothrombin to thrombin.
disorders, such as deep vein thrombosis and pul-
4. These drugs act as haptens and induce
monary embolism. Warfarin acts by decreasing
antigenantibody response and formation of
prothrombin and other procoagulation factors. It
immune complexes that cause platelet destruction
alters vitamin K in a manner that reduces its abil-
by complement-mediated lysis (see Chapter 15).
ity to participate in synthesis of the vitamin
In persons with drug-associated thrombocyto-
Kdependent coagulation factors in the liver.
penia, there is a rapid fall in the platelet count
4. Answer: b
within 2 to 3 days of resuming a drug or
RATIONALE: Heparin binds to antithrombin III,
7 or more days (i.e., the time needed to mount
causing a conformational change that increases
an immune response) after starting a drug for the
the ability of antithrombin III to inactivate throm-
first time.
bin, factor Xa, and other clotting factors. By
5. Activation through the extrinsic pathway occurs
promoting the inactivation of clotting factors,
with liberation of tissue factors, associated with
heparin ultimately suppresses the formation of
obstetric complications, trauma, bacterial sepsis,
fibrin. Heparin does not bind to factors X and Xa.
and cancers. The intrinsic pathway may be
Heparin does not inactivate factor VIII.
activated through extensive endothelial damage
5. Answer: b
with activation of factor XII. DIC begins with mas-
RATIONALE: Platelets, through the action of their
sive activation of the coagulation sequence as a
actin and myosin filaments, also contribute to clot
result of unregulated generation of thrombin,

Copyright 2011. Wolters Kluwer Health | Lippincott Williams & Wilkins. Study Guide for Porths Essentials of Pathophysiology, Third Edition.
LWBK707-Ans_p280-402.qxd 8/20/10 6:38PM Page 310 Aptara Inc

310 ANSWER KEY

retraction. Clot retraction therefore requires large uble fibrin clot, (4) clot retraction, and (5) clot dis-
numbers of platelets and failure of clot retraction solution.
is indicative of a low platelet count. Factor Xa is 12. Answer: intravascular
necessary factor in blood coagulation. It does not 13. Answer: Heparin
cause failure of clot retraction. 14. Answers: a, b, c, e
6. Answer: c RATIONALE: Platelets that adhere to the vessel wall
RATIONALE: The common underlying causes of release growth factors that cause proliferation of
secondary thrombocytosis include tissue damage smooth muscle and thereby contribute to the
due to surgery, infection, cancer, and chronic development of atherosclerosis. Smoking, elevated
inflammatory conditions such as rheumatoid levels of blood lipids and cholesterol, hemodynamic
arthritis and Crohn disease. Lyme disease, caused stress, diabetes mellitus, and immune mechanisms
by a tick bite, does not cause thrombocytosis. may cause vessel damage, platelet adherence, and,
Hirschsprung disease and megacolon are the eventually, thrombosis.
same thing, and they are not inflammatory 15. Answers: a, c, e
conditions. RATIONALE: In DIC, microemboli may obstruct
7. Answer: a blood vessels and cause tissue hypoxia and
RATIONALE: A reduction in platelet number, also necrotic damage to organ structures, such as the
referred to as thrombocytopenia, is an important kidneys, heart, lungs, and brain. As a result,
cause of generalized bleeding. Thrombocytopenia common clinical signs may be due to renal, circu-
usually refers to a decrease in the number of circu- latory, or respiratory failure, acute bleeding ulcers
lating platelets to a level less than 100,000/ L. or convulsions and coma. A form of hemolytic
The greater the decrease in the platelet count, the anemia may develop as red cells are damaged as
greater the risk of bleeding. Thrombocytopenic they pass through vessels partially blocked by
can result from a decrease in platelet production, thrombus.
increased sequestration of platelets in the spleen,
or decreased platelet survival.
8. Answer: b CHAPTER 13
RATIONALE: Hemophilia A is an X-linked
recessive disorder that primarily affects males. SECTION II: ASSESSING YOUR
Approximately 90% of persons with hemophilia UNDERSTANDING
produce insufficient quantities of the factor VIII.
Activity A
The prevention of trauma is important in persons
with hemophilia. 1. biconcave, cell membrane
9. Answer: c 2. iron
RATIONALE: In persons with bleeding disorders 3. nucleus
caused by vascular defects, the platelet count and 4. 4
results of other tests for coagulation factors are 5. glycolytic
normal. A shift to the left indicates an infectious 6. methemoglobin
or inflammatory process, not a clotting disorder. A 7. red blood cell count (RBC)
lack of iron indicates iron deficiency anemia, not a 8. hematocrit
clotting disorder. A normal hematocrit indicates a 9. mean corpuscular hemoglobin concentration
normal number of packed red blood cells, not a (MCHC)
clotting disorder. 10. Anemia
10. Answer: a 11. hypoxia
RATIONALE: Disseminated intravascular coagulation 12. Hemolytic
is a paradox in the hemostatic sequence and is 13. sickle cell, thalassemias
characterized by widespread coagulation and 14. spherocytosis
bleeding in the vascular compartment. It is not a 15.
-Thalassemias, -thalassemias
primary disease but occurs as a complication of a 16. glucose-6-phosphatase (G6PD)
wide variety of conditions such as disease or 17. Iron-deficiency
injury, such as septicemia, acute hypotension, poi- 18. chronic blood loss
sonous snake bites, neoplasms, obstetric emergen- 19. Megaloblastic
cies, severe trauma, extensive surgery, and 20. Pernicious
hemorrhage. 21. Aplastic anemia
11. Answer: c 22. Polycythemia
RATIONALE: Hemostasis is divided into five stages: 23. oxygen
(1) vessel spasm, (2) formation of the platelet plug, 24. conjugate
(3) blood coagulation or development of an insol- 25. anemia

Copyright 2011. Wolters Kluwer Health | Lippincott Williams & Wilkins. Study Guide for Porths Essentials of Pathophysiology, Third Edition.
LWBK707-Ans_p280-402.qxd 8/20/10 6:38PM Page 311 Aptara Inc

ANSWER KEY 311

Activity B 3. (1) Manifestations of impaired oxygen transport and


the resulting compensatory mechanisms, (2) reduc-
tion in red cell indices and hemoglobin levels, and
(3) signs and symptoms associated with the patho-
Spleen logic process that is causing the anemia.
4. Premature destruction of the cells due to the rigid
nondeformable membrane occurs in the spleen,
causing hemolysis and anemia due to a decrease
Hemoglobin in red cell numbers. Secondly, vessel occlusion, a
complex process involving an interaction among
the sickled cells, endothelial cells, leukocytes,
Heme Globin platelets, and other plasma proteins will interrupt
blood flow. The adherence of sickled cells to the
Iron Amino acids vessel endothelium causes endothelial activation
(reutilized) with liberation of inflammatory mediators and
Free, unconjugated substances that increase platelet activation and
bilirubin
promote blood coagulation.
5. Exposure to high doses of radiation, chemicals,
Liver and toxins that suppress cellular activity directly or
through immune mechanisms are the standard
Reused by bone cancer treatments. Chemotherapy and irradiation
marrow or stored in commonly result in bone marrow depression,
spleen and liver
which causes anemia, thrombocytopenia, and neu-
tropenia. Identified toxic agents include benzene,
Conjugated bilirubin
the antibiotic chloramphenicol, and the alkylating
Bone agents and antimetabolites used in the treatment of
marrow Secreted in bile; cancer will decrease bone marrow of stem cells,
excreted in feces thus affecting the production of RBCs.
or urine 6. Viscosity rises exponentially with the hematocrit
and interferes with cardiac output and blood flow.
Hypertension is common and there may be
Activity C complaints of headache, dizziness, inability to con-
centrate, and some difficulty with hearing and
1. e 2. a 3. f 4. c 5. i
vision because of decreased cerebral blood flow.
6. j 7. b 8. d 9. h 10. g
Venous stasis gives rise to a plethoric appearance or
Activity D dusky redness, even cyanosis, particularly of the
1. The hemoglobin molecule is composed of two pairs lips, fingernails, and mucous membranes. Because
of structurally different  and
polypeptide chains. of the increased concentration of blood cells, the
Each of the four-polypeptide chains consists of a person may experience itching and pain in the fin-
globin (protein) portion and heme unit, which sur- gers or toes, and the hypermetabolism may induce
rounds an atom of iron that binds oxygen. Thus, night sweats and weight loss.
each molecule of hemoglobin can carry four mole- 7. Hyperbilirubinemia in the neonate is treated with
cules of oxygen. The binding that occurs is cooper- phototherapy or exchange transfusion. Photother-
ative, or allosteric. When one oxygen molecule apy is more commonly used to treat jaundiced
binds, it makes it easier for the next to bind. The infants and reduce the risk of kernicterus. Exposure
process also works in reverse. to fluorescent light in the blue range of the visible
2. A group of large phagocytic cells found in the spectrum (420- to 470-nm wavelength) reduces
spleen, liver, bone marrow, and lymph nodes facili- bilirubin levels. Bilirubin in the skin absorbs the
tates the destruction of RBCs. These phagocytic light energy and is converted to a structural isomer
cells recognize old and defective red cells and then that is more water-soluble and can be excreted in
ingest and destroy them in a series of enzymatic the stool and urine.
reactions. During these reactions, the amino acids
from the globulin chains and iron from the heme SECTION III: APPLYING YOUR KNOWLEDGE
units are salvaged and reused. The bulk of the heme
unit is converted to bilirubin, which is insoluble in Activity E
plasma and attaches to plasma proteins for transport. 1. A sensation of heat along the vein the transfusion
Bilirubin is removed from the blood by the liver is going in
and conjugated with glucuronide to render it Urticaria
water-soluble so that it can be excreted in the bile. Headache

Copyright 2011. Wolters Kluwer Health | Lippincott Williams & Wilkins. Study Guide for Porths Essentials of Pathophysiology, Third Edition.
LWBK707-Ans_p280-402.qxd 8/20/10 6:38PM Page 312 Aptara Inc

312 ANSWER KEY

Pain in the low back bleeding is controlled and sufficient, iron stores
Chills are available. The red cell concentration returns to
Fever normal within 3 to 4 weeks.
Chest pain 5. Answer: c
Abdominal cramps RATIONALE: Chronic blood loss does not affect blood
Nausea volume, but instead leads to iron-deficiency anemia
Vomiting when iron stores are depleted. It is commonly
Tachycardia caused by gastrointestinal bleeding and menstrual
Hypotension disorders. Because of compensatory mechanisms,
Dyspnea patients are commonly asymptomatic until the
RATIONALE: The most feared and lethal transfusion hemoglobin level is less than 8 g/dL. The red cells
reaction is the destruction of donor red cells by that are produced have too little hemoglobin, giv-
reaction with antibody in the recipients serum. ing rise to microcytic hypochromic anemia. Macro-
This immediate hemolytic reaction usually is cytic anemia is when the RBCs are larger than
caused by ABO incompatibility. The signs and normal. Hyperchromic means the cells are a darker
symptoms of such a reaction include sensation of color red then they should be.
heat along the vein where the blood is being 6. Answer: d
infused, flushing of the face, urticaria, headache, RATIONALE: Hemolytic anemia is characterized by
pain in the lumbar area, chills, fever, constricting the premature destruction of red cells, the
pain in the chest, cramping pain in the abdomen, retention in the body of iron and the other prod-
nausea, vomiting, tachycardia, hypotension, and ucts of hemoglobin destruction, and an increase in
dyspnea. erythropoiesis. Almost all types of hemolytic ane-
2. Most transfusion reactions result from administra- mia are distinguished by normocytic and
tive errors or misidentification, and care should be normochromic red cells.
taken to correctly identify the recipient and the 7. Answer: d
transfusion source. RATIONALE: In hemolytic anemia, intravascular
hemolysis is less common than extravascular
SECTION IV: PRACTICING FOR NCLEX hemolysis and occurs as a result of complement
fixation in transfusion reactions, mechanical
Activity F
injury, or toxic factors. It is characterized by hemo-
1. Answer: a globinemia, hemoglobinuria, jaundice, and hemo-
RATIONALE: When RBCs age and are destroyed in siderinuria. Spherocytosis is the most common
the spleen, the iron from their hemoglobin is inherited disorder of the red cell membrane and is
released into the circulation and returned to the not associated with hemolytic anemia.
bone marrow for incorporation into new RBCs or 8. Answer: b
to the liver and other tissues for storage. Iron is RATIONALE: Therapy for aplastic anemia in the
not bound to RBCs in the liver. Iron does not bind young and severely affected includes stem cell
with oxygen in the lung without first being incor- replacement by bone marrow or peripheral blood
porated into an RBC. Iron is stored in tissues of transplantation. Histocompatible donors supply
the body, but not for strength, only for its oxygen- the stem cells to replace the patients destroyed
binding capacity. marrow cells. A liver transplant will not produce
2. Answer: d new blood cells for the body. Spleen transplants
RATIONALE: The plasma-insoluble form of bilirubin are not done and would not produce new blood
is referred to as unconjugated bilirubin and the cells for the body.
water-soluble form as conjugated bilirubin. Serum 9. Answer: a
levels of conjugated and unconjugated bilirubin RATIONALE: Chronic renal failure almost always
can be measured in the laboratory and are results in anemia, primarily because of a deficiency
reported as direct and indirect, respectively. of erythropoietin. Unidentified uremic toxins and
3. Answer: a retained nitrogen also interfere with the actions
RATIONALE: Hyperbilirubinemia, an increased level of erythropoietin, and red cell production and
of serum bilirubin, is a common cause of jaundice survival. Hemolysis and blood loss associated
in the neonate. A benign, self-limited condition, it with hemodialysis and bleeding tendencies also
most often is related to the developmental state of contribute to the anemia of renal failure. Fibrino-
the neonate. Rarely, cases of hyperbilirubinemia gen is essential for blood clotting, not oxygen
are pathologic and may lead to kernicterus and transportation.
serious brain damage. 10. Answer: c
4. Answer: b RATIONALE: Erythroblastosis fetalis, or hemolytic
RATIONALE: It takes about 5 days for the progeny of disease of the newborn, occurs in Rh-positive
stem cells to fully differentiate, an event marked infants of Rh-negative mothers who have been
by increased reticulocytes in the blood. If the sensitized. The Rh-negative mother usually

Copyright 2011. Wolters Kluwer Health | Lippincott Williams & Wilkins. Study Guide for Porths Essentials of Pathophysiology, Third Edition.
LWBK707-Ans_p280-402.qxd 8/20/10 6:38PM Page 313 Aptara Inc

ANSWER KEY 313

becomes sensitized during the first few days after stasis gives rise to a plethoric appearance or dusky
delivery, when fetal Rh-positive red cells from the redness, even cyanosis, particularly of the lips, fin-
placental site are released into the maternal circu- gernails, and mucous membranes.
lation. Because the antibodies take several weeks 16. Answer: transfusion
to develop, the first Rh-positive infant of an Rh- RATIONALE: Persons who are homozygous for the
negative mother usually is not affected. There is trait (thalassemia major) have severe, transfusion-
no such thing as microcytic or macrocytic disease dependent anemia that is evident at 6 to 9 months
of the newborn, nor is there a hemolytic iron-defi- of age when the hemoglobin switches from HbF to
ciency anemia. HbA. If transfusion therapy is not started early in
11. Answer: vitamin B12 life, severe growth retardation occurs in children
RATIONALE: Pernicious anemia is believed to result with the disorder.
from immunologically mediated, possibly autoim-
mune, destruction of the gastric mucosa. The
resultant chronic atrophic gastritis is marked by CHAPTER 14
loss of parietal cells and production of antibodies
that interfere with binding of vitamin B12 to SECTION II: ASSESSING YOUR
intrinsic factor. UNDERSTANDING
12. Answers: a, b, d
Activity A
RATIONALE: Factors associated with sickling and
vessel occlusion include cold, stress, physical exer- 1. commensalism
tion, infection, and illnesses that cause hypoxia, 2. infection
dehydration, or acidosis. 3. parasitic
13. Answers: 1-c, 2-a, 3-b 4. opportunistic
RATIONALE: Red cell indices are used to differentiate 5. transmissible neurodegenerative
types of anemias by size or color of red cells. The 6. Viruses
mean corpuscular volume (MCV) reflects the volume 7. prokaryotes
or size of the red cells. The MCV falls in microcytic 8. Staining
(small cell) anemia and rises in macrocytic (large 9. spirochetes
cell) anemia. Some anemias are normocytic (i.e., cells 10. mycoplasmas
are of normal size or MCV). The mean corpuscular 11. fungal
hemoglobin concentration (MCHC) is the concen- 12. yeasts, molds
tration of hemoglobin in each cell. 13. feces
14. Answers: a, b 14. prodromal stage
RATIONALE: In anemia, the oxygen-carrying capac- 15. acute stage
ity of hemoglobin is reduced, causing tissue 16. convalescent period
hypoxia. Tissue hypoxia can give rise to fatigue, 17. itis
weakness, dyspnea, and sometimes angina. 18. emia
Hypoxia of brain tissue results in headache, faint- 19. Virulence
ness, and dim vision. The redistribution of the 20. exotoxins
blood from cutaneous tissues or a lack of hemo- Activity B
globin causes pallor of the skin, mucous mem-
branes, conjunctiva, and nail beds. Tachycardia
and palpitations may occur as the body tries to Death
compensate with an increase in cardiac output. Critical threshold
Ruddy skin and bradycardia are not signs or symp-
toms of anemia.
replication of pathogens

15. Answers: a, b, e
Severity of illness

RATIONALE: Primary polycythemia, or polycythemia Chronic disease


vera, is a neoplastic disease of the pluripotent cells
of the bone marrow characterized by an absolute
increase in total RBC mass accompanied by elevated
white cell and platelet counts. It most commonly Clinical
is seen in men with a median age of 62 years, but threshold
Subclinical disease
may occur at any age. In addition, early findings
include splenomegaly and depletion of iron stores. Incubation Acute Convalescent
Hypertension is common, and there may be com- Prodromal
plaints of headache, dizziness, inability to concen-
Infection Resolution
trate, and some difficulty with hearing and vision
because of decreased cerebral blood flow. Venous

Copyright 2011. Wolters Kluwer Health | Lippincott Williams & Wilkins. Study Guide for Porths Essentials of Pathophysiology, Third Edition.
LWBK707-Ans_p280-402.qxd 8/20/10 6:38PM Page 314 Aptara Inc

314 ANSWER KEY

Activity C 5. Potential agents of bioterrorism have been catego-


rized into three levels (A, B, and C) based on risk
1.
of use, transmissibility, invasiveness, and mortality
1. b 2. f 3. d 4. g 5. a
rate. The agents considered to be in the highest
6. e 7. c 8. h 9. i 10. j
biothreat levelplague, tularemia, smallpox, and
2.
hemorrhagic feverare category A. The category B
1. f 2. a 3. j 4. e 5. c
agents include agents of food-borne and water-
6. d 7. b 8. h 9. g 10. i
borne disease, agents of zoonotic infections, and
Activity D viral encephalitides. Category C agents are defined
1. as emerging pathogens and potential risks for the
future even though many of these organisms are
d S e S a S c S f S b causes of ancient diseases, such as tuberculosis and
tick-borne fever viruses.

Activity E SECTION III: APPLYING YOUR KNOWLEDGE


1. Viruses are incapable of replication outside a living Activity F
cell. They must penetrate a susceptible living cell 1. An antibiotic is considered bactericidal if it causes
and use the biosynthetic machinery of the cell to irreversible and lethal damage to the bacterial
produce viral progeny. Not every viral agent causes pathogen, and bacteriostatic if its inhibitory effects
lysis and death of the host cell during the course on bacterial growth are reversed when the agent is
of replication. Some viruses enter the host cell and eliminated.
insert their genome into the host cell chromosome, 2. The drugs used to treat HIV infections are not
where it remains in a latent, nonreplicating state antibiotics or antiviral agents. They are classified as
for long periods without causing disease. Under the antiretroviral agents. These drugs are acyclovir, gan-
appropriate stimulation, the virus undergoes active ciclovir, vidarabine, ribavirin, zidovudine, lamivu-
replication and produces symptoms of disease dine, didanosine, stavudine, zalcitabine,
months to years later. nevirapine, efavirenz, and delavirdine.
2. The portal of entry refers to the process by which a
pathogen enters the body, gains access to suscepti- SECTION IV: PRACTICING FOR NCLEX
ble tissues, and causes disease. Among the poten-
tial modes of transmission are penetration, direct Activity G
contact, ingestion, and inhalation. In terms of patho- 1. Answer: a
physiology, symptoms are the outward expression of RATIONALE: A parasitic relationship is one in which
the struggle between invading organisms and the only the infecting organism benefits from the rela-
retaliatory inflammation and immune responses of tionship and the host either gains nothing from
the host. the relationship or sustains injury from the inter-
3. The course of any infectious disease can be divided action. If the host sustains injury or pathologic
into several distinguishable stages after the point damage in response to a parasitic infection, the
of time in which the potential pathogen enters the process is called an infectious disease. Mutual and
host. These stages are the incubation period, the commensal relationships do not harm the human
prodromal stage, the acute stage, the convalescent body. Communicable diseases can be passed from
stage, and the resolution stage. The stages are based one human to another; they are not parasitic.
on the progression and intensity of the hosts 2. Answer: b
symptoms over time. The duration of each phase RATIONALE: The rickettsiae are accidentally
and the pattern of the overall illness can be specific transmitted to humans through the bite of the
for different pathogens, thereby aiding in the diag- arthropod (i.e., vector) and produce a number of
nosis of an infectious disease. potentially lethal diseases, including Rocky Moun-
4. The goal of treatment for an infectious disease is tain spotted fever and epidemic typhus. Viruses,
complete removal of the pathogen from the host Chlamydiae, and Ehrlichiae do not cause either
and the restoration of normal physiologic function epidemic typhus or Rocky Mountain spotted fever.
to damaged tissues. When an infectious process 3. Answer: c
gains the upper hand and therapeutic intervention RATIONALE: Severe Acute Respiratory Syndrome
is essential, the choice of treatment may be medici- (SARS) was recognized in the Guangdong province
nal, using antimicrobial agents; immunologic, with in southern China beginning in November 2002.
antibody preparations, vaccines, or substances that The illness was highly transmissible as evidenced
stimulate and improve the hosts immune function; by the first recognized occurrence in Taiwan. Four
or surgical, by removing infected tissues. days after returning to Taiwan from work in the
Guangdong province, a businessman developed a

Copyright 2011. Wolters Kluwer Health | Lippincott Williams & Wilkins. Study Guide for Porths Essentials of Pathophysiology, Third Edition.
LWBK707-Ans_p280-402.qxd 8/20/10 6:38PM Page 315 Aptara Inc

ANSWER KEY 315

febrile illness and was admitted to a local hospital. after the point of time in which the potential
Within 1 month, a large nosocomial outbreak of pathogen enters the host. These stages are the
SARS was documented to have affected 3000 incubation period, the prodromal stage, the acute
people in Taipei City, Taiwan. Since the SARS out- stage, the convalescent stage, and the resolution
break began in China and crossed continental bor- stage. There are no postacute, subacute or postdro-
ders for the first time, it was classified as not only mal stages to a disease.
an epidemic but also a pandemic. Regional and 9. Answer: a
endemic mean the same thing, a specific area RATIONALE: An abscess is a localized pocket of
where the disease occurs. Nosocomial is an infec- infection composed of devitalized tissue, microor-
tion acquired in a health care facility. ganisms, and the hosts phagocytic white blood
4. Answer: d cells: in essence, a stalemate in the infectious
RATIONALE: The term symptomatology refers to the process. A pimple is a small papule or pustule. A
collection of signs and symptoms expressed by the lesion is a pathologic change in body tissue. Acne
host during the disease course. This is also known is a disease of the skin.
as the clinical picture or disease presentation. The 10. Answer: c
virulence of the disease is its power to produce the RATIONALE: Other exotoxins that have gained
disease. The source of the disease is the place where notoriety include the Shiga toxins produced by
it came from. The diagnosis of the disease is the Escherichia coli O157:H7 and other select strains.
naming of the disease process in the body. The ingestion of undercooked hamburger meat
5. Answer: a or unpasteurized fruit juices contaminated with
RATIONALE: The diagnosis of an infectious disease this organism produces hemorrhagic colitis and a
requires two criteria: the recovery of a probable sometimes fatal illness called hemolytic uremic
pathogen or evidence of its presence from the syndrome, characterized by vascular endothelial
infected sites of a diseased host, and accurate doc- damage, acute renal failure, and thrombocytopenia.
umentation of clinical signs and symptoms com- E. coli does not cause nephritic syndrome or
patible with an infectious process. Culture and hemolytic thrombocytopenia or neuroleptic
sensitivity are the growing of microorganisms out- malignant syndrome.
side the body and the testing to see what kills it. 11. Answer: prions
Identifying a microorganism by microscopic RATIONALE: Prions, protein particles that lack any
appearance and Gram stain reaction are not the kind of a demonstrable genome, have been found
criteria for diagnosis. Serology, an indirect means to cause pathologic processes in humans. The vari-
of identifying infectious agents by measuring ous prion-associated diseases produce very similar
serum antibodies in the diseased host, and the symptoms and pathology in the host and are col-
quantification of those antibodies, an antibody lectively called transmissible neurodegenerative
titer, are not criteria for diagnosis. diseases.
6. Answer: b 12. Answer: Congenital
RATIONALE: Potential agents of bioterrorism have RATIONALE: When an infectious disease is transmit-
been categorized into three levels (A, B, and C) ted from mother to child during gestation or birth,
based on risk of use, transmissibility, invasiveness, it is classified as a congenital infection.
and mortality rate. 13. Answers: 1-c, 2-a, 3-d, 4-b
7. Answer: c 14. Answers: a, c, d, e
RATIONALE: Aided by a global market and the ease RATIONALE: Virulence factors are substances or
of international travel, the past 5 years has products generated by infectious agents that
witnessed the importation or emergence of a host enhance their ability to cause disease. Although
of novel infectious diseases. During the late sum- the number and type of microbial products that fit
mer and early fall of 1999, West Nile virus (WNV) this description are numerous, they can generally
was identified as the cause of an epidemic involv- be grouped into four categories: toxins, adhesion
ing 56 patients in the New York City area. This factors, evasive factors, and invasive factors.
outbreak, which led to seven deaths (primarily in Prodromal means occurring first or prior to a spe-
the elderly), marked the first time that WNV had cific event. It is not a virulence factor.
been recognized in the Western hemisphere since 15. Answers: a, d, e
its discovery in Uganda nearly 60 years earlier. RATIONALE: A number of factors produced by
Coxsackie diseases, caused by the coxsackie virus; microorganisms enhance virulence by evading var-
respiratory syncytial disease, better known as RSV; ious components of the hosts immune system.
and hand, foot, and mouth disease are not consid- Extracellular polysaccharides including capsules,
ered global diseases. slime, and mucous layers discourage engulfment
8. Answer: d and killing of pathogens by the hosts phagocytic
RATIONALE: The course of any infectious disease white blood cells. Phospholipases and collagenases
can be divided into several distinguishable stages are enzymes that are invasive virulence factors.

Copyright 2011. Wolters Kluwer Health | Lippincott Williams & Wilkins. Study Guide for Porths Essentials of Pathophysiology, Third Edition.
LWBK707-Ans_p280-402.qxd 8/20/10 6:38PM Page 316 Aptara Inc

316 ANSWER KEY

CHAPTER 15 pocket that is complementary to the antigen, allowing


recognition and binding.
SECTION II: ASSESSING YOUR Activity C
UNDERSTANDING 1.
1. c 2. j 3. f 4. h 5. e
Activity A
6. d 7. a 8. b 9. i 10. g
1. immune system 2.
2. allergies, autoimmune 1. g 2. d 3. f 4. e 5. a
3. innate 6. b 7. c 8. h 9. j 10. i
4. Adaptive
Activity D
5. antigens
6. Humoral 1. Although cells of both the innate and adaptive
7. Cell-mediated immune systems communicate critical information
8. macrophages by cell-to-cell contact, many interactions and effec-
9. neutrophils, macrophages tor responses depend on the secretion of short-act-
10. Neutrophils ing soluble molecules called cytokines. One type of
11. macrophage cytokine, chemokines, direct leukocyte movement
12. B, T and migration, and another group of cytokines, the
13. Natural killer cells colony-stimulating factors, promote the proliferation
14. Dendritic and differentiation of bone marrow progenitor cells.
15. Chemokines Chemokines give the cells of the immune system the
16. colony-stimulating factors ability to act systemically as one.
17. epithelial 2. The innate immune system consists of the epithe-
18. pathogens lial barriers, phagocytic cells (mainly neutrophils
19. Opsonization and macrophages), natural killer (NK) cells, and
20. Antigens several plasma proteins including those of the com-
21. immunoglobulins plement system. These mechanisms are present in
22. Humoral the body before an encounter with an infectious
23. CD4 helper T cell (TH) agent and are rapidly activated by microbes before
24. Regulatory the development of adaptive immunity. The activa-
25. bone marrow, thymus tion and regulation of inflammation is also a major
26. spleen job of innate immunity.
3. These phagocytic cells recruited during an inflamma-
Activity B
tory response to recognize and kill infectious
The figure is a schematic model of an immunoglobulin invaders. The early-responding innate immune cell
G molecule showing the constant and variable regions is the neutrophil, followed shortly by the more
of the light and dark chains. Each immunoglobulin efficient, multifunctional macrophage. They are acti-
is composed of two identical light (L) chains and vated to engulf and digest microbes that attach to
two identical heavy (H) chains to form a Y-shaped their cell membrane. Once the cell is activated and
molecule. The two forked ends of the immunoglobulin the microbe is ingested, the cell generates digestive
molecule bind antigen and are called Fab (i.e., antigen- enzymes, toxic oxygen, and nitrogen intermediates
binding) fragments, and the tail of the molecule, (i.e., hydrogen peroxide or nitric oxide) through
which is called the Fc fragment, determines the bio- metabolic pathways. The phagocytic killing of
logic properties that are characteristic of a particular microorganisms helps to contain infectious agents.
class of immunoglobulins. The amino acid sequence of 4. There are three pathways for recognizing microor-
the heavy and light chains shows constant (C) regions ganisms that result in activation of the
and variable (V) regions. The constant regions have complement system: the classical, the lectin, and
sequences of amino acids that vary little among the the alternative pathway. The reactions of the com-
antibodies of a particular class of immunoglobulin. The plement systems are divided into three phases:
constant regions allow separation of immunoglobulins (1) initiation or activation, (2) amplification of
into classes (e.g., IgM, IgG) and allow each class of inflammation, and (3) membrane attack response.
antibody to interact with certain effectors cells and mol- 5. The major histocompatibility complex (MHC) mol-
ecules. The variable regions contain the antigen-binding ecules involved in self-recognition and cell-to-cell
sites of the molecule. The wide variation in the amino communication fall into two classes, class I and
acid sequence of the variable regions seen from anti- class II. Class I MHC molecules are cell surface gly-
body to antibody allows this region to recognize its coproteins that interact with the antigen receptor-
complementary epitope. A unique amino acid sequence foreign peptide complex and the CD8 molecule
in this region determines a distinctive three-dimensional on T cytotoxic lymphocytes. MHC-I molecules are

Copyright 2011. Wolters Kluwer Health | Lippincott Williams & Wilkins. Study Guide for Porths Essentials of Pathophysiology, Third Edition.
LWBK707-Ans_p280-402.qxd 8/20/10 6:38PM Page 317 Aptara Inc

ANSWER KEY 317

found on nearly all nucleated cells in the body and


thereby are capable of alerting the immune system
SECTION IV: PRACTICING FOR NCLEX
of any cell changes due to viruses, intracellular bac- Activity F
teria, or cancer. 1. Answer: a
6. Macrophages are key members of the mononuclear RATIONALE: The major components of innate
phagocytic system that engulf and digest microbes immunity are the skin and mucous membranes;
and other foreign substances. The monocytes phagocytic cells (mainly neutrophils and
migrate from the blood to various tissues where macrophages); specialized lymphocytes called
they mature into the major tissue phagocyte, the (NK) cells; and several plasma proteins, including
macrophages. As the general scavenger cells of the the proteins of the complement system. Adaptive,
body, the macrophage can be fixed in a tissue or humoral, and cell-mediated immunity do not use
can be free to migrate from an organ to lymphoid NK cells.
tissues. The tissue macrophages are scattered in 2. Answer: b
connective tissue or clustered in organs such as the RATIONALE: The actions of cytokines are often
lung (i.e., alveolar macrophages), liver (i.e., Kupffer pleiotropic and redundant. Cytokines are not
cells), spleen, lymph nodes, peritoneum, central described as rapid and self-limiting, or cell-specific
nervous system (i.e., microglial cells), and other and targeted, or dendritic and morphologic.
areas. Macrophages are activated to engulf and 3. Answer: c
break down complex antigens into peptide fragments RATIONALE: The T lymphocytes (T cells) are gener-
for association with class II MHC molecules. ated from stem cells in the bone marrow and com-
Macrophages can then present these complexes to plete their maturation in the thymus and
the helper T cell so that selfnonself recognition and functions in the peripheral tissues to produce cell-
activation of the immune response can occur. mediated immunity, as well as aiding antibody
7. The immunoglobulins have been divided into five production.
classes: IgG, IgA, IgM, IgD, and IgE. 4. Answer: d
I. IgG protects against bacteria, toxins, and RATIONALE: Activation of macrophages ensures
viruses in body fluids and activates the comple- enhanced phagocytic, metabolic, and enzymatic
ment system. potential, resulting in more efficient destruction
II. IgA is a primary defense against local infections of infected cells. This type of defense is important
in mucosal tissues. against intracellular pathogens such as Mycobac-
III. IgM is the first circulating immunoglobulin to terium species and Listeria monocytogenes. Contact
appear in response to an antigen. dermatitis due to a poison ivy reaction or sensitiv-
IV. IgD serves as an antigen receptor for initiating ity to dyes is an example of delayed or cell-medi-
the differentiation of B cells. ated hypersensitivity caused by haptencarrier
V. IgE is involved in inflammation, allergic complexes. Blood transfusions do not cause
responses, and combating parasitic infections. hypersensitivity reactions by haptencarrier
8. Active immunity is acquired through complexes.
immunization or actually having a disease. It is 5. Answer: a
active as it depends on a response to the antigen by RATIONALE: Passive immunity also can be
the persons immune system. Because of memory, artificially provided by the transfer of antibodies
the immune system usually is able to react within produced by other people or animals. Some
hours to subsequent exposure to the same agent protection against infectious disease can be
because of the presence of memory B and T provided by the injection of hyperimmune serum,
lymphocytes and circulating antibodies. Passive which contains high concentrations of antibod-
immunity is immunity transferred from another ies for a specific disease, or immune serum or
source. An infant receives passive immunity -globulin, which contains a pool of antibodies
naturally from the transfer of antibodies from its from many individuals providing protection
mother in utero and through a mothers breast against many infectious agents. Immunizations
milk. and allergy shots are examples of active immunity.
Exposure to poison ivy can be the cause of a
SECTION III: APPLYING YOUR KNOWLEDGE hypersensitivity reaction; it is not immunity.
Activity E 6. Answer: b
RATIONALE: Self-regulation is an essential property
1. Every baby is born with passive immunity. Baby
of the immune system. An inadequate immune
receives antibodies from mother through placenta
response may lead to immunodeficiency, but an
and colostrums. Passive immunity lasts up to
inappropriate or excessive response may lead to
6 months. Passive immunity is replaced by immu-
conditions varying from allergic reactions to
nity gotten from immunizations.
autoimmune diseases. All answers are autoimmune
diseases except for Huntington disease.

Copyright 2011. Wolters Kluwer Health | Lippincott Williams & Wilkins. Study Guide for Porths Essentials of Pathophysiology, Third Edition.
LWBK707-Ans_p280-402.qxd 8/20/10 6:38PM Page 318 Aptara Inc

318 ANSWER KEY

7. Answer: c 15. Answers: b, e


RATIONALE: The term tolerance is used to define the RATIONALE: The T and B lymphocytes are the
ability of the immune system to be nonreactive to only cells in the body capable of specifically recog-
self-antigens while producing immunity to foreign nizing different antigenic determinants of micro-
agents. All other responses have nothing to do bial agents and other pathogens, and therefore
with the recognition and tolerance to self- responsible for two defining characteristics of
antigens. adaptive immunity, specificity and memory.
8. Answer: d Phagocytes, dendritic cells, and NK cells all partici-
RATIONALE: Cord blood does not normally contain pate in innate immunity.
IgM or IgA. If present, these antibodies are of fetal
origin and represent exposure to intrauterine
infection. CHAPTER 16
9. Answer: a
RATIONALE: Aging is characterized by a declining SECTION II: ASSESSING YOUR
ability to adapt to environmental stresses. One of UNDERSTANDING
the factors thought to contribute to this problem
Activity A
is a decline in immune responsiveness. This
includes changes in cell-mediated and humoral 1. immune
immune responses. Elderly persons tend to be 2. Immunodeficiency
more susceptible to infections, have more 3. innate
evidence of autoimmune and immune complex 4. adaptive
disorders than younger persons, and have a higher 5. humoral, cellular
incidence of cancer. None of the other answers are 6. X
true or acceptable. 7. pyogenic
10. Answer: b 8. maternal
RATIONALE: Among the functions of the innate 9. antibody
immune system is induction of a complex cascade 10. kidney
of events known as the inflammatory response. 11. malignancies
Recent evidence suggests that inflammation plays 12. CD4 helper, CD8 cytotoxic
a key role in the pathogenesis of a number of dis- 13. T lymphocytes
orders such as atherosclerosis and coronary artery 14. combined immunodeficiency
disease, bronchial asthma, type 2 diabetes 15. severe combined immunodeficiency (SCID)
mellitus, rheumatoid arthritis, multiple sclerosis, 16. boys
and systemic lupus erythematosus. Osteoporosis is 17. complement
the abnormal loss of bone tissue and density. 18. secondary
Osteogenesis imperfecta is a genetic disease caus- 19. degranulation
ing multiple bone fractures in a newborn. 20. respiratory burst
Hydronephrosis is a condition of the kidney caus- 21. Hypersensitivity
ing distention of the pelvis and calyces because of 22. allergic reactions
an obstruction in the ureter causing an inability of 23. Anaphylaxis
urine to pass. 24. atopic
11. Answer: Antigens 25. rhinitis
12. Answers: 1-d, 2-e, 3-a, 4-c, 5-b 26. type II
13. Answer: epithelial 27. Antibody-dependent cellular cytotoxicity
14. Answers: bc 28. Type III
RATIONALE: While cells of both the innate and 29. Serum
adaptive immune systems communicate critical 30. Arthus reaction
information about the invading microbe or 31. type IV reactions
pathogen by cell-to-cell contact, many interactions 32. contact dermatitis
and effector responses depend on the secretion of 33. transplantation
chemical mediators (cytokines, chemokines, and 34. autologous, syngeneic, allogeneic
colony-stimulating factors). Virulence factors 35. Graft-versus-host-disease
define how much power an organism has to 36. Autoimmune
produce disease. Coxiellas are organisms that 37. self-tolerance
cause Q fever. 38. autoantibodies

Copyright 2011. Wolters Kluwer Health | Lippincott Williams & Wilkins. Study Guide for Porths Essentials of Pathophysiology, Third Edition.
LWBK707-Ans_p280-402.qxd 8/20/10 6:38PM Page 319 Aptara Inc

ANSWER KEY 319

Activity B 3. Type I hypersensitivity reactions begin with mast


cell or basophil sensitization. During the sensitiza-
tion or priming stage, allergen-specific IgE antibod-
Bone marrow
ies attach to receptors on the surface of mast cells
and basophils. With subsequent exposure, the sen-
sitizing allergen binds to the cell-associated IgE and
triggers a series of events that ultimately lead to
degranulation of the sensitized mast cells or
Pre-T cells basophils, causing release of their preformed media-
tors. Mast cells are also the source of lipid-derived
membrane products (e.g., prostaglandins and
Thymus
leukotrienes) and cytokines that participate in the
continued response to the allergen.
4. In direct cell-mediated cytotoxicity, CD8 cytolytic T
Self-antigen lymphocytes (CTLs) directly kill the antigen-present-
not expressed ing target cells. In viral infections, CTL responses
in thymus
can lead to tissue injury by killing infected target
cells even if the virus itself has no cytotoxic effects.
Self-reactive clones Nonreactive clones Self-reactive clones
Because CTLs cannot distinguish between cytopathic
and noncytopathic viruses, they kill virtually all
infected cells, regardless of whether the infection is
Apoptosis
Failure of Activation- harmful or not. Delayed-type hypersensitivity reac-
antigens to induced tions occur in response to soluble protein antigens
activate apoptosis
Development of
lymphocyte and primarily involve antigen-presenting cells such
central tolerance
A Induction of normal as macrophages and CD4 helper T cells of the Th1
immune function with
self versus nonself Clonal anergy type. During the reaction, Th1 cells are activated and
B recognition secrete an array of cytokines that recruit and activate
monocytes, lymphocytes, fibroblasts, and other
C Induction of peripheral tolerance inflammatory cells. These T-cellmediated responses
require the synthesis of effector molecules and take
24 to 72 hours to develop, which is why they are
Activity C called delayed-type hypersensitivity disorders.
1. e 2. h 3. l 4. a 5. f 5. Severe combined immunodeficiency (SCID) is the
6. m 7. d 8. k 9. i 10. c result of genetic mutations that lead to absence of
11. n 12. j 13. g 14. b 15. o all T-and B-cell function and, in some cases, a lack
Activity D of natural killer cells. Affected infants have a disease
course that resembles AIDS, with failure to thrive,
1. chronic diarrhea, and opportunistic infections. Sur-
vival beyond the first year of life is rare without
3 S 5 S 1 S 4 S 2 prompt immune reconstitution through bone mar-
row or hematopoietic stem cell transplantation.
Early diagnosis is critical because the chances of suc-
Activity E cessful treatment are highest in infants who have
not experienced severe opportunistic infections.
1. A primary deficiency or immunodeficiency is con-
There is also hope that gene therapy will someday
genital or inherited. Secondary immunodeficiency
be available for some, if not all, forms of SCID.
is acquired.
2. During the first few months of life, infants are
protected from infection by IgG antibodies that SECTION III: APPLYING YOUR KNOWLEDGE
have been transferred from the maternal circulation Activity F
during fetal life. An infants level of maternal
1. Antinuclear antibodies test. The basis for most
IgG gradually declines over a period of appro-
serologic assays is the demonstration of antibod-
ximately 6 months. Concomitant with the loss of
ies directed against tissue antigens or cellular compo-
maternal antibody, the infants immature humoral
nents. For example, a child with chronic or acute
immune system begins to function, and between
history of fever, arthritis, and a macular rash along
the ages of 1 and 2 years, the childs antibody pro-
with high levels of antinuclear antibody has a proba-
duction reaches adult levels. Once the level of
ble diagnosis of SLE. The detection of autoantibodies
maternal antibodies drops, the infant is susceptible
in the laboratory usually is accomplished by one of
to infection.
three methods: indirect fluorescent antibody assays

Copyright 2011. Wolters Kluwer Health | Lippincott Williams & Wilkins. Study Guide for Porths Essentials of Pathophysiology, Third Edition.
LWBK707-Ans_p280-402.qxd 8/20/10 6:38PM Page 320 Aptara Inc

320 ANSWER KEY

(IFA), enzyme-linked immunosorbent assay (ELISA), 6. Answer: a


or particle agglutination of some kind. RATIONALE: Disorders caused by immune responses
2. Medications used in the treatment of SLE include are collectively referred to as hypersensitivity reactions.
corticosteroids (prednisone) and immunosuppres- Antigens cause allergic reactions. Mediator response
sive (cytotoxic) agents (azathioprine, cyclophos- action and allergen stimulating reaction have noth-
phamide, methotrexate). ing to do with hypersensitivity reactions.
7. Answer: b
SECTION IV: PRACTICING FOR NCLEX RATIONALE: Anaphylaxis is a systemic life-threaten-
ing hypersensitivity reaction characterized by
Activity G
widespread edema, vascular shock secondary to
1. Answer: c vasodilation, and difficulty breathing. It is not
RATIONALE: During the first few months of life, called an antigen reaction, neither is it called an
infants are protected from infection by IgG Arthus reaction.
antibodies that have been transferred from the 8. Answer: c
maternal circulation during fetal life. IgA, IgM, RATIONALE: Serum sickness is a systemic immune
IgD, and IgE do not normally cross the placenta. complex disorder that is triggered by the deposition
2. Answers: a, b, c, d of insoluble antigen-antibody (IgM, IgG, and occa-
RATIONALE: Medications that cause reversible second- sionally IgA) complexes in blood vessels, joints,
ary hypogammaglobulinemia include the disease- heart, and kidney tissue. This is not anti-immune
modifying antirheumatic drugs, corticosteroid disease, SLE or antigen-antibody sickness.
agents, and the antiepileptic drugs, phenytoin and 9. Answer: d
carbamazepine. Interferon beta-1a drugs are used in RATIONALE: Cornstarch powder is applied to the
the treatment of autoimmune disorders. gloves during the manufacturing process to prevent
3. Answer: a stickiness and give the gloves a smooth feel. The
RATIONALE: In general, persons with cell-mediated cornstarch glove powder has an important role in
immunodeficiency disorders have infections or the allergic response. Latex proteins are readily
other clinical problems that are more severe than absorbed by glove powder and become airborne
antibody disorders. Children with defects in this during removal of the gloves. Baking powder is not
branch of the immune response rarely survive used inside the gloves. Pieces of latex that become
beyond infancy or childhood, unless immunologic airborne and latex proteins that attach to clothing
reconstitution is achieved through bone marrow are not significant contributors to the incidence of
transplantation. In DeGeorge syndrome children latex allergy.
who survive the immediate neonatal period may 10. Answer: antidonor
have recurrent or chronic infections because of RATIONALE: When preformed antidonor antibodies
impaired T-cell immunity. Children also may have are present, rejection occurs immediately after
an absence of immunoglobulin production, caused transplantation.
by a lack of helper T-cell function. The X-linked 11. Answers: a, b, d
immunodeficiency of hyper-IgM, also known as RATIONALE: Because autoimmunity does not
the hyper-IgM syndrome, is characterized by low develop in all persons with genetic predisposition,
IgG and IgA levels with normal or, more frequently, it appears that other factors such as a trigger
high IgM concentrations. X-linked agammaglobu- event interact to precipitate the altered immune
linemia is a primary humeral immunodeficiency state. The event or events that trigger the develop-
disorder. Y-linked agammaglobulinemia does not ment of an autoimmune response are unknown. It
exist. has been suggested that the trigger may be a virus
4. Answer: b or other microorganism, a chemical substance, or a
RATIONALE: Disorders that affect both B and T lym- self-antigen from a body tissue that has been hid-
phocytes with resultant defects in both humoral den from the immune system during development.
and cell-mediated immunity fall under the broad
classification of combined immunodeficiency syn-
drome. A single mutation in any one of the many CHAPTER 17
genes that influence lymphocyte development or
response, including lymphocyte receptors, SECTION II: ASSESSING YOUR
cytokines, or major histocompatibility antigens, UNDERSTANDING
could lead to combined immunodeficiency.
5. Answer: c Activity A
RATIONALE: Ataxia-telangiectasia is a complex syn- 1. oxygen, waste, hormones
drome of neurologic, immunologic, endocrinologic, 2. pulmonary, systemic
hepatic, and cutaneous abnormalities. Pierre-Robin 3. pulmonary
syndrome, Angelman syndrome, and Adair-Dighton 4. systemic
syndrome are not immunologic deficiencies. 5. low

Copyright 2011. Wolters Kluwer Health | Lippincott Williams & Wilkins. Study Guide for Porths Essentials of Pathophysiology, Third Edition.
LWBK707-Ans_p280-402.qxd 8/20/10 6:38PM Page 321 Aptara Inc

ANSWER KEY 321

6. atria, ventricles 2.
7. same
8. volume, pressure Superior vena cava
9. hemodynamics
10. large
11. Viscosity Right pulmonary Left pulmonary
artery artery
12. Turbulent
Pulmonic valve
13. thicker Pulmonary veins
14. distensibility Pulmonary Left atrium
15. aortic, pulmonic veins Aortic valve
Right atrium Mitral valve
16. precedes
Tricuspid valve Chordae
17. elastic tendineae
18. Diastole Left
19. stroke volume Right ventricle ventricle
20. ejection Inferior vena cava Papillary
21. cardiac output muscles
22. cardiac reserve Papillary muscles
23. Frank-Starling
24. heart rate Descending
aorta
25. tunica adventitia, tunica media, tunica intima
26. arterial pressure pulse
27. decreases
28. central venous pressure
29. Valves Activity C
30. Autoregulation 1. d 2. b 3. j 4. c 5. i
31. hyperemia 6. h 7. e 8. f 9. a 10. g
32. anastomotic Activity D
33. microcirculation
1. The most important factors governing the flow of
34. capillary pores
blood in the cardiovascular system are pressure,
35. colloidal osmotic
resistance, and flow. Blood flow (F) through a vessel
36. medulla oblongata
or series of blood vessels is determined by the pres-
37. sympathetic, parasympathetic
sure difference ( P) between the two ends of a ves-
38. Cushing reflex
sel (the inlet and the outlet) and the resistance (R)
Activity B that blood must overcome as it moves through the
1. vessel (F  P/R).
2. This is because, even though each individual capil-
External
lary is very small, the total cross-sectional area of
jugular vein Internal
jugular vein all the systemic capillaries greatly exceeds the cross-
Subclavian vein
sectional area of other parts of the circulation.
Superior Because of this large surface area, the slower move-
vena cava
ment of blood allows ample time for exchange of
Aortic arch
nutrients, gases, and metabolites between the
tissues and the blood.
3. The anatomic arrangement of the actin and myosin
Right Left atrium
atrium
filaments in the myocardial muscle fibers is such
that the tension or force of contraction depends on
Left coronary the degree to which the muscle fibers are stretched
artery just before the ventricles begin to contract. The
maximum force of contraction and cardiac output is
Left achieved when venous return produces an increase
ventricle in left ventricular end-diastolic filling (i.e., preload)
Right Pericardium
ventricle such that the muscle fibers are stretched about two
Pleura Right coronary Posterior
artery and one-half times their normal resting length.
When the muscle fibers are stretched to this degree,
there is optimal overlap of the actin and myosin fil-
Left
Right ventricle ventricle aments needed for maximal contraction.
Interventricular 4. Sympathetic innervation via -adrenergic receptors
septum is excitatory in that they produce vasoconstriction;
Anterior
-adrenergic receptors are inhibitory in that they

Copyright 2011. Wolters Kluwer Health | Lippincott Williams & Wilkins. Study Guide for Porths Essentials of Pathophysiology, Third Edition.
LWBK707-Ans_p280-408.qxd 9/2/10 8:02 AM Page 322 Aptara Inc

322 ANSWER KEY

produce vasodilation. Smooth muscle contraction mechanism. The brain and other cerebral
and relaxation also occur in response to local tissue structures are located within the rigid confines of
factors such as lack of oxygen, increased hydrogen the skull, with no room for expansion, and any
ion concentrations, and excess carbon dioxide. increase in intracranial pressure tends to compress
Nitric oxide acts locally to produce smooth muscle the blood vessels that supply the brain.
relaxation and regulate blood flow. 5. Answer: c
5. Norepinephrinepotent vasoconstrictor RATIONALE: In clinical practice, the measurement of
Epinephrinemild vasoconstriction or dilation the cardiac forms of troponin T and troponin I are
depending on the receptor type found in target used in the diagnosis of myocardial infarction. Tro-
tissue ponin C is not diagnostic of a myocardial infarction.
Angiotensin IIpowerful vasoconstrictor Troponin A is not one of the troponin complexes.
Histaminepowerful vasodilator and can 6. Answer: d
increase permeability RATIONALE: Approximately 60% of the stroke
Serotoninvasoconstrictor volume is ejected during the first quarter of
Bradykininvasodilator systole, and the remaining 40% is ejected during
Prostaglandinsvasodilator or vasoconstrictor the next two quarters of systole. Little blood is
depending on type of prostaglandin ejected from the heart during the last quarter of
systole, although the ventricle remains contracted.
SECTION III: PRACTICING FOR NCLEX 7. Answer: a
RATIONALE: With peripheral arterial disease, there is
Activity E
a delay in the transmission of the reflected wave
1. Answer: a so that the pulse decreases rather than increases in
RATIONALE: The total blood volume is a function of amplitude.
age and body weight, ranging from 85 to 90 mL/kg 8. Answer: b
in the neonate and from 70 to 75 mL/kg in the adult. RATIONALE: The efficiency of the heart as a pump
2. Answer: b often is measured in terms of cardiac output (CO)
RATIONALE: The blood vessels and the blood vessel or the amount of blood the heart pumps each
itself constitute resistance to flow. A helpful equa- minute. The CO is the product of the stroke volume
tion for understanding the relationship between (SV) and the heart rate (HR), and can be expressed
resistance, blood vessel diameter (radius), and by the equation: CO  SV  HR. AV stands for atri-
blood viscosity factors that affect blood flow was oventricular and EF stands for ejection fraction.
derived by the French physician Poiseuille more Neither is part of the equation for CO.
than a century ago. The other laws do not address 9. Answers: b, c, d, e
resistance to flow. RATIONALE: The hearts ability to increase its output
3. Answer: b according to body needs mainly depends on four
RATIONALE: Compliance refers to the total quantity factors: the preload, or ventricular filling; the after-
of blood that can be stored in a given portion of load, or resistance to ejection of blood from the
the circulation for each millimeter rise in pressure. heart; cardiac contractility; and the heart rate. Car-
Compliance reflects the distensibility of the blood diac reserve does not add to the hearts ability to
vessel. Wall tension, laminar blood flow, and increase its output.
resistance are not major factors in the distensibil- 10. Answer: d
ity of the blood vessel. RATIONALE: The fact that nitric oxide is released
4. Answer: c into the vessel lumen (to inactivate platelets) and
RATIONALE: The Cushing reflex is a special type of away from the lumen (to relax smooth muscle)
CNS reflex resulting from an increase in intracra- suggests that it protects against both thrombosis
nial pressure. When the intracranial pressure rises and vasoconstriction. Nitroglycerin, which is used
to levels that equal intra-arterial pressure, blood in treatment of angina, produces its effects by
vessels to the vasomotor center become compressed, releasing nitric oxide in vascular smooth muscle of
initiating the CNS ischemic response. The purpose the target tissues. None of the other answers are
of this reflex is to produce a rise in arterial pressure released by nitroglycerin.
to levels above intracranial pressure so that the 11. Answer: a
blood flow to the vasomotor center can be re- RATIONALE: The osmotic pressure caused by the
established. Should the intracranial pressure rise to plasma proteins in the blood tends to pull fluid
the point that the blood supply to the vasomotor from the interstitial spaces back into the capillary.
center becomes inadequate, vasoconstrictor tone is This pressure is termed colloidal osmotic pressure to
lost, and the blood pressure begins to fall. The differentiate the osmotic effects of the plasma pro-
elevation in blood pressure associated with the teins, which are suspended colloids, from the
Cushing reflex is usually of short duration and osmotic effects of substances such as sodium and
should be considered a protective homeostatic glucose, which are dissolved crystalloids.

Copyright 2011. Wolters Kluwer Health | Lippincott Williams & Wilkins. Study Guide for Porths Essentials of Pathophysiology, Third Edition.
LWBK707-Ans_p280-402.qxd 8/20/10 6:38PM Page 323 Aptara Inc

ANSWER KEY 323

12. Answer: c 34. valves


RATIONALE: The lymph capillaries drain into larger 35. deep vein thrombosis
lymph vessels that ultimately empty into the right 36. Venous insufficiency
and left thoracic ducts. The thoracic ducts empty 37. stasis of blood, increased blood coagulability,
into the circulation at the junctions of the subcla- vessel wall injury
vian and internal jugular veins. The lymphatic sys- 38. Arterial
tem only joins the vascular system in one place, so 39. systolic, diastolic
no other answer is accurate. 40. pulse pressure
13. Answer: b 41. mean arterial pressure
RATIONALE: The medullary cardiovascular neurons 42. cardiac output, peripheral vascular resistance
are grouped into three distinct pools that lead to 43. vessel constriction, fluid retention
sympathetic innervation of the heart and blood 44. equilibrium
vessels and parasympathetic innervation of the 45. kidneys, sodium, water
heart. The first two, which control sympathetic- 46. Primary, secondary
mediated acceleration of heart rate and blood vessel 47. 140 mm Hg, 90 mm Hg
tone, are called the vasomotor center. The third, 48. constitutional
which controls parasympathetic-mediated slowing 49. left ventricle
of heart rate, is called the cardioinhibitory center. 50. nephrosclerosis
51. ischemic, hemorrhage
52. 140/90 mm Hg
CHAPTER 18 53. Diuretics
54.
-adrenergic
SECTION II: ASSESSING YOUR 55. calcium channel receptor-blocking
UNDERSTANDING 56. systole
57. secondary
Activity A
58. oral contraceptive
1. blood vessels 59. Preeclampsia-eclampsia
2. endothelium 60. orthostatic hypotension
3. vasoconstriction, dilation
Activity B
4. ischemia
5. Infarction 1.
6. cholesterol LUMEN
CAP Macrophage
7. cholesterol, triglyceride
Smooth muscle cells
8. lipoproteins
9. lipolytic Endothelial cell
10. small intestine, liver Lymphocytes
11. Chylomicrons
SHOULDER
12. bad cholesterol
13. LDL receptors, scavenger NECROTIC
14. atherosclerosis CORE
15. good cholesterol Lipid-laden
16. coronary heart disease macrophage
17. genetic ELASTIC MEDIA
18. secondary
19. lower, elevate
20. Atherosclerosis
21. hypercholesterolemia
22. Cigarette smoking
2. Mechanisms of blood pressure regulation. The
23. inflammation
solid lines represent the mechanisms for renal and
24. C-reactive protein (CRP)
baroreceptor control of blood pressure through
25. Homocysteine
changes in cardiac output and peripheral vascular
26. free radicals
resistance. The dashed lines represent the stimulus
27. vasculitides
for regulation of blood pressure by the
28. embolus
baroreceptors and the kidneys.
29. Thromboangiitis obliterans
30. Raynaud phenomenon Activity C
31. aneurysm 1.
32. asymptomatic, rupture 1. h 2. i 3. g 4. j 5. d
33. hemorrhage 6. a 7. f 8. c 9. e 10. b

Copyright 2011. Wolters Kluwer Health | Lippincott Williams & Wilkins. Study Guide for Porths Essentials of Pathophysiology, Third Edition.
LWBK707-Ans_p280-402.qxd 8/20/10 6:38PM Page 324 Aptara Inc

324 ANSWER KEY

2. superficial gangrene of the fingers, although infre-


1. i. 2. h 3. d 4. j 5. a quent, may occur.
6. b 7. e 8. g 9. f 10. c 6. During muscle contraction, which is similar to sys-
Activity D tole, valves in the communicating channels close
to prevent backward flow of blood into the superfi-
1. Decrease in blood pressure
cial system, as blood in the deep veins is moved
2. Stimulation of juxtaglomerular apparatus
forward by the action of the contracting muscles.
3. Release of renin
During muscle relaxation, which is similar to dias-
4. Conversion of angiotensinogen to angiotensin I
tole, the communicating valves open, allowing
5. Conversion of ANG I to ANG II by angiotensin-
blood from the superficial veins to move into the
converting enzyme
deep veins.
6. Increased vascular resistance, release of aldosterone
7. Short-term regulation is accomplished through the
7. Na retention, stimulation of ADH release
cardiovascular center of the ANS, baroreceptors,
8. Water retention
and chemoreceptors. The cardiovascular center
Activity E transmits parasympathetic impulses to the heart
1. Once thought to be nothing more than a lining for through the vagus nerve and sympathetic impulses
blood vessels, it is now known that the endothelium to the heart and blood vessels through the spinal
is a versatile, multifunctional tissue that plays an cord and peripheral sympathetic nerves. The
active role in controlling vascular function. As a baroreceptors are pressure-sensitive receptors
semipermeable membrane, the endothelium located in the walls of blood vessels and the heart.
controls the transfer of molecules across the vascu- They respond to changes in the stretch of the vessel
lar wall. The endothelium also plays a role in the wall by sending impulses to cardiovascular centers
control of platelet adhesion and blood clotting, in the brain stem to effect appropriate changes in
modulation of blood flow and vascular resistance, heart rate and vascular smooth muscle tone. The
metabolism of hormones, regulation of immune arterial chemoreceptors are chemosensitive cells
and inflammatory reactions, and elaboration of fac- that monitor the oxygen, carbon dioxide, and
tors that influence the growth of other cell types, hydrogen ion content of the blood.
particularly vascular smooth muscle cells. 8. Most acute kidney disorders result in decreased
2. High-calorie diets increase the production of VLDL urine formation, retention of salt and water, and
with triglyceride elevation and high conversion of hypertension. Renovascular hypertension refers to
VLDL to LDL. Excess ingestion of cholesterol may hypertension caused by reduced renal blood flow
reduce the formation of LDL receptors and thereby and activation of the renin-angiotensin-aldosterone
decrease LDL removal. Diets that are high in mechanism. The reduced renal blood flow that
triglycerides and saturated fats increase cholesterol occurs with renovascular disease causes the affected
synthesis and suppress LDL receptor activity. In dia- kidney to release excessive amounts of renin,
betes mellitus and the metabolic syndrome, typical increasing circulating levels of angiotensin II.
dyslipidemia is seen with elevation of triglycerides, Angiotensin II, in turn, acts as a vasoconstrictor to
low HDL and minimal or modest elevation of LDL. increase peripheral vascular resistance and as a
3. Lipid-lowering drugs work in several ways including stimulus for increased aldosterone levels and
decreasing cholesterol production, decreasing cho- sodium retention by the kidney.
lesterol absorption from the intestine, or removing
cholesterol from the bloodstream. Drugs that act SECTION III: PRACTICING FOR NCLEX
directly to decrease cholesterol levels also have the
Activity F
beneficial effect of further lowering cholesterol lev-
els by stimulating the production of additional LDL 1. Answers: 1-f, 2-e, 3-b, 4-a, 5-d, 6-c
receptors. RATIONALE: Disturbances in blood flow can result
4. (1) Pistol shot (acute onset), (2) pallor, (3) polar from pathologic changes in the vessel wall (i.e., ath-
(cold), (4) pulselessness, (5) pain, (6) paresthesia, erosclerosis and vasculitis), acute vessel obstruction
and (7) paralysis. due to thrombus or embolus, vasospasm (i.e., Ray-
5. Ischemia due to vasospasm causes changes in skin naud phenomenon), or abnormal vessel dilation
color that progress from pallor to cyanosis, a sensa- (i.e., arterial aneurysms or varicose veins).
tion of cold, and changes in sensory perception, 2. Answers: a, d
such as numbness and tingling. After the ischemic RATIONALE: There are two sites of lipoprotein syn-
episode, there is a period of hyperemia with intense thesis: the small intestine and the liver. The
redness, throbbing, and paresthesias. In severe, pro- chylomicrons, which are the largest of the lipopro-
gressive cases usually associated with Raynaud phe- tein molecules, are synthesized in the wall of the
nomenon, trophic changes may develop. The nails small intestine. The liver synthesizes and releases
may become brittle and the skin over the tips of VLDL and HDL. The large intestine and the
the affected fingers may thicken. Ulceration and pancreas play no part in synthesizing lipoprotein.

Copyright 2011. Wolters Kluwer Health | Lippincott Williams & Wilkins. Study Guide for Porths Essentials of Pathophysiology, Third Edition.
LWBK707-Ans_p280-402.qxd 8/20/10 6:38PM Page 325 Aptara Inc

ANSWER KEY 325

3. Answer: a disease, primary and secondary. The secondary


RATIONALE: Many types of primary type, called Raynaud phenomenon, is associated
hypercholesterolemia have a genetic basis. There with other disease states or known causes of
may be a defective synthesis of the apoproteins, a vasospasm. Raynaud phenomenon is associated
lack of receptors, defective receptors, or defects in with previous vessel injury, such as frostbite, occu-
the handling of cholesterol in the cell that are pational trauma associated with the use of heavy
genetically determined. For example, the LDL vibrating tools, collagen diseases, neurologic disor-
receptor is deficient or defective in the genetic dis- ders, and chronic arterial occlusive disorders. The
order known as familial hypercholesterolemia initial diagnosis is based on history of vasospastic
(type 2A). This autosomal dominant type of hyper- attacks supported by other evidence of the
lipoproteinemia results from a mutation in the disorder. Treatment measures are directed toward
gene specifying the receptor for LDL. Although eliminating factors that cause vasospasm and pro-
heterozygotes commonly have an elevated choles- tecting the digits from trauma during an ischemic
terol level from birth, they do not develop symp- episode. Abstinence from smoking and protection
toms until adult life, when they often develop from cold are priorities. The presenting symptoms
xanthomas (i.e., cholesterol deposits) along the of this patient do not support a diagnosis of or
tendons and atherosclerosis appears. Myocardial treatment for arterial thrombosis or peripheral
infarction before 40 years of age is common. artery disease.
Homozygotes are much more severely affected; 8. Answers: c, d
they have cutaneous xanthomas in childhood and RATIONALE: Abdominal aortic aneurysms can
may experience myocardial infarction by as early involve any part of the vessel circumference
as 1 to 2 years of age. Homozygotic cutaneous (saccular) or extend to involve the entire circum-
xanthoma and adult-onset hypercholesterolemia ference (fusiform). Berry aneurysms typically occur
(type 1A) are not known diseases. Causes of in the circle of Willis. Dissecting aneurysms are
secondary hyperlipoproteinemia include obesity false aneurysms and typically occur in the thoracic
with high-calorie intake and diabetes mellitus. It aorta. Aneurysms can occur at the bifurcation of a
does not have a genetic basis. blood vessel but are not termed bifurcating
4. Answer: b aneurysms.
RATIONALE: The cause or causes of atherosclerosis 9. Answer: b
have not been determined with certainty. RATIONALE: Sclerotherapy, which often is used in
However, epidemiologic studies have identified the treatment of small residual varicosities,
predisposing risk factors, which include a major involves the injection of a sclerosing agent into
risk factor of hypercholesterolemia. Other risk fac- the collapsed superficial veins to produce fibrosis
tors include increasing age, family history of pre- of the vessel lumen. Surgical treatment consists of
mature coronary heart disease, and male sex. removing the varicosities and the incompetent
5. Answer: c perforating veins, but it is limited to persons
RATIONALE: Temporal arteritis (i.e., giant cell arteri- with patent deep venous channels. Sclerotherapy
tis), the most common of the vasculitides, is a produces fibrosis of the vessel lumen. There is
focal inflammatory condition of medium-sized no fibrotherapy for varicose veins. There is no
and large arteries. It predominantly affects Trendelenburg therapy for varicose veins. There
branches of arteries originating from the aortic is a Trendelenburg test that is diagnostic for primary
arch, including the superficial temporal, vertebral, or secondary varicose veins.
ophthalmic, and posterior ciliary arteries. Neither 10. Answer: d
Polyarteritis Nodosa nor Raynaud disease are the RATIONALE: In 1846, Virchow described the triad
most common of the vasculitides. Varicose veins that has come to be associated with venous
are not vasculitides. thrombosis: stasis of blood, increased blood coag-
6. Answer: c ulability, and vessel wall injury. Inflammation is a
RATIONALE: Acute arterial occlusion is a sudden symptom of venous thrombosis, not a risk factor.
event that interrupts arterial flow to the affected Decreased venous blood flow can occur because
tissues or organ. Most acute arterial occlusions are of venous thrombosis, if the thrombus does not
the result of an embolus or a thrombus. Other completely obstruct the vein; it is not a risk factor.
answers are not appropriate for the nurse to give Hypocoagulability would not cause a thrombus to
the client. form.
7. Answers: a, d 11. Answer: c
RATIONALE: Raynaud disease or phenomenon is a RATIONALE: At normal heart rates, mean arterial
functional disorder caused by intense vasospasm pressure can be estimated by adding one third of
of the arteries and arterioles in the fingers and, less the pulse pressure to the diastolic pressure (i.e.,
often, the toes. There are two types of Raynaud diastolic blood pressure  pulse pressure/3).

Copyright 2011. Wolters Kluwer Health | Lippincott Williams & Wilkins. Study Guide for Porths Essentials of Pathophysiology, Third Edition.
LWBK707-Ans_p280-402.qxd 8/20/10 6:38PM Page 326 Aptara Inc

326 ANSWER KEY

12. Answers: a, b, e 16. Answer: c


RATIONALE: The constitutional risk factors include a RATIONALE: Liver damage, when it occurs, may
family history of hypertension, race, and age- range from mild hepatocellular necrosis with ele-
related increases in blood pressure. Another factor vation of liver enzymes to the more ominous
that is thought to contribute to hypertension is hemolysis, elevated liver function tests, and low
insulin resistance and the resultant hyperinsuline- platelet count (HELLP) syndrome that is associated
mia that occurs in metabolic abnormalities such as with significant maternal mortality.
type 2 diabetes. Lifestyle factors can contribute to 17. Answer: d
the development of hypertension by interacting RATIONALE: Hypertension in infants is associated
with other risk factors. These lifestyle factors most commonly with high umbilical catheteriza-
include high salt intake, excessive calorie intake tion and renal artery obstruction caused by throm-
and obesity, excessive alcohol consumption, and bosis. Cerebral vascular bleeds, coarctation of the
low intake of potassium. Although stress can raise aorta, and pheochromocytoma all can raise blood
blood pressure acutely, there is less evidence link- pressure; they are not the most common cause of
ing it to chronic elevations in blood pressure. hypertension in an infant.
Smoking and a diet high in saturated fats and cho- 18. Answer: a
lesterol, although not identified as primary risk fac- RATIONALE: Among the aging processes that
tors for hypertension, are independent risk factors contribute to an increase in blood pressure are a
for coronary heart disease and should be avoided. stiffening of the large arteries, particularly the
13. Answer: d aorta; decreased baroreceptor sensitivity; increased
RATIONALE: Like adrenal medullary cells, the tumor peripheral vascular resistance; and decreased renal
cells of a pheochromocytoma produce and secrete blood flow.
the catecholamines epinephrine and norepineph- 19. Answer: b
rine. The hypertension that develops is a result of RATIONALE: Pseudohypertension should be suspected
the massive release of these catecholamines. Their in older persons with hypertension in whom the
release may be paroxysmal rather than continuous, radial or brachial artery remains palpable but pulse-
causing periodic episodes of headache, excessive less at higher cuff pressures. The presenting parame-
sweating, and palpitations. Headache is the most ters of the patient are not compatible with essential,
common symptom and can be quite severe. orthostatic, or secondary hypertension.
Nervousness, tremor, facial pallor, weakness, 20. Answer: c
fatigue, and weight loss occur less frequently. RATIONALE: The renin-angiotensin-aldosterone sys-
Marked variability in blood pressure between tem plays a central role in blood pressure
episodes is typical. regulation. Angiotensin II has two major functions
14. Answer: a in the rennin-angiotensin-aldosterone system and
RATIONALE: Because chronic hypertension is associ- acts as both a short- and long-term regulation of
ated with autoregulatory changes in coronary blood pressure. It is a strong vasoconstrictor, espe-
artery, cerebral artery, and kidney blood flow, care cially of the arterioles regulating blood pressure in
should be taken to avoid excessively rapid decreases the short term. However, its second major action,
in blood pressure, which can lead to hypoperfusion the stimulation of aldosterone secretion from the
and ischemic injury. Therefore, the goal of initial adrenal gland, is the end of the rennin-
treatment measures should be to obtain a partial angiotensin-aldosterone loop. The aldosterone
reduction in blood pressure to a safer, less critical that is secreted notifies the kidneys to stop produc-
level, rather than to normotensive levels. tion of renin (the negative feedback in the loop)
15. Answer: b and contributes to the long-term regulation of
RATIONALE: Cerebral vasoconstriction probably is blood pressure by increasing salt and water reten-
an exaggerated homeostatic response designed to tion by the kidney.
protect the brain from excesses of blood pressure
and flow. The regulatory mechanisms often are
insufficient to protect the capillaries, and cerebral CHAPTER 19
edema frequently develops. As it advances,
papilledema (i.e., swelling of the optic nerve at SECTION II: ASSESSING YOUR
its point of entrance into the eye) ensues, giving UNDERSTANDING
evidence of the effects of pressure on the optic
Activity A
nerve and retinal vessels. The patient may have
headache, restlessness, confusion, stupor, motor 1. pericardium
and sensory deficits, and visual disturbances. 2. frictional
In severe cases, convulsions and coma follow. 3. pericarditis
Lethargy, nervousness, and hyperreflexia are not 4. effusion
signs or symptoms of cerebral edema in malignant 5. tamponade
hypertension. 6. constrictive

Copyright 2011. Wolters Kluwer Health | Lippincott Williams & Wilkins. Study Guide for Porths Essentials of Pathophysiology, Third Edition.
LWBK707-Ans_p280-402.qxd 8/20/10 6:38PM Page 327 Aptara Inc

ANSWER KEY 327

7. atherosclerosis 2.
8. metabolic activity, autoregulatory 1. g 2. e 3. b 4. h 5. a
9. increased activity 6. f 7. c 8. j 9. i 10. d
10. 12-Lead ECG Activity D
11. Echocardiography
1. The pericardial cavity has little reserve volume, so
12. Atherosclerosis
small additions of fluid increase the pericardial
13. stable, unstable
pressure. Right heart filling pressures are lower
14. chronic ischemic heart disease, acute coronary
than the left, and increases in pericardial fluid
syndrome
pressure will result in decreased right-side filling.
15. T-wave inversion, ST-segment elevation, develop-
2. Myocardial oxygen supply is determined by the
ment of an abnormal Q wave
coronary arteries, capillary inflow, and the ability of
16. resting membrane potential
hemoglobin to transport and deliver oxygen to the
17. troponin assays
heart muscle. Important factors in the transport
18. Acute ST-segment
and delivery of oxygen include the fraction of
19. 20 to 40
inspired oxygen in the blood and the number of
20. ventricular remodeling
red blood cells with normal functioning hemoglo-
21. vagal
bin. There are three major determinants of myocar-
22. nitroglycerin
dial oxygen demand (MVO2): the heart rate,
23. Atherectomy
myocardial contractility, and myocardial wall
24. papillary muscle
stress or tension. The heart rate is the most impor-
25. Stable angina
tant factor in myocardial oxygen demand for two
26. exertion, emotional
reasons: (1) as the heart rate increases, myocardial
27. genetic
oxygen consumption or demand also increases;
28. mixed
and (2) subendocardial coronary blood flow is
29. hypertrophic cardiomyopathy
reduced because of the decreased diastolic filling
30. Dilated
time with increased heart rates.
31. Polyarthritis
3. On rupture, lipid core provides a stimulus for
32. neurologic
platelet aggregation and thrombus formation.
33. valves
Both smooth muscle and foam cells in the lipid
34. stenosis
core contribute to the expression of tissue factor in
35. regurgitation
unstable plaques. Once exposed to blood, tissue
36. prolapse
factor initiates the extrinsic coagulation pathway,
37. stenosis
resulting in the local generation of thrombin and
38. regurgitation
deposition of fibrin.
39. fetal heart
4. Biomarkers for ACS include cardiac-specific
40. blood, cyanosis, pulmonary
troponin I (cTnI) and troponin T (cTnT), myoglo-
41. acyanotic
bin, and creatine kinase MB (CK-MB). As the
42. ventricular septal
myocardial cells become necrotic, their intracellu-
43. Kawasaki
lar enzymes begin to diffuse into the surrounding
Activity B interstitium and then into the blood.
Right 5. The pathophysiology is divided into three phases:
Superior Left Superior
vena pulmonary pulmonary vena
veins
development of the unstable plaque that ruptures,
cava Aortic veins cava
arch Aortic valve Left the acute ischemic event, and the long-term risk of
atrium Coronary
sinus Inferior recurrent events that remain after the acute event.
Circumflex branch vena
of left coronary cava Inflammation plays a prominent role in plaque
Right artery instability, with inflammatory cells releasing
atrium Anterior descending Right
branch of left atrium cytokines that cause the fibrous cap to become
coronary artery
thinner and more vulnerable to rupture. The acute
Right
coronary Left circumflex ischemic event can be caused by an increase in
branch Right
artery ventricle myocardial oxygen demand precipitated by tachy-
Right
ventricle Left Posterior descending cardia or hypertension or, more commonly, by a
ventricle branch of right
coronary artery decrease in oxygen supply related to a reduction in
coronary lumen diameter due to platelet-rich
thrombi or vessel spasm.
Activity C 6. The extent of the infarct depends on the location
1. and extent of occlusion, amount of heart tissue
1. i 2. h 3. c 4. d 5. a supplied by the vessel, duration of the occlusion,
6. f 7. g 8. b 9. e 10. j metabolic needs of the affected tissue, extent of

Copyright 2011. Wolters Kluwer Health | Lippincott Williams & Wilkins. Study Guide for Porths Essentials of Pathophysiology, Third Edition.
LWBK707-Ans_p280-402.qxd 8/20/10 6:38PM Page 328 Aptara Inc

328 ANSWER KEY

collateral circulation, and other factors such as


heart rate, blood pressure, and cardiac rhythm.
SECTION III: APPLYING YOUR KNOWLEDGE
An infarct may involve the endocardium, Activity E
myocardium, epicardium, or a combination of 1. Classic symptoms of a STEMI include:
these. Onset of event that is abrupt and having pain as
7. The term reperfusion refers to re-establishment of the significant symptom.
blood flow through use of fibrinolytic therapy, Pain is typically severe and crushing and usually
percutaneous coronary intervention, or coronary substernal
artery bypass grafting. Early reperfusion (within The pain can radiate to the left arm, neck, or jaw
15 to 20 minutes) after onset of ischemia can pre- Pain is not relieved by rest or nitroglycerin
vent necrosis and improve myocardial perfusion Gastrointestinal distress, including nausea and
in the infarct zone. Reperfusion after a longer vomiting
interval can salvage some of the myocardial cells Fatigue and weakness, especially of the arms and
that would have died owing to longer periods of legs
ischemia. It also may prevent microvascular injury Tachycardia, anxiety, restlessness, and feelings of
that occurs over a longer period. impending doom
8. A cardiomyopathy is a heterogeneous group of Pale, cool, moist skin
diseases of the myocardium associated with 2. The emergency department goals of management
mechanical and/or electrical dysfunction that usu- for a patient with a STEMI are:
ally exhibit inappropriate ventricular hypertrophy Identification of persons who are candidates for
or dilatation and are due to a variety of causes reperfusion therapy
that frequently are genetic. Cardiomyopathies Evaluation of the persons chief complaint, typi-
either are confined to the heart or are part of cally chest pain, along with other associated
generalized systemic disorders, often leading to symptoms to differentiate ACS from other
cardiovascular death or progressive heart diagnoses
failurerelated disability. Monitoring should be instituted: a 12-lead ECG
9. Rheumatic heart disease is a complication of should be obtained and read by a physician
immune-mediated response to group A streptococ- within 10 minutes of arrival within the
cal throat infection. The acute stage of rheumatic emergency department
fever includes a history of an initiating streptococ- Administration of oxygen, aspirin, nitrates, pain
cal infection and subsequent involvement of the medications, antiplatelet and anticoagulant ther-
connective tissue elements of the heart, blood ves- apy,
-adrenergic blocking agents, and an
sels, joints, and subcutaneous tissues. The recurrent angiotensin converting enzyme inhibitor
phase usually involves extension of the cardiac Persons with ECG evidence of infarction should
effects of the disease. The chronic phase of receive immediate reperfusion therapy with a
rheumatic fever is characterized by permanent thrombolytic agent or percutaneous coronary
deformity of the heart valves. intervention
10. Blood typically shunts across the ductus from the
higher pressure left side to the lower pressure right
SECTION IV: PRACTICING FOR NCLEX
side. A murmur is typically detected within days or
weeks of birth. The murmur is loudest at the Activity F
second left intercostal space, continuous through 1. Answer: a
systole and diastole, and has a characteristic RATIONALE: The pain typically is worse with deep
machinery sound. A widened pulse pressure is breathing, coughing, swallowing, and positional
common because of the continuous runoff of aor- changes because of changes in venous return and
tic blood into the pulmonary artery. cardiac filling. All other answers make the pain
11. Tetralogy of Fallot consists of four associated worse.
defects: (1) a ventricular septal defect involving 2. Answer: b
the membranous septum and the anterior RATIONALE: A key diagnostic finding is pulsus para-
portion of the muscular septum; (2) dextro- doxus or an exaggeration of the normal variation
position or shifting to the right of the aorta; in the systemic arterial pulse volume with respira-
(3) obstruction or narrowing of the pulmonary tion. None of the other answers occur in cardiac
outflow channel, including pulmonic valve tamponade.
stenosis, a decrease in the size of the pulmonary 3. Answer: c
trunk, or both; and (4) hypertrophy of the right RATIONALE: Kussmaul sign is an inspiratory disten-
ventricle because of the increased work required tion of the jugular veins caused by the inability of
to pump blood through the obstructed the right atrium, encased in its rigid pericardium,
pulmonary channels. to accommodate the increase in venous return

Copyright 2011. Wolters Kluwer Health | Lippincott Williams & Wilkins. Study Guide for Porths Essentials of Pathophysiology, Third Edition.
LWBK707-Ans_p280-402.qxd 8/20/10 6:38PM Page 329 Aptara Inc

ANSWER KEY 329

that occurs with inspiration. None of the other described as a constricting, squeezing, or suffocat-
physiologic signs occur in constrictive pericarditis. ing sensation. It usually is steady, increasing in
4. Answers: a, c, e intensity only at the onset and end of the attack.
RATIONALE: The major determinants of plaque vul- Changing positions abruptly does not cause an
nerability to disruption include the size of the attack of angina pectoris.
lipid-rich core, the stability and thickness of its 11. Answer: a
fibrous cap, the presence of inflammation, and RATIONALE: Serum biochemical markers for MI are
lack of smooth muscle cells. A decrease in blood normal in patients with chronic stable angina. All
pressure and coronary blood flow are not determi- other answers are tests used in the diagnosis of
nants of plaque vulnerability to rupture. angina.
5. Answer: d 12. Hypertrophic cardiomyopathygenetic
RATIONALE: The troponin assays have high Left ventricular noncompactiongenetic
specificity for myocardial tissue and have become Myocarditisacquired
the primary biomarker for the diagnosis of myocar- Dilated cardiomyopathymixed
dial infarction (MI). The troponin complex, which Peripartum cardiomyopathyacquired
is part of the actin filament, consists of three sub- 13. Answer: b
units (i.e., TnC, TnT, and TnI) that regulate RATIONALE: Alcoholic cardiomyopathy is the single
calcium-mediated actin-myosin contractile process most common identifiable cause of DCM in the
in striated muscle (see Chapter 1, Fig. 1-19). TnI United States and Europe. The other answers are
and TnT, which are present in cardiac muscle, incorrect.
begin to rise within 3 hours after the onset of MI 14. Answer: c
and may remain elevated for 7 to 10 days after the RATIONALE: The intracardiac vegetative lesions also
event. This is especially adventitious in the late have local and distant systemic effects. The loose
diagnosis of MI. The other blood work may be organization of these lesions permits the organisms
ordered, but not to confirm the diagnosis of MI. and fragments of the lesions to form emboli and
6. Answers: b, d travel in the bloodstream, causing cerebral, systemic,
RATIONALE: UA/NSTEMI is classified as either low or pulmonary emboli. Preventing the valves of the
or intermediate risk of acute MI, the diagnosis of heart from either opening or closing completely is
which is based on the clinical history, ECG not a systemic effect of the lesions. Fragmentation of
pattern, and serum biomarkers. The other answers the lesions does not make them larger.
are not diagnostic of UA/NSTEMI. 15. Answer: d
7. Answer: a RATIONALE: It is thought that antibodies directed
RATIONALE: The principal biochemical consequence against the M protein of certain strains of strepto-
of MI is the conversion from aerobic to anaerobic cocci cross-react with glycoprotein antigens in the
metabolism with inadequate production of energy heart, joint, and other tissues to produce an
to sustain normal myocardial function. As a result, autoimmune response through a phenomenon
a striking loss of contractile function occurs within called molecular mimicry. None of the other
60 seconds of onset. None of the other answers answers are correct.
occur. 16. Answer: a
8. Answer: b RATIONALE: Persons with palpitations and mild
RATIONALE: Although a number of analgesic agents tachyarrhythmias or increased adrenergic symp-
have been used to treat the pain of STEMI, toms and those with chest discomfort, anxiety,
morphine is usually the drug of choice. It usually and fatigue often respond to therapy with the
is indicated if chest pain is unrelieved with oxygen
-adrenergicblocking drugs. None of the other
and nitrates. The reduction in anxiety that accom- types of drugs are used in the treatment of mitral
panies the administration of morphine contributes valve prolapse to relieve symptoms or prevent
to a decrease in restlessness and autonomic nerv- complications.
ous system activity, with a subsequent decrease in 17. Answer: b
the metabolic demands of the heart. Morphine RATIONALE: Heart failure manifests itself as tachyp-
does not cause a feeling of depression to the client. nea or dyspnea at rest or on exertion. For the
9. Answer: c infant, this most commonly occurs during
RATIONALE: If blood flow can be restored within the feeding. The other answers are incorrect.
20- to 40-minute time frame, loss of cell viability 18. Answer: c
does not occur or is minimal. RATIONALE: The degree of obstruction may be
10. Answer: d dynamic and can increase during periods of stress
RATIONALE: Angina pectoris usually is precipitated causing hypercyanotic attacks (tet spells). None
by situations that increase the work demands of of the other answers occur in association with
the heart, such as physical exertion, exposure to tetralogy of Fallot or tet spells.
cold, and emotional stress. The pain typically is

Copyright 2011. Wolters Kluwer Health | Lippincott Williams & Wilkins. Study Guide for Porths Essentials of Pathophysiology, Third Edition.
LWBK707-Ans_p280-402.qxd 8/20/10 6:38PM Page 330 Aptara Inc

330 ANSWER KEY

CHAPTER 20 Activity C

SECTION II: ASSESSING YOUR


UNDERSTANDING
Activity A
1. Heart failure
2. coronary artery disease, hypertension, Right heart failure Left heart failure
valvular
3. large Congestion of peripheral tissues Decreased cardiac output Pulmonary congestion

4. Cardiac output
5. sympathetic, parasympathetic Dependent Liver congestion Activity Impaired gas Pulmonary
edema intolerance exchange edema
6. stroke volume and ascites and signs of
decreased
7. Ejection fraction GI tract Signs related tissue
Cyanosis Orthopnea
congestion to impaired liver perfusion
and signs of
8. decrease function
hypoxia
9. normal Anorexia, GI distress,
weight loss
Cough with Paroxysmal
frothy sputum nocturnal dyspnea
10. compensatory mechanisms
11. Frank-Starling
12. contractile, volume overload, pressure Activity D
overload 1. A number of factors determine cardiac contractility
13. diastolic by altering the systolic Ca levels. Catecholamines
14. compress, increase, delay increase Ca entry into the cell by phosphorylation
15. tachycardia of the Ca channels via a cAMP-dependent protein
16. side kinase. Another mechanism that can modulate
17. peripheral edema inotropy is the sodium ion (Na)/Ca exchange
18. hepatic pump and the ATPase dependent Ca pump on
19. Left ventricular failure the myocardial cell membrane. These pumps trans-
20. left port Ca out of the cell, thereby preventing the
21. High-output failure cell from becoming overloaded with Ca. If Ca
22. Low-output failure extrusion is inhibited, the rise in intracellular Ca
23. myocardial hypertrophy can increase inotropy.
24. acute heart failure 2. With both systolic and diastolic ventricular dysfunc-
25. Paroxysmal nocturnal tion, compensatory mechanisms are usually able to
26. Acute pulmonary edema maintain adequate resting cardiac function until the
27. brain later stages of heart failure. Therefore, cardiac func-
28. right tion measured at rest is a poor clinical indicator of
29. oxygenation the extent of cardiac impairment because cardiac
30. ventricular output may be relatively normal at rest.
31. brain natriuretic peptide (BNP) 3. With diastolic dysfunction, blood is unable to
32. left ventricular move freely into the left ventricle, causing an
33. Circulatory shock increase in intraventricular pressure at any given
34. myocardial infarction volume. The elevated pressures are transferred back-
35. Hypovolemic ward from the left ventricle into the atria and pul-
36. Vasodilatory monary venous system causing a decrease in lung
37. neurogenic shock compliance, which increases the work of breathing
38. immunologically and evokes symptoms of dyspnea. Cardiac output is
39. Structural decreased because of a decrease in the volume (pre-
40. Aging load) available for adequate cardiac output. Inade-
Activity B quate cardiac output during exercise may lead to
1. fatigue of the legs and the accessory muscles of
1. j 2. f 3. d 4. i 5. c respiration.
6. b 7. e 8. a 9. g 10. h 4. Adaptive responses:
2. Frank-Starling mechanism: increases inotropy but
1. g 2. c 3. a 4. d 5. b eventually increases metabolic demand of cardiac
6. f 7. h 8. e tissue
3. Activation of the sympathetic nervous system:
1. c 2. e 3. b 4. d 5. a increases inotropy, but increases wall tension and
metabolic demand

Copyright 2011. Wolters Kluwer Health | Lippincott Williams & Wilkins. Study Guide for Porths Essentials of Pathophysiology, Third Edition.
LWBK707-Ans_p280-402.qxd 8/20/10 6:38PM Page 331 Aptara Inc

ANSWER KEY 331

The renin-angiotensin-aldosterone mechanism: 3. Answers: a, b, d, e


increases blood volume and maintains cardiac RATIONALE: Shock is not a specific disease but a syn-
output, but eventually increases wall tension drome that can occur in the course of many life-
Natriuretic peptides: inhibit sympathetic and threatening traumatic conditions or disease states.
renal compensation and decrease work of heart, It can be caused by an alteration in cardiac function
but inactivation results in decreased cardiac (cardiogenic shock), a decrease in blood volume
output (hypovolemic shock), excessive vasodilation with
Myocardial hypertrophy and remodeling: maldistribution of blood flow (distributive shock),
cardiomyocyte hypertrophy and increased or obstruction of blood flow through the circulatory
inotropy, but increase demands more oxygen and system (obstructive shock). Excessive vasoconstric-
increases metabolic needs tion and hypervolemia are not causes of shock.
5. The signs and symptoms include shortness of 4. Answers: a, c, e
breath and other respiratory manifestations, fatigue RATIONALE: Signs and symptoms of cardiogenic
and limited exercise tolerance, fluid retention and shock include indications of hypoperfusion with
edema, cachexia and malnutrition, and cyanosis. hypotension, although a preshock state of hypo-
Persons with severe heart failure may exhibit perfusion may occur with a normal blood pressure.
diaphoresis and tachycardia. These signs are the The lips, nail beds, and skin may become cyanotic
result of decreased tissue perfusion and resultant because of stagnation of blood flow and increased
hypoxia. extraction of oxygen from the hemoglobin as it
6. Diuretics promote the excretion of fluid and help to passes through the capillary bed. Mean arterial and
sustain cardiac output and tissue perfusion by systolic blood pressures decrease due to poor stroke
reducing preload and allowing the heart to operate volume, and there is a narrow pulse pressure and
at a more optimal part of the Frank-Starling curve. near-normal diastolic blood pressure due to arterial
7. In severe shock, cellular metabolic processes are vasoconstriction. Urine output decreases due to
essentially anaerobic because of the decreased avail- lower renal perfusion pressures and the increased
ability of oxygen. Excess amounts of lactic acid release of aldosterone. Elevation of preload is
accumulate in the cellular and the extracellular reflected in a rise in CVP and pulmonary capillary
compartment, and limited amounts of ATP are wedge pressure. Neurologic changes, such as alter-
produced and normal cell function cannot be ations in cognition or consciousness, may occur
maintained. The sodium-potassium membrane due to low cardiac output and poor cerebral perfu-
pump is impaired, resulting in cellular edema and sion. The other physiologic occurrences are not
an increase in the permeability of cell membranes. signs or symptoms of shock.
Mitochondrial activity becomes severely depressed 5. Answer: c
and lysosomal membranes may rupture, resulting RATIONALE: The treatment of hypovolemic shock is
in the release of enzymes that cause further directed toward correcting or controlling the
intracellular destruction. This is followed by cell underlying cause and improving tissue perfusion.
death and the release of intracellular contents into Ongoing loss of blood must be corrected, such as
the extracellular spaces. The destruction of the cell in surgery. Oxygen is administered to increase oxy-
membrane activates the arachidonic acid cascade, gen delivery to the tissues. Medications usually are
release of inflammatory mediators, and production administered intravenously. In hypovolemic shock,
of oxygen free radicals that extend cellular damage. the goal of treatment is to restore vascular volume.
8. (1) Pulmonary injury, (2) acute renal failure, (3) gas- This can be accomplished through intravenous
trointestinal ulceration, (4) disseminated intravas- administration of fluids and blood. The crystalloids
cular coagulation, and (5) multiple organ (e.g., isotonic saline and Ringers lactate) are readily
dysfunction syndrome. available and effective, at least temporarily. Plasma
volume expanders (e.g., pentastarch and colloidal
SECTION III: PRACTICING FOR NCLEX albumin) have a high molecular weight, do not
necessitate blood typing, and remain in the vascu-
Activity E
lar space for longer periods than the crystalloids,
1. Answers: 1-f, 2-a, 3-c, 4-b, 5-e, 6-d such as dextrose and saline. Blood or blood products
2. Answers: a, c (packed or frozen red cells) are administered based
RATIONALE: The signs and symptoms of heart failure on hematocrit and hemodynamic findings. Fluids
include shortness of breath and other respiratory and blood are best administered based on volume
manifestations, fatigue and limited exercise indicators such as CVP and urine output. Vasoactive
tolerance, fluid retention and edema, cachexia and medications are agents capable of constricting or
malnutrition, and cyanosis. Persons with severe dilating blood vessels. Considerable controversy
heart failure may exhibit diaphoresis and tachycar- exists about the advantages or disadvantages
dia. A ruddy complexion, bradycardia, and a related to the use of these drugs. As a general rule,
chronic productive cough are not signs or vasoconstrictor agents are not used as a primary
symptoms of heart failure.

Copyright 2011. Wolters Kluwer Health | Lippincott Williams & Wilkins. Study Guide for Porths Essentials of Pathophysiology, Third Edition.
LWBK707-Ans_p280-402.qxd 8/20/10 6:38PM Page 332 Aptara Inc

332 ANSWER KEY

form of therapy in hypovolemic shock and may be 3. conducting


detrimental. These agents are given only when vol- 4. warmed, filtered, moistened
ume deficits have been corrected but hypotension 5. mucus
persists. 6. glottis
6. Answer: a 7. hyaline
RATIONALE: In contrast to other shock states due to 8. hilum
the loss of blood volume or impaired cardiac func- 9. pulmonary lobule
tion, the heart rate in neurogenic shock often is 10. alveoli
slower than normal, and the skin is dry and warm. 11. pulmonary
This type of distributive shock is rare and usually 12. lymphatic
transitory. The other answers are not correct. 13. parasympathetic
7. Answer: b 14. sympathetic
RATIONALE: Anaphylaxis is a clinical syndrome that 15. partial pressure
represents the most severe form of systemic allergic 16. pressure difference
reaction. Anaphylactic shock results from an 17. intrapleural
immunologically mediated reaction in which 18. Valsalva
vasodilator substances such as histamine are 19. compliance
released into the blood. The vascular response in 20. tidal volume (TV)
anaphylaxis is often accompanied by life-threaten- 21. inspiratory reserve volume (IRV), expiratory
ing laryngeal edema and bronchospasm, circulatory reserve volume (ERV)
collapse, contraction of gastrointestinal and uterine 22. inspiratory capacity
smooth muscle, and urticaria (hives) or angioedema. 23. vital capacity
8. Answer: a 24. minute volume
RATIONALE: Although activated neutrophils kill 25. Pulmonary, alveolar
microorganisms, they also injure the endothelium 26. collapse
by releasing mediators that increase vascular per- 27. Dead space
meability. In addition, activated endothelial cells 28. mismatching
release nitric oxide, a potent vasodilator that acts 29. blood
as a key mediator of septic shock. 30. Hemoglobin
9. Answer: b 31. cooperatively
RATIONALE: The primary physiologic result of 32. pH, carbon dioxide, temperature.
obstructive shock is elevated right heart pressure 33. dissolved carbon dioxide, hemoglobin,
due to impaired right ventricular function. The bicarbonate
other answers are not correct. 34. pneumotaxic, apneustic
10. Answer: c 35. chemoreceptors, lung
RATIONALE: The degree of renal damage in shock is 36. carbon dioxide
related to the severity and duration of shock. 37. Dyspnea
None of the other answers relate to the damage to Activity B
the renal system in shock.
11. Answer: c
RATIONALE: Major risk factors for the development of
Trachea
MODS are severe trauma, sepsis, prolonged periods
of hypotension, hepatic dysfunction, infarcted
bowel, advanced age, and alcohol abuse. Respiratory
Left primary
dysfunction is not a major risk factor in MODS.
bronchus
12. Answer: b
Secondary
RATIONALE: Structural (congenital) heart defects are bronchi
the most common cause of heart failure in
children. The other answers are not correct.
Segmental
bronchi

CHAPTER 21
Terminal
SECTION II: ASSESSING YOUR bronchioles
UNDERSTANDING
Activity A
1. gas exchange
2. conducting, respiratory

Copyright 2011. Wolters Kluwer Health | Lippincott Williams & Wilkins. Study Guide for Porths Essentials of Pathophysiology, Third Edition.
LWBK707-Ans_p280-402.qxd 8/20/10 6:38PM Page 333 Aptara Inc

ANSWER KEY 333

Activity C area (SA) of the membrane, and the diffusion coeffi-


cient (D), and is inversely proportional to the thick-
1. f 2. g 3. h 4. b 5. i
ness (T) of the membrane.
6. a 7. c 8. j 9. d 10. e
5. Arterial blood is commonly used for measuring
Activity D blood gases. Venous blood is not used because
venous levels of oxygen and carbon dioxide reflect
a S c S b S g S f S e S d the metabolic demands of the tissues rather than
the gas exchange function of the lungs.
6. Coughing is a neurally mediated reflex that
Activity E protects the lungs from accumulation of secretions
1. The pleural membrane lines the thoracic cavity and and from entry of irritating and destructive sub-
encases the lungs. The outer parietal layer lines the stances. It is one of the primary defense
pulmonary cavities and adheres to the thoracic mechanisms of the respiratory tract. The cough
wall, the mediastinum, and the diaphragm. The reflex is initiated by receptors located in the tra-
inner visceral pleura closely covers the lung and is cheobronchial wall; these receptors are extremely
adherent to all its surfaces. It is continuous with sensitive to irritating substances and to the pres-
the parietal pleura at the hilum of the lung, where ence of excess secretions. Afferent impulses from
the major bronchus and pulmonary vessels enter these receptors are transmitted through the vagus
and leave the lung. A thin film of serous fluid sepa- to the medullary center, which integrates the
rates the two pleural layers, allowing the two layers cough response.
to glide over each other and yet hold together, so
there is no separation between the lungs and the SECTION III: APPLYING YOUR KNOWLEDGE
chest wall.
Activity F
2. During inspiration, the size of the chest cavity
increases, the intrathoracic pressure becomes more 1. When the oxygen levels in the body drop below a
negative, and air is drawn into the lungs. The specific set-point, the small blood vessels in the
diaphragm is the principal muscle of inspiration. lungs go into a vasoconstrictive state; they squeeze
When the diaphragm contracts, the abdominal down, so very little blood can go through them.
contents are forced downward and the chest This means that no oxygen is exchanged at this
expands from top to bottom. The external point either. This vasoconstriction can occur in a
intercostal muscles, which also aid in inspiration, limited part of the lung, or it can occur throughout
connect to the adjacent ribs and slope downward the lung. This is called generalized hypoxia.
and forward. When they contract, they raise the 2. Blood gas and pulmonary function tests should be
ribs and rotate them slightly so that the sternum is ordered.
pushed forward; this enlarges the chest from side to
side and from front to back. The scalene muscles SECTION IV: PRACTICING FOR NCLEX
elevate the first two ribs, and the sternocleidomas- Activity G
toid muscles raise the sternum to increase the size
of the chest cavity. Expiration is largely passive. It 1. Answers: a, c
occurs as the elastic components of the chest wall RATIONALE: The lungs are the functional structures
and lung structures that were stretched during of the respiratory system. In addition to their gas
inspiration recoil, causing air to leave the lungs as exchange function, they inactivate vasoactive sub-
the intrathoracic pressure increases. When needed, stances such as bradykinin; they convert
the abdominal and the internal intercostal muscles angiotensin I to angiotensin II. They also serve as
can be used to increase expiratory effort. a reservoir for blood storage. Heparin-producing
3. Pulmonary surfactant forms a monolayer with its cells are particularly abundant in the capillaries of
hydrophilic surface binding to liquid film on the the lung, where small clots may be trapped. The
surface of the alveoli and its hydrophobic surface other functions do not occur in the lungs.
facing outward toward the gases in the alveolar air. 2. Answer: a
This monolayer interrupts the surface tension that RATIONALE: The bronchial blood vessels are the
develops at the air-liquid interface in the alveoli, only ones that can undergo angiogenesis (forma-
keeping them from collapsing and allowing equal tion of new vessels) and develop collateral circula-
inflation. tion when vessels in the pulmonary circulation are
4. Gas diffusion in the lung is described by the Fick obstructed, as in pulmonary embolism. The devel-
law of diffusion. The Fick law states that the opment of new blood vessels helps to keep lung
volume of a gas diffusing across the membrane per tissue alive until the pulmonary circulation can
unit time is directly proportional to the partial be restored. The blood in the bronchiole blood
pressure difference of the gas (P1 P2), the surface vessels is unoxygenated, so they neither carry

Copyright 2011. Wolters Kluwer Health | Lippincott Williams & Wilkins. Study Guide for Porths Essentials of Pathophysiology, Third Edition.
LWBK707-Ans_p280-402.qxd 8/20/10 6:38PM Page 334 Aptara Inc

334 ANSWER KEY

oxygen-rich blood to the lung tissues nor partici- ide reflect the metabolic demands of the tissues
pate in gas exchange. Bronchiole blood vessels rather than the gas exchange function of the
drain blood into the bronchial veins. lungs. The other answers are not correct.
3. Answers: 1-b, 2-d, 3-c, 4-a 10. Answers: a, c, e
4. Answer: b RATIONALE: The automatic and voluntary
RATIONALE: Specifically, lung compliance (C) components of respiration are regulated by
describes the change in lung volume ( V) that afferent impulses that are transmitted to the respi-
can be accomplished with a given change in res- ratory center from a number of sources. Afferent
piratory pressure ( P); thus, (C  V/ P). This input from higher brain centers is evidenced by
equation has nothing to do with surface tension, the fact that a person can consciously alter the
airway resistance, or a change in peak expiratory depth and rate of respiration. Fever, pain, and
flow. emotion exert their influence through lower brain
5. Answer: c centers.
RATIONALE: The work of breathing is determined by 11. Answers: d-c-e-a-b
the amount of effort required to move air through RATIONALE: Coughing itself requires the rapid
the conducting airways and by the ease of lung inspiration of a large volume of air (usually about
expansion, or compliance. Expansion of the lungs 2.5 L), followed by rapid closure of the glottis and
is difficult for persons with stiff and noncompliant forceful contraction of the abdominal and expira-
lungs; they usually find it easier to breathe if they tory muscles. As these muscles contract, intratho-
keep their TV low and breathe at a more rapid rate racic pressures are elevated to levels of 100 mm Hg
(e.g., 300 20  6000 mL) to achieve their or more. The rapid opening of the glottis at this
minute volume and meet their oxygen needs. In point leads to an explosive expulsion of air.
contrast, persons with obstructive airway disease 12. Answer: d
usually find it less difficult to inflate their lungs RATIONALE: Dyspnea is observed in at least three
but expend more energy in moving air through major cardiopulmonary disease states: primary
the airways. As a result, these persons take lung diseases, such as pneumonia, asthma, and
deeper breaths and breathe at a slower rate (e.g., emphysema; heart disease that is characterized by
600 10  6000 mL) to achieve their oxygen pulmonary congestion; and neuromuscular disor-
needs. People with COPD do not have hyperpneic ders, such as myasthenia gravis and muscular
breathing under normal conditions. dystrophy that affect the respiratory muscles. Dys-
6. Answer: c pnea is not an identified component of multiple
RATIONALE: The distribution of ventilation between sclerosis.
the apex and base of the lung varies with body
position and the effects of gravity on intrapleural
pressure. Intrapleural pressure impacts the CHAPTER 22
distribution of ventilation, not intrathoracic or
alveolar pressures. SECTION II: ASSESSING YOUR
7. Answer: d UNDERSTANDING
RATIONALE: Generalized hypoxia occurs at high
Activity A
altitudes and in persons with chronic hypoxia due
to lung disease, and causes vasoconstriction 1. Viruses
throughout the lung. Prolonged hypoxia can lead 2. bronchial, obstruct, bacterial
to pulmonary hypertension and increased 3. upper
workload on the right heart. 4. rhinoviruses
8. Answer: a 5. Antihistamines
RATIONALE: Physiologic shunting of blood usually 6. Rhinitis, paranasal
results from destructive lung disease that impairs 7. oxygen
ventilation or from heart failure that interferes 8. hemagglutinin, neuraminidase
with movement of blood through sections of the 9. upper, viral, bacterial
lungs. Obstructive lung disease, pulmonary hyper- 10. vaccination
tension, and regional hypoxia usually do not 11. reassortment
cause the physiologic shunting of blood. 12. pneumonia
9. Answer: b 13. Lobar pneumonia, bronchopneumonia
RATIONALE: In the clinical setting, blood gas mea- 14. nosocomial
surements are used to determine the partial 15. immunocompromised
pressure of oxygen (PO2) and carbon dioxide (PCO2) 16. Legionnaire
in the blood. Arterial blood commonly is used for 17. mycoplasma
measuring blood gases. Venous blood is not used 18. Tuberculosis
because venous levels of oxygen and carbon diox- 19. waxy

Copyright 2011. Wolters Kluwer Health | Lippincott Williams & Wilkins. Study Guide for Porths Essentials of Pathophysiology, Third Edition.
LWBK707-Ans_p280-402.qxd 8/20/10 6:38PM Page 335 Aptara Inc

ANSWER KEY 335

20. Primary Activity D


21. tuberculin skin, x-rays
22. Histoplasmosis
23. Fungal Inhalation of
24. smoking tubercle bacillus
25. 80%
26. Lung cancers
27. small cell lung cancers (SCLCs)
28. nonsmall cell lung cancers (NSCLCs)
29. Croup
30. 25th to 28th
Activity B
Primary Secondary
tuberculosis tuberculosis

Cell-mediated Development of
hypersensitivity cell-mediated Reinfection
response immunity
Frontal sinus

Ethmoid sinuses Granulomatous


Positive skin
inflammatory
test
response
Maxillary sinus

Progressive
Ghon
or disseminated
complex
tuberculosis

Healed dormant Reactivated


lesion tuberculosis
Frontal sinus

Sphenoidal Activity E
sinus
Superior turbinate
1. The fingers are the greatest source of spread, and
Middle turbinate the nasal mucosa and conjunctival surface of the
eyes are the most common portals of entry of the
Inferior turbinate virus. The most highly contagious period is during
the first 3 days after the onset of symptoms, and
the incubation period is approximately 5 days.
Cold viruses have been found to survive for more
than 5 hours on the skin and hard surfaces, such
as plastic countertops. Aerosol spread of colds,
through coughing and sneezing, is much less
important than the spread by fingers picking up
the virus from contaminated surfaces and carrying
Activity C it to the nasal membranes and eyes.
1. i 2. c 3. a 4. b 5. d 2. Contagion results from the ability of the influenza
6. j 7. e 8. g 9. g 10. h A virus to develop new HA and NA subtypes
against which the population is not protected. An
antigenic shift, which involves a major genetic
rearrangement in either antigen, may lead to epi-
demic or pandemic infection. Lesser changes,
called antigenic drift, find the population partially
protected by cross-reacting antibodies.
3. Viral pneumonia occurs as a complication of
influenza. It typically develops within 1 day after

Copyright 2011. Wolters Kluwer Health | Lippincott Williams & Wilkins. Study Guide for Porths Essentials of Pathophysiology, Third Edition.
LWBK707-Ans_p280-402.qxd 8/20/10 6:38PM Page 336 Aptara Inc

336 ANSWER KEY

onset of influenza and is characterized by rapid present their antigens to T lymphocytes. The sen-
progression of fever, tachypnea, tachycardia, sitized T lymphocytes, in turn, stimulate the
cyanosis, and hypotension. The clinical course of macrophages to increase their concentration of
influenza pneumonia progresses rapidly. It can lytic enzymes and ability to kill the mycobacteria.
cause hypoxemia and death within a few days of When released, these lytic enzymes also damage
onset. Survivors often develop diffuse pulmonary lung tissue. The development of a population of
fibrosis. activated T lymphocytes and related development
4. The lung below the main bronchi is normally ster- of activated macrophages capable of ingesting and
ile, despite frequent entry of microorganisms into destroying the bacilli constitutes the cell-mediated
the air passages by inhalation during ventilation immune response.
or aspiration of nasopharyngeal secretions. Bacter- 8. Lung cancer is classified as squamous cell lung car-
ial pneumonia results due to loss of the cough cinoma, adenocarcinoma, small cell carcinoma,
reflex, damage to the ciliated endothelium that and large cell carcinoma.
lines the respiratory tract, or impaired immune 9. The manifestations of lung cancer can be divided
defenses. Bacterial adherence also plays a role in into three categories: (1) those due to involvement
colonization of the lower airways. The epithelial of the lung and adjacent structures, (2) the effects
cells of critically and chronically ill persons are of local spread and metastasis, and (3) nonmetasta-
more receptive to binding microorganisms that tic paraneoplastic manifestations involving
cause pneumonia. Other clinical risk factors favor- endocrine, neurologic, and connective tissue
ing colonization of the tracheobronchial tree function.
include antibiotic therapy that alters the normal 10. Pulmonary immaturity, together with surfactant
bacterial flora, diabetes, smoking, chronic bronchi- deficiency, lead to alveolar collapse. The type II
tis, and viral infection. alveolar cells that produce surfactant do not begin
5. During the first stage, alveoli become filled with to mature until approximately the 25th to 28th
protein-rich edema fluid containing numerous weeks of gestation; consequently, many premature
organisms. Marked capillary congestion follows, infants are born with poorly functioning type II
leading to massive outpouring of polymorphonu- alveolar cells and have difficulty producing
clear leukocytes and red blood cells. Because the sufficient amounts of surfactant. Without
first consistency of the affected lung resembles surfactant, the large alveoli remain inflated,
that of the liver, this stage is referred to as the red whereas the small alveoli become difficult to
hepatization stage. The next stage involves the inflate, resulting in respiratory distress syndrome.
arrival of macrophages that phagocytose the frag-
mented polymorphonuclear cells, red blood cells, SECTION III: APPLYING YOUR KNOWLEDGE
and other cellular debris. During this stage, which
is termed the gray hepatization stage, the
Activity F
congestion has diminished but the lung is still 1. Diagnostic tests for squamous cell cancer of the lung
firm. The alveolar exudate is then removed and include chest radiography, bronchoscopy, cytologic
the lung returns to normal. studies (Papanicolaou [Pap] test) of the sputum or
6. M. tuberculosis hominis is an airborne infection bronchial washings, percutaneous needle biopsy of
spread by minute, invisible particles called droplet lung tissue, Scalene lymph node biopsy, computed
nuclei that are harbored in the respiratory secre- tomographic scans, MRI studies, ultrasonography to
tions of persons with active tuberculosis. Cough- locate lesions and evaluate the extent of the disease,
ing, sneezing, and talking all create respiratory and positron-emission tomography, a noninvasive
droplets; these droplets evaporate, leaving the alternative for identifying metastatic lesions in the
organisms, which remain suspended in the air and mediastinum or distant sites.
are circulated by air currents. Thus, living under 2. Treatments used for squamous cell (NSCLC) cancer
crowded and confined conditions increases the of the lung include surgery for the removal of
risk for spread of the disease. small, localized NSCLC tumors; radiation therapy, a
7. Inhaled droplet nuclei pass down the bronchial definitive or main treatment modality for palliation
tree without settling on the epithelium and are of symptoms; and chemotherapy, often using a
deposited in the alveoli. Soon after entering the combination of drugs. Often, a combination of
lung, the bacilli are phagocytosed by alveolar these treatments is used.
macrophages, but resist killing, because cell wall
lipids of the M. tuberculosis block fusion of phago- SECTION IV: PRACTICING FOR NCLEX
somes and lysosomes. Although the macrophages
Activity G
that first ingest M. tuberculosis cannot kill the
organisms, they initiate a cell-mediated immune 1. Answer: a
response that eventually contains the infection. RATIONALE: Decongestant drugs (i.e.,
As the tubercle bacilli multiply, the infected sympathomimetic agents) are available in over-
macrophages degrade the mycobacteria and the-counter nasal sprays, drops, and oral cold

Copyright 2011. Wolters Kluwer Health | Lippincott Williams & Wilkins. Study Guide for Porths Essentials of Pathophysiology, Third Edition.
LWBK707-Ans_p280-402.qxd 8/20/10 6:38PM Page 337 Aptara Inc

ANSWER KEY 337

medications. These drugs constrict the blood ves- 7. Answer: b


sels in the swollen nasal mucosa and reduce nasal RATIONALE: The pathogenesis of tuberculosis in a
swelling. Rebound nasal swelling can occur with previously unexposed immunocompetent person
indiscriminate use of nasal drops and sprays. Oral is centered on the development of a cell-mediated
preparations containing decongestants may cause immune response that confers resistance to the
systemic vasoconstriction and elevation of blood organism and development of tissue hypersensitiv-
pressure when given in doses large enough to ity to the tubercular antigens. The destructive
relieve nasal congestion. They should be avoided nature of the disease, such as caseating necrosis
by persons with hypertension, heart disease, and cavitation, results from the hypersensitivity
hyperthyroidism, diabetes mellitus, or other immune response rather than the destructive
health problems. capabilities of the tubercle bacillus. Tuberculosis
2. Answer: b does not have rapidly progressing pulmonary
RATIONALE: One distinguishing feature of an lesions, nor does it have purulent necrosis or puru-
influenza viral infection is the rapid onset, lent pulmonary lesions.
sometimes in as little as 1 to 2 minutes, of 8. Answer: c
profound malaise. None of the other answers RATIONALE: The oral antifungal drugs itraconazole
are distinguishing characteristics of an influenza and fluconazole are used for treatment of less
viral infection. severe forms of infection. Intravenous
3. Answer: c amphotericin B is used in the treatment of persons
RATIONALE: Recently, a highly pathogenic influenza with progressive disease. Long-term treatment is
A subtype H5N1 was found in poultry in East and often required. BCG is an attenuated strain of live
Southeast Asian Countries. Although the H5N1 tubercle vaccine. Rifampin is an oral drug used in
strain is highly contagious from one bird to the treatment of tuberculosis.
another, the transmission from human to human 9. Answer: d
is relatively inefficient and not sustained. The RATIONALE: The NSCLCs include squamous cell car-
result is only rare cases of person-to-person trans- cinomas, adenocarcinomas, and large cell carcino-
mission. Most cases occur after exposure to mas. As with the SCLCs, these cancers have the
infected poultry or surfaces contaminated with capacity to synthesize bioactive products and pro-
poultry droppings. Because infection in humans is duce paraneoplastic syndromes. NSCLCs do not
associated with high mortality, there exists consid- neutralize bioactive syndromes. In addition, they
erable concern that H5N1 strain might mutate and neither synthesize ACTH nor produce pan-
initiate a pandemic. neoplastic syndromes.
4. Answers: a, c, d 10. Answer: a
RATIONALE: Community-acquired pneumonia may RATIONALE: The infant with BPD often
be further categorized according to risk of mortal- demonstrates a barrel chest, tachycardia, rapid and
ity and need for hospitalization based on age, pres- shallow breathing, chest retractions, cough, and
ence of coexisting disease, and severity of illness poor weight gain. Other signs and symptoms listed
using physical examination findings and laboratory are not those of BPD.
and radiologic findings. The other answers are not 11. Epiglottitis: upper airway
categories used to classify community-acquired Acute bronchiolitis: lower airway
pneumonia. Asthma: lower airway
5. Answer: d Spasmodic croup: upper airway
RATIONALE: Neutropenia and impaired granulocyte Laryngotracheobronchitis: upper airway
function, as occurs in persons with leukemia, 12. Answer: b
chemotherapy, and bone marrow depression, RATIONALE: The child with bronchiolitis is at risk
predispose to infections caused by S. aureus, for respiratory failure resulting from impaired gas
Aspergillus, gram-negative bacilli, and candida. exchange. The other answers are not applicable.
All the other organisms can cause pneumonia, but
they are not usually seen in people with neutrope-
nia and impaired granulocyte function. CHAPTER 23
6. Answer: a
RATIONALE: Pleuritic pain, a sharp pain that is SECTION II: ASSESSING YOUR
more severe with respiratory movements, is UNDERSTANDING
common. With antibiotic therapy, fever usually
Activity A
subsides in approximately 48 to 72 hours, and
recovery is uneventful. Elderly persons are less 1. carbon dioxide (CO2), oxygen (O2)
likely to experience marked elevations in temper- 2. Ventilation
ature; in these persons, the only sign of pneumo- 3. oxygenation, removal of CO2
nia may be a loss of appetite and deterioration in 4. Hypoxemia
mental status. 5. hypoxia

Copyright 2011. Wolters Kluwer Health | Lippincott Williams & Wilkins. Study Guide for Porths Essentials of Pathophysiology, Third Edition.
LWBK707-Ans_p280-402.qxd 8/20/10 6:38PM Page 338 Aptara Inc

338 ANSWER KEY

6. ventilation, vasoconstriction, red blood Activity D


7. Hypercapnia
1.
8. pH, acidosis
9. Pleural effusion
10. Hemothorax
e S b S c S a S f S d
11. inflated
12. expiratory Activity E
13. asthma
14. T lymphocyte 1. The mechanisms that result in hypoxemia are hypo-
15. Chronic obstructive ventilation, impaired diffusion of gases, inadequate
16. inflammation, fibrosis circulation of blood through the pulmonary capillar-
17. Emphysema, proteases ies, and mismatching of ventilation and perfusion.
18. 1-antitrypsin 2. The clinical manifestations of atelectasis include
19. hypersecretion of mucus tachypnea, tachycardia, dyspnea, cyanosis, signs of
20. pink puffers hypoxemia, diminished chest expansion, absence
21. blue bloaters of breath sounds, and intercostal retractions. Both
22. Bronchiectasis chest expansion and breath sounds are decreased on
23. Cystic fibrosis the affected area. There may be intercostal retraction
24. interstitial over the involved area during inspiration.
25. collagen, elastic 3. The symptoms of the acute response are caused by
26. embolism the release of chemical mediators from the presen-
27. pulmonary embolism sitized mast cells. Mediator release results in the
28. Pulmonary hypertension infiltration of inflammatory cells, opening of the
29. hypoxemia mucosal intercellular junctions, and increased
30. Respiratory failure access of antigen to submucosal mast cells. There is
bronchospasm caused by direct stimulation of
Activity B parasympathetic receptors, mucosal edema caused
by increased vascular permeability, and increased
mucus secretions. The late-phase response involves
Attraction of
inflammation and increased airway responsiveness
Smoking
inflammatory cells that prolong the asthma attack. An initial trigger in
the late-phase response causes the release of inflam-
matory mediators from mast cells, macrophages,
Release of elastase and epithelial cells. These substances induce the
migration and activation of other inflammatory
cells, which then produce epithelial injury and
1 edema, changes in mucociliary function and
reduced clearance of respiratory tract secretions,
and increased airway responsiveness.
4. The two processes that are critical to the pathogen-
1 1 esis of bronchiectasis are airway obstruction and
chronic persistent infection, causing damage to the
Macrophages bronchial walls, leading to weakening and dilation.
and neutrophils 5. Cystic fibrosis is caused by mutations in a single
Destruction of
elastic fibers in lung gene on the long arm of chromosome that
encodes for the cystic fibrosis transmembrane reg-
ulator (CFTR), which functions as a chloride
Emphysema channel in epithelial cell membranes. Mutations
in the CFTR gene render the epithelial membrane
relatively impermeable to the chloride ion. The
impaired transport of Cl ultimately leads to a series
of secondary events, including increased absorption
Activity C of Na and water from the airways into the blood.
1. This lowers the water content of the mucociliary
1. d 2. b 3. i 4. a 5. j blanket coating the respiratory epithelium, causing
6. e 7. f 8. g 9. h 10. c it to become more viscid. The resulting dehydration
2. of the mucous layer leads to defective mucociliary
1. e 2. b 3. g 4. i 5. d function and accumulation of viscid secretions that
6. j 7. a 8. h 9. f 10. c obstruct the airways and predispose to recurrent
pulmonary infections. The obstruction develops

Copyright 2011. Wolters Kluwer Health | Lippincott Williams & Wilkins. Study Guide for Porths Essentials of Pathophysiology, Third Edition.
LWBK707-Ans_p280-402.qxd 8/20/10 6:38PM Page 339 Aptara Inc

ANSWER KEY 339

from the thick mucous and recurrent infections RATIONALE: Hypoxemia can result from an
damage lung tissue leading to the development of inadequate amount of O2 in the air, disease of the
bronchiectasis. respiratory system, dysfunction of the neurologic
6. Obstruction of pulmonary blood flow causes reflex system, or alterations in circulatory function. The
bronchoconstriction in the affected area of the lung, mechanisms whereby respiratory disorders lead to
wasted ventilation and impaired gas exchange, and a significant reduction in PO2 are hypoventilation,
loss of alveolar surfactant. Pulmonary hypertension impaired diffusion of gases, inadequate circulation
and right heart failure may develop when there is of blood through the pulmonary capillaries, and
massive vasoconstriction because of a large embolus. mismatching of ventilation and perfusion.
7. Pathologic lung changes include diffuse epithelial 2. Answers: a, b, c, e
cell injury with increased permeability of the alveo- RATIONALE: Hypercapnia refers to an increase in
lar-capillary membrane, which permits fluid, plasma carbon dioxide levels. In the clinical setting, four
proteins, and blood cells to move out of the vascu- factors contribute to hypercapnia: alterations in
lar compartment into the interstitium and alveoli of carbon dioxide production, disturbance in the gas
the lung. Diffuse alveolar cell damage leads to accu- exchange function of the lungs, abnormalities in
mulation of fluid, surfactant inactivation, and respiratory function of the chest wall and respira-
formation of a hyaline membrane. The work of tory muscles, and changes in neural control of res-
breathing becomes greatly increased as the lung piration. A decrease in carbon dioxide production
stiffens and becomes more difficult to inflate. There does not cause hypercapnia.
is increased intrapulmonary shunting of blood, 3. Answer: b
impaired gas exchange, and hypoxemia despite RATIONALE: One of the complications of untreated
high supplemental oxygen therapy. Gas exchange is moderate or large hemothorax is fibrothoraxthe
further compromised by alveolar collapse resulting fusion of the pleural surfaces by fibrin, hyalin, and
from abnormalities in surfactant production. When connective tissueand in some cases, calcification
injury to the alveolar epithelium is severe, disorgan- of the fibrous tissue, which restricts lung
ized epithelial repair may lead to fibrosis. expansion. Calcification of the lung tissue does
not occur because of a hemothorax, neither does
SECTION III: APPLYING YOUR KNOWLEDGE pleuritis or an atelectasis.
4. Answer: c
Activity F RATIONALE: Persons with talc lung are also highly
1. Diagnostic tests that the nurse would expect to be susceptible to the occurrence of pneumothorax.
ordered to confirm the diagnosis of asthma include Talc lung may result from inhalation of talc parti-
spirometry, inhalation challenge tests, and labora- cles, but is more commonly an occurrence of
tory findings. injected or inhaled talc powder that is used as a
2. A plan of care will be developed with the input of filler with heroin, methamphetamine, or codeine.
both you and your daughter to encourage A hemothorax is not a complication of talc lung,
independence as it relates to the control of her neither are chylothorax or fibrothorax.
symptoms, along with measures directed at helping 5. Answer: a
her develop and keep a positive self-concept. RATIONALE: Treatment of pleuritis consists of treat-
ing the underlying disease and inflammation.
SECTION IV: PRACTICING FOR NCLEX Analgesics and nonsteroidal anti-inflammatory
drugs (e.g., indomethacin) may be used for pleural
Activity G
pain. Although these agents reduce inflammation,
1. they may not entirely relieve the discomfort asso-
ciated with deep breathing and coughing. The
Mechanism Outcome
other answers are not used to treat pleural pain.
Decreased oxygen in air Hypoxemia 6. Answer: c
RATIONALE: If the collapsed area is large, the medi-
Inadequate circulation through Decreased PO2 astinum and trachea shift to the affected side. In
pulmonary capillaries compression atelectasis, the mediastinum shifts
away from the affected lung. None of the other
Hypoventilation Decreased PO2
answers are correct.
Disease in respiratory system Hypoxemia 7. Answer: a
RATIONALE: For children younger than 2 years of
Mismatched ventilation & perfusion Decreased PO2
age, nebulizer therapy usually is preferred.
Dysfunction of neurologic Hypoxemia Children between 3 and 5 years of age may begin
system using an MDI with a spacer and holding chamber.
The other answers are not correct.

Copyright 2011. Wolters Kluwer Health | Lippincott Williams & Wilkins. Study Guide for Porths Essentials of Pathophysiology, Third Edition.
LWBK707-Ans_p280-402.qxd 8/20/10 6:38PM Page 340 Aptara Inc

340 ANSWER KEY

8. Answer: b 14. Answer: b


RATIONALE: The term chronic obstructive pulmonary RATIONALE: Continued exposure of the pulmonary
disease encompasses two types of obstructive vessels to hypoxemia is a common cause of
airway disease: emphysema, with enlargement of pulmonary hypertension. Unlike blood vessels in
air spaces and destruction of lung tissue; and the systemic circulation, most of which dilate in
chronic obstructive bronchitis, with increased response to hypoxemia and hypercapnia, the pul-
mucus production, obstruction of small airways, monary vessels constrict. None of the other
and a chronic productive cough. Persons with answers are correct.
COPD often have overlapping features of both dis- 15. Answer: c
orders. Asthma and chronic bronchitis have not RATIONALE: Management of cor pulmonale focuses
been identified as components of COPD. on the treatment of the lung disease and heart
9. Answer: c failure. Low-flow oxygen therapy may be used
RATIONALE: In the past, bronchiectasis often to reduce the pulmonary hypertension and
followed a necrotizing bacterial pneumonia that polycythemia associated with severe hypoxemia
frequently complicated measles, pertussis, or caused by chronic lung disease. Low-flow oxygen
influenza. Chickenpox has never been linked to used in treating cor pulmonale does not stimulate
bronchiectasis. the body to breathe; it does not act in an
10. Answer: d inhibitory way on the respiratory center in the
RATIONALE: In addition to airway obstruction, the brain; nor does it reduce the formation of
basic genetic defect that occurs with CF predisposes pulmonary emboli.
to chronic infection with a surprising small number 16. Answers: b, d, e
of organisms, the most common being Pseudomonas RATIONALE: Clinically, ALI/ARDS is marked by a
aeruginosa, Burkholderia cepacia, Staphylococcus rapid onset, usually within 12 to 18 hours of the
aureus, and Haemophilus influenzae. The other initiating event, of respiratory distress, an increase
disease causing organisms are not linked to CF. in respiratory rate, and signs of respiratory failure.
11. Answers: a, b, e Chest radiography shows diffuse bilateral
RATIONALE: Important etiologic determinants in infiltrates of the lung tissue in the absence of car-
the development of the pneumoconioses are the diac dysfunction. Marked hypoxemia occurs that
size of the dust particle, its chemical nature and is refractory to treatment with supplemental oxy-
ability to incite lung destruction, and the concen- gen therapy, which results in a decrease in the PF
tration of dust and the length of exposure to it. ratio. Many persons with ARDS have a systemic
The density and biologic nature of the dust parti- response that results in multiple organ failure, par-
cles are not linked to their ability to cause ticularly the renal, gastrointestinal, cardiovascular,
pneumoconioses. and central nervous systems. The other answers
12. Answer: a are not clinical signs of ARDS.
RATIONALE: Drugs can cause a variety of both 17. Answer: d
acute and chronic alterations in lung function. RATIONALE: Many of the adverse consequences of
For example, some of the cytotoxic drugs (e.g., hypercapnia are the result of respiratory acidosis.
bleomycin, busulfan, methotrexate, and Direct effects of acidosis include depression of car-
cyclophosphamide) used in treatment of cancer diac contractility, decreased respiratory muscle
cause pulmonary damage as a result of direct toxi- contractility, and arterial vasodilation. Raised lev-
city of the drug and by stimulating the influx of els of PCO2 greatly increase cerebral blood flow,
inflammatory cells into the alveoli. Amiodarone, a which may result in headache, increased cerebral
drug used to treat resistant cardiac arrhythmias, is spinal fluid pressure, and sometimes papilledema.
preferentially sequestered in the lung and causes
significant pneumonitis in 5% to 15% of persons
receiving it. Inderal does not cause a direct toxicity CHAPTER 24
in the lungs.
13. Answer: b SECTION II: ASSESSING YOUR
RATIONALE: Chest pain, dyspnea, and increased res- UNDERSTANDING
piratory rate are the most frequent signs and
Activity A
symptoms of pulmonary embolism. Pulmonary
infarction often causes pleuritic pain that changes 1. kidneys
with respiration; it is more severe on inspiration 2. hilus
and less severe on expiration. Mediastinal and tra- 3. Nephrons
cheal shifts are not signs of a pulmonary 4. cortex
infarction, and neither is pericardial pain. 5. renal pyramids
6. aorta

Copyright 2011. Wolters Kluwer Health | Lippincott Williams & Wilkins. Study Guide for Porths Essentials of Pathophysiology, Third Edition.
LWBK707-Ans_p280-402.qxd 8/20/10 6:38PM Page 341 Aptara Inc

ANSWER KEY 341

7. glomeruli sive transport mechanisms. Water and urea are pas-


8. glomerulus sively absorbed along concentration gradients.
9. Peritubular capillaries Sodium, K, chloride, calcium, and phosphate ions,
10. filtrate as well as urate, glucose, and amino acids, are reab-
11. glomerular filtration rate (GFR) sorbed using primary or secondary active transport
12. concentration, basolateral mechanisms to move across the tubular membrane.
13. Cotransport Some substances, such as hydrogen, potassium, and
14. proximal urate ions, are secreted into the tubular fluids.
15. renal threshold 3. The juxtaglomerular complex is a feedback control
16. loop of Henle system that links changes in the GFR with renal
17. NaK2Cl blood flow. It is located at the site where the distal
18. distal convoluted tubule extends back to the glomerulus and then
19. antidiuretic hormone passes between the afferent and efferent arteriole.
20. sympathetic The distal tubular site that is nearest the glomerulus
21. clearance is characterized by densely nucleated cells called
22. Aldosterone the macula densa. In the adjacent afferent arteriole,
23. sodium the smooth muscle cells of the media are modified
24. bicarbonate, hydrogen as special secretory cells called juxtaglomerular
25. Urea cells. These cells contain granules of inactive renin,
26. erythropoietin an enzyme that functions in the conversion of
27. Proteinuria angiotensinogen to angiotensin. Renin functions
28. specific gravity by means of angiotensin II to produce vasoconstric-
29. Creatinine tion of the efferent arteriole as a means of prevent-
30. BUN ing serious decreases in GFR. Angiotensin II also
Activity B increases sodium reabsorption indirectly by stimu-
lating aldosterone secretion from the adrenal gland
1. e 2. d 3. i 4. a 5. j
and directly by increasing sodium reabsorption by
6. g 7. b 8. h 9. f 10. c
the proximal tubule cells. The increase in sodium
Activity C will result in an increase in water retention, which
1. Decreased GFR S juxtaglomerular release of renin will increase blood volume and in turn increase
S conversion of angiotensinogen to angiotensin GFR.
I by renin S conversion of angiotensin I to 4. The actions of ANP include vasodilation of the
angiotensin II by angiotensin converting enzyme afferent and efferent arterioles, which results in an
S angiotensin II stimulates release of ADH and increase in renal blood flow and GFR. ANP inhibits
aldosterone S sodium and water retention. aldosterone secretion by the adrenal gland and
Activity D sodium reabsorption from the collecting tubules
through its action on aldosterone and through
1. The glomerular capillary membrane is composed of direct action on the tubular cells. It also inhibits
three layers: the capillary endothelial layer, the base- ADH release from the posterior pituitary gland,
ment membrane, and the single-celled capsular thereby increasing excretion of water by the
epithelial layer. The endothelial layer contains many kidneys. ANP also has vasodilator properties.
small perforations, called fenestrations. The epithe- 5. The kidneys function as an endocrine organ in that
lial layer that covers the glomerulus is continuous they produce chemical mediators that travel through
with the epithelium that lines Bowmans capsule. the blood to distant sites where they exert their
The cells of the epithelial layer have unusual actions. The kidneys participate in control of
octopus-like structures that possess a large number blood pressure by way of the renin-angiotensin
of extensions, or foot processes. These foot processes mechanism, in calcium metabolism by activating
form slit pores through which the glomerular filtrate vitamin D, and in regulating red blood cell produc-
passes. The basement membrane consists of a homo- tion through the synthesis of erythropoietin.
geneous acellular meshwork of collagen fibers, glyco- 6. By blocking the reabsorption of these solutes,
proteins, and mucopolysaccharides. The spaces diuretics create an osmotic pressure gradient within
between the fibers that make up the basement mem- the nephron that prevents the passive reabsorption
brane represent the pores of a filter and determine of water. Thus, diuretics cause water and sodium to
the size-dependent permeability barrier of the be retained within the nephron, thereby promoting
glomerulus. the excretion of both. The increase in urine flow
2. The basic mechanisms of transport across the tubu- that a diuretic produces is related to the amount of
lar epithelial cell membrane include active and pas- sodium and chloride reabsorption that it blocks.

Copyright 2011. Wolters Kluwer Health | Lippincott Williams & Wilkins. Study Guide for Porths Essentials of Pathophysiology, Third Edition.
LWBK707-Ans_p280-402.qxd 8/20/10 6:38PM Page 342 Aptara Inc

342 ANSWER KEY

7. Answer: b
SECTION III: APPLYING YOUR KNOWLEDGE RATIONALE: The increase in urine flow that a
Activity E diuretic produces is related to the amount of
1. Tests that the nurse would expect to be ordered to sodium and chloride reabsorption that it blocks.
either confirm or deny the diagnosis include urine The other answers are not correct.
specific gravity, urinalysis with culture and sensitiv- 8. Answer: b
ity, urine osmolality, GFR, BUN, and serum RATIONALE: With diminished renal function, there
electrolytes. is a loss of renal concentrating ability, and the
2. A simple flat-plat radiograph will show the kidneys, urine specific gravity may fall to levels of 1.006 to
ureters, and any radio-opaque stones that may be 1.010 (usual range is 1.010 to 1.025 with normal
in the kidney pelvis or ureters. fluid intake). These low levels are particularly sig-
nificant if they occur during periods that follow a
decrease in water intake (e.g., during the first urine
SECTION IV: PRACTICING FOR NCLEX
specimen on arising in the morning). The other
Activity F answers are incorrect.
1. Answer: a 9. Answer: d
RATIONALE: The plasma level at which the RATIONALE: Creatinine is freely filtered in the
substance appears in the urine is called the renal glomeruli, is not reabsorbed from the tubules into
threshold. Renal clearance, renal filtration rate, the blood, and is only minimally secreted into the
and renal transport levels are not the right tubules from the blood; therefore, its blood values
answers. depend closely on the GFR. A normal serum creati-
2. Answer: b nine level usually indicates normal renal function.
RATIONALE: With ingestion of a high-protein diet, In addition to its use in calculating the GFR, the
renal blood flow increases 20% to 30% within 1 to serum creatinine level is used in estimating the
2 hours. Although the exact mechanism for this functional capacity of the kidneys. If the value
increase is uncertain, it is thought to be related to doubles, the GFRand renal functionprobably
the fact that amino acids and sodium are absorbed has fallen to half of its normal state. A rise in the
together in the proximal tubule (secondary active serum creatinine level to three times its normal
transport). The same mechanism is thought to value suggests that there is a 75% loss of renal
explain the large increases in renal blood flow and function. A BUN, 24-hour urine test, and urine test
GFR that occur with high blood glucose levels in of first void in the morning do not tell you about
persons with uncontrolled diabetes mellitus. serum creatinine levels.
3. Answers: a, b 10. Answer: a
RATIONALE: With inulin, after intravenous RATIONALE: The actions of ANP include
injection, the amount that appears in the urine is vasodilation of the afferent and efferent arterioles,
equal to the amount that is filtered in the which results in an increase in renal blood flow
glomeruli (i.e., the clearance rate is equal to the and GFR. ANP inhibits aldosterone secretion by
GFR). Because of these properties, inulin can be the adrenal gland and sodium reabsorption from
used as a laboratory measure of the GFR. The other the collecting tubules through its action on aldos-
answers are not correct. terone and through direct action on the tubular
4. Answer: c cells. It also inhibits ADH release from the
RATIONALE: Small doses of aspirin compete with posterior pituitary gland, thereby increasing excre-
uric acid for secretion into the tubular fluid and tion of water by the kidneys. ANP also has
reduce uric acid secretion, and large doses compete vasodilator properties.
with uric acid for reabsorption and increase uric
acid excretion in the urine.
5. Answer: d
CHAPTER 25
RATIONALE: Alkaline or acid diuresis may be used to
increase elimination of drugs in the urine, particu- SECTION II: ASSESSING YOUR
larly in situations of drug overdose. The other UNDERSTANDING
answers are incorrect. Activity A
6. Answer: a
1. shape, position
RATIONALE: Persons with end-stage kidney disease
2. agenesis
often are anemic because of an inability of the kid-
3. Potter syndrome
neys to produce erythropoietin. This anemia usu-
4. hypoplasia
ally is managed by the administration of a
5. dysplasia
recombinant erythropoietin (epoetin alfa),
6. multicystic
produced through DNA technology, to stimulate
7. Polycystic
erythropoiesis.

Copyright 2011. Wolters Kluwer Health | Lippincott Williams & Wilkins. Study Guide for Porths Essentials of Pathophysiology, Third Edition.
LWBK707-Ans_p280-402.qxd 8/20/10 6:38PM Page 343 Aptara Inc

ANSWER KEY 343

8. autosomal dominant Activity C


9. destructive
1. c 2. a 3. f 4. g 5. e
10. Stagnation
6. i 7. d 8. h 9. b 10. j
11. Hydronephrosis
12. distention Activity D
13. calculi
14. nidus
15. calcium
16. pain
17. second
18. Escherichia coli
19. urethra
20. more
21. anatomic, functional Glomerular damage
22. Catheter
23. cystitis
24. Glomerulonephritis Increased permeability to proteins
25. nephritic, nephrotic Proteinuria ( 3.5 g/24 h)
26. Acute nephritic
27. deposition
28. hypercellularity Hypoproteinemia
29. Goodpasture
30. Nephrotic
31. Membranous Decreased plasma Compensatory synthesis
32. Berger disease oncotic pressure of proteins by liver
33. basement membrane
34. Renal tubular acidosis Edema Hyperlipidemia
35. HCO3
36. Acute pyelonephritis
37. Wilms tumor
38. hematuria, mass
Activity B Activity E
1. The destructive effects of urinary obstruction on
kidney structures are determined by the degree (i.e.,
partial versus complete, unilateral versus bilateral)
and the duration of the obstruction. The two most
damaging effects of urinary obstruction are stasis of
urine, which predisposes to infection and stone for-
mation, and progressive dilation of the renal
collecting ducts and renal tubular structures, which
causes destruction and atrophy of renal tissue.
2. Kidney stone formation requires supersaturated
Kidney urine and an environment that allows the stone to
stone
grow. The risk for stone formation is increased
Pregnancy when the urine is supersaturated with stone com-
or tumor ponents (e.g., calcium salts, uric acid, magnesium
Scar ammonium phosphate, cystine). Supersaturation
tissue depends on urinary pH, solute concentration,
Uretero-
vesical ionic strength, and complexation. The greater the
junction concentration of two ions, the more likely they are
stricture to precipitate. Complexation influences the avail-
ability of specific ions.
Neurogenic 3. The risk factors for UTI are higher:
bladder a. in persons with urinary obstruction and reflux
Bladder b. in people with neurogenic disorders that impair
outflow bladder emptying
obstruction

Copyright 2011. Wolters Kluwer Health | Lippincott Williams & Wilkins. Study Guide for Porths Essentials of Pathophysiology, Third Edition.
LWBK707-Ans_p280-402.qxd 8/20/10 6:38PM Page 344 Aptara Inc

344 ANSWER KEY

c. in women who are sexually active number of toxic metabolites. Drugs and toxic
d. in postmenopausal women substances can damage the kidneys by causing a
e. in men with diseases of the prostate decrease in renal blood flow, obstructing urine
f. in elderly persons. flow, directly damaging tubulointerstitial
g. in those who have undergone catheterization structures, or producing hypersensitivity
h. in women with diabetes reactions.
4. The host defenses of the bladder include the
washout phenomenon, in which bacteria are SECTION III: APPLYING YOUR KNOWLEDGE
removed from the bladder and urethra during
Activity F
voiding; the protective mucin layer that lines the
bladder and protects against bacterial invasion; 1. Urine analysis, urine culture and sensitivity, and
and local immune responses. In the ureters, broad-spectrum antibiotic given intravenously.
peristaltic movements facilitate the movement
of urine from the renal pelvis through the ureters SECTION IV: PRACTICING FOR NCLEX
and into the bladder. Immune mechanisms, Activity G
particularly secretory immunoglobulin (Ig) A,
appear to provide an important antibacterial 1. Answers: a, b; c, e
defense. Phagocytic blood cells further assist in RATIONALE: Bilateral renal dysplasia causes
the removal of bacteria from the urinary tract. In oligohydramnios and the resultant Potter facies,
women, the normal flora of the periurethral area, pulmonary hypoplasia, and renal failure. Multicystic
which consists of organisms such as lactobacillus, kidneys are a disorder, not the result of a congenital
provides defense against the colonization of problem.
uropathic bacteria. In men, the prostatic fluid has 2. Answers: 1-b, 2-a, 3-d, 4-c
antimicrobial properties that protect the urethra 3. Answer: a
from colonization. RATIONALE: Urinary tract obstruction encourages
5. The cellular changes that occur with glomerular the growth of microorganisms and should be sus-
disease include increases in glomerular and/or pected in persons with recurrent UTIs. The other
inflammatory cell number, basement membrane answers can cause lower UTIs, but an obstruction
thickening, and changes in noncellular glomerular would be considered because of the frequency of
components. the infections.
6. The development of glomerulonephritis follows 4. Answer: b
a streptococcal infection by approximately 7 to RATIONALE: Phosphate levels are increased in alka-
12 days, the time needed for the production of line urine and magnesium, always present in the
antibodies. The primary infection usually involves urine, and combine to form struvite stones. These
the pharynx. Oliguria, which develops as the GFR stones can increase in size until they fill an entire
decreases, is one of the first symptoms. Proteinuria renal pelvis. Because of their shape, they often are
and hematuria follow because of increased called staghorn stones. The other minerals can
glomerular capillary wall permeability. The red form stones, but not staghorn stones.
blood cells are degraded by materials in the urine, 5. Answer: c
and cola-colored urine may be the first sign of the RATIONALE: Most uncomplicated lower UTIs are
disorder. Sodium and water retention gives rise to caused by Escherichia coli. The other organisms can
edema (particularly of the face and hands) and cause UTIs, but are not the most common cause of
hypertension. infection.
7. Widespread thickening of the glomerular capillary 6. Answers: b, c, d
basement membrane occurs in almost all persons RATIONALE: Toddlers often present with abdominal
with diabetes and can occur without evidence of pain, vomiting, diarrhea, abnormal voiding
proteinuria. This is followed by a diffuse increase patterns, foul-smelling urine, fever, and poor
in mesangial matrix, with mild proliferation of growth. Toddlers do not typically have frequency
mesangial cells. As the disease progresses, the in voiding, nor do they complain of burning when
mesangial cells impinge on the capillary lumen, they urinate.
reducing the surface area for glomerular filtration. 7. Answer: d
8. Drug-related nephropathies involve functional or RATIONALE: Group A
-hemolytic streptococci have
structural changes in the kidneys that occur after the ability to seed from one area of the body to
exposure to a drug. Because of their large blood another. One area it seeds to is the kidney, where
flow and high filtration pressure, the kidneys are it causes acute postinfectious glomerulonephritis.
exposed to any substance that is in the blood. The Other organisms can cause acute postinfectious
kidneys also are active in the metabolic transfor- glomerulonephritis but they are not the most
mation of drugs and therefore are exposed to a common cause of the disease.

Copyright 2011. Wolters Kluwer Health | Lippincott Williams & Wilkins. Study Guide for Porths Essentials of Pathophysiology, Third Edition.
LWBK707-Ans_p280-402.qxd 8/20/10 6:38PM Page 345 Aptara Inc

ANSWER KEY 345

8. Answer: a Activity B
RATIONALE: The lesions of diabetic nephropathy
most commonly involve the glomeruli and are
associated with three glomerular syndromes:
Prerenal
nonnephrotic proteinuria, nephrotic syndrome, Intrinsic (marked decrease
and chronic renal failure. The other answers are (damage to in renal blood flow)
not commonly associated with diabetic structures
nephropathy. within the
9. Answer: b kidney)
RATIONALE: The most common causative agents of
acute pyelonephritis are Gram-negative bacteria,
including E. coli and Proteus, Klebsiella, Enterobac-
ter, and Pseudomonas. The other answers are not
considered a common causative agent of acute
pyelonephritis. Postrenal
10. Answer: c (obstruction of
RATIONALE: The tolerance to drugs varies with age urine outflow
and depends on renal function, state of hydration, from the kidney)
blood pressure, and the pH of the urine. None of
the other answers are correct.
11. Answer: d
RATIONALE: The common presenting signs of a
Wilms tumor are a large asymptomatic abdominal Activity C
mass and hypertension. The tumor is often discov-
ered inadvertently, and it is not uncommon for 1. b 2. j 3. d 4. g 5. a
the mother to discover it while bathing the child. 6. e 7. h 8. i 9. f 10. c
Some children may present with abdominal pain, Activity D
vomiting, or both. Hypotension, oliguria, and 1. Acute tubular necrosis (ATN) is characterized by the
diarrhea are not common presenting signs of a destruction of tubular epithelial cells with acute
Wilms tumor. suppression of renal function. ATN can be caused
by a variety of conditions, including acute tubular
damage due to ischemia, sepsis, nephrotoxic effects
CHAPTER 26 of drugs, tubular obstruction, and toxins from a
massive infection. Tubular epithelial cells are partic-
SECTION II: ASSESSING YOUR ularly sensitive to ischemia and are vulnerable to
UNDERSTANDING toxins. The tubular injury that occurs in ATN
Activity A frequently is reversible.
2. The onset or initiating phase, which lasts hours or
1. Acute renal failure days, is the time from the onset of the precipitating
2. prerenal, intrinsic, postrenal event until tubular injury occurs. The maintenance
3. Prerenal phase of ATN is characterized by a marked decrease
4. tubular epithelial in the glomerular filtration rate (GFR), causing
5. blood urea nitrogen sudden retention of endogenous metabolites,
6. Postrenal such as urea, potassium, sulfate, and creatinine
7. cause that normally are cleared by the kidneys. Fluid
8. chronic kidney disease retention gives rise to edema, water intoxication,
9. 120 to 130 and pulmonary congestion. If the period of
10. creatinine oliguria is prolonged, hypertension frequently
11. tubulointerstitial, albumin develops and, with it, signs of uremia. The
12. uremic recovery phase is the period during which repair
13. dehydration, overload of renal tissue takes place. Its onset usually is her-
14. sodium alded by a gradual increase in urine output and a
15. bone fall in serum creatinine, indicating that the
16. osteodystrophy nephrons have recovered to the point at which
17. Hypertension urine excretion is possible.
18. uremia
19. atrophy, demyelination
20. GFR

Copyright 2011. Wolters Kluwer Health | Lippincott Williams & Wilkins. Study Guide for Porths Essentials of Pathophysiology, Third Edition.
LWBK707-Ans_p280-402.qxd 8/20/10 6:38PM Page 346 Aptara Inc

346 ANSWER KEY

3. GFR is used to classify chronic kidney disease into growth and developmental delays; medication
five stages, beginning with kidney damage with regimen, including side effects; and dietary restric-
normal or elevated GFR, progressing to chronic kid- tions including protein, caloric, sodium, and fluid
ney disease and, potentially, to kidney failure. restrictions.
4. As kidney structures are destroyed, the remaining 2. Chronic kidney disease is a progressive disorder
nephrons undergo structural and functional hyper- that can be slowed by adherence to dietary restric-
trophy, each increasing its function as a means of tions and medication regimen. The disorder usually
compensating for those that have been lost. In the progresses to the point where the child needs either
process, each of the remaining nephrons must filter hemodialysis or peritoneal dialysis or a kidney
more solute particles from the blood. It is only when transplant. All forms of renal replacement therapy
the few remaining nephrons are destroyed that the are considered safe in the pediatric population, and
manifestations of kidney failure become evident. renal transplantation is considered the best
5. The manifestations of CKD include an treatment for a child.
accumulation of nitrogenous wastes; alterations in
water, electrolyte, and acid-base balance; mineral SECTION IV: PRACTICING FOR NCLEX
and skeletal disorders; anemia and coagulation dis-
Activity F
orders; hypertension and alterations in cardiovascu-
lar function; gastrointestinal disorders; neurologic 1. Answer: a
complications; disorders of skin integrity; and dis- RATIONALE: The most common indicator of acute
orders of immunologic function. The point at renal failure is azotemia, an accumulation of
which these disorders make their appearance and nitrogenous wastes (urea nitrogen, uric acid, and
the severity of the manifestations are determined creatinine) in the blood and a decrease in the GFR.
largely by the extent of renal function that is pres- The other answers are not common indicators of
ent and the coexisting disease conditions. acute renal failure.
6. The anemia of CKD is due to several factors includ- 2. Answers: a, c, d
ing chronic blood loss, hemolysis, bone marrow RATIONALE: Ischemic ATN occurs most frequently
suppression due to retained uremic factors, and in persons who have major surgery, severe
decreased red cell production due to impaired pro- hypovolemia, overwhelming sepsis, trauma, and
duction of erythropoietin and iron deficiency. The burns. Hypervolemia and hypertension are not
kidneys are the primary site for the production of the considered contributing factors to ischemic ATN.
hormone erythropoietin, which controls red blood 3. Answer: b
cell production. In renal failure, erythropoietin pro- RATIONALE: In clinical practice, GFR is usually esti-
duction usually is insufficient to stimulate adequate mated using the serum creatinine concentration.
red blood cell production by the bone marrow. The other answers are not used to estimate the GFR.
7. People with CKD tend to have an increased 4. Answer: c
prevalence of left ventricular dysfunction, with both RATIONALE: The number one hematologic disorder
depressed left ventricular ejection fraction, as in sys- that accompanies CKD is anemia. The other
tolic dysfunction, and impaired ventricular filling, answers are incorrect.
as in diastolic failure. Multiple factors lead to devel- 5. Answers: a, b, c
opment of left ventricular dysfunction, including RATIONALE: Uremic pericarditis resembles viral peri-
extracellular fluid overload, shunting of blood carditis in its presentation. This includes all poten-
through an arteriovenous fistula for dialysis, and tial complications, up to and including cardiac
anemia. Coupled with the hypertension that often tamponade. The presenting signs include mild to
is present, they cause increased myocardial work severe chest pain with respiratory accentuation
and oxygen demand, with eventual development and a pericardial friction rub. Fever is variable in
of heart failure. Congestive heart failure and the absence of infection and is more common in
pulmonary edema tend to occur in the late stages of dialysis than uremic pericarditis. Shortness of
kidney failure. Coexisting conditions that have breath and thromboangiitis are not indicative of
been identified as contributing to the burden of car- uremic pericarditis.
diovascular disease include hypertension, anemia, 6. Answer: d
diabetes mellitus, dyslipidemia, and coagulopathies. RATIONALE: Restless legs syndrome is a manifestation
Anemia, in particular, has been correlated with the of peripheral nerve involvement and can be seen
presence of left ventricular hypertrophy. in as many as two-thirds of patients on dialysis.
The other answers are not correct.
SECTION III: APPLYING YOUR KNOWLEDGE 7. Answer: a
RATIONALE: Many persons with CKD fail to mount
Activity E a fever with infection, making the diagnosis more
1. Description of the disease process; prognosis; man- difficult. All of the other answers occur.
ifestations of the disease, including physical

Copyright 2011. Wolters Kluwer Health | Lippincott Williams & Wilkins. Study Guide for Porths Essentials of Pathophysiology, Third Edition.
LWBK707-Ans_p280-402.qxd 8/20/10 6:38PM Page 347 Aptara Inc

ANSWER KEY 347

8. Answers: a, b, c Activity B
RATIONALE: The cause of sexual dysfunction in
men and women with CKD is unclear. The cause Epithelium when Epithelium when
bladder is empty bladder is full
probably is multifactorial and may result from
high levels of uremic toxins, neuropathy, altered
endocrine function, psychological factors,
and medications (e.g., antihypertensive drugs).
The other answers do not apply in this
situation.
Detrusor
9. Answer: b muscle
RATIONALE: Access to the vascular system is accom-
plished through an external arteriovenous shunt Ureters
(i.e., tubing implanted into an artery and a vein)
or, more commonly, through an internal arteriove-
nous fistula (i.e., anastomosis of a vein to an
artery, usually in the forearm). The other answers
are incorrect. Trigone
10. Answer: c
RATIONALE: At least 50% of the protein intake for Internal sphincter
clients with CKD should consist of proteins of
high biologic value, such as those in eggs, lean External sphincter
meat, and milk, which are rich in essential amino
acids. The other sources of protein contribute to
high levels of nitrogen. Activity C
1. i 2. f 3. a 4. d 5. j
CHAPTER 27 6. g 7. b 8. e 9. h 10. c
Activity D
SECTION II: ASSESSING YOUR 1. The bladder is composed of four layers. The first is
UNDERSTANDING an outer serosal layer, which covers the upper sur-
Activity A face and is continuous with the peritoneum; the
second is a network of smooth muscle fibers called
1. bladder the detrusor muscle; the third is a submucosal layer
2. retroperitoneally, symphysis of loose connective tissue; and the fourth is an
3. prostate inner mucosal lining of transitional epithelium.
4. ureters 2. The pelvic nerve carries sensory fibers from
5. epithelial lining the stretch receptors in the bladder wall; the puden-
6. external sphincter dal nerve carries sensory fibers from the external
7. parasympathetic, sympathetic sphincter and pelvic muscles; and the hypogastric
8. sacral, pelvic nerve nerve carries sensory fibers from the trigone area.
9. pons 3. As bladder filling occurs, ascending spinal afferents
10. Cortical relay this information to the micturition center,
11.
2-adrenergic which also receives important descending informa-
12. 1 receptors tion from the forebrain concerning behavioral cues
13. obstruction, incontinence for bladder emptying and urine storage. Descending
14. prostate gland pathways from the pontine micturition center pro-
15. store, empty duce coordinated inhibition or relaxation of the
16. micturition reflex external sphincter. Cortical brain centers enable
17. stroke inhibition of the micturition center in the pons and
18. Atony conscious control of urination. Neural influences
19. Stress incontinence from the subcortical centers in the basal ganglia
20. neurogenic, myogenic modulate the contractile response. They modify and
21. transitional delay the detrusor contractile response during filling
22. hematuria

Copyright 2011. Wolters Kluwer Health | Lippincott Williams & Wilkins. Study Guide for Porths Essentials of Pathophysiology, Third Edition.
LWBK707-Ans_p280-402.qxd 8/20/10 6:38PM Page 348 Aptara Inc

348 ANSWER KEY

and then modulate the expulsive activity of the neling of the bladder neck, and backward and
bladder to facilitate complete emptying. downward rotation of the bladder occur, so that the
4. The detrusor muscle of the bladder fundus and bladder and urethra are already in an anatomic
bladder neck contract down on the urine; the position for the first stage of voiding. Any activity
ureteral orifices are forced shut; the bladder neck is that causes downward pressure on the bladder is
widened and shortened as it is pulled up by the sufficient to allow the urine to escape involuntarily.
globular muscles in the bladder fundus; the resist- 8. The neurogenic theory for overactive bladder pos-
ance of the internal sphincter in the bladder neck is tulates that the CNS functions as an on-off switch-
decreased; and the external sphincter relaxes as ing circuit for voluntary control of bladder
urine moves out of the bladder. function. Therefore, damage to the CNS inhibitory
5. The necessary factors that every child must possess pathways may trigger bladder overactivity owing to
in order to attain conscious control of bladder func- uncontrolled voiding reflexes. Neurogenic causes of
tion are (1) normal bladder growth, (2) myelination overactive bladder include stroke, Parkinson
of the ascending afferents that signal awareness of disease, and multiple sclerosis.
bladder filling, (3) development of cortical control 9. The overall capacity of the bladder is reduced, as is
and descending communication with the sacral the urethral closing pressure. Detrusor muscle func-
micturition center, (4) ability to consciously tighten tion also tends to decline with aging; thus, there is
the external sphincter to prevent incontinence, (5) a trend toward a reduction in the strength of blad-
and motivation of the child to stay dry. der contraction and impairment in emptying that
6. During the early stage of obstruction, the bladder leads to larger postvoid residual volumes.
begins to hypertrophy and becomes hypersensitive
to afferent stimuli arising from stretch receptors in SECTION III: APPLYING YOUR KNOWLEDGE
the bladder wall. The ability to suppress urination
is diminished, and bladder contraction can become
Activity E
so strong that it virtually produces bladder spasm. 1. In people who have multiple sclerosis, the demyeli-
There is further hypertrophy of the bladder muscle, nation of the nerves can cause an interruption in
the thickness of the bladder wall may double, the messages from the brain and the spinal cord in
and the pressure generated by detrusor contraction reaching the bladder. This causes a condition
will increase to overcome the resistance from the known as a neurogenic bladder.
obstruction. As the force needed to expel urine from 2. The nurse would expect the client to be given
the bladder increases, compensatory mechanisms an antimuscarinic drug, such as oxybutynin,
may become ineffective, causing muscle fatigue tolterodine, or propantheline, to decrease detrusor
before complete emptying can be accomplished. muscle tone and increase bladder capacity.
The inner smooth surface of the bladder is replaced
with coarsely woven structures called trabeculae. SECTION IV: PRACTICING FOR NCLEX
Small pockets of mucosal tissue commonly develop
Activity F
between the trabecular ridges. These pockets form
diverticula, making the patient more susceptible to 1. Answers: a, c, e
secondary infections. Along with hypertrophy of RATIONALE: Disruption of pontine control of
the bladder wall, there is hypertrophy of the trigone micturition, as in spinal cord injury, results in
area and the interureteric ridge, which is located uninhibited spinal reflex-controlled contraction of
between the two ureters. This causes backpressure the bladder without relaxation of the external
on the ureters, the development of hydroureters sphincter, a condition known as detrusor-sphincter
and eventually, kidney damage. dyssynergia. The other answers are not true.
7. The angle between the bladder and the posterior 2. Answer: a
proximal urethra normally is 90 to 100 degrees, RATIONALE: As the child grows, the bladder gradu-
with at least one-third of the bladder base ally enlarges, with an increase in capacity, in
contributing to the angle when not voiding. ounces, that approximates the age of the child
During the first stage of voiding, this angle is lost as plus 2. The other answers are not true.
the bladder descends. In women, diminution of 3. Answer: b
muscle tone associated with childbirth can cause RATIONALE: Sphincter EMG allows the activity of the
weakness of the pelvic floor muscles and result in striated (voluntary) muscles of the perineal area to
stress incontinence by obliterating the critical pos- be studied. Cystometry measures the ability of the
terior urethrovesical angle. In these women, loss of bladder to store urine as well as the pressure of the
the posterior urethrovesical angle, descent and fun- bladder during filling and emptying. Uroflowmetry
measures the flow rate during urination.

Copyright 2011. Wolters Kluwer Health | Lippincott Williams & Wilkins. Study Guide for Porths Essentials of Pathophysiology, Third Edition.
LWBK707-Ans_p280-402.qxd 8/20/10 6:38PM Page 349 Aptara Inc

ANSWER KEY 349

4. Answer: b significant reduction in the rate of relapse and pro-


RATIONALE: During the early stage of obstruction, longs relapse-free interval in persons with cancer
the bladder begins to hypertrophy and becomes in situ. The other drugs are used to treat bladder
hypersensitive to afferent stimuli arising from cancer, but not cancer in situ.
stretch receptors in the bladder wall. The ability
to suppress urination is diminished, and bladder
contraction can become so strong that it virtually CHAPTER 28
produces bladder spasm. There is urgency, some-
times to the point of incontinence, and frequency SECTION II: ASSESSING YOUR
during the day and at night. The other answers UNDERSTANDING
are wrong.
Activity A
5. Answer: c
RATIONALE: The most common causes of spastic 1. gastrointestinal (GI) system
bladder dysfunction are spinal cord lesions such as 2. pharyngoesophageal
spinal cord injury, herniated intervertebral disk, 3. gastroesophageal
vascular lesions, tumors, and myelitis. The other 4. stomach
answers are wrong. 5. duodenum, jejunum, ileum
6. Answer: d 6. jejunum
RATIONALE: With acute overdistention of the blad- 7. epithelial, mucus
der, usually no more than 1,000 mL of urine is 8. Serous
removed from the bladder at one time. The other 9. mesentery
answers are incorrect. 10. pacemaker
7. Answer: a 11. enteric
RATIONALE: In women, the angle between the 12. Mechanoreceptors, chemoreceptors
bladder and the posterior proximal urethra 13. vagovagal
(i.e., urethrovesical junction) is important to con- 14. oral, pharyngeal, esophageal
tinence. This angle normally is 90 to 100 degrees. 15. small intestine
The other answers are incorrect. 16. Defecation
8. Answers: b, c, e 17. hormones
RATIONALE: Among the transient causes of 18. gastrin
urinary incontinence are recurrent urinary 19. Ghrelin
tract infections, medications that alter bladder 20. Cholecystokinin
function or perception of bladder filling and 21. parietal, vitamin B12
the need to urinate, diuretics and conditions 22. pepsinogen
that increase bladder filling, stool impaction, 23. gastrin
restricted mobility, and confusional states. The 24. Brunner glands
other answers are not associated with transient 25. bacteria
urinary incontinence. 26. indigestible dietary residue
9. Answer: b 27. Digestion
RATIONALE: Habit training with regularly scheduled 28. Absorption
toiletingusually every 2 to 4 hoursoften is 29. enterocytes
effective. The other answers are incorrect. 30. brush border enzymes
10. Answer: c 31. lipase
RATIONALE: The intervesicular administration of 32. Anorexia
bacillus Calmette-Gurin (BCG) vaccine, made 33. Nausea
from a strain of Mycobacterium bovis that formerly 34. Vomiting
was used to protect against tuberculosis, causes a

Copyright 2011. Wolters Kluwer Health | Lippincott Williams & Wilkins. Study Guide for Porths Essentials of Pathophysiology, Third Edition.
LWBK707-Ans_p280-402.qxd 8/20/10 6:38PM Page 350 Aptara Inc

350 ANSWER KEY

Activity B
1.

Mesentery
Muscularis externa

Epithelium
Longitudinal Circular
muscle muscle Lamina propria Mucosa
Muscularis
mucosa

Serosa
(mesothelium)
Serosa
(connective
tissue)
Submucosa

2. Activity D
Enterocyte being extruded 1. The upper partthe mouth, esophagus, and stom-
from a villus achacts as an intake source and receptacle
through which food passes and in which initial
digestive processes take place. The middle
Enterocyte portionthe duodenum, jejunum, and ileumis
the place where most digestive and absorptive
processes occur. The lower segmentthe cecum,
colon, and rectumserves as a storage channel for
the efficient elimination of waste.
Vein 2. The emptying of the stomach is regulated by
Lacteal hormonal and neural mechanisms. The hor-
mones cholecystokinin and glucose-dependent
Artery insulinotropic polypeptide are thought to partly
control gastric emptying, which are released in
response to the pH and the osmolar and fatty acid
composition of the chyme. Afferent receptor fibers
Crypt of
Lieberkihn
synapse with the neurons in the intramural plexus or
trigger intrinsic reflexes by means of vagal or sympa-
thetic pathways that participate in extrinsic reflexes.
3. With segmentation waves, slow contractions of the
circular muscle layer occlude the lumen and drive
the contents forward and backward. Most of the
contractions that produce segmentation waves are
local events involving only 1 to 4 cm of intestine at
Activity C a time. They function mainly to mix the chyme
1. c 2. d 3. b 4. f 5. e with the digestive enzymes from the pancreas and
6. i 7. h 8. d 9. a 10. j to ensure adequate exposure of all parts of the

Copyright 2011. Wolters Kluwer Health | Lippincott Williams & Wilkins. Study Guide for Porths Essentials of Pathophysiology, Third Edition.
LWBK707-Ans_p280-402.qxd 8/20/10 6:38PM Page 351 Aptara Inc

ANSWER KEY 351

chyme to the mucosal surface of the intestine, chymotrypsin, carboxypeptidase, and elastase. The
where absorption takes place. Peristaltic move- pancreatic enzymes are secreted as precursor mole-
ments are rhythmic propulsive movements cules. Trypsinogen, which lacks enzymatic activity,
designed to propel the chyme along the small is activated by an enzyme located on the brush bor-
intestine toward the large intestine. der cells of the duodenal enterocytes. Activated
4. The incretin effect is the increase in insulin release trypsin activates additional trypsinogen molecules
after an oral glucose load. The two hormones that and other pancreatic precursor proteolytic enzymes.
account for about 90% of the incretin effect are GLP- The amino acids are liberated on the surface of the
1, which is released from L cells in the distal small mucosal surface of the intestine by brush border
bowel, and GIP, which is released by K cells in the enzymes that degrade proteins into peptides that
upper gut (mainly the jejunum). Because increased are one, two, or three amino acids long. Similar to
levels of GLP-1 and GIP can lower blood glucose glucose, many amino acids are transported across
levels by augmenting insulin release in a glucose- the mucosal membrane in a sodium-linked process
dependent manner (i.e., at low blood glucose levels that uses ATP as an energy source. Some amino
no further insulin is secreted, minimizing the risk of acids are absorbed by facilitated diffusion processes
hypoglycemia), these hormones have been targeted that do not require sodium.
as possible antidiabetic drugs. Moreover, GLP-1 can
exert other metabolically beneficial effects, including SECTION III: APPLYING YOUR KNOWLEDGE
suppression of glucagon release, slowing of gastric
emptying, augmenting of net glucose clearance, and
Activity E
decreasing appetite and body weight. 1. The gastrointestinal tract is the largest endocrine
5. The first function of saliva is protection and lubri- gland in the body. Many nerves make the GI tract
cation. Saliva is rich in mucus, which protects the work. The stomach begins digestion by kneading
oral mucosa and coats the food as it passes through and churning the food we eat. Food then progresses
the mouth, pharynx, and esophagus. The sublingual to the small intestine, where most of the food is
and buccal glands produce only mucus-type secre- digested and absorbed. Our food then goes into the
tions. The second function of saliva is its protective large intestine, where it is compacted into the feces
antimicrobial action. The saliva cleans the mouth that we expel from our bodies.
and contains the enzyme lysozyme, which has
an antibacterial action. Third, saliva contains SECTION IV: PRACTICING FOR NCLEX
ptyalin and amylase, which initiate the digestion of
Activity F
dietary starches.
6. The cellular mechanism for hydrochloric acid 1. Answer: a
(HCl) secretion by the parietal cells in the stomach RATIONALE: At the end of the pyloric channel, the
involves the hydrogen (H)/potassium (K) adeno- circular layer smooth muscle thickens to form the
sine triphosphatase (ATPase) transporter and pyloric sphincter. This muscle serves as a valve
chloride (Cl) channels located on their luminal that controls the rate of stomach emptying and
membrane. During the process of HCl secretion, prevents the regurgitation of intestinal contents
carbon dioxide (CO2) produced by aerobic metabo- back into the stomach. There is no cardiac sphinc-
lism combines with water (H2O), catalyzed by ter in the GI tract. The antrum is a portion of the
carbonic anhydrase, to form carbonic acid (H2CO3), stomach that is the wider, upper portion of the
which dissociates into H and bicarbonate (HCO3). pyloric region. The cardiac orifice is the opening
The H is secreted with Cl into the stomach, and between the esophagus and the stomach.
the HCO3 moves out of the cell and into blood 2. Answer: b
from the basolateral membrane. At the luminal side RATIONALE: It is in the jejunum and ileum that
of the membrane, H is secreted into the stomach food is digested and absorbed. The other answers
via the H-K ATPase transporter and chloride are incorrect.
follows H into the stomach by diffusing through 3. Answer: c
Cl channels in the luminal membrane. RATIONALE: No contraction can occur without an
7. Digestion of starch begins in the mouth with the action potential and an action potential cannot occur
action of amylase. Pancreatic secretions also contain unless the slow wave brings the membrane potential
an amylase. Amylase breaks down starch into several to threshold. The other answers are incorrect.
disaccharides, including maltose, isomaltose, and 4. Answer: d
-dextrins. The brush border enzymes convert the RATIONALE: The external sphincter is controlled by
disaccharides into monosaccharides that can be nerve fibers in the pudendal nerve, which is part
absorbed. of the somatic nervous system and therefore under
8. Protein digestion begins in the stomach with voluntary control. The other answers are incorrect.
the action of pepsin. Proteins are broken down 5. Answer: a
further by pancreatic enzymes, such as trypsin, RATIONALE: Ghrelin is a newly discovered peptide
hormone produced by endocrine cells in the

Copyright 2011. Wolters Kluwer Health | Lippincott Williams & Wilkins. Study Guide for Porths Essentials of Pathophysiology, Third Edition.
LWBK707-Ans_p280-402.qxd 8/20/10 6:38PM Page 352 Aptara Inc

352 ANSWER KEY

mucosal layer of the fundus of the stomach. It dis- 13. Acute gastritis
plays potent growth hormone-releasing activity 14. Chronic gastritis
and has a stimulatory effect on food intake and 15. autoantibodies
digestive function, while reducing energy expendi- 16. Peptic ulcer
ture. The isolation of this hormone has led to new 17. hemorrhage, obstruction
insights into the gut-brain regulation of growth 18. bleeding ulcers
hormone secretion and energy balance. The other 19. Histamine
hormones are secreted elsewhere in the GI tract. 20. stress ulcers
6. Answers: a, b, c, d 21. carcinoma
RATIONALE: Saliva has three functions. The first is 22. Irritable bowel syndrome
protection and lubrication. Saliva is rich in mucus, 23. Crohn, ulcerative colitis
which protects the oral mucosa and coats the food 24. Crohn
as it passes through the mouth, pharynx, and 25. colon, rectum
esophagus. The second function of saliva is its pro- 26. Nutritional
tective antimicrobial action. Third, saliva contains 27. Lieberkhn
ptyalin and amylase, which initiate the digestion 28. Cancer
of dietary starches. The other answer is incorrect. 29. bacterial enterocolitis
7. Answer: b 30. Diverticulosis
RATIONALE: The major metabolic function of colonic 31. Diverticulitis
microflora is the fermentation of undigestible 32. appendicitis
dietary residue and endogenous mucus produced 33. diarrhea
by the epithelial cells. The other answers are not 34. noninflammatory diarrhea
their main function. 35. Chronic
8. Answer: c 36. Inflammatory diarrhea
RATIONALE: Absorption is accomplished by active 37. Constipation
transport and diffusion. The other answers are 38. Fecal impaction
incorrect. 39. cephalocaudal
9. Answer: d 40. Paralytic
RATIONALE: A common cause of nausea is distention 41. serous membrane
of the duodenum or upper small intestinal tract. 42. gluten
The other answers are not associated with nausea. 43. Colonoscopy
10. Answer: d Activity B
RATIONALE: Serotonin is believed to be involved in
1.
the nausea and emesis associated with cancer
1. g 2. f 3. a 4. e 5. j
chemotherapy and radiation therapy. Serotonin
6. h 7. d 8. i 9. c 10. b
antagonists (e.g., granisetron and ondansetron) are
2.
effective in treating the nausea and vomiting asso-
1. c 2. g 3. a 4. b 5. e
ciated with these stimuli. The other answers are
6. j 7. d 8. f 9. i 10. h
incorrect.
Activity C
1. GERD is gastroesophageal reflux disease. It is thought
CHAPTER 29 to be associated with a weak or incompetent lower
esophageal sphincter that allows reflux to occur, the
SECTION II: ASSESSING YOUR irritant effects of the refluxate, and decreased clear-
UNDERSTANDING ance of the refluxed acid from the esophagus after
Activity A it has occurred. In most cases, reflux occurs during
transient relaxation of the esophagus. Gastric
1. esophagus distention and meals high in fat increase the
2. Congenital frequency of relaxation. Delayed gastric emptying
3. Dysphagia also may contribute to reflux by increasing gastric
4. Hiatal hernia volume and pressure with greater chance for reflux.
5. GERD Esophageal mucosal injury is related to the destruc-
6. asthma tive nature of the refluxate and the amount of time
7. Reflux esophagitis it is in contact with mucosa. Acidic gastric fluids
8. infant (pH 4.0) are particularly damaging.
9. alcohol, tobacco 2. Several factors contribute to the protection of the
10. impermeable gastric mucosa, including an impermeable epithelial
11. prostaglandins cell surface covering, mechanisms for the selective
12. Gastritis transport of hydrogen and bicarbonate ions, and the

Copyright 2011. Wolters Kluwer Health | Lippincott Williams & Wilkins. Study Guide for Porths Essentials of Pathophysiology, Third Edition.
LWBK707-Ans_p280-402.qxd 8/20/10 6:38PM Page 353 Aptara Inc

ANSWER KEY 353

characteristics of gastric mucus. The gastric epithe- in the normal intestinal flora of genetically suscep-
lial cells are connected by tight junctions that pre- tible individuals. Thus, as in many other auto-
vent acid penetration, and they are covered with an immune disorders, the pathogenesis of Crohn
impermeable hydrophobic lipid layer that prevents disease and ulcerative colitis involves a failure of
diffusion of ionized water-soluble molecules. The immune regulation, genetic predisposition, and an
secretion of hydrochloric acid by the parietal cells environmental trigger, especially microbial flora.
of the stomach is accompanied by secretion of 7. In a manner similar to the small intestine, bands of
bicarbonate ions (HCO3). For every hydrogen ion circular muscle constrict the large intestine. As the
(H) that is secreted, an HCO3 is produced, and as circular muscle contracts at each of these points
long as HCO3 production is equal to H secretion, (approximately every 2.5 cm), the lumen of the bowel
mucosal injury does not occur. Water-insoluble becomes constricted, so that it is almost occluded.
mucus forms a thin, stable gel that adheres to the gas- The combined contraction of the circular muscle and
tric mucosal surface and provides protection from the the lack of a continuous longitudinal muscle layer
proteolytic (protein-digesting) actions of pepsin. It cause the intestine to bulge outward into pouches
also forms an unstirred layer that traps bicarbonate, called haustra. Diverticula develop between the longi-
forming an alkaline interface between the luminal tudinal muscle bands of the haustra, in the area
contents of the stomach and its mucosal surface. The where the blood vessels pierce the circular muscle
water-soluble mucus is washed from the mucosal layer to bring blood to the mucosal layer. An increase
surface and mixes with the luminal contents; its in intraluminal pressure in the haustra provides the
viscid nature makes it a lubricant that prevents force for creating these herniations. The increase in
mechanical damage to the mucosal surface. pressure is thought to be related to the volume of the
3. A peptic ulcer can affect one or all layers of the colonic contents. The scantier the contents, the more
stomach or duodenum. The ulcer may penetrate vigorous are the contractions and the greater is the
only the mucosal surface, or it may extend into the pressure in the haustra.
smooth muscle layers. Occasionally, an ulcer pene- 8. The pathophysiology of constipation can be classi-
trates the outer wall of the stomach or duodenum. fied into three broad categories: normal-transit con-
Spontaneous remissions and exacerbations are com- stipation, slow-transit constipation, and disorders
mon. Healing of the muscularis layer involves of defecatory or rectal evacuation. Normal-transit
replacement with scar tissue; although the mucosal constipation (or functional constipation) is charac-
layers that cover the scarred muscle layer regenerate, terized by perceived difficulty in defecation and
the regeneration often is less than perfect, which usually responds to increased fluid and fiber intake.
contributes to repeated episodes of ulceration. Slow-transit constipation, which is characterized by
4. Chronic infection with H. pylori appears to serve infrequent bowel movements, is often caused by
as a cofactor in some types of gastric carcinomas. alterations in intestinal innervation. Hirschsprung
The bacterial infection causes gastritis, followed by disease is an extreme form of slow-transit constipa-
atrophy, intestinal metaplasia, and carcinoma. This tion in which the ganglion cells in the distal bowel
sequence of cellular events depends on both the are absent because of a defect that occurred during
presence of the bacterial proteins and the host embryonic development; the bowel narrows at the
immune response; the latter being influenced by area that lack ganglionic cells. Although most per-
the host genetic background. However, most people sons with this disorder present in infancy or early
with H. pylori infection will not develop gastric can- childhood, some with a relatively short segment of
cer, and not all H. pylori infections increase the risk involved colon do not have symptoms until later in
of gastric cancer, suggesting that other factors must life. Defecatory disorders are most commonly due
be involved. to dysfunction of the pelvic floor or anal sphincter.
5. The condition is believed to result from deregula- 9. The cause of colon cancer is unknown, but
tion of intestinal motor and sensory functions attention has focused on dietary fat intake, refined
modulated by the CNS. Irritable bowel disease is sugar intake, fiber intake, and the adequacy of such
characterized by persistent or recurrent symptoms protective micronutrients as vitamins A, C, and E
of abdominal pain, altered bowel function, and in the diet. It has been hypothesized that a high
varying complaints of flatulence, bloatedness, nau- level of fat in the diet increases the synthesis of bile
sea and anorexia, constipation or diarrhea, and anx- acids in the liver, which may be converted to
iety or depression. A hallmark of irritable bowel potential carcinogens by the bacterial flora in the
syndrome is abdominal pain that is relieved by defe- colon. Bacterial organisms in particular are
cation and associated with a change in consistency suspected of converting bile acids to carcinogens;
or frequency of stools. Abdominal pain usually is their proliferation is enhanced by a high dietary
intermittent, cramping, and in the lower abdomen. level of refined sugars. Dietary fiber is thought to
6. According to the currently accepted hypothesis, increase stool bulk and thereby dilute and remove
this normal state of homeostasis is disrupted in potential carcinogens. Refined diets often contain
inflammatory bowel disease leading to unregulated reduced amounts of vitamins A, C, and E, which
and exaggerated immune responses against bacteria may act as oxygen free radical scavengers.

Copyright 2011. Wolters Kluwer Health | Lippincott Williams & Wilkins. Study Guide for Porths Essentials of Pathophysiology, Third Edition.
LWBK707-Ans_p280-402.qxd 8/20/10 6:38PM Page 354 Aptara Inc

354 ANSWER KEY

studies can prove particularly useful as routine


SECTION III: APPLYING YOUR KNOWLEDGE screening tests for persons with atrophic gastritis
Activity D or gastric polyps. Computed tomography and endo-
1. The doctor wants to try giving you the chemother- scopic ultrasonography often are used to delineate
apy medicine to try to reduce the size of your the spread of a diagnosed stomach cancer. Papani-
tumor so the surgery will not be as extensive as it colaou smears are done on gastric secretions but not
would be if the surgery were done today. by the nurse. A lower gastrointestinal study would
2. Even though your cancer has already spread, be of no value in diagnosing this client. A techni-
removing the tumor in your esophagus will make cian does not do an endoscopic ultrasound.
you more comfortable and, hopefully, allow you to 7. Answer: b
live longer than you would without the surgery. RATIONALE: A hallmark of irritable bowel syndrome
is abdominal pain that is relieved by defecation
and associated with a change in consistency or fre-
SECTION IV: PRACTICING FOR NCLEX
quency of stools. Nausea, altered bowel function,
Activity E and diarrhea are also symptoms of irritable bowel
1. Answer: b syndrome but not combined with abdominal pain
RATIONALE: Esophageal acid clearance can be that is unrelieved by defecation. A bowel impaction
retarded in cases of severe erosive esophagitis is not a symptom of irritable bowel syndrome.
where gastroesophageal reflux and a large hiatal 8. Answer: c
hernia coexist. The other answers are incorrect. RATIONALE: A characteristic feature of Crohn
2. Answer: c disease is the sharply demarcated, granulomatous
RATIONALE: Tilting of the head to one side and lesions that are surrounded by normal-appearing
arching of the back may be noted in children mucosal tissue. When the lesions are multiple,
with severe reflux. Early satiety is another indica- they often are referred to as skip lesions because
tion of gastroesophageal reflux, but not coupled they are interspersed between what appear to be
with consolable crying. The other answers are normal segments of the bowel.
not correct. 9. Answer: d
3. Answers: a, c, e RATIONALE: Rotavirus infection typically begins after
RATIONALE: The stomach lining usually is imperme- an incubation period of less than 24 hours, with
able to the acid it secretes, a property that allows mild to moderate fever, and vomiting, followed by
the stomach to contain acid and pepsin without onset of frequent watery, stools. The fever and vom-
having its wall digested. Several factors contribute iting usually disappear on about the second day,
to the protection of the gastric mucosa, including but the diarrhea continues for 5 to 7 days. Dehydra-
an impermeable epithelial cell surface covering, tion may develop rapidly, particularly in infants.
mechanisms for the selective transport of hydrogen The other answers are incorrect.
and bicarbonate ions, and the characteristics of 10. Answer: a
gastric mucus. These mechanisms are collectively RATIONALE: One of the most common complaints
referred to as the gastric mucosal barrier. The other of diverticulitis is pain in the lower left quadrant,
answers are incorrect. accompanied by nausea and vomiting, tenderness
4. Answer: d in the lower left quadrant, a slight fever, and an
RATIONALE: Helicobacter pylori gastritis can be a elevated white blood cell count. Both B and D
chronic infection that can lead to gastric atrophy, describe a suspected appendicitis, and C describes
peptic ulcer, and is associated with increased risk symptoms of a peptic ulcer.
of gastric adenocarcinoma and low-grade B-cell 11. Answers: b, c, d
gastric lymphoma (mucosa-associated lymphoid RATIONALE: Noninflammatory diarrhea is associated
tissue [MALToma]). The other answers are with large-volume watery and nonbloody stools,
incorrect. periumbilical cramps, bloating, and nausea and/or
5. Answer: a vomiting. The other answers are incorrect.
RATIONALE: Diagnostic procedures for peptic ulcer 12. Answer: b
include history taking, laboratory tests, radiologic RATIONALE: One of the most important manifesta-
imaging, and endoscopic examination. The other tions of peritonitis is the translocation of extra-
answers are not expected orders for a suspected cellular fluid into the peritoneal cavity (through
peptic ulcer. weeping or serous fluid from the inflamed
6. Answers: a, c peritoneum) and into the bowel as a result of bowel
RATIONALE: Diagnosis of gastric cancer is obstruction. The other answers are incorrect.
accomplished by means of a variety of techniques, 13. Answer: c
including barium radiographic studies, endoscopic RATIONALE: The primary treatment of celiac disease
studies with biopsy, and cytologic studies (e.g., consists of removal of gluten and related proteins
Papanicolaou smear) of gastric secretions. Cytologic from the diet. No other answer is correct.

Copyright 2011. Wolters Kluwer Health | Lippincott Williams & Wilkins. Study Guide for Porths Essentials of Pathophysiology, Third Edition.
LWBK707-Ans_p280-402.qxd 8/20/10 6:38PM Page 355 Aptara Inc

ANSWER KEY 355

14. Answer: d 14. Phase 1 reactions


RATIONALE: To reduce the likelihood of false- 15. CYP or cytochrome P450 gene
positive tests, persons are instructed to avoid non- 16. Phase 2 reactions
steroidal anti-inflammatory drugs such as 17. toxic metabolites
ibuprofen and aspirin for 7 days prior to testing, to 18. Cholestatic
avoid vitamin C in excess of 250 mg from either 19. Hepatitis
supplements or citrus fruits for 3 before testing, 20. injection drug, blood transfusions, high-risk sexual
and to avoid red meats for 3 days before testing. behavior
The other answers are incorrect. 21. Autoimmune
22. Intrahepatic
23. Secondary
CHAPTER 30 24. fatty
25. Cirrhosis
SECTION II: ASSESSING YOUR 26. Portal hypertension
UNDERSTANDING 27. increased, dilatation
28. Ascites
Activity A
29. Spontaneous bacterial peritonitis
1. accessory 30. hepatorenal
2. artery, portal vein 31. encephalopathy
3. hepatic portal vein 32. liver cancer
4. bile 33. gallbladder
5. albumin 34. Cholecystokinin
6. cholesterol, bile 35. cholesterol, bilirubin
7. fat 36. cholecystitis
8. triglycerides 37. exocrine
9. emulsifying, micelles 38. pancreatitis
10. Cholestasis 39. Chronic pancreatitis
11. bile 40. cigarette smoking
12. Hemolytic
Activity B
13. Conjugation

Diaphragm

Liver

Gallbladder Spleen

Cystic duct Hepatic duct

Common
bile duct
Ampulla of Vater
Tail of the
pancreas
Sphincter of Oddi
Duodenum Pancreatic duct

Head of the pancreas

Activity C
1.
1. c 2. f 3. h 4. b 5. e
6. g 7. a 8. d 9. i 10. j
2.
1. d 2. c 3. e 4. a 5. b

Copyright 2011. Wolters Kluwer Health | Lippincott Williams & Wilkins. Study Guide for Porths Essentials of Pathophysiology, Third Edition.
LWBK707-Ans_p280-402.qxd 8/20/10 6:38PM Page 356 Aptara Inc

356 ANSWER KEY

Activity D
1.

Amino acids Glycerol Lactic acid

Gluconeogenesis

Glucose Glycogen

Triglycerides

Bloodstream

2.

Portal hypertension

Increased pressure in Portosystemic


peritoneal capillaries shunting of blood Splenomegaly

Ascites Development of Shunting of ammonia Anemia Leukopenia


collateral channels and toxins from the
intestine into the
general circulation
Caput Esophageal Thrombocytopenia
medusae varices
Hepatic
Hemorrhoids encephalopathy Bleeding

Activity E neoplasms, the effects begin with increased


pressure in the large bile ducts. Genetic disorders
1. The liver is one of the most versatile and active
involving the transport of bile into the canaliculi
organs in the body. It produces bile; metabolizes
also can result in cholestasis.
hormones and drugs; synthesizes proteins,
3. The four major causes of jaundice are excessive
glucose, and clotting factors; stores vitamins and
destruction of red blood cells, impaired uptake of
minerals; changes ammonia produced by
bilirubin by the liver cells, decreased conjugation
deamination of amino acids to urea; and converts
of bilirubin, and obstruction of bile flow in the
fatty acids to ketones. The liver degrades excess
canaliculi of the hepatic lobules or in the
nutrients and converts them into substances
intrahepatic or extrahepatic bile ducts. From an
essential to the body. In its capacity for metaboliz-
anatomic standpoint, jaundice can be categorized
ing drugs and hormones, the liver serves as an
as prehepatic, intrahepatic, and posthepatic.
excretory organ.
4. Elevated serum enzyme tests usually indicate
2. A number of mechanisms are implicated in the
liver injury earlier than other indicators of
pathogenesis of cholestasis. Primary biliary cirrho-
liver function. The key enzymes are alanine
sis and primary sclerosing cholangitis are caused
aminotransferase (ALT) and aspartate aminotrans-
by disorders of the small intrahepatic canaliculi and
ferase (AST), which are present in liver cells.
bile ducts. In the case of extrahepatic obstruction,
Alanine aminotransferase is liver specific, whereas
such as that caused by conditions such as cholelithi-
AST is derived from organs other than the liver. In
asis, common duct strictures, or obstructing

Copyright 2011. Wolters Kluwer Health | Lippincott Williams & Wilkins. Study Guide for Porths Essentials of Pathophysiology, Third Edition.
LWBK707-Ans_p280-402.qxd 8/20/10 6:38PM Page 357 Aptara Inc

ANSWER KEY 357

most cases of liver damage, there are parallel rises lead to the development of ascites. Diminished
in ALT and AST. The most dramatic rise is seen in blood volume (i.e., underfill theory) and excessive
cases of acute hepatocellular injury. blood volume (i.e., overfill theory) have been used
5. The clinical course of viral hepatitis involves a to explain the increased salt and water retention
number of syndromes, including asymptomatic by the kidney.
infection with only serologic evidence of disease, 10. With the gradual obstruction of venous blood
acute hepatitis, the carrier state without clinically flow in the liver, the pressure in the portal vein
apparent disease or with chronic hepatitis, chronic increases, and large collateral channels develop
hepatitis with or without progression to cirrhosis, between the portal and systemic veins that supply
with rapid onset of liver failure. Not all hepatotoxic the lower rectum. The dilation of the collaterals
viruses provoke each of the clinical syndromes. between the inferior and internal iliac veins may
6. The metabolic end products of alcohol metabolism give rise to hemorrhoids.
(e.g., acetaldehyde, free radicals) are responsible
for a variety of metabolic alterations that can SECTION III: APPLYING YOUR KNOWLEDGE
cause liver injury. Acetaldehyde, for example, has
multiple toxic effects on liver cells and liver func-
Activity F
tion. The metabolism of alcohol leads to chemical 1. Serum aminotransferase, liver biopsy, complete
attack on certain membranes of the liver. Acetalde- blood count, and complete metabolic panel.
hyde is known to impede the mitochondrial elec- 2. Interferons, nucleotide and nucleotide analog anti-
tron transport system, which is responsible for retroviral agents, and pegylated interferon alfa-2a.
oxidative metabolism and generation of ATP; as a
result, the hydrogen ions that are generated in the SECTION IV: PRACTICING FOR NCLEX
mitochondria are shunted into lipid synthesis and
Activity G
ketogenesis. Binding of acetaldehyde to other mol-
ecules impairs the detoxification of free radicals 1. Answer: a
and synthesis of proteins. Acetaldehyde also pro- RATIONALE: Kupffer cells are reticuloendothelial
motes collagen synthesis and fibrogenesis. cells that are capable of removing and phagocytiz-
7. Fatty liver is characterized by the accumulation of ing old and defective blood cells, bacteria, and
fat in hepatocytes, a condition called steatosis. The other foreign material from the portal blood as it
liver becomes yellow, enlarges owing to excessive flows through the sinusoid. Langerhans cells are
fat accumulation, and is characterized by inflam- stellate dendritic cells found mostly in the stratum
mation and necrosis of liver cells. Alcoholic hepa- spinosum of the epidermis. Epstein cells do not
titis is the intermediate stage between fatty changes exist. Davidoff cells are large granular epithelial
and cirrhosis. It often is seen after an abrupt cells found in intestinal glands.
increase in alcohol intake and is common in 2. Answer: b
spree drinkers. Alcoholic cirrhosis is the result of RATIONALE: The morphologic features of cholestasis
repeated bouts of drinking-related liver injury and depend on the underlying cause. Common to all
designates the onset of end-stage alcoholic liver dis- types of obstructive and hepatocellular cholestasis
ease. The gross appearance of the early cirrhotic is the accumulation of bile pigment in the liver.
liver is one of fine, uniform nodules on its surface. The other answers are incorrect.
8. Cirrhosis is characterized by diffuse fibrosis and 3. Answer: c
conversion of normal liver architecture into nod- RATIONALE: Usually, only a small amount of biliru-
ules containing proliferating hepatocytes encircled bin is found in the blood; the normal level of total
by fibrosis. The formation of nodules represents serum bilirubin is 0.1 to 1.2 mg/dL. The other
a balance between regenerative activity and con- answers are incorrect.
strictive scarring. The fibrous tissue that replaces 4. Answer: d
normally functioning liver tissue forms RATIONALE: Because of the greater activity of the
constrictive bands that disrupt flow in the vascular drug-metabolizing enzymes in the central zones of
channels and biliary duct systems of the liver. The the liver, these agents typically cause centrilobular
disruption of vascular channels predisposes to por- necrosis. The other answers are incorrect.
tal hypertension and its complications; 5. Answer: a
obstruction of biliary channels and exposure to RATIONALE: The earliest symptoms are unexplained
the destructive effects of bile stasis; and loss of pruritus or itching, weight loss, and fatigue, followed
liver cells, leading to liver failure. by dark urine and pale stools. The other answers are
9. An increase in capillary pressure due to portal not indicative of primary biliary cirrhosis.
hypertension and obstruction of venous flow 6. Answer: b
through the liver, salt and water retention by the RATIONALE: When the capacity of the liver to
kidney, and decreased colloidal osmotic pressure export triglyceride is saturated, excess fatty acids
due to impaired synthesis of albumin by the liver contribute to the formation of fatty liver.

Copyright 2011. Wolters Kluwer Health | Lippincott Williams & Wilkins. Study Guide for Porths Essentials of Pathophysiology, Third Edition.
LWBK707-Ans_p280-402.qxd 8/20/10 6:38PM Page 358 Aptara Inc

358 ANSWER KEY

7. Answer: c Activity C
RATIONALE: Because of the many limitations in
1. Hormones generally are thought of as chemical
sodium restriction, the use of diuretics has become
messengers that are transported in body fluids.
the mainstay of treatment for ascites. A paracente-
They are highly specialized organic molecules pro-
sis may be done if the diuretics do not correct the
duced by endocrine organs that exert their action
problem. A thoracentesis would never be done for
on specific target cells. Hormones do not initiate
ascites. DDAVP is given to decrease urine output,
reactions but function as modulators of cellular and
not increase it.
systemic responses. Most hormones are present in
8. Answer: d
body fluids at all times, but in greater or lesser
RATIONALE: Diagnostic methods include ultrasound,
amounts depending on the needs of the body.
CT scans, and MRI. Liver biopsy may be used to
2. Hormones are divided into three categories: (1)
confirm the diagnosis. The serum -fetoprotein
amines and amino acids; (2) peptides, polypeptides,
can be indicative of liver cancer but it is not
proteins, and glycoproteins; and (3) steroids. The
confirmatory. An endoscopy is of no value. An
first category, the amines, includes norepinephrine
ultrasound of the liver is not confirmatory for
and epinephrine, which are derived from a single
liver cancer.
amino acid, and the thyroid hormones, which are
9. Answer: a
derived from two iodinated tyrosine amino acid
RATIONALE: Gallbladder sludge (thickened gallblad-
residues. The second category, the peptides, polypep-
der mucoprotein with tiny trapped cholesterol
tides, proteins, and glycoproteins, can be as small as
crystals) is thought to be a precursor of gallstones.
only to contain three amino acids, and as large and
The other answers are incorrect.
complex to consist of approximately 200 amino
10. Answer: b
acids. The third category consists of the steroid hor-
RATIONALE: Serum amylase and lipase are the labo-
mones, which are derivatives of cholesterol.
ratory markers most commonly used to establish a
3. The response of a target cell to a hormone varies
diagnosis of acute pancreatitis. Cholesterol and
with the number of receptors present and with the
triglycerides are not used as laboratory markers for
affinity of these receptors for hormone binding.
acute pancreatitis.
The number of hormone receptors on a cell may be
11. Answer: c
altered for any of several reasons. Antibodies may
RATIONALE: In pancreatic cancer, the most
destroy or block the receptor proteins. Increased or
significant and reproducible environmental risk
decreased hormone levels often induce changes in
factor is cigarette smoking. The other answers
the activity of the genes that regulate receptor syn-
are incorrect.
thesis. For example, decreased hormone levels often
produce an increase in receptor numbers by means
CHAPTER 31 of a process called up-regulation; this increases the
sensitivity of the body to existing hormone levels.
SECTION II: ASSESSING YOUR Likewise, sustained levels of excess hormone often
bring about a decrease in receptor numbers by
UNDERSTANDING
down-regulation, producing a decrease in hormone
Activity A sensitivity.
1. hormones 4. The intracellular signal system is termed the second
2. nervous, immune messenger, and the hormone is considered the first
3. paracrine messenger. The most widely distributed second
4. autocrine messenger is cyclic adenosine monophosphate
5. free, bound (cAMP). Adenylate cyclase is functionally coupled
6. high-affinity receptors to various cell surface receptors by the regulatory
7. receptors actions of G proteins. The second major cell surface
8. Lipid-soluble receptor involves the binding of a hormone or neu-
9. hypothalamus rotransmitter to a surface receptor acts directly to
10. master gland open an ion channel in the cell membrane. The
11. metabolites, hormone levels influx of ions, then, serves as an intracellular signal
to convey the hormonal message to the interior of
Activity B
the cell.
1. c 2. a 3. e 4. d 5. b 5. Hormones produced by the anterior pituitary
6. f 7. g control body growth and metabolism (growth
hormone, GH), function of the thyroid gland
(thyrotropin, TSH), glucocorticoid hormone levels
(corticotropin, ACTH), function of the gonads

Copyright 2011. Wolters Kluwer Health | Lippincott Williams & Wilkins. Study Guide for Porths Essentials of Pathophysiology, Third Edition.
LWBK707-Ans_p280-402.qxd 8/20/10 6:38PM Page 359 Aptara Inc

ANSWER KEY 359

(follicle-stimulating hormone, FSH, and luteinizing These two structures are connected by blood flow
hormone, LH), and breast growth and milk produc- in the hypophyseal portal system, which begins in
tion (prolactin). Melanocyte-stimulating hormone, the hypothalamus and drains into the anterior
which is involved in the control of pigmentation of pituitary gland, and by the nerve axons that con-
the skin, is produced by the pars intermedia of the nect the supraoptic and paraventricular nuclei of
pituitary gland. the hypothalamus with the posterior pituitary
6. The level of hormones in the body is regulated by gland. The other answers are not correct.
negative feedback mechanisms. Sensors detect a 6. Answer: b
change in the hormone level and adjust hormone RATIONALE: The level of many of the hormones in
secretion so that body levels are maintained within the body is regulated by negative feedback mecha-
an appropriate range. When the sensors detect a nisms. The other answers are incorrect.
decrease in hormone levels, they initiate changes 7. Answer: c
that cause an increase in hormone production; RATIONALE: Real progress in measuring plasma hor-
when hormone levels rise above the set point of the mone levels came more than 40 years ago with the
system, the sensors cause hormone production and use of competitive binding and the development
release to decrease the level. of radioimmunoassay methods. The other answers
are incorrect.
SECTION III: APPLYING YOUR KNOWLEDGE 8. Answer: d
RATIONALE: The advantages of a urine test include
Activity D
the relative ease of obtaining urine samples and
1. The nurse would expect a dual electron x-ray the fact that blood sampling is not required. The
absorptiometry (DEXA) to be ordered as the nurse other answers are not true.
knows that this test is used routinely for the 9. Answer: a
diagnosis and monitoring of osteoporosis and RATIONALE: A suppression test may be useful to
metabolic bone diseases. confirm this situation. The other answers are
2. The nurse would expect an assessment of insulin incorrect.
function through a blood glucose level. 10. Answer: b
RATIONALE: Isotopic imaging includes radioactive
SECTION IV: PRACTICING FOR NCLEX scanning of the thyroid. The other answers are all
examples of nonisotopic imaging.
Activity E
1. Answer: a
RATIONALE: Neurotransmitters such as epinephrine CHAPTER 32
can act as neurotransmitters or as hormones. The
other answers are not correct. SECTION II: ASSESSING YOUR
2. Answer: b UNDERSTANDING
RATIONALE: When hormones act locally on cells
other than those that produced the hormone, the Activity A
action is called paracrine. Hormones also can exert 1. hypofunction, hyperfunction
an autocrine action on the cells from which they 2. Congenital
were produced. The other terms are incorrect. 3. growth
3. Answer: c 4. insulinlike growth factors
RATIONALE: Hormones that are synthesized by 5. Growth hormone
nonvesicle-mediated pathways include the gluco- 6. Constitutional short stature
corticoids, androgens, estrogens, and mineralocor- 7. constitutional tall stature
ticoidsall steroids derived from cholesterol. The 8. gigantism
other answers are incorrect. 9. acromegaly
4. Answer: d 10. overstimulation
RATIONALE: Unbound adrenal and gonadal steroid 11. Precocious
hormones are conjugated in the liver, which ren- 12. Thyroid
ders them inactive, and then excreted in the bile 13. metabolism, protein
or urine. Adrenal and gonadal steroid hormones 14. metabolism
are not excreted in the feces, cell metabolites, or 15. immunoassay
the lungs. 16. preventable mental retardation
5. Answer: a 17. myxedema
RATIONALE: The hypothalamus and pituitary (i.e., 18. Thyrotoxicosis
hypophysis) form a unit that exerts control over 19. Graves
many functions of several endocrine glands as well 20. oxygen, metabolic
as a wide range of other physiologic functions. 21. Thyroid storm

Copyright 2011. Wolters Kluwer Health | Lippincott Williams & Wilkins. Study Guide for Porths Essentials of Pathophysiology, Third Edition.
LWBK707-Ans_p280-402.qxd 8/20/10 6:38PM Page 360 Aptara Inc

360 ANSWER KEY

22. adrenal cortex 26. Addison


23. Aldosterone 27. Cushing syndrome
24. glucocorticoid Activity B
25. Cortisol

Hypothalamus

Anterior
pituitary

Growth-promoting actions Anti-insulin effects


Growth hormone

Liver

IGF-1

Adipose Carbohydrate
Increased protein synthesis tissue metabolism

Bone and Body Decreased


Muscle Increased lipolysis
cartilage organs glucose use
Increased FFA use

Increased Increased Increased


linear size and Decrease in Increased blood
lean muscle
growth function adiposity glucose
mass

Activity C growth hormone; one of the most striking effects of


growth hormone is on linear bone growth, resulting
1. i 2. j 3. d 4. e 5. b
from its action on the epiphyseal growth plates of
6. g 7. f 8. c 9. a 10. h
long bones. The width of bone increases because of
Activity D enhanced periosteal growth; visceral and endocrine
1. Primary defects in endocrine function originate in the organs, skeletal and cardiac muscle, skin, and
target gland responsible for producing the hormone. connective tissue all undergo increased growth in
In secondary disorders of endocrine function, the response to growth hormone. In many instances, the
target gland is essentially normal, but its function is increased growth of visceral and endocrine organs is
altered by defective levels of stimulating hormones accompanied by enhanced functional capacity.
or releasing factors from the pituitary system. A terti- 4. Growth hormone secretion is stimulated by hypo-
ary disorder results from hypothalamic dysfunction. glycemia, fasting, starvation, increased blood levels
2. Hormones directly affected by hypopituitarism of amino acids (particularly arginine), and stress
are ACTH, thyrotropin, growth hormone, the conditions such as trauma, excitement, emotional
gonadotrophic hormones, and prolactin. Hypopitu- stress, and heavy exercise. Growth hormone is
itarism is characterized by a decreased secretion of inhibited by increased glucose levels, free fatty acid
pituitary hormones, which affects many of the release, cortisol, and obesity. Impairment of secre-
other endocrine systems by under stimulation. tion, leading to growth retardation, is common in
3. Growth hormone is necessary for growth and con- children with severe emotional deprivation.
tributes to the regulation of metabolic functions. 5. The secretion of thyroid hormone is regulated by
All aspects of cartilage growth are stimulated by the hypothalamic-pituitary-thyroid feedback

Copyright 2011. Wolters Kluwer Health | Lippincott Williams & Wilkins. Study Guide for Porths Essentials of Pathophysiology, Third Edition.
LWBK707-Ans_p280-402.qxd 8/20/10 6:38PM Page 361 Aptara Inc

ANSWER KEY 361

system. In this system, thyrotropin-releasing 2. Thyroid hormone is necessary for the brain to grow
hormone (TRH) controls the release of thyrotropin and develop. If the babys thyroid gland is not
(TSH) from the anterior pituitary gland. TSH working correctly, the doctor will order thyroid
increases the overall activity of the thyroid gland medicine for the baby. As long as the baby receives
by increasing thyroglobulin breakdown and the the medication as the doctor orders, the babys brain
release of thyroid hormone from follicles into the will grow and develop just as it is supposed to.
bloodstream, activating the iodide pump (by increas-
ing Na/I Symporter [NIS] activity), increasing the SECTION IV: PRACTICING FOR NCLEX
oxidation of iodide and the coupling of iodide to
tyrosine, and increasing the number and the size of
Activity F
the follicle cells. Increased levels of thyroid hormone 1. Answer: a
act in the feedback inhibition of TRH or TSH. RATIONALE: When further information regarding
6. The manifestations of the disorder are related pituitary function is required, combined hypothal-
largely to two factors: the hypometabolic state amic-pituitary function tests are undertaken
resulting from thyroid hormone deficiency, and (although these are performed less often today).
myxedematous involvement of body tissues. The These tests consist mainly of hormone stimulation
hypometabolic state associated with hypothyroidism tests (e.g., rapid ACTH stimulation test) or suppres-
is characterized by a gradual onset of weakness and sion tests (e.g., GH suppression test). The other
fatigue, a tendency to gain weight despite a loss of answers are incorrect.
appetite, and cold intolerance. As the condition 2. Answer: b
progresses, the skin becomes dry and rough and RATIONALE: The secretion of GH fluctuates over a
acquires a pale yellowish cast, which primarily 24-hour period, with peak levels occurring 1 to
results from carotene deposition, and the hair 4 hours after onset of sleep. The other answers are
becomes coarse and brittle. There can be loss of the incorrect.
lateral third of the eyebrows. Gastrointestinal 3. Answers: a, b, c
motility is decreased, producing constipation, flatu- RATIONALE: In addition to its effects on growth, GH
lence, and abdominal distention. Nervous system facilitates the rate of protein synthesis by all of the
involvement is manifested in mental dullness, cells of the body, enhances fatty acid mobilization
lethargy, and impaired memory. and increases the use of fatty acids for fuel, and
7. Addison disease is a relatively rare disorder in which maintains or increases blood glucose levels by
all the layers of the adrenal cortex are destroyed. decreasing the use of glucose for fuel. Growth hor-
Autoimmune destruction is the most common cause. mone has an initial effect of increasing insulin lev-
Because of a lack of glucocorticoids, the person with els. Growth hormone does not decrease the
Addison disease has poor tolerance to stress. Hyper- production of ACTH.
pigmentation results from elevated levels of ACTH. 4. Answer: b
The skin looks bronzed or suntanned in exposed RATIONALE: When the production of excessive GH
and unexposed areas, and the normal creases and occurs after the epiphyses of the long bones have
pressure points tend to become especially dark. The closed, as in the adult, the person cannot grow
gums and oral mucous membranes may become taller, but the soft tissues continue to grow. Enlarge-
bluish-black. Mineralocorticoid deficiency causes ment of the small bones of the hands and feet and
increased urinary losses of sodium, chloride, and of the membranous bones of the face and skull
water, along with decreased excretion of potassium. results in a pronounced enlargement of the hands
The result is hyponatremia, loss of extracellular fluid, and feet, a broad and bulbous nose, a protruding
decreased cardiac output, and hyperkalemia. lower jaw, and a slanting forehead. The other
8. The major manifestations of Cushing syndrome answers are incorrect.
represent an exaggeration of the many actions of 5. Answer: c
cortisol. Altered fat metabolism causes a peculiar dep- RATIONALE: Persons with precocious puberty
osition of fat characterized by a protruding abdomen, usually are tall for their age as children, but short
subclavicular fat pads or buffalo hump on the back, as adults because of the early closure of the
and a round, plethoric moon face. There is mus- epiphyses. The other answers are incorrect.
cle weakness, and the extremities are thin because 6. Answer: d
of protein breakdown and muscle wasting. RATIONALE: The assessment of thyroid autoanti-
bodies (e.g., antithyroid peroxidase antibodies in
SECTION III: APPLYING YOUR KNOWLEDGE Hashimoto thyroiditis) is important in the
diagnostic workup and consequent follow-up of
Activity E thyroid patients.
1. We are testing the baby for a disorder called congen- 7. Answer: a
ital hypothyroidism. This means that the babys thy- RATIONALE: As a result of myxedematous fluid accu-
roid gland is not functioning normally, and it is mulation, the face takes on a characteristic puffy
not producing thyroid hormone. look, especially around the eyes. The tongue is

Copyright 2011. Wolters Kluwer Health | Lippincott Williams & Wilkins. Study Guide for Porths Essentials of Pathophysiology, Third Edition.
LWBK707-Ans_p280-402.qxd 8/20/10 6:38PM Page 362 Aptara Inc

362 ANSWER KEY

enlarged, and the voice is hoarse and husky. The 10. glucose transporter
other answers are incorrect. 11. GLUT-4
8. Answers: a, c, e 12. Glucagon
RATIONALE: Thyroid storm is manifested by a very 13. glycogenolysis, gluconeogenesis
high fever, extreme cardiovascular effects (i.e., 14. insulin
tachycardia, congestive failure, and angina), and 15. Diabetes
severe CNS effects (i.e., agitation, restlessness, and 16. 100 mg/dL, 140 mg/dL
delirium). The mortality rate is high. Very low 17. Type 1
fever and bradycardia are not manifestations of a 18. idiopathic
thyroid storm. 19. Type 2
9. Answer: b 20. resistance
RATIONALE: Chronic suppression causes atrophy of 21. obesity, physical inactivity
the adrenal gland, and the abrupt withdrawal of 22. obesity
drugs can cause acute adrenal insufficiency. The 23. Gestational
other answers are incorrect. 24. fasting
10. Answer: c 25. casual, greater than ()
RATIONALE: In female infants, an increase in andro- 26. glycated hemoglobin
gens is responsible for creating the virilization syn- 27. insulin
drome of ambiguous genitalia with an enlarged 28. ketoacidosis
clitoris, fused labia, and urogenital sinus. The 29. hyperosmolar hyperglycemic
other answers are incorrect. 30. Advanced glycation end products
11. Answer: d 31. diabetic nephropathy
RATIONALE: Hydrocortisone usually is the drug of 32. Diabetic retinopathy
choice. The other answers are not drugs; they are 33. macrovascular disease
naturally occurring steroids. Activity B
12. Answers: a, b, c, e
1. d 2. c 3. f 4. e 5. h
RATIONALE: If Addison disease is the underlying
6. a 7. j 8. i 9. b 10. g
problem, exposure to even a minor illness or stress
11. k
can precipitate nausea, vomiting, muscular weak-
ness, hypotension, dehydration, and vascular Activity C
collapse.
13. Answer: a in insulin glucagon and gluconeogenesis
RATIONALE: The major manifestations of Cushing
syndrome represent an exaggeration of the many
actions of cortisol (see Table 32-2). Altered fat
metabolism causes a peculiar deposition of fat
blood glucose
characterized by a protruding abdomen, subclavic-
ular fat pads or buffalo hump on the back, and a
round, plethoric moon face. There is muscle
weakness, and the extremities are thin because of insulin release
protein breakdown and muscle wasting. The other glucagon
from beta cells
answers are incorrect.

CHAPTER 33 removal of
hepatic glucose
glucose from blood
production
SECTION II: ASSESSING YOUR
UNDERSTANDING
Activity A blood glucose
1. glucose
2. brain
3. hypoglycemia Activity D
4. glycogen 1. The actions of insulin are threefold: (1) it promotes
5. glycogenolysis glucose uptake by target cells and provides for glu-
6. gluconeogenesis cose storage as glycogen, (2) it prevents fat and
7. 9, 4 glycogen breakdown, and (3) it inhibits gluconeo-
8. Proteins genesis and increases protein synthesis
9. fatty acids, proteins

Copyright 2011. Wolters Kluwer Health | Lippincott Williams & Wilkins. Study Guide for Porths Essentials of Pathophysiology, Third Edition.
LWBK707-Ans_p280-402.qxd 8/20/10 6:38PM Page 363 Aptara Inc

ANSWER KEY 363

2. The release of insulin from the pancreatic beta eventually lead to beta cell dysfunction, increased
cells is regulated by blood glucose levels, insulin resistance, and greater hepatic glucose
increasing as blood glucose levels rise and decreas- production.
ing when blood glucose levels decline. Blood glu- 9. The most commonly identified signs and
cose enters the beta cell by means of the glucose symptoms of diabetes are referred to as the three
transporter, is phosphorylated by an enzyme polys: (1) polyuria (i.e., excessive urination), (2)
called glucokinase, and metabolized to form the polydipsia (i.e., excessive thirst), and (3) polypha-
adenosine triphosphate (ATP) needed to close the gia (i.e., excessive hunger). These three symptoms
potassium channels and depolarize the cell. Depo- are closely related to the hyperglycemia and glyco-
larization, in turn, results in opening of the suria of diabetes.
calcium channels and insulin secretion. 10. Weight loss despite normal or increased appetite is
3. The absolute lack of insulin in people with type 1 a common occurrence in people with uncontrolled
diabetes mellitus means that they are particularly type 1 diabetes. First, loss of body fluids results
prone to the development of ketoacidosis. One of from osmotic diuresis. Second, body tissue is lost
the actions of insulin is the inhibition of lipolysis because the lack of insulin forces the body to use
and release of free fatty acids (FFA) from fat cells. its fat stores and cellular proteins as sources of
In the absence of insulin, ketosis develops when energy.
these fatty acids are released from fat cells and 11. This technique involves the insertion of a small
converted to ketones in the liver. needle or plastic catheter into the subcutaneous
4. Type 1A diabetes is thought to be an autoimmune tissue of the abdomen. Tubing from the catheter is
disorder resulting from a genetic predisposition; connected to a syringe set into a small infusion
an environmental triggering event, such as an pump worn on a belt or in a jacket pocket. The
infection; and a T-lymphocytemediated hypersen- computer-operated pump then delivers one or
sitivity reaction against some beta cell antigen. more set basal amounts of insulin. In addition to
Much evidence has focused on the inherited major the basal amount delivered by the pump, a bolus
histocompatibility complex (MHC) genes on chro- amount of insulin may be delivered when needed
mosome 6. In addition to the MHC susceptibility (e.g., before a meal) by pushing a button.
genes for type 1 diabetes on chromosome 6, an 12. The three major metabolic derangements in
insulin gene regulating beta cell replication and diabetic ketoacidosis (DKA) are hyperglycemia,
function has been identified on chromosome 11. ketosis, and metabolic acidosis. Hyperglycemia
5. The metabolic abnormalities that lead to type 2 leads to osmotic diuresis, dehydration, and a criti-
diabetes include (1) insulin resistance, (2) deranged cal loss of electrolytes. Serum potassium levels
secretion of insulin by the pancreatic beta cells, and may be normal or elevated, despite total
(3) increased glucose production by the liver. potassium depletion resulting from protracted
6. Specific causes of beta cell dysfunction include an polyuria and vomiting. Metabolic acidosis is
initial decrease in the beta cell mass related to caused by the excess ketoacids that require buffer-
genetic or prenatal factors, increased apoptosis ing by bicarbonate ions; this leads to a marked
and/or decreased beta cell regeneration, beta cell decrease in serum bicarbonate levels.
exhaustion due to long-standing insulin resistance, 13. The chronic complications of diabetes include dis-
glucotoxicity, lipotoxicity, and amyloid deposition orders of the microvasculature (i.e., neuropathies,
or other conditions that have the potential to nephropathies, and retinopathies), macrovascular
reduce beta cell mass. complications (i.e., coronary artery, cerebral
7. The manifestations include obesity, high levels vascular, and peripheral vascular disease), and
of plasma triglycerides, and low levels of high- foot ulcers. In the sorbitol pathway, glucose is
density lipoproteins, hypertension, systemic transformed first to sorbitol and then to fructose.
inflammation, abnormal fibrinolysis, abnormal Although glucose is converted readily to sorbitol,
function of the vascular endothelium, and the rate at which sorbitol can be converted to fruc-
macrovascular disease. tose and then metabolized is limited. Sorbitol is
8. This has several consequences: first, excessive and osmotically active, and it has been hypothesized
chronic elevation of FFAs can cause beta cell dys- that the presence of excess intracellular amounts
function (lipotoxicity); second, FFAs act at the level may alter cell function in those tissues that use
of the peripheral tissues to cause insulin resistance this pathway.
and glucose underutilization by inhibiting glucose 14. Pathologic changes include thickening of the walls
uptake and glycogen storage; and third, the accu- of the nutrient vessels that supply the nerve, lead-
mulation of FFAs and triglycerides reduce hepatic ing to the assumption that vessel ischemia plays a
insulin sensitivity, leading to increased hepatic major role in the development of neural changes.
glucose production and hyperglycemia, especially In addition, segmental demyelinization process
fasting plasma glucose levels. Thus, an increase that affects the Schwann cell. This demyelinization
in FFAs that occurs in obese individuals with a process is accompanied by a slowing of nerve
genetic predisposition to type 2 diabetes may conduction.

Copyright 2011. Wolters Kluwer Health | Lippincott Williams & Wilkins. Study Guide for Porths Essentials of Pathophysiology, Third Edition.
LWBK707-Ans_p280-402.qxd 8/20/10 6:38PM Page 364 Aptara Inc

364 ANSWER KEY

15. Various glomerular changes may occur in people hyperglycemia and the development of diabetes
with diabetic nephropathy, including capillary mellitus and starvation. They stimulate gluconeo-
basement membrane thickening, diffuse glomeru- genesis by the liver, sometimes producing a 6- to
lar sclerosis, and nodular glomerulosclerosis. 10-fold increase in hepatic glucose production. A
Changes in the capillary basement membrane take prolonged increase in glucocorticoid hormones
the form of thickening of basement membranes does not cause hepatomegaly, portal hypertension,
along the length of the glomeruli. Diffuse glomeru- or adrenal hyperplasia.
losclerosis consists of thickening of the basement 4. Answer: a
membrane and the mesangial matrix. Nodular RATIONALE: Type 1A diabetes is thought to be an
glomerulosclerosis, Kimmelstiel-Wilson disease, is autoimmune disorder resulting from a genetic pre-
a form of glomerulosclerosis that involves the disposition (i.e., diabetogenic genes); an environ-
development of nodular lesions in the glomerular mental triggering event, such as an infection; and
capillaries of the kidneys, causing impaired blood a T-lymphocytemediated hypersensitivity reaction
flow with progressive loss of kidney function and, against some beta cell antigen. The other answers
eventually, renal failure. Changes in the base- are incorrect.
ment membrane in diffuse glomerulosclerosis 5. Answers: a, c, d
and Kimmelstiel-Wilson syndrome allow plasma RATIONALE: The metabolic abnormalities that lead
proteins to escape in the urine, causing proteinuria to type 2 diabetes include (1) insulin resistance,
and the development of hypoproteinemia, edema, (2) deranged secretion of insulin by the pancreatic
and others signs of impaired kidney function. beta cells, and (3) increased glucose production by
the liver. The other answers are incorrect.
SECTION III: APPLYING YOUR KNOWLEDGE 6. Answer: a
RATIONALE: Such diabetes can occur with pancreatic
Activity E disease or the removal of pancreatic tissue and with
1. Type 1A diabetes mellitus is thought to be a endocrine diseases, such as acromegaly, Cushing
chronic autoimmune disease that has a genetic pre- syndrome, or pheochromocytoma. Endocrine disor-
disposition. Type 1A diabetes mellitus is character- ders that produce hyperglycemia do so by increasing
ized by a total lack of insulin, an elevation of blood the hepatic production of glucose or decreasing the
glucose, and a breakdown of body fats and cellular use of glucose. Dwarfism, hepatomegaly,
proteins. Type 1A diabetics are prone to the devel- and pancreatic hyperplasia do not cause secondary
opment of ketoacidosis. Type 1A diabetics require diabetes.
daily injections of exogenous insulin to control 7. Answer: b
blood glucose levels and prevent ketosis. RATIONALE: Diagnosis and careful medical manage-
2. Presently, there is no cure for diabetes mellitus. ment are essential because women with gestational
There is research being conducted into prevention diabetes mellitus are at higher risk for complications
of the disease, but none has been successful to date. of pregnancy, mortality, and fetal abnormalities.
Fetal abnormalities include macrosomia (i.e.,
SECTION IV: PRACTICING FOR NCLEX large body size), hypoglycemia, hypocalcemia,
polycythemia, and hyperbilirubinemia. Microsomia
Activity F
and hypercalcemia are not fetal abnormalities are
1. Answer: b associated with gestational diabetes mellitus.
RATIONALE: Each islet is composed of beta cells that 8. Answer: c
secrete insulin and amylin, alpha cells that secrete RATIONALE: The most commonly identified signs
glucagon, and delta cells that secrete somatostatin. and symptoms of diabetes are referred to as the
In addition, at least one other type of cell, the PP three polys: (1) polyuria (i.e., excessive urination),
cell, is present in small numbers in the islets and (2) polydipsia (i.e., excessive thirst), and (3)
secrets a hormone of uncertain function called polyphagia (i.e., excessive hunger). Pheochromo-
pancreatic polypeptide. cytoma and polycythemia are not hallmark signs
2. Answers: a, b, c, e of diabetes mellitus.
RATIONALE: These hormones, along with glucagon, 9. Answers: 1-d, 2-c, 3-e, 4-b, 5-f, 6-a
are sometimes called counterregulatory hormones 10. Answer: d
because they counteract the storage functions of RATIONALE: The definitive diagnosis of DKA
insulin in regulating blood glucose levels during consists of hyperglycemia (blood glucose levels
periods of fasting, exercise, and other situations 250 mg/dL), low bicarbonate (15 mEq/L), and
that either limit glucose intake or deplete glucose low pH (7.3), with ketonemia (positive at 1:2
stores. Mineralocorticoids are not considered dilution) and moderate ketonuria. The other
counterregulatory hormones. answers are not diagnostic for DKA.
3. Answer: c 11. Answer: a
RATIONALE: In predisposed persons, the prolonged RATIONALE: Alcohol decreases liver gluconeogene-
elevation of glucocorticoid hormones can lead to sis, and people with diabetes need to be cautioned

Copyright 2011. Wolters Kluwer Health | Lippincott Williams & Wilkins. Study Guide for Porths Essentials of Pathophysiology, Third Edition.
LWBK707-Ans_p280-402.qxd 8/20/10 6:38PM Page 365 Aptara Inc

ANSWER KEY 365

about its potential for causing hypoglycemia, espe- infections may bear some relation to the presence
cially if alcohol is consumed in large amounts or of a neurogenic bladder or nephrosclerotic
on an empty stomach. changes in the kidneys. Urinary retention and
12. Answer: b urinary incontinence can both be the result of a
RATIONALE: The signs and symptoms of neurogenic bladder. Nephrotic syndrome is not
hypoglycemia can be divided into two categories: thought to be related to a neurogenic bladder in
(1) those caused by altered cerebral function and diabetics.
(2) those related to activation of the autonomic
nervous system. Because the brain relies on blood
glucose as its main energy source, hypoglycemia CHAPTER 34
produces behaviors related to altered cerebral
function. Headache, difficulty in problem solving, SECTION II: ASSESSING YOUR
disturbed or altered behavior, coma, and seizures UNDERSTANDING
may occur. Muscle spasms are not one of the signs
Activity A
or symptoms of hypoglycemia.
13. Answer: c 1. neurons
RATIONALE: The Somogyi effect describes a cycle 2. Schwann cells, neuroglial
of insulin-induced posthypoglycemic episodes. 3. body, dendrites, axons, synapses
In 1924, Joslin and associates noticed that hypo- 4. Dendrites
glycemia was associated with alternate episodes of 5. Schwann, satellite
hyperglycemia. The other answers are not correct. 6. Satellite
14. Answer: d 7. myelin
RATIONALE: The loss of feeling, touch, and 8. nodes of Ranvier, saltatory conduction
position sense, which increases the risk of falling. 9. oligodendrocytes
Impairment of temperature and pain sensation 10. Glucose
increases the risk of serious burns and injuries to 11. action potentials
the feet. Denervation of the small muscles of the 12. resting membrane potential
foot result in clawing of the toes and displacement 13. synapses
of the submetatarsal fat pad anteriorly. These 14. Chemical
changes together with joint and connective 15. synaptic cleft
tissue changes alter the biomechanics of the foot, 16. depolarization, hyperpolarization
increasing plantar pressure and predisposing to 17. excitatory postsynaptic
development of foot trauma and ulcers. The other 18. neurotransmission
answers are incorrect. 19. Neuromodulator
15. Answer: a 20. Neurotrophic
RATIONALE: Diabetic nephropathy is the leading 21. reticular activating system
cause of chronic kidney disease, accounting for 22. pia mater
40% of new cases. Also, diabetes is the leading 23. spinal nerves
cause of acquired blindness in the United States. 24. paired segmental spinal nerves
The liver and pancreas are not organs that diabetes 25. plexuses
attacks. 26. reflex
16. Answer: a 27. withdrawal
RATIONALE: Multiple risk factors for macrovascular 28. hindbrain, midbrain, forebrain
disease, including obesity, hypertension, 29. hypoglossal
hyperglycemia, hyperinsulinemia, hyperlipidemia, 30. vagus
altered platelet function, endothelial dysfunction, 31. spinal accessory nerve
systemic inflammation (as evidenced by increased 32. glossopharyngeal nerve
C-reactive protein), and elevated fibrinogen levels, 33. vestibulocochlear nerve
frequently are found in people with diabetes. 34. facial nerve
Hypotension, hypoinsulinemia and decreased fib- 35. abducens
rinogen levels are not risk factors for macrovascu- 36. trigeminal nerve
lar disease in diabetics. 37. cerebellum
17. Answer: b 38. thalamus
RATIONALE: Foot problems have been reported as 39. gyrus, sulcus
the most common complication leading to hospi- 40. basal ganglia
talization among people with diabetes. 41. primary somatosensory cortex
18. Answer: c 42. meninges
RATIONALE: Pyelonephritis and urinary tract infec- 43. CSF
tions are relatively common in persons with 44. autonomic nervous system.
diabetes, and it has been suggested that these 45. parasympathetic nervous system

Copyright 2011. Wolters Kluwer Health | Lippincott Williams & Wilkins. Study Guide for Porths Essentials of Pathophysiology, Third Edition.
LWBK707-Ans_p280-402.qxd 8/20/10 6:38PM Page 366 Aptara Inc

366 ANSWER KEY

Activity B dense line. Linking proteins, proteolipid protein


(PLP) found only in the CNS and myelin protein
1.
zero (MPZ) found only in the PNS, help stabilize
Segments adjacent plasma membranes of the myelin sheath.
2. Nervous tissue has a high rate of metabolism.
Although the brain comprises only 2% of the
Dorsal bodys weight, it receives approximately 15% of
root the resting cardiac output and consumes 20%
of its oxygen. Despite its substantial energy require-
ments, the brain cannot store oxygen or effectively
engage in anaerobic metabolism. An interruption in
the blood or oxygen supply to the brain rapidly
Dorsal root leads to clinically observable signs and symptoms.
ganglion Without oxygen, brain cells continue to function for
neuron approximately 10 seconds. Unconsciousness occurs
Spinal
almost simultaneously with cardiac arrest, and the
IA neuron Ventral
nerve death of brain cells begins within 4 to 6 minutes.
root
Dorsal root
3. The local currents resulting from an excitatory
ganglion postsynaptic potential (EPSP, sometimes called a
generator potential) are usually insufficient to reach
threshold and cause depolarization of the axons
2. initial segment. However, if several EPSPs occur
simultaneously, the area of depolarization can
Corpus become large enough and the currents at the initial
callosum
Septum segment can become strong enough to exceed the
pellucidum
Third threshold potential and initiate an action potential.
Frontal ventricle This summation of depolarized areas is called spa-
lobe
tial summation. Excitatory postsynaptic potentials
Occipital
lobe also can summate and cause an action potential if
Pineal they occur in rapid succession. This temporal
body aspect of the occurrence of two or more EPSPs is
Interventricular
foramen Cerebral called temporal summation. Inhibitory postsynaptic
Anterior aqueduct
commissure potentials (IPSPs) also can undergo spatial and tem-
Fourth
Midbrain ventricle poral summation with each other and with EPSPs,
Pons Cerebellum reducing the effectiveness of the latter by a roughly
Medulla Central canal algebraic summation. If the sum of EPSPs and IPSPs
oblongata
keeps the depolarization at the initial segment
Spinal cord
below threshold levels, no action potential occurs.
4. (1) They can be broken down into inactive
Activity C substances by enzymes; (2) they can be taken back
up into the presynaptic neuron in a process called
1. reuptake; or (3) they can diffuse away into the inter-
1. f 2. h 3. e 4. b 5. d cellular fluid until its concentration is too low to
6. c 7. a 8. j 9. g 10. i influence postsynaptic excitability.
2. 5. The nervous system appears very early in embryonic
1. b 2. i 3. e 4. g 5. f development. At the beginning of week 3, the ecto-
6. a 7. h 8. c 9. d derm begins to invaginate and migrates between the
Activity D two layers, forming a third layer called the mesoderm.
1. Myelin formation is essentially the same in both Mesoderm along the entire midline of the embryo
the peripheral nervous system (PNS) and CNS; both forms a specialized rod of embryonic tissue called
contain myelin basic protein and both involve the the notochord. The notochord and adjacent meso-
winding of plasma membranes around the nerve derm provide the necessary induction signal for the
fiber. During the wrapping of myelin, the cytoplasm overlying ectoderm to differentiate and form a thick-
between two adjacent inner leaflets of the plasma ened structure called the neural plate. Within the
membrane is expelled. The two adjacent inner neural plate an axial groove develops and sinks into
leaflets and any remaining cytoplasm appear as a the underlying mesoderm, allowing its walls to fuse
dark line called the major dense line. Likewise, dur- across the top and form an ectodermal tube called
ing the wrapping of the plasma membranes to form the neural tube. As the neural tube closes, ectoder-
myelin, adjacent outer plasma membrane leaflets mal cells called neural crest cells migrate away from
become opposed creating the interperiod or minor the dorsal surface of the neural tube to become the

Copyright 2011. Wolters Kluwer Health | Lippincott Williams & Wilkins. Study Guide for Porths Essentials of Pathophysiology, Third Edition.
LWBK707-Ans_p280-402.qxd 8/20/10 6:38PM Page 367 Aptara Inc

ANSWER KEY 367

progenitors of the neurons and supporting cells of extent of the dysfunction cannot be determined
the PNS. During development, the more rostral until the infant is born and can be better assessed.
portions of the embryonic neural tubeappro-
ximately 10 segmentsundergoes extensive modifi- SECTION IV: PRACTICING FOR NCLEX
cation and enlargement to form the brain.
Activity F
6. Four columns of afferent (sensory) neurons in the
dorsal root ganglia directly innervate four correspon- 1. Answer: a
ding columns of input association neurons in the RATIONALE: The supporting cells, such as Schwann
dorsal horn. These columns are categorized as spe- cells in the PNS and the neuroglial cells in the
cial and general afferents: special somatic afferent, CNS, protect the nervous system and provide
general somatic afferent, special visceral afferent, metabolic support for the neurons. The other
and general visceral afferent. The ventral horn answers are incorrect.
contains three longitudinal cell columns: general 2. Answer: b
visceral efferent, pharyngeal efferent, and general RATIONALE: These membrane channels are guarded
somatic efferent (Fig. 34-9). Each of these cell by voltage-dependent gates that open and close
columns contains output association and efferent with changes in the membrane potential. The
neurons. The output association neurons other answers are incorrect.
coordinate and integrate the function of the effer- 3. Answer: c
ent motor neurons cells of its column. RATIONALE: The most common type of synapse is the
7. Maintenance of a chemically stable environment chemical synapse. The other answers are incorrect.
is essential to the function of the brain. In most 4. Answer: d
regions of the body, extracellular fluid undergoes RATIONALE: Neurotransmitters are synthesized in
small fluctuations in pH and concentrations of hor- the cytoplasm of the axon terminal. The other
mones, amino acids, and potassium ions during answers are incorrect.
routine daily activities such as eating and exercising. 5. Answer: a
If the brain were to undergo such fluctuations, the RATIONALE: Neuromodulator molecules react with
result would be uncontrolled neural activity because presynaptic or postsynaptic receptors to alter the
some substances such as amino acids act as release of or response to neurotransmitters. The
neurotransmitters, and ions such as potassium influ- other answers are incorrect.
ence the threshold for neural firing. Two barriers, 6. Answer: b
the blood-brain barrier and the CSF-brain barrier, RATIONALE: With rare exceptions, peripheral nerves
provide the means for maintaining the stable chem- including the cranial nerves contain afferent and
ical environment of the brain. Only water, carbon efferent processes of more than one of the four
dioxide, and oxygen enter the brain with relative afferent and three efferent cell columns. This pro-
ease; the transport of other substances between the vides the basis for assessing the function of the any
brain and the blood is slower and more controlled. peripheral nerve. The other answers are incorrect.
8. The blood-brain barrier prevents many drugs from 7. Answer: c
entering the brain. Most highly water-soluble com- RATIONALE: On the lateral sides of the spinal
pounds are excluded from the brain, especially mol- cord, extensions of the pia mater, the denticulate
ecules with high ionic charge such as many of the ligaments, attach the sides of the spinal cord to the
catecholamines. In contrast, many lipid-soluble bony walls of the spinal canal. Thus, the cord is sus-
molecules cross the lipid layers of the blood-brain pended by both the denticulate ligaments and the
barrier with ease. Some drugs, such as the antibiotic segmental nerves. The posterior vertebra and verte-
chloramphenicol, are highly lipid-soluble and there- bral blood vessels do not support the spinal cord.
fore enter the brain readily. Other medications have 8. Answer: d
a low solubility in lipids and enter the brain slowly RATIONALE: The myotatic or stretch reflex controls
or not at all. Alcohol, nicotine, and heroin are very muscle tone and helps maintain posture. Special-
lipid-soluble and rapidly enter the brain. Some sub- ized sensory nerve terminals in skeletal muscles
stances that enter the capillary endothelium are and tendons relay information on muscle stretch
converted by metabolic processes to a chemical and joint tension to the CNS. This information,
form incapable of moving into the brain. which drives postural reflex mechanisms, also is
relayed to the thalamus and the sensory cortex
SECTION III: APPLYING YOUR KNOWLEDGE and is experienced as proprioception, the sense of
body movement and position.
Activity E 9. Answer: a
1. Your baby has a meningomyeloceles. RATIONALE: The cerebellum compares what is actu-
2. Most children with meningomyeloceles have clini- ally happening with what is intended to happen.
cal dysfunction in both the motor and sensory It then transmits the appropriate corrective signals
nerves of the lower extremities. Dysfunction back to the motor system, instructing it to increase
usually extends to bowel and bladder control. The or decrease the activity of the participating muscle

Copyright 2011. Wolters Kluwer Health | Lippincott Williams & Wilkins. Study Guide for Porths Essentials of Pathophysiology, Third Edition.
LWBK707-Ans_p280-402.qxd 8/20/10 6:38PM Page 368 Aptara Inc

368 ANSWER KEY

groups so that smooth and accurate movements 24. threshold


can be performed. Answer B describes the trigemi- 25. Cutaneous
nal nerve, which exits the brain stem. Answer C 26. Deep somatic
describes the pons. Answer D describes midbrain. 27. warning
10. Answer: b 28. analgesic
RATIONALE: Parkinson disease, Huntington chorea, 29. hyperalgesia
and some forms of cerebral palsy, among other 30. Analgesia
dysfunctions involving the basal ganglia, result in 31. Neuralgia
a frequent or continuous release of abnormal pos- 32. Cluster
tural or axial and proximal movement patterns. If 33. tension-type
damage to the basal ganglia is localized to one 34. temporomandibular joint
side, the movements occur on the opposite side of Activity B
the body. The other answers are incorrect.
1.
11. Answer: c
RATIONALE: The sympathetic and parasympathetic Somatosensory
nervous systems are continually active. The effect cortex
of this continual or basal (baseline) activity is
referred to as tone. Third-
12. Answer: d order
RATIONALE: Dopamine, which is an intermediate
compound in the synthesis of norepinephrine,
also acts as a neurotransmitter. It is the principal Thalamus
inhibitory transmitter of internuncial neurons in
the sympathetic ganglia. It also has vasodilator
Dorsal root
effects on renal, splanchnic, and coronary blood Second-
ganglion
vessels when given intravenously and is Receptor order
sometimes used in the treatment of shock.

CHAPTER 35 First-
order
SECTION II: ASSESSING YOUR
UNDERSTANDING
Activity A
2. Lips
1. somatosensory
Trunk/back
2. General
Lips
3. Special
4. visceral Activity C
5. trigeminal 1.
6. dermatome 1. f 2. a 3. d 4. i 5. j
7. discriminative 6. e 7. g 8. c 9. h 10. b
8. anterolateral 2.
9. modalities 1. d 2. f 3. c 4. a 5. b
10. action potentials 6. e
11. acuity Activity D
12. tactile
1. Sensory systems are organized in a serial succession
13. Thermal
of neurons consisting of first-order, second-order,
14. pain
and third-order neurons. First-order neurons trans-
15. sensory, perception
mit sensory information from the periphery to the
16. Neuropathic
CNS. Second-order neurons communicate with var-
17. neuromatrix
ious reflex networks and sensory pathways in the
18. Nociceptive
spinal cord and travel directly to the thalamus.
19. C fibers
Third-order neurons relay information from the
20. neospinothalamic
thalamus to the cerebral cortex. This organizing
21. paleospinothalamic
framework corresponds with the three primary
22. periaqueductal gray
levels of neural integration in the somatosensory
23. enkephalins, endorphins, dynorphins

Copyright 2011. Wolters Kluwer Health | Lippincott Williams & Wilkins. Study Guide for Porths Essentials of Pathophysiology, Third Edition.
LWBK707-Ans_p280-402.qxd 8/20/10 6:38PM Page 369 Aptara Inc

ANSWER KEY 369

system: the sensory units, which contain the sensory nates in the abdominal or thoracic viscera is
receptors; the ascending pathways; and the central diffuse, poorly localized, and often perceived at a
processing centers in the thalamus and cerebral site far removed from the affected area.
cortex. 6. Heat dilates blood vessels and increases local blood
2. These somatosensory receptors monitor four major flow; it also can influence the transmission of pain
types or modalities of sensation: discriminative impulses and increase collagen extensibility. An
touch, which is required to identify the size and increase in local circulation can reduce the level of
shape of objects and their movement across the nociceptive stimulation by reducing local ischemia
skin; temperature sensation; sense of movement of caused by muscle spasm or tension, increase the
the limbs and joints of the body; and nociception, removal of metabolites and inflammatory mediators
or pain. that act as nociceptive stimuli, and help to reduce
3. A pinpoint pressed against the skin of the sole of swelling and relieve pressure on local nociceptive
the foot that results in a withdrawal reflex and a endings. It also may trigger the release of endogenous
complaint of skin pain confirms the functional opioids. Heat also alters the viscosity of collagen
integrity of the afferent terminals in the skin, the fibers in ligaments, tendons, and joint structures
entire pathway through the peripheral nerves of so that they are more easily extended and can
the foot, leg, and thigh to the sacral (S1) dorsal be stretched further before the nociceptive endings
root ganglion, and through the dorsal root into are stimulated.
the spinal cord segment. It confirms that the 7. The pain often begins as sensations of tingling,
somatosensory input association cells receiving heat and cold, or heaviness, followed by burning,
this information are functioning and that the cramping, or shooting pain. It may disappear spon-
reflex circuitry of the cord segments (L5 to S2) is taneously or persist for many years. Several theories
functioning. In addition, the lower motor neurons have been proposed as to the causes of phantom
of the L4 to S1 ventral horn can be considered pain. One theory is that the end of a regenerating
operational, and their axons through the ventral nerve becomes trapped in the scar tissue of the
roots, the mixed peripheral nerve, and the motor amputation site. It is known that when a peripheral
neuron to the muscles producing the withdrawal nerve is cut, the scar tissue that forms becomes a
response can be considered intact and functional. barrier to regenerating outgrowth of the axon. The
The communication between the lower motor growing axon often becomes trapped in the scar tis-
neuron and the muscle cells is functional, and sue, forming a tangled growth of small-diameter
these muscles have normal responsiveness and axons, including primary nociceptive afferents and
strength. Observation of a normal withdrawal sympathetic efferents. It has been proposed that
reflex rules out peripheral nerve disease, disorders these afferents show increased sensitivity to
of the dorsal root and ganglion, diseases of the innocuous mechanical stimuli and to sympathetic
myoneural junction, and severe muscle diseases. activity and circulating catecholamines. A related
Normal reflex function also indicates that many theory moves the source of phantom limb pain
major descending CNS tract systems are function- to the spinal cord, suggesting that the pain is
ing within normal limits. If the person is able to due to the spontaneous firing of spinal cord neu-
report the pinprick sensation and accurately iden- rons that have lost their normal sensory input
tify its location, many ascending systems through from the body. In one hypothesis, the pain is
much of the spinal cord and brain also are caused by changes in the flow of signals through
functioning normally, as are basic intellect and somatosensory areas of the brain.
speech mechanisms. 8. Migraine without aura is a pulsatile, throbbing,
4. According to the gate control theory, the internun- unilateral headache that typically lasts 1 to 2 days
cial neurons involved in the gating mechanism are and is aggravated by routine physical activity. The
activated by large-diameter, faster-propagating headache is accompanied by nausea and vomiting,
fibers that carry tactile information. The simultane- which often is disabling, and sensitivity to light
ous firing of the large-diameter touch fibers has the and sound. Visual disturbances occur quite
potential for blocking the transmission of impulses commonly and consist of visual hallucinations
from the small-diameter myelinated and unmyeli- such as stars, sparks, and flashes of light. Migraine
nated pain fibers. Pain therapists have long known with aura has similar symptoms, but with the addi-
that pain intensity can be temporarily reduced dur- tion of reversible visual symptoms including posi-
ing active tactile stimulation. tive features (e.g., flickering lights spots, or lines)
5. Referred pain is pain that is perceived at a site and/or negative features (loss of vision); fully
different from its point of origin but innervated by reversible sensory symptoms including positive
the same spinal segment. It is hypothesized that features (feeling of pins or needles) or negative
visceral and somatic afferent neurons converge on features (numbness); and fully reversible speech
the same dorsal horn projection neurons (Fig. 35-10). disturbance.
For this reason, it can be difficult for the brain to 9. Activation of the trigeminal sensory fibers may lead
identify the original source of pain. Pain that origi- to the release of neuropeptides, causing painful

Copyright 2011. Wolters Kluwer Health | Lippincott Williams & Wilkins. Study Guide for Porths Essentials of Pathophysiology, Third Edition.
LWBK707-Ans_p280-402.qxd 8/20/10 6:38PM Page 370 Aptara Inc

370 ANSWER KEY

neurogenic inflammation within the meningeal 9. Answer: a


vasculature characterized by plasma protein extrava- RATIONALE: Part of the reluctance of health care
sation, vasodilation, and mast cell degranulation. workers to provide adequate relief for acute pain
Another possible mechanism implicates neurogenic has been fear of addiction. However, addiction to
vasodilation of meningeal blood vessels as a key opioid medications is thought to be virtually non-
component of the inflammatory processes that existent when these drugs are prescribed for acute
occur during migraine. Activation of trigeminal sen- pain. The other answers are not the major concern.
sory fibers evokes a neurogenic dural vasodilation 10. Answer: b
mediated by calcitonin gene-related peptide. It also RATIONALE: The World Health Organization has
has been observed that calcitonin gene-related pep- created an analgesic ladder for cancer pain that
tide level is elevated during migraine. assists clinicians in choosing the appropriate anal-
gesic. The other answers are incorrect.
SECTION III: APPLYING YOUR KNOWLEDGE 11. Answers: a, b, c
RATIONALE: The ideal analgesic would be effective,
Activity E
nonaddictive, and inexpensive. In addition, it
1. It is difficult to assess pain and discomfort in some- would produce minimal adverse effects and not
one suffering with dementia. In our facility we use affect the persons level of consciousness.
The Assessment for Discomfort in Dementia Protocol as 12. Answer: c
it has been shown to improve pain management in RATIONALE: Surgery for severe, intractable pain of
this population. peripheral or central origin has met with some
2. Acetaminophen is the drug of choice to manage success. It can be used to remove the cause or
this clients discomfort. You can also place ice on block the transmission of intractable pain from
the cast at the point of fracture for 20 minutes, phantom limb pain, severe neuralgia, inoperable
each hour, to help reduce the discomfort. cancer of certain types, and causalgia. The other
answers are incorrect.
SECTION IV: PRACTICING FOR NCLEX 13. Answer: d
RATIONALE: Primary hyperalgesia describes pain
Activity F
sensitivity that occurs directly in damaged tissues.
1. Answers: 1-e, 2-a, 3-g, 4-b, 5-c, 6-h, 7-d, 8-f The other answers are incorrect.
2. Answers: 1-c, 2-g, 3-i, 4-d, 5-h, 6-a, 7-f, 8-j, 9-b, 14. Answers: 1-d, 2-b, 3-a, 4-c
10-k, 11-e 15. Answer: a
3. Answer: a RATIONALE: Treatment of phantom limb pain has
RATIONALE: Clinically, neurologic assessment of been accomplished by the use of sympathetic
somatosensory function can be done by testing blocks, transcutaneous electrical nerve stimulation
the integrity of spinal segmental nerves. The other of the large myelinated afferents innervating the
answers are incorrect. area, hypnosis, and relaxation training.
4. Answer: b 16. Answer: b
RATIONALE: Stimuli used include pressure from a RATIONALE: Based on clinical trials, first-line agents
sharp object, strong electric current to the skin, or include acetylsalicylic acid, combinations of acet-
application of heat or cold of approximately 10C aminophen, acetylsalicylic acid, and caffeine and
above or below normal skin temperature. The nonsteroidal anti-inflammatory drug analgesics
other answers are incorrect. (e.g., naproxen sodium, ibuprofen), serotonin
5. Answer: c (5-HT1) receptor agonists (e.g., sumatriptan, nara-
RATIONALE: The amino acid glutamate is a major triptan, rizatriptan, zolmitriptan), ergotamine
excitatory neurotransmitter released from the cen- derivatives (e.g., dihydroergotamine), and
tral nerve endings of the nociceptive neurons. The antiemetic medications (e.g., ondansetron, meto-
other answers are incorrect. clopramide). Morphine, tramadol, and syrup of
6. Answer: d ipecac are not first-line drugs in the treatment of
RATIONALE: The paleospinothalamic tract is a migraine.
slower-conducting, multisynaptic tract 17. Answer: c
concerned with the diffuse, dull, aching, and RATIONALE: Cluster headache is a type of primary
unpleasant sensations that commonly are associ- neurovascular headache that typically includes
ated with chronic and visceral pain. The other severe, unrelenting, unilateral pain located, in
answers are incorrect. order of decreasing frequency, in the orbital, retro-
7. Answers: 1-c, 2-d, 3-f, 4-g, 5-e, 6-b, 7-a orbital, temporal, supraorbital, and infraorbital
8. Answers: a, c region. The other answers are incorrect.
RATIONALE: Assessment includes such things as the 18. Answer: d
nature, severity, location, and radiation of the RATIONALE: With children 3 to 8 years old, scales
pain. Spinal reflex involvement and spinal tract with faces of actual children or cartoon faces can
involvement are not assessed when assessing pain.

Copyright 2011. Wolters Kluwer Health | Lippincott Williams & Wilkins. Study Guide for Porths Essentials of Pathophysiology, Third Edition.
LWBK707-Ans_p280-402.qxd 8/20/10 6:38PM Page 371 Aptara Inc

ANSWER KEY 371

be used to obtain a report of pain. The other pain Activity B


rating scales are inappropriate in this age group.
19. Answer: a Premotor
RATIONALE: The overriding principle in all pediatric cortex Motor Somatosensory
pain management is to treat each childs pain on (8) (6) cortex cortex
Frontal
an individual basis and to match the analgesic (4) (3,1,2)
eye fields
agent with the cause and the level of pain. The (part of
other answers are incorrect. area 8)

CHAPTER 36
SECTION II: ASSESSING YOUR
UNDERSTANDING
Activity A Broca
area Primary
1. Motor function Primary
(45,44) visual
2. spinal cord Vestibular auditory cortex
3. polysynaptic cortex cortex (17)
4. posture (44)
5. cortex
6. motor
7. cerebellum, basal ganglia Activity C
8. circuits 1.
9. muscle spindles 1. d 2. c 3. i 4. e 5. a
10. Golgi tendon organs 6. f 7. j 8. h 9. g 10. b
11. peripheral 2.
12. motor 1. c 2. e 3. f 4. i 5. b
13. atrophy 6. j 7. g 8. a 9. h 10. d
14. muscle tone Activity D
15. UMN
1.
16. Hyporeflexia
17. lower motor neuron disorders, peripheral,
2 S 5 S 4 S 3 S 1
myopathies
18. dystrophy
19. fibrillations Activity E
20. Duchenne 1. The lowest level of the hierarchy occurs at the spinal
21. neuromuscular junction cord, which contains the basic reflex circuitry
22. acetylcholine needed to coordinate the function of the motor
23. Myasthenia gravis units involved in the planned movement. Above the
24. spinal cord, cranial nerve spinal cord is the brain stem, and above the brain
25. peripheral nerve stem is the cerebellum and basal ganglia, structures
26. Mononeuropathies that modulate the actions of the brain stem systems.
27. Polyneuropathies Overseeing these supraspinal structures are the
28. herniated disk motor centers in the cerebral cortex. The highest
29. cerebellar level of function, which occurs at the level of the
30. basal ganglia frontal cortex, is concerned with the purpose and
31. tremor planning of the motor movement. The efficiency of
32. Parkinson movement depends on input from sensory systems
33. bradykinesia that operate in parallel with the motor systems.
34. Amyotrophic lateral sclerosis 2. The motor neuron and the group of muscle fibers it
35. Multiple sclerosis (MS) innervates in a muscle is called a motor unit. When
36. demyelination the motor neuron develops an action potential, all
37. spinal cord injury of the muscle fibers in the motor unit it innervates
38. loss develop action potentials, causing them to contract
39. Tetraplegia simultaneously. Thus, a motor neuron and the
40. Paraplegia muscle fibers it innervates function as a single
41. vasovagal unitthe basic unit of motor control. Each motor
42. Orthostatic neuron undergoes multiple branching, making it
43. deep venous thrombosis

Copyright 2011. Wolters Kluwer Health | Lippincott Williams & Wilkins. Study Guide for Porths Essentials of Pathophysiology, Third Edition.
LWBK707-Ans_p280-402.qxd 8/20/10 6:38PM Page 372 Aptara Inc

372 ANSWER KEY

possible for a single motor neuron to innervate a process of repair and regeneration, and progres-
few to thousands of muscle fibers. In general, large sive fibrosis.
musclesthose containing hundreds or thousands 6. Segmental demyelination occurs when there is a
of muscle fibers and providing gross motor disorder of the Schwann cell (as in Guillain-Barr
movementhave large motor units. This sharply syndrome) or damage to the myelin sheath
contrasts with those that control the hand, tongue, (e.g., sensory neuropathies), without a primary
and eye movements, for which the motor units are abnormality of the axon. It typically affects some
small and permit very precise control. Schwann cells while sparing others. Axonal degen-
3. The muscle spindles consist of a group of special- eration is caused by primary injury to a neuronal
ized miniature skeletal muscle fibers called intrafusal cell body or its axon. Damage to the axon may be
fibers that are encased in a connective tissue capsule due either to a focal event occurring at some point
and attached to the extrafusal fibers of a skeletal along the length of the nerve (e.g., trauma or
muscle. In the center of the receptor area, a large ischemia) or to a more generalized abnormality
sensory neuron spirals around the intrafusal fiber affecting the neuronal cell body (neuropathy).
forming the so-called primary or annulospiral 7. The condition can be caused by a variety of events
ending. The intrafusal muscle fibers function that produce a reduction in the capacity of the
as stretch receptors. When a skeletal muscle is carpal tunnel (i.e., bony or ligament changes) or
stretched, the spindle and its intrafusal fibers are an increase in the volume of the tunnel contents
stretched, resulting in increased firing of their (i.e., inflammation of the tendons, synovial
afferent nerve fibers. Segmental branches make swelling, or tumors). Carpal tunnel syndrome is an
connections, along with other branches, that example of a compression-type mononeuropathy
pass directly to the anterior gray matter of the that is relatively common. It is caused by compres-
spinal cord and establish monosynaptic contact sion of the median nerve as it travels with the
with each of the LMNs that have motor units in flexor tendons through a canal made by the carpal
the muscle containing the spindle receptor. This bones and transverse carpal ligament.
produces an opposing muscle contraction. 8. Guillain-Barr syndrome is an acute immune-
Another segmental branch of the same afferent mediated polyneuropathy that is characterized by
neuron innervates an internuncial neuron that rapidly progressive limb weakness and loss of ten-
is inhibitory to motor units of antagonistic mus- don reflexes. The disorder is marked by progressive
cle groups. This disynaptic inhibitory pathway is ascending muscle weakness of the limbs, produc-
the basis for the reciprocal activity of agonist and ing a symmetric flaccid paralysis. Symptoms of
antagonist muscles (i.e., when an agonist muscle paresthesia and numbness often accompany the
is stretched, the antagonists relax). loss of motor function. Paralysis may progress to
4. Coordination of muscle movement requires that involve the respiratory muscles. Autonomic nerv-
four areas of the nervous system function in an ous system involvement that causes postural
integrated mannerthe motor system for muscle hypotension, arrhythmias, facial flushing, abnor-
strength, the cerebellar system for rhythmic move- malities of sweating, and urinary retention is com-
ment and steady posture, the vestibular system for mon. Pain is another common feature.
posture and balance, and the sensory system for 9. The primary brain abnormality found in all persons
position sense. with Parkinson disease is degeneration of the
5. Duchenne muscular dystrophy is caused by muta- nigrostriatal dopamine neurons. On microscopic
tions in a gene located on the short arm of the examination, there is loss of pigmented substan-
X chromosome that codes for a protein called dys- tia nigra neurons. Some residual nerve cells are
trophin. Dystrophin is a large cytoplasmic protein atrophic, and few contain Lewy bodies, which are
located on the inner surface of the sarcolemma or visualized as spherical, eosinophilic cytoplasmic
muscle fiber membrane. The dystrophin molecules inclusions. Although the cause of Parkinson dis-
are concentrated over the Z-bands of the muscle, ease is still unknown, it is widely believed that
where they form a strong link between the actin fila- most cases are caused by an interaction of envi-
ments of the intracellular contractile apparatus and ronmental and genetic factors. Over the past sev-
the extracellular connective tissue matrix. Abnormal- eral decades, several pathologic processes (e.g.,
ities in the dystrophin-associated protein complex oxidative stress, apoptosis, and mitochondrial dis-
compromise sarcolemma integrity, particularly with orders) that might lead to degeneration have been
sustained contractions. This disruption in integrity identified. One theory is that the auto-oxidation
may be responsible for the observed increased of catecholamines such as dopamine during
fragility of dystrophic muscle, excessive influx melanin synthesis injures neurons in the substan-
of calcium ions, and release of soluble muscle tia nigra. There is increasing evidence that the
enzymes such as creatine kinase into the serum. development of Parkinson disease may be related
The degenerative process in Duchenne muscular to oxidative metabolites of this process and the
dystrophy consists of a relentless necrosis of inability of neurons to render these products
muscle fibers, accompanied by a continuous harmless.

Copyright 2011. Wolters Kluwer Health | Lippincott Williams & Wilkins. Study Guide for Porths Essentials of Pathophysiology, Third Edition.
LWBK707-Ans_p280-402.qxd 8/20/10 6:38PM Page 373 Aptara Inc

ANSWER KEY 373

10. In amyotrophic lateral sclerosis, the death of muscle or with an interneuron that synapses with
LMNs leads to denervation, with subsequent an effector neuron.
shrinkage of musculature and muscle fiber 4. Answers: a, b, c, e
atrophy. It is this fiber atrophy, called amyotrophy, RATIONALE: These signs and symptoms include
which appears in the name of the disease. The loss changes in muscle characteristics (strength, bulk,
of nerve fibers in lateral columns of the white mat- and tone), spinal reflex activity, and motor coordi-
ter of the spinal cord, along with fibrillary gliosis, nation. Muscle innervation is incorrect.
imparts a firmness or sclerosis to this CNS tissue; 5. Answer: c
the term lateral sclerosis designates these changes. RATIONALE: The postural muscles of hip and shoul-
11. The primary neurologic injury occurs at the time der are usually the first to be affected. The other
of mechanical injury and is irreversible. It is char- answers are incorrect.
acterized by small hemorrhages in the gray matter 6. Answer: d
of the cord, followed by edematous changes in the RATIONALE: The aminoglycoside antibiotics (e.g.,
white matter that lead to necrosis of neural tissue. gentamicin) may produce a clinical disturbance
This type of pathology results from the forces of similar to botulism by preventing the release of
compression, stretch, and shear associated with acetylcholine from nerve endings. These drugs are
fracture or compression of the spinal vertebrae, particularly dangerous in persons with pre-existing
dislocation of vertebrae, and contusions due to disturbances of neuromuscular transmission,
jarring of the cord in the spinal canal. Secondary such as myasthenia gravis. The other answers are
injuries follow the primary injury and promote incorrect.
the spread of injury. Although there is considerable 7. Answer: a
debate about the pathogenesis of secondary RATIONALE: Myasthenia crisis occurs when muscle
injuries, the tissue destruction that occurs ends in weakness becomes severe enough to compromise
progressive neurologic damage. After spinal cord ventilation to the extent that ventilatory support
injury, several pathologic mechanisms come into and airway protection are needed. The other
play, including vascular damage, neuronal injury answers are incorrect.
that leads to loss of reflexes below the level of 8. Answer: b
injury, and release of vasoactive agents and cellu- RATIONALE: Carpal tunnel syndrome is an example
lar enzymes. of a compression-type mononeuropathy that is
relatively common. The other answers are not
SECTION III: APPLYING YOUR KNOWLEDGE mononeuropathies.
9. Answer: c
Activity F RATIONALE: The straight-leg test is an important
1. The medicine that we are giving your husband is diagnostic maneuver for a herniated disk in the
methylprednisolone, a short-acting corticosteroid. lumbar area. The other answers are incorrect.
In a case of spinal cord injury, the drug is thought 10. Answers: 1-a, 2-b, 3-c
to enhance the generation of impulses down the 11. Answer: d
spinal cord and improve the blood flow around the RATIONALE: The function of the striatum also
site of the injury. involves local cholinergic interneurons and their
2. destruction is thought to be related to the
1. Bed rest with log rolling only choreiform movements of Huntington disease,
2. Continuous pulse oximetry another basal ganglia-related syndrome. The other
3. Vital signs hourly until stable answers do not involve the cholinergic
4. Methylprednisolone intravenously interneurons of the striatum.
5. Monitor for gastric bleeding, venous thrombosis, 12. Answer: a
and steroid myopathy RATIONALE: In Parkinson disease, also known as
idiopathic parkinsonism, dopamine depletion
SECTION IV: PRACTICING FOR NCLEX results from degeneration of the dopamine nigros-
triatal system. The other answers are incorrect.
Activity G
13. Answer: b
1. Answer: a RATIONALE: The most common clinical
RATIONALE: The highest level of function, which presentation is slowly progressive weakness and
occurs at the level of the frontal cortex, is atrophy in distal muscles of one upper extremity.
concerned with the purpose and planning of the The other answers do not describe the clinical
motor movement. The other answers are incorrect. presentation of amyotrophic lateral sclerosis.
2. Answers: 1-b, 2-a, 3-d, 4-c 14. Answer: c
3. Answer: b RATIONALE: A large percentage of patients with
RATIONALE: The anatomic basis of a reflex consists multiple sclerosis have elevated immunoglobulin
of an afferent neuron, which synapses either G (IgG) levels, and some have oligoclonal patterns
directly with an effector neuron that innervates a

Copyright 2011. Wolters Kluwer Health | Lippincott Williams & Wilkins. Study Guide for Porths Essentials of Pathophysiology, Third Edition.
LWBK707-Ans_p280-402.qxd 8/20/10 6:38PM Page 374 Aptara Inc

374 ANSWER KEY

(i.e., discrete electrophoretic bands) even with nor- 23. ministroke, angina
mal IgG levels. 24. Thrombi
15. Answer: d 25. Lacunar
RATIONALE: A functional C7 injury allows full 26. embolic
elbow flexion and extension, wrist plantar 27. hemorrhage
flexion, and some finger control. At the C8 level, 28. cerebral artery
finger flexion is added. The other answers are 29. cerebral aneurysm
incorrect. 30. Arteriovenous
16. Answer: a 31. Encephalitis
RATIONALE: Cord injuries involving C1 to C3 32. Vomiting
result in a lack of respiratory effort, and affected 33. chemotherapy
patients require assisted ventilation. The other 34. seizure
answers involve injuries further down the spinal 35. Simple partial
column. 36. Complex partial
17. Answers: a, c, e 37. muscle contractions
RATIONALE: Autonomic dysreflexia is characterized 38. Tonic-clonic
by vasospasm, hypertension ranging from mild 39. status epilepticus
(20 mm Hg above baseline) to severe (as high as Activity B
240/120 mm Hg or higher), skin pallor, and goose-
flesh associated with the piloerector response. Anterior
Fever and vasoconstriction are not manifestations Epidural Subdural
of autonomic dysreflexia. hematoma hematoma
18. Answer: b
RATIONALE: Even though the enteric nervous system
innervation of the bowel remains intact, without
the defecation reflex, peristaltic movements are
ineffective in evacuating stool. The other answers
are incorrect.

CHAPTER 37
SECTION II: ASSESSING YOUR
UNDERSTANDING
Activity A
1. brains
2. 20
3. hypoxia
4. focal, global
5. sodium
6. Laminar necrosis
7. amino acids, proteases
8. intracranial
9. herniation Intracerebral
10. edema hematoma
Posterior
11. vasogenic
12. Cytotoxic
13. primary, cerebral hypoxia Activity C
14. Epidural hematomas
15. tear 1. f 2. g 3. d 4. j 5. b
16. Consciousness 6. c 7. e 8. h 9. a 10. i
17. brain Activity D
18. vegetative 1.
19. autoregulation
20. carbon dioxide, hydrogen ion, oxygen 5 S 1 S 3 S 7 S 2 S 6 S 4 S 8
21. Stroke
22. Ischemic, hemorrhagic

Copyright 2011. Wolters Kluwer Health | Lippincott Williams & Wilkins. Study Guide for Porths Essentials of Pathophysiology, Third Edition.
LWBK707-Ans_p280-402.qxd 8/20/10 6:38PM Page 375 Aptara Inc

ANSWER KEY 375

2. Additionally, intracellular glutamate is released


from the damaged cells. This glutamate excess
Glutamate
Glutamate then drives the uncontrolled opening of N-
methyl-D-aspartate (NMDA) receptoroperated
channels producing an increase in intracellular
NMDA receptor calcium. Excess intracellular calcium leads to a
series of calcium-mediated processes called the
calcium cascade (Fig. 37-2), including the release
Opening calcium channels of intracellular enzymes that cause protein break-
down, free radical formation, lipid peroxidation,
fragmentation of DNA, mitochondrial injury,
Calcium cascade nuclear breakdown, and eventually cell death.
4. Although recovery usually takes place within 24
hours, mild symptoms, such as headache, irritabil-
ity, insomnia, and poor concentration and mem-
Release of intracellular enzymes
Protein breakdown ory, may persist for months. The memory loss
Free radical formation usually includes an interval of time preceding the
Lipid peroxidation accident (retrograde amnesia) and following the
Fragmentation of DNA injury (anterograde amnesia).
Nuclear breakdown 5. Global brain injury, whether due to head trauma,
stroke, or other pathologies, is manifested by alter-
ations in sensory, motor, and cognitive function
Activity E and by changes in the level of consciousness. In
1. Global ischemia occurs at a time when blood flow contrast to focal injury, which causes focal neuro-
is inadequate to meet the metabolic needs of the logic deficits without altered consciousness, global
entire brain. The result is a spectrum of neuro- injury nearly always results in altered levels of
logic disorders reflecting global brain dysfunction. consciousness ranging from inattention to stupor
Unconsciousness occurs within seconds of severe or coma. Severe injury that seriously compromises
global ischemia, such as that resulting from com- brain function may result in brain death.
plete cessation of blood flow, as in cardiac arrest, 6. The two components of consciousness are (1)
or with marked decrease in blood flow, as in serious arousal and wakefulness, and (2) content and
cardiac arrhythmias. If cerebral circulation is cognition. The content and cognition aspects
restored immediately, consciousness is regained of consciousness are determined by a functioning
quickly. However, if blood flow is not promptly cerebral cortex. Arousal and wakefulness requires
restored, severe pathologic changes take place. the concurrent functioning of both cerebral hemi-
Energy sources, glucose and glycogen, are spheres and an intact reticular activating system in
exhausted in 2 to 4 minutes, and cellular ATP the brain stem. The earliest signs of diminution in
stores are depleted in 4 to 5 minutes. When level of consciousness are inattention, mild confu-
ischemia is sufficiently severe or prolonged, infarc- sion, disorientation, and blunted responsiveness.
tion or death of all the cellular elements of the With further deterioration, the delirious person
brain occurs. Even if blood flow is restored, if becomes markedly inattentive and variably lethar-
ischemic thresholds for injury were exceeded, gic or agitated. The person may progress to become
then permanent cell death ensues. obtunded and may respond only to vigorous or
2. Watershed infarcts are concentrated in anatomically noxious stimuli.
vulnerable border zones between the overlapping 7. A bilateral loss of the pupillary light response is
territories supplied by the major cerebral arteries, indicative of lesions of the brain stem. A unilateral
notably the middle, anterior, and posterior cerebral loss of the pupillary light response may be due to a
arteries. The overlapping territory at the distal lesion of the optic or oculomotor pathways. The
ends of these vessels forms extremely vulnerable oculocephalic reflex (dolls-head eye movement)
areas in terms of global ischemia, called watershed can be used to determine whether the brain stem
zones. During events such as severe hypotension, centers for eye movement are intact and the
these distal territories undergo a profound lowering oculovestibular may be used to elicit nystagmus.
of blood flow, predisposing to focal ischemia and 8. During the evolution of a stroke, there usually is a
infarction of brain tissues. Therefore, global central core of dead or dying cells, surrounded by
ischemia can result in focal infarcts that occur in an ischemic band or area of minimally perfused
the border zones between major vascular territories. cells called the penumbra. Brain cells of the penum-
3. During prolonged ischemia, the glutamate bra receive marginal blood flow, and their metabolic
transport mechanisms become immobilized, caus- activities are impaired; although the area undergoes
ing extracellular glutamate to accumulate. an electrical failure, the structural integrity of

Copyright 2011. Wolters Kluwer Health | Lippincott Williams & Wilkins. Study Guide for Porths Essentials of Pathophysiology, Third Edition.
LWBK707-Ans_p280-402.qxd 8/20/10 6:38PM Page 376 Aptara Inc

376 ANSWER KEY

the brain cells is maintained. Whether the cells of 3. Answers: a, b, d, e


the penumbra continue to survive depends on the RATIONALE: The direct brain injuries include diffuse
successful timely return of adequate circulation, axonal injury and the focal lesions of laceration,
the volume of toxic products released by the contusion, and hemorrhage. A hypoxic brain
neighboring dying cells, the degree of cerebral injury is considered a secondary type of injury.
edema, and alterations in local blood flow. If the 4. Answer: a
toxic products result in additional death of cells RATIONALE: In contrast to focal injury, which
in the penumbra, the core of dead or dying tissue causes focal neurologic deficits without altered
enlarges, and the volume of surrounding ischemic consciousness, global injury nearly always results
tissue increases. in altered levels of consciousness ranging from
9. First, blood is shunted from the high-pressure inattention to stupor or coma. The other answers
arterial system to the low-pressure venous system are manifestations of different types of brain
without the buffering advantage of the capillary injury, not a global injury.
network. The draining venous channels are 5. Answer: b
exposed to high levels of pressure, predisposing RATIONALE: Decorticate (flexion) posturing is char-
them to rupture and hemorrhage. Second, the acterized by flexion of the arms, wrists, and fin-
elevated arterial and venous pressures divert gers, with abduction of the upper extremities,
blood away from the surrounding tissue, impair- internal rotation, and plantar flexion of the lower
ing tissue perfusion. extremities. Decerebrate and extensor posturing are
10. Seizures may be caused by alterations in cell mem- the same thing and are incorrect. Diencephalon
brane permeability or distribution of ions across posturing does not exist, so it is incorrect.
the neuronal cell membranes. Another cause may 6. Answer: c
be decreased inhibition of cortical or thalamic RATIONALE: Clinical examination must disclose at
neuronal activity or structural changes that alter least the absence of responsiveness, brain stem
the excitability of neurons. Neurotransmitter reflexes, and respiratory effort. Brain death is a
imbalances such as an acetylcholine excess or clinical diagnosis, and a repeat evaluation at
-aminobutyric acid (GABA, an inhibitory neuro- least 6 hours later is recommended. An electrocar-
transmitter) deficiency have been proposed as diogram is not assessed in an examination for
causes. Certain epilepsy syndromes have been brain death.
linked to specific genetic mutations causing ion 7. Answers: a, c, e
channel defects. RATIONALE: The criteria for diagnosis of vegetative
state include the absence of awareness of self and
SECTION III: APPLYING YOUR KNOWLEDGE environment and an inability to interact with oth-
ers; the absence of sustained or reproducible
Activity F CASE STUDY
voluntary behavioral responses; lack of language
a. 1. Family history of stroke comprehension; sufficiently preserved hypothala-
2. History of hypertension mic and brain stem function to maintain life;
3. History of smoking bowel and bladder incontinence; and variably
4. History of diabetes mellitus preserved cranial nerve (e.g., pupillary, gag) and
6. History of sickle cell disease spinal cord reflexes. People in a persistent vegeta-
7. History of hyperlipidemia tive state can open their eyes and have enough
8. History of atrial fibrillation hypothalamic function to maintain life.
9. Weight 8. Answer: d
10. Alcohol and drug use RATIONALE: If blood pressure falls below 60 mm Hg,
11. Hormone replacement therapy cerebral blood flow becomes severely compromised,
12. Oral contraceptive use and, if it rises above the upper limit of autoregula-
13. Activity level tion, blood flow increases rapidly and overstretches
b. 1. Administration of tissue plasminogen activator the cerebral vessels. The other answers are
to be given within 3 hours of onset incorrect.
2. Administration of neuroprotective drugs 9. Answer: a
3. Hypothermia treatment RATIONALE: The diagnosis of subarachnoid hemor-
rhage and intracranial aneurysms is made by clini-
SECTION IV: PRACTICING FOR NCLEX cal presentation, CT scan, lumbar puncture, and
Activity G angiography. An MRI is not necessary for the diag-
nosis of subarachnoid hemorrhage and intracranial
1. Answers: 1-g, 2-c, 3-a, 4-f, 5-b, 6-e, 7-d aneurysm. Loss of cranial nerve reflexes is not
2. Answers: 1-e, 2-a, 3-b. 4-g, 5-c, 6-d, 7-f diagnostic of subarachnoid hemorrhage and
intracranial aneurysm and neither is venography.

Copyright 2011. Wolters Kluwer Health | Lippincott Williams & Wilkins. Study Guide for Porths Essentials of Pathophysiology, Third Edition.
LWBK707-Ans_p280-402.qxd 8/20/10 6:38PM Page 377 Aptara Inc

ANSWER KEY 377

10. Answer: b 14. degenerative


RATIONALE: Two assessment techniques can help 15. Congenital
determine whether meningeal irritation is present. 16. astigmatism
Kernig sign is resistance to extension of the knee 17. Accommodation
while the person is lying with the hip flexed at a 18. cataract
right angle. Brudzinski sign is elicited when 19. retina
flexion of the neck induces flexion of the hip and 20. color-blind
knee. The other answers are incorrect. 21. Retinitis pigmentosa
11. Answer: c 22. Macular
RATIONALE: Intracranial tumors give rise to focal 23. visual field
disturbances in brain function and increased ICP. 24. inferior recti, lateral recti, obliques
Focal disturbances occur because of brain compres- 25. Conjugate
sion, tumor infiltration, disturbances in blood 26. Strabismus
flow, and brain edema. Blood pressure, either 27. Amblyopia
increased or decreased, is not a manifestation of a 28. ear
brain tumor. 29. cerumen
12. Answers: 1-f, 2-e, 3-d, 4-a, 5-b, 6-c 30. Otitis externa
13. Answer: d 31. temporal
RATIONALE: The most common surgery consists of 32. eustachian
removal of the amygdala and an anterior part of 33. abnormally patent
the hippocampus and entorhinal cortex, as well as 34. Otitis media
a small part of the temporal pole, leaving the lat- 35. Acute otitis media
eral temporal neocortex intact. Only a portion of 36. Otosclerosis
the hippocampus and entorhinal cortex, and tem- 37. cochlea
poral pole are removed. 38. Sensorineural
14. Answer: a 39. bacterial meningitis
RATIONALE: Treatment consists of appropriate 40. cranial nerve VIII
life-support measures. Medications are given to 41. cytomegalovirus
control seizure activity. Intravenously administered 42. vestibular
diazepam or lorazepam is considered first-line 43. vertigo
therapy for the condition. Lorazepam is not given 44. Motion sickness
intramuscularly in status epilepticus. Cyclobenza- 45. paroxysmal positional
prine and cyproheptadine are not used to treat 46. vestibular neuronitis
status epilepticus. 47. Mnire
48. cerebellum
Activity B
CHAPTER 38
1.
SECTION II: ASSESSING YOUR Retina
Choroid
UNDERSTANDING Sclera
Superior
Activity A rectus
Bulbar and
1. eyeball Levator palpebral
palpebrae conjunctiva
2. sclera superioris
Cornea
3. levator palpebrae superioris, orbicularis oculi
Lens
4. conjunctivitis
Superior tarsal
5. Hyperacute plate
6. cornea Iris
7. Keratitis Optic Meibomian
nerve gland in
8. keratitis tarsal plate
Inferior
9. ophthalmicus rectus
Orbicularis
10. iris Inferior oculi muscle
11. uveitis oblique
muscle
12. pupils Ciliary body

13. Glaucoma

Copyright 2011. Wolters Kluwer Health | Lippincott Williams & Wilkins. Study Guide for Porths Essentials of Pathophysiology, Third Edition.
LWBK707-Ans_p280-402.qxd 8/20/10 6:38PM Page 378 Aptara Inc

378 ANSWER KEY

2.

Normal focal length

B
Hyperopia

C
Myopia

3.
Middle Inner
Cochlear
ear ear Cranial portion
nerve
Semicircular Vestibular
Tympanic VIII
canals portion
membrane

Incus

Cochlea

Eustachian
tube

Malleus
External Stapes
Auricle acoustic
meatus
Pharynx

Copyright 2011. Wolters Kluwer Health | Lippincott Williams & Wilkins. Study Guide for Porths Essentials of Pathophysiology, Third Edition.
LWBK707-Ans_p280-402.qxd 8/20/10 6:38PM Page 379 Aptara Inc

ANSWER KEY 379

Activity C trigeminal ganglion and possibly in the cornea


without causing signs of infection.
1.
5. Acanthamoeba keratitis is a rare but serious and
1. e 2. f 3. g 4. a 5. j
sight-threatening complication caused by wearing
6. c 7. d 8. b 9. i 10. h
soft contact lenses, particularly when they are
2.
worn overnight beyond doctor-recommended
1. f 2. c 3. g 4. b 5. a
periods or when poor disinfection techniques are
6. e 7. j 8. d 9. i 10. h
used. It also may occur in noncontact lens wear-
3.
ers after exposure to contaminated water or soil. It
1. j 2. h 3. e 4. c 5. i
is characterized by pain that is disproportionate to
6. a 7. g 8. d 9. b 10. f
the clinical manifestations, redness of the eye, and
Activity D photophobia.
1. The lacrimal system includes the major lacrimal 6. Changes in pupil size are controlled by contraction
gland, which produces the tears, the puncta, or relaxation of the sphincter and radial muscles
canaliculi, and tear sac, which collect the tears, of the iris. The pupillary reflex, which controls
and the nasolacrimal duct, which empties the tears the size of the pupillary opening, is controlled by
into the nasal cavity. Tears contain approximately the autonomic nervous system, with the parasym-
98% water, 1.5% sodium chloride, and small pathetic nervous system producing pupillary con-
amounts of potassium, albumin, and glucose. striction or miosis and the sympathetic nervous
The function of tears is to provide a smooth system producing pupillary dilation or mydriasis.
optical surface by abolishing minute surface The sphincter muscle that produces pupillary con-
irregularities. Tears also wet and protect the striction is innervated by postganglionic parasym-
delicate surface of the cornea and conjunctiva. pathetic neurons of the ciliary ganglion and other
They flush and remove irritating substances and scattered ganglion cells between the scleral and
microorganisms, and provide the cornea with choroid layers. The pupillary reflex is controlled
necessary nutrient substances. Tears also contain by a region in the midbrain called the pretectum.
lysozymes and immunoglobulin A (IgA), IgG, 7. Glaucoma usually results from congenital or
and IgE, which synergistically act to protect acquired lesions of the anterior segment of the eye
against infection. that mechanically obstruct aqueous outflow.
2. Chronic bacterial conjunctivitis most commonly is Primary glaucoma occurs without evidence of pre-
caused by Staphylococcus species, although other existing ocular or systemic disease. Secondary
bacteria may be involved. It is often associated glaucoma can result from inflammatory processes
with blepharitis and bacterial colonization of eye- that affect the eye, from tumors, or from blood
lid margins. The symptoms of chronic bacterial cells of trauma-produced hemorrhage that
conjunctivitis vary and can include itching, burn- obstruct the outflow of aqueous humor.
ing, foreign body sensation, and morning eyelash 8. The term presbyopia refers to decrease in accommo-
crusting. Other symptoms include flaky debris and dation that occurs because of aging. The lens con-
erythema along the lid margins, eyelash loss, and sists of transparent fibers arranged in concentric
eye redness. Some people with chronic bacterial layers, of which the external layers are the newest
conjunctivitis also have recurrent styes and and softest. No loss of lens fibers occurs with
chalazia of the lid margins. aging; instead, additional fibers are added to the
3. Trauma that causes abrasions of the cornea can outermost portion of the lens. As the lens ages,
be extremely painful, but if minor, the abrasions it thickens, and its fibers become less elastic, so
usually heal in a few days. The epithelial layer that the range of focus or accommodation is
can regenerate, and small defects heal without diminished to the point where reading glasses
scarring. If the stroma is damaged, healing become necessary for near vision.
occurs more slowly, and the danger of infection 9. Hemorrhage can be preretinal, intraretinal, or sub-
is increased. Injuries to Bowman membrane and retinal. Preretinal hemorrhages occur between the
the stromal layer heal with scar formation and retina and the vitreous. These hemorrhages are
permanent opacification. Opacities of the cornea usually large because the blood vessels are only
impair the transmission of light. A minor scar can loosely restricted; they may be associated with a
severely distort vision because it disturbs the subarachnoid or subdural hemorrhage and are
refractive surface. usually regarded as a serious manifestation of the
4. Primary epithelial infections are the optical coun- disorder. They usually reabsorb without complica-
terpart of labial herpes with similar immunologic tions unless they penetrate into the vitreous.
and pathologic features as well as a similar time Intraretinal hemorrhages occur because of abnor-
course. During childhood, mild primary herpes malities of the retinal vessels, diseases of the
simplex virus infection may go unnoticed. After blood, increased pressure in the retinal vessels, or
the initial primary infection, the virus may persist vitreous traction on the vessels. Systemic causes
in a quiescent or latent state that remains in the include diabetes mellitus, hypertension, and blood

Copyright 2011. Wolters Kluwer Health | Lippincott Williams & Wilkins. Study Guide for Porths Essentials of Pathophysiology, Third Edition.
LWBK707-Ans_p280-402.qxd 8/20/10 6:38PM Page 380 Aptara Inc

380 ANSWER KEY

dyscrasias. Subretinal hemorrhages are those that contribute to the development of tinnitus,
develop between the choroid and pigment layer sensorineural hearing loss, and vertigo.
of the retina. A common cause of subretinal hem- 15. A number of causes and conditions have been asso-
orrhage is neovascularization. Photocoagulation ciated with subjective tinnitus. Intermittent periods
may be used to treat microaneurysms and neovas- of mild, high-pitched tinnitus lasting for several
cularization. minutes are common in normal-hearing persons.
10. Proliferative diabetic retinopathy represents a Impacted cerumen is a benign cause of tinnitus,
more severe retinal change than background which resolves after the earwax is removed. Med-
retinopathy. It is characterized by formation of ications such as aspirin and stimulants such as
new fragile blood vessels (i.e., neovascularization) nicotine and caffeine can cause transient tinnitus.
at the disk and elsewhere in the retina. These ves- Conditions associated with more persistent tinnitus
sels grow in front of the retina along the posterior include noise-induced hearing loss, presbycusis
surface of the vitreous or into the vitreous. They (sensorineural hearing loss that occurs with aging),
threaten vision in two ways. First, because they are hypertension, atherosclerosis, head injury, and
abnormal, they often bleed easily, leaking blood cochlear or labyrinthine infection or inflammation.
into the vitreous cavity and decreasing visual acu- 16. Conductive hearing loss occurs when auditory
ity. Second, the blood vessels attach firmly to the stimuli are not adequately transmitted through
retinal surface and posterior surface of the the auditory canal, tympanic membrane, middle
vitreous, such that normal movement of the vitre- ear, or ossicle chain to the inner ear. Temporary
ous may exert a pull on the retina, causing retinal hearing loss can occur as the result of impacted
detachment and progressive blindness. cerumen in the outer ear or fluid in the middle ear.
11. Persistently elevated blood pressure results in the Foreign bodies, including pieces of cotton and
compensatory thickening of arteriolar walls, insects, may impair hearing. More permanent
which effectively reduces capillary perfusion pres- causes of hearing loss are thickening or damage of
sure. With severe uncontrolled hypertension, there the tympanic membrane or involvement of the
is disruption of the blood-retinal barrier, necrosis bony structures (ossicles and oval window) of the
of smooth muscle and endothelial cells, exudation middle ear due to otosclerosis or Paget disease.
of blood and lipids, and retinal ischemia. These 17. The hair cells in both utricular and saccular macu-
changes are manifested in the retina by lae are embedded in a flattened gelatinous mass,
microaneurysms, intraretinal hemorrhages, hard the otolithic membrane, which is studded with
exudates, and cotton-wool spots. tiny stones called otoliths. Although they are
12. The eustachian tube serves three basic functions: small, the density of the otoliths increases the
(1) ventilation of the middle ear, along with equal- membranes weight and its resistance to change in
ization of middle ear and ambient pressures; (2) motion. When the head is tilted, the gelatinous
protection of the middle ear from unwanted mass shifts its position because of the pull of the
nasopharyngeal sound waves and secretions; and gravitational field, bending the stereocilia of the
(3) drainage of middle ear secretions into the macular hair cells. While each hair cell becomes
nasopharynx. more or less excitable depending on the direction
13. Hearing loss, which is a common complication of in which the cilia are bending, the hair cells are
otitis media, usually is conductive and temporary oriented in all directions, making these sense
based on the duration of the effusion. Hearing loss organs sensitive to static or changing head
that is associated with fluid collection usually position in relation to the gravitational field. In a
resolves when the effusion clears. Permanent hear- condition called benign positional vertigo (to be dis-
ing loss may occur as the result of damage to the cussed), the otoliths become dislodged from their
tympanic membrane or other middle ear gelatinous base, causing positional vertigo.
structures. Cases of sensorineural hearing loss are 18. Caloric testing involves elevating the head 30
rare. Persistent and episodic conductive hearing degrees and irrigating each external auditory canal
loss in children may impair their cognitive, separately with 30 to 50 mL of ice water. The
linguistic, and emotional development. resulting changes in temperature, which are con-
14. During active bone resorption, the bone structure ducted through the petrous portion of the tempo-
appears spongy and softer than normal. The ral bone, set up convection currents in the
resorbed bone is replaced by an overgrowth of endolymph that mimic the effects of angular
new, hard, sclerotic bone. The process is slowly acceleration. In an unconscious person with a
progressive, involving more areas of the temporal functional brain stem and intact oculovestibular
bone, especially in front of and posterior to the reflexes, the eyes exhibit a jerk nystagmus lasting
stapes footplate. As it invades the footplate, the 2 to 3 minutes, with the slow component toward
pathologic bone increasingly immobilizes the the irrigated ear followed by rapid movement
stapes, reducing the transmission of sound. away from the ear (see Fig. 37-10). With
Pressure of otosclerotic bone on middle ear struc- impairment of brain stem function, the response
tures or the vestibulocochlear nerve (CN VIII) may becomes perverted and eventually disappears.

Copyright 2011. Wolters Kluwer Health | Lippincott Williams & Wilkins. Study Guide for Porths Essentials of Pathophysiology, Third Edition.
LWBK707-Ans_p280-402.qxd 8/20/10 6:38PM Page 381 Aptara Inc

ANSWER KEY 381

8. Answer: b
SECTION III: APPLYING YOUR KNOWLEDGE RATIONALE: Age-related cataracts, which are the
Activity E CASE STUDY most common type, are characterized by increas-
1. ingly blurred vision and visual distortion. The
a. Ophthalmoscopic examination under anesthesia other answers are incorrect.
by an ophthalmologist; CT or MRI scans are 9. Answer: c
used to evaluate the extent intraocular disease RATIONALE: With the loss of gel structure, fine
and extraocular spread. fibers, membranes, and cellular debris develop.
b. Laser thermotherapy, cryotherapy, chemotherapy, When this occurs, floaters (images) can often be
and nucleation noticed as these substances move within the vitre-
2. ous cavity during head movement. Blind spots,
a. Aminoglycosides, antimalarial drugs, chemo- meshlike structures, and red spots are not seen
therapeutic drugs, loop diuretics, and salicylates. during head movement with a loss of the gel
structure of the vitreous humor.
10. Answer: d
SECTION IV: PRACTICING FOR NCLEX
RATIONALE: Neovascularization occurs in many
Activity F conditions that impair retinal blood flow, includ-
1. Answer: b ing stasis because of hyperviscosity of blood or
RATIONALE: The symptoms include tearing and dis- decreased flow, vascular occlusion, sickle cell
charge, pain, swelling, and tenderness. The other disease, sarcoidosis, diabetes mellitus, and
answers are incorrect. retinopathy of prematurity. The other answers
2. Answer: c are incorrect.
RATIONALE: Infection should be suspected when 11. Answer: a
conjunctivitis develops 48 hours after birth. The RATIONALE: Nonexudative age-related macular
other answers are not correct. degeneration is characterized by various degrees
3. Answer: c of atrophy and degeneration of the outer retina,
RATIONALE: The treatment of herpes simplex virus Bruch membrane, and the choriocapillary layer of
keratitis focuses on eliminating viral replication the choroid. It does not involve leakage of blood
within the cornea while minimizing the damaging or serum; hence, it is called dry age-related mac-
effects of the inflammatory process. The other ular degeneration. The other answers are charac-
answers are not goals in the treatment of herpes terizations of the wet form of macular
simplex virus keratitis. degeneration.
4. Answers: b, c, d 12. Answer: b
RATIONALE: The low rejection rate is due to several RATIONALE: Crude analysis of visual stimulation
factors: the cornea is avascular, including lymphat- at reflex levels, such as eye-orienting and head-
ics, thereby limiting perfusion by immune orienting responses to bright moving lights, pupil-
elements; major histocompatibility complexes lary reflexes, and blinking at sudden bright lights,
(class II) are virtually absent in the cornea; may be retained even though vision has been lost.
antigen-presenting cells are not present in great The other answers are incorrect.
numbers; the cornea secretes immunosuppressive 13. Answer: c
factors, and corneal cells secrete substances (e.g., RATIONALE: Paralytic strabismus is uncommon
Fas ligand) that protect against apoptosis, thereby in children but accounts for nearly all cases of
minimizing inflammation. The other answers are adult strabismus. It can be caused by infiltrative
incorrect. processes including: Graves disease, myasthenia
5. Answer: d gravis, stroke, and direct optical trauma. The
RATIONALE: Miotic drugs (e.g., pilocarpine), which other diseases have nothing to do with adult
are used in the treatment of angle-closure glaucoma strabismus.
(to be discussed), produce pupil constriction and, 14. Answer: b
in that manner, facilitate aqueous humor circula- RATIONALE: The reversibility of amblyopia depends
tion. The other answers are classes of drugs that do on the maturity of the visual system at the time
not affect papillary constriction. of onset and the duration of the abnormal experi-
6. Answer: a ence. The other answers are incorrect.
RATIONALE: Primary open-angle glaucoma usually 15. Answer: a
occurs because of an abnormality of the trabecular RATIONALE: The most common bacterial pathogens
meshwork that controls the flow of aqueous are gram-negative rods (Pseudomonas aeruginosa,
humor into the canal of Schlemm. The other proteus sp) and fungi (Aspergillus) that grow in the
answers are incorrect. presence of excess moisture. The other answers are
7. Answers: 1-d, 2-b, 3-a, 4-c, 5-e not fungi.

Copyright 2011. Wolters Kluwer Health | Lippincott Williams & Wilkins. Study Guide for Porths Essentials of Pathophysiology, Third Edition.
LWBK707-Ans_p280-402.qxd 8/20/10 6:38PM Page 382 Aptara Inc

382 ANSWER KEY

16. Answer: a 25. Answer: b


RATIONALE: The abnormally patent tube does not RATIONALE: The disorder first reduces the ability to
close or does not close completely. In infants and understand speech and, later, the ability to detect,
children with an abnormally patent tube, air and identify, and localize sounds.
secretions often are pumped into the eustachian
tube during crying and nose blowing. Cerumen
and saliva are not let into the eustachian tube. CHAPTER 39
17. Answers: a, b, d
RATIONALE: Risk factors include premature birth, SECTION II: ASSESSING YOUR
male gender, ethnicity (Native American, Inuit), fam- UNDERSTANDING
ily history of recurrent otitis media, presence of sib-
Activity A
lings in the household, genetic syndromes, and low
socioeconomic status. Being an only child and being 1. Hypospadias, epispadias
a female are not risk factors for acute otitis media. 2. Peyronie disease
18. Answer: c 3. Peyronie
RATIONALE: Because much of the conductive hear- 4. parasympathetic, sympathetic
ing loss associated with otosclerosis is caused by 5. nitric oxide, arterial
stapedial fixation, surgical treatment involves 6. Erectile dysfunction
stapedectomy with stapedial reconstruction using 7. cardiovascular, metabolic
the patients own stapes or a stapedial prosthesis. 8. Priapism
The other answers are incorrect. 9. penile cancer
19. Answer: d 10. cryptorchidism
RATIONALE: The scala vestibuli and scala media are 11. motility
separated from each other by the vestibular mem- 12. Epididymitis
brane, also known as Reissner membrane. The 13. Prostatitis
other answers are incorrect. 14. acute bacterial prostatitis
20. Answer: a 15. prostate
RATIONALE: In some vascular disorders, for 16. chronic bacterial prostatitis
example, sounds generated by turbulent blood 17. extraprostatic
flow (e.g., arterial bruits or venous hums) are con- 18. prostate-specific antigen (PSA)
ducted to the auditory system. Vascular disorders Activity B
typically produce a pulsatile form of tinnitus. The
other answers are incorrect. A
21. Answer: b
RATIONALE: More permanent causes of hearing loss
Deep dorsal vein
are thickening or damage of the tympanic
Dorsal artery Cavernous
membrane or involvement of the bony structures
Dorsal nerve nerve
(ossicles and oval window) of the middle ear due
(somatic) (autonomic)
to otosclerosis or Paget disease. Huntington,
Alzheimer, and Parkinson disease are not
associated with conductive hearing loss.
22. Answer: c
RATIONALE: Acoustic neuromas are benign Schwann Circumflex
cell tumors affecting CN VIII. The other answers artery and vein
are incorrect.
B Circumflex vein Subtunical
23. Answer: d venular plexus
RATIONALE: Tuning forks are used to differentiate
conductive and sensorineural hearing loss. Audio-
scope, audiometer, and tone analysis do not differ- Deep dorsal vein
entiate between conductive and sensorineural
Tunica albuginea
hearing loss.
24. Answer: a Corpora cavernosa
RATIONALE: Genetic causes are probably responsible Sinusoidal spaces
for as much as 50% of sensorineural hearing loss Cavernous artery
in children. The other diseases are not the correct
answer.

Copyright 2011. Wolters Kluwer Health | Lippincott Williams & Wilkins. Study Guide for Porths Essentials of Pathophysiology, Third Edition.
LWBK707-Ans_p280-402.qxd 8/20/10 6:38PM Page 383 Aptara Inc

ANSWER KEY 383

Activity C an increase in prostatic size, and gives rise to symp-


toms such as a weak urinary stream, postvoid drib-
1. a 2. c 3. h 4. d 5. b
bling, frequency of urination, and nocturia. The
6. e 7. f 8. j 9. g 10. i
dynamic component of BPH is related to prostatic
Activity D smooth muscle tone. 1-Adrenergic receptors are the
1. Neurogenic disorders such as Parkinson disease, main receptors for the smooth muscle component
stroke, and cerebral trauma often contribute to of the prostate.
erectile dysfunction by decreasing libido or 6. The diagnosis of prostate cancer is based on history
preventing the initiation of erection. In spinal and physical examination and confirmed through
cord injury, the extent of neural impairment biopsy methods. Transrectal ultrasonography is
depends on the level, location, and extent of the used to guide a biopsy needle and document the
lesion. Hormonal causes of erectile dysfunction exact location of the biopsied tissue. Radiologic
include a decrease in androgen levels due to examination of the bones of the skull, ribs, spine,
both primary and secondary hypogonadism. Com- and pelvis can be used to reveal metastases. PSA
mon risk factors for generalized penile arterial insuf- levels are important in the staging and management
ficiency include hypertension, hyperlipidemia, of prostatic cancer. In untreated cases, the level
cigarette smoking, diabetes mellitus, and pelvic of PSA correlates with the volume and stage of
irradiation. disease.
2. Sildenafil (Viagra) is a selective inhibitor of
phosphodiesterase type 5, the enzyme that inacti- SECTION III: APPLYING YOUR KNOWLEDGE
vates cyclic guanosine monophosphate. This acts
Activity E
by facilitating corporeal smooth muscle relaxation
in response to sexual stimulation. 1. The cause of penile cancer is not known, although
3. Testicular torsion is a twisting of the spermatic cord several risk factors are thought to be linked to this
that suspends the testis. Extravaginal torsion, cancer.
which occurs almost exclusively in neonates, is the 2. Research data has shown that the most important
less common form. It occurs when the testicle and prognostic indicator is the status of your lymph
the fascial tunicae that surround it rotate around nodes. The more lymph nodes that are involved,
the spermatic cord at a level well above the tunica the more advanced your cancer has become. It is
vaginalis. The torsion probably occurs during fetal important that you ask your physician what your
or neonatal descent of the testes before the tunica prognosis is.
adheres to the scrotal wall. Intravaginal torsion is
considerably more common than extravaginal tor- SECTION IV: PRACTICING FOR NCLEX
sion. It occurs when the testis rotates on the long Activity F
axis in the tunica vaginalis. In most cases, congeni-
tal abnormalities of the tunica vaginalis or 1. Answers: a, c, e
RATIONALE: Factors that influence the timing of
spermatic cord exist. The tunica vaginalis normally
surrounds the testes and epididymis, allowing the surgical repair include anesthetic risk, penile size,
testicle to rotate freely in the tunica. Patients and the psychological effects of the surgery on the
usually present in severe distress within hours of child. In mild cases, the surgery is done for
onset and often have nausea, vomiting, and tachy- cosmetic reasons only. In more severe cases, repair
cardia. The affected testis is large and tender, with becomes essential for normal sexual functioning
pain radiating to the inguinal area. and to prevent the psychological sequelae of hav-
4. The clinical staging for testicular cancer is as follows: ing malformed genitalia. Testicular involvement
stage I, tumor confined to testes, epididymis, or and presence of an abdominal hernia have no
spermatic cord; stage II, tumor spread to retroperi- bearing on the timing of the surgery.
toneal lymph nodes below the diaphragm; and 2. Answer: a
RATIONALE: Peyronie disease involves a localized
stage III, metastases outside the retroperitoneal
nodes or above the diaphragm. and progressive fibrosis of unknown origin that
5. The anatomic location of the prostate at the affects the tunica albuginea (i.e., the tough, fibrous
bladder neck contributes to the pathophysiology sheath that surrounds the corpora cavernosa) of
and symptomatology of benign prostatic hyperpla- the penis. The disorder is characterized initially
sia (BPH). There are two prostatic components to by an inflammatory process that results in dense
the obstructive properties of BPH and development fibrous plaque formation. The plaque usually is on
of lower urinary tract symptoms: dynamic and the dorsal midline of the shaft, causing upward
static. The static component of BPH is related to bowing of the shaft during erection. The other
answers are incorrect.

Copyright 2011. Wolters Kluwer Health | Lippincott Williams & Wilkins. Study Guide for Porths Essentials of Pathophysiology, Third Edition.
LWBK707-Ans_p280-402.qxd 8/20/10 6:38PM Page 384 Aptara Inc

384 ANSWER KEY

3. Answer: b dysuria, and urethral discharge. Dull, aching pain


RATIONALE: Priapism can occur at any age, in the often is present in the perineum, rectum, or
newborn as well as other age groups. Sickle cell sacrococcygeal region. The other answers are
disease or neoplasms are the most common cause incorrect.
in boys between 5 and 10 years of age. Hemophilia 10. Answer: d
and hypospadias are not linked to priapism in any RATIONALE: The incidence of BPH increases with
age group. advanced age and is highest in African Americans
4. Answer: c and lowest in native Japanese. The other answers
RATIONALE: The treatment goals for the boys with are incorrect.
cryptorchidism include measures to enhance
future fertility potential, placement of the gonad
in a favorable place for cancer detection, and CHAPTER 40
improved cosmetic appearance. The other answers
are incorrect. SECTION II: ASSESSING YOUR
5. Answer: d UNDERSTANDING
RATIONALE: Hydroceles are palpated as cystic
Activity A
masses that may attain massive proportions. If
there is enough fluid, the mass may be mistaken 1. hair-covered
for a solid tumor. Transillumination of the 2. vulva
scrotum (i.e., shining a light through the scrotum 3. Bartholin gland cyst
for the purposes of visualizing its internal 4. Chronic dermatitis
structures) or ultrasonography can help to 5. ecology
determine whether the mass is solid or cystic and 6. Vaginitis
whether the testicle is normal. The other answers 7. bleeding
are incorrect. 8. metaplasia
6. Answer: a 9. Cervical
RATIONALE: The most common acute scrotal disor- 10. cervicitis
der in the pediatric population is testicular 11. Polyps
torsion. The other answers are incorrect. 12. Human papilloma virus (HPV)
7. Answer: b 13. Endometriosis
RATIONALE: Sexually transmitted acute epididymitis 14. Adenomyosis
occurs mainly in young men without underlying 15. Dilatation, curettage
genitourinary disease and is most commonly 16. leiomyomas
caused by Chlamydia trachomatis and Neisseria gon- 17. Pelvic inflammatory disease
orrhoeae. Candida albicans and Escherichia coli are 18. Ectopic pregnancy
not the most common causes of epididymitis in 19. menstrual, fertility
young men without underlying genitourinary 20. Polycystic ovary
disease. 21. luteinizing hormone
8. Answers: a, b, c 22. Ovarian
RATIONALE: Signs of metastatic spread include 23. ovulatory age
swelling of the lower extremities, back pain, 24. tumor suppressor
neck mass, cough, hemoptysis, or dizziness. 25. Rectocele
Gynecomastia (breast enlargement) may result 26. cardinal
from human chorionic gonadotropin-producing 27. uterine prolapse
tumors and occurs in about 5% of men with germ 28. amenorrhea
cell tumors. The other answers are not signs of 29. Galactorrhea
metastatic spread of a testicular cancer. 30. Mastitis
9. Answer: c 31. Fibroadenomas
RATIONALE: The manifestations of acute bacterial 32. Fibrocystic
prostatitis include fever and chills, malaise, myal- 33. Paget
gia, arthralgia, frequent and urgent urination, 34. Infertility

Copyright 2011. Wolters Kluwer Health | Lippincott Williams & Wilkins. Study Guide for Porths Essentials of Pathophysiology, Third Edition.
LWBK707-Ans_p280-402.qxd 8/20/10 6:38PM Page 385 Aptara Inc

ANSWER KEY 385

Activity B

Umbilicus

Ovary

Small bowel

Colon
Fallopian tube

Uterine serosa
Rectovaginal septum
and uterosacral
Peritoneum ligaments

Bladder

Uterovesical fold

Activity C 2. A quadrivalent vaccine (Gardasil) to prevent infec-


tion with the HPV subtypes 16, 18, 6, and 11 was
1.
licensed by the Food and Drug Administration
1. i 2. h 3. c 4. b 5. d
(FDA) in June 2006. The vaccine targets the two
6. f 7. e 8. i 9. a 10. g
strains of HPV (HPV 16 and 18), which are respon-
2.
sible for 70% of cervical cancer, and the two most
1. d 2. f 3. b 4. a 5. g
common benign strains (HPV 6 and 11), which
6. c 7. e
account for up to 90% of genital warts. The target
Activity D population for the vaccine is females between the
1. The prevention and treatment of vaginal infections ages of 9 and 26 years, optimally prior to initiating
depend on proper health habits and accurate diag- sexual activity. Clinical studies provided to the FDA
nosis and treatment of ongoing infections (see have confirmed that the vaccine appears safe and
Chapter 41). Measures to prevent infection include effective in inducing long-term immunity to HPV.
measures that keep the genital area clean and dry, 3. Diagnosis of cervical cancer requires pathologic con-
maintenance of normal vaginal flora and healthy firmation. Pap smear results demonstrating a squa-
vaginal mucosa, and avoidance of contact with mous intraepithelial lesion often require further
organisms known to cause vaginal infections. evaluation by colposcopy, during which a biopsy
Perfumed products, such as feminine deodorant sample may be obtained from suspect areas and
sprays, douches, bath powders, soaps, and even examined microscopically. An alternate diagnostic
toilet paper, can be irritating and may alter the nor- tool in areas where colposcopy is not readily avail-
mal vaginal flora. Tight clothing prevents the dissi- able is a noninvasive photographic technique, in
pation of body heat and evaporation of skin which a cervicography camera is used to take pho-
moisture and promotes favorable conditions for tographs of the cervix. The projected cervicogram
irritation and the growth of pathogens. Cotton (a slide made from the film) is then sent for expert
undergarments that can withstand hot water and evaluation. In one study, the cervicogram was found
bleach (a fungicide) may be preferable for women to give a greater yield of cervical intraepithelial neo-
to prevent such infections. plasia (CIN) than Pap smear alone in patients with
previous abnormal pap smears.

Copyright 2011. Wolters Kluwer Health | Lippincott Williams & Wilkins. Study Guide for Porths Essentials of Pathophysiology, Third Edition.
LWBK707-Ans_p280-402.qxd 8/20/10 6:38PM Page 386 Aptara Inc

386 ANSWER KEY

4. The first theory, the regurgitation/implantation from the failure of any of the developing ovarian
theory, suggests that menstrual blood containing follicles to mature to the point of ovulation, with
fragments of endometrium is forced upward the subsequent formation of the corpus luteum and
through the fallopian tubes into the peritoneal cav- production and secretion of progesterone.
ity. Retrograde menstruation is not an uncommon 9. Approximately 5% to 10% of all breast cancers are
phenomenon, and it is unknown why endometrial hereditary, with genetic mutations causing up to 80%
cells implant and grow in some women but not in of breast cancers in women under age 50. Two breast
others. A second theory, the metaplastic theory, pro- cancer susceptibility genesBRCA1 on chromosome
poses that dormant, immature cellular elements, 17 and BRCA2 on chromosome 13may account for
spread over a wide area during embryonic develop- most inherited forms of breast cancer (see Chapter 7).
ment, persist into adult life and then differentiate BRCA1 is known to be involved in tumor sup-
into endometrial tissue. A third theory, the vascular pression. A woman with known mutations in
or lymphatic theory, suggests that the endometrial BRCA1 has a lifetime risk of 60% to 85% for breast
tissue may metastasize through the lymphatics or cancer and an increased risk of ovarian cancer. BRCA2
vascular system. Genetic and immune factors also is another susceptibility gene that carries an elevated
have been studied as contributing factors to the cancer risk similar to that of BRCA1.
development of endometriosis.
5. The organisms ascend through the endocervical SECTION III: APPLYING YOUR KNOWLEDGE
canal to the endometrial cavity, and then to the
Activity E
tubes and ovaries. The endocervical canal is slightly
dilated during menstruation, allowing bacteria to 1. A colposcopy is the examination of the vagina and
gain entrance to the uterus and other pelvic struc- cervix with an optical magnifying instrument. It is
tures. After entering the upper reproductive tract, usually done after a Pap smear shows abnormal cells.
the organisms multiply rapidly in the favorable 2. The LEEP procedure uses a thin, rigid, wire loop
environment of the sloughing endometrium and that is attached to a generator. It blends high-
ascend to the fallopian tube. frequency, low-voltage current for cutting with a
6. There is also concern that women with PCOS who higher voltage current for coagulation. The wire
are anovulatory do not produce significant amounts loop allows the physician to remove the entire
of progesterone. This may, in turn, subject the uter- transformation zone of the cervix. This removes the
ine lining to an unopposed estrogen environment, entire lesion while providing a specimen for further
which is a significant risk factor for development histologic evaluation. The procedure is done under
of endometrial cancer. Although is there also a local anesthesia in the physicians office at a lower
reported association with breast cancer and ovarian cost than a cone biopsy, which is done in the hos-
cancer, PCOS has not been conclusively shown to pital or outpatient surgery clinic.
be an independent risk factor for either malignancy.
7. The uterus and the pelvic structures are maintained SECTION IV: PRACTICING FOR NCLEX
in proper position by the uterosacral ligaments, Activity F
round ligaments, broad ligament, and cardinal liga-
ments. The two cardinal ligaments maintain the 1. Answer: a
cervix in its normal position (Fig. 40-13A). The RATIONALE: Surgical treatment of a Bartholin cyst
uterosacral ligaments hold the uterus in a forward that has abscessed or blocks the introitus is called
position and the broad ligaments suspend the marsupialization, a procedure that involves
uterus, fallopian tubes, and ovaries in the pelvis removal of a wedge of vulvar skin and the cyst
(see Fig. 40-4). The vagina is encased in the semirigid wall. The other answers are incorrect.
structure of the strong supporting fascia. The 2. Answer: b
muscular floor of the pelvis is a strong, slinglike RATIONALE: One-third to one-half of vulvar intraep-
structure that supports the uterus, vagina, urinary ithelial neoplasia (VIN) cases appear to be caused
bladder, and rectum. by the cancer-promoting potential of certain strains
8. Dysfunctional menstrual cycles are related to alter- (subtypes 16 and 18) of HPV that are sexually
ations in the hormones that support normal cyclic transmitted and are associated with the type of
endometrial changes. Estrogen deprivation causes vulvar cancer found in younger women. The other
retrogression of a previously built-up endometrium answers are not thought to be associated with vul-
and bleeding. Such bleeding often is irregular in var cancer in younger women.
amount and duration, with the flow varying with 3. Answers: a, b, c
the time and degree of estrogen stimulation and RATIONALE: In premenarchal girls, most vaginal
with the degree of estrogen withdrawal. A lack of infections have nonspecific causes, such as poor
progesterone can cause abnormal menstrual bleed- hygiene, intestinal parasites, or the presence of
ing; in its absence, estrogen induces development foreign bodies. Vaginal deodorants and Tampax
of a much thicker endometrial layer with a richer are not associated with vaginal infections in
blood supply. The absence of progesterone results premenarchal girls.

Copyright 2011. Wolters Kluwer Health | Lippincott Williams & Wilkins. Study Guide for Porths Essentials of Pathophysiology, Third Edition.
LWBK707-Ans_p280-402.qxd 8/20/10 6:38PM Page 387 Aptara Inc

ANSWER KEY 387

4. Answer: c gas and an increased appetite are not highly corre-


RATIONALE: Blockage of the mucosal glands results lated with ovarian cancer.
in trapping of mucus in the deeper glands leading 11. Answer: a
to the formation of dilated cysts within the cervix, RATIONALE: A pessary may be inserted to hold the
called nabothian cysts. The other answers are uterus in place and may stave off surgical interven-
incorrect. tion in women who want to have children or in
5. Answer: d older women for whom the surgery may pose a sig-
RATIONALE: Risk factors for endometriosis may nificant health risk. The other answers are incorrect.
include early menarche; regular periods with shorter 12. Answer: b
cycles (less than 27 days), longer duration (greater RATIONALE: Although analgesic agents such as aspirin
than 7 days), or heavier flow; increased menstrual and acetaminophen may relieve minor uterine
pain; and other first-degree relatives with the con- cramping or low back pain, prostaglandin synthetase
dition. Late menarche, light flow, and periods inhibitors (e.g., ibuprofen, naproxen, mefenamic
shorter than 7 days are not risk factors for acid, indomethacin) are more specific for dysmenor-
endometriosis. rhea and the treatment of choice, if contraception
6. Answer: a is not desired. Metformic acid is incorrect.
RATIONALE: Leiomyomas are asymptomatic approx- 13. Answer: c
imately half of the time and may be discovered RATIONALE: Treatment for mastitis symptoms
during a routine pelvic examination, or they may include application of heat or cold, excision, aspi-
cause menorrhagia (excessive menstrual bleeding), ration, mild analgesics, antibiotics, and a support-
anemia, urinary frequency, rectal pressure/consti- ive brassiere or breast binder. The other answers
pation, abdominal distention, and, infrequently, are incorrect.
pain. Diarrhea and urinary retention are not 14. Answer: d
symptoms of leiomyomas. RATIONALE: Diagnosis of fibrocystic changes is
7. Answer: b made by physical examination, mammography,
RATIONALE: New-onset breakthrough bleeding in ultrasonography, and biopsy (i.e., aspiration or tis-
women who are taking oral contraceptives or sue sample). Client history and galactography are
medroxyprogesterone contraceptive injection not used to diagnose fibrocystic changes in the
(Depo-Provera) has been associated with pelvic breast.
inflammatory disease. The other forms of contra- 15. Answer: a
ception have not been associated with pelvic RATIONALE: Postmenopausal women and women
inflammatory disease. who have had a hysterectomy can perform the
8. Answer: c examination any day of the month. The other
RATIONALE: Diagnostic tests for ectopic pregnancy answers are incorrect.
include a urine pregnancy test, ultrasonography, 16. Answer: b
and
-hCG (hormone produced by placental cells) RATIONALE: The specimen is best collected by mas-
levels. Serial
-hCG tests may detect lower-than- turbation into a sterile container after 3 days of
normal hCG production. Transvaginal ultrasound abstinence. The other answers are incorrect.
studies after 5 weeks gestation may demonstrate 17. Answer: c
an empty uterine cavity or presence of the gesta- RATIONALE: Between 12 and 24 hours after insemi-
tional sac outside the uterus. In a comparison nation, the ova are evaluated for signs of fertiliza-
of various protocols for diagnosing ectopic preg- tion. If signs are present, the ova are returned to
nancy, ultrasound followed by serial hCG levels the incubator, and 48 to 72 hours after egg
was found to yield the best results. The other retrieval, the fertilized eggs are placed into the
answers are incorrect. womans uterus by means of a transcervical
9. Answer: d catheter. The other answers are incorrect.
RATIONALE: Metformin, an insulin-sensitizing drug,
used with or without ovulation-inducing medica-
tions, is emerging as an important component of CHAPTER 41
PCOS treatment. DHEAS is often found in the
blood of women with PCOS; Methotrexate is used SECTION II: ASSESSING YOUR
in ectopic pregnancies; Spironolactone, an UNDERSTANDING
antimineralocorticoid, is used in treating PCOS,
Activity A
not mineralocorticoids.
10. Answers: a, c, e 1. mucocutaneous
RATIONALE: Symptoms that are believed to have a 2. fetus or newborn
strong correlation to ovarian cancer include 3. Genital warts
abdominal or pelvic pain, increased abdominal 4. external genitalia
size or bloating, and difficulty eating or feeling full 5. Genital herpes
quickly after ingesting food. Increased intestinal 6. neurotropic

Copyright 2011. Wolters Kluwer Health | Lippincott Williams & Wilkins. Study Guide for Porths Essentials of Pathophysiology, Third Edition.
LWBK707-Ans_p280-402.qxd 8/20/10 6:38PM Page 388 Aptara Inc

388 ANSWER KEY

7. type-2 the spread of other organisms, carrying pathogens


8. primary genital herpes attached to their surface into the fallopian tubes. In
9. yeast men, it is a common cause of nongonococcal urethri-
10. Trichomonads tis and is a risk factor for infertility, altering sperm
11. Bacterial motility and viability. It has also been associated
12. Chlamydiae with chronic prostatitis.
13. fallopian tube 5. Men are more likely to be symptomatic than
14. gonococcus women. In men, the initial symptoms include ure-
15. Syphilis thral pain and a creamy yellow, sometimes bloody,
Activity B discharge. The disorder may become chronic and
affect the prostate, epididymis, and periurethral
1. j 2. h 3. a 4. d 5. g
glands. Rectal infections are common in homosex-
6. f 7. i 8. b 9. e 10. c
ual men. In women, recognizable symptoms
Activity C include unusual genital or urinary discharge,
1. Risk factors for acquiring HPV include young age dysuria, dyspareunia, pelvic pain or tenderness,
(25 years), early age of first intercourse (16 unusual vaginal bleeding (including bleeding after
years), increasing numbers of sex partners, and hav- intercourse), fever, and proctitis. Symptoms may
ing a male partner with multiple sex partners. The occur or increase during or immediately after
HPV infection can occur with any type of vaginal menses because the bacterium is an intracellular
or anal penetration and is common in men having diplococcus that thrives in menstrual blood but
sex with men and women having sex with women. cannot survive long outside the human body. There
Oral-genital and manual-genital contact are less may be infections of the uterus and development of
likely means of spreading this infection. acute or chronic infection of the fallopian tubes,
2. Herpes simplex virus type is transmitted by contact with ultimate scarring and sterility.
with infectious lesions or secretions. Herpes simplex 6. The clinical disease is divided into three stages: pri-
virus type-1 is transmitted by oral secretions, and mary, secondary, and tertiary. Primary syphilis is
infections frequently occur in childhood. Herpes characterized by the appearance of a chancre at the
simplex virus type-1 may be spread to the genital site of exposure. These lesions usually are painless
area by autoinoculation after poor hand washing and located at the site of sexual contact. The timing
or through oral-genital contact. Herpes simplex of the second stage of syphilis varies even more than
virus type-2 usually is transmitted by sexual contact that of the first, lasting from 1 week to 6 months.
but can be passed to an infant during childbirth The symptoms of a rash, fever, sore throat, stomati-
if the virus is actively being shed from the genital tis, nausea, loss of appetite, and inflamed eyes may
tract. In genital herpes, the virus ascends through come and go for a year but usually last for 3 to
the peripheral nerves to the sacral dorsal root 6 months. Secondary manifestations may include
ganglia. The virus can remain dormant in the dor- alopecia and genital condylomata latum. Condylo-
sal root ganglia or it can reactivate, in which case mata latum are elevated, red-brown lesions that
the viral particles are transported back down the may ulcerate and produce a foul discharge. They
nerve root to the skin, where they multiply and are 2 to 3 cm in diameter; they contain many spiro-
cause a lesion to develop. chetes and are highly infectious. Tertiary syphilis is
3. Reported risk factors for the overgrowth of a delayed response of the untreated disease. It can
Candida albicans include recent antibiotic therapy, occur as long as 20 years after the initial infection.
which suppresses the normal protective bacterial When syphilis does progress to the symptomatic
flora; high hormone levels owing to pregnancy tertiary stage, it commonly takes one of three
or the use of oral contraceptives, which cause forms: development of localized destructive lesions
an increase in vaginal glycogen stores; and called gummas, development of cardiovascular
uncontrolled diabetes mellitus or human immun- lesions, or development of central nervous system
odeficiency virus (HIV) infection, because they lesions.
compromise the immune system.
4. It is a risk factor for HIV transmission and infectivity SECTION III: APPLYING YOUR KNOWLEDGE
in both men and women. In women, it increases the
Activity D
risk of tubal infertility and atypical pelvic inflamma-
tory disease (see Chapter 40), and it is associated 1. The nurse, while taking the nursing history, would
with adverse outcomes such as premature birth in find it important to note the following: urethral
pregnant women. Trichomonads attach easily to itching, meatal erythema and tenderness, urethral
mucous membrane. They may serve as vectors for discharge, history of sexual relations with someone

Copyright 2011. Wolters Kluwer Health | Lippincott Williams & Wilkins. Study Guide for Porths Essentials of Pathophysiology, Third Edition.
LWBK707-Ans_p280-402.qxd 8/20/10 6:38PM Page 389 Aptara Inc

ANSWER KEY 389

being treated for a chlamydial infection, and atypical pelvic inflammatory disease, and it is asso-
history of recent conjunctivitis. ciated with adverse outcomes such as premature
2. The expected treatment for chlamydial infection birth in pregnant women. The other answers are
includes: pharmacologic treatment with either incorrect.
azithromycin or doxycycline, simultaneous 8. Answer: c
treatment of both sexual partners, and abstinence RATIONALE: The predominant symptom of bacterial
from sexual activity to facilitate cure. vaginosis is a thin, grayish-white discharge that
has a foul, fishy odor. The other answers are
SECTION IV: PRACTICING FOR NCLEX incorrect.
9. Answers: a, b, e
Activity E
RATIONALE: A specimen should be collected from
1. Answer: a the appropriate site (i.e., endocervix, urethra, anal
RATIONALE: The incubation period for HPV-induced canal, or oropharynx), inoculated onto a suitable
genital warts ranges from 6 weeks to 8 months, medium, and transported under appropriate con-
with a mean of 2 to 3 months. The other answers ditions. The nasal passages and the exocervix are
are incorrect. not sites that would be used for the collection of
2. Answers: a, c, e Neisseria gonorrhoeae.
RATIONALE: The initial symptoms of primary geni- 10. Answer: d
tal herpes infections include tingling, itching, RATIONALE: The syphilitic gumma is a peculiar, rub-
and pain in the genital area, followed by eruption bery, necrotic lesion that is caused by noninflam-
of small pustules and vesicles. Chancres and matory tissue necrosis. Gummas can occur singly
eczemalike lesions are not indicative of genital or multiply and vary in size from microscopic
herpes. lesions to large, tumorous masses. They most com-
3. Answer: b monly are found in the liver, testes, and bone.
RATIONALE: The antiviral drugs acyclovir, Chancres occur in primary syphilis. Chancroid is
valacyclovir, and famciclovir have become the cor- an STD. Gummies are candy.
nerstone for management of genital herpes. The
other drugs are not used in the treatment of geni-
tal herpes. CHAPTER 42
4. Answer: c
RATIONALE: The organism has shown resistance to SECTION II: ASSESSING YOUR
treatment with sulfamethoxazole alone and to UNDERSTANDING
tetracycline. The Centers for Disease Control
and Prevention recommends treatment with Activity A
azithromycin, erythromycin, or ceftriaxone. The 1. muscles, tendons, ligaments
other answers are incorrect. 2. calcium, blood
5. Answer: d 3. axial, appendicular
RATIONALE: An important characteristic of 4. Compact
lymphogranuloma venereum is the early 5. Bones
(1 to 4 weeks later) development of large, tender, 6. periosteum
and sometimes fluctuant inguinal lymph nodes 7. marrow
called buboes. 8. nutritional arteries
6. Answer: a 9. connective, calcium
RATIONALE: Antifungal agents such as clotrimazole, 10. osteoprogenitor
miconazole, butaconazole, and terconazole, in var- 11. Osteoclasts
ious forms, are effective in treating candidiasis. 12. Elastic
These drugs, with the exception of terconazole, are 13. Fibrocartilage
available without prescription for use by women 14. Hyaline
who have had a previously confirmed diagnosis of 15. Calcitonin
candidiasis. 16. Tendons
7. Answer: b 17. Ligaments
RATIONALE: Trichomoniasis can cause a number of 18. Synarthroses
complications. It is a risk factor for HIV transmis- 19. Synovial
sion and infectivity in both men and women. In 20. bursa
women, it increases the risk of tubal infertility and

Copyright 2011. Wolters Kluwer Health | Lippincott Williams & Wilkins. Study Guide for Porths Essentials of Pathophysiology, Third Edition.
LWBK707-Ans_p280-402.qxd 8/20/10 6:38PM Page 390 Aptara Inc

390 ANSWER KEY

Activity B Activity E
1. A typical long bone has a shaft, or diaphysis, and
Compact bone Proximal epiphysis
two ends, called epiphyses. Long bones usually are
Yellow
Epiphyseal narrow in the midportion and broad at the ends so
marrow
line that the weight they bear can be distributed over a
wider surface. The shaft of a long bone is formed
Medullary
cavity mainly of compact bone roughly hollowed out to
form a marrow-filled medullary canal. The ends of
long bones are covered with articular cartilage.
Periosteum 2. Red bone marrow contains developing red blood
cells and is the site of blood cell formation. Yellow
bone marrow is composed largely of adipose cells.
At birth, nearly all of the marrow is red and
A hematopoietically active. As the need for red blood
cell production decreases during postnatal growth,
Nutrient
artery red marrow is gradually replaced with yellow bone
Compact bone marrow in most of the bones. In the adult, red mar-
Spongy bone
row persists in the vertebrae, ribs, sternum, and ilia.
3. The intercellular matrix is composed of two types of
substances: organic matter and inorganic salts. The
organic matter, including bone cells, blood vessels,
and nerves, constitutes approximately one third of
the dry weight of bone; the inorganic salts make up
B C the other two-thirds. The organic matter consists pri-
marily of collagen fibers embedded in an amorphous
Epiphyseal ground substance. The inorganic matter consists of
line
hydroxyapatite, an insoluble macrocrystalline struc-
Distal epiphysis
ture of calcium phosphate salts, and small amounts
of calcium carbonate and calcium fluoride.
Activity C 4. Both of these types of connective tissue consist of
1. j 2. f 3. b 4. d 5. e living cells, nonliving intercellular fibers, and an
6. i 7. a 8. c 9. g 10. h amorphous (shapeless) ground substance. The
Activity D tissue cells are responsible for secreting and main-
taining the intercellular substances in which they
are housed. However, cartilage consists of more
Parathyroid extracellular substance than bone, and fibers are
glands
embedded in a firm gel rather a calcified cement
Kidney substance. Hence, cartilage has the flexibility of a
Reabsorption
firm plastic material rather than the rigid character-
of calcium
istics of bone.
Bone
5. Parathyroid hormone maintains serum calcium
Release of
calcium and levels by initiation of calcium release from bone,
phosphate by conservation of calcium by the kidney, by
enhanced intestinal absorption of calcium through
Calcium
activation of vitamin D, and by reduction of serum
concentration
in extracellular phosphate levels. Parathyroid hormone also
fluid increases the movement of calcium and phosphate
from bone into the extracellular fluid.
Urinary excretion 6. The most potent of the vitamin D metabolites is
of phosphate 1,25-(OH)2D3. This metabolite increases intestinal
absorption of calcium and promotes the actions of
Activation of parathyroid hormone on resorption of calcium and
vitamin D phosphate from bone. Bone resorption by the
osteoclasts is increased and bone formation by the
Intestine osteoblasts is decreased; there is also an increase in
Reabsorption of acid phosphatase and a decrease in alkaline phos-
calcium via activated phatase. Intestinal absorption and bone resorption
vitamin D increase the amount of calcium and phosphorus
available to the mineralizing surface of the bone.

Copyright 2011. Wolters Kluwer Health | Lippincott Williams & Wilkins. Study Guide for Porths Essentials of Pathophysiology, Third Edition.
LWBK707-Ans_p280-402.qxd 8/20/10 6:38PM Page 391 Aptara Inc

ANSWER KEY 391

7. Answer: b
SECTION III: APPLYING YOUR KNOWLEDGE RATIONALE: Synchondroses are joints in which
Activity F CASE STUDY bones are connected by hyaline cartilage and have
1. Tendinitis occurs because of overuse of the tendon, limited motion. The other answers are incorrect.
which causes inflammation of the tendon. 8. Answer: c
b. Some tendons are enclosed in sheaths, and they RATIONALE: Diarthrodial joints are the joints most
slide inside the sheath and are cushioned by frequently affected by rheumatic disorders. The
synovial fluid. Other tendons are not encased in other types of joint are not the ones most
a sheath. All tendons attach muscles to bone frequently affected by rheumatic disorders.
and they do not stretch very much. 9. Answer: d
RATIONALE: The tendons and ligaments of the joint
capsule are sensitive to position and movement,
SECTION IV: PRACTICING FOR NCLEX
particularly stretching and twisting. The other
Activity G answers are incorrect.
1. Answer: a 10. Answer: a
RATIONALE: The metaphysis is composed of bony RATIONALE: These sacs, or bursae, contain synovial
trabeculae that have cores of cartilage. The other fluid. Their purpose is to prevent friction on a ten-
answers are incorrect. don. Bursae do not prevent injury to a joint, nor
2. Answer: b do they cushion joints. Bursae do not prevent fric-
RATIONALE: Yellow bone marrow is composed tion on a ligament.
largely of adipose cells. Hematopoietic cells are in
red bone marrow. Cancellous cells are in spongy
bone. Osteogenic cells line the latticelike pattern
CHAPTER 43
that forms bone marrow.
3. Answer: c SECTION II: ASSESSING YOUR
RATIONALE: Lamellar bone is composed largely of UNDERSTANDING
cylindrical units called osteons or haversian Activity A
systems. Hematopoietic cells, spicules, and macro-
1. musculoskeletal
crystalline cells do not make up lamellar bone.
2. falls
4. Answer: d
3. Soft-tissue
RATIONALE: Fibrocartilage is found in the interver-
4. strain
tebral disks, in areas where tendons are connected
5. sprain
to bone, and in the symphysis pubis. The other
6. dislocation
answers are incorrect.
7. Loose
5. Answers: a, b, c
8. Rotator cuff
RATIONALE: Parathyroid hormone maintains serum
9. rotational
calcium levels by initiation of calcium release from
10. Dislocations
bone, by conservation of calcium by the kidney,
11. sudden injury, pathologic
by enhanced intestinal absorption of calcium
12. pathologic
through activation of vitamin D, and by reduction
13. fracture
of serum phosphate levels.
14. Traction
6. Answer: a
15. Fracture blisters
RATIONALE: The most potent of the vitamin D
16. thromboemboli
metabolites is 1,25(OH)2D3. This metabolite
17. fat embolism
increases intestinal absorption of calcium and pro-
18. Tuberculosis
motes the actions of parathyroid hormone on
19. Osteonecrosis
resorption of calcium and phosphate from bone.
20. osteosarcoma
None of the other answers are correct.
21. Benign
22. chondroma
23. Chondrosarcoma

Copyright 2011. Wolters Kluwer Health | Lippincott Williams & Wilkins. Study Guide for Porths Essentials of Pathophysiology, Third Edition.
LWBK707-Ans_p280-402.qxd 8/20/10 6:38PM Page 392 Aptara Inc

392 ANSWER KEY

Activity B occurs within the first 2 weeks, and although tensile


strength increases steadily thereafter, it is not suffi-
cient to permit strong tendon pulls for 6 to 8 weeks.
There is a danger that muscle contraction will pull
Proximal the injured ends apart, causing the tendon to heal
in the lengthened position. There is also a danger
that adhesions will develop in areas where tendons
Midshaft
pass through fibrous channels, such as in the distal
palm of the hands, rendering the tendon useless.
3. Motion of the arm involves the coordinated move-
Distal ment of muscles of the rotator cuff (supraspinous,
teres minor, infraspinatus, and subscapularis) and
Spiral
their musculotendinous attachments. These muscles
Transverse Oblique
are separated from the overlying coracoacromial
arch by two bursae, the subdeltoid and subcoracoid.
These two bursae, sometimes referred to as the sub-
acromial bursa, often communicate and are affected
by lesions of the rotator cuff. Rotator cuff injuries
and impingement disorders can result from a num-
ber of causes, including excessive use, a direct blow,
or stretch injury, usually involving throwing or
swinging, as with baseball pitchers or tennis players.
Overuse and degenerative disorders have a slower
Impacted onset and are seen in older persons with minor or
no trauma. The tendons of the rotator cuff fuse
Comminuted Segmental Butterfly
together near their insertions into the tuberosities of
the humerus to form the musculotendinous cuff.
Activity C 4. A hip fracture is generally a fracture of the proximal
femur. Such fractures are commonly categorized
1. a 2. e 3. i 4. c 5. j according to the anatomic site in which they occur.
6. f 7. g 8. d 9. h 10. b Femoral-neck fractures are located in the area distal
Activity D to the femoral head but proximal to the greater and
1. lesser trochanters and are considered intracapsular
because they are located within the capsule of the
a S c S b S d hip joint. Intertrochanteric fractures occur in the
metaphyseal region between the greater and lesser
trochanter. Subtrochanteric fractures are those that
occur just below the greater trochanter. Femoral-
Activity E neck and intertrochanteric fractures account for
1. Any joint may be sprained, but the ankle joint is over 90% of hip fractures, occurring in approxi-
most commonly involved, especially in fast-moving mately equal proportions.
injuries in which an ankle or knee can be suddenly 5. The compartment syndrome has been described as
twisted. Most ankle sprains occur in the lateral ankle a condition of increased pressure within a limited
when the foot is turned inward under a person, forc- space (e.g., abdominal and limb compartments)
ing the ankle into inversion beyond the structural that compromises the circulation and function
limits. Other common sites of sprain are the knee of the tissues within the space. The muscles and
(the collateral ligament and anterior cruciate liga- nerves of an extremity are enclosed in a tough,
ment) and elbow (the ulnar side). As with a strain, inelastic fascial envelope called a muscle compart-
the soft-tissue injury that occurs with a sprain is not ment. If the pressure in the compartment is suffi-
evident on the radiograph. Wrist sprains most often ciently high, tissue circulation is compromised,
occur with a fall on an outstretched hand. causing death of nerve and muscle cells. Permanent
2. If properly treated, injuries usually heal with the loss of function may occur. The amount of
restoration of the original tensile strength. Repair is pressure required to produce a compartment syn-
accomplished by fibroblasts from the inner tendon drome depends on many factors, including the
sheath or, if the tendon has no sheath, from the duration of the pressure elevation, the metabolic
loose connective tissue that surrounds the tendon. rate of the tissues, vascular tone, and local blood
Capillaries infiltrate the injured area during the ini- pressure. Compartment syndrome can result from
tial healing process and supply the fibroblasts with a decrease in compartment size, an increase in the
the materials they need to produce large amounts volume of its contents, or a combination of the
of collagen. Formation of the long collagen bundles two factors.

Copyright 2011. Wolters Kluwer Health | Lippincott Williams & Wilkins. Study Guide for Porths Essentials of Pathophysiology, Third Edition.
LWBK707-Ans_p280-402.qxd 8/20/10 6:38PM Page 393 Aptara Inc

ANSWER KEY 393

6. Osteomyelitis after trauma or bone surgery usually 3. An arthroscopic meniscectomy may be performed if
is associated with persistent or recurrent fevers, there is recurrent or persistent locking, recurrent
increased pain at the operative or trauma site, and fluid buildup in the knee, or disabling pain.
poor incisional healing, which often is accompanied
by continued wound drainage and wound separation. SECTION IV: PRACTICING FOR NCLEX
Prosthetic joint infections often present with joint
pain, fever, and cutaneous drainage.
Activity G
7. The pathologic features of bone necrosis are the 1. Answer: a
same, regardless of cause. The site of the lesion is RATIONALE: Overuse injuries have been described as
related to the vessels involved. There is necrosis of chronic injuries, including stress fractures that
cancellous bone and marrow. The cortex usually is result from constant high levels of physiologic
not involved because of collateral blood flow. In stress without sufficient recovery time. They com-
subchondral necrosis, a triangular or wedge-shaped monly occur in the elbow (Little League elbow
segment of tissue that has the subchondral bone or tennis elbow) and in tissue in which tendons
plate as its base and the center of the epiphysis as its attach to the bone, such as the heel, knee, and
apex, undergoes necrosis. When medullary infarcts shoulder. The other answers are incorrect.
occur in fatty bone marrow, the death of bone cells 2. Answers: 1-g, 2-d, 3-b, 4-f, 5-c, 6-a, 7-e
causes calcium release and necrosis of fat cells, with 3. Answers: b, c, d
the formation of free fatty acids. Released calcium RATIONALE: Conservative treatment with anti-
forms an insoluble soap with free fatty acids. inflammatory agents, corticosteroid injections, and
Because bone lacks mechanisms for resolving the physical therapy often is undertaken. A period of
infarct, the lesions remain for life. rest is followed by a customized exercise and reha-
8. There are three major manifestations of bone bilitation program to improve strength, flexibility,
tumors: pain, presence of a mass, and impairment and endurance. Pain medicine and anesthetic
of function. Pain is a feature common to almost all injections are not usually prescribed for conserva-
malignant tumors, but may or may not occur with tive treatment of a shoulder or rotator cuff injury.
benign tumors. A mass or hard lump may be the 4. Answer: b
first sign of a bone tumor. A malignant tumor is RATIONALE: Hip dislocation is an emergency. In the
suspected when a painful mass exists that is enlarg- dislocated position, great tension is placed on the
ing or eroding the cortex of the bone. The ease of blood supply to the femoral head, and avascular
discovery of a mass depends on the location of the necrosis may result. Pain caused by a dislocated
tumor; a small lump arising on the surface of the hip is not considered an emergency. The longer the
tibia is easy to detect, whereas a tumor that is deep hip is dislocated, the more time it takes to heal and
in the medial portion of the thigh may grow to a remain in place, but this does not make the situation
considerable size before it is noticed. Benign and an emergency, and dislocation of the hip does not
malignant tumors may cause the bone to erode to interrupt the blood supply to the femoral head.
the point at which it cannot withstand the strain of 5. Answer: b
ordinary use. In such cases, even a small amount of RATIONALE: Various growth factors, such as bone
bone stress or trauma precipitates a pathologic frac- morphologic protein, are thought to induce bone
ture. A tumor may produce pressure on a peripheral formation and repair bone defects. The other
nerve, causing decreased sensation, numbness, a answers are not used to induce healing in fractures.
limp, or limitation of movement. 6. Answers: b, c, d
9. Metastatic lesions are seen most often in the spine, RATIONALE: A thorough history includes the mech-
femur, pelvis, ribs, sternum, proximal humerus, and anism, time, and place of the injury; first recogni-
skull, and are less common in anatomic sites that tion of symptoms; and any treatment initiated. It
are further removed from the trunk of the body is unimportant if anyone else in the family is prone
that are secondary tumors. Tumors that frequently to fractures. It is also unimportant what the patient
spread to the skeletal system are those of the breast, has eaten. If surgery were indicated than it would be
lung, prostate, kidney, and thyroid, although any important to find out if the patient has eaten.
cancer can ultimately involve the skeleton. More 7. Answers: 1-a, 2-c, 3-b
than 85% of bone metastases result from primary 8. Answer: c
lesions in the breast, lung, or prostate. RATIONALE: The main clinical features of fat emboli
syndrome are respiratory failure, cerebral dysfunc-
SECTION III: APPLYING YOUR KNOWLEDGE tion, and skin and mucosal petechiae. Cerebral
manifestations include encephalopathy, seizures,
Activity F CASE STUDY and focal neurologic deficits unrelated to head
1. Magnetic resonance imaging of the injured knee. injury. The other answers are incorrect.
2. Place the knee in a removable knee immobilizer 9. Answer: d
and prescribe isometric quadriceps exercises. RATIONALE: Intravenous therapy is usually needed
for up to 6 weeks. Initial antibiotic therapy is

Copyright 2011. Wolters Kluwer Health | Lippincott Williams & Wilkins. Study Guide for Porths Essentials of Pathophysiology, Third Edition.
LWBK707-Ans_p280-402.qxd 8/20/10 6:38PM Page 394 Aptara Inc

394 ANSWER KEY

followed by surgery to remove foreign bodies Activity B


(e.g., metal plates, screws) or sequestra and by
long-term antibiotic therapy. The other answers
are incorrect. Bone Osteophyte
10. Answer: a cysts
RATIONALE: Any bone, joint, or bursa may be
affected, but the spine is the most common site,
followed by the knees and hips. The ankles and
shoulders are not common sites for tuberculosis to
be found.
11. Answer: b
RATIONALE: Besides fracture, the most common
causes of bone necrosis are idiopathic (i.e., those
of unknown cause) and prior steroid therapy. Ves-
sel injury, radiation therapy, and embolism can
cause osteonecrosis, but not as commonly as the
others mentioned.
12. Answer: c
Erosion of
RATIONALE: The primary clinical feature of
Joint space cartilage and
osteosarcoma is deep localized pain with narrows bone
nighttime awakening and swelling in the affected
bone. In osteosarcoma, the pain is worse at night.
There may be erythema in the overlaying skin, but
that is not the primary clinical feature of the Activity C
disease. Osteosarcoma does not cause soreness in
1. c 2. e 3. f 4. j 5. a
the nearest joint; it may impede range of motion.
6. b 7. d 8. g 9. h 10. i
13. Answers: a, c, e
RATIONALE: The primary goals in treatment of Activity D
metastatic bone disease are to prevent pathologic 1.
fractures and promote survival with maximum
functioning, allowing the person to maintain as e S g S d S a S b S f S c
much mobility and pain control as possible. Cure
of the disease and preventing bone segment
ischemia are not primary goals of treatment in Activity E
metastatic bone disease. 1. The pathogenesis of rheumatoid arthritis (RA) can
be viewed as an aberrant immune response that
leads to synovial inflammation and destruction of
CHAPTER 44 the joint architecture. It has been suggested that
the disease is initiated by the activation of helper
SECTION II: ASSESSING YOUR T cells, release of cytokines, and antibody formation.
UNDERSTANDING Approximately 70% to 80% of those with the dis-
ease have a substance called the rheumatoid factor
Activity A
(RF), which is an autologous (self-produced) anti-
1. rheumatoid arthritis body (Ig RF) that reacts with a fragment of immuno-
2. T-cellmediated globulin G (IgG) to form immune complexes.
3. chronic inflammatory Immune complexes (IgG RF  IgG) and comple-
4. scleroderma ment components are found in the synovium,
5. Ankylosing spondylitis synovial fluid, and extra-articular lesions of persons
6. arthropathies with RA.
7. Reiter syndrome 2. At the cellular level, neutrophils, macrophages,
8. enteropathic and lymphocytes are attracted to the area. The neu-
9. Osteoarthritis trophils and macrophages phagocytize the immune
10. wear and tear complexes and, in the process, release lysosomal
11. Gout enzymes capable of causing destructive changes in
12. Juvenile idiopathic arthritis the joint cartilage. The inflammatory response that
13. systemic lupus erythematosus follows attracts additional inflammatory cells, set-
14. dermatomyositis ting into motion a chain of events that perpetuates
15. Arthritis the condition. As the inflammatory process
16. Osteoarthritis progresses, the synovial cells and subsynovial
17. Polymyalgia rheumatica

Copyright 2011. Wolters Kluwer Health | Lippincott Williams & Wilkins. Study Guide for Porths Essentials of Pathophysiology, Third Edition.
LWBK707-Ans_p280-402.qxd 8/20/10 6:38PM Page 395 Aptara Inc

ANSWER KEY 395

tissues undergo reactive hyperplasia. Vasodilation


and increased blood flow cause warmth and redness.
SECTION IV: PRACTICING FOR NCLEX
The joint swelling that occurs is the result of the Activity G
increased capillary permeability that accompanies 1. Answers: c, e, d, i, f, g, a, h, b
the inflammatory process. RATIONALE: The role of the autoimmune process in
3. Arthralgias and arthritis are among the most the joint destruction of RA remains obscure. At
commonly occurring early symptoms of SLE. The the cellular level, neutrophils, macrophages, and
polyarthritis of SLE initially can be confused with lymphocytes are attracted to the area. The neu-
other forms of arthritis, especially rheumatoid trophils and macrophages phagocytize the immune
arthritis, because of the symmetric arthropathy. complexes and, in the process, release lysosomal
Flexion contractures, hyperextension of the enzymes capable of causing destructive changes
interphalangeal joint, and subluxation of the car- in the joint cartilage. The inflammatory response
pometacarpal joint contribute to the deformity that follows attracts additional inflammatory cells,
and subsequent loss of function in the hands. setting into motion a chain of events that perpetu-
Other musculoskeletal manifestations of SLE ates the condition. As the inflammatory process
include tenosynovitis, rupture of the intrapatellar progresses, the synovial cells and subsynovial tis-
and Achilles tendons, and avascular necrosis, sues undergo reactive hyperplasia. Vasodilation
frequently of the femoral head. and increased blood flow cause warmth and redness.
4. The joint changes associated with osteoarthritis, The joint swelling that occurs is the result of the
which include a progressive loss of articular increased capillary permeability that accompanies
cartilage and synovitis, result from the the inflammatory process.
inflammation caused when cartilage attempts to 2. Answer: a
repair itself, creating osteophytes or spurs. These RATIONALE: Arthralgias and arthritis are among the
changes are accompanied by joint pain, stiffness, most commonly occurring early symptoms of SLE.
limitation of motion, and, in some cases, joint The other answers are symptoms of SLE in differ-
instability and deformity. ing stages of the disease.
5. The pathogenesis of gout resides in an elevation 3. Answers: c, d
of the serum uric acid levels. Uric acid is the end RATIONALE: Almost all persons with scleroderma
product of purine (adenine and guanine from develop polyarthritis and Raynaud phenomenon,
DNA and RNA) metabolism. The elevation of uric a vascular disorder characterized by reversible
acid and the subsequent development of gout vasospasm of the arteries supplying the fingers.
can result from overproduction of purines, Dumping syndrome, chronic diarrhea, and
decreased salvage of free purine bases, augmented chronic vasoconstriction are not diseases
breakdown of nucleic acids because of increased developed by people with scleroderma.
cell turnover, or decreased urinary excretion of 4. Answer: b
uric acid. Primary gout, which constitutes 90% of RATIONALE: Corticosteroids are the mainstay of
cases, may be a consequence of enzyme defects, treatment for these conditions. The other drug
which result in an overproduction of uric acid, types are not the treatment of choice for
inadequate elimination of uric acid by the kidney, polymyositis and dermatomyositis.
or a combination of the two. In secondary gout, 5. Answer: c
the hyperuricemia may be caused by increased RATIONALE: The pain, which becomes worse when
breakdown in the production of nucleic acids, resting, particularly when lying in bed, initially
as occurs with rapid tumor cell lysis during treat- may be blamed on muscle strain or spasm from
ment for lymphoma or leukemia. Other cases physical activity. The other answers are incorrect.
of secondary gout result from chronic renal 6. Answer: d
disease. RATIONALE: Reiter syndrome was the first rheumatic
disease to be recognized in association with HIV
SECTION III: APPLYING YOUR KNOWLEDGE infection. Symptoms of arthritis may precede any
Activity F CASE STUDY overt signs of HIV disease. The other sexually
transmitted diseases have not been associated with
1. Juvenile idiopathic arthritis can be regarded not as
Reiter syndrome.
a single disease, but as a category of diseases with
7. Answer: a
three principle types of onset: (1) systemic onset
RATIONALE: The biologic response modifiers,
disease, (2) pauciarticular arthritis, and (3)
specifically the tumor necrosis factor inhibitors
polyarticular disease.
(e.g., etanercept, infliximab, and adalimumab)
2. The nurse would expect blood work for rheumatoid
have been found to be beneficial in controlling the
factor and a complete metabolic panel along with a
arthritis as well as the psoriasis in patients with
complete blood count to be ordered.
psoriatic arthritis. The other drugs have not been
found to be beneficial in psoriatic arthritis.

Copyright 2011. Wolters Kluwer Health | Lippincott Williams & Wilkins. Study Guide for Porths Essentials of Pathophysiology, Third Edition.
LWBK707-Ans_p280-402.qxd 8/20/10 6:38PM Page 396 Aptara Inc

396 ANSWER KEY

8. Answers: a, c, e CHAPTER 45
RATIONALE: The joint changes associated with
osteoarthritis, which include a progressive loss of
SECTION II: ASSESSING YOUR
articular cartilage and synovitis, result from the
inflammation caused when cartilage attempts to
UNDERSTANDING
repair itself, creating osteophytes or spurs. These Activity A
changes are accompanied by joint pain, stiffness, 1. integumentum
limitation of motion, and, in some cases, joint 2. thickness
instability and deformity. The other answers are 3. epidermis
incorrect. 4. stratum corneum
9. Answer: b 5. Keratinocytes
RATIONALE: A definitive diagnosis of gout can be 6. Melanocytes
made only when monosodium urate crystals are in 7. eumelanin
the synovial fluid or in tissue sections of 8. Langerhans
tophaceous deposits. The other answers are not 9. basement membrane
diagnostic of gout. 10. epidermis
10. Answer: c 11. dermis
RATIONALE: In terms of medications, the selection 12. papillary
of drugs used in the treatment of arthritic dis- 13. Eccrine
orders and their dosages may need to be considered 14. Apocrine
when prescribing for the elderly. For example, the 15. keratinized
NSAIDs may be less well tolerated by the elderly, 16. keratinized
and their side effects are more likely to be serious. 17. blister
In addition to bleeding from the gastrointestinal 18. callus
tract and renal insufficiency, there may be cogni- 19. Corns
tive dysfunction, manifested by forgetfulness, 20. xerosis
inability to concentrate, sleeplessness, paranoid
ideation, and depression. Malaise, lethargy, and Activity B
mania are not side effects of NSAIDs. 1.

Epidermis lifted to reveal


papillae of the dermis

Papillae

Dermis

Arrector pili muscle

Blood vessel

Sebaceous gland

Subcutaneous tissue

Nerve endings

Nerve to hair follicle


Sweat gland

Copyright 2011. Wolters Kluwer Health | Lippincott Williams & Wilkins. Study Guide for Porths Essentials of Pathophysiology, Third Edition.
LWBK707-Ans_p280-402.qxd 8/20/10 6:38PM Page 397 Aptara Inc

ANSWER KEY 397

2. 4. Dark-skinned and light-skinned people have


approximately the same number of melanocytes,
but the production and packaging of pigment is
Hair shaft
different. In dark-skinned people, larger melanin-
Arrector pili muscle containing melanosomes are produced and trans-
ferred individually to the keratinocyte; whereas,
in light-skinned people, smaller melanosomes are
Epidermis
produced and then packaged together in a mem-
Sebaceous brane before being transferred to the keratinocyte.
gland Although the number of melanosomes in dark
Keratinized and white skin is the same, black skin produces
cells more melanin and produces it faster than white
Dermis skin.
Hair 5. The sebaceous glands are located over the entire
follicle skin surface except for the palms, soles, and sides
Hair of the feet. They are part of the pilosebaceous unit.
papilla They secrete a mixture of lipids, including triglyc-
erides, cholesterol, and wax. This mixture is called
Dermal
blood sebum; it lubricates hair and skin. Sebum is not the
vessels same as the surface lipid film. Sebum prevents
undue evaporation of moisture from the stratum
corneum during cold weather and helps to
conserve body heat. Sebum production is under the
Activity C
control of genetic and hormonal influences.
1. d 2. j 3. i 4. c 5. a 6. It is generally agreed that itch is a sensation that
6. h 7. f 8. g 9. b 10. e originates in free nerve endings in the skin, is car-
Activity D ried by small myelinated type C nerve fibers to the
1. The skin serves several other vital functions, dorsal horn of the spinal cord, and is then trans-
including somatosensory function, temperature mitted to the somatosensory cortex via the
regulation, and vitamin D synthesis. The skin is spinothalamic tract, differing from the pain path-
richly innervated with pain, temperature, and touch ways.
receptors. Skin receptors relay the numerous qualities
of touch, such as pressure, sharpness, dullness, and SECTION III: APPLYING YOUR KNOWLEDGE
pleasure to the central nervous system for localiza- Activity E CASE STUDY
tion and fine discrimination. The rate at which
a. Skin color is determined by the melanin produced
heat is dissipated from the body is determined by
by the melanocytes. Black skin produces more
constriction or dilation of the arterioles that supply
melanin and produces it faster than white skin.
blood to the skin and through evaporation of mois-
Because of their skin color, dark-skinned persons
ture and sweat from the skin surface. Vitamin D3 is
are better protected against skin cancer, premature
the most important of these and is formed in the
wrinkling, and aging of the skin that occurs with
skin as the result of irradiation of 7-dehydrocholes-
sun exposure. Dry or ashy skin also can be a
terol, a substance normally found in the skin, by
problem for people with dark skin. It often is
ultraviolet rays from the sun.
uncomfortable, and it also is easily noticed because
2. The epidermis is composed of stratified squamous
it gives the skin an ashen or grayish appearance.
keratinized epithelium, which, when viewed under
The darker pigmentation can make skin pallor,
the microscope, is seen to consist of five distinct
cyanosis, and erythema more difficult to observe.
layers, or strata, that represent a progressive differ-
Changes in skin color; in particular hypopigmenta-
entiation or maturation of the keratinocytes: the
tion and hyperpigmentation, often accompany dis-
stratum germinativum, or basal layer; the stratum
orders of dark skin and are very important signs to
spinosum; the stratum granulosum; the stratum
observe when diagnosing skin conditions.
lucidum; and the stratum corneum.
3. The ability to synthesize melanin depends on the
ability of the melanocytes to produce an enzyme
SECTION IV: PRACTICING FOR NCLEX
called tyrosinase, which converts the amino acid Activity F
tyrosine to a precursor of melanin. A genetic lack of 1. Answer: a
this enzyme results in a clinical condition called RATIONALE: The skin also serves as an immunologic
albinism. Persons with this disorder lack pigmenta- barrier. The Langerhans cells detect foreign anti-
tion in the skin, hair, and iris of the eye. gens, playing an important part in allergic skin

Copyright 2011. Wolters Kluwer Health | Lippincott Williams & Wilkins. Study Guide for Porths Essentials of Pathophysiology, Third Edition.
LWBK707-Ans_p280-402.qxd 8/20/10 6:38PM Page 398 Aptara Inc

398 ANSWER KEY

conditions and skin graft rejections. The other CHAPTER 46


cell types are not part of the immunologic
barrier.
2. Answers: 1-c, 2-b, 3-a, 4-d
SECTION II: ASSESSING YOUR
3. Answer: b UNDERSTANDING
RATIONALE: The basement membrane is also a Activity A
major site of immunoglobulin and complement 1. Primary
deposition in skin disease. The other answers are 2. Melanin
incorrect. 3. Fungi
4. Answer: c 4. Candidiasis
RATIONALE: The reticular dermis (pars reticularis) is
5. bacterial
the thicker area of the dermis and forms the bulk 6. Impetigo
of the dermal layer. The other three answers are 7. Cellulitis
part of the reticular dermis but it is not character- 8. Verrucae plantaris, verrucae palmaris
ized by them. 9. zoster
5. Answers: a, c, e 10. Acne
RATIONALE: Because the eccrine glands and deep
11. Sebum
hair follicles extend to this layer and several skin 12. comedones, blackheads, whiteheads
diseases involve the subcutaneous tissue, the sub- 13. Inflammatory
cutaneous tissue may be considered part of the 14. dermatoses
skin. The keratinocytes and the Merkel cells are 15. Allergic contact
not part of the subcutaneous tissue. 16. urticaria
6. Answer: d 17. Topical, systemic
RATIONALE: This mixture is called sebum; it
18. Papulosquamous
lubricates hair and skin. Sweat comes from sweat 19. Lichen planus
glands. Chalasia is an abnormal relaxation or the 20. repeated
cardiac sphincter of the stomach. Cerumen is 21. scabies
found in the ear. 22. UV-B rays
7. Answer: a 23. photosensitive
RATIONALE: The nearly transparent nail plate
24. Sunburn
provides a useful window for viewing the amount 25. Burns
of oxygen in the blood, providing a view of the 26. Burn
color of the blood in the dermal vessels. The other 27. ischemic
answers are incorrect. 28. dysplastic nevus
8. Answer: b 29. melanocytes
RATIONALE: When a blister occurs, histologically,
30. melanoma
there is degeneration of the epidermal cells and a 31. Basal cell carcinoma
disruption of the intercellular junctions that 32. Squamous cell carcinomas
causes the layers of the skin to separate. Lichenifi- 33. birthmarks
cations are thickened areas of the skin. Petechiae 34. Hemangiomas
are pinpoint rashes on the skin. A pressure ulcer 35. Port-wine stains
is sometimes referred to as a bed sore and is 36. Irritant diaper dermatitis
caused by continuous pressure on a bony promi- 37. Prickly heat
nence. 38. Cradle cap
9. Answer: c
RATIONALE: Given these new findings, it has been Activity B
postulated that itch exists both locally and 1.
centrally, that, in addition to localized itch, an 1. b 2. e 3. a 4. g 5. f
itch center exists in the somatosensory cortex. 6. h 7. d 8. c 9. j 10. i
The other answers are incorrect. 2.
10. Answer: d 1. h 2. a 3. j 4. e 5. d
RATIONALE: Moisturizing agents are the cornerstone 6. g 7. b 8. f 9. c 10. i
of treatment for dry skin. These agents exert their Activity C
effects by repairing the skin barrier, increasing the
1. The most common type is recessively inherited
water content of the skin, reducing transepidermal
oculocutaneous albinism, in which there are a
water loss, and restoring the lipid barriers ability
normal number of melanocytes, but they lack
to attract, hold, and redistribute water. The other
tyrosinase, the enzyme needed for synthesis of
answers are incorrect.

Copyright 2011. Wolters Kluwer Health | Lippincott Williams & Wilkins. Study Guide for Porths Essentials of Pathophysiology, Third Edition.
LWBK707-Ans_p280-402.qxd 8/20/10 6:38PM Page 399 Aptara Inc

ANSWER KEY 399

melanin. Individuals have pale or pink skin, white Individuals who are immunocompromised may
or yellow hair, and light-colored or sometimes have severe attacks.
pink eyes. Persons with albinism have ocular prob- 6. Factors that are believed to contribute to the
lems, such as extreme sensitivity to light, development of acne are (1) increased sebum
refractive errors, lack of stereopsis, and nystagmus. production, (2) increased proliferation of the kera-
2. The fungi that cause superficial mycoses live on tinizing epidermal cells that form the sebaceous
the dead keratinized cells of the epidermis. They cells, (3) the colonization and proliferation of Pro-
emit an enzyme that enables them to digest pionibacterium acnes, and (4) inflammation.
keratin, which results in superficial skin scaling, 7. Atopic dermatitis (atopic eczema) is an itchy,
nail disintegration, or hair breakage, depending on inflammatory skin disorder that is characterized by
the location of the infection. Deeper reactions poorly defined erythema with edema, vesicles, and
involving vesicles, erythema, and infiltration are weeping at the acute stage and skin thickening
caused by the inflammation that results from exo- (lichenification) in the chronic stage. The infantile
toxins liberated by the fungus. Fungi also are capa- form of atopic dermatitis is characterized by vesi-
ble of producing an allergic or immune response. cle formation, oozing, and crusting with excoria-
3. There are two common types of tinea capitis: pri- tions. The skin of the cheeks may be paler, with
mary (noninflammatory) and secondary (inflam- extra creases under the eyes. Adolescents and
matory). The infection is spread most often among adults usually have dry, red patches affecting the
household members who share combs and brushes face, neck, and upper trunk, but without the thick-
on which the spores are shed and remain viable ening and discrete demarcation associated with
for long periods. Depending on the invading fun- psoriasis. The bends of the elbows and knees are
gus, the lesions of the noninflammatory type can usually involved. In chronic cases, the skin is dry,
vary from grayish, round, hairless patches to bald- leathery, and lichenified.
ing spots, with or without black dots on the head. 8. It is thought that activated T lymphocytes (mainly
The individual usually is asymptomatic, although CD4 helper cells) produce chemical messengers
pruritus may exist. The inflammatory type of tinea that stimulate abnormal growth of keratinocytes
capitis is caused by virulent strains. The onset is and dermal blood vessels. Accompanying inflam-
rapid, and inflamed lesions usually are localized to matory changes are caused by infiltration of
one area of the head. The inflammation is believed neutrophils and monocytes. Skin trauma (i.e.,
to be a delayed hypersensitivity reaction to the prepsoriasis) is a common precipitating factor in
invading fungus. The initial lesion consists of a people predisposed to the disease. The reaction of
pustular, scaly, round patch with broken hairs. A the skin to an original trauma of any type is called
secondary bacterial infection is common and may the Koebner reaction. Stress, infections, trauma, xero-
lead to a painful, circumscribed, boggy, and sis, and use of medications such as angiotensin-
indurated lesion called a kerion. converting enzyme inhibitors,
-adrenergic block-
4. Pre-existing wounds (e.g., ulcers, erosions) and ing drugs, lithium, and the antimalarial agent,
tinea pedis are often portals of entry. Legs are the hydroxychloroquine (Plaquenil), may precipitate
most common sites, followed by the hands and or exacerbate the condition.
pinnas of the ears, but cellulitis may be seen on 9. Skin damage induced by UV-B is believed to be
many body parts. The lesion consists of an expand- caused by the generation of reactive oxygen species
ing red, swollen, tender plaque with an indefinite and by damage to melanin. Cellular proteins and
border, covering a small to wide area. Cellulitis is DNA are primarily damaged because of their abun-
frequently accompanied by fever, erythema, heat, dance and ability to absorb UV radiation. Both
edema, and pain. Cellulitis often involves the UV-A and UV-B also deplete Langerhans cells and
lymph system and, once compromised, repeat immune cells. It is believed that these effects pre-
infections may impair lymphatic drainage, leading vent immune cells from detecting and removing
to chronically swollen legs, and eventually dermal sun-damaged cells with malignant potential.
fibrosis and lymphedema. 10. A patient should: (1) wear a wide-brimmed hat, (2)
5. The recurrent lesions of HSV-1 usually begin with cover up in the sun, (3) seek shade, (4) wear wrap-
a burning or tingling sensation. Umbilicated vesi- around sunglasses, and (5) avoid the sun during
cles and erythema follow and progress to pustules, the hours of 10 AM to 4 PM, while using a broad-
ulcers, and crusts before healing. Lesions are most spectrum sunscreen with an sun protection factor
common on the lips, face, mouth, nasal septum, of 15 or higher. It is also important to avoid sun-
and nose. When a lesion is active, HSV-1 is shed tanning booths, perform a self-assessment of the
and there is risk of transmitting the virus to skin every month, and obtain a professional skin
others. Pain is common, and healing takes place examination every year.
within 10 to 14 days. Precipitating factors may be 11. The massive loss of skin tissue not only predisposes
stress, menses, or injury. In particular, UV-B expo- to attack by microorganisms that are present in
sure seems to be a frequent trigger for recurrence. the environment but it allows for the massive loss

Copyright 2011. Wolters Kluwer Health | Lippincott Williams & Wilkins. Study Guide for Porths Essentials of Pathophysiology, Third Edition.
LWBK707-Ans_p280-402.qxd 8/20/10 6:38PM Page 400 Aptara Inc

400 ANSWER KEY

of body fluids and their contents, it interferes with Heat sensitivity and telangiectasia occur later in
temperature regulation, it challenges the immune the disease and are not considered prominent
system, and it imposes excessive demands on the symptoms.
metabolic and reparative processes that are needed 6. Answer: c
to restore the bodys interface with the environment. RATIONALE: The lesions of allergic contact dermati-
12. Basal cell carcinoma usually is a nonmetastasizing tis range from a mild erythema with edema to
tumor that extends wide and deep if left untreated. vesicles or large bullae. The other answers are
Nodular ulcerative basal cell carcinoma is the most incorrect.
common, accounting for 60% of all basal cell car- 7. Answer: d
cinomas. It has a nodulocystic structure that RATIONALE: In persons with black skin, pigmenta-
begins as a small, flesh-colored or pink, smooth, tion may be lost from lichenified skin. The other
translucent nodule that enlarges over time. Telang- answers do not occur in people with black skin
iectatic vessels frequently are seen beneath the who have eczema.
surface. Over the years, a central depression forms 8. Answer: a
that progress to an ulcer surrounded by the origi- RATIONALE: Intravenous immunoglobulin may
nal shiny, waxy border. The second most common hasten the healing response of the skin. Broad-
form is superficial basal cell carcinoma, which is spectrum antibiotics and corticosteroids may be
seen most often on the chest or back. It begins as a given but they do not hasten the healing response
flat, nonpalpable, erythematous plaque. The red, of the skin. Diflucan is given for vaginal candidiasis.
scaly areas slowly enlarge, with nodular borders 9. Answer: b
and telangiectatic bases. This type of skin cancer is RATIONALE: In psoriasis vulgaris the primary lesions
difficult to diagnose because it mimics other are sharply demarcated, thick, red plaques with a
dermatologic problems. silvery scale that vary in size and shape. The other
answers are incorrect.
SECTION III: APPLYING YOUR KNOWLEDGE 10. Answer: c
RATIONALE: Most persons with lichen planus who
Activity D
have skin lesions also have oral lesions, appearing
1. Attack by microorganisms in the environment; mas- as milky white lacework on the buccal mucosa or
sive loss of body fluids; interferes with temperature tongue. The other answers are incorrect.
regulation; imposes excessive demands on the meta- 11. Answer: d
bolic system; and challenges the immune system. RATIONALE: Oral ivermectin, a broad-spectrum
2. Hemodynamic instability due to fluid loss; smoke antiparasitic agent, has been used for treatment-
inhalation and postburn lung injury; hypermetabo- resistant scabies. The other drugs are not used for
lism, characterized by increased oxygen consump- treatment-resistant scabies.
tion, increased glucose use, and protein and fat 12. Answer: b
wasting; impaired function of the kidneys; hypo- RATIONALE: Methods for preventing pressure ulcers
volemic shock and impaired organ perfusion; and include frequent position change, meticulous skin
sepsis. care, and frequent and careful observation to
detect early signs of skin breakdown. The other
SECTION IV: PRACTICING FOR NCLEX answers are incorrect.
13. Answer: c
Activity E
RATIONALE: Another form of nevi, the dysplastic
1. Answers: 1-a, 2-c, 3-b nevus, is important because of its capacity to
2. Answer: a transform to malignant melanoma. The other
RATIONALE: Treatment of fungal infections usually answers are incorrect.
follows diagnosis confirmed by potassium hydrox- 14. Answer: d
ide preparation or culture. The other answers are RATIONALE: Other risk factors include a family his-
incorrect. tory of malignant melanoma, presence of marked
3. Answers: 1-b, 2-a, 3-f, 4-g, 5-e, 6-d, 7-c freckling on the upper back, history of three or
4. Answer: b more blistering sunburns before 20 years of age,
RATIONALE: Benzoyl peroxide is a topical agent that and presence of actinic keratoses. The other
has both antibacterial and comedolytic properties. answers are incorrect.
It is the topical agent most effective in reducing 15. Answer: a
the P. acnes population. The other topical agents RATIONALE: The most important treatment goal is
do not act both as comedolytic and antibacterial complete elimination of the lesion. Also important
agents. is the maintenance of function and optimal
5. Answers: a, c, e cosmetic effect. Curettage with electrodesiccation,
RATIONALE: Prominent symptoms include eyes that surgical excision, irradiation, laser, cryosurgery,
are itchy, burning, or dry; a gritty or foreign sensa- and chemosurgery are effective in removing all
tion; and erythema and swelling of the eyelid. cancerous cells. The other answers are incorrect.

Copyright 2011. Wolters Kluwer Health | Lippincott Williams & Wilkins. Study Guide for Porths Essentials of Pathophysiology, Third Edition.
LWBK707-Ans_p280-402.qxd 8/20/10 6:38PM Page 401 Aptara Inc

ANSWER KEY 401

16. Answer: b 18. Answer: d


RATIONALE: In black persons, the lesions may RATIONALE: Immunization is accomplished by live-
appear as hyperpigmented nodules and occur virus injection. Rubella vaccination has nearly
more frequently on nonsun-exposed areas. The 100% immunity response in immunized children.
other answers do not describe squamous cell carci- The other answers are incorrect.
noma in black-skinned people. 19. Answer: a
17. Answer: c RATIONALE: Lentigines can be removed surgically
RATIONALE: Hemangiomas of infancy typically (cryotherapy, laser therapy, liquid nitrogen).
undergo an early period of a proliferation during Topical creams and lotions containing adapalene,
which they enlarge, followed by a period of slow tretinoin, have been used. The other answers are
involution where the growth is reversed until com- incorrect.
plete resolution. Surgical excision, laser surgery, and
chemotherapy are not used on hemangiomas of
infancy.

Copyright 2011. Wolters Kluwer Health | Lippincott Williams & Wilkins. Study Guide for Porths Essentials of Pathophysiology, Third Edition.
LWBK707-Ans_p280-408.qxd 8/21/10 2:08 AM Page 402 Aptara Inc.
LWBK707-Ans_p280-408.qxd 8/21/10 2:08 AM Page 403 Aptara Inc.
LWBK707-Ans_p280-408.qxd 8/21/10 2:08 AM Page 404 Aptara Inc.
LWBK707-Ans_p280-408.qxd 8/21/10 2:08 AM Page 405 Aptara Inc.
LWBK707-Ans_p280-408.qxd 8/21/10 2:08 AM Page 406 Aptara Inc.
LWBK707-Ans_p280-408.qxd 8/21/10 2:08 AM Page 407 Aptara Inc.
LWBK707-Ans_p280-408.qxd 8/21/10 2:08 AM Page 408 Aptara Inc.

Das könnte Ihnen auch gefallen